100% found this document useful (15 votes)
8K views498 pages

Psychiatry Test Preparation and Review Manual

Review Manual Notes

Uploaded by

Kalai selvan
Copyright
© © All Rights Reserved
We take content rights seriously. If you suspect this is your content, claim it here.
Available Formats
Download as PDF, TXT or read online on Scribd
100% found this document useful (15 votes)
8K views498 pages

Psychiatry Test Preparation and Review Manual

Review Manual Notes

Uploaded by

Kalai selvan
Copyright
© © All Rights Reserved
We take content rights seriously. If you suspect this is your content, claim it here.
Available Formats
Download as PDF, TXT or read online on Scribd
You are on page 1/ 498

Any screen.

Any time.
Anywhere.
Activate the eBook version
of this title at no additional charge.

Expert Consult eBooks give you the power to browse and find content,
view enhanced images, share notes and highlights—both online and offline.

Unlock your eBook today.


1 Visit expertconsult.inkling.com/redeem Scan this QR code to redeem your
eBook through your mobile device:
2 Scratch off your code
3 Type code into “Enter Code” box

4 Click “Redeem”
5 Log in or Sign up
6 Go to “My Library”
Place Peel Off
It’s that easy! Sticker Here

For technical assistance:


email expertconsult.help@elsevier.com
call 1-800-401-9962 (inside the US)
call +1-314-447-8200 (outside the US)
Use of the current edition of the electronic version of this book (eBook) is subject to the terms of the nontransferable, limited license granted on
expertconsult.inkling.com. Access to the eBook is limited to the first individual who redeems the PIN, located on the inside cover of this book,
at expertconsult.inkling.com and may not be transferred to another party by resale, lending, or other means.
2015v1.0
Psychiatry
Test Preparation
& Review Manual
This page intentionally left blank

     
FOURTH EDITION

Psychiatry
Test Preparation
& Review Manual

J. Clive Spiegel, MD • John M. Kenny, MD

For additional online content, visit ExpertConsult.com


Elsevier
1600 John F. Kennedy Blvd.
Ste 1800
Philadelphia, PA 19103-2899

PSYCHIATRY TEST PREPARATION & REVIEW MANUAL, FOURTH EDITION ISBN: 978-0-323-64272-9
Copyright © 2021 by Elsevier, Inc. All rights reserved.

No part of this publication may be reproduced or transmitted in any form or by any means, electronic or
mechanical, including photocopying, recording, or any information storage and retrieval system, without
permission in writing from the publisher. Details on how to seek permission, further information about the
Publisher’s permissions policies and our arrangements with organizations such as the Copyright Clearance
Center and the Copyright Licensing Agency, can be found at our website: www.elsevier.com/permissions.

This book and the individual contributions contained in it are protected under copyright by the Publisher
(other than as may be noted herein).

Notice

Practitioners and researchers must always rely on their own experience and knowledge in evaluating and
using any information, methods, compounds or experiments described herein. Because of rapid advances
in the medical sciences, in particular, independent verification of diagnoses and drug dosages should be
made. To the fullest extent of the law, no responsibility is assumed by Elsevier, authors, editors or con-
tributors for any injury and/or damage to persons or property as a matter of products liability, negligence
or otherwise, or from any use or operation of any methods, products, instructions, or ideas contained in
the material herein.

Previous editions copyrighted 2007, 2013, and 2017.

Library of Congress Control Number

Senior Content Strategist: Joslyn Chaiprasert-Paguio


Content Development Manager: Ellen Wurm-Cutter
Content Development Specialist: Laura Klein
Publishing Services Manager: Shereen Jameel
Project Manager: Aparna Venkatachalam
Designer: Bridget Hoette

Printed in the United States of America

Last digit is the print number: 9 8 7 6 5 4 3 2 1


Contents

Video Vignettes ........................................................................vi Video Vignettes - Available online 


Preface ........................................................................................vii Vignette One
References and Figure Credits ........................................ viii Vignette Two
Dedication ..................................................................................ix Vignette Three
Vignette Four
Test Number One Vignette Five
Questions .......................................................1 Vignette Six
Answer Key ..................................................29 Vignette Seven
Answer Explanations ...................................30 Vignette Eight
Test Number Two Vignette Nine
Questions .....................................................62 Vignette Ten
Answer Key ..................................................91 Vignette Eleven
Answer Explanations ...................................92 Vignette Twelve
Test Number Three Vignette Thirteen
Questions ...................................................128
Answer Key ................................................156 Topic Index ............................................................................. 455
Answer Explanations .................................157
Index ......................................................................................... 458
Test Number Four
Questions ...................................................191
Answer Key ................................................218
Answer Explanations .................................219
Test Number Five
Questions ...................................................254
Answer Key ................................................283
Answer Explanations .................................284
Test Number Six
Questions ...................................................321
Answer Key ................................................350
Answer Explanations .................................351
Vignettes
Questions ...................................................391
Answer Key ................................................424
Answer Explanations .................................426

v
Video Vignettes

The fourth edition of Psychiatry Test Preparation & Review Manual features video vignettes and accompanying
multiple-choice questions online.

Instructions on how to access these are available on the inside front cover.

vi
Preface

It has been 14 years since the first edition of this book hit the shelves and rapidly became a best seller in the field
of psychiatric test preparation. We were overwhelmed with the positive response our book received. We sought to
write a question-and-answer book that presented comprehensive answer explanations to save the reader time and
effort by having all of the information needed for exam preparation in one volume. We sought to emphasize high-
yield information for standardized exams and to present it in a format that would allow students to practice test-
taking technique and timing in addition to bolstering their knowledge base. The first three editions have helped
many psychiatrists become board-certified and pass their maintenance of certification exams, and I am delighted
to present you with the fourth edition, newly updated with the most recent standardized exam content.

I hope that you will find this new edition an invaluable asset in your test preparation, whether you are studying
for the ABPN certification exam, ABPN maintenance of certification exam, PRITE, or USMLE. I have worked very
hard to keep this book up to date with the latest information likely to show up on standardized exams. Keep in
mind that each test closely mimics the balance of material found on the psychiatry board exam, and that all of the
answer choices are valuable opportunities to learn. Even wrong answer choices can form the basis of questions on
the board exam, which is why I spend so much effort on them in the answer explanations. Each answer explana-
tion is also followed by a “K&S” or “B&D” reference. These refer to Kaplan and Sadock’s Synopsis of Psychiatry
and Bradley’s Neurology in Clinical Practice (Daroff et  al.). I believe these authoritative texts in psychiatry and
neurology are the gold standard, and candidates need only refer to the chapter corresponding with a particular
question to gain a more in-depth coverage of the subject involved. And don’t forget to test your timing for the
actual exam; take each test in one sitting, giving yourself 2½ hours for a 150-question test. When timing yourself
on vignettes, allow 1 minute per question and an extra 3 minutes to read the vignette.

I would like to thank you, our many readers, for your support. To those of you who took the time to e-mail with
questions, concerns, corrections, critiques, and comments, I extend my sincere thanks. I listened to your feedback,
and it has made this edition better than the first three. And finally, I would like to thank all those at Elsevier whose
hard work has been essential in bringing this book to the shelves.

John M. Kenny, MD
Avondale, Arizona

vii
References

Sadock, B. J., Kaplan V. Kaplan & Sadock’s Synopsis of Psychiatry. (2015). Behavioral sciences/clinical psychiatry (11th ed.).
Philadelphia: Lippincott Williams & Wilkins.
Daroff, R. B., Jankovic, J., Mazziotta, J. C., & Scott L. (2016). Pomeroy: Bradley’s neurology in clinical practice (7th ed.).
Philadelphia: Elsevier.

Figure Credits
All figures in this book are reproduced from Daroff, R. B., Fenichel, G. M., Jankovic, J., Mazziotta, J. (Eds.). (2012). Bradley’s
neurology in clinical practice (6th ed.) Philadelphia: Elsevier Saunders, with permission; and Bradley, W. G., Daroff, R. B.,
Fenichel, G. M., & Jankovic, J. (Eds.). (2003). Neurology in clinical practice (4th ed.) Philadelphia: Elsevier Butterworth-
Heinemann, with permission.

viii
Dedication

I would like to dedicate this book to:

John Clive Spiegel, MD, 1964–2014


Clive was a respected psychiatrist, neurologist, and author. His vision and perseverance made this book a reality.
He was a trusted friend and an extraordinary teacher. His life was far too short, and he is deeply missed.

ix
This page intentionally left blank

     
Psychiatry
Test Preparation
& Review Manual
This page intentionally left blank

     
Test Number One

ONE Test Number One


1. A 16-year-old male suffers from irritable mood, increased energy, decreased need for sleep, and pressured speech. He
was recently started on medication by his psychiatrist to control these symptoms. He comes into your office complaining
of a significant worsening of his acne since starting this new medication. What drug was he started on?

A. 
Oxcarbazepine
B. 
Cariprazine
C. 
Risperidone
D. 
Lithium
E. 
Lamotrigine

2. Which one of the following is not true regarding bonding and attachment?

A. 
Attachment lasts for life
B. 
Attachment is the emotional dependence of an infant on its mother
C. 
Bonding is the emotional attachment of a mother to her child
D. 
Bonding is anchored by resources and security
E. 
Poor attachments may lead to personality disorders

3. What is the best test for diagnosing Huntington’s disease?

A. 
Karyotype of chromosomes
B. 
Serum ceruloplasmin
C. 
Urine porphobilinogens
D. 
Serum polymerase chain reaction (PCR)
E. 
Cerebrospinal fluid (CSF) assay for 14-3-3 proteinase inhibitor protein

4. A patient with lesions in the orbitofrontal region of the brain will present as:

A. 
Profane, irritable, and irresponsible
B. 
Manic
C. 
Depressed
D. 
Apathetic
E. 
Psychotic

Full test - and additional VIDEO vignettes - available online - see inside front cover for details.

Psychiatry Test Preparation and Review


Manual E-Book
Psychiatry Test Preparation & Review Manual

5. A 65-year-old woman with schizophrenia believes that she is pregnant with God’s child. She has been convinced of this
for the past 5 years. When you confront her about this, she tells you that she is certain that she is pregnant and that God
is the father. She will not agree that this is not true. Her thinking is an example of which one of the following?

A. 
Egomania
B. 
Coprolalia
C. 
Delusion
D. 
Ailurophobia
E. 
Obsession

6. A group of patients is looked at with regard to a risk factor for heart disease. The group is divided into those who have
the risk factor and those who do not. These groups are then followed for a number of years to see who does and who
does not develop heart disease. This is an example of a:

A. 
Cohort study
B. 
Case–control study
C. 
Clinical trial
D. 
Cross-sectional survey
E. 
Crossover study

7. Who developed the theory of “good enough mothering”?

A. 
Piaget
B. 
Freud
C. 
Mahler
D. 
Winnicott
E. 
Erikson

8. A 47-year-old man presents to the emergency room in an acute state of disorientation, with tachycardia, ophthalmopa-
resis, diaphoresis, and ataxia. He dies in the hospital 48 hours later. A brain autopsy of this patient would reveal:

A. 
Frontal and temporal lobe atrophy
B. 
Substantia nigra depigmentation
C. 
Hemorrhages in the ependyma of the third ventricle and superior vermis
D. 
Diffuse Lewy bodies in cortex
E. 
Subcortical white matter lesions perpendicular to the ventricles

9. Which one of the following statements is true concerning monoamine oxidase inhibitors (MAOIs)?

A. 
MAOIs are not likely to cause orthostatic hypotension
B. 
To switch between an MAOI and a selective serotonin reuptake inhibitor (SSRI) you need a 3-day washout period
C. 
Giving meperidine with an MAOI is contraindicated
D. 
Lithium is contraindicated with MAOIs
E. 
All MAOIs require adherence to a tyramine-free diet

Psychiatry Test Preparation and Review


Manual E-Book
Test Number One

10. You are called to consult on an agitated patient in the medical unit. The patient is elderly and confused and is pulling
out her I.V. lines. You decide that she must be tranquilized for her own safety. Which one of the following drugs would
be the best choice?

A. 
Lorazepam
B. 
Lithium
C. 
Esketamine
D. 
Aripiprazole
E. 
Haloperidol

11. You are talking to one of your colleagues from surgery. He tells you about a postoperative patient that he is caring
for who keeps complaining of pain. He tells you that the patient was originally on intramuscular meperidine and was
switched to the same dose of oral meperidine just yesterday. The patient has been complaining constantly and is getting
the nursing staff upset. What factor do you think is responsible for this situation?

A. 
The patient has low pain tolerance
B. 
The patient has borderline personality disorder and is splitting the staff
C. 
The patient has an intractable pain disorder
D. 
The analgesic potency of oral meperidine is less than that of intramuscular meperidine
E. 
The patient has a conversion disorder (functional neurological symptom disorder)

12. A therapist is assigned a new patient in his clinic. While looking at the materials the patient filled out in the waiting area,
the therapist finds out that the patient has a substance abuse history. He immediately says, “Stupid drug addicts, they’re
so annoying. They’re such a waste of time. They never want to get better.” This is an example of:

A. 
Projection
B. 
Transference
C. 
Countertransference
D. 
Resistance
E. 
Confrontation

13. What is the likelihood of a patient acquiring Huntington’s disease if his father is a carrier and has the illness?

A. 
25%
B. 
50%
C. 
75%
D. 
90%
E. 
100%

14. Which one of the following is not a developmental task of middle adulthood?

A. 
Taking stock of accomplishments
B. 
Reassessing commitments to family, work, and marriage
C. 
Using accumulated power ethically
D. 
Engaging in risk-taking behavior
E. 
Dealing with parental illness and death

Psychiatry Test Preparation and Review


Manual E-Book
Psychiatry Test Preparation & Review Manual

15. A patient with chronic schizophrenia has been taking medication for 20 years. Every morning he goes to his pill bottle
and takes the pills his doctor prescribes. This is an example of:

A. 
Primary prevention
B. 
Secondary prevention
C. 
Tertiary prevention
D. 
Malingering
E. 
Noncompliance

16. A patient with metastatic carcinoma of the lung presents with generalized muscle weakness and is found to have
improved muscle strength with minimal exercise. The most likely diagnosis is:

A. 
Myasthenia gravis
B. 
Multiple sclerosis
C. 
Guillain–Barré syndrome
D. 
Polymyositis
E. 
Lambert–Eaton myasthenic syndrome

17. Which one of the following tests is not projective?

A. 
Draw a person test
B. 
Minnesota Multiphasic Personality Inventory (MMPI)
C. 
Sentence completion test
D. 
Thematic apperception test
E. 
Rorschach test

18. A patient presents with slowly progressive muscle weakness, fasciculations of arm muscles and tongue, and difficulty
swallowing, and he or she becomes wheelchair bound. The diagnosis is amyotrophic lateral sclerosis. Postmortem stud-
ies of this patient’s central nervous system would reveal:

A. 
Nigrostriatal depigmentation and atrophy
B. 
Frontal and temporal lobe atrophy
C. 
Anterior horn cell degeneration
D. 
Corpus callosum thinning and atrophy
E. 
Dorsal column volume loss

19. Which one of the following is not part of the ethics guidelines of the American Psychiatric Association?

A. 
It is unethical to accept a commission for patient referrals
B. 
It is unethical to have sexual relations with patients
C. 
It is a psychiatrist’s obligation to report other psychiatrists’ unethical behavior
D. 
Retiring psychiatrists must provide patients with sufficient notice of their retirement and make every reasonable
effort to find follow-up care for their patients
E. 
Psychiatrists have an obligation to participate in executions

Psychiatry Test Preparation and Review


Manual E-Book
Test Number One

20. You are called by the medicine team to do a psychiatric consultation on a 90-year-old female with sepsis who is agitated,
confused, and disoriented and is pulling out her lines. The medical team tells you that her mentation has been waxing
and waning throughout the day. Your first consideration in approaching the case is:

A. 
Determining capacity to refuse treatment
B. 
Speaking to the patient’s family
C. 
Examining the patient’s medication regimen
D. 
Developing a therapeutic relationship with the patient
E. 
Protecting the patient from unintended harm

21. Positive reinforcement, negative reinforcement, the operant, and the reinforcing stimulus are integral parts of what
theory?

A. 
Operant conditioning developed by Skinner
B. 
Operant conditioning developed by Bandura
C. 
Attribution theory developed by Hull
D. 
Learned helplessness developed by Kandel
E. 
Habituation theory developed by Pavlov

22. A 7-year-old girl with staring spells and 3-per-second spike and wave activity on electroencephalogram (EEG) fails ther-
apy with ethosuximide and has breakthrough spells. The next best medication of choice to treat this patient is:

A. 
Phenytoin
B. 
Divalproex sodium
C. 
Phenobarbital
D. 
Diazepam
E. 
Levomilnacipran

23. Episodes of sudden-onset sleep accompanied by sudden loss of muscle tone and followed by quick entry into rapid eye
movement (REM) sleep are characteristic of which one of the following?

A. 
Sleep changes associated with depression
B. 
Obstructive sleep apnea hypopnea
C. 
Insomnia disorder
D. 
Narcolepsy
E. 
Circadian rhythm sleep–wake disorder, shift-work type

24. Which one of the following neurotransmitters is not associated with the inhibition of aggressive behavior?

A. 
Dopamine
B. 
Serotonin
C. 
GABA
D. 
Glycine

Psychiatry Test Preparation and Review


Manual E-Book
Psychiatry Test Preparation & Review Manual

25. Which one of the following anticonvulsant agents needs rapid dosage increases early in therapy owing to autoinduction
of its own metabolism?

A. 
Carbamazepine
B. 
Divalproex sodium
C. 
Phenytoin
D. 
Phenobarbital
E. 
Pimavanserin

26. Giving positive reinforcement intermittently at a variable schedule is the best way to prevent:

A. 
Discrimination
B. 
Generalization
C. 
Extinction
D. 
Respondent conditioning
E. 
Transference

27. A young woman presents to the emergency room with a history of intractable seizures and mental retardation. You dis-
cover she has severe acne, skin depigmentation on her back, and blotchy patches on her retinal surface on funduscopic
examination. The most likely diagnosis is:

A. 
Down’s syndrome
B. 
Rett’s disorder
C. 
Neurofibromatosis
D. 
Tuberous sclerosis
E. 
Williams’ syndrome

28. Which one of the following drugs does not work by blocking the catabolism of acetylcholine?

A. 
Donepezil
B. 
Memantine
C. 
Tacrine
D. 
Rivastigmine
E. 
Galantamine

29. Which one of the following tests would be best used for testing executive function?

A. 
Thematic apperception test
B. 
Halstead–Reitan neuropsychological battery
C. 
MMPI
D. 
Brief psychiatric rating scale (BPRS)
E. 
Trail-making tests

30. You are asked as a psychiatrist to determine if a patient has the capacity to make a will. To make the will, which one of
the following does the patient not have to prove to you?

A. 
He knows that he is making a will
B. 
He knows how the will distributes his property
C. 
He knows the nature of the property to be distributed

Psychiatry Test Preparation and Review


Manual E-Book
Test Number One

D. 
He knows who will inherit the property
E. 
He understands court procedure

31. Which one of the following is not correct regarding the onset of puberty?

A. 
Onset of puberty is triggered by the maturation of the hypothalamic–pituitary–adrenal–gonadal axis
B. 
Primary sex characteristics are those directly involved in coitus and reproduction
C. 
The average age of onset of puberty is 11 years for boys and 13 years for girls
D. 
Increases in height and weight occur earlier in girls than in boys
E. 
In adolescent boys, testosterone levels correlate with libido

32. Sumatriptan (Imitrex) is contraindicated in patients with:

A. 
Ischemic heart disease
B. 
Kidney disease
C. 
Obstructive pulmonary disease
D. 
Inflammatory bowel disease
E. 
Carcinoma

33. Which one of the following does not increase tricyclic antidepressant concentrations?

A. 
Clozapine
B. 
Haloperidol
C. 
Risperidone
D. 
Cigarette smoking
E. 
Methylphenidate

34. Which one of the following is true regarding suicide?

A. 
Completed suicide is most frequently related to bipolar disorder
B. 
Adolescents most frequently succeed in committing suicide by hanging
C. 
In recent years the suicide rate has increased dramatically among middle-aged adults
D. 
Previous suicidal behavior is the best predictor of risk for future suicide
E. 
Women successfully commit suicide more often than men

35. Which one of the following symptoms is not part of the classical stroke condition known as Gerstmann’s syndrome?

A. 
Acalculia
B. 
Right and left confusion
C. 
Finger agnosia
D. 
Alexia without agraphia
E. 
Pure agraphia

36. Which one of the following is most appropriate for treatment with dialectical behavioral therapy?
A. 
Histrionic personality disorder
B. 
Borderline personality disorder
C. 
Dependent personality disorder
D. 
Schizoid personality disorder
E. 
Obsessive–compulsive personality disorder

Psychiatry Test Preparation and Review


Manual E-Book
Psychiatry Test Preparation & Review Manual

37. A 45-year-old woman with bipolar disorder complains of amenorrhea, galactorrhea, decreased libido, and anorgasmia.
She presents to the emergency room with an elevated serum prolactin level and is on risperidone 4 mg daily for bipolar
disorder. On neurologic examination, you discover decreased vision in both lateral visual fields. The most likely diagno-
sis is:

A. 
Acute right parietal stroke
B. 
Thalamic hemorrhage
C. 
Pituitary macroadenoma
D. 
Acute left parietal stroke
E. 
Midbrain infarct

38. Which one of the following is not true regarding the mental status examination?

A. 
Racing thoughts are considered part of thought process
B. 
Blunted is a term used to describe affect
C. 
Hallucinations are part of thought content
D. 
Delusions are part of thought content
E. 
Circumstantiality is part of thought process

39. A Malaysian man was brought into the emergency room after trying to commit suicide. The family describes an unusual
course of events preceding the suicide attempt. The patient was depressed, preoccupied, and brooding. He suddenly had
an unprovoked outburst of rage during which he went around the neighborhood and indiscriminately maimed two peo-
ple and three dogs. Two of the dogs died. Afterward he had no memory of the episode and was exhausted. He then went
into the kitchen of his home, picked up a knife, and slit his wrists. The most appropriate diagnosis is:

A. 
Koro
B. 
Amok
C. 
Piblokto
D. 
Wihtigo
E. 
Mal de ojo

40. 
An 80-year-old man with known vascular dementia presents to your emergency room with caregivers, complain-
ing of new-onset right hemiparesis and mutism. Which one of the following signs is not compatible with this clinical
presentation?

A. 
Myerson’s sign
B. 
Right-sided Hoffman’s sign
C. 
Right-sided Babinski’s sign
D. 
A positive palmomental reflex
E. 
Complete loss of the gag reflex

41. Glutamate is not:

A. 
One of the two major amino acid neurotransmitters
B. 
An inhibitory neurotransmitter
C. 
Involved in learning and memory
D. 
The primary neurotransmitter in cerebellar granule cells
E. 
A precursor of GABA

Psychiatry Test Preparation and Review


Manual E-Book
Test Number One

42. Down’s syndrome is associated with defects in chromosome 21. This is a feature also shared by:

A. 
Turner’s syndrome
B. 
Klinefelter’s syndrome
C. 
Huntington’s disease
D. 
Alzheimer’s disease
E. 
Parkinson’s disease

43. Which one of the following is not compatible with the diagnosis of brain death?

A. 
Eyes fully open
B. 
Absence of corneal reflexes
C. 
Presence of oculovestibular reflexes
D. 
Spontaneous activity seen on EEG
E. 
Large, fixed pupils

44. Patients with compromised liver function should not use which one of the following drugs?

A. 
Temazepam
B. 
Diazepam
C. 
Oxazepam
D. 
Lorazepam
E. 
Chlorazepate

45. Which one of the following is not an appropriate part of family therapy?

A. 
Exploring family members’ beliefs about the meanings of their behaviors
B. 
Reframing problematic behaviors positively
C. 
Focusing most of the session on the most dysfunctional member of the family
D. 
Encouraging family members to interact differently and observe the effects
E. 
Giving the family members things to think about and work on outside of sessions

46. Which one of the following statements is true regarding neurotransmitters and anxiety?

A. 
GABA has nothing to do with anxiety
B. 
GABA, norepinephrine, and serotonin are associated with anxiety in some way
C. 
Dopamine, glutamate, and histamine are associated with anxiety in some way
D. 
Only acetylcholine is associated with anxiety
E. 
Anxiety can be treated with injection of epinephrine

47. Which one of the following is not used in treating myasthenia gravis?

A. 
Pyridostigmine
B. 
Edrophonium chloride
C. 
Plasmapheresis
D. 
Intravenous immunoglobulin administration
E. 
Thymectomy

Psychiatry Test Preparation and Review


Manual E-Book
Psychiatry Test Preparation & Review Manual

48. You are called to evaluate a potentially delirious patient on a medical unit. As part of your workup you order an EEG.
What do you expect to find on EEG if this is truly a delirium?

A. 
3-per-second spike and wave pattern
B. 
Frontocentral beta activity
C. 
Posterior alpha rhythm
D. 
Generalized slow-wave activity consisting of theta and delta waves, as well as some focal areas of hyperactivity
E. 
Right temporal spikes

49. Gowers’ maneuver or sign is typically seen in which one of the following neurologic conditions?

A. 
Myasthenia gravis
B. 
Multiple sclerosis
C. 
Huntington’s disease
D. 
Duchenne’s muscular dystrophy
E. 
Myotonic dystrophy

50. Which one of the following antidepressants does not have strong sedative effects?

A. 
Trazodone
B. 
Paroxetine
C. 
Doxepin
D. 
Clomipramine
E. 
Mirtazapine

51. One of your patients of the opposite sex begins to act seductively and proceeds to ask you out for dinner. Which one of
the following would be an appropriate response?

A. 
Ignore the patient’s advances
B. 
Compliment the patient on the way she or he is dressed
C. 
Tell the patient that you are seeing someone and therefore can’t accept the offer
D. 
Examine your own countertransference and explore the meaning of the patient’s behavior
E. 
Have sex with the patient and then make the patient find a new doctor

52. While on call one night in the emergency room, you are asked to evaluate a distraught couple that has been brought
in by the police after a fight that started after the wife found out that her husband was wearing her panties to work.
It turns out that he has been wearing women’s undergarments for more than a year because he finds this very sexually
arousing. He has developed several fantasies imagining himself in women’s undergarments. The most appropriate diag-
nosis for the husband is:

A. 
Exhibitionistic disorder
B. 
Fetishistic disorder
C. 
Frotteuristic disorder
D. 
Voyeuristic disorder
E. 
Transvestism

10

Psychiatry Test Preparation and Review


Manual E-Book
Test Number One

53. A 48-year-old woman presents to your office with complaints of lancinating, brief, sharp pain to the left side of her face.
The pain is short-lived and recurrent. It is triggered frequently by cold air touching her face. The pharmacologic treat-
ment of choice for this condition would be:

A. 
Divalproex sodium
B. 
Clonazepam
C. 
Carbamazepine
D. 
Tiagabine
E. 
Risperidone

54. A 75-year-old man presents to the emergency room with newly onset headache, fever, vague joint pains, and complaints
of recently diminished vision. The first test of choice in this case is:

A. 
Head computed tomography (CT) scan
B. 
Brain magnetic resonance imaging (MRI) with diffusion-weighted imaging
C. 
Lumbar puncture looking for CSF xanthochromia
D. 
Serum sedimentation rate
E. 
Carotid ultrasound looking for dissection

55. The concept that different mental disorders have different outcomes was pioneered by:

A. 
Freud
B. 
Bleuler
C. 
Winnicott
D. 
Kraepelin
E. 
Kohut

56. A patient comes into your practice after referral from his primary care physician. He is convinced that he has cancer. He
thinks that it hasn’t been found yet but is convinced that it is there. He remains convinced despite a full workup with
negative results. Despite further reassurance by his doctors, he remains convinced that he has cancer. Which is the most
appropriate diagnosis?

A. 
Conversion disorder (functional neurological symptom disorder)
B. 
Illness anxiety disorder
C. 
Body dysmorphic disorder
D. 
Somatic symptom disorder
E. 
Briquet’s syndrome

57. Which one of the following is not characteristic of cluster headaches?

A. 
Attacks of short duration of 3 hours or less
B. 
Daytime attacks
C. 
Male predominance
D. 
Sharp, severe, retro-orbital pain
E. 
Cyclical pattern of occurrence mainly in spring and fall seasons

11

Psychiatry Test Preparation and Review


Manual E-Book
Psychiatry Test Preparation & Review Manual

58. Which one of the following is not a contraindication to bupropion?

A. 
Seizure
B. 
Anorexia
C. 
Use of an MAOI in the past 14 days
D. 
Head trauma
E. 
Hypertension

59. Which one of the following is safe to combine with MAOIs?

A. 
Meperidine
B. 
Lithium
C. 
Levodopa
D. 
SSRIs
E. 
Spinal anesthetic containing epinephrine

60. Which one of the following will not result in an acquired peripheral neuropathy?

A. 
Systemic lupus erythematosus
B. 
Toluene intoxication
C. 
Acetaminophen overdose
D. 
Vincristine therapy
E. 
Epstein–Barr virus infection

61. Which one of the following is false regarding tricyclic antidepressants (TCAs)?

A. 
Cigarette smoking decreases TCA levels
B. 
Clozapine will increase TCA levels
C. 
Methylphenidate will decrease TCA levels
D. 
TCAs can have adverse cardiac effects
E. 
TCAs have strong anticholinergic effects

62. A 73-year-old male presents with depression and low energy for the past month. He has no significant medical issues.
Memory and cognition are intact. You prescribe him paroxetine 20 mg po AM for depression. Two weeks later he is
brought into the emergency room by his family with disorientation, nausea, and weakness. Which laboratory test is
most likely to establish his diagnosis?

A. 
Lipase
B. 
Sodium
C. 
TSH
D. 
Alkaline phosphatase
E. 
Serum paroxetine level

63. Self-mutilation is most common in which one of the following personality disorders?

A. 
Borderline
B. 
Narcissistic
C. 
Histrionic
D. 
Dependent
E. 
Schizoid

12

Psychiatry Test Preparation and Review


Manual E-Book
Test Number One

64. A young man is admitted to the hospital with progressive proximal muscle weakness, generalized fatigue, and a red non-
pruritic rash on the face and body, especially around the knees and elbows. His workup should include screening for:

A. 
Carcinoma
B. 
Heart disease
C. 
Intestinal bleeding
D. 
Fibrotic lung disease
E. 
Stroke

65. You are on call and get paged to go see a schizophrenic patient in the inpatient unit. The patient has a tremor, is ataxic,
and is restless. During the interview the patient vomits. The nurse tells you he has been having diarrhea and has been
urinating very frequently. What question would be most useful to ask the patient?

A. 
Can you count from 100 backward by 7s?
B. 
Where are you right now?
C. 
Who is the current president?
D. 
How much water have you been drinking recently?
E. 
Are you HIV positive?

66. Susan reports to her primary care physician that she regularly picks at her skin, resulting in scabbing and scarring. She
has tried to stop repeatedly but has not been successful. She often misses days of school because of scabs on her face
from picking. Her most likely diagnosis is:

A. 
Excoriation disorder
B. 
Trichotillomania
C. 
Obsessive–compulsive disorder
D. 
Cocaine use
E. 
Ichthyosis vulgaris

67. Which one of the following is not associated with good outcomes in schizophrenia?

A. 
High premorbid functioning
B. 
Little prodrome
C. 
Early age at onset
D. 
Acute onset
E. 
Absence of family history of schizophrenia

68. During a workup you send a patient for an EEG. The results reveal shortened latency of REM sleep, decreased stage IV
sleep, and increased REM density. These findings are most consistent with:

A. 
Tumor
B. 
Petit mal epilepsy
C. 
Hepatic encephalopathy
D. 
Delirium
E. 
Depression

69. Homozygosity for which one of the following is believed to predispose patients to Alzheimer-type dementia?

A. 
Tau
B. 
Apolipoprotein E4

13

Psychiatry Test Preparation and Review


Manual E-Book
Psychiatry Test Preparation & Review Manual

C. 
Amyloid precursor protein
D. 
Trisomy 21
E. 
Presenilin

70. Which one of the following is not characteristic of narcissistic personality disorder?

A. 
Grandiosity
B. 
Need for admiration
C. 
Showing self-dramatization, theatricality, and exaggerated expression of emotion
D. 
Preoccupation with fantasies of unlimited success, power, and brilliance
E. 
Interpersonally exploitative

71. Which one of the following is not a criterion of posttraumatic stress disorder?

A. 
Reexperiencing the event
B. 
Increased arousal
C. 
Avoidance of stimuli associated with the trauma
D. 
The duration of the disturbance is more than 2 months
E. 
Experiencing repeated exposure to aversive details of a traumatic event

72. After her mother died, Sarah felt extreme sadness, cried, blamed God, felt guilty, and became convinced that she was
worthless, and she eventually tried to hang herself. Her diagnosis is:

A. 
Normal bereavement
B. 
Bipolar disorder
C. 
Delusional disorder
D. 
Anticipatory grief
E. 
Pathological grief

73. The brains of patients with schizophrenia often reveal enlargement of the:

A. 
Hippocampus
B. 
Caudate
C. 
Ventricles
D. 
Corpus callosum
E. 
Cerebellum

74. A 20-year-old patient comes into the emergency room while you are on call. She is 5 feet (152 cm) tall and has difficulty
maintaining her body weight above 67 pounds (30 kg). She has lost weight in the past by dieting and was encouraged by
her progress. She continued to decrease food intake and increase exercising until her weight dropped below 63 pounds
(28 kg). At this time she is no longer having her menstrual periods. She comes to the emergency room with symptoms of
peptic ulcer disease. Which one of the following would be considered the most important and urgent part of her initial
medical workup?

A. 
Bone scan
B. 
Head CT scan
C. 
Gastric emptying study
D. 
Cholesterol level
E. 
Serum potassium level

14

Psychiatry Test Preparation and Review


Manual E-Book
Test Number One

75. Which one of the following is not true of delusional disorder?

A. 
It may involve nonbizarre delusions that could happen in real life
B. 
It may involve tactile hallucinations
C. 
The erotomanic type involves another person of higher social standing being in love with the patient
D. 
Daily functioning is markedly impaired
E. 
The person’s behavior is not markedly odd or bizarre

76. Which one of the following is false regarding female orgasmic disorder?

A. 
Female orgasmic disorder is the persistent absence of orgasm approximately 75% to 100% of the time
B. 
The incidence of orgasm in women increases with age
C. 
Fears of impregnation or damage to the vagina, as well as guilt, are psychological factors involved in this disorder
D. 
Female orgasmic disorder can be either lifelong or acquired
E. 
Criteria include involuntary spasm of the vaginal musculature that interferes with intercourse

77. A patient presents to your office with a complaint of intense fear of going to social functions at her child’s school. On
further examination you note that she has fears that she will act in a way that will be humiliating or embarrassing. She
is also made anxious by having to meet new people that she does not know. Your differential diagnosis of this patient
should include which one of the following Axis II disorders?

A. 
Borderline personality disorder
B. 
Obsessive–compulsive personality disorder
C. 
Narcissistic personality disorder
D. 
Avoidant personality disorder
E. 
Dependent personality disorder

78. Which one of the following anticonvulsant agents is known to cause hirsutism, facial changes, and hypertrophy of the
gingiva?

A. 
Carbamazepine
B. 
Valproate
C. 
Phenobarbital
D. 
Levetiracetam
E. 
Phenytoin

79. A 52-year-old man is brought to the emergency room after being found by police prone on the edge of the sidewalk
outside. He is moderately intoxicated with alcohol and unable to give an adequate history. Upon neurologic examina-
tion you discover that his right wrist and fingers are limp, and he cannot lift them. He displays weakness when he tries
to extend his arm from a bent to a straight position. He also has trouble turning his forearm over when it is placed palm
down on a flat surface. The lesion in question here is most likely a(n):

A. 
Radial nerve entrapment
B. 
Ulnar nerve entrapment
C. 
Median nerve entrapment
D. 
Musculocutaneous nerve entrapment
E. 
Suprascapular nerve entrapment

15

Psychiatry Test Preparation and Review


Manual E-Book
Psychiatry Test Preparation & Review Manual

80. A patient comes into the emergency room complaining that twice during the past week he experienced a sudden loss of
muscle tone. The first time occurred when he was told that his mother was diagnosed with cancer. The second came dur-
ing a track meet while he was warming up before his turn to run. These episodes are most likely to be associated with
which one of the following diagnoses:

A. 
Obstructive sleep apnea hypopnea
B. 
Insomnia disorder
C. 
Hypersomnolence disorder
D. 
Narcolepsy
E. 
Circadian rhythm sleep–wake disorder

81. Which one of the following is true regarding conversion disorder (functional neurological symptom disorder)?

A. 
It is intentionally produced
B. 
It consists of complaints in multiple organ systems
C. 
It involves neurologic symptoms
D. 
It can be limited to pain
E. 
It can be limited to sexual dysfunction

82. Which one of the following is an important diagnostic feature of chronic fatigue syndrome?

A. 
Rash
B. 
Pain in 11 of 18 tender point sites
C. 
Mucus in the stool, but no blood
D. 
Postexertive malaise lasting longer than 24 hours
E. 
Opportunistic infections

83. Piaget’s stage of concrete operations includes which one of the following?

A. 
Identity versus role confusion
B. 
Good enough mothering
C. 
Conservation
D. 
Inductive reasoning
E. 
Object permanence

84. A 10-year-old child engages in sex play. This should be viewed as:

A. 
A sign of homosexuality
B. 
A sign of hormonal imbalance
C. 
The result of excessive television viewing
D. 
Normal development
E. 
Premature development

85. You see a child in the clinic who has fragile X syndrome. Which one of the following would you not expect him to have?

A. 
Mental retardation
B. 
Long ears
C. 
Narrow face
D. 
Arched palate
E. 
Short palpebral fissures

16

Psychiatry Test Preparation and Review


Manual E-Book
Test Number One

86. What would you expect from 18-month-old children with secure attachments after their parents leave them alone with
you in a room?

A. 
They would try to bring the parents back into the room
B. 
They would immediately run to you and sit on your lap
C. 
They would become more inquisitive
D. 
They would not notice the parents’ absence
E. 
They would become aggressive and violent

87. 
Which one of the following agents is a potent cytochrome P450 inhibitor and can dangerously increase levels of
lamotrigine in patients?

A. 
Phenytoin
B. 
Diazepam
C. 
Valproate
D. 
Phenobarbital
E. 
Gabapentin

88. You are introduced to a child with a physical deformity. When would you predict that the deformity would have the
greatest psychological effect on the child?

A. 
Infancy
B. 
Preschool
C. 
Elementary school age
D. 
Early adolescence
E. 
Adulthood

89. Which one of the following is a selective inhibitor of monoamine oxidase type B when given at lower doses?

A. 
Moclobemide
B. 
Phenelzine
C. 
Tranylcypromine
D. 
Selegiline
E. 
Befloxatone

90. Which one of the following will produce a hypodopaminergic state when used chronically?

A. 
Heroin
B. 
Phencyclidine (PCP)
C. 
Alcohol
D. 
Amphetamines
E. 
Cocaine

91. The anticonvulsant agent valproic acid can cause which one of the following problems in the fetus of pregnant patients?

A. 
Spina bifida
B. 
Macrocephaly
C. 
Hypertelorism
D. 
Oligohydramnios
E. 
Intrauterine growth retardation

17

Psychiatry Test Preparation and Review


Manual E-Book
Psychiatry Test Preparation & Review Manual

92. Which one of the following agents is a dopamine agonist?

A. 
Haloperidol
B. 
Pergolide
C. 
Quetiapine
D. 
Buspirone
E. 
Fluphenazine

93. A 25-year-old man is brought to see you because of a change in personality after a boating accident. He fell off of his
boat and landed head first on the dock. He was previously friendly, happy, and high-functioning. Now his speech is pres-
sured and his mood is labile. He has been irresponsible at work and has been fired from his job. His memory is intact.
Which one of the following brain areas did he damage?

A. 
Temporal lobe
B. 
Occipital lobe
C. 
Basal ganglia
D. 
Substantia nigra
E. 
Frontal lobe

94. Which one of the following inhibits norepinephrine reuptake?

A. 
Haloperidol
B. 
Ziprasidone
C. 
Chlorpromazine
D. 
Olanzapine
E. 
Aripiprazole

95. Which one of the following is a partial agonist at the dopamine D2 receptor?

A. 
Haloperidol
B. 
Ziprasidone
C. 
Chlorpromazine
D. 
Olanzapine
E. 
Aripiprazole

96. Damage to which one of the following brain areas is most likely to present with depression?

A. 
Occipital lobe
B. 
Right prefrontal cortex
C. 
Left prefrontal cortex
D. 
Right parietal lobe
E. 
Left parietal lobe

97. Which one of the following brain areas is characteristically serotonergic?

A. 
Ventral tegmental area
B. 
Substantia nigra
C. 
Nucleus accumbens
D. 
Cerebellum
E. 
Raphe nuclei

18

Psychiatry Test Preparation and Review


Manual E-Book
Test Number One

98. A patient presents to your office with a history of wing-flapping coarse tremor of the upper extremities, ataxia, and a
rapidly progressive confusional state developing over several months. The test of choice to diagnose this patient is:

A. 
Serum angiotensin-converting enzyme (ACE) level
B. 
Chromosomal analysis for CAG triplet repeats
C. 
Serum ceruloplasmin level
D. 
Lumbar puncture and CSF titer for oligoclonal bands and myelin basic protein
E. 
Edrophonium hydrochloride testing (Tensilon test)

99. A patient comes into the emergency room high on cocaine. Which one of the following brain regions would you expect
to be most active in terms of the reward he is experiencing from the drug?

A. 
Neocortex
B. 
Substantia nigra
C. 
Nucleus accumbens
D. 
Locus ceruleus
E. 
Raphe nuclei

100. What is the therapeutic focus of motivational enhancement therapy?

A. 
Anger
B. 
Depression
C. 
Medical comorbidity
D. 
Ambivalence
E. 
Environment

101. You are teaching a class to a group of first-year psychiatric residents. You review some of the psychological tests with
them and describe their use. One of the anal-retentive types in the front row asks which of the tests has the highest reli-
ability. The correct answer is:

A. 
Wechsler adult intelligence scale
B. 
Thematic apperception test
C. 
Draw a person test
D. 
MMPI
E. 
Projective personality assessment

102. Freud is best associated with which one of the following?

A. 
Learning theory
B. 
Mesolimbic dopamine theory of positive psychotic symptoms
C. 
Conflict theory
D. 
Self-psychology
E. 
Drive theory

103. The most common cause of intracerebral hemorrhage is:

A. 
Hypertension
B. 
Intracranial tumors or metastases
C. 
Disorders of coagulation (coagulopathies)
D. 
Vascular malformations
E. 
Trauma

19

Psychiatry Test Preparation and Review


Manual E-Book
Psychiatry Test Preparation & Review Manual

104. How would Beck describe the problem found in depression?

A. 
Learned helplessness
B. 
Not good enough mothering
C. 
Neurochemical imbalance
D. 
Cognitive distortion
E. 
Lack of social skills

105. A type I error occurs when:

A. 
The null hypothesis is rejected when it should have been retained
B. 
The null hypothesis is retained when it should have been rejected
C. 
There is false rejection of a difference that was truly significant
D. 
The probability of an event occurring is 0
E. 
The probability of an event occurring is 1

106. The process by which a patient in a clinical trial has an equal likelihood of being in a control group versus an experi-
mental group is:

A. 
Probability
B. 
Risk
C. 
Percentile rank
D. 
Power
E. 
Randomization

107. A 32-year-old man who is HIV-positive presents to the emergency room with mild fever to 101° F (38.3° C), headache,
stiff neck, photophobia, and lethargy. His CD4 count is 0, and he has a highly elevated viral load. The most useful
immediate diagnostic test for his current condition would be:

A. 
Head CT scan with contrast
B. 
MRI of the brain with and without gadolinium
C. 
Lumbar puncture for CSF analysis and India ink staining
D. 
Chest radiography and blood cultures
E. 
Serum cold agglutinin assay

108. The probability of finding a true difference between two samples is:

A. 
Probability
B. 
Risk
C. 
Percentile rank
D. 
Power
E. 
Randomization

109. The number of people who have a disorder at a specified point in time is:

A. 
Probability
B. 
Risk
C. 
Point prevalence
D. 
Power
E. 
Randomization

20

Psychiatry Test Preparation and Review


Manual E-Book
Test Number One

110. A young girl is brought to the pediatrician because she is not eating well. She presents with a lack of interest in food. She
has had significant weight loss and is not growing properly. She is not getting enough nutrition and is missing days of
school because she feels physically ill. She suffers no disturbances in the way her weight or body is experienced. Which
one of the following is the most appropriate diagnosis?

A. 
Avoidant/restrictive food intake disorder
B. 
Anorexia nervosa
C. 
Bulimia nervosa
D. 
Binge eating disorder
E. 
Atypical anorexia nervosa

111. How long after taking PCP can it still be found in the urine?

A. 
1 day
B. 
2 days
C. 
5 days
D. 
8 days
E. 
10 days

112. Which one of the following is associated with the amyloid precursor protein?

A. 
Wilson’s disease
B. 
Schizophrenia
C. 
Alzheimer’s disease
D. 
Bipolar disorder
E. 
Huntington’s disease

113. A patient you put on carbamazepine has weakness and a rash. Which laboratory test would you order first?

A. 
Liver profile
B. 
Electrolytes, blood–urea–nitrogen (BUN), creatinine, glucose (Chem-7)
C. 
Complete blood count
D. 
Thyroid function tests
E. 
Venereal Disease Research Laboratory (VDRL)

114. Which one of the following should not be part of an initial workup of a patient with anorexia nervosa?

A. 
Complete blood count
B. 
Chem-7
C. 
Thyroid function tests
D. 
Electrocardiogram
E. 
Head CT scan

115. The test of choice to diagnose human central nervous system prion disease is:

A. 
Serum assay for 14-3-3 proteins
B. 
CSF assay for 14-3-3 and tau proteins
C. 
EEG
D. 
MRI of the brain with and without gadolinium
E. 
Head CT scan with contrast

21

Psychiatry Test Preparation and Review


Manual E-Book
Psychiatry Test Preparation & Review Manual

116. Which eye findings are common in schizophrenia?

A. 
Failure of adduction
B. 
Failure of accommodation
C. 
Pupillary dilatation
D. 
Abnormal smooth pursuit and saccades
E. 
Weakness of the third cranial nerve

117. A patient on risperidone comes into your office and reports that she intends on going to her gynecologist because she
hasn’t been having her menstrual periods. She has taken a pregnancy test and it was negative. Which laboratory test
would you order?

A. 
Lumbar puncture
B. 
Risperidone level
C. 
Complete blood count
D. 
Liver profile
E. 
Prolactin level

118. A 20-year-old man comes into the emergency room. He has superficial cuts on his arms, legs, and abdomen. He reports
being very depressed and feels that his neighbors are out to harm him. His most likely diagnosis is:

A. 
Dysthymic disorder
B. 
Schizoaffective disorder
C. 
Borderline personality disorder
D. 
Bipolar disorder
E. 
Adjustment disorder with mixed anxiety and depressed mood

119. Which one of the following conditions has the highest prevalence?

A. 
Depressive disorders
B. 
Anxiety disorders
C. 
Schizophrenia
D. 
Dementia
E. 
Substance abuse

120. Which one of the following has the greatest comorbidity with pathological gambling?

A. 
Schizophrenia
B. 
Posttraumatic stress disorder
C. 
Agoraphobia
D. 
Major depressive disorder
E. 
Intermittent explosive disorder

121. A couple comes into the emergency room. The wife says that her husband has become convinced that she is cheating
on him, and that it is not true. He has been following her, smelling her clothing, going through her purse, and making
regular accusations. He does not meet criteria for a mood disorder. He denies other psychotic symptoms. Medical and
substance abuse history are negative. What is his diagnosis?

A. 
Schizophrenia
B. 
Major depressive disorder with psychotic features

22

Psychiatry Test Preparation and Review


Manual E-Book
Test Number One

C. 
Delusional disorder
D. 
Delirium
E. 
Dementia

122. Which one of the following disorders presents with the patient being preoccupied with having a given illness based on
misinterpretation of bodily sensations?

A. 
Pseudocyesis
B. 
Factitious disorder
C. 
Conversion disorder (functional neurological symptom disorder)
D. 
Somatic symptom disorder with predominant pain
E. 
Illness anxiety disorder

123. A patient asks you about the data proving that alcoholism is hereditary. During your discussion, the patient asks you the
following question: “The study of which group most strongly supports the heredity of alcoholism?” The correct answer is:

A. 
Siblings
B. 
Cousins
C. 
Parents
D. 
Mothers–daughters
E. 
Adopted siblings

124. A patient falls down on the floor of your office. He states that he has a terrible headache. He begins to hyperventilate.
He has asynchronous tonic–clonic movements on both sides of his body. He is not incontinent and is not injured. He is
conscious the whole time. What is the most likely explanation for this presentation?

A. 
Complex seizure
B. 
Simple seizure
C. 
Psychogenic seizure
D. 
Myoclonus
E. 
Carpal tunnel syndrome

125. The lesion that produces the classical signs of internuclear ophthalmoplegia in multiple sclerosis is most often found in
the:

A. 
Superior colliculus
B. 
Medial longitudinal fasciculus
C. 
Inferior colliculus
D. 
Nucleus of the third nerve
E. 
Nucleus of the sixth nerve

126. A child who actively approaches and interacts with unfamiliar adults after experiencing social neglect or deprivation is
most accurately diagnosed as having:

A. 
Posttraumatic stress disorder
B. 
Disinhibited social engagement disorder
C. 
Reactive attachment disorder
D. 
Attention deficit hyperactivity disorder
E. 
Rumination disorder

23

Psychiatry Test Preparation and Review


Manual E-Book
Psychiatry Test Preparation & Review Manual

127. A middle-aged man comes to you with the complaint that he cannot stop gambling. He has wasted tens of thousands
of dollars in casinos and his wife just left him. He has also been fired from his job because he misses so much work to
gamble. Where would his diagnosis best fit in the following choices according to the Diagnostic and Statistical Manual
of Mental Disorders 5 (DSM 5)?

A. 
Personality disorders
B. 
Psychotic disorders
C. 
Anxiety disorders
D. 
Impulse-control disorders
E. 
Substance-related and addictive disorders

128. Disruption of identity characterized by two or more distinct personality states is most consistent with a diagnosis of:

A. 
Dissociative amnesia
B. 
Depersonalization/derealization disorder
C. 
Dissociative fugue
D. 
Borderline personality disorder
E. 
Dissociative identity disorder

129. PCP intoxication is associated with which one of the following?

A. 
Slurred speech
B. 
Conjunctival injection
C. 
Memory impairment
D. 
Vertical nystagmus
E. 
Depressed reflexes

130. A subjective sense that the environment is changed or unreal is:

A. 
Depersonalization
B. 
Derealization
C. 
Fugue
D. 
Amnesia
E. 
Anosognosia

131. A young patient presents to your office with dementia. He has been involved in heavy drug use. He has used heroin,
PCP, lysergic acid (LSD), amphetamines, and inhalants. If you were to postulate which most likely caused his dementia,
which one would you choose?

A. 
Heroin
B. 
LSD
C. 
PCP
D. 
Amphetamines
E. 
Inhalants

132. Which is the best matched pair among the following?

A. 
Family therapy–seclusion and restraint
B. 
Vocational assessment–social skills training

24

Psychiatry Test Preparation and Review


Manual E-Book
Test Number One

C. 
Assertive community treatment–psychoanalysis
D. 
Attention deficit hyperactivity disorder (ADHD)–electroconvulsive therapy
E. 
Psychiatric rehabilitation–social skills training

133. Which one of the following is not a clinical feature highly suggestive of multiple sclerosis?

A. 
Optic neuritis
B. 
Worsening with elevated body temperature
C. 
Fatigue
D. 
Steady progression from initial onset
E. 
Lhermitte’s sign

134. A 20-year-old woman comes to the emergency room with hypokalemic alkalosis, enlarged parotids, hypotension, and
Russell’s sign. What diagnosis do you suspect?

A. 
Psychosis
B. 
Major depressive disorder
C. 
Bulimia
D. 
Inhalant-induced euphoria
E. 
HIV

135. While on call in the emergency room, you receive a phone call from emergency medical services (EMS) to say that they
are bringing in a patient who is highly intoxicated and behaviorally out of control. The patient’s friend told EMS that
the patient has been taking amphetamines. If this is true, what is the most prominent psychiatric symptom you would
expect to see?

A. 
Hallucinations
B. 
Suicidal tendencies
C. 
Disorganized speech
D. 
Paranoia
E. 
Anxiety

136. A 29-year-old woman has begun hearing voices since seeing her child hit by a car 3 weeks ago. She has become irritable
and fearful and is not sleeping well. The most likely diagnosis is:

A. 
Schizophrenia
B. 
Acute stress disorder
C. 
Dysthymic disorder
D. 
Bipolar II disorder
E. 
Adjustment disorder with depressed mood

137. Children with depression often present with which one of the following?

A. 
Urinary incontinence
B. 
Violence
C. 
Irritability
D. 
Hallucinations
E. 
Delusions

25

Psychiatry Test Preparation and Review


Manual E-Book
Psychiatry Test Preparation & Review Manual

138. A patient comes into the clinic carrying a diagnosis of schizoid personality disorder. To confirm this diagnosis, you
would look for which one of the following?
A. 
Bright, revealing clothing
B. 
Grandiosity
C. 
Paranoia
D. 
Lack of close relationships
E. 
Magical thinking

139. If you apply your abilities solely for the patient’s well-being and do no harm to the patient, you are said to have:
A. 
Beneficence
B. 
Malignancy
C. 
Justice
D. 
Validity
E. 
Autonomy

140. Which one of the following is not true of Tourette’s syndrome?


A. 
The course is usually not progressive
B. 
Symptoms increase in times of stress
C. 
Initial symptoms may decrease, increase, or persist
D. 
Vocal tics are done to intentionally provoke others
E. 
Medication can be helpful

141. On your drive in to work you wonder if you will encounter any violent patients during your day. If you encounter the
following types of patients today, which group of patients is the most likely to attack you?
A. 
Bipolar patients
B. 
Schizophrenic patients
C. 
Borderline patients
D. 
Substance abusers
E. 
Major depressive disorder patients

142. What is the best indicator that a patient has the ego strength for psychodynamic psychotherapy?
A. 
Diagnosis
B. 
Age
C. 
Quality of relationships
D. 
Gender
E. 
Mental status examination

143. T2-weighted MRI brain imaging of a patient reveals the scan pictured adjacent. The
patient is a 36-year-old woman who presented to the emergency room with recurrent
episodes of unilateral arm and leg weakness and numbness with gait instability. The
treatment of first choice in this case would be:
A. 
Intravenous ceftriaxone administration
B. 
Intravenous immunoglobulin therapy
C. 
Plasmapheresis
D. 
Sublingual aspirin and intravenous heparin therapy
E. 
Intravenous corticosteroid therapy

26

Psychiatry Test Preparation and Review


Manual E-Book
Test Number One

144. A patient with borderline personality disorder and past suicide attempts calls you after a fight with her boyfriend.
She has been cutting herself since the fight and is hearing voices. What is the best level of care for this patient at
this time?

A. 
Inpatient hospitalization
B. 
Outpatient therapy
C. 
Speak to her again in 5 days
D. 
Extended inpatient stay (1+ months)
E. 
Group therapy session

145. A 75-year-old woman is referred to your practice by an internist for depression. On initial examination you discover
that the patient has recently recovered from a heart attack. Which one of the following medications would be the best
choice for this patient?

A. 
Amitriptyline
B. 
Doxepin
C. 
Bupropion
D. 
Methylphenidate
E. 
Sertraline

146. Which one of the following is not a classical characteristic of neurofibromatosis type 1 (von Recklinghausen’s
disease)?

A. 
Café-au-lait spots and cutaneous neurofibromas
B. 
Bilateral acoustic schwannomas
C. 
Optic gliomas
D. 
Lisch nodules
E. 
Axillary or inguinal freckling

147. The sign that best differentiates between delirium and dementia is:

A. 
Sleep disturbance
B. 
Hallucinations
C. 
Disorientation to place
D. 
Violent behavior
E. 
Alteration of consciousness

148. What is the first step toward treating a 23-year-old medical student who comes to your office with a complaint of
insomnia?

A. 
Prescribe zolpidem
B. 
Prescribe benzodiazepines
C. 
Prescribe diphenhydramine
D. 
Restrict the use of the bed to sleep and intimacy only
E. 
Obtain a sleep study

149. 
Which of the following is true concerning the benefits of collaborative care between psychiatry and primary care
physicians?

A. 
Psychiatric illness is rare among primary care populations
B. 
Managed care organizations make referral to a psychiatrist a simple and quick process

27

Psychiatry Test Preparation and Review


Manual E-Book
Psychiatry Test Preparation & Review Manual

C. 
Primary care physicians often have sufficient time to adequately explore mental health concerns
D. 
Most patients feel more comfortable and less stigmatized in a general medical setting than in a mental health clinic
E. 
Most primary care physicians are comfortable managing complex regimens of psychiatric medications

150. A 20-year-old college student is brought into the emergency room after a party. He has tenting of the skin on the backs
of his hands, is nauseated and vomits, acts seductively toward the nursing staff, and thinks the security guards are out
to kill him. He tells you: “The one with the red hair is out to slay me.” The emergency medical technician tells you the
patient apparently collapsed while dancing at a “rave.” What substance has he most likely taken?

A. 
Cannabis
B. 
Ketamine
C. 
Diacetylmorphine
D. 
Methylenedioxyamphetamine
E. 
Some form of volatile inhalant

28

Psychiatry Test Preparation and Review


Manual E-Book
Test Number One

One
Answer Key – Test Number One
1. D 26. C 51. D 76. E 101. A 126. B
2. D 27. D 52. B 77. D 102. E 127. E
3. D 28. B 53. C 78. E 103. A 128. E
4. A 29. E 54. D 79. A 104. D 129. D
5. C 30. E 55. D 80. D 105. A 130. B
6. A 31. C 56. B 81. C 106. E 131. E
7. D 32. A 57. B 82. D 107. C 132. E
8. C 33. D 58. E 83. C 108. D 133. D
9. C 34. D 59. B 84. D 109. C 134. C
10. E 35. D 60. C 85. E 110. A 135. D
11. D 36. B 61. C 86. A 111. D 136. B
12. C 37. C 62. B 87. C 112. C 137. C
13. B 38. C 63. A 88. D 113. C 138. D
14. D 39. B 64. A 89. D 114. E 139. A
15. C 40. E 65. D 90. E 115. B 140. D
16. E 41. B 66. A 91. A 116. D 141. D
17. B 42. D 67. C 92. B 117. E 142. C
18. C 43. C 68. E 93. E 118. C 143. E
19. E 44. B 69. B 94. B 119. B 144. A
20. E 45. C 70. C 95. E 120. D 145. E
21. A 46. B 71. D 96. C 121. C 146. B
22. B 47. B 72. E 97. E 122. E 147. E
23. D 48. D 73. C 98. C 123. E 148. D
24. A 49. D 74. E 99. C 124. C 149. D
25. A 50. B 75. D 100. D 125. B 150. D

29

Psychiatry Test Preparation and Review


Manual E-Book
Psychiatry Test Preparation & Review Manual

One
Explanations – Test Number One
Question 1. D. Several cutaneous side effects are possible with lithium, including acne and follicular and maculopapu-
lar eruptions. Lithium can both cause and exacerbate psoriasis. Alopecia has also been reported. Major
side effects of lithium include gastrointestinal complaints, tremors, diabetes insipidus, hypothyroidism,
weight gain, cardiac arrhythmia, and edema. Frequently tested is the fact that in patients suffering from
psoriasis, lithium can precipitate psoriasis flare-ups. Another high-yield fact about lithium is that lithium
decreases the kidney’s ability to respond to antidiuretic hormone (ADH), resulting in decreased fluid
resorption from the distal tubules and increased urine output. Lamotrigine is an anticonvulsant that is
also used for mood stabilization. Side effects can include Stevens–Johnson syndrome, anemia, thrombo-
cytopenia, liver failure, and pancreatitis. Cariprazine is an atypical antipsychotic sold in the U.S. under
the brand name Vraylar, with indications for schizophrenia and bipolar disorder. It can cause extrapy-
ramidal symptoms, neuroleptic malignant syndrome, and akathisia, among other effects. Risperidone is
an antipsychotic that can cause extrapyramidal side effects, neuroleptic malignant syndrome, metabolic
syndrome, gastrointestinal upset, increased salivation, and lactation, among other effects. Oxcarbazepine
is an anticonvulsant that may cause leukopenia, thrombocytopenia, Stevens–Johnson syndrome, and sev-
eral other side effects. With the exception of lithium, the other choices do not worsen acne.
Psychopharmacology
K&S Chapter 29

Question 2. D. Attachment, which is the emotional dependence of the infant on its mother, involves resources and secu-
rity, because the infant depends on the mother for these things. Attachment theory was developed by John
Bowlby and says that a secure attachment between mother and child affects the child’s ability to form
healthy relationships later in life. Attachment occurs when there is a warm, intimate, and continuous rela-
tionship between child and mother. The attachment gives the infant a feeling of security. Bonding refers to
the mother’s feelings for her infant. The mother does not rely on her baby for food and protection; there-
fore bonding does not involve resources and security. It is thought that bonding occurs with skin-to-skin
contact between infant and mother. All other choices given are true regarding attachment theory.
Human Development
K&S Chapter 2

Question 3. D. Serum PCR is the test of choice to examine the number of trinucleotide repeats (>35 in adults and >60
in children) to diagnose Huntington’s disease (HD). The HD gene resides on the short arm of chro-
mosome 4. A chromosomal karyotype can reveal only macroscopic defects in chromosomes such as
deletions, translocations, or trisomies. Serum ceruloplasmin, when low, is diagnostic of Wilson’s dis-
ease. Urine porphobilinogens and aminolevulinic acid, when detected in urine in excessive amounts, are
diagnostic of acute intermittent porphyria. Acute intermittent porphyria presents with abdominal pain,
tachycardia, tingling in hands and feet, paranoia, and hallucinations. Creutzfeldt–Jakob disease is diag-
nosed by CSF assay for 14-3-3 proteinase inhibitor proteins.
Neurology
B&D Chapter 97

30

Psychiatry Test Preparation and Review


Manual E-Book
Test Number One

Question 4. A. Orbitofrontal lobe lesions cause patients to appear profane, irritable, and irresponsible. When pre-
sented with cases that involve personality changes, one should suspect pathology in the frontal lobes.
Also, deficits in executive functioning usually involve the frontal lobes. Medial frontal lesions cause apa-
thy, characterized by limited spontaneous movement, gesture, and speech. Left frontal lesions can cause
depression. Right frontal lobe lesions can cause mania.
Neurocognitive Disorders
K&S Chapter 21

Question 5. C. This is an example of a delusion, which is a fixed false belief that is not accepted by members of the same
cultural background. Delusions may be mood congruent or mood incongruent. They may have themes that
are bizarre, persecutory, paranoid, grandiose, jealous, somatic, guilty, or erotic. Coprolalia is the compul-
sive utterance of obscene words, as seen in Tourette’s disorder. Egomania is a pathological self-preoccupa-
tion. Ailurophobia is a dread of cats. An obsession is the pathological persistence of an irresistible thought
or feeling that cannot be eliminated from consciousness and is associated with anxiety.
Psychotic Disorders
K&S Chapter 7

Question 6. A. This is a description of a cohort study in which a well-defined population is followed over a period of
time. Cohort studies are also known as longitudinal studies. Cohort studies provide direct estimates of
risk associated with a suspected causative factor. A case–control study is a retrospective study that exam-
ines persons without a particular disease. In a clinical trial, specially selected patients receive a course of
treatment, whereas another group does not. Patients are assigned to either group on a random basis. The
goal is to determine the effectiveness of the treatment. Cross-sectional surveys describe the prevalence of
a disease in a population at a particular point in time. Crossover studies are a variation of the double-
blind study, wherein the placebo and treatment groups switch at some point during the study.
Statistics
K&S Chapter 5

Question 7. D. Although all of the choices contributed to our understanding of child development, it is Winnicott who devel-
oped the concept of good enough mothering. This concept is based on the understanding that the mother
plays a vital role in bringing the world to the infant and offering empathic anticipation of the infant’s needs.
If she does these things well enough, the baby will move toward the development of a healthy sense of self.
Piaget described stages of cognitive development consisting of sensorimotor, preoperational thought, concrete
operations, and formal operations. Freud was the founder of psychoanalysis, giving us the oral, anal, phal-
lic, and latency stages of development. Mahler developed stages of separation–individuation to describe how
children develop identity that is separate from their mothers. Her stages were normal autism, symbiosis, dif-
ferentiation, practicing, rapprochement, and object constancy. Erikson developed an eight-stage life cycle. The
stages are trust vs mistrust, autonomy vs shame and doubt, initiative vs guilt, industry vs inferiority, identity
vs role diffusion, intimacy vs self-absorption, generativity vs stagnation, and integrity vs despair and isolation.
Human Development
K&S Chapter 2

Question 8. C. The clinical picture presented is that of Wernicke’s encephalopathy. Classically seen in alcoholics, the
clinical triad is that of mental confusion, ophthalmoplegia, and gait ataxia. The usual brain autopsy
finding is that of microhemorrhages in the periventricular gray matter, particularly around the aque-
duct and third and fourth ventricles. Frontal and temporal lobe atrophy is consistent with Pick’s demen-
tia. Parkinson’s disease would result in depigmentation of the pars compacta of the substantia nigra
in the midbrain. Diffuse Lewy bodies can be seen in both Parkinson’s disease and Alzheimer’s disease.
Subcortical white matter lesions perpendicular to the ventricles (also called Dawson’s fingers) are consis-
tent with a demyelinating disease, such as multiple sclerosis.
Neurology
B&D Chapter 57

31

Psychiatry Test Preparation and Review


Manual E-Book
Psychiatry Test Preparation & Review Manual

Question 9. C. MAOIs increase levels of biogenic amine neurotransmitters (serotonin, norepinephrine, and dopamine) by
preventing their degradation. There are two types of MAO enzyme: MAO-A, which breaks down sero-
tonin, norepinephrine, and dopamine; and MAO-B, which breaks down dopamine. It is contraindicated
to give meperidine with an MAOI. Because these drugs increase intrasynaptic levels of biogenic amine
neurotransmitters, they should not be given with other drugs that do the same. There have been reports of
death in patients given MAOIs and meperidine simultaneously. Patients should inform each of the doctors
that they are seeing that they are taking an MAOI. Lithium can be given with MAOIs. When switching a
patient from an SSRI to an MAOI, you need to allow a 14-day washout (28 days for fluoxetine). This is
because the combination of these drugs in a patient’s system at the same time can potentiate a serotonin
syndrome. Orthostatic hypotension is a major side effect of the MAOIs. Other side effects include weight
gain, edema, sexual dysfunction, and insomnia. Moclobemide and the selegiline patch only weakly poten-
tiate the pressor effects of tyramine, so often do not require a tyramine-free diet at low doses.
Psychopharmacology
K&S Chapter 29

Question 10. E. The best choice for tranquilizing agitated patients is haloperidol. Given that the patient in question is
elderly, starting with a small dose of haloperidol would be appropriate. Benzodiazepines should be avoided
in cases of suspected delirium, which based on the question stem, is a concern for this patient. Hence
answer choice A is out. A benzodiazepine given to a delirious patient can worsen the delirium and further
disinhibit the patient, making him or her more agitated. In general one should use great caution in giv-
ing benzodiazepines to the elderly, and when used, they should be given in small doses. Esketamine is a
nasal spray derived from ketamine, which is sold in the U.S. under the name Spravato, and is indicated
for treatment of treatment-resistant depression. It is a N-methyl-D-aspartate (NMDA) receptor antagonist,
has potential for abuse and dissociative symptoms, and is monitored by a risk evaluation and manage-
ment program similar to clozapine. Aripiprazole, an atypical antipsychotic, comes only in oral form, which
would probably be unfeasible for an acutely agitated patient. Other atypical antipsychotic drugs that come
in intramuscular injectable form, such as olanzapine or ziprasidone, would be appropriate choices. Lithium
is not standardly used to tranquilize patients. It is a mood stabilizer used in the treatment of bipolar disor-
der and can be administered only orally.
Psychopharmacology
K&S Chapter 21

Question 11. D. Oral meperidine has lower analgesic potency than intramuscular meperidine; therefore a given dose of
the oral agent will not cover pain as well as the same dose of the intramuscular agent. There is no reason
to suspect that the patient has low pain tolerance or an intractable pain disorder, because she is only
very recently postoperative and would be expected to be in pain. There is no evidence of a personality
disorder given. Conversion disorder (functional neurological symptom disorder) presents with neuro-
logic symptoms that are not solely limited to pain, and as such this is not a conversion disorder (func-
tional neurological symptom disorder).
Somatic Symptom Disorders
K&S Chapter 13

Question 12. C. This is an example of countertransference, which refers to the conscious and unconscious feelings the thera-
pist has toward the patient. Transference refers to the feelings the patient has toward the therapist. Resistance
is when ideas that are unacceptable to the patient are prevented from reaching awareness. The term is usu-
ally used in reference to a patient withholding relevant information, remaining silent, being late, or missing
appointments while undergoing therapy. Confrontation is addressing an issue that the patient does not want
to accept. Projection is reacting to unacceptable inner impulses as if they were outside the self. It may often
take the form of perceiving one’s own feelings in another and then acting on that perception.
Psychotherapy
K&S Chapter 28

Question 13. B. HD is transmitted by an autosomal dominant inheritance pattern. If one parent is an affected carrier, the likelihood
of transmission to any given child is 50%. The protein huntingtin is coded on the short arm of chromosome 4. The
gene contains an expanded trinucleotide repeat sequence of CAG (normally fewer than 29 repeats occur).
Neurology
B&D Chapter 71

32

Psychiatry Test Preparation and Review


Manual E-Book
Test Number One

Question 14. D. Middle adulthood spans the years between ages 40 and 65. At the end of early adulthood, people review
the past and decide what the future will hold for them. In their occupation they start to see differences
between early aspirations and what they have actually achieved. In middle adulthood people take stock
of accomplishments; reassess commitment to family, work, and marriage; use power ethically; and deal
with the illness of their parents. Hence, all of the choices are life tasks faced in middle adulthood, except
risk-taking behavior. This takes place traditionally in adolescence. Adulthood typically begins with
selecting a mate, deciding on an occupation, and achieving independence and self-sufficiency.
Human Development
K&S Chapter 2

Question 15. C. This is an example of tertiary prevention. Primary prevention is when a clinician does something to
prevent the onset of a disease. This is done by reducing causative agents, reducing risk factors, increas-
ing host resistance, or interfering with the transmission of a disease. Secondary prevention is when one
identifies a disease in its early stages and seeks prompt treatment. Tertiary prevention involves reduc-
ing deficits caused by an illness to obtain the highest possible level of functioning. The other answer
choices have nothing to do with prevention. Malingering is consciously faking illness for secondary gain.
Noncompliance is a term that refers to not following a doctor’s instructions.
Statistics
K&S Chapter 5

Question 16. E. Lambert–Eaton myasthenic syndrome is a paraneoplastic abnormality of presynaptic acetylcho-


line release, often described in conjunction with small-cell lung carcinoma. The likely mechanism
is immune-mediated and directed against voltage-gated calcium channels on the presynaptic side of
the neuromuscular junction. The clinical hallmark of the disorder is generalized weakness with ini-
tial improvement in strength after minimal exercise. Multiple sclerosis would be expected to cause
numerous different deficits—motor, sensory, or both—that are diffuse in space and time. Guillain–
Barré syndrome, also known as acute inflammatory demyelinating polyneuropathy (AIDP), is a rap-
idly occurring demyelinating disease that can present with ascending pain, paralysis, sensory loss, or
any combination of these symptoms. The clinical hallmark of AIDP is a loss of deep tendon reflexes in
the extremities. The tests of choice are EMG and nerve conduction studies, which usually reveal loss
of the H reflex and decreased nerve conduction velocities. Polymyositis is an inflammatory disease of
the muscle.
Neurology
B&D Chapter 78

Question 17. B. The only choice that is not a projective test is the MMPI, which is a self-report inventory used to assess
personality and areas of psychopathologic functioning. The draw a person test consists of the patient
being asked to draw a person. The level of detail is thought to correlate with intelligence and develop-
ment level. Next the patient is asked to draw a person of the opposite sex. The patient is then questioned
on what he or she drew. The assumption is that the drawing represents the expression of the self or
the body in the environment. The sentence completion test consists of asking the patient to complete a
series of incomplete sentences. The tester focuses attention on strong affect, repeated answers, humor, or
unusual responses. The thematic apperception test is a series of pictures shown to a patient. The patient
then generates a story to explain the pictures. The patient’s most accepted and conscious traits and
motives are attributed to the character closest to the patient in sex, age, and appearance. More uncon-
scious or unacceptable traits are attributed to those characters most unlike the patient. The Rorschach
test is a series of 10 inkblots that serve as inspiration for free association. The patient’s responses to each
card are recorded and closely interpreted.
Psychological Theory and Psychometric Testing
K&S Chapter 5

Question 18. C. Amyotrophic lateral sclerosis (ALS) is a disorder of the upper and lower motor neurons. The spinal
cord lower motor neurons are also known as the anterior horn cells. These classically degenerate in ALS
and can be demonstrated on autopsy. Callosal thinning and atrophy are hallmarks of multiple sclerosis.

33

Psychiatry Test Preparation and Review


Manual E-Book
Psychiatry Test Preparation & Review Manual

Frontotemporal atrophy can be seen in Pick’s dementia. Nigrostriatal depigmentation is a result of


Parkinson’s disease. Dorsal column pathology can be seen in vitamin B12 deficiency polyneuropathy with
loss of vibration and joint position sensation.
Neurology
B&D Chapter 74

Question 19. E. It is considered unethical for psychiatrists to participate in executions. According to the American
Psychiatric Association, it is unethical to accept commission for patient referrals. It is unethical to have
romantic or sexual relationships with patients. Psychiatrists are expected to report the unethical behav-
ior of other psychiatrists. When retiring, the psychiatrist needs to give patients sufficient notice and
make an effort to find them follow-up care. These are ethical issues often questioned on standardized
examinations.
Ethics
K&S Chapter 36

Question 20. E. This patient is clearly delirious based on the description. Although all of the other choices are logical
steps, the first and most important priority is protecting the patient from harm. In this case that would
involve sedating the patient before she is harmed as a result of her own agitation. This is a good rule to
keep in mind whenever dealing with an agitated patient. The first responsibility is to keep both patient
and staff from being harmed.
Management in Psychiatry
K&S Chapter 21

Question 21. A. The use of positive and negative reinforcement is part of operant conditioning developed by Skinner. In
operant conditioning the animal is active and behaves in a way that produces a reward. Learning occurs
as a consequence of action. The desired behavior yields a positive reward. An undesired behavior gets a
negative reward. Bandura is a proponent of social learning theory, which says we learn through model-
ing others and through social interaction. Attribution theory says that people are likely to attribute their
own behavior to situational causes and the behavior of others to personality traits. This then affects
their feelings and behavior. Hull did work in the neurophysiologic aspects of learning, developing a drive
reduction theory of learning. Learned helplessness is a model for depression developed by Seligman,
in which an organism learns that no behavioral change can influence the environment. The organism
becomes depressed and apathetic because no matter what it does, its environmental circumstances never
change. Kandel studied habituation and sensitization in snails. Habituation theory says that an animal
can learn to stop responding to a repeated stimulus. Sensitization theory says that an organism can be
taught to respond more easily to a stimulus or be made more sensitive to that stimulus. Pavlov devel-
oped classical conditioning. In classical conditioning, a neutral stimulus is paired with one that evokes
a response so that eventually the neutral stimulus comes to evoke the same response. He did the classic
experiments with dogs salivating when hearing their master’s footsteps.
Psychological Theory and Psychometric Testing
K&S Chapter 2

Question 22. B. Ethosuximide is the treatment of choice for uncomplicated absence seizures, the clinical presentation
depicted in this question. Failing ethosuximide, the next best choice would be valproic acid, which has
efficacy in partial complex, primary generalized, and absence seizure types. Carbamazepine would be a
very poor choice to treat absence seizures, as it is ineffective in absence seizures and may even worsen
the condition. Phenobarbital is not indicated for use in absence seizures. Diazepam is useful only for
emergencies, such as status epilepticus, and is usually administered in rectal, intramuscular, or intra-
venous forms. Phenytoin is indicated for partial and generalized tonic–clonic seizures, not for absence
seizures. Levomilnacipran is a serotonin-norepinephrine reuptake inhibitor (SNRI) sold in the U.S.
under the brand name Fetzima, indicated for treatment of depression. It has nothing to do with absence
seizures.
Neurology
B&D Chapter 67

34

Psychiatry Test Preparation and Review


Manual E-Book
Test Number One

Question 23. D. The description in this question is that of narcolepsy. Narcolepsy consists of irresistible attacks of
refreshing daytime sleep that occur at least 3 times per week for 3 months or more. Importantly, nar-
colepsy has been associated with low levels of CSF hypocretin (orexin). The sudden loss of muscle tone
described is known as cataplexy. One also sees increased intrusion of REM sleep into the transition
between sleep and wakefulness, causing hypnopompic (while awakening) and hypnagogic (while falling
asleep) hallucinations, as well as sleep paralysis. This disorder can be dangerous because it can lead to
automobile or industrial accidents. Treatments can involve stimulants, such as amphetamines, methyl-
phenidate (Ritalin), or modafinil (Provigil), as well as structured napping times during the day. Modafinil
is a nonstimulant medication U.S. Food and Drug Administration (FDA)-approved for narcolepsy. Its
mechanism of action is on histamine neurons in the reticular activating system in the pons. Sleep changes
associated with depression include early morning awakening and difficulty falling asleep. Obstructive
sleep apnea presents with daytime irritability and drowsiness, as well as prominent snoring at night.
Insomnia disorder is characterized by difficulty initiating or maintaining sleep at least 3 nights per week
for 3 months. Circadian rhythm sleep–wake disorder, shift-work type, is a type of circadian rhythm sleep
disorder that occurs in those who repeatedly and rapidly change their work schedules. This can lead to
somnolence, insomnia, and somatic problems, such as an increased likelihood of peptic ulcer.
Sleep Wake Disorders
K&S Chapter 16

Question 24. A. Dopamine is associated with the induction of aggression. Serotonin is associated with decreased aggres-
sion. In particular, the CSF levels of 5-hydroxyindoleacetic acid (5-HIAA), a major serotonin metabolite,
have been shown to be inversely correlated with the frequency of aggression. GABA is the major inhibi-
tory neurotransmitter of the brain and is associated with decreased aggression. Glycine is an inhibitory
neurotransmitter and, as such, is not associated with increased aggression. As a general rule, it is thought
that cholinergic and catecholaminergic mechanisms seem to be involved in the induction of aggression, and
serotonin and GABA seem to inhibit such behavior.
Basic Neuroscience
K&S Chapter 1

Question 25. A. Carbamazepine induces its own metabolism. This effect decreases its 24-hour half-life by at least 50%
during the first 3 to 4 weeks of therapy. Increments in dosages after the first few weeks of therapy are
often necessary to maintain therapeutic serum levels. None of the other mentioned anticonvulsants have
this unique pharmacokinetic profile. Pimavanserin is an antipsychotic sold in the U.S. under the brand
name Nuplazid, indicated for treatment of psychosis in Parkinson’s disease. It is not an anticonvulsant
and is not known to induce its own metabolism.
Psychopharmacology
B&D Chapter 67

Question 26. C. This question reviews aspects of both operant and classical conditioning. In classical conditioning a neu-
tral (or conditioned) stimulus is repeatedly paired with one that evokes a response (the unconditioned
stimulus), such that the neutral stimulus comes to evoke the response. In operant (Skinnerian) condi-
tioning, a random behavior is reinforced with reward. Initially, every desirable response is rewarded,
which enables the behavior to be learned. Giving positive reinforcement intermittently and variably is
the best way to prevent a behavior from going extinct. Extinction occurs when the conditioned stimulus
is constantly repeated without the unconditioned stimulus until the response evoked by the uncondi-
tioned stimulus eventually disappears. Generalization is the transfer of a conditioned response from one
stimulus to another. For example, the dog that learned to salivate to a bell now salivates to the sound
of a cabinet being opened. Discrimination is recognizing and responding to differences between similar
stimuli. For example, a dog can be trained to respond differently to two similar bells. Transference that
takes place during psychotherapy can be thought of as a form of stimulus generalization. Respondent
conditioning is just another term for classical conditioning.
Psychological Theory and Psychometric Testing
K&S Chapter 2

35

Psychiatry Test Preparation and Review


Manual E-Book
Psychiatry Test Preparation & Review Manual

Question 27. D. Tuberous sclerosis is an autosomal dominant neurocutaneous disorder with a prevalence of about 1
in 6000 to 9000 individuals. The classical neurologic features of the disease are seizures, mental retar-
dation, and behavioral problems. Cutaneous lesions include the ash leaf spot (hypomelanotic mac-
ule), adenoma sebaceum (facial angiofibromas), and shagreen spots (irregularly shaped, often raised
or textured skin lesion on the back or flank). Retinal hamartomas can be observed in many patients.
Neuropathologic lesions include subependymal nodules and cortical hamartomas. Down’s syndrome, or
trisomy 21, frequently results in early-onset Alzheimer’s-type changes in the brain, including neurofi-
brillary tangles and cholinergic deficits. Rett’s disorder, a pervasive developmental disorder seen only
in girls, involves deceleration of head growth from ages 5 months to 4 years, loss of purposeful hand
skills and development of stereotyped hand movements between ages 5 months and 2.5 years, loss of
social engagement, and acquired impairment in expressive and receptive language skills. Although sei-
zures can be observed in up to 75% of Rett’s patients, there are typically no skin lesions associated with
the disorder.

Neurofibromatosis has two types: NF1 and NF2. NF1 (classic von Recklinghausen’s disease) presents
with café-au-lait spots, subcutaneous neurofibromas, axillary freckling, Lisch nodules (pigmented iris
hamartomas), optic nerve glioma, neurofibromas, and schwannomas. NF1 is caused by a mutation of
the NF1 gene on chromosome 17. You can remember this by the mnemonic “There are 17 letters in
von Recklinghausen.” NF2 is caused by a mutation of the NF2 gene on chromosome 22. Remember by
the mnemonic “Chromosome 22 carries NF2.” NF2 patients have few cutaneous lesions. The diagnostic
hallmark of NF2 is bilateral vestibular (nerve VIII) schwannomas. Williams’ syndrome is an autosomal
dominant mental retardation syndrome that occurs by a hemizygous deletion that includes the elastin
locus on chromosome 7q11–q23. Patients with the disorder have short stature, unusual facial features
that include depressed nasal bridge (an upturned nose), broad forehead, widely spaced teeth, and elfin-
like facies, as well as thyroid, renal, and cardiovascular anomalies. Psychiatric symptoms include anxiety,
hyperactivity, and hypermusicality. Seizures and skin lesions are not observed in Williams’ syndrome.
Neurology
B&D Chapter 65

Question 28. B. The mechanism described is that of the cholinesterase inhibitors used in Alzheimer’s disease. By poten-
tiating cholinergic transmission, these drugs cause modest improvement in memory and goal-directed
thought. These drugs include medications such as tacrine, donepezil, galantamine, and rivastigmine. All
of the answer choices in this question are cholinesterase inhibitors except for memantine. Memantine is
also used for Alzheimer’s dementia but works by binding to N-methyl-d-aspartate (NMDA) receptors,
acting as an antagonist and thereby slowing calcium influx into cells. The slowing of calcium influx halts
cell destruction. Bonus fact: The most common side effects of the cholinesterase inhibitors tend to be
gastrointestinal, such as nausea, vomiting, and diarrhea.
Psychopharmacology
K&S Chapter 29

Question 29. E. Of the tests listed, the only one that tests executive function is the trail-making test. The trail-making test
involves connecting letters and numbers in order, alternating between letters and numbers (i.e., connect A-1-
B-2-C-3, etc.). Another acceptable answer would be the Wisconsin Card Sorting Test, but it is not an answer
choice. The Wisconsin Card Sorting Test evaluates abstract reasoning and flexibility in problem solving. The
thematic apperception test is used to test normal personality and involves showing pictures and having the
patient come up with stories. The patient’s most accepted and conscious traits and motives are attributed to
the character closest to the patient in sex, age, and appearance. More unconscious or unacceptable traits are
attributed to those characters most unlike the patient. The Halstead–Reitan battery helps find the location
of brain lesions and differentiates between those who are brain damaged and those who are neurologically
intact. It consists of a series of 10 tests. The MMPI is a personality assessment used to find areas of psycho-
pathologic functioning. It consists of more than 500 statements to which the patient must respond “true,”
“false,” or “cannot say.” The BPRS is used to assess the severity of psychosis in schizophrenia.
Psychological Theory and Psychometric Testing
K&S Chapter 5

36

Psychiatry Test Preparation and Review


Manual E-Book
Test Number One

Question 30. E. The first four choices are all very important pieces in determining whether a person can make a will,
including whether the person knows he or she is making a will. To have the capacity to make a will,
three things are needed. The first is the ability to understand the nature and extent of one’s property.
The second is that one must know that one is making a will. The third is that one must know to whom
the property will be bequeathed. The last answer choice is part of the McGarry instrument, which deter-
mines whether someone is competent to stand trial. It has nothing to do with making a will.
Forensic Psychiatry
K&S Chapter 36

Question 31. C. The average age of onset of puberty is 11 years for girls and 13 years for boys. All other answer choices
are true. The onset of puberty is triggered by maturation of the hypothalamic–pituitary–adrenal–gonadal
axis. This leads to secondary sex characteristics, such as enlarged breasts and hips in girls and facial hair
and lowered voice in boys. Primary sex characteristics are those involved in coitus—the external genitals
and reproductive organs. Increases in height and weight occur earlier in girls than in boys. Sex hormones
increase slowly through adolescence and correlate with bodily changes. Follicle-stimulating hormone
and luteinizing hormone increase through adolescence, being above normal adult values by age 17 or 18.
Testosterone seems to increase around age 16 or 17 and then stabilize at adult levels in males.
Human Development
K&S Chapter 2

Question 32. A. Sumatriptan (Imitrex) is an antimigraine medication indicated for acute, abortive therapy of migraine
headache. All drugs in the triptan class act as potent agonists at 5-HT 1B and 5-HT 1D receptors.
Although these receptors reside principally on intracranial blood vessels, they may have an effect on the
coronary arteries as well and could theoretically cause vasoconstriction, vasospasm, and acute myocar-
dial infarction. Therefore these agents are contraindicated in patients with coronary ischemic heart dis-
ease and those with uncontrolled hypertension.
Neurology
B&D Chapter 69

Question 33. D. Antipsychotic drugs and methylphenidate increase TCA concentrations through their interaction with
the cytochrome P450 system. Other drugs that increase TCA concentrations include acetazolamide, aspi-
rin, cimetidine, thiazides, fluoxetine, and sodium bicarbonate. Cigarette smoking decreases their con-
centration through its action on the cytochrome P450 1A2 enzyme. Other drugs that decrease TCA
concentrations include ascorbic acid, lithium, barbiturates, and primidone.
Psychopharmacology
K&S Chapter 29

Question 34. D. Completed suicide is most often associated with depression, not bipolar disorder. Adolescents most fre-
quently commit suicide with guns, not by hanging. In recent years the suicide rate has gone up dramati-
cally among adolescents, not among middle-aged adults. Previous suicide attempts are the best predictor
of future risk of suicide. This is a very important factor that should be taken into account whenever
taking a patient history. Men successfully commit suicide three times more often than women. Another
factor contributing to completed suicides is age. For men, the highest risk period is after 45 years of age.
For women, the highest risk period is after 55 years of age. Married people are less likely to commit
suicide than single or widowed people. As far as religion is concerned, rates of suicide among Roman
Catholics are less than those for Protestants or Jewish people. Concerning race, whites are more likely to
commit suicide than others, especially white males. Physical health may play a role. Thirty-two percent
of people who commit suicide have seen a doctor within the past 6 months. With regard to occupation,
the higher a person’s social status, the higher the rate of suicide. A fall in social status also increases the
risk. The best legal protection for a psychiatrist treating a suicidal patient is thorough, regular docu-
mentation of a suicide assessment. No-suicide contracts signed by the patient offer no legal protection,
though they may demonstrate the existence of a therapeutic relationship between doctor and patient.
Depressive Disorders
K&S Chapter 23

37

Psychiatry Test Preparation and Review


Manual E-Book
Psychiatry Test Preparation & Review Manual

Question 35. D. Alexia without agraphia is seen with lesions involving the splenium of the corpus callosum. Gerstmann’s
syndrome usually involves left parietal lobe damage. The clinical picture is the classic tetrad of acalculia,
agraphia (without alexia), right and left confusion, and finger agnosia (the inability to name fingers). The
lesion in Gerstmann’s syndrome localizes to the left angular gyrus.
Neurology
B&D Chapter 51

Question 36. B. Dialectical behavioral therapy is a form of therapy developed by Marsha Linehan for the treatment
of borderline personality disorder. The therapist is supportive and directive. Specific exercises are per-
formed to help solve problems and improve interpersonal skills. The focus of therapy is on reducing
impulses to self-mutilate.
Psychotherapy
K&S Chapter 28

Question 37. C. The clinical picture portrayed in this question is that of hyperprolactinemia induced by dopamine block-
ade in the tuberoinfundibular system by a neuroleptic medication. Conventional neuroleptics and risper-
idone can increase the volume of pituitary microadenomas by blocking dopamine and increasing serum
prolactin levels. When an adenoma grows beyond 1.5 cm in diameter, it can encroach on the medial
portion of both optic nerves outside of the sella turcica. This optic nerve involvement results in the clas-
sical clinical sign of bitemporal hemianopsia. Appropriate treatment would be the discontinuation of the
offending drug and possibly administration of bromocriptine. Some adenomas require surgical interven-
tion if they are unresponsive to medication therapy.
Psychopharmacology
B&D Chapter 52

Question 38. C. The mental status examination is the description of the patient’s appearance, speech, actions, and
thoughts during the interview. All of the choices are correct, with the exception of C. Content of thought
includes such things as delusions, preoccupations, obsessions, compulsions, phobias, suicidality, and
homicidality. It is a common mistake to put hallucinations in the thought content section of the mental
status examination. Hallucinations are false sensory perceptions and fall under the category of percep-
tion. The categories of the mental status examination are appearance, psychomotor activity, attitude,
mood, affect, speech, perception, thought content and process, consciousness, orientation, memory, con-
centration, attention, reading and writing, visuospatial ability, abstract thought, information and intel-
ligence, impulsivity, judgment and insight, and reliability.
Diagnostic and Treatment Procedures in Psychiatry
K&S Chapter 5

Question 39. B. This is a clear description of the Malaysian cultural syndrome of Amok. It consists of a sudden rampage,
which can include homicide and/or suicide, and ends in exhaustion and amnesia. Koro is a delusion that
occurs in Asian males who believe the penis will disappear into the abdomen and cause death. Piblokto
occurs in female Eskimos of northern Greenland. It involves anxiety, depression, confusion, depersonali-
zation, and derealization and ends in stuporous sleep and amnesia. Wihtigo is a delusional fear displayed
by Native American Indians of being turned into a cannibal through possession by a supernatural mon-
ster, the Wihtigo. Mal de ojo is a syndrome found in those of Mediterranean descent involving vomiting,
fever, and restless sleep. It is thought to be caused by the evil eye.
Cultural Issues in Psychiatry
K&S Chapter 7

Question 40. E. The patient has clearly suffered a left-hemispheric stroke, possibly in the middle cerebral artery territory.
Any hemispheric stroke that involves the corticospinal tract can result in an appearance of contralateral
Babinski and Hoffman signs. The Babinski sign is the upward motion of the big toe and fanning of the
other toes when the plantar surface of the foot is stroked upwardly from bottom to top with a noxious
stimulus or blunt instrument, like the butt of a reflex hammer. The Hoffman sign is positive when the
adduction of the thumb is noted upon a fast downward flick being administered to the index or middle

38

Psychiatry Test Preparation and Review


Manual E-Book
Test Number One

finger of the same hand. Hoffman’s sign is equivalent to Babinski’s sign, except it is in the upper extrem-
ity. The palmomental reflex and Myerson’s sign are two of the classical, so-called frontal release signs.
The palmomental reflex is positive when the chin muscle contracts as the thenar eminence of the palm
contralateral to the brain lesion is stroked with a blunt instrument. Myerson’s sign is the presence of
a persistence of the glabellar reflex of blinking upon confrontation of the forehead by tapping with a
finger. The blinking normally should extinguish after several taps of the forehead, but in the presence of
frontal lobe damage, the response does not extinguish as rapidly. Complete loss of the gag reflex would
be expected only in a devastating stroke involving the brainstem or complete brain death.
Neurology
B&D Chapter 51

Question 41. B. Glutamate is the major excitatory neurotransmitter in the brain. Glutamate is the precursor to GABA.
The major inhibitory neurotransmitters are GABA and glycine. Glutamate works on the NMDA recep-
tor and four types of non-NMDA receptors. The NMDA receptor is bound by PCP. Glutamate is
thought to be very important in learning and memory. Glutamate is also important in the theory of exci-
totoxicity, which postulates that excessive glutamate stimulation leads to excessive intracellular calcium
and nitric oxide concentrations and cell death. Understimulation of the NMDA receptor by glutamate
has been found to cause psychosis; therefore glutamate is thought to play some role in schizophrenia,
although the exact nature of that role is yet unclear. Locations for glutamate in the brain include cerebel-
lar granule cells, striatum, hippocampus, pyramidal cells of the cortex, thalamocortical projections, and
corticostriatal projections.
Basic Neuroscience
K&S Chapter 1

Question 42. D. Alzheimer’s disease has been associated with defects in chromosome 21. The gene for amyloid precursor
protein is found on the long arm of chromosome 21. This protein plays a significant role in the develop-
ment of Alzheimer’s disease. These defects have been shown to run in families. Some studies have shown
that as high as 40% of Alzheimer’s patients have a positive family history for the disease. Also note the
Apo E4 gene on chromosome 19 is highly associated with Alzheimer’s. Turner’s syndrome results from
a missing sex chromosome, XO. The result is absent or minimal development of the gonads. No sex
hormones are produced. Individuals with Turner’s syndrome are female but with no secondary sex char-
acteristics and an absence or minimal development of the gonads. Klinefelter’s syndrome is the presence
of an extra chromosome, making the patient XXY. They have a male habitus because of the presence of
the Y chromosome, but because of the extra X chromosome they do not develop strong male character-
istics. They have small, underdeveloped genitals. They are infertile and can develop breast tissue during
adolescence. HD results from the expansion of trinucleotide repeat sequences at chromosome 4p16.3.
The disease typically presents with dementia and chorea. Parkinson’s disease results from the loss of
dopaminergic neurons from the substantia nigra. It can present with dementia, as well as a clear pattern
of symptoms that include shuffling gait, pill-rolling tremor, and masked facies. Other than Alzheimer’s,
the diseases listed have nothing to do with chromosome 21.
Neurodevelopmental and Pervasive Developmental Disorders
K&S Chapter 21

Question 43. C. The diagnosis of brain death can be made only in the complete absence of the brainstem reflexes (i.e.,
absent gag, fixed pupils, absent oculocephalic and oculovestibular reflexes, absent corneal reflexes). The
eyes may be either open or closed in the presence of brain death. The EEG pattern need not be flat line
to diagnose brain death. There have been known cases of preserved cortical function and consequent
positive activity on EEG despite a complete lack of brainstem functioning.
Neurology
B&D Chapters 5 and 55

Question 44. B. Of the drugs listed, diazepam is the one that needs oxidative metabolism by the liver. The other four
are safe choices for patients with compromised liver function because they have no active metab-
olites or do not need oxidation by the liver. Patients with hepatic disease and elderly patients are at

39

Psychiatry Test Preparation and Review


Manual E-Book
Psychiatry Test Preparation & Review Manual

particular risk from adverse effects due to the benzodiazepines, especially if repeated high doses are
given. Benzodiazepines should be used with caution in anyone with a history of substance abuse, cogni-
tive disorders, renal disease, liver disease, central nervous system depression, or myasthenia gravis.
Psychopharmacology
K&S Chapter 29

Question 45. C. All of the answer choices are reasonable things to do with a family in family therapy, except focusing
most of the attention on the most dysfunctional member. The focus of the therapy should be on the
whole family as a system in which everyone plays a role. The problems that the family are having should
not be treated as one person’s fault.
Psychotherapy
K&S Chapter 28

Question 46. B. The neurotransmitters associated with anxiety are norepinephrine, serotonin, and GABA. Poor regula-
tion of norepinephrine is thought to be involved in anxiety disorders. Noradrenergic neurons are found
primarily in the locus ceruleus. Stimulation of the locus ceruleus increases anxiety, and ablation of the
locus ceruleus blocks anxiety responses. Serotonin is also thought to be involved in anxiety, although
its role is less clear. Serotonergic drugs have shown a clear propensity to decrease anxiety. Serotonergic
neurons are located primarily in the raphe nuclei in the pons. The role of GABA in anxiety is clearly
supported by the strong effect that benzodiazepines have on lessening anxiety. Benzodiazepines enhance
the effect of GABA at the GABA receptor, thus decreasing anxiety. Those neurotransmitters not directly
associated with anxiety include dopamine, glutamate, histamine, and acetylcholine. There is no evidence
as yet that these neurotransmitters play a role in the pathophysiology of anxiety. Injection of epinephrine
would worsen anxiety.
Basic Neuroscience
K&S Chapter 1

Question 47. B. Myasthenia gravis (MG) is an autoimmune neurologic disorder involving the production of autoanti-
bodies against postsynaptic nicotinic acetylcholinergic receptor sites on muscle. There is passive transfer
of the offending antibodies to the fetus across the placenta. The clinical picture is that of diplopia, dys-
arthria, dysphagia, and other signs and symptoms of bulbar palsy, fatigue, and muscle weakness. Mental
status and cognition are usually intact. Deep tendon reflexes are generally preserved. There is a rela-
tionship between MG and thymoma. About 10% of patients with MG have thymoma. Edrophonium
chloride (Tensilon), a short-acting cholinergic agent, is used to diagnose the disorder clinically, and pyr-
idostigmine (Mestinon) is used to treat the disorder on an ongoing basis. Other diagnostic tests include
EMG and nerve conduction studies, which reveal a classical decreased amplitude (decrement) upon
rapid repetitive muscle stimulation. Serum antibody levels can also be titered. Other therapeutic modali-
ties include steroids, plasmapheresis, intravenous immunoglobulin administration, and immunosuppres-
sive agents, for example, azathioprine (Imuran).
Neurology
B&D Chapter 78

Question 48. D. Generalized slow activity consisting of theta and delta waves with focal areas of hyperactivity is the EEG
pattern of delirium. An important characteristic of this pattern is that the rhythm is slowed. Choice A is
the pattern for absence seizures. This is a commonly asked pattern on examinations. Choice B is normal
adult drowsiness. Choice C is a normal pattern seen when the eyes are closed. Upon awakening, the pos-
terior alpha rhythm is replaced by random activity. Right temporal spikes, choice E, are significant for
a seizure focus. In addition to the aforementioned information, the appearance of delta waves is consid-
ered abnormal and should raise concern regarding a structural lesion, except if the patient is asleep.
Neurology
K&S Chapter 1

Question 49. D. Gower’s maneuver or sign is a classic bedside indicator of muscular dystrophy or myopathy. Usually
seen in children, the sign is present when a patient gets up from the floor or a chair by using the hands

40

Psychiatry Test Preparation and Review


Manual E-Book
Test Number One

because of muscle weakness in the legs. Duchenne’s is called a dystrophinopathy because it is an autosomal
recessive hereditary disease of muscle due to a lack of dystrophin, a protein found in muscle membrane.
Duchenne’s is the most common of the childhood muscular dystrophies. Muscular weakness is usually
greater proximally. Other features include diminished deep tendon reflexes, elevated creatine phosphoki-
nase (CPK), mental retardation in about one-third of cases, and enlarged muscles due to fat infiltration,
particularly the calves.
Neurology
B&D Chapter 79

Question 50. B. The TCAs, trazodone, and mirtazapine are all sedating drugs. Sedation is a common effect of the TCAs
and can be a welcome one if the patient isn’t sleeping well. The most sedating of the TCAs are amitripty-
line, trimipramine, and doxepin. The least sedating are desipramine and protriptyline, with other TCAs
falling between these two groups in the amount of sedation they cause. Trazodone is an antidepres-
sant that can be extremely sedating. For this reason, it is sometimes used independently for insomnia.
Trazodone can also cause priapism, in which case the patient should be switched to another medication.
The SSRIs and the serotonin and norepinephrine reuptake inhibitors in general are not very sedating.
Psychopharmacology
K&S Chapter 29

Question 51. D. The appropriate response to this situation is to examine your own behavior and countertransference.
You should then share your observations about the patient’s behavior with the patient and examine the
meaning of the patient’s behavior. Answer choice A is a bad idea because ignoring the problem will not
make it go away. Flirting with the patient is inappropriate, and having sex with the patient is a violation
of ethics, which is strictly forbidden for psychiatrists. Revealing personal information is also not appro-
priate for the therapist to do and does not address the patient’s underlying motivations.
Psychotherapy
K&S Chapters 5 and 28

Question 52. B. This is an example of fetishistic disorder. In fetishism a person, usually a male, obtains sexual arousal
from an inanimate object, such as women’s undergarments, a glove, or a shoe. This needs to go on for
at least 6 months to qualify for the diagnosis and often involves sexual fantasies directed at the object.
When fetishism lasts for at least 6 months and causes clinically significant distress or problems in social
or occupational functioning, it can be classified as fetishistic disorder. Exhibitionistic disorder involves
being sexually aroused by exposing one’s genitals to a stranger. Frotteuristic disorder involves becom-
ing sexually aroused by touching and rubbing against a nonconsenting person. Voyeuristic disorder is a
pattern of obtaining sexual arousal from watching an unsuspecting person who is naked, disrobing, or
engaged in sexual activity. Transvestism is a pattern of sexual arousal from cross-dressing, usually seen
in a heterosexual male. The answer to this question is not transvestism because the patient was wear-
ing women’s undergarments only under his traditional male work clothes. He was not going to work
dressed as a female. It is not dressing like a woman that arouses him, but a fantasy connected with an
inanimate object, namely his wife’s undergarments.
Paraphilic Disorders
K&S Chapter 17

Question 53. C. This case describes trigeminal neuralgia (tic douloureux). It is usually unilateral. It usually affects the
upper two branches of the fifth nerve (V2 and V3). Treatments of choice are carbamazepine (Tegretol)
and oxcarbazepine (Trileptal), which modulate pain centrally and peripherally. About 75% of patients
respond to carbamazepine therapy. Other treatments include gabapentin (Neurontin), TCAs, tiagabine
(Gabitril), opioid analgesics, nonsteroidal anti inflammatory agents, lidocaine patches, and benzodiaz-
epine sedatives. Some patients opt for an invasive intervention: radiation therapy in the form of stereo-
tactic Gamma Knife treatment to alleviate the pain.
Neurology
B&D Chapters 69 and 70

41

Psychiatry Test Preparation and Review


Manual E-Book
Psychiatry Test Preparation & Review Manual

Question 54. D. This is the classical clinical picture of temporal (also called giant cell) arteritis, a systemic vasculitis of
the medium-sized vessels. Women are affected more often than men (about 3:1). The disease occurs in
the elderly, usually over 50 years of age. Clinically, the disease can present as new-onset or change in
headache with fever, fatigue, myalgia, night sweats, weight loss, and jaw claudication (tiredness upon
chewing). About 25% of patients have polymyalgia rheumatica. The temporal artery can demonstrate
tenderness to palpation with induration and diminished or absent pulse. The most feared complication
is irreversible and sudden vision loss as a result of central retinal artery occlusion. The initial test of
choice is the serum erythrocyte sedimentation rate (ESR), which is virtually always elevated. Temporal
artery biopsy is the gold standard diagnostic test of choice in the face of an elevated ESR. The treat-
ment of choice is prednisone. Brain imaging would not reveal the abnormality. Lumbar puncture for
CSF xanthochromia is done to diagnose subarachnoid hemorrhage. Carotid dissection does not involve
systemic constitutional symptoms but usually presents with ipsilateral stroke-like deficits due to arterial
embolization.
Neurology
B&D Chapter 69

Question 55. D. The concept that mental disorders have different outcomes was pioneered by Emil Kraepelin. He was the
first to differentiate between the course of chronic schizophrenia and that of manic psychosis. He used
the term dementia praecox, borrowing it from the work of French psychiatrist Morel. Eugen Bleuler
later renamed it as schizophrenia and stressed that it need not have a deteriorating course. Winnicott
was one of the central figures in the school of object relations theory. He developed the concepts of the
“good enough mother” and the “transitional object.” Sigmund Freud is the founder of classical psycho-
analysis. Heinz Kohut is best known for his writings on narcissism and self-psychology.
History of Psychiatry
K&S Chapter 7

Question 56. B. This is a clear case of illness anxiety disorder. The patient believes that he has a specific serious disease
despite a negative workup and the reassurance of his doctors. Conversion disorder (functional neuro-
logical symptom disorder) is when a patient has a neurologic symptom that is attributed to psychologi-
cal conflict and cannot be explained medically. Body dysmorphic disorder is preoccupation with an
imagined defect in appearance. Often slight physical imperfections cause markedly excessive concern.
Somatic symptom disorder (previously known as Briquet’s syndrome or somatization disorder) is a con-
dition in which a patient has one or more somatic symptoms that are distressing or disrupt daily life.
There are excessive thoughts, feelings, or behaviors related to the somatic symptoms, which can involve
high levels of anxiety or excessive time and energy devoted to the symptoms. These symptoms cannot
be explained by a medical diagnosis. The symptoms are not intentionally produced. Somatic symptom
disorder can be described as mild, moderate, or severe, depending on how many somatic symptoms the
patient has.
Somatic Symptom Disorders
K&S Chapter 13

Question 57. B. Cluster headache is a rare type of headache occurring in approximately 0.5% of the population.
Sufferers are usually males in their 20s and 30s. Most sufferers experience episodic cycles of 4 to 12
weeks’ duration that are predominant in the spring and fall seasons. Attack periods can be considered
chronic, that is, lasting 1 year or more without remission or with remission periods of less than 2 weeks’
duration. Attacks can last anywhere from 15 minutes to 3 hours. They can occur as often as eight times
a day or as infrequently as once every other day. The attacks are generally nocturnal. Alcohol consump-
tion is a common trigger. The attacks are excruciatingly painful and retro-orbital in location. Pain can
radiate to the teeth, neck, and temporal regions and can be accompanied by ipsilateral autonomic symp-
toms. Patients prefer moving their head or pacing rather than lying still. Abortive therapies include
oxygen by nasal cannula, sumatriptan subcutaneous injection, and ergotamine. Prophylactic therapies
include prednisone, verapamil, divalproex sodium, methysergide, and lithium.
Neurology
B&D Chapter 75

42

Psychiatry Test Preparation and Review


Manual E-Book
Test Number One

Question 58. E. Seizure, anorexia, head trauma, and use of a MAOI in the past 14 days are all contraindications to using
bupropion. Bupropion can lower the seizure threshold. One does not want to use this medication in a
situation where the seizure threshold may already be lowered or if a seizure focus is present. The medi-
cation can also cause weight loss, so use in those who are underweight is not a good idea. It can also
lead to increased rates of seizures in patients with eating disorders. Although it can increase blood pres-
sure in some patients, it does not cause hypertensive crises by itself and is not contraindicated in patients
with high blood pressure. Hypertension is a strong concern when using venlafaxine because of its abil-
ity to potentiate hypertensive crisis. Bupropion is also used for smoking cessation. Waiting for 14 days
when switching to or from an MAOI is a hard-and-fast rule to prevent a hypertensive crisis. Bupropion
is not associated with sexual side effects in the way that the selective serotonin reuptake inhibitors are.
Psychopharmacology
K&S Chapter 29

Question 59. B. Lithium and phentolamine are not contraindicated with MAOIs. Meperidine and SSRIs cannot be given
at the same time as MAOIs, and this often comes up on standardized tests. There should be a 14-day
washout period between giving an SSRI and an MAOI. Levodopa and spinal anesthetics containing epi-
nephrine are also part of a long list of medications that should not be mixed with MAOIs.
Psychopharmacology
K&S Chapter 29

Question 60. C. There are numerous causes of acquired peripheral neuropathy. The more notable causes include vincris-
tine and isoniazid (INH) therapies, excess vitamin B6 therapy, inhalant abuse (e.g., toluene or nitrous
oxide), heavy metal poisoning, hydrocarbon exposure, vitamin B12 deficiency, niacin deficiency, and com-
plications of mononucleosis (Epstein–Barr virus infection). Autoimmune diseases such as lupus can also
cause peripheral neuropathy. Acetaminophen overdose does not generally affect the peripheral nervous
system.
Neurology
B&D Chapter 76

Question 61. C. Of all of the answer choices, the only one that is not true is C. Methylphenidate will increase TCA
levels, as will some antipsychotics. Smoking decreases TCA levels. Antipsychotics and methylphenidate
increase TCA concentrations through their interaction with the cytochrome P450 system. Other drugs
that increase TCA concentrations include acetazolamide, aspirin, cimetidine, thiazides, fluoxetine, and
sodium bicarbonate. Cigarette smoking decreases their concentration through induction of the cyto-
chrome P450 1A2 enzyme. Drugs that decrease TCA concentrations include ascorbic acid, lithium, bar-
biturates, and primidone.
Psychopharmacology
K&S Chapter 29

Question 62. B. Based on the information given, this is most likely a case of SIADH that has been caused by the parox-
etine. SSRI-induced SIADH is most common in elderly patients. It can present with irritability, personal-
ity changes, decreased appetite, nausea and vomiting, weakness, confusion, hallucinations, and in severe
cases can lead to seizure, stupor, and coma. As ADH levels rise, the body holds onto water. Urine output
goes down, and urine is more concentrated. Increased blood volume dilutes sodium, and sodium levels
drop. To avoid confusion on standardized examinations, let’s compare this with the picture seen in psy-
chogenic polydipsia and with that in diabetes insipidus. In psychogenic polydipsia the patient is drinking
excessively. Urine output goes up, and urine is more dilute. Because there is so much water coming into
the body from excessive drinking, the serum sodium drops. In diabetes insipidus there is an inability to
properly concentrate the urine. Urine output goes up, and urine is more dilute. As water is moved out
of the body and is not replaced, serum sodium becomes more concentrated, and laboratory values for
sodium increase.
Psychopharmacology
K&S Chapter 29

43

Psychiatry Test Preparation and Review


Manual E-Book
Psychiatry Test Preparation & Review Manual

Question 63. A. Self-mutilation is most often associated with borderline personality disorder. Borderline patients often
use this behavior to express anger, elicit help from others, or numb themselves to overwhelming affect.
They have tumultuous interpersonal relationships and strong mood swings. They can have short-lived
psychotic episodes. Their behavior is often unpredictable. They can rarely tolerate being alone and are
known for splitting people into all good or all bad categories. They lack a consistent sense of identity.
Personality Disorders
K&S Chapter 22

Question 64. A. The clinical picture depicted in this question is that of dermatomyositis. Dermatomyositis is an autoim-
mune disease that affects skin and muscle. Skin rash appears generally with the onset of muscle weak-
ness. The rash is classically purplish and is mainly seen on the face and eyelids. It can also appear on the
neck, elbows, and knees, which are often reddened and indurated. Serum CPK levels are often elevated.
EMG results reflect myopathy and muscle irritability. The hallmark finding on muscle biopsy is perifas-
cicular atrophy and “ghost” fibers. There is a strong relationship between dermatomyositis and occult
neoplasm in up to 50% of patients with the disorder. The usual neoplasm is carcinoma and can be in
the lung, breast, stomach, or ovary most typically. A cancer workup is essential in patients found to have
dermatomyositis.
Neurology
B&D Chapter 79

Question 65. D. The case presented in this question is a common description of water intoxication. Symptoms include
tremor, ataxia, restlessness, diarrhea, vomiting, polyuria, and eventual stupor. This is a problem that can
be found in up to 20% of patients with chronic schizophrenia. When found, these patients need close
monitoring of their electrolytes, and in many cases must be water-restricted with close monitoring of
their intake and output. The electrolyte disturbances that result from drinking enormous quantities of
water can become serious medical issues and in some cases prompt medical hospitalization. Although
the other questions could be useful in doing a thorough evaluation, the patient’s symptoms and psychiat-
ric diagnosis should suggest water intoxication.
Psychotic Disorders
K&S Chapter 7

Question 66. A. Excoriation (skin-picking) disorder is new to the DSM 5. The criteria include recurrent skin picking
resulting in skin lesions and repeated attempts to decrease or stop picking. It leads to significant distress
or impairment in social, occupational, or other areas of functioning. It is not attributable to the physio-
logical effect of a substance such as cocaine or a medical condition such as scabies. It is not explained by
symptoms of another mental disorder. Also new to the DSM 5 is trichotillomania (hair-pulling disorder).
It involves recurrently pulling out hair, causing hair loss, and repeated attempts to decrease or stop hair
pulling. The hair pulling causes impairment in social or occupational functioning. Obsessive–compulsive
disorder (OCD) consists of obsessions (recurrent intrusive negative thoughts or urges that cause marked
anxiety or distress), which lead to compulsions (repetitive behaviors or mental acts aimed at prevent-
ing or reducing the anxiety or distress). Though both excoriation and trichotillomania fall under the
category of obsessive–compulsive and related disorders now that they have their own diagnostic criteria
in DSM 5, they are no longer considered under OCD but as independent disorders. Cocaine withdrawal
can present with itching in addition to depression, paranoia, sleep disturbance, increased appetite, sweat-
ing, headache, fatigue, muscle cramps, and stomach cramps. Ichthyosis vulgaris is a skin condition
wherein dry dead cells accumulate in patches on the surface of the skin. It is also known as fish-scale
disease and is inherited in an autosomal dominant pattern. This is simply a distractor in this question.
Susan did not have a flaking scalp or polygon-shaped scales on her skin that are brown-gray or white, as
would be expected in ichthyosis vulgaris.
Obsessive–Compulsive and Related Disorders
K&S Chapter 10

Question 67. C. All of the answer choices are associated with good outcomes except early age of onset. The older the
patient is at onset, the better the prognosis. Good prognostic indicators for schizophrenia include late

44

Psychiatry Test Preparation and Review


Manual E-Book
Test Number One

onset, obvious precipitating factors, acute onset, good premorbid functioning, mood disorder symp-
toms, being married, family history of mood disorders, good support systems, and positive symptoms.
Poor prognostic factors include young onset; no precipitating factors; insidious onset; poor premorbid
functioning; withdrawn or autistic-like behavior; being single, divorced, or widowed; family history of
schizophrenia; poor support systems; negative symptoms; neurological symptoms; history of perinatal
trauma; no remissions in early years; many relapses; and history of assaultiveness.
Psychotic Disorders
K&S Chapter 7

Question 68. E. The findings given in this question are descriptors of sleep patterns one would find in depression. One
might also find increased awakening during the second half of the night and increased length of the first
REM sleep episode. EEG can be used to evaluate sleep, but in clinical psychiatry, it is most often used
to separate temporal lobe seizures from pseudoseizures and to distinguish dementia from pseudodemen-
tia caused by depression. The other answer choices given are clear distractors. A tumor is unrelated to
sleep changes and could potentially show up on an EEG as a seizure focus, but we have no history here
of either seizures or seizure focus on EEG. Petit mal epilepsy has a classic 3-per-second spike and wave
pattern, which is clearly not mentioned in the question. Hepatic encephalopathy would cause a delir-
ium, making answer choices C and D very similar. EEG patterns in delirium would show generalized
slow activity, that is, theta and delta waves, with possible areas of hyperactivity. Hepatic encephalopathy
often shows on EEG as bilaterally synchronous triphasic slow waves. None of that is mentioned in the
question because we are talking solely about sleep patterns. Therefore E is the only reasonable answer.
Diagnostic and Treatment Procedures in Psychiatry
K&S Chapter 8

Question 69. B. The genetics of Alzheimer’s dementia (AD) are the subject of ongoing research. A positive family history
of the disorder is found in about one-quarter of cases. This type of AD is further classified as familial
AD, or FAD. The most significant genetic risk factor is believed to be homozygosity for the inheritance of
the E4 allele of apolipoprotein E (apo E). Apo E4 has been associated with chromosome 19. Other less
significant risks may be mutations in the presenilin 1 (on chromosome 14) and presenilin 2 (on chro-
mosome 1) genes and amyloid precursor protein (APP). Apo E4 genotyping may be useful in patients
with cognitive deficits because it points very strongly to the clinical diagnosis of AD. Neurofibrillary
tangles are a neuropathologic hallmark of AD, and a major component of these tangles is the microtu-
bule-associated protein tau. Abnormal hyperphosphorylation of tau results in the destruction of the neu-
ronal cytoskeleton and the aggregation of tangles. Trisomy 21 (Down’s syndrome) predisposes patients
to early onset of AD in as many as 90% of cases. Neuropathologic findings in these cases are identical
to those seen in elderly patients. The reason for the early onset of the condition in Down’s patients is
believed to be overexpression of the APP gene and thus increased β-amyloid deposition.
Neurocognitive Disorders
B&D Chapter 66

Question 70. C. The answer choices are all characteristics of narcissism except C. Answer choice C describes characteris-
tics of histrionic personality disorder. Histrionic personality disorder is marked by a pattern of excessive
emotionality and attention seeking. Narcissism is marked by grandiosity, need for admiration, and lack
of empathy.
Personality Disorders
K&S Chapter 22

Question 71. D. For posttraumatic stress disorder (PTSD) the duration of the disturbance must be at least 1 month. All
other answer choices are correct. In PTSD the person was exposed to an event that involved actual or
threatened death or injury, exposed to a threat to his or her or others’ physical integrity, witnessed a trau-
matic event occur to others, learned that a traumatic event had occurred to a close family member or
friend, or experienced extreme or repeated exposure to aversive details of a traumatic event. The event is
reexperienced as flashbacks or recurring dreams. Patients may act or feel as if the event were recurring. If
they perceive anything that reminds them of the event, they experience intense psychological distress and

45

Psychiatry Test Preparation and Review


Manual E-Book
Psychiatry Test Preparation & Review Manual

show physiological reactivity. Patients with PTSD often spend great energy avoiding stimuli that remind
them of the event. They can also demonstrate a numbing of general responsiveness, as shown by inability
to remember certain aspects of the trauma, loss of interest in significant activities, feelings of detachment
from others, restricted affect, and feelings of a foreshortened future. They can experience negative altera-
tions in cognition and mood. They also show signs of increased arousal, such as problems with sleep, irrita-
bility and outbursts of anger, poor concentration, hypervigilance, and excited startle response.
Trauma and Stress Related Disorders
K&S Chapter 11

Question 72. E. Uncomplicated grief is often manifested as a state of shock, numbness, or bewilderment. It may be fol-
lowed by sighing or crying. This may lead to weakness, decreased appetite, weight loss, problems con-
centrating, and sleep disturbances. This is all considered part of normal grief. Once a person begins to
manifest worthlessness, suicidality, excessive guilt, hallucinations, or psychomotor retardation, the
grief is no longer normal. Pathological grief can take many forms, ranging from absent or delayed grief
to excessively intense or prolonged grief, psychosis, and suicidality. Anticipatory grief is expressed in
advance of a loss deemed to be inevitable. This grief ends at the time when the loss occurs, regardless of
what happens after. The grief may intensify as time goes on and realization of the loss grows near, and
it may then turn to acute grief when the loss occurs. Delusional disorder and bipolar disorder are unre-
lated to the information in the question stem.
Trauma and Stress Related Disorders
K&S Chapter 34

Question 73. C. Numerous neuropathologic abnormalities accompany schizophrenia on both a microscopic and a mac-
roscopic level. Hippocampal neurons can be atrophic. Lamina III neurons in the hippocampus can be
disorganized and scattered. This is not solidly replicated in neuropathologic specimens. One of the most
replicable findings is enlargement of the cerebral ventricular system, particularly the lateral ventricles.
This finding has been extensively replicated over numerous neuropathologic specimens. Other affected
areas include the thalamus and the dorsolateral prefrontal cortex.
Psychotic Disorders
K&S Chapter 7

Question 74. E. The patient presented in this question displays a clear case of anorexia nervosa. No single laboratory
test can diagnose the disease, but a battery of tests is needed to properly evaluate the patient medically.
Tests to order include serum electrolytes, renal function tests, thyroid function tests, glucose, amylase,
complete blood count, electrocardiogram, cholesterol, dexamethasone suppression test, and carotene. Of
these, one of the most important tests is the serum potassium level. Eating-disorder patients can com-
monly become hypokalemic, develop a hypokalemic hypochloremic alkalosis, and have cardiac compli-
cations, including arrhythmias and sudden death. Osteoporosis can be found in anorexic patients, but
a bone scan is not a vital initial procedure. A head CT scan is not warranted. Delayed gastric emptying
can occur with eating disorders, but a study to prove slowed gastric emptying is not urgent. Cholesterol
is often increased in these patients but is not, again, in urgent need of address.
Feeding and Eating Disorders
K&S Chapter 15

Question 75. D. All of the answer choices regarding delusional disorder are true except D. You do not have to have impair-
ment of daily functioning to qualify for delusional disorder. The most prominent feature of the disorder is
delusions. These delusions can be persecutory, grandiose, erotic, jealous, somatic, or mixed. These patients
lack significant mood symptoms, and their delusions can be bizarre or nonbizarre. “I’m being followed
by the police” is a nonbizarre delusion because it is possible that it could be true. “I’m being tracked by
aliens” is a bizarre delusion and is not possible. The primary medicinal treatment is with antipsychotics
in conjunction with individual therapy, and sometimes family therapy. Keep in mind that the diagnosis of
delusional disorder is relatively stable over the long term. Very few patients will spontaneously improve,
and only a minority of instances will end up being rediagnosed as schizophrenia or a mood disorder.
Psychotic Disorders
K&S Chapter 7

46

Psychiatry Test Preparation and Review


Manual E-Book
Test Number One

Question 76. E. Female orgasmic disorder is the persistent absence of orgasm in women approximately 75% to 100% of
the time. It is based on the clinician’s judgment that the woman’s orgasmic capacity is less than would be
reasonable for her age, sexual experience, and adequacy of sexual stimulation she receives. The overall
prevalence is thought to be somewhere around 30%. It is true that the incidence of orgasm increases
with age, which is attributable to less psychological inhibition and more experience. Psychological fac-
tors, like those listed in answer choice C, may play a role. It can be either lifelong or acquired, depending
on whether the patient has ever had an orgasm at any point in life. Answer choice E refers to vaginis-
mus, which is an involuntary contraction of the outer third of the vagina, preventing intercourse. It can
occur after rape or in women with psychosexual conflicts. It is diagnosed in the DSM 5 as part of genito-
pelvic pain/penetration disorder.
Sexual Dysfunctions
K&S Chapter 17

Question 77. D. The case described in this question is consistent with social anxiety disorder (social phobia). Lifetime
prevalence of social anxiety disorder is 3% to 13%. Social anxiety disorder involves certain specific social
situations that provoke intense anxiety because of fear of embarrassment or humiliation. An important dif-
ferential to consider would be avoidant personality disorder. In this disorder there is a pervasive pattern of
social inhibition, feelings of inadequacy, and hypersensitivity to negative evaluation. It leads to the avoid-
ance of other people unless the sufferer is sure that he or she is going to be liked. Avoidant personality dis-
order leads to restraint of intimate relationships for fear of being shamed or ridiculed. These patients often
view themselves as socially inept or personally unappealing. They avoid jobs with significant interpersonal
contact. Very importantly, they desire the closeness and warmth of relationships but avoid them for fear of
rejection. Borderline personality disorder is characterized by a pattern of instability of interpersonal rela-
tionships, self-image, and affect, as well as marked impulsivity. OCD is defined by a pervasive pattern of
preoccupation with orderliness, perfectionism, and mental and interpersonal control at the expense of flex-
ibility, openness, and efficiency. Narcissistic personality disorder is defined by a pattern of grandiosity, need
for admiration, and lack of empathy. Dependent personality disorder is defined by a pervasive need to be
taken care of that leads to submissive and clinging behavior and fears of separation.
Anxiety Disorders
K&S Chapters 9 and 22

Question 78. E. Phenytoin sodium (Dilantin) is notorious for causing hirsutism in women, facial dysmorphism, and
gingival hypertrophy. The drug can also cause cerebellar atrophy when taken over a long period of
time, resulting in cerebellar signs and symptoms, such as ataxic gait and dysmetria of the extremities.
Carbamazepine has the distinction of inducing its own metabolism. It can also cause rash that can
lead to Stevens–Johnson syndrome. Another side effect of carbamazepine includes hyponatremia by an
antidiuretic hormone-like effect. It can also cause leukopenia and toxic hepatitis. Valproate can cause
leukopenia, liver failure, weight gain, hair loss, fetal neural tube defects such as spina bifida, and poly-
cystic ovary syndrome. Levetiracetam is an anticonvulsant with minimal side effects. One of the more
worrisome, but infrequent, side effects of levetiracetam is agitation or hyperactivity. Phenobarbital is a
barbiturate anticonvulsant and shares the side effects of that class—central nervous system depression,
sedation, respiratory compromise, and depression. Phenobarbital can of course be deadly in overdose.
Neurology
B&D Chapter 67

Question 79. A. The clinical picture depicted in this question is known as a “Saturday night palsy,” which is synony-
mous for a radial nerve entrapment. Entrapment of the radial nerve at the axilla often results from
prolonged armpit compression when the arm is draped over the edge of a chair or when a patient is
on crutches. A radial nerve palsy of this kind results in weakness of the extensor muscles of the wrist
and fingers, triceps weakness, and supinator weakness. Such compression injury usually resolves in
1 to 2 months. Ulnar nerve entrapment can occur either at the elbow or at the wrist. Elbow trauma
may result in ulnar nerve entrapment in the cubital tunnel. Other causes include arm compression
during surgery under general anesthesia. Ulnar nerve compression results in weakness of the flexor
carpi ulnaris, intrinsic hand muscles, and fourth and fifth finger deep flexor weakness. Median nerve

47

Psychiatry Test Preparation and Review


Manual E-Book
Psychiatry Test Preparation & Review Manual

entrapment at the wrist can result in a classic carpal tunnel syndrome. This is the most common of
the entrapment neuropathies. Tenosynovitis of the transverse carpal ligament places pressure on the
median nerve in the tunnel, resulting in nocturnal hand paresthesias of the thumb and index and mid-
dle fingers. There may be sensory loss, thenar atrophy, and a positive Tinel’s sign in about 60% of
cases. Tinel’s sign is positive when percussion of the nerve over the wrist results in paresthesias in the
median nerve territory. Flexing the hand at the wrist for about 1 minute or more is called Phalen’s
maneuver and can result in similar paresthesias. Injury to the median nerve is sustained with use of
handheld vibrating tools and repetitive forceful use of the hands and wrists compromising the carpal
tunnel. The diagnostic test of choice for carpal tunnel syndrome is needle EMG and nerve conduc-
tion studies, which reveal delayed sensory latency across the wrist. Musculocutaneous nerve injury
can occur with brachial plexus injuries, such as by shoulder dislocation, compression to the shoulder
during surgical anesthesia, or by repetitively carrying heavy objects over the shoulder (carpet carrier’s
palsy). Weakness in the biceps and brachialis is the hallmark of musculocutaneous nerve injury. The
suprascapular nerve is a pure motor nerve of the brachial plexus.

Entrapment injury can occur after repetitive forward traction at the shoulder. Diffuse aching pain in the
posterior shoulder is a usual symptom. EMG demonstrates denervation of the infraspinatus and supra-
spinatus muscles.
Neurology
B&D Chapter 76

Question 80. D. The case described shows loss of muscle tone in times of extreme emotion or physical exertion. This
is found in association with narcolepsy. In narcolepsy, there are irresistible attacks of sleep that occur
daily. They are characterized by either cataplexy, which is the loss of muscle tone described here, or the
recurrent intrusions of REM sleep into the transition between sleep and wakefulness, causing hypno-
pompic or hypnogogic hallucinations or sleep paralysis. Narcolepsy has been associated with low levels
of CSF hypocretin (orexin). Sleep apnea is a medical condition that can be either central or obstruc-
tive and leads to snoring, daytime drowsiness, and irritability. It can have negative long-term cardiac
consequences as well. Insomnia disorder is difficulty in initiating or maintaining sleep, or experiencing
nonrestorative sleep, for at least 1 month. It is independent of any known physical or medical condition.
It is often treated with benzodiazepines or zolpidem. Hypersomnolence disorder is excessive sleepiness 3
times per week for 3 months, as demonstrated by prolonged sleep episodes at night that are nonrestor-
ative or daily sleep episodes during the day. Treatment consists of stimulants such as amphetamines and
the nonstimulant modafinil (Provigil). Sodium oxybate (Xyrem) is FDA-approved for cataplexy associ-
ated with narcolepsy. Xyrem is essentially a synthetic analogue of γ-hydroxybutyrate, notorious for its
use as a date rape drug. Circadian rhythm sleep–wake disorder is a persistent pattern of sleep disruption
resulting from a mismatch between the sleep–wake cycle of the environment and the circadian sleep–
wake pattern. Modafinil has a specific FDA indication in circadian rhythm sleep–wake disorder, shift-
work type, as well as narcolepsy, obstructive sleep apnea, and hypersomnolence disorder.
Sleep Wake Disorders
K&S Chapter 16

Question 81. C. Conversion disorder (functional neurological symptom disorder) involves neurologic symptoms.
Multiple organ system complaints may be found in somatic symptom disorder, though they are not nec-
essary for the diagnosis. According to the DSM 5, for the patient to have somatic symptom disorder he
or she must have one or more somatic symptoms that are distressing or result in significant disruption
of daily life. In addition the patient has excessive thoughts, feelings, or behaviors related to the symp-
toms, which can be manifested by persistently high anxiety, disproportionate persistent thoughts about
the seriousness of the symptoms, or excessive time and energy devoted to the symptoms. If symptoms
are limited to pain, it is a somatic symptom disorder with predominant pain, not a conversion disor-
der. If symptoms are limited to sexual dysfunction, it is a sexual dysfunction, not conversion disorder.
Conversion disorder symptoms are not intentionally produced.
Somatic Symptom Disorders
K&S Chapter 13

48

Psychiatry Test Preparation and Review


Manual E-Book
Test Number One

Question 82. D. Chronic fatigue syndrome is characterized by a decline in energy with no clear medical etiology. It is diagnosed
after 6 months of severe fatigue. Associated symptoms can include poor sleep, muscle pain, headaches, impaired
memory or concentration, joint pain, postexertive malaise lasting longer than 24 hours, and an improvement in
symptoms when lying down. It can often present with flulike symptoms. The cause is unknown, and up to 80%
of patients have comorbid depression. Treatment involves graded exercise therapy and CBT. Aggressive exercise
should be avoided. Medications have not been shown to be effective. Other answer choices in this question are
associated with conditions that have psychiatric comorbidity, such as lupus, fibromyalgia, irritable bowel syn-
drome, and HIV. You will read more about them elsewhere in this volume.
Somatic Symptom Disorders
K&S Ch. 14

Question 83. C. Piaget proposed four stages of cognitive development: the sensorimotor stage, the stage of preoperational
thought, the concrete operations stage, and the formal operations stage. During the concrete operations stage
the child begins to deal with information outside of him or herself and to see things from others’ perspectives.
The child also develops conservation, which is the idea that although objects may change, they can maintain
characteristics that allow them to be recognized as the same (for example, different leaves may be different
shapes and colors but are all leaves). The concept of reversibility is also understood at this stage. It says that
things can change form and shape and then go back again (for example, ice to water to ice).
Human Development
K&S Chapter 2

Question 84. D. By 2 to 3 years of age, almost all children have a concept of being either male or female. Infants begin
exploring their genitalia by 15 months of age. Children also develop interest in others’ genitals, leading
to exploration and exhibition. Sexual curiosity and sex play increase during puberty but are normally
present before puberty. They are not a sign of anything abnormal, nor are they a result of watching tele-
vision, homosexuality, hormonal imbalance, or premature development.
Human Development
K&S Chapter 2

Question 85. E. Short palpebral fissures are found in children with fetal alcohol syndrome, not fragile X syndrome.
Fragile X syndrome presents with mental retardation, long ears, narrow face, short stature, hyperextend-
able joints, arched palate, macro-orchidism, seizures, and autistic features. The syndrome coincides with
a high rate of ADHD and learning disorders. It is the second most common cause of mental retardation.
It results from a mutation of the X chromosome.
Neurodevelopmental and Pervasive Developmental Disorders
K&S Chapter 31

Question 86. A. In normal attachment, children at 18 months of age would use a transitional object in the absence of
the mother. There would be less anxiety at separation than in previous stages, but it would not be com-
pletely gone. The child would try to master strange situations when the mother was nearby. There is
object permanence. It is not until 25 months of age that the child would be expected to tolerate the
mother’s absence without distress. In the situations described in the other answer choices, the child
would be scared, not violent. The child would not immediately run to you because he or she would have
some stranger anxiety. He or she would not feel safe enough in the mother’s absence to become more
inquisitive. The child would definitely notice the mother’s absence.
Human Development
K&S Chapter 2

Question 87. C. Valproate is the classic inhibitor of cytochrome P450 3A4, which causes inhibition of enzymatic clear-
ance of lamotrigine. Doses of lamotrigine need to be lowered and generally started at lower doses when
administered concomitantly with valproic acid to avoid lamotrigine toxicity. The other agents noted in
this question do not inhibit cytochrome P450 3A4 in this fashion.
Psychopharmacology
K&S Chapter 29

49

Psychiatry Test Preparation and Review


Manual E-Book
Psychiatry Test Preparation & Review Manual

Question 88. D. It is during adolescence that children move away from the family, and the friend group provides the
most important relationships. During this time, any deviation in appearance, dress, or behavior can lead
to a decrease in self-esteem. For this reason the child would most likely suffer the most psychological
effect from a deformity during adolescence.
Human Development
K&S Chapter 2

Question 89. D. Whereas some MAOIs work on both MAO-A and MAO-B, selegiline works selectively on MAO-B at
lower doses. MAO-A is involved in the metabolism of serotonin and norepinephrine. MAO-B is involved
in the metabolism of phenylethylamine. Both are involved in the metabolism of dopamine. MAO-A in
the gastrointestinal tract is involved in the metabolism of tyramine. If you block these enzymes, tyramine
is not broken down, which can lead to hypertensive crisis.
Psychopharmacology
K&S Chapter 29

Question 90. E. Cocaine blocks dopamine reuptake from the synaptic cleft, leading to increased levels of dopamine.
When chronically used, this disturbance of normal dopamine metabolism leads to depletion of dopa-
mine. Cocaine has also been shown to be associated with decreased levels of cerebral blood flow.
Patients recovering from cocaine addiction show a drop in neuronal activity and a decreased activity of
dopamine, which can persist for up to a year and a half after stopping the drug.
Substance Abuse and Addictive Disorders
K&S Chapter 20

Question 91. A. Neural tube defects are the most worrisome fetotoxic side effect of valproic acid in the pregnant patient.
The classic presentation is that of fetal spina bifida. With formation of the neural tube early in gestation,
spina bifida can usually be detected by fetal ultrasound in the first trimester. The other noted problems
are not attributable to side effects of valproic acid.
Psychopharmacology
K&S Chapter 29

Question 92. B. Dopamine agonists are used to treat Parkinson’s disease. The classic agents of this class available for
use in the United States are pergolide, bromocriptine, pramipexole, and ropinirole. Ropinirole is also
indicated in restless legs syndrome and is one of the treatments of choice for that disorder. Worrisome
side effects of the dopamine agonists include hallucinations, sedation, and orthostatic hypotension.
There is a much lower incidence of dyskinesias with the dopamine agonists than with levodopa therapy.
Haloperidol and fluphenazine are conventional antipsychotic agents and hence are dopamine antago-
nists. Quetiapine is a second-generation atypical antipsychotic that has dopamine antagonist properties
as well. Buspirone is approved for generalized anxiety disorder and is a 5-HT 1A partial agonist.
Neurology
B&D Chapter 71

Question 93. E. The frontal lobes are the seat of executive functioning. They also play a large role in the personality.
Damage to the orbitofrontal region can cause disinhibition, irritability, mood lability, euphoria, lack of
remorse, poor judgment, and distractibility. Damage to the dorsolateral frontal regions leads to extensive
executive functioning deficits. Damage to the medial frontal region leads to an apathy syndrome.
Basic Neuroscience
K&S Chapter 1

Question 94. B. Ziprasidone (Geodon) stands alone as the one atypical antipsychotic that inhibits serotonin and norepi-
nephrine reuptake. All of the atypical antipsychotics block the 5-HT 2A and dopamine D2 receptors.
The reuptake inhibition seen with ziprasidone, however, is unique, as is its blockade of the 5-HT 1A
receptor.
Psychopharmacology
K&S Chapter 29

50

Psychiatry Test Preparation and Review


Manual E-Book
Test Number One

Question 95. E. Aripiprazole (Abilify) is a partial dopamine agonist at the D2 receptor. It is postulated to work on posi-
tive symptoms of schizophrenia by competing with dopamine in the mesolimbic pathway and negative
symptoms of schizophrenia by being an agonist at dopamine receptors in the prefrontal cortex. It is also
a partial agonist at the 5-HT 1A receptor and an antagonist at the 5-HT 2A receptor. All of the other
antipsychotics available block only dopamine receptors. This blockade of dopamine receptors in the
frontal cortex theoretically leads to a worsening of negative symptoms by the medication, particularly
by the typical antipsychotics. Another commonly tested fact is that aripiprazole causes less weight gain
than most of the other atypical antipsychotics. Interestingly, because it has less risk of prolactin eleva-
tion, aripiprazole would be a good choice for a patient with a prolactin-secreting tumor who develops
psychosis. Studies have shown lower prolactin elevation with aripiprazole than other antipsychotics.
Psychopharmacology
K&S Chapter 29

Question 96. C. The prefrontal cortices influence mood differently. If one activates the left prefrontal cortex, mood is
lifted. If the right prefrontal cortex is activated, mood is depressed. Therefore a lesion to the left prefron-
tal cortex would cause depression, and a lesion to the right prefrontal cortex would cause euphoria and
laughter. The parietal and occipital lobes are not the predominant lobes involved in emotion.
Basic Neuroscience
K&S Chapter 1

Question 97. E. The raphe nuclei of the brainstem, predominantly in the pons, are the major sites of serotonergic cell
bodies. The ventral tegmental area, substantia nigra, and nucleus accumbens are all dopaminergic areas
and are parts of the major neuronal pathways involved in the pathophysiology of schizophrenia. The
cerebellum is a distractor.
Basic Neuroscience
K&S Chapter 1

Question 98. C. The clinical picture depicted in this question is that of Wilson’s disease. Wilson’s disease is an autosomal
recessive disorder of abnormal copper metabolism. It is linked to chromosome 13. The disorder results
in a problem with the incorporation of copper into ceruloplasmin and with diminished biliary excretion
of copper. This results in excessive deposition of copper in the brain, with a predilection for the basal
ganglia. The most useful laboratory test is serum ceruloplasmin, which is decreased. The most frequent
neurologic manifestations are parkinsonism, flapping tremor, ataxia, dystonia, and bulbar signs, such as
dysphagia and dysarthria. Signs of liver failure are usually present. The treatment of choice is penicil-
lamine, a copper-chelating agent, which in many cases can reverse the deficits of the disease. Serum ACE
level would be a screening test for sarcoidosis. Chromosomal analysis for CAG triplet repeats by PCR
would be the test of choice for Huntington’s disease. Lumbar puncture for CSF oligoclonal bands and
myelin basic protein would be a useful supportive test (in addition to brain and/or spinal cord MRI) for
multiple sclerosis. The Tensilon test is for the diagnosis of myasthenia gravis.
Neurology
B&D Chapter 71

Question 99. C. The mesolimbic pathway of dopaminergic neurons, starting at the ventral tegmental area and projecting
to the nucleus accumbens, is thought to be highly involved in the sense of reward one gets from cocaine
use and is a major mediator of cocaine’s effects. It is very involved in amphetamine’s effects as well. The
locus ceruleus of the brainstem contains a high number of adrenergic neurons and mediates the effects of
opiates and opioids.
Basic Neuroscience
K&S Chapter 20

Question 100. D. The therapeutic focus of motivational enhancement therapy is on the patient’s ambivalence toward staying
off the drug of abuse. It is a type of therapy specifically used with patients addicted to drugs of abuse.
Psychotherapy
K&S Chapter 28

51

Psychiatry Test Preparation and Review


Manual E-Book
Psychiatry Test Preparation & Review Manual

Question 101. A. Of the many psychological tests used today, the reliability of the Wechsler Adult Intelligence Scale is
among the highest. Retesting of people, even at later ages, rarely reveals higher IQ scores. The scores
are consistent and repeatable. As such, it is the most reliable of the choices given. It also has a very high
validity in identifying mental retardation and predicting future school performance. There is also a child-
hood version of the same test, the Wechsler Intelligence Scale for Children.
Psychological Theory and Psychometric Testing
K&S Chapter 5

Question 102. E. Freud’s drive theory focused on basic instincts or drives that motivated human behavior. These drives
were libido and aggression. In Freud’s model, a drive has four parts. The “source” is the part of the body
from which the drive comes. The “impetus” is the amount of intensity of the drive. The “aim” is any
action that discharges the tension. The “object” is the target of the action. The other theories listed have
nothing to do with Freud. Self-psychology is the theory of Kohut. Learning theory cannot be attributed
to any one individual but has many theories and contributors. Conflict theory is a distractor, as is the
mesolimbic dopamine theory.
Psychological Theory and Psychometric Testing
K&S Chapter 4

Question 103. A. Numerous studies have shown the principal cause of intracerebral hemorrhage (ICH) to be hyperten-
sion. Chronic hypertension probably causes lipohyalinosis of the small intraparenchymal arteries and
Charcot–Bouchard microaneurysms that rupture owing to increased vascular pressure. ICH accounts for
about 10% of all strokes. The most common area of predilection for ICH is the putamen, in about one-
third of cases, followed by the thalamus in about 10% to 15% of cases. The other choices listed in this
question are all less frequent causes of ICH.
Neurology
B&D Chapter 51

Question 104. D. Aaron Beck is the originator of cognitive behavioral therapy. In this theory, patients’ assumptions affect
their cognitions, which in turn affect their mood. As such, it would be cognitive distortion that Beck
would most likely find as the cause of depression. The other answer choices may be things to which
other theorists attribute depression or are totally unrelated answer choices included to distract.
Psychotherapy
K&S Chapter 28

Question 105. A. A type I error occurs when the null hypothesis is rejected when it should have been retained. It is the equiva-
lent of saying that a true difference exists between two samples when the difference is due solely to chance.
Statistics
K&S Chapter 5

Question 106. E. Randomization is the process by which each patient in a clinical trial has an equal chance to be assigned
to a control group or an experimental group. This process protects against selection bias. Power is the
probability of finding the difference between two samples. It is the probability of rejecting the null
hypothesis when it should be rejected. Probability is the likelihood that an event will occur. A probability
of 1 means it will occur, a probability of 0 means that it will not. Risk is a distractor.
Statistics
K&S Chapter 5

Question 107. C. The clinical picture here is that of cryptococcal meningitis in an AIDS patient who is severely immu-
nocompromised. About 10% of AIDS patients develop this infection by the encapsulated yeast
Cryptococcus neoformans. The CD4 count is generally less than 200/μL. Although MRI of the brain is a
good test, in this case the results would be nonspecific. The scan might demonstrate meningeal enhance-
ment with gadolinium, suggesting a subacute or chronic meningitis. There may also be multiple small
abscesses seen on scan due to fungal invasion of the Virchow–Robin spaces surrounding meningeal ves-
sels. Hydrocephalus due to obstruction of CSF flow may also be seen. In rarer cases a mass lesion, or

52

Psychiatry Test Preparation and Review


Manual E-Book
Test Number One

cryptococcoma, with surrounding edema can be seen due to consolidation of the infection. The most
important immediate test is the lumbar puncture. Opening pressure should be measured and is usually
elevated. CSF is most often colorless and clear. CSF analysis can reveal a leukocytosis of 50 to 1000
cells/mm3 with lymphocytic predominance. CSF protein is usually elevated from 50 to 1000 mg/dL.
India ink staining of CSF viewed under the microscope will quickly reveal an identifiable capsule and
budding yeasts and requires no special laboratory machinery or testing. CSF cryptococcal antigen assay
is indeed more sensitive than India ink staining and should be done concomitantly, as it is now readily
available in most centers. Chest radiography would be helpful only with a suspicion of lung involvement
or pulmonary symptoms. Blood cultures are generally negative in fungal infection and should be done
only if concomitant bacterial infection is suspected. Amphotericin B intravenous administration is the
treatment of choice for central nervous system fungal infections. The problem with amphotericin B is a
high rate of up to 80% renal toxicity as a side effect.
Neurology
B&D Chapter 53

Question 108. D. Power is the probability of finding the difference between two samples. It is the probability of rejecting
the null hypothesis when it should be rejected. Randomization is the process by which each patient in a
clinical trial has an equal chance to be assigned to a control group or an experimental group. This pro-
cess protects against selection bias. Probability is the likelihood that an event will occur. A probability of
1 means it will occur, a probability of 0 means that it will not. Risk is a distractor.
Statistics
K&S Chapter 5

Question 109. C. The number of people who have a disorder at a specific point in time is the point prevalence. It is cal-
culated by dividing the number of people with the disorder at that time by the total population at that
time. Randomization is the process by which each patient in a clinical trial has an equal chance to be
assigned to a control group or an experimental group. This process protects against selection bias. Power
is the probability of finding the difference between two samples. It is the probability of rejecting the null
hypothesis when it should be rejected. Probability is the likelihood that an event will occur. A probability
of 1 means it will occur, a probability of 0 means that it will not. Risk is a distractor.
Statistics
K&S Chapter 5

Question 110. A. Avoidant/restrictive food intake disorder is an eating disturbance characterized by a lack of interest in eating
or food and which causes in a patient significant weight loss, significant nutritional deficiency, or a depen-
dence on enteral feeding or oral nutritional supplements. The patients do not meet criteria for anorexia
or bulimia, and notably there is not a disturbance in the way their body weight or shape is experienced.
Anorexia nervosa is an eating disorder that causes in a patient significantly low body weight that is less than
minimally normal or minimally expected. There is an intense fear of gaining weight or becoming fat, and
there is a disturbance in the way one’s body weight or shape is experienced. Anorexia can be described as
restricting type or as binge eating/purging type. In bulimia nervosa there are recurrent episodes of binge eat-
ing followed by inappropriate compensatory behaviors to prevent weight gain, such as self-induced vomiting,
misuse of laxatives or diuretics, fasting, or excessive exercise. An important differentiator from anorexia is
that the bulimic patient does not have less than minimally normal body weight. Similar to anorexia there
is a disturbance in how body shape or weight is experienced. In binge eating disorder there are recurrent
episodes of binge eating consisting of eating rapidly, eating until uncomfortably full, eating large amounts of
food when not hungry, eating alone because of embarrassment about eating, or feeling disgusted with oneself,
depressed, or guilty after eating. It is differentiated from bulimia nervosa by a lack of inappropriate compen-
satory behaviors, such as vomiting, laxative use, or excessive exercise. Atypical anorexia would be categorized
as “Other Specialized Feeding or Eating Disorder.” This descriptor is used when the patient doesn’t meet full
criteria for another eating disorder. In atypical anorexia all of the criteria for anorexia are met except that,
despite significant weight loss, the individual’s weight is still within or above the normal range.
Feeding and Eating Disorders
K&S Chapter 15

53

Psychiatry Test Preparation and Review


Manual E-Book
Psychiatry Test Preparation & Review Manual

Question 111. D. PCP can be found in the urine up to 8 days after use. Some other drugs of note include cannabis, up to 4
weeks; cocaine, up to 8 hours; and heroin, up to 72 hours.
Laboratory Tests in Psychiatry
K&S Chapter 20

Question 112. C. APP is the protein that makes up the amyloid plaques found in the brain in Alzheimer’s disease. The
protein is encoded by a gene found on chromosome 21. The amyloid deposits found in Alzheimer’s dis-
ease are the hallmark of the disease’s neuropathology. Wilson’s disease is the result of abnormal copper
metabolism, not amyloid. Schizophrenia, bipolar disorder, and HD have nothing to do with amyloid.
Neurocognitive Disorders
K&S Chapter 10

Question 113. C. A complete blood count (CBC) would be the first test to order because of the risk of significant side
effects on the hematopoietic system. Carbamazepine can cause decreased white blood cell count, agranu-
locytosis, pancytopenia, and aplastic anemia. Carbamazepine also has a vasopressin-like effect and can
cause water intoxication and hyponatremia. Carbamazepine interacts significantly with the cytochrome
P450 system and as such has many interactions with many drugs. Great care should be taken when pre-
scribing carbamazepine with other medications.
Laboratory Tests in Psychiatry
K&S Chapter 29

Question 114. E. Because the starvation associated with anorexia affects a multitude of organ systems, a battery of tests
is warranted when working up the disease. These include electrolytes, renal function tests, thyroid tests,
glucose, amylase, CBC, electrocardiogram, cholesterol, carotene level, and dexamethasone suppression
test. There is not an indication for a head CT, because one would not find changes on the head CT of
an anorexic patient that would differentiate it from a normal head CT. With respect to laboratory tests
and anorexia, a frequently tested fact is that in anorexia, levels of luteinizing hormone LH and follicle-
stimulating hormone FSH are decreased.
Laboratory Tests in Psychiatry
K&S Chapter 15

Question 115. B. Creutzfeldt–Jakob disease (CJD) is one of a number of human spongiform encephalopathies and is associ-
ated with prion infection. The worldwide incidence of CJD is about 0.5 to 1 in 1 million per year. A new
variant was thought to have developed during the late 1990s resulting from consumption of meat from
cattle infected with bovine spongiform encephalopathy. The clinical picture is that of a prodromal period
of vegetative symptoms, such as asthenia and sleep and appetite disturbances. This is followed by the onset
of a rapidly progressive dementia with deficits in memory and concentration, depression, self-neglect, and
personality changes. The condition progresses to global dementia over time, and death typically occurs
from 2 to 7 months after onset of symptoms. The diagnostic test of choice today is lumbar puncture with
CSF assay of 14-3-3 and tau proteins, the specificity and sensitivity of which exceed 90%. CT scans of the
brain are useless because they remain normal in a majority of cases. There may be atrophy seen on CT
scan with ventricular enlargement, but this is nonspecific and diagnostically unhelpful. MRI of the brain
may reveal atrophy with symmetrical increased signal intensity in the basal ganglia, which is again not
particularly helpful in diagnosing CJD. EEG is more helpful and is expected to reveal periodic sharp wave
complexes.
Neurology
B&D Chapter 53

Question 116. D. Disorders of smooth visual pursuit and disinhibition of saccadic eye movements are commonly found in
patients with schizophrenia. This has been proposed by some as a trait marker for schizophrenia because
it is found regardless of medication use and is also present in first-degree relatives. It is thought that the
eye movement disorders are the function of pathology in the frontal lobes.
Psychotic Disorders
K&S Chapter 7

54

Psychiatry Test Preparation and Review


Manual E-Book
Test Number One

Question 117. E. Many of the antipsychotic medications block dopamine in the tuberoinfundibular tract. Because of
this dopamine blockade, the patient develops an elevated prolactin level. That elevated prolactin level
leads to galactorrhea and amenorrhea. In the case given, the risperidone is the most likely cause of the
patient’s symptoms. You would want to check the serum prolactin level and adjust the risperidone dose
or consider switching the patient to another medication.
Laboratory Tests in Psychiatry
K&S Chapter 5

Question 118. C. Patients with borderline personality disorder have frequent mood swings. They can develop short-lived
psychotic episodes. They often cut or mutilate themselves to elicit help from others, to express anger, or
to numb themselves to strong affect. Both men and women can have borderline personality disorder,
though it is more common in women. The other answer choices do not fit the case as well as border-
line personality disorder. Schizoaffective disorder patients do not usually self-mutilate. Dysthymic disor-
der is not consistent with psychotic symptoms. There is no description of mania, so bipolar disorder is
unlikely. There is no acute stressor, so adjustment disorder doesn’t fit well. Whenever a question involves
cutting or self-mutilation, strongly consider borderline personality disorder. Psychopharmacologic treat-
ment of impulsive-aggressive behavior in borderlines involves SSRIs as first line and the addition of a
neuroleptic as second line. Third line would be the addition of, or switch to, a mood stabilizer.
Personality Disorders
K&S Chapter 22

Question 119. B. Of the choices given, the highest prevalence is for anxiety disorders. More than 30 million people in
the United States have an anxiety disorder. About 17.5 million have depression. About 2 million have
schizophrenia. About 5 million have dementia. About 12.8 million use illicit drugs.
Anxiety Disorders
K&S Chapter 9

Question 120. D. There is an association between pathological gambling and mood disorders, particularly major depres-
sive disorder (MDD). There is also an association with panic, OCD, and agoraphobia, but the asso-
ciation with MDD is greater. Criteria for pathological gambling include preoccupation with gambling,
gambling increased sums of money to obtain excitement, being unsuccessful at stopping or cutting back,
gambling to escape dysphoric mood, lying to significant others about gambling, loss of important rela-
tionships over gambling, committing illegal acts to be able to gamble, relying on others to pay the bills
because of money lost gambling, and a desire to keep going back to break even.
Disruptive, Impulse Control Disorders, Conduct Disorders, and ADHD
K&S Chapter 20

Question 121. C. This is a case of delusional disorder. In delusional disorder the patient has only delusions and does not
meet criteria for schizophrenia. Functioning in day-to-day life is relatively well preserved. The disor-
der may take various forms, such as erotomanic type, grandiose type, jealous type, persecutory type,
somatic type, or mixed type. These patients do not meet criteria for schizophrenia. There are no mood
symptoms, ruling out depression. They are not confused, disoriented, and waxing and waning in con-
sciousness, ruling out a delirium. There is no evidence of memory impairment, so that makes dementia
unlikely. Given all of this, the correct answer is delusional disorder.
Psychotic Disorders
K&S Chapter 7

Question 122. E. Illness anxiety disorder involves being convinced that one has a serious disease based on misinterpre-
tation of bodily sensations. The preoccupation with having the illness persists despite reassurance by
doctors. It causes clinically significant impairment in functioning. Somatoform disorders is a general cat-
egory that includes somatic symptom disorder, conversion disorder (functional neurological symptom
disorder), and illness anxiety disorder. Factitious disorder is when a patient feigns illness for primary
gain (i.e., benefits of the sick role). Conversion disorder (functional neurological symptom disorder) is
the development of a neurologic deficit as a result of psychological conflict. Somatic symptom disorder

55

Psychiatry Test Preparation and Review


Manual E-Book
Psychiatry Test Preparation & Review Manual

with predominant pain manifests the presence of pain as the predominant clinical focus, and the pain is
thought to be substantially mediated by psychological factors.
Somatic Symptom Disorders
K&S Chapter 13

Question 123. E. There are multiple studies that all point to a genetic predisposition for alcoholism. The studies that sepa-
rate environmental from genetic factors are some of the most convincing. Studies of adoptees clearly
demonstrate that children whose biological parents were alcoholics are at increased risk for alcoholism,
even when brought up by adopted families in which neither parent has an alcohol problem. In addition,
children whose biological parents do not have an alcohol problem are not more likely to become alco-
holic if raised in a home with parents who have alcohol problems.
Substance Abuse and Addictive Disorders
K&S Chapter 20

Question 124. C. This is a psychogenic seizure (also called nonepileptic seizure). Keys to a psychogenic seizure, or pseudo-
seizure, are lack of an aura, no cyanotic skin changes, no self-injury, no incontinence, no postictal con-
fusion, asynchronous body movements, absent EEG changes, and seizure activity being affected by the
suggestion of the doctor. Prolactin will not be elevated in a psychogenic seizure, as it often is with an epi-
leptic seizure. One of the best ways to tell the difference between an epileptic seizure and a psychogenic
seizure is video EEG monitoring.
Neurology
K&S Chapter 10

Question 125. B. Internuclear ophthalmoplegia is a classic brainstem finding on neurologic examination of patients with
demyelinating lesions of multiple sclerosis (MS). The lesion localizes to the medial longitudinal fascicu-
lus (MLF) of the brainstem. The deficit involves abnormal horizontal ocular movements with absence
or delayed adduction of the eye ipsilateral to the MLF lesion and coarse horizontal nystagmus in the
abducting eye. Convergence is preserved. Bilateral internuclear ophthalmoplegia is highly suggestive
of MS but can also be seen with other brainstem lesions, particularly Arnold–Chiari malformation,
Wernicke’s encephalopathy, vascular lesions, and brainstem gliomas.
Neurology
B&D Chapter 54

Question 126. B. In disinhibited social engagement disorder, a child actively approaches and interacts with unfamiliar adults.
There is reduced or absent reticence in approaching unfamiliar adults, overly familiar verbal or physical
behavior, diminished or absent checking back with adult caregivers after venturing away even in unfamiliar
settings, and a willingness to go off with unfamiliar adults with minimal or no hesitation. These behaviors
are not limited to impulsivity, as would be found in ADHD. The child has experienced a pattern of insuf-
ficient care, such as social neglect or deprivation, repeated changes of primary caregivers that impair forma-
tion of stable attachments, or rearing in unusual settings that severely limit opportunities to form selective
attachments. In PTSD the patient has experienced actual or threatened death or serious injury, which leads
to symptoms of intrusion, avoidance, negative alterations in cognition and mood, and alterations in arousal
and reactivity. In reactive attachment disorder the patient would be the opposite of what was described in the
question. He or she would have a pattern of inhibited, emotionally withdrawn behavior toward adults and
minimal social or emotional responsiveness to others. However, he or she would share a history of insufficient
care characterized by social neglect or deprivation, change in primary caregivers, or rearing in unusual set-
tings. Rumination disorder is one of the feeding and eating disorders characterized by repeated regurgitation
of food, which may be rechewed, reswallowed, or spat out. This behavior must persist for at least 1 month to
meet criteria, and the patient cannot meet criteria for another eating disorder.
Trauma and Stress Related Disorders
K&S Chapter 31

Question 127. E. Pathological gambling is categorized by the DSM 5 as a non–substance-related addictive disorder
found under Substance-Related and Addictive Disorders. Criteria for pathological gambling include

56

Psychiatry Test Preparation and Review


Manual E-Book
Test Number One

preoccupation with gambling, gambling increased sums of money to obtain excitement, being unsuc-
cessful at stopping or cutting back, gambling to escape dysphoric mood, lying to significant others about
gambling, loss of important relationships over gambling, committing illegal acts to be able to gamble,
relying on others to pay the bills because of money lost gambling, and a desire to keep going back to
break even.
Substance-Related and Addictive Disorders
K&S Chapter 20

Question 128. E. Dissociative identity disorder is characterized by a disruption of identity involving two or more distinct per-
sonality states. This disruption involves a marked discontinuity in sense of self and sense of agency accompa-
nied by related alterations in affect, behavior, consciousness, memory, perception, cognition, and/or sensory
motor functioning. Patients experience recurrent gaps in the recall of everyday events, important personal
information, and/or traumatic events that are inconsistent with ordinary forgetting. Dissociative amnesia
is characterized by an inability to recall important autobiographical information, usually of a traumatic or
stressful nature. It most often consists of localized or selective amnesia for a specific event or events but can
present as generalized amnesia for identity and life history. Dissociative fugue is travel or wandering that is
associated with amnesia for identity or other important biographical information. Depersonalization/dereal-
ization disorder involves the persistent or recurrent experiences of depersonalization, derealization, or both.
Depersonalization is an experience of unreality, detachment, or feeling like an outside observer with respect to
one’s own thoughts, feelings, sensations, body, or actions. Derealization is an experience of unreality or detach-
ment with respect to one’s surroundings in which individuals or objects are experienced as unreal, dreamlike,
foggy, lifeless, or visually distorted. During depersonalization/derealization disorder, reality testing remains
intact. Borderline personality consists of a pervasive pattern of instability of interpersonal relationships, self-
image, and affects and marked impulsivity beginning by early adulthood and present in a variety of contexts.
Dissociative Disorders
K&S Chapter 12

Question 129. D. Within an hour of PCP intoxication, patients may develop vertical or horizontal nystagmus, hyper-
tension or tachycardia, numbness or diminished responsiveness to pain, ataxia, dysarthria, muscular
rigidity, seizures or coma, or hyperacusis. Depressed reflexes are often seen with inhalant intoxication.
Conjunctival injection is often seen with cannabis intoxication. Slurred speech and memory impairment
are often associated with alcohol intoxication.
Substance-Related and Addictive Disorders
K&S Chapter 20

Question 130. B. Depersonalization is an experience of unreality, detachment, or feeling like an outside observer with respect
to one’s own thoughts, feelings, sensations, body, or actions. Derealization is an experience of unreality or
detachment with respect to one’s surroundings in which individuals or objects are experienced as unreal,
dreamlike, foggy, lifeless, or visually distorted. Fugue involves having amnesia for your identity and assum-
ing a new identity. It usually also involves wandering to new places. Amnesia is the inability to recall past
experiences. Anosognosia is an inability to recognize a neurological deficit that is occurring to oneself.
Dissociative Disorders
K&S Chapter 12

Question 131. E. Inhalants can cause a persisting dementia. It is irreversible, except for the mildest cases. It may be the result
of the neurotoxic effects of the inhalants, the metals they contain, or the effects of hypoxia. Inhalant use
can also lead to delirium, psychosis, and mood and anxiety disorders. Signs of intoxication with inhalants
include maladaptive behavior, such as assaultiveness; impaired judgment; and neurological signs, such as
dizziness, slurred speech, ataxia, tremor, blurred vision, stupor, and coma. The other answer choices have
various effects but do not cause a persisting dementia. One of our answer choices is LSD. Keep in mind
that LSD is thought to work as a partial agonist at postsynaptic serotonin receptors. LSD use therefore
alters postsynaptic serotonin binding. This is a useful little fact for the standardized test taker to know.
Substance Abuse and Addictive Disorders
K&S Chapter 20

57

Psychiatry Test Preparation and Review


Manual E-Book
Psychiatry Test Preparation & Review Manual

Question 132. E. Social skills training is an important part of psychiatric rehabilitation. Social skills are behaviors necessary for
survival in the community. These are disrupted by severe illnesses, such as schizophrenia. Social skills training
has proven important in correcting deficits in patients’ behaviors. Severely ill patients make slow progress but
can learn some necessary skills that enable them to engage in conversation and decrease social anxiety. Social
skills training can be done in both a group and an individual format. The other answer choices in this ques-
tion consist of unrelated pairs, some of which border on the ridiculous and are distractors.
Psychosocial Interventions
K&S Chapter 28

Question 133. D. MS is the most common inflammatory demyelinating disease. The classic onset of the disease is between the
ages of 15 and 50 years. About two-thirds of patients have the relapsing–remitting form of the disease at
onset, which is the most common form of the illness. Only about 20% of patients have primary progressive
disease at onset. Optic neuritis (ON) is a common sign of MS and is frequently the cause of initial presenting
symptoms. ON usually presents with eye pain that increases with eye movement, followed by central visual
loss (scotoma) in the affected eye. ON patients will have a relative afferent pupillary defect (Marcus Gunn
pupil). This is tested by the swinging flashlight test, which demonstrates that the abnormal pupil paradoxi-
cally dilates when a light is moved away from the normal to the affected eye. Internuclear ophthalmoplegia
is a common sign of MS and involves a lesion in the MLF of the brainstem that produces a characteristic eye
movement abnormality. The eye ipsilateral to the lesions cannot adduct past the midline, whereas the contra-
lateral eye fully abducts and displays a coarse end-gaze nystagmus. The finding can sometimes be bilateral.
Fatigue is a common complaint in patients with MS. It often has little to do with the amount of physical exer-
tion carried out by the patient. It may occur upon waking despite a good night’s sleep the night before. Heat
sensitivity is a well-described phenomenon in MS. Increases in core body temperature can bring on symp-
toms or worsen already-existing symptoms. This is known as Uhthoff’s phenomenon. The condition is due
to a conduction block that occurs as body temperature rises. Lhermitte’s sign is a transient neurologic sign
described by patients as a sensation of an electric shock that descends down the spine or the extremities upon
neck flexion. It is most often suggestive of MS but can also be seen in other conditions involving the cervical
spinal cord, such as disk herniations, trauma, and tumors.
Neurology
B&D Chapter 54

Question 134. C. Bulimia is categorized by a recurrent pattern of binge eating and self-induced vomiting. Bulimic patients
often develop a hypochloremic alkalosis and are at risk for gastric and esophageal tears. Dehydration
(hence low blood pressure) and electrolyte imbalances are likely. Many female bulimic patients have
menstrual disturbances. Russell’s sign is positive when cuts or scrapes to the backs of the hands are
noted, which are a result of the teeth scraping the fingers while vomiting.
Feeding and Eating Disorders
K&S Chapter 15

Question 135. D. Amphetamine intoxication presents with euphoria, anxiety, anger, hypervigilance, and impaired judg-
ment and functioning. The effects are similar to those of cocaine. There is a risk for an amphetamine-
induced psychotic disorder as well, which is characterized by paranoia. One can also note visual
hallucinations, hypersexuality, hyperactivity, confusion, and incoherence.
Substance Abuse and Addictive Disorders
K&S Chapter 20

Question 136. B. Acute stress disorder is characterized by symptoms similar to those of PTSD, but with a different time frame.
Symptoms occur for a minimum of 2 days and a maximum of 4 weeks, and they begin within 4 weeks of the
traumatic event. The patient must have undergone a traumatic event, witnessed a traumatic event, learned
that a traumatic event happened to a close family member or friend, or experienced repeated or extreme
exposure to aversive details of a traumatic event. The patient then may experience emotional numbing, lack
of awareness of surroundings, derealization, depersonalization, dissociative amnesia, flashbacks, avoidance of
stimuli that remind him or her of the event, anxiety, irritability, increased arousal, or poor sleep.
Trauma and Stress-Related Disorders
K&S Chapter 11

58

Psychiatry Test Preparation and Review


Manual E-Book
Test Number One

Question 137. C. Children who are depressed can often present with irritability instead of, or in addition to, depressed
mood. Prepubertal children can report somatic complaints, psychomotor agitation, and mood-congruent
hallucinations. Depressed children can also fail to make expected weight gains. Other signs of depres-
sion that children can present with include school phobia and excessive clinging to parents. Teens with
depression often report poor school performance, substance abuse, promiscuity, antisocial behavior, tru-
ancy, and running away from home. They can withdraw from social activities and be grouchy and sulky.
Depressive Disorders
K&S Chapter 31

Question 138. D. Schizoid personality disorder is characterized by a pervasive pattern of detachment from social relation-
ships. The patient neither desires nor enjoys close relationships. They choose solitary activities. They lack
close friends or romantic relationships. They are indifferent to the opinions of others and are emotion-
ally cold and detached. Some of the other choices in this question are references to schizotypal person-
ality disorder. In the schizotypal patient, there are ideas of reference, magical thinking, paranoia, and
excessive social anxiety that is fueled by paranoid thinking.
Personality Disorders
K&S Chapter 22

Question 139. A. Beneficence is the duty to do no harm to the patient. Autonomy is the duty to protect a patient’s freedom
to choose. Autonomy theory views the relationship between patient and doctor as between two adults,
not as parent and child. Justice in this context means a fair distribution and application of services.
Validity is a statistical word meaning that a test measures what it claims to measure.
Ethics
K&S Chapter 36

Question 140. D. Tourette’s disorder often involves both motor and vocal tics. The onset is usually around 7 years of
age but may come as early as 2 years. Motor tics usually start in the face and head and progress down
the body. Vocal tics are not done intentionally to provoke others but are the result of sudden, intrusive
thoughts and urges that the patient cannot control. These intrusive thoughts may involve socially unac-
ceptable subject matter or obscenity.
Disruptive, Impulse Control Disorders, Conduct Disorders, and ADHD
K&S Chapter 46

Question 141. D. Substance abusers have the highest risk of becoming violent. Large doses of alcohol promote aggres-
sion, as do large doses of barbiturates. Paradoxical aggression can be observed with anxiolytics. Opioid
dependence is associated with increased aggression. Stimulants, cocaine, hallucinogens, and sometimes
cannabis can also lead to aggression. Aggressive behavior is more likely with those who have become
acutely psychologically decompensated. More than half of people who commit homicide and engage
in assaultive behavior are under the influence of significant amounts of alcohol at the time the crime is
committed. Although many major psychiatric disorders can lead to aggression, you are more likely to
face substance-induced aggression simply because of the sheer number of cases of aggression and vio-
lence that are substance-induced.
Substance Abuse and Addictive Disorders
K&S Chapter 20

Question 142. C. An ideal patient for psychodynamic psychotherapy should have the capacity for psychological mind-
edness, have at least one meaningful relationship, be able to tolerate affect, respond well to transfer-
ence interpretation, be highly motivated, have flexible defenses, and lack tendencies toward splitting,
projection, or denial. A useful screening tool for whether a patient has these characteristics is to under-
stand the quality of their relationships, as the aforementioned qualities often contribute to productive
relationships.
Psychotherapy
K&S Chapter 28

59

Psychiatry Test Preparation and Review


Manual E-Book
Psychiatry Test Preparation & Review Manual

Question 143. E. The clinical picture and scan are classic for MS. The MRI scan reveals
numerous subcortical white matter demyelinating lesions that are typi-
cal of MS. The lesions would be expected to enhance with gadolinium
contrast early on during an attack, and enhancement can persist up to 8
weeks after an acute attack. The treatment of an acute attack is gener-
ally with intravenous corticosteroids. This may or may not be followed
with an oral prednisone taper. Antibiotics such as ceftriaxone have no
place in MS. Intravenous immunoglobulin therapy and plasmapheresis
are treatments for MG and Guillain–Barré syndrome and not for MS.
Aspirin and heparin therapies are generally instituted in the emergency
room setting for acute ischemic stroke when recombinant tissue plas-
minogen activator cannot be given.
Neurology
B&D Chapter 54

Question 144. A. Borderline patients often cut, self-mutilate, and make suicide attempts. The patient in question has made
past suicide attempts, and past attempts are the best predictor of future attempts. She is emotionally
labile after an interpersonal conflict. She is already doing harm to herself through cutting and is becom-
ing psychotic. All of these factors add up to one very important point: this patient is highly unpredict-
able and could very easily kill herself. The only reasonable answer choice is hospital admission, by which
her impulsive, self-destructive, and self-mutilating impulses can be limited and her behavior observed.
The other answer choices do not take her unpredictability and self-destructiveness seriously enough. The
choice for extended inpatient stay is wrong because you have no way of knowing how long she is going
to need to stay based on the information given. She could potentially stabilize in a few days and be safe
for discharge. She could also be in the hospital for several months. There is no way to predict length of
stay based on the question stem.
Management in Psychiatry
K&S Chapter 22

Question 145. E. Research in recent years has found that depression after a heart attack increases the likelihood of
another heart attack. There has been evidence to suggest that there are serotonin receptors on the sur-
face of platelets that can modify and reduce platelet aggregation and thereby reduce heart attack risk.
The prescription of an SSRI antidepressant after a myocardial infarction has been shown to increase the
amount of serotonin in the body as a whole. This in turn modulates platelet serotonin receptors, thus
decreasing platelet aggregation and making a future heart attack less likely.
Somatic Symptom Disorders
K&S Chapter 29

Question 146. B. NF1 (von Recklinghausen’s disease) is caused by a mutation in the NF1 gene on chromosome 17. This
can be remembered by the mnemonic “von Recklinghausen has 17 letters.” NF1 is the most common of
the neurocutaneous illnesses, occurring in about 1 in 3000 individuals. NF2 is caused by a mutation in
the NF2 gene on chromosome 22. This is remembered by the mnemonic “Chromosome 22 carries NF2.”
It is less common than NF1 and appears in about 1 in 50,000 individuals. Patients with NF1 need to
have any two of the following seven criteria to carry the diagnosis: six or more café-au-lait spots, axil-
lary or inguinal freckling, optic glioma, two or more neurofibromas or one plexiform neurofibroma, a
first-degree relative with NF1, two or more Lisch nodules (hamartomas of the iris), and characteristic
bony lesion, such as thinning of long bones or sphenoid dysplasia. Patients with NF2 must have bilateral
acoustic schwannomas to meet criteria for this condition. If the schwannoma is unilateral, the patient
meets criteria only with a first-degree relative with NF2.
Neurology
B&D Chapter 65

Question 147. E. Both delirium and dementia can present with sleep problems, disorientation as to place, violent behavior,
and hallucinations. The hallmark of delirium, however, is alteration of consciousness. Criteria include

60

Psychiatry Test Preparation and Review


Manual E-Book
Test Number One

disturbance of consciousness with reduced ability to sustain attention, changes in cognition (memory
problems, language disturbance, disorientation), and perceptual disturbances. These develop over a short
period of time and can fluctuate during the course of a day. Dementia, on the other hand, consists of
multiple cognitive deficits, including memory loss, aphasia, apraxia, agnosia, and disturbance of execu-
tive function.
Neurocognitive Disorders
K&S Chapter 21

Question 148. D. The first step in treating a sleep problem is to rule out any problems in the environment that could cause
insomnia and to alter the environment to make it more conducive to sleep. This approach starts with
the rule that the bed is to be used for sleep and sex only. Reading in bed or watching television in bed
should not be permitted. If this should fail, then pharmacologic aids can be pursued. A sleep study is not
warranted by a simple complaint of insomnia. That would be overkill. Of course, a detailed history is
the best tool to determine whether a more serious sleep disturbance is present.
Sleep Wake Disorders
K&S Chapter 16

Question 149. D. Models of collaborative care between psychiatry and primary care physicians differ, but many involve
patients being seen in a general medical clinic. This reduces the stigma associated with going to mental
health treatment, and patients tend to be more open to receiving help. The epidemiologic catchment area
study found that about half of general medical outpatients had some form of psychiatric symptoms.
Managed care organizations often “carve out” psychiatric coverage making psychiatrist referral compli-
cated and time consuming. Limited formularies with onerous prior authorization requirements act as a
further barrier to care. Most primary care physicians have 10 to 15 minutes to see each patient, which
is not sufficient to adequately explore psychiatric symptoms or detect the psychiatric underpinnings of
vague somatic complaints. Primary care physicians are not trained in the use of complex regimens of
psychiatric medications and often use suboptimal doses of medications, lacking in comfort to use higher
doses. Several studies have demonstrated better outcomes for patients treated collaboratively by the pri-
mary care physician and psychiatrist.
Management in Psychiatry
MGH Comprehensive Clinical Psychiatry, Chapter 59

Question 150. D. Methylenedioxyamphetamine (MDMA) is also known as ecstasy. It is in the amphetamine family and
is a common drug of abuse at clubs and raves. Symptoms of intoxication with amphetamines include
euphoria, changes in sociability, hypervigilance, changes in interpersonal sensitivity, anxiety, anger, and
impaired judgment. Amphetamines can induce a psychosis that includes paranoia, hyperactivity, and
hypersexuality. Physical effects include fever, headache, cyanosis, vomiting (leading to dehydration),
shortness of breath, ataxia, and tremor. More serious effects can include myocardial infarction, severe
hypertension, and ischemic colitis. Cannabis intoxication presents as impaired coordination, eupho-
ria or anxiety, sense of slowed time, social withdrawal, and impaired judgment. Physical signs include
conjunctival injection, increased appetite, tachycardia, and dry mouth. Ketamine is a relative of PCP.
Intoxication presents as belligerence, impulsivity, psychomotor agitation, and impaired judgment.
Physical signs include nystagmus, hypertension, ataxia, dysarthria, or muscle rigidity. Psychosis may be
present and can persist for up to 2 weeks after intoxication. Diacetylmorphine is heroin. Intoxication
results in euphoria followed by apathy, psychomotor agitation or retardation, impaired judgment, pupil-
lary dilation, sedation, slurred speech, and impaired attention or memory. Volatile inhalant intoxication
presents as belligerence, assaultiveness, apathy, impaired judgment, dizziness, nystagmus, impaired coor-
dination, unsteady gait, lethargy, tremor, psychomotor retardation, muscle weakness, euphoria, or coma.
Low doses of these substances can cause feelings of euphoria. High doses can cause paranoia, fearful-
ness, and hallucinations.
Substance Abuse and Addictive Disorders
K&S Chapter 20

61

Psychiatry Test Preparation and Review


Manual E-Book
Psychiatry Test Preparation & Review Manual

TWO
Test Number Two
1. Which one of the following is false?

A. 
Carl Jung focused on the growth of the personality and individuation
B. 
Harry Stack Sullivan saw human development as a function of social interaction
C. 
Erik Erikson developed a model of the life cycle that spanned from childhood to old age
D. 
Jean Piaget developed a theory of cognitive development
E. 
The work of Freud, Jung, and Erikson was a function of carefully crafted psychological and neurodevelopmental
studies

2. A 75-year-old woman presents to the emergency room with an acute onset of right hemisensory loss, mild right hemi-
paresis, and a right-sided Babinski sign. Upon examination of her mental status you note that she cannot repeat simple
phrases; she can follow simple task instructions both verbal and on paper; she cannot write well; and she is having
word-finding difficulties with multiple paraphasic errors. This clinical picture is consistent with:

A. 
Broca’s aphasia
B. 
Wernicke’s aphasia
C. 
Transcortical sensory aphasia
D. 
Conduction aphasia
E. 
Transcortical motor aphasia

3. Which one of the following is not a goal of collaborative care between psychiatry and primary care physicians?

A. 
Improved access
B. 
Improved formularies
C. 
Improved treatment
D. 
Improved outcomes
E. 
Improved communication

Full test - and additional VIDEO vignettes - available online - see inside front cover for details.

62

Psychiatry Test Preparation and Review


Manual E-Book
Test Number Two

4. Correcting hyponatremia too rapidly with hypertonic saline replacement can result in:

A. 
Guillain–Barré syndrome
B. 
Acute thalamic hemorrhage
C. 
Acute demyelinating encephalomyelitis
D. 
Acute cerebellar syndrome
E. 
Acute locked-in syndrome

5. A patient comes into your office and explains away why he beat his brother with a baseball bat. He gives several exam-
ples of how his brother had mistreated him in the past and says that if he had not given him this beating, the mistreat-
ment would have continued. Which of the following defenses does this represent?

A. 
Projection
B. 
Blocking
C. 
Externalization
D. 
Rationalization
E. 
Denial

6. A 34-year-old obese African American woman presents to the emergency room with a complaint of 6 weeks of inter-
mittent bifrontal headache and vague visual obscurations. She is on oral contraceptive medication and has a history of
being on tetracycline therapy for a recent sexually transmitted disease. The immediate diagnostic test of choice in the
emergency room is:

A. 
Noncontrast head computed tomography (CT) scan
B. 
Lumbar puncture with cerebrospinal fluid (CSF) opening pressure
C. 
Brain magnetic resonance imaging (MRI) without gadolinium
D. 
Serum sedimentation rate (ESR)
E. 
Serum prolactin level

7. Which of the following organizations is made up of family members of the mentally ill?

A. 
American Association for Mental Health
B. 
National Mental Health Assembly
C. 
National Alliance for the Mentally Ill
D. 
Council for Mental Health Reform
E. 
Association for the Advancement of Psychotherapy

8. Which one of the following is not a diagnostic criterion of migraine without aura?

A. 
Headache must last 4 to 72 hours
B. 
Pulsatile quality
C. 
Photophobia
D. 
Nausea and vomiting
E. 
Mild to moderate intensity

63

Psychiatry Test Preparation and Review


Manual E-Book
Psychiatry Test Preparation & Review Manual

9. What is the lifetime prevalence of schizophrenia?

A. 
10%
B. 
5%
C. 
1%
D. 
0.5%
E. 
0.1%

10. Which one of the following is not a contraindication to the use of recombinant tissue plasminogen activator (r-TPA) in
acute ischemic stroke?

A. 
Stroke occurrence 2 hours before r-TPA administration
B. 
Major surgery within 2 weeks of r-TPA administration
C. 
Uncontrolled hypertension
D. 
Prothrombin time (PT) >15
E. 
CT shows signs of intracranial hemorrhage

11. Which of the following is based on active outreach to patients in the community?

A. 
Traditional social work
B. 
Assertive community treatment
C. 
Day hospitals
D. 
Psychiatric rehabilitation
E. 
Electroconvulsive therapy

12. A 45-year-old woman presents to your office complaining of long-standing lower extremity discomfort, particularly at
night before sleep onset. She reports shooting pains in the lower extremities that are relieved upon standing or walking.
The discomfort is described as a “crawling” sensation. The treatment of choice for her condition is:

A. 
Sertraline
B. 
Cyproheptadine
C. 
Ropinirole
D. 
Levetiracetam
E. 
Suvorexant

13. Which of the following is the best diagnostic procedure to determine if a 12-year-old boy is depressed?

A. 
Minnesota Multiphasic Personality Inventory (MMPI)
B. 
Scholastic achievement test
C. 
Dexamethasone suppression test
D. 
Face-to-face interview with the child
E. 
Interview the child’s teacher by phone

64

Psychiatry Test Preparation and Review


Manual E-Book
Test Number Two

14. A 65-year-old man presents to the emergency room with acute onset of vertigo, nausea, vomiting, dysarthria, and nys-
tagmus. On further examination, he is noted to have loss of pain and temperature sensation to the left-hand side of his
face. He has right-sided loss of pain and temperature sensation to his trunk and leg. He has a left Horner’s syndrome
and falls to his left-hand side when you ask him to walk, and he has left finger-to-nose dysmetria. You diagnose an acute
stroke, which is most likely localized to the:

A. 
Left hemisphere
B. 
Left lateral medulla
C. 
Left pons
D. 
Right pons
E. 
Right lateral medulla

15. Which of the following is associated with violence and aggression?

A. 
Blunted response to corticotropin-releasing hormone (CRH) stimulation test
B. 
Blunted growth hormone response to hypoglycemia
C. 
Decreased 5-hydroxyindoleacetic acid (5-HIAA) in the CSF
D. 
Decreased dopamine in the CSF
E. 
Increased levels of norepinephrine in the CSF

16. Metachromatic leukodystrophy is inherited by an _____ pattern of inheritance and results in a deficiency in _____:

A. 
Autosomal recessive; hexosaminidase A
B. 
Autosomal dominant; hexosaminidase A
C. 
Autosomal dominant; arylsulfatase A
D. 
Autosomal recessive; arylsulfatase A
Autosomal recessive; galactocerebroside β-galactosidase
E. 

17. Which one of the following is not true with respect to seasonal affective disorder (SAD)?

A. 
Patients are likely to respond well to light therapy
B. 
The “with seasonal pattern” specifier can be applied to bipolar I, bipolar II, and major depressive disorders accord-
ing to the Diagnostic and Statistical Manual of Mental Disorders (DSM)
C. 
It is not necessary to have full remission of symptoms at other times of the year to make this diagnosis
D. 
SAD involves a regular temporal relationship between the onset of symptoms and the time of year
E. 
The physician must demonstrate at least two depressive episodes at the same time of year to make the diagnosis

18. Which one of the following primitive reflexes is not generally expected to disappear by about 6 months of age?

A. 
Rooting
B. 
Moro
C. 
Palmar grasp
D. 
Parachute response
E. 
Tonic neck reflex

65

Psychiatry Test Preparation and Review


Manual E-Book
Psychiatry Test Preparation & Review Manual

19. Which of the following would fall under the heading of “Other Specified Somatic Symptom and Related Disorders”?

A. 
A patient with pain in one or more areas that is thought to be significantly mediated by psychological factors
B. 
A patient with a persistent belief that he or she has cancer despite reassurance by his or her physician that nothing is
wrong
C. 
A patient who develops a motor deficit following significant psychological stressors
D. 
A patient who feels that she is pregnant and presents with amenorrhea, enlarged abdomen, and breast engorgement
but a negative pregnancy test
E. 
A patient who has medical complaints involving pain, gastrointestinal complaints, neurological complaints, and
sexual complaints, but no medical explanation can be found for these symptoms

20. An AIDS patient presents with decreased visual acuity. The most likely offending infectious agent responsible for this
presentation is:

A. 
Cytomegalovirus
B. 
Toxoplasmosis
C. 
Tuberculosis (Mycobacterium)
D. 
Cryptococcus neoformans
E. 
JC virus

21. Which one of the following is not true regarding schizophrenia?

A. 
The disorder is chronic and usually has a prodromal phase
B. 
Eugen Bleuler coined the term schizophrenia
C. 
The patient’s overall functioning declines or fails to reach the expected level
D. 
The most frequent hallucinations are olfactory
E. 
Social withdrawal and emotional disengagement are common

22. Which one of the following is not seen in narcolepsy?

A. 
Cataplexy
B. 
Nighttime awakening
C. 
Excessive daytime sleepiness
D. 
Sleep paralysis
E. 
Hypnagogic hallucinations

23. Uncontrollable excessive talking, as seen in mania, is also known as:

A. 
Alexithymia
B. 
Logorrhea
C. 
Echolalia
D. 
Flight of ideas
E. 
Stilted speech

66

Psychiatry Test Preparation and Review


Manual E-Book
Test Number Two

24. Which one of the following is not an appropriate therapy for status epilepticus?

A. 
Rectal diazepam
B. 
Intravenous lorazepam
C. 
Intramuscular phenytoin
D. 
Intravenous valproic acid
E. 
Oxygen by nasal cannula with airway protection

25. Which one of the following statements is true regarding atypical antipsychotics?

A. 
Ziprasidone is an agonist at the serotonin (5-HT) 1A receptor and an inhibitor of reuptake of both serotonin and
norepinephrine
B. 
Risperidone is a significantly weaker antagonist of the dopamine D2 receptor than haloperidol
C. 
Quetiapine is known for its high incidence of extrapyramidal symptoms
D. 
Olanzapine has been associated with weight loss in the majority of patients
E. 
Clozapine has been shown to increase suicidality in chronically ill patients

26. Which of the following anticonvulsant agents is most appropriate for primary generalized seizures, including tonic–
clonic, absence, atonic, and myoclonic seizure types?

A. 
Divalproex sodium
B. 
Phenytoin
C. 
Oxcarbazepine
D. 
Carbamazepine
E. 
Valbenazine

27. Which of the following is not a side effect of the tricyclic antidepressants?

A. 
Tachycardia
B. 
Prolonged PR interval
C. 
Prolonged QRS interval
D. 
Orthostatic hypotension
E. 
Diarrhea

28. The L5 motor nerve root innervates the nerves responsible for:

A. 
Foot extension
B. 
Foot flexion
C. 
Leg extension
D. 
Hip flexion
E. 
The ankle jerk reflex

29. Which one of the following antidepressants can be used as an antipruritic agent and for the treatment of gastric ulcer
because of its potent histamine blockade?

A. 
Trazodone
B. 
Fluoxetine
C. 
Vilazodone
D. 
Amitriptyline
E. 
Amoxapine

67

Psychiatry Test Preparation and Review


Manual E-Book
Psychiatry Test Preparation & Review Manual

30. A patient involved in a car accident is found on MRI to have a spinal fracture and a partial crush lesion to the cervical
spinal cord that effectively causes a functional hemisection of the cord. His deficits would be expected to include:

A. 
Contralateral loss of motor control and pain and temperature sensation with ipsilateral loss of proprioception and
vibration sensation
B. 
Ipsilateral loss of motor control and pain and temperature sensation with contralateral loss of proprioception and
vibration sensation
C. 
Ipsilateral loss of motor control and proprioception and vibration sensation with contralateral loss of pain and tem-
perature sensation
D. 
Contralateral loss of motor control and proprioception and vibration sensation with ipsilateral loss of pain and
temperature sensation
E. 
Ipsilateral loss of motor control and contralateral loss of proprioception, vibration, pain, and temperature sensation

31. A child is able to use some symbols and language. Her reasoning is intuitive. She is unable to think logically or deduc-
tively. Which of Piaget’s stages does this child fit into?

A. 
Sensorimotor
B. 
Preoperational thought
C. 
Concrete operations
D. 
Formal operations
E. 
Trust vs mistrust

32. A 72-year-old man suffers a stroke with loss of motor functioning in


the left leg and, to a lesser extent, the left arm. He has abulia, and his
eyes and head seem preferentially deviated to the right. His left arm is
apraxic. His head CT is shown. The arterial territory involved is that of
the:

A. 
Right middle cerebral artery
B. 
Right posterior cerebral artery
C. 
Right vertebral artery
D. 
Right anterior cerebral artery
E. 
Right posterior communicating artery

33. Which of the following is true regarding norepinephrine (NE) and/or the locus ceruleus?

A. 
NE is synthesized in the locus ceruleus
B. 
Dopamine is synthesized in the locus ceruleus, NE in the dorsal raphe nuclei
C. 
Acetylcholine is synthesized with NE in the substantia nigra
D. 
5-HT is synthesized in the locus ceruleus
E. 
The locus ceruleus is the site of the formation of serotonin

68

Psychiatry Test Preparation and Review


Manual E-Book
Test Number Two

34. Pure motor hemiparesis is most likely to result from a stroke localized to the:

A. 
Midbrain
B. 
Cerebellum
C. 
Medulla
D. 
Thalamus
E. 
Internal capsule

35. Which one of the following receptor types is associated with weight gain and sedation?

A. 
5-HT 2A
α1
B. 
C. 
5-HT 1A
D. 
H1
E. 
M1

36. The lesion causing a left-arm hemiballismus would most likely localize to the:

A. 
Right subthalamic nucleus
B. 
Left subthalamic nucleus
C. 
Right putamen
D. 
Left putamen
E. 
Right globus pallidus interna

37. Which of the following is not a biogenic amine neurotransmitter?

A. 
Dopamine
γ-Aminobutyric acid (GABA)
B. 
C. 
Epinephrine
D. 
Acetylcholine
E. 
Serotonin

38. The mechanism of action by which reserpine both improves the symptoms of adult-onset primary focal dystonia and
can cause depression is:

A. 
Direct postsynaptic dopamine antagonism
B. 
Direct postsynaptic serotonin agonism
C. 
Direct postsynaptic serotonin antagonism
D. 
Presynaptic dopaminergic depletion
E. 
Direct postsynaptic cholinergic antagonism

39. In the psychotic patient, the defense mechanism of projection takes the form of:

A. 
Feelings of persecution
B. 
Feelings of abandonment
C. 
Feelings of sadness
D. 
Feelings of gratification
E. 
Feelings of isolation

69

Psychiatry Test Preparation and Review


Manual E-Book
Psychiatry Test Preparation & Review Manual

40. The drainage of CSF into the blood is a function of the:

A. 
Choroid plexus
B. 
Virchow–Robin spaces
C. 
Dural mitochondria
D. 
Ventricular ependymal cells
E. 
Arachnoid granulations

41. Which one of the following is a method of making a prediction to compare the value of one variable to another?

A. 
Probability
B. 
Point prevalence
C. 
Incidence
D. 
Regression analysis
E. 
Kappa

42. A 72-year-old woman with a history of smoking, diabetes, hypertension, hyperlipidemia, and myocardial infarction
presents to your emergency room by ambulance with an acute onset of obtundation with dense right hemiplegia; right
hemisensory loss to light touch, pain, and temperature; and mutism. You suspect a left lobar hemorrhage because of the
acuteness of onset of her symptoms and a blood pressure reading of 210/100 mm Hg in the emergency room. Once sta-
bilized, the best immediate diagnostic test of choice from the emergency room would be:

A. 
Lumbar puncture with opening pressure and CSF assay for xanthochromia
B. 
Brain MRI scan without gadolinium
C. 
Blood work for coagulation panel (PT, PTT [partial thromboplastin time], INR [international normalized ratio])
D. 
Noncontrast head CT scan
E. 
Routine bedside electroencephalogram (EEG)

43. Which one of the following is most closely associated with prognostic outcome in psychodynamic therapy?

A. 
Length of training
B. 
Neutrality of the therapist
C. 
Age of the therapist
D. 
Gender of the therapist
E. 
Empathy and warmth

44. Melatonin is a neuronal hormone that promotes sleep and is produced in the brain by the:

A. 
Pineal gland
B. 
Anterior pituitary gland
C. 
Posterior pituitary gland
D. 
Hypothalamus
E. 
Thalamus

70

Psychiatry Test Preparation and Review


Manual E-Book
Test Number Two

45. Which one of the following is an objective psychological test?

A. 
Rorschach
B. 
Sentence completion test
C. 
Thematic apperception test
D. 
MMPI
E. 
Draw a person test

46. Subacute sclerosing panencephalitis is a rare late complication of which one of the following organisms?

A. 
Measles virus
B. 
Herpes simplex virus
C. 
Epstein–Barr virus
D. 
Mumps virus
E. 
JC virus

47. Which one of the following does not describe a patient with attention deficit hyperactivity disorder (ADHD)?
A. 
The patient fails to follow through on instructions and fails to finish schoolwork
B. 
The patient often fidgets with hands or feet or squirms in his or her seat
C. 
The patient often has difficulty awaiting his or her turn
D. 
The patient often seems not to listen when spoken to directly
E. 
The patient shows impairment from symptoms at school but not at home

48. A 13-year-old boy is brought to the emergency room from a group home because of acute agitation. On examination
you note choreoathetotic movements, hyperreflexia, acute agitation, self-scratching and mutilating behavior, and marked
cognitive impairment. You peruse the group-home chart and note that this young boy has an enzymatic deficiency in
hypoxanthine–guanine phosphoribosyltransferase. Your keen memory brings you back to your pediatrics rotation in
medical school, and you realize the diagnosis is:

A. 
Tay–Sachs disease
B. 
Metachromatic leukodystrophy
C. 
Krabbe’s disease
D. 
Gaucher’s disease
E. 
Lesch–Nyhan syndrome

49. Which one of the following is not a DSM criterion for schizophrenia?

A. 
Delusions
B. 
Presence of active-phase symptoms for 6 months
C. 
Hallucinations
D. 
Disorganized speech
E. 
Grossly disorganized or catatonic behavior

50. Botulinum toxin type A is not the treatment of choice for which one of the following disorders?

A. 
Hemifacial spasm
B. 
Blepharospasm
C. 
Cervical dystonia
D. 
Restless legs syndrome
E. 
Limb spasticity related to multiple sclerosis

71

Psychiatry Test Preparation and Review


Manual E-Book
Psychiatry Test Preparation & Review Manual

51. A patient presents with a delusion about being poisoned that has been present for 5 months. The patient has no halluci-
nations or other psychotic symptoms. There has been no major effect on the patient’s daily functioning. The patient has
no mood symptoms. The most likely diagnosis is:

A. 
Dementia
B. 
Schizophrenia
C. 
Schizoaffective disorder, bipolar type
D. 
Delusional disorder
E. 
Brief psychotic disorder

52. A young woman presents to the emergency room with a complaint of a band-like, bifrontal, squeezing headache that
began 6 hours earlier. She denies nausea, vomiting, or any other associated symptoms. She describes the pain as waxing
and waning in intensity throughout the 6-hour period. Physical examination is unremarkable. She reports suffering from
similar attacks in the past. The most likely diagnosis is:

A. 
Tension-type headache
B. 
Migraine without aura
C. 
Migraine with aura
D. 
Paroxysmal hemicrania
E. 
Basilar migraine

53. Which one of the following is not consistent with a major depressive episode?

A. 
Anhedonia
B. 
Withdrawal from social situations
C. 
Low frustration tolerance
D. 
Weight loss
E. 
Increased libido

54. Sophie has been seeing her primary care physician due to hirsutism, obesity, and the recent diagnosis of type II diabetes.
Her doctor has now diagnosed her with polycystic ovarian syndrome. Based on this new diagnosis, which psychiatric
comorbidity is she most likely to experience?

A. 
Anorexia
B. 
Depression
C. 
Psychosis
D. 
Disturbance of body image
E. 
Bulimia

55. A patient comes to your clinic with complaint of hypersomnia, hyperphagia, psychomotor slowing, and depressed
mood. He states that this happens yearly, usually around October or November. The treatment plan for this man should
include:

A. 
Risperidone
B. 
Naloxone
C. 
Exposure to bright artificial light for 2 to 6 hours per day.
D. 
Flooding
E. 
Alprazolam

72

Psychiatry Test Preparation and Review


Manual E-Book
Test Number Two

56. What is the mechanism of action of carbidopa in the combination agent carbidopa–levodopa that is used for the treat-
ment of Parkinson’s disease?

A. 
Postsynaptic dopamine receptor agonism
B. 
Monoamine oxidase type B inhibition
C. 
DOPA decarboxylase inhibition
D. 
Catechol-O-methyltransferase inhibition
E. 
Acetylcholine receptor antagonism

57. A young woman presents to the emergency room with complaints of palpitations, sweating, shortness of breath, chest
pain, and nausea. She thinks that she is having a heart attack. Electrocardiogram (ECG) reveals normal sinus rhythm
with no ischemic changes. Cardiac enzymes are not elevated. Given her symptoms, an alternative diagnosis would be:

A. 
Manic episode
B. 
Myxedema madness
C. 
Mad Hatter syndrome
D. 
Psychotic disorder not otherwise specified (NOS)
E. 
Panic attack

58. The phenomenon of apoptosis refers to:

A. 
Neuronal migration
B. 
Neuronal maturation
C. 
Neurogenesis
D. 
Neuronal myelination
E. 
Neuronal programmed cell death

59. You are reviewing previous treatment records for a new patient that you are about to meet for the first time. You note
that they carry a diagnosis of paranoid personality disorder. Based on this information, which defense mechanism is this
patient most likely to exhibit?

A. 
Splitting
B. 
Altruism
C. 
Projective identification
D. 
Projection
E. 
Denial

60. A 2-year-old toddler is brought to the emergency room because of seizures, hemiparesis, and apparent blindness. You
immediately notice a marked reddish discoloration of the left side of the forehead and face. The parents tell you that
their child has not met appropriate developmental milestones. Your most likely diagnosis is:

A. 
Tuberous sclerosis
B. 
Sturge–Weber syndrome
C. 
Von Hippel–Lindau disease
D. 
Ataxia–telangiectasia
E. 
Fabry’s disease

73

Psychiatry Test Preparation and Review


Manual E-Book
Psychiatry Test Preparation & Review Manual

61. How is a doctor who agrees to take Medicare paid?

A. 
He or she agrees to take only what Medicare pays for the service
B. 
He or she is allowed to bill the patient for the difference between what Medicare pays and what he or she charges
C. 
He or she is paid by a third party to make up the difference between the fee-for-service rate and the fee allowed by
Medicare
D. 
He or she can sue the government if the full fee is not paid
E. 
He or she is not allowed to charge copays

62. The pain syndrome known as reflex sympathetic dystrophy does not involve which one of the following characteristics?

A. 
Hypersensitivity to painful stimuli
B. 
Myofascial trigger points
C. 
Cyanosis of the extremities
D. 
Sweating and shiny skin
E. 
Warm or hot skin on the extremities

63. The police bring an acutely paranoid patient who was found wandering the streets into the emergency room. In your
initial approach to this patient you should first:

A. 
Sedate the patient with haloperidol and lorazepam
B. 
Assess the dangerousness of the patient to self or others
C. 
Obtain an ECG
D. 
Contact the patient’s family if possible
E. 
Obtain any old charts from medical records

64. Carbamazepine will not lower the levels or the efficacy of which one of the following agents?

A. 
Warfarin
B. 
Clozapine
C. 
Alprazolam
D. 
Propranolol
E. 
Citalopram

65. Which one of the following is false concerning the right to die and surrogate decision making?

A. 
Patients who believe that continuing treatment would lessen their quality of life have the right to demand that treat-
ment be withheld or withdrawn
B. 
Advanced directives or a living will is a way for patients to express their preferences before anything happens that
would cause them to lose capacity
C. 
If a patient leaves no clear instructions, the state will carry out a course of action to protect and preserve
human life
D. 
Surrogate decision makers can be appointed by the patient or the courts
E. 
The standard of substituted judgment means that the surrogate will do whatever is in the patient’s best
interests

74

Psychiatry Test Preparation and Review


Manual E-Book
Test Number Two

66. Patients who smoke tobacco heavily can markedly reduce levels of psychotropic medications they are taking. Which one
of the following medications is not affected by tobacco smoking in this way?

A. 
Clozapine
B. 
Olanzapine
C. 
Haloperidol
D. 
Risperidone
E. 
Amitriptyline

67. A consultation-liaison psychiatrist is called to evaluate a patient who is in denial of a major illness. The most important
obligation of the psychiatrist at the first evaluation is to:

A. 
Confront the denial forcefully
B. 
Tell the staff to “play along” with the patient’s denial
C. 
Obtain neuropsychological testing
D. 
Meet with the patient’s family
E. 
Make sure the patient has been informed about the illness and treatment

68. A 38-year-old delivers twins by uncomplicated caesarean section at 37 weeks. The pregnancy, her first, was unremark-
able. On day 3 after her delivery, she experiences an acute onset of what she describes as the worst headache of her life.
The neurologist is called and discovers that she has a notable bitemporal hemianopsia, neck stiffness, a positive Kernig’s
sign, persistent hypotension, and a right third nerve palsy. The most likely diagnosis is:

A. 
Sheehan’s syndrome
B. 
Cushing’s disease
C. 
Subarachnoid hemorrhage
D. 
Acute bacterial meningitis
E. 
Familial hemiplegic migraine

69. A patient describes feeling anxious about being in places or situations from which escape may be difficult or in which
help may not be available should the patient begin to panic. The patient avoids various situations because of these fears.
The term that best describes this patient’s symptoms is:

A. 
Agonothete
B. 
Agoniada
C. 
Agoraphobia
D. 
Agora
E. 
Agouara

70. Normal-pressure hydrocephalus presents as the triad of:

A. 
Dementia, parkinsonism, and visual hallucinations
B. 
Dementia, ophthalmoplegia, and ataxia
C. 
Dementia, incontinence, and gait disturbance
D. 
Chorea, irritability, and obsessive–compulsive traits
E. 
Dementia, axial rigidity, and vertical ophthalmoplegia

75

Psychiatry Test Preparation and Review


Manual E-Book
Psychiatry Test Preparation & Review Manual

71. Which one of the following is not true concerning cyclothymic disorder?

A. 
It is similar to bipolar disorder but less severe
B. 
Symptoms must be present for at least 2 years
C. 
It is equally common in men and women
D. 
Substance abuse is common in patients with cyclothymia
E. 
There are often psychotic symptoms found in patients with cyclothymia

72. Which one of the following is not a potential risk factor for ischemic stroke?

A. 
Prior cardiac disease
B. 
Depression
C. 
Gender
D. 
Family history
E. 
Obesity

73. A patient enters your office. She is agitated, acts seductively, wears colorful clothes that are bizarre in appearance, has an
excessive amount of makeup on, and vacillates between being entertaining, hyperexcited, and threatening. Based on this
information, her most likely diagnosis is:

A. 
Major depressive disorder
B. 
Brief psychotic disorder
C. 
Body dysmorphic disorder
D. 
Bipolar disorder
E. 
Delusional disorder

74. Which one of the following symptoms is not suggestive of a carotid territory transient ischemic attack or stroke?

A. 
Ataxia with vertigo
B. 
Aphasia
C. 
Ipsilateral monocular blindness
D. 
Contralateral body weakness
E. 
Contralateral homonymous visual field defects

75. Which of the following is not commonly part of the thought process of the manic bipolar patient?

A. 
Flight of ideas
B. 
Clang associations
C. 
Racing thoughts
D. 
Tangentiality
E. 
Suicidal ideation

76. Which one of the following agents is not potentially useful for the treatment of essential tremor?

A. 
Lorazepam
B. 
Primidone
C. 
Propranolol
D. 
Desipramine
E. 
Botulinum toxin type A

76

Psychiatry Test Preparation and Review


Manual E-Book
Test Number Two

77. To meet criteria for a major depressive disorder, a patient must have symptoms for:

A. 
1 week
B. 
2 weeks
C. 
1 month
D. 
2 months
E. 
6 weeks

78. Which one of the following tumors is associated with myasthenia gravis?

A. 
Thyroid carcinoma
B. 
Thymoma
C. 
Glioblastoma multiforme
D. 
Breast papilloma
E. 
Non-Hodgkin’s lymphoma

79. Which one of the following is not true regarding schizophrenia?

A. 
Lifetime prevalence is about 1%
B. 
Prevalence is greater in rural than in urban areas
C. 
The male-to-female ratio is 1:1
D. 
Onset is rare before age 10 years or after age 40 years
E. 
There is a higher incidence of the disease in babies born in winter and early spring

80. Which one of the following genetically inherited neurological disorders is not acquired by autosomal dominant heredity?

A. 
Friedrich’s ataxia
B. 
Myotonic dystrophy
C. 
Tuberous sclerosis
D. 
Huntington’s disease
E. 
Neurofibromatosis

81. Which one of the following is not a characteristic of a major depressive episode?

A. 
Constipation
B. 
Dry mouth
C. 
Headache
D. 
Disinhibited behavior
E. 
Early morning awakening

82. Migraine is most likely a hereditary disorder that maps to which chromosome?

A. 
Chromosome 14
B. 
Chromosome 15
C. 
Chromosome 17
D. 
Chromosome 18
E. 
Chromosome 19

77

Psychiatry Test Preparation and Review


Manual E-Book
Psychiatry Test Preparation & Review Manual

83. Which one of the following would be listed under thought content in the mental status examination?

A. 
Obsessions
B. 
Word salad
C. 
Flight of ideas
D. 
Circumstantiality
E. 
Tangentiality

84. Central nervous system (CNS) cysticercosis is caused by brain parenchymal invasion by which one of the following
organisms?

A. 
Leishmania major
B. 
Taenia solium
C. 
Toxoplasma gondii
D. 
Echinococcus granulosus
E. 
Trichinella spiralis

85. Which of the following laboratory tests can be used to detect chronic alcohol abuse?

A. 
Red blood cell count
B. 
White blood cell (WBC) count
γ-Glutamyl transpeptidase (GGT)
C. 
D. 
Creatine phosphokinase
E. 
Alkaline phosphatase

86. The most frequent neurological complication of chronic alcohol abuse is:

A. 
Wernicke’s encephalopathy
B. 
Alcoholic cerebellar degeneration
C. 
Alcoholic neuropathy
D. 
Marchiafava–Bignami disease
E. 
Alcoholic dementia

87. A study in which a group comes from a well-defined population and is followed over a long period of time is a:

A. 
Case history study
B. 
Cohort study
C. 
Cross-sectional study
D. 
Case–control study
E. 
Retrospective study

88. The Miller–Fisher syndrome involves the classic symptom complex of:

A. 
Dementia, parkinsonism, psychosis
B. 
Gait ataxia, urinary incontinence, dementia
C. 
Ataxia, areflexia, ophthalmoplegia
D. 
Alexia without agraphia
E. 
Right–left confusion, finger agnosia, acalculia

78

Psychiatry Test Preparation and Review


Manual E-Book
Test Number Two

89. Konrad Lorenz, during his work with animals, demonstrated which one of the following concepts, which may be used to
understand early human psychological development?

A. 
Sensory deprivation
B. 
Altruism
C. 
Imprinting
D. 
Stress syndromes
E. 
Episodic dyscontrol

90. A 29-year-old woman presents to the emergency room by ambulance in a wheelchair. She was brought from home with
a complaint of rapidly progressive bilateral leg weakness over the past 2 weeks. Her legs were also painful, and she com-
plained of numbness and tingling in the lower part of both legs. Just before the onset of symptoms, she had a 3-day bout
of bad diarrhea with fever and chills that resolved spontaneously. Which one of the following would not be a likely find-
ing on diagnostic testing and examination of this patient?

A. 
Diminished deep tendon reflexes
B. 
High cell count with absent protein in CSF
C. 
Conduction block and prolonged F-wave latencies on nerve conduction studies
D. 
Positive Campylobacter jejuni antibody serology
E. 
Complement-fixing antibodies to peripheral nerve myelin on nerve biopsy

91. Which one of the following is not a second messenger?

A. 
Adenylyl cyclase
B. 
cGMP
Ca2+
C. 
D. 
cAMP
E. 
Inositol triphosphate (IP3)

92. Which one of the following is not more typical of a cortical dementia than of a subcortical dementia, such as dementia
of the Alzheimer type?

A. 
Apathy and depression
B. 
Aphasia
C. 
Dyspraxia
D. 
Absence of motor abnormalities
E. 
Insidious progression of cognitive decline

93. A child is playing in his home, and at the same time that his dog barks, the doorbell also rings. The child believes that
the doorbell rang because the dog barked. This child would fit best into which of Piaget’s stages?

A. 
Sensorimotor
B. 
Preoperational thought
C. 
Concrete operations
D. 
Formal operations
E. 
Latency

79

Psychiatry Test Preparation and Review


Manual E-Book
Psychiatry Test Preparation & Review Manual

94. The sensory dermatomes of the nipples and the umbilicus are respectively located at the levels of:

A. 
T2 and T8
B. 
T3 and T9
C. 
T4 and T10
D. 
T5 and T11
E. 
T6 and T12

95. Which one of the following receptor subtypes is associated with the neurotransmitter glutamate?

A. 
Nicotinic
B. 
Muscarinic
α1
C. 
α-Amino-3-hydroxy-5-methyl-4-isoxazolepropionic acid (AMPA)
D. 
E. 
GABA

96. Buspirone’s mechanism of action is predominantly linked to which one of the following receptors?

A. 
Serotonin 2A
B. 
Serotonin 1A
C. 
N-methyl-d-aspartate (NMDA)
D. 
Dopamine 2
E. 
NE

97. Which one of the following statements is true regarding excitatory neurotransmitters?

A. 
They open anion channels that depolarize the cell membrane and increase the likelihood of generating an action
potential
B. 
They open cation channels that hyperpolarize the cell membrane and increase the likelihood of generating an action
potential
C. 
They open cation channels that hyperpolarize the cell membrane and decrease the likelihood of generating an action
potential
D. 
They open anion channels that hyperpolarize the cell membrane and decrease the likelihood of generating an action
potential
E. 
They open cation channels that depolarize the cell membrane and increase the likelihood of generating an action
potential

98. Phencyclidine (PCP) exerts its hallucinogenic effects primarily by mediation of which of the following receptors?

A. 
Serotonin 2A
B. 
Serotonin 1A
C. 
NMDA
D. 
Dopamine 2
E. 
NE

80

Psychiatry Test Preparation and Review


Manual E-Book
Test Number Two

99. Which one of the following is not an immature defense?

A. 
Hypochondriasis
B. 
Introjection
C. 
Sublimation
D. 
Regression
E. 
Passive aggression

100. Which one of the following agents is least likely to exacerbate the extrapyramidal symptoms of Parkinson’s disease?

A. 
Amoxapine
B. 
Perphenazine
C. 
Thorazine
D. 
Fluphenazine
E. 
Phenelzine

101. Who was the author of The Ego and Mechanisms of Defense and gave us the first comprehensive study of defense
mechanisms?

A. 
Sigmund Freud
B. 
Kohut
C. 
Erich Fromm
D. 
Anna Freud
E. 
Carl Jung

102. Which one of the following is not necessary for a patient to be declared competent to stand trial?

A. 
Understanding of the nature of the charges against him or her
B. 
Not having a mental illness
C. 
Having the ability to consult a lawyer
D. 
Helping the lawyer in his defense
E. 
Understanding of court procedure

103. Which one of the following is a required DSM 5 criterion for schizoaffective disorder?

A. 
Catatonic features
B. 
A mood episode separate from schizophrenic symptoms
C. 
Delusions or hallucinations for 1 month in the absence of a major mood episode
D. 
Symptoms of a major mood episode are present for the majority of the illness
E. 
Mood symptoms may be substance induced

104. How long should a patient remain on antidepressant medication after having experienced four major depressive epi-
sodes in the past 5 years?

A. 
3 months
B. 
6 months
C. 
12 months
D. 
2 years
E. 
Indefinitely

81

Psychiatry Test Preparation and Review


Manual E-Book
Psychiatry Test Preparation & Review Manual

105. Which one of the following symptoms is not part of dysthymic disorder?

A. 
Poor appetite
B. 
Low self-esteem
C. 
Feelings of hopelessness
D. 
Hallucinations
E. 
Fatigue

106. Which ruling determined that the physician–patient relationship imposes an obligation on the psychiatrist for care and
safety of the patient and others?

A. 
Wyatt v Stickney
B. 
Durham v the United States
C. 
O’Connor v Donaldson
D. 
Tarasoff v Regents of the University of California
E. 
Clites v State

107. Which one of the following defense mechanisms is considered healthy and adaptive throughout the life cycle?

A. 
Sublimation
B. 
Acting out
C. 
Projection
D. 
Somatization
E. 
Introjection

108. Susan presented to her doctor with fatigue, weight loss, arthritis, rash, photosensitivity, anemia, cognitive dysfunction,
and depression. Her primary care doctor treated her with corticosteroids, and after 2 weeks her symptoms resolved,
with the exception of cognitive dysfunction. This history is most consistent with:

A. 
Fibromyalgia
B. 
Systemic lupus erythematosus
C. 
Chronic fatigue syndrome
D. 
Irritable bowel syndrome
E. 
Sheehan’s syndrome

109. The obligation to act in the patient’s best interests is known as:

A. 
Beneficence
B. 
Fiduciary duty
C. 
Nonmalfeasance
D. 
Altruism
E. 
Parens patriae

82

Psychiatry Test Preparation and Review


Manual E-Book
Test Number Two

110. A patient presents with a nondominant hemispheric stroke in the middle cerebral artery territory. Which one of the fol-
lowing symptoms and signs would you not find on neurologic examination?

A. 
Hemi-inattention
B. 
Anosognosia
C. 
Right–left disorientation
D. 
Impaired prosody of speech
E. 
Visual and tactile extinction

111. Upon graduation from residency, you start a new practice in your home town. You go to a local internist and a local
psychologist and tell them that for every patient they refer you, you will pay them $100. This arrangement is:

A. 
Ethical because people need doctors and you need patients
B. 
Unethical because it puts the doctor’s interests ahead of the patient’s
C. 
Ethical because all parties involved benefit in some way
D. 
Ethical because the referring clinician is fairly compensated for the referral
E. 
Unethical because the primary care physician should know how to treat depression and should not need to refer to
you

112. A 50-year-old man presents to the emergency room with a complaint of acute onset of right eye pain, ptosis, and diplo-
pia. The symptoms began that morning immediately upon his awakening from sleep. Your examination reveals a nor-
mal-sized pupil that is fully reactive to light and a right eye that cannot adduct nasally with a ptotic right eyelid. The
most likely cause of this condition is:

A. 
Diabetes
B. 
Stroke
C. 
Posterior communicating artery aneurysm
D. 
Multiple sclerosis
E. 
Myasthenia gravis

113. Which one of the following is not a major problem area addressed by interpersonal psychotherapy?

A. 
Cognitive distortions
B. 
Grief and loss
C. 
Role transitions
D. 
Relational role disputes
E. 
Social deficits

114. Which one of the following agents would not be useful for the treatment of tics in Tourette’s syndrome?

A. 
Fluphenazine
B. 
Molindone
C. 
Botulinum toxin type A
D. 
Haloperidol
E. 
Protriptyline

83

Psychiatry Test Preparation and Review


Manual E-Book
Psychiatry Test Preparation & Review Manual

115. Difficulty discarding possessions, which results in congested and cluttered living spaces, is most characteristic of:

A. 
Obsessive–compulsive disorder
B. 
Obsessive–compulsive personality disorder
C. 
Shubo-kyofu
D. 
Amphetamine use
E. 
Hoarding disorder

116. A right middle cerebral artery territory ischemic stroke posterior to the optic chiasm would be expected to cause:

A. 
Bitemporal hemianopsia
B. 
Left monocular blindness
C. 
Left homonymous hemianopsia
D. 
Right homonymous hemianopsia
E. 
Right upper quadrant anopsia

117. Which one of the following statements is not true regarding chemical signaling between neurons?

Neurotransmitter synthesis may be stimulated by influx of Ca2+


A. 
NE-releasing neurons have presynaptic α receptors, which are involved in a negative feedback system to stop NE
B. 
release
C. 
Once dopamine is released into the synaptic cleft, it works until it diffuses away or is removed by reuptake
mechanisms
D. 
Exocytosis is the process by which neurotransmitter storage vesicles release their contents into the synaptic cleft
E. 
Monoamine oxidase type B (MAO-B) metabolizes NE and serotonin

118. A 4-year-old boy presents to the emergency room in an acute state of agitation. Careful history-taking and examination
reveal hypotonia, delayed developmental milestones, athetotic movements of the upper extremities, and mental retarda-
tion. The parents explain that their son constantly bites his hands and lips to the point of bleeding. Lesch–Nyhan syn-
drome is the clinical diagnosis. This syndrome is caused by a deficiency of which of the following enzymes?

A. 
Ornithine transcarbamylase
B. 
Hypoxanthine–guanine phosphoribosyltransferase
C. 
Adenylosuccinate deficiency
D. 
Arginase
E. 
Carbamoyl-phosphate synthase

119. A child is brought to your office for depression. During the course of your interview you see that the patient can think
abstractly, reason deductively, and define abstract concepts. This child would fit into which of Piaget’s developmental
stages?

A. 
Sensorimotor
B. 
Preoperational thought
C. 
Concrete operations
D. 
Formal operations
E. 
Symbiosis

84

Psychiatry Test Preparation and Review


Manual E-Book
Test Number Two

120. Predisposition for dementia pugilistica is increased in carriers with defects on which one of the following chromosomes?

A. 
Chromosome 16
B. 
Chromosome 17
C. 
Chromosome 18
D. 
Chromosome 19
E. 
Chromosome 20

121. Which one of the tricyclic antidepressants has the most antihistaminic activity?

A. 
Amoxapine
B. 
Clomipramine
C. 
Desipramine
D. 
Nortriptyline
E. 
Doxepin

122. Which one of the following is not helpful in the treatment of obsessive–compulsive disorder?

A. 
Bupropion
B. 
Fluvoxamine
C. 
Clomipramine
D. 
Sertraline
E. 
Fluoxetine

123. Which one of the following is a systemic approach in which psychiatrists and primary care physicians work together to
deliver treatment for common mental disorders in a primary care setting?

A. 
Population-based care
B. 
Illness management
C. 
Tertiary prevention
D. 
Universal interventions
E. 
Collaborative care

124. Which one of the following is not effective in treating enuresis in childhood?

A. 
Desmopressin
B. 
Bell-and-pad conditioning
C. 
Amitriptyline
D. 
Imipramine
E. 
Olanzapine

125. Which one of the following ions uses the second ion channel to open during an action potential, acts as a second mes-
senger once in the neuron, activates the release of neurotransmitter, and activates ion channels that allow for influx of
other ions that halt the action potential?

Na+
A. 
K+
B. 
Cl−
C. 
Ca2+
D. 
E. 
IP3

85

Psychiatry Test Preparation and Review


Manual E-Book
Psychiatry Test Preparation & Review Manual

126. Patients with which one of the following disorders may have clinically significant side effects from tricyclic antidepres-
sant drugs?

A. 
Insomnia
B. 
Benign prostatic hypertrophy
C. 
Migraine
D. 
Parkinson’s disease
E. 
Pituitary adenoma

127. Which one of the following statements is true regarding inhibitory neurotransmitters?

A. 
They open chloride channels that depolarize the cell membrane and increase the likelihood of an action potential
B. 
They open cation channels that depolarize the cell membrane and increase the likelihood of an action potential
C. 
They open chloride channels that hyperpolarize the cell membrane and increase the likelihood of an action potential
D. 
They open chloride channels that hyperpolarize the cell membrane and decrease the likelihood of an action potential
E. 
They open potassium channels that depolarize the cell membrane and decrease the likelihood of an action potential

128. A patient with a fear of spiders is put in a room with many spiders, and immediately a live tarantula is placed on his
hand for as long as necessary until the dissipation of his anxiety. This behavioral technique is called:

A. 
Graded exposure
B. 
Aversion therapy
C. 
Flooding
D. 
Assertiveness training
E. 
Modeling

129. Which one of the following is true regarding psychoanalytic psychotherapy?

A. 
All of the patient’s remarks should be taken at face value
B. 
Most of what the patient says is unimportant
C. 
Disclaimers often precede emotionally charged material and are important to note
D. 
It is important to point out to the patient every instance in which he or she exhibits low self-esteem
E. 
One should interpret the patient’s resistance at each and every opportunity

130. An 82-year-old patient in a skilled nursing facility displays confusion, restlessness, agitation, and disorganized speech
only during the evening hours. Which one of the following is not an appropriate treatment approach?

A. 
Increased lighting in the room
B. 
Low-dose haloperidol at bedtime
C. 
Having a calendar on the wall
D. 
Flurazepam at bedtime for sleep
E. 
Companionship and family support during the day

131. Which one of the following is not a technique used in cognitive therapy?

A. 
Reattribution
B. 
Role playing
C. 
Thought recording
D. 
Abreaction
E. 
Developing alternatives

86

Psychiatry Test Preparation and Review


Manual E-Book
Test Number Two

132. Heroin withdrawal does not involve which one of the following symptoms?

A. 
Pinpoint pupils
B. 
Abdominal pain
C. 
Piloerection
D. 
Muscle twitching
E. 
Dysphoria

133. Which one of the following is among the least sedating of the tricyclic antidepressants?

A. 
Desipramine
B. 
Amitriptyline
C. 
Trimipramine
D. 
Doxepin
E. 
Imipramine

134. Which one of the following is not an example of secondary gain?

A. 
Getting money
B. 
Getting medical help
C. 
Getting out of having to work
D. 
Getting out of family responsibilities
E. 
Getting drugs of abuse

135. In hypothyroidism one would expect to find:

A. 
Serum free T4 is increased
B. 
Serum total T4 concentration is increased
C. 
Serum thyroid-stimulating hormone (TSH) is increased
D. 
Serum T3 uptake is increased
E. 
Serum T3–T4 ratio is decreased

136. Which one of the following conditions involves increased risk in electroconvulsive shock therapy (ECT)?

A. 
Pregnancy
B. 
Hypopituitarism
C. 
Uncontrolled epilepsy
D. 
Neuroleptic malignant syndrome
E. 
Cerebral aneurysm

137. Caution should be taken when prescribing which one of the following to a woman on oral contraception?

A. 
Risperidone
B. 
Valproic acid
C. 
Gabapentin
D. 
Lithium
E. 
Carbamazepine

87

Psychiatry Test Preparation and Review


Manual E-Book
Psychiatry Test Preparation & Review Manual

138. An 8-year-old child has a mental age of 6 years. For school placement purposes, the intelligence quotient (IQ) should be
reported as:

A. 
50
B. 
75
C. 
100
D. 
120
E. 
135

139. Psychiatrist: What’s on your mind? Patient: I’ve been feeling depressed. Psychiatrist: Can you tell me more about what’s
been happening? Patient: I haven’t been eating as much as I used to. Psychiatrist: Could you explain to me what you’ve
been going through? The psychiatrist’s approach is an example of:

A. 
Closed-ended questions
B. 
Open-ended questions
C. 
Countertransference
D. 
Detailed mini-mental status examination
E. 
Negative reinforcement

140. Violent or aggressive behavior is associated with:

A. 
Decreased levels of 5-HIAA in spinal fluid
B. 
Decreased growth hormone response to insulin-induced hypoglycemia
C. 
Abnormal dexamethasone suppression test
D. 
Decreased response to corticotrophin-releasing hormone stimulation test
E. 
Decreased response to thyrotropin-releasing hormone suppression test

141. Which one of the following is not a potential repercussion of lithium intoxication?

A. 
Seizure
B. 
Renal toxicity
C. 
Ataxia and coarse tremor
D. 
Nonspecific T-wave changes
E. 
Jaundice

142. Obsessive–compulsive disorder is associated with abnormality of which one of the following neurotransmitters?

A. 
NE
B. 
Serotonin
C. 
Melatonin
D. 
Acetylcholine
E. 
Dopamine

88

Psychiatry Test Preparation and Review


Manual E-Book
Test Number Two

143. A patient comes into the clinic because his family (whom he sees only one or two times per year) keeps telling him to
“go see a shrink.” He has no close relationships. He participates mainly in solitary activities. He has no desire for sexual
activity with others. He is indifferent to the praise or criticism of others. On examination his affect is flat. His most
likely diagnosis is:

A. 
Schizoid personality disorder
B. 
Schizotypal personality disorder
C. 
Narcissistic personality disorder
D. 
Major depressive disorder
E. 
Dysthymic disorder

144. Which one of the following characteristics pertaining to vascular dementia is false?

A. 
There is a stepwise decline in functioning
B. 
Hypertension is a known risk factor
C. 
There is abrupt onset of symptoms
D. 
There is a good response to cholinergic therapies
E. 
Smoking is a known risk factor

145. A 70-year-old male comes in to you for evaluation. He reports that about 90 minutes into sleep he begins screaming and
moving in response to dreams. He has hit and kicked his wife during these episodes. Upon awakening he is alert, is not
confused, and remembers his dreams. You order a sleep study, which confirms that he is not experiencing atonia during
REM sleep. Based on this information, which neurocognitive disorder is this patient at increased risk of developing?

A. 
Frontotemporal dementia
B. 
Subacute sclerosing panencephalitis
C. 
Lewy body disease
D. 
Alzheimer’s disease
E. 
Vascular dementia

146. A team comprising a psychiatrist, psychologist, social worker, nurse, and medical student discharges a patient because
the insurance will no longer pay for her stay. She commits suicide. Who will be held legally responsible for the team’s
actions?

A. 
The medical student
B. 
The psychologist
C. 
The social worker
D. 
The nurse
E. 
The psychiatrist

147. Which one of the following is not a side effect of treatment with tricyclic antidepressants?

A. 
Termination of ventricular fibrillation
B. 
Increased collateral blood supply to ischemic heart muscle
C. 
Decreased contractility
D. 
Tachycardia
E. 
Hypertension

89

Psychiatry Test Preparation and Review


Manual E-Book
Psychiatry Test Preparation & Review Manual

148. The best indicator for future suicidal behavior is:

A. 
Age
B. 
Psychosis
C. 
Past suicidal behavior
D. 
Substance use
E. 
Personality disorder

149. A young woman comes to a psychiatrist’s office seeking help because of problems on her job. She describes nervousness
talking in front of other coworkers at conferences and difficulty at social events. She thinks that her boss knows her
inner feelings and that there was wording put in everyone’s contracts with her specifically in mind. Her dress is eccentric
and out of date. She complains that she does not have any friends at the office. Given this picture, she most likely has
which one of the following diagnoses?

A. 
Borderline personality disorder
B. 
Dependent personality disorder
C. 
Schizotypal personality disorder
D. 
Histrionic personality disorder
E. 
Schizoid personality disorder

150. Which one of the tricyclic antidepressants is used to treat childhood enuresis?

A. 
Desipramine
B. 
Clomipramine
C. 
Maprotiline
D. 
Amoxapine
E. 
Imipramine

90

Psychiatry Test Preparation and Review


Manual E-Book
Test Number Two

Two
Answer Key – Test Number Two
1. E 26. A 51. D 76. D 101. D 126. B
2. D 27. E 52. A 77. B 102. B 127. D
3. B 28. B 53. E 78. B 103. D 128. C
4. E 29. D 54. B 79. B 104. E 129. C
5. D 30. C 55. C 80. A 105. D 130. D
6. B 31. B 56. C 81. D 106. D 131. D
7. C 32. D 57. E 82. E 107. A 132. A
8. E 33. A 58. E 83. A 108. B 133. A
9. C 34. E 59. D 84. B 109. B 134. B
10. A 35. D 60. B 85. C 110. C 135. C
11. B 36. A 61. A 86. C 111. B 136. E
12. C 37. B 62. E 87. B 112. A 137. E
13. D 38. D 63. B 88. C 113. A 138. B
14. B 39. A 64. B 89. C 114. E 139. B
15. C 40. E 65. E 90. B 115. E 140. A
16. D 41. D 66. D 91. A 116. C 141. E
17. C 42. D 67. E 92. A 117. E 142. B
18. D 43. E 68. A 93. B 118. B 143. A
19. D 44. A 69. C 94. C 119. D 144. D
20. A 45. D 70. C 95. D 120. D 145. C
21. D 46. A 71. E 96. B 121. E 146. E
22. B 47. E 72. B 97. E 122. A 147. E
23. B 48. E 73. D 98. C 123. E 148. C
24. C 49. B 74. A 99. C 124. E 149. C
25. A 50. D 75. E 100. E 125. D 150. E

91

Psychiatry Test Preparation and Review


Manual E-Book
Psychiatry Test Preparation & Review Manual

Two
Explanations – Test Number Two
Question 1. E. There are several life-cycle theorists who are important to know. Freud’s theory of development involv-
ing oral, anal, and phallic stages is important. Carl Jung felt that development of the personality occurs
through experiences that teach a person who they intrinsically are. For him, libido included sexual
energy but also spiritual urges and a drive to understand the meaning of life. Harry Stack Sullivan
focused on social interaction. In his theory he defined each stage of life through the need to interact with
certain individuals. These interactions shaped the development of the personality. Erik Erikson devel-
oped a life cycle from childhood to adulthood. It consists of:
• S tage 1: Trust vs mistrust. Trust is shown by ease of feeding and depth of sleep and depends on consistency of
experience provided by the caretaker. If trust is strong, the child develops self-confidence. (Birth to 1 year of age)
• S tage 2: Autonomy vs shame and doubt. This stage includes the child learning to walk and feed him-
or herself. There is a need for outer control. Shame happens through excessive punishment, and self-
doubt occurs if the child is made to feel ashamed of his or her actions. (Ages 1 to 3 years)
• S tage 3: Initiative vs guilt. In this stage the child initiates both motor and intellectual activities. If rein-
forced, his or her intellectual curiosity is satisfied. If made to feel inadequate, he or she will develop
guilt about self-initiated activities. (Ages 3 to 5 years)
• S tage 4: Industry vs inferiority. In this stage the child is busy building, creating, and accomplishing. If
inferior to his or her peers with the use of tools and skills, he or she will have less status and develop
a sense of inadequacy and inferiority. (Ages 6 to 11 years)
• S tage 5: Ego identity vs role confusion. There is a struggle to develop a sense of inner sameness and
continuity. The adolescent shows preoccupation with appearance, hero worship, and ideology. There
are dangers of role confusion and doubts about his or her sexual orientation and vocational identity.
(Ages 11 years to end of adolescence)
• S tage 6: Intimacy vs isolation. Intimacy is marked by formation of lifelong attachments and self-abandon-
ment. Separation occurs if the individual is isolated and views others as dangerous. (Ages 21 to 40 years)
• S tage 7: Generativity vs stagnation. Generativity is marked by raising children, altruism, creativity,
and guiding the next generation. Stagnation occurs if there is isolation, excessive self-concern, and an
absence of intimacy. (Ages 40 to 65 years)
• S tage 8: Ego identity vs despair. Integrity is a feeling of satisfaction that life has been worthwhile,
along with an acceptance of one’s place in life. Despair is the feeling of loss of hope, disgust, and fear
of death. (Ages 65+ years)
Piaget developed a theory of cognitive development. It consists of four stages: sensorimotor, preopera-
tional thought, concrete operations, and formal operations. The work of Freud, Jung, and Erikson was
the result of observations of children, not of carefully crafted studies. Their theories are a framework
to understand development but are not intended to describe objective reality. Their work has been fol-
lowed by more scientific longitudinal studies of development and more neurobiological understandings
of human behavior.
Human Development
K&S Chapter 2

92

Psychiatry Test Preparation and Review


Manual E-Book
Test Number Two

Question 2. D. The clinical picture given is that of a conduction aphasia. Conduction aphasia results from left-hemi-
spheric (dominant) lesions, particularly in the inferior parietal or superior temporal regions. The area
of predilection for this lesion is considered to be the arcuate fasciculus that connects Wernicke’s and
Broca’s areas. Conduction aphasia is characterized by the inability to repeat relatively normal, spon-
taneous speech and the possibility of paraphasic errors and hesitancy. Naming may be impaired, but
auditory comprehension is intact. Writing may or may not be impaired. Associated symptoms of a con-
duction aphasia may include right hemiparesis, right hemisensory loss, right hemianopsia, and limb
apraxia. Broca’s aphasia would involve a lesion of Broca’s area in the left posterior inferior frontal gyrus.
This results in broken, stuttering, staccato speech, with inability to repeat and phonemic and paraphasic
errors. Reading is often impaired, although auditory comprehension is usually intact. Broca’s aphasia is
frequently associated with depression and right hemiparesis and hemisensory loss.

Wernicke’s aphasia results from a lesion in the superior temporal gyrus in Wernicke’s area. Speech is
generally fluent, but comprehension is impaired. Speech may also be logorrheic or overproductive. The
speech displays paragrammatism, which involves neologisms, verbal paraphasic errors, and production
of jargon. Repetition, naming, and auditory comprehension are impaired. Reading comprehension is
impaired. Wernicke’s aphasia is often accompanied by a right homonymous hemianopsia, with a marked
absence of motor and sensory signs and symptoms.

Global aphasia results from large lesions involving the superior temporal, inferior frontal, and parietal
lobes. The region generally corresponds to the territory of the middle cerebral artery (MCA), and indeed
a thrombus at the trifurcation of the MCA can produce a global aphasia. Speech is nonfluent or mute,
and comprehension is impaired. Naming, reading, repetition, and writing are all poor. Most patients
present with a dense right hemiparesis, hemisensory loss, and hemianopsia.

Transcortical aphasias involve the areas around and adjacent to Broca’s and Wernicke’s areas and essen-
tially present with features similar to those of the two perisylvian aphasias, except that repetition is
spared. Transcortical motor aphasia presents with features similar to those of Broca’s aphasia, with tele-
graphic, stuttering speech; sparing of comprehension; and fluent repetition. The area involved is usu-
ally anterior to Broca’s area in the anterior cerebral artery territory. Transcortical sensory aphasia is the
analogue of Wernicke’s aphasia and presents with fluent paraphasic speech, paraphasic naming, impaired
reading and auditory comprehension, and normal repetition. The lesion usually localizes to the temporo-
occipital area. Transcortical mixed aphasia is a rare condition also known as isolation of the speech
area. The classic presentation is that of a global aphasia with echolalic repetition only and no propo-
sitional speech or comprehension. The patient can only repeat. The lesion localizes to large watershed
areas in the left hemisphere, sparing the perisylvian areas.
Neurology
B&D Chapter 13

Question 3. B. Collaborative care models seek to expand access to psychiatric treatment to the many primary care
patients who would otherwise not be able to gain access to a psychiatrist. Patients are more com-
fortable being seen in a primary care setting than in the mental health clinic. Treatment is improved
when primary care physicians and psychiatrists work together, leading to better diagnosis and more
effective treatment regimens. Studies support improved outcomes for patients treated with collabora-
tive care. Improved communication is an important factor that contributes to better overall outcomes.
Formularies are determined by the patient’s insurance, not by the primary care physician or psychiatrist,
so they are not affected by the collaborative care model. However, having the support of the psychiatrist
often helps the primary care physician select medications from the patient’s formulary that are appropri-
ate for their condition and covered by insurance as well.
Management in Psychiatry
MGH Comprehensive Clinical Psychiatry, Chapter 59

Question 4. E. The classic complication of rapid sodium replacement in hyponatremia is central pontine myelinolysis.
This can result in a clinical transection of the pons and a locked-in syndrome. Locked-in syndrome is the
result of a pontine lesion that is clinically devastating and produces quadriplegia, mutism, and a lower
cranial nerve palsy that leaves a patient with only the ability to move the eyes up and down and blink

93

Psychiatry Test Preparation and Review


Manual E-Book
Psychiatry Test Preparation & Review Manual

the eyelids. Only the upper brainstem function is preserved, and patients usually need to be on a respira-
tor. Cognition and comprehension are usually grossly intact, and the patient is often quite aware of the
predicament. Prognosis is very poor. The locked-in syndrome is most often the result of a ventral pontine
infarct as a consequence of basilar artery thrombosis but can in certain instances result from central
pontine myelinolysis, which produces a more central pontine lesion. Other possible causes of locked-in
syndrome are acute inflammatory demyelinating polyneuropathy (or Guillain–Barré syndrome), myas-
thenia gravis, and neuromuscular blocking agents.
Neurology
B&D Chapter 5

Question 5. D. This question contains an example of rationalization. Rationalization is characterized by offering ratio-
nal explanations in an attempt to justify attitudes, beliefs, or behaviors that may otherwise be unac-
ceptable. Projection is perceiving and reacting to unacceptable inner impulses as if they are outside the
self. Blocking is temporarily halting thinking. It often occurs in psychosis because of hallucinations and
thought disorganization. Externalization is perceiving elements of your own personality in the external
world or in external objects. It is a more general term than projection. Denial is avoiding awareness
of some painful aspect of reality by negating sensory data. The most primitive of the defenses are the
narcissistic defenses (denial, distortion, projection). The least primitive defenses are the mature defenses
(altruism, anticipation, asceticism, humor, sublimation, suppression).
Psychological Theory and Psychometric Testing
K&S Chapter 4

Question 6. B. The clinical condition depicted in this question is that of pseudotumor cerebri, also called benign intra-
cranial hypertension. Classically a problem seen in young, obese, African American women, the exact
etiology is unknown. Risk factors include hypervitaminosis A, high-dose corticosteroid therapy, tetra-
cycline therapy, oral contraceptive use, and head trauma. Patients typically present with a waxing and
waning headache and intermittent visual obscurations. Neurologic examination can reveal papilledema
on funduscopic examination and enlargement of the blind spot on visual field testing. Brain imaging
is usually normal, although some scans reveal slit-like ventricles. The diagnosis is established by lum-
bar puncture with measurement of the opening pressure, which is elevated over 20 cm H2O. The treat-
ment modality of choice is pharmacologic, acetazolamide or prednisone. In certain cases, patients opt
for repeated lumbar punctures that siphon off fluid to maintain normal CSF pressure. Surgical options
for treatment include ventriculoperitoneal shunting, or lumboperitoneal shunting if the ventricles are too
small, and optic nerve sheath fenestration, which can siphon off CSF. ESR would be used as a diagnostic
test for temporal arteritis. Serum prolactin level is sometimes drawn after a seizure to determine if it is
epileptic or nonepileptic. The prolactin level would be expected to be elevated greater than twice normal
within an hour after a true epileptic seizure.
Neurology
B&D Chapter 46

Question 7. C. The National Alliance for the Mentally Ill (NAMI) is an advocacy group made up of families of the
mentally ill that works at local, state, and federal levels to improve services for the mentally ill. They are
involved with lawmakers, outreach, and education. The other answer choices are distractors. The most
important organization to know is NAMI.
Public Policy
K&S Chapters 7 and 35

Question 8. E. The criteria for migraine without aura are as follows: at least five headache attacks lasting 4 to 72 hours
(untreated or unsuccessfully treated) that have at least two of four characteristics: unilateral location,
pulsating quality, moderate or severe intensity, or are aggravated by walking stairs or similar routine
physical activity. During the headache at least one of the two following symptoms occurs: phonophobia
and photophobia, nausea and/or vomiting.

Migraine with aura involves at least two attacks with at least three of the following: one or more fully
reversible aura symptoms indicating focal cerebral cortical and/or brainstem functions; at least one aura

94

Psychiatry Test Preparation and Review


Manual E-Book
Test Number Two

symptom develops gradually over more than 4 minutes, or two or more symptoms occur in succession;
no aura symptom lasts more than 60 minutes; or headache follows aura with free interval of at least
60 minutes (it may also simultaneously begin with the aura). At least one of the following aura features
establishes a diagnosis of migraine with typical aura: homonymous visual disturbance, unilateral pares-
thesias and/or numbness, unilateral weakness, or aphasia or unclassifiable speech difficulty.
Neurology
B&D Chapter 69

Question 9. C. The lifetime prevalence of schizophrenia is 1%. It is estimated that there are somewhere in the vicinity of
2 million patients with schizophrenia in the United States. The other answer choices are distractors.
Psychotic Disorders
K&S Chapter 7

Question 10. A. r-TPA is used for acute thrombolysis of ischemic cerebrovascular accidents within 3 hours of the occur-
rence of the event for most patients and 4.5 hours for select patients. Contraindications to r-TPA include
major surgery within the past 14 days, blood pressure over 185/110 mm Hg, bleeding parameters falling
outside a narrow range (PT > 15 or INR > 1.7), or a CT showing signs of intracranial hemorrhage.
Neurology
B&D Chapter 51

Question 11. B. Assertive community treatment is based on the model whereby teams of psychiatrists and other mental
health workers go out into the community to patients’ homes to maintain contact with them, monitor
their status, and encourage medication compliance. The goal is to prevent decompensation of severely
mentally ill patients and catch them before they severely decompensate and require hospitalization.
Public Policy
K&S Chapter 7

Question 12. C. The condition described in this question is that of restless legs syndrome (RLS). One of the most com-
mon treatments is ropinirole (Requip), a dopamine agonist, which can improve symptoms at low doses
starting at 0.5 mg before bedtime. Other dopamine agonists, pergolide, pramipexole, and bromocriptine,
can be useful as well. Other helpful agents include benzodiazepines, opiate analgesics, gabapentin, and
levodopa-carbidopa. It is usually idiopathic but has been associated with polyneuropathy, uremia, and
iron deficiency. The classic symptoms of RLS are crawling or creeping sensations (paresthesias or dys-
esthesias) of the lower extremities that are worse when lying down or in bed and occur most often at
the onset of sleep. There is an urge to move the legs during rest or when lying down or sitting. The urge
to move is generally relieved by movement, like walking or stretching. Symptoms are worse at night or
occur only in the evening or at night. Family history can be positive in 40% to 50% of patients, which
suggests an autosomal dominant pattern of inheritance. Answer choice E, Suvorexant, is a sleep medica-
tion sold in the U.S. under the brand name Belsomra. It is not indicated for treatment of restless legs. It
has a unique mechanism of action via antagonism of orexin receptors.
Neurology
B&D Chapter 68

Question 13. D. The best method to diagnose depression is the standard psychiatric interview. Psychiatric interviews
serve two functions: to find and classify symptoms and to find psychological determinants of behavior.
Interviews can be either insight- or symptom-oriented. The other answer choices all have major flaws.
The MMPI is a self-report inventory used to assess personality traits and, as such, is not appropri-
ate to the task. The scholastic achievement test is completely unrelated to symptom identification for
depression. The dexamethasone suppression test is used to demonstrate abnormal activity of the hypo-
thalamic–pituitary–adrenal axis, which can be found in 50% of major depression patients. However,
the test has limited clinical usefulness because of the frequency of false-positive and false-negative test
results. Interviewing the patient’s teacher is a good idea, but in no way can it replace a face-to-face meet-
ing with the child or be the sole basis for diagnosing depression.
Diagnostic and Treatment Procedures in Psychiatry
K&S Chapter 5

95

Psychiatry Test Preparation and Review


Manual E-Book
Psychiatry Test Preparation & Review Manual

Question 14. B. This question describes a left lateral medullary syndrome (also called Wallenberg’s syndrome). The
Wallenberg syndrome is a brainstem stroke syndrome usually caused by occlusion of one of the vertebral
arteries or, less commonly, one of the posterior inferior cerebellar arteries. The area of infarction is the
lateral medulla. The clinical syndrome involves an ipsilateral Horner’s syndrome, ipsilateral loss of pain
and temperature sensation in the face, cerebellar ataxia, and weakness of the vocal cords, pharynx, and
palate. There is contralateral loss of pain and temperature sensation to the body. The visual system is not
affected in this syndrome.
Neurology
B&D Chapter 51

Question 15. C. Decreased levels of serotonin in the CSF have been shown to be linked to higher levels of aggression in
patients. In general, dopamine seems to promote aggression, whereas NE and serotonin seem to inhibit
it, as does GABA. Rapid declines in serotonin levels have been linked to irritability and aggression,
and low CSF serotonin has been linked to increased frequency of suicide. CRH is a hormone that may
increase in major depression, anorexia, and anxiety disorders. It is produced by the hypothalamus. It has
no relation to aggression. Growth hormone is also unrelated to aggression.
Basic Neuroscience
K&S Chapter 1

Question 16. D. Metachromatic leukodystrophy (MLD) is an autosomal recessive metabolic disorder of myelin that
results in a deficiency in arylsulfatase A (ASA). The result is an abnormal accumulation of sulfa-
tides in the brain and peripheral nerves. High sulfatide levels lead to progressive demyelination. The
ASA gene is located on chromosome 22q13. Gait disorder with hypotonia and lower-limb areflexia
are early manifestations of the disease and often precede CNS involvement in infantile and juve-
nile forms of the disease. Adult-onset MLD tends to present with progressive dementia and behav-
ioral problems. Nerve conduction velocities are slowed in both juvenile and adult patients. Delayed
visual and somatosensory-evoked potential latencies are noted more often in adult cases. The clas-
sic neuropathologic findings are segmental nerve demyelination on nerve biopsy and metachromatic
inclusions within Schwann cells and macrophages. Although nerve biopsy is diagnostic of MLD,
MRI of the brain combined with urine assay for increased sulfatide excretion and abnormal ASA
enzyme assay in leukocytes is less invasive and the preferred diagnostic method today. Treatment
with bone marrow transplantation may increase brain ASA levels sufficiently to slow or stop disease
progression.

Tay–Sachs disease is the severe infantile form of the autosomal recessive gangliosidosis, which results in
hexosaminidase A deficiency. Hexosaminidase A codes to chromosome 15q23–q24. The classic infantile
picture is that of developmental retardation, paralysis, dementia, and blindness, with death in the sec-
ond or third year of life. The classic finding is a “cherry-red spot” on funduscopic examination. Babies
with Tay–Sachs disease have a persistent hypersensitivity to loud noise with a marked hyperreactivity of
startle response.

Krabbe’s disease, also called globoid cell leukodystrophy, is an autosomal recessive disease caused by a
deficiency in the lysosomal enzyme galactocerebrosidase β-galactosidase. The gene has been localized to
chromosome 14q31. Multinucleated macrophages in central white matter are accompanied by extensive
central and peripheral demyelination. Infants present with rapid deterioration in motor and intellectual
development, hypertonicity, optic atrophy, opisthotonic posture, and seizures. Stem cell transplantation
can improve CNS manifestations by providing a source of missing enzyme.
Neurology
B&D Chapter 76

Question 17. C. SAD is a non-DSM term used to describe a seasonal pattern specifier added to the diagnoses of bipolar
I and II disorders and major depressive disorder. It is associated with depressive symptoms that occur
at a certain time of year and complete remission of symptoms at other times of the year. One must
show a pattern of two episodes during the same season over the previous 2 years to make the diagnosis.
In addition, the seasonal depressive episodes must substantially outnumber any non-seasonally related

96

Psychiatry Test Preparation and Review


Manual E-Book
Test Number Two

depressive episodes during the patient’s lifetime. The treatment is light therapy. Keep in mind that sea-
sonal affective disorder is often associated with carbohydrate cravings, that light therapy is most effec-
tive in the morning, and that light therapy can precipitate hypomania in bipolar patients. These are very
useful little facts often mentioned on standardized examinations.
Depressive Disorders
K&S Chapter 8

Question 18. D. The parachute response can persist longer than 6 months. The Moro reflex is elicited by startle and is
usually present in the normal infant up to 6 months of age. The child extends the arms symmetrically
when the head is rapidly but gently dropped a few centimeters into the examiner’s hand with the baby
in a supine position. The tonic neck reflex is normally present from birth to about 3 months of age.
Grasp and rooting reflexes usually disappear at or about the 6-month age mark. Any persistence of these
reflexes beyond this period or recurrence later in life would be considered abnormal. Positive grasp and
rooting reflexes in the adult would be considered frontal release signs and would signal extensive frontal
white matter damage or disease.
Neurology
B&D Chapter 27

Question 19. D. Answer choice D describes pseudocyesis, which is listed in the DSM under “Other Specified Somatic
Symptom and Related Disorders.” It involves a false belief that one is pregnant, and it can involve physi-
cal signs associated with pregnancy, such as those described. However, the patient is not pregnant, and
there is no endocrine disorder present to explain the findings. Other patients that fall under the heading
of “Other Specified Somatic Symptom and Related Disorders” include those with other somatic symp-
toms that do not meet the time criteria for other diagnoses. Answer choice A describes somatic symp-
tom disorder with predominant pain, in which a patient experiences pain that cannot be explained by
a medical condition and is thought to be significantly mediated by psychological factors. Answer choice
B describes illness anxiety disorder, in which a patient is convinced that he or she has a serious dis-
ease, based on misinterpretation of bodily symptoms and despite the reassurance of doctors. Choice C
describes conversion disorder (functional neurological symptom disorder), in which the patient develops
neurologic symptoms with no medical explanation that are thought to be mediated by psychological
factors. Choice E describes somatic symptom disorder, in which the patient has somatic complaints that
have no medical explanation and are thought to be associated with psychological factors. To make the
diagnosis of somatic symptom disorder, the patient must exhibit one or more somatic symptoms that are
distressing or result in significant disruption of daily life. He or she has excessive thoughts, feelings, or
behaviors related to the somatic symptoms, which can be manifested as disproportionate thoughts about
the symptoms’ seriousness, persistent anxiety about the symptoms, or excessive time and energy devoted
to health concerns.
Somatic Symptom Disorders
K&S Chapter 13

Question 20. A. Cytomegalovirus is the most common offending infectious agent in AIDS-related retinopathy. It accounts
for 30% of cases of HIV-related retinopathy. Toxoplasmosis is less common, probably involved in about
5% of retinitis cases. Tuberculosis is a rare cause of AIDS-related retinitis. Cryptococcus neoformans is
usually responsible for a fungal meningitis and not a retinitis in AIDS patients with low CD4 counts. JC
virus is the offending agent in progressive multifocal leukoencephalopathy in AIDS patients.
Neurology
B&D Chapter 53

Question 21. D. The most common hallucinations in schizophrenia are auditory. Visual, olfactory, and tactile hallucina-
tions are also possible. Schizophrenia was first discovered by Morel, who called it “démence precoce.”
Later Kraepelin used “dementia praecox” to describe a group of illnesses that started in adolescence and
ended in dementia. Bleuler coined the term “schizophrenia.” The disorder is chronic and has a prodro-
mal, active, and residual phase.
Psychotic Disorders
K&S Chapter 7

97

Psychiatry Test Preparation and Review


Manual E-Book
Psychiatry Test Preparation & Review Manual

Question 22. B. Nighttime awakening is not considered to be part of the clinical picture of narcolepsy. Narcolepsy is a
lifelong sleep disorder that is believed to have a hereditary component. In the United States, the prev-
alence is about 3 to 6 in 10,000. One to two percent of first-degree relatives of narcoleptic patients
manifest the illness compared with 0.02% to 0.18% of the population at large. Narcolepsy is linked to
dysfunction of the hypocretin (orexin) peptide system.

The hallmark of the disease is excessive daytime sleepiness and sleep attacks. Sleep attacks manifest as
the irresistible desire to fall asleep. Attacks can occur at any waking moment and last from a few min-
utes up to 30 minutes. Performance at school, at work, and in social situations usually suffers.

The second major manifestation of narcolepsy is cataplexy. Cataplexy is the sudden loss of muscle tone
in the voluntary muscles, while the respiratory and ocular muscles are spared. Attacks of cataplexy
involve the patient falling to the ground after a rapid and complete loss of muscle tone. Consciousness
is preserved during cataplexic attacks. Anywhere from 60% to 100% of narcoleptic patients suffer from
cataplexy.

Sleep paralysis is the third major symptom of narcolepsy. It can be noted in one-quarter to one-half of
narcoleptic patients. These attacks are characterized by hypnagogic or hypnopompic sudden bilateral or
unilateral limb paralysis. Consciousness is preserved during these attacks, but the patient cannot move
or speak.

Hypnagogic hallucinations are the fourth major symptom of narcolepsy. These can occur at onset of
sleep or during early morning awakening. About 20% to 40% of narcoleptic patients experience these
hallucinations. Other major symptoms of narcolepsy include disturbance of night sleep in about 70% to
80% of patients and automatic behavior that can resemble a fugue-like state, which can occur in 20% to
40% of narcoleptic patients.

Medications that are used for narcolepsy include modafinil, armodafinil, and sodium oxybate.
Modafinil, which can be given in a single dose of 100 to 200 mg daily, modulates the hypocretin (orexin)
neurons in the brainstem and is a nonamphetamine stimulant. Armodafinil (Nuvigil) is the R-enantiomer
of modafinil. Sodium oxybate (Xyrem) is a drug that was initially FDA-approved specifically for cata-
plexy, but now has an approval for narcolepsy as well. In many cases a combination of modafinil and
Xyrem is most effective.
Sleep–Wake Disorders
B&D Chapter 68

Question 23. B. Logorrhea is uncontrollable, excessive talking. Alexithymia is a difficulty in recognizing and describing
one’s emotions. Echolalia is the imitative repetition of the speech of another. Flight of ideas is rapid shift-
ing from one topic to another. Stilted speech is a formal, stiff speech pattern.
Diagnostic and Treatment Procedures in Psychiatry
K&S Chapter 5

Question 24. C. Intramuscular phenytoin is a poor choice for treatment of status epilepticus because of its erratic rate
of absorption. Useful status epilepticus treatments include rectal or intravenous diazepam, intravenous
lorazepam, intravenous phenytoin or fosphenytoin, intravenous valproic acid, oxygen by nasal cannula
and airway protection, and intravenous phenobarbital. Phenobarbital is acceptable treatment but not the
best choice because of the narrow therapeutic window and possibility of overdose leading to respiratory
depression and possible death.
Neurology
B&D Chapter 67

Question 25. A. Of the statements listed, the only correct one is answer choice A. Ziprasidone is an agonist at 5-HT
1A and does inhibit reuptake of serotonin and NE. It is also an antagonist at 5-HT 1D, 2A, and 2C, as
well as dopamine D2 and D3. It has low affinity for histaminic, muscarinic, and α2 receptors. Because
of its action on serotonin and NE, it has been postulated to be of benefit not only in psychosis but also

98

Psychiatry Test Preparation and Review


Manual E-Book
Test Number Two

in anxiety and depression. Risperidone is a similar blocker of D2 compared with haldol and, as such,
carries one of the highest risks for extrapyramidal symptoms (EPS) among the atypicals. Quetiapine is
known for its low incidence of EPS, as it has only a moderate affinity for the D2 receptor. It has a
high affinity for 5-HT types 2 and 6, H1, and α1 and α2. It has low affinity for muscarinic receptors.
Olanzapine has been associated with weight gain in many patients. It is very anticholinergic and carries
with it the corresponding side effects. Clozapine has been shown to be associated with a decreased risk
of suicide in schizophrenic patients. Major side effects of clozapine include seizures and agranulocytosis,
among others. And while we’re on the topic of anticholinergic side effects, the treatment of choice for
urinary retention is Urecholine (bethanechol).
Psychopharmacology
K&S Chapter 29

Question 26. A. Divalproex sodium (valproic acid) has the broadest spectrum of coverage and indication of all the anti-
convulsant medications. It has proven efficacy in primary generalized tonic–clonic seizures, absence
seizures, and myoclonic seizures. Its likely mechanism of action is by blockade of voltage-dependent
sodium channels and GABA enhancement. Phenytoin is not indicated for absence seizures and has no
place in their treatment. It is indicated for treatment of partial and generalized tonic–clonic seizures. Its
mechanism of action is by blockade of voltage-dependent sodium channels. Oxcarbazepine and carba-
mazepine are indicated for partial complex and secondary generalized seizures, but both can worsen
absence seizures. Oxcarbazepine does not produce autoinduction of its own metabolism in the way
that carbamazepine does. Ethosuximide is indicated only in uncomplicated absence seizures and has no
place in the treatment of partial complex or secondary generalized tonic–clonic seizures. Its mechanism
of action is by lowering voltage-dependent calcium conductance in thalamic neurons. Valbenazine is a
medication indicated for treatment of tardive dyskinesia and is sold in the United States under the brand
name Ingrezza. It works by inhibiting the vesicular monoamine transporter type 2 (VMAT2), which
leads to depleted monoamine stores. It is not used to treat seizures.
Neurology
B&D Chapter 67

Question 27. E. Constipation is a common side effect of the tricyclic antidepressants as a result of the anticholinergic
activity of these drugs. Other tricyclic side effects include dry mouth, blurry vision, sweating, orthostatic
hypotension, sedation, lethargy, agitation, tremor, slowed cardiac conduction (as evidenced by prolonged
PR and QRS intervals), and tachycardia.
Psychopharmacology
K&S Chapter 29

Question 28. B. Foot flexion (also called dorsiflexion; the bending of the foot upward) involves the tibialis anterior mus-
cle innervated by nerves that emanate from the L5 motor nerve root. L5 also controls the extension
(bending upward) of the big toe (extensor hallucis longus). A lesion at L5 often causes a foot drop from
dorsiflexor muscle weakness. Foot extension (pushing down on the gas pedal) involves the gastrocne-
mius (calf) and soleus muscles innervated by nerves that emanate from the S1 nerve root predominantly.
Leg extension is a function of the quadriceps muscles (anterior thigh), which are innervated by nerves
emanating from the L3 and L4 nerve roots. Hip flexion (raising of the knee in the air) is a function of
the iliopsoas muscles, which are innervated by the L1, L2, and L3 nerve roots. The Achilles or ankle jerk
motor reflex is a function of the S1 motor nerve root.
Neurology
B&D Chapter 23

Question 29. D. Amitriptyline can be used for gastric ulcer because of its strong histamine blockade. Other drugs that can also
be used for this purpose include doxepin and trimipramine. Of note, answer choice C, vilazodone, is a sero-
tonin modulator sold in the U.S. under the brand name Viibryd, indicated for major depression. It is unre-
lated to gastric ulcer. It works by inhibiting serotonin reuptake and partially agonizing the 5HT 1A receptor.
Psychopharmacology
K&S Chapter 29

99

Psychiatry Test Preparation and Review


Manual E-Book
Psychiatry Test Preparation & Review Manual

Question 30. C. The condition noted in this question is that of the classic Brown–Séquard syndrome due to a hemisec-
tion of the spinal cord. The correct combination of deficits is that of loss of motor control and posterior
column function ipsilateral to and below the level of the lesion coupled with contralateral loss of pain
and temperature sensation one to two dermatomal levels below the level of the lesion. Frequent causes
include disk herniation, penetrating trauma, spinal fracture, and radiation injury. The cervical spinal
cord is most commonly affected.
Neurology
B&D Chapter 24

Question 31. B. This question focuses on Piaget’s stages of cognitive development. In the sensorimotor stage, children
respond to stimuli in the environment, learning during the process. They eventually develop object per-
manence (objects exist independently of the child’s awareness of their existence) and begin to understand
symbols (as in the use of words to express thoughts).

During the preoperational stage, there is a sense that punishment for bad deeds is unavoidable (imma-
nent justice). There is a sense of egocentrism and phenomenalistic causality (the thought that events that
occur together cause one another). Animistic thinking (inanimate objects are given thoughts and feel-
ings) is also seen.

In the concrete operations stage, egocentric thought changes to operational thought, in which another’s
point of view can be taken into consideration. In concrete operations children can put things in order
and group objects according to common characteristics. They develop the understanding of conservation
(a tall cup and a wide cup can both hold equal volumes of water) and reversibility (ice can change to
water and back to ice again).

During formal operations children can think abstractly, reason deductively, and define abstract concepts.
Trust vs mistrust is one of Erik Erikson’s stages and is a distractor in this question.
Human Development
K&S Chapter 2

Question 32. D. The anterior cerebral artery (ACA) territory is the area
affected by this stroke. The ACA irrigates the medial
frontal lobes; therefore when it’s affected it causes pref-
erential leg greater than arm and face weakness contra-
lateral to the side of the lesion. The destruction of the
nearby frontal eye fields causes loss of tonic opposi-
tion of the gaze to the opposite side, so the eyes will
often be noted as looking toward the side of the lesion.
Sphincteric incontinence is sometimes seen if the corti-
cal bowel and bladder areas are affected. Patients often
suffer from abulia, a loss of will power or the lack of
ability to do things independently. Bilateral ACA terri-
tory damage can result in akinetic mutism.
Neurology
B&D Chapter 51

Question 33. A. NE is made in the locus ceruleus. Serotonin is made in the dorsal raphe nuclei. Dopamine is made in the
substantia nigra. Acetylcholine is made in the nucleus basalis of Meynert.
Basic Neuroscience
K&S Chapter 1

100

Psychiatry Test Preparation and Review


Manual E-Book
Test Number Two

Question 34. E. Pure motor hemiparesis is one of the classic lacunar stroke syndromes and would be expected from an
infarct in the area of the internal capsule, the basis pontis, or the corona radiata. Lacunar strokes are
characterized as ischemic strokes resulting from small-vessel lipohyalinosis, for which hypertension and
diabetes are the two major risk factors. Lacunar infarcts by definition are small and range from 0.5 mm
to 1.5 cm in diameter. Microembolism may also be a mechanism of lacunar strokes.

Other lacunar syndromes include pure sensory stroke, resulting usually from a small lesion in the ventro-
posterolateral nucleus of the thalamus; sensorimotor stroke, resulting from a stroke to the internal cap-
sule and thalamus or the posterior limb of the internal capsule; ataxic hemiparesis, which results from a
lacunar infarct to either the basis pontis or the posterior limb of the internal capsule; and the dysarthria–
clumsy hand syndrome, resulting from a stroke to the deep areas of the basis pontis.
Neurology
B&D Chapter 51

Question 35. D. Of the receptors listed, it is the histamine receptor that is associated with weight gain and sedation. The
M1 receptor is associated with constipation, blurred vision, dry mouth, and drowsiness. The α1 recep-
tors are associated with dizziness and decreased blood pressure. The 5-HT 1A receptor is a presynaptic
autoreceptor involved in the response of neurons to the SSRIs. The 5-HT 2A receptor is one of the post-
synaptic serotonin receptors involved in the neuron’s response to the SSRIs. The serotonin receptors are
associated with modulation of depression and anxiety but not with weight gain or sedation.
Basic Neuroscience
K&S Chapter 1

Question 36. A. The contralateral subthalamic nucleus of Luys is most often the area where a lesion will produce hemi-
ballismus. Hemiballismus is a dramatic, flinging movement of the proximal extremities. It can affect
both the upper and the lower limbs and is most often unilateral. Most frequently, the cause of hemibal-
lismus is an acute stroke.
Neurology
B&D Chapter 71

Question 37. B. The six biogenic amine neurotransmitters are dopamine, epinephrine, NE, acetylcholine, histamine, and
serotonin. Of these, dopamine, NE, and epinephrine are all synthesized from the precursor tyrosine and
are known as a group as the catecholamines. GABA is an amino acid neurotransmitter, not a biogenic
amine. Of note is that cocaine works by blocking the reuptake of the biogenic amines, more specifically
serotonin, NE, and dopamine.
Basic Neuroscience
K&S Chapter 1

Question 38. D. Reserpine interferes with the magnesium and ATP-dependent uptake of biogenic amines, which results in
the depletion of dopamine, NE, and serotonin neurotransmitters. In this way, it can relieve symptoms of
dystonia and can also cause depression, sedation, and a Parkinson’s-like syndrome.
Neurology
B&D Chapter 71

Question 39. A. Projection is perceiving and reacting to unacceptable inner impulses as though they were outside the self.
On a psychotic level, it takes the form of delusions about external realities that are usually persecutory
in nature. A patient’s own impulses and hostilities are projected onto another, who is now assumed to
have intentions to persecute the patient. The other answer choices are distractors that have nothing to
do with projection.
Psychological Theory and Psychometric Testing
K&S Chapter 5

101

Psychiatry Test Preparation and Review


Manual E-Book
Psychiatry Test Preparation & Review Manual

Question 40. E. The arachnoid granulations are the major sites for drainage of CSF into the blood. These granulations
protrude through the dura into the superior sagittal sinus and act as one-way valves or siphons for the
CSF. Equilibrium is maintained by the arachnoid granulations, for if the CSF pressure drops below a
certain level, absorption stops. If the CSF pressure increases, more fluid is absorbed. The choroid plex-
uses that protrude into the ventricles are responsible for the majority of CSF production in humans.
In humans, the rate of CSF production is about 0.35 mL/min. Drugs that can temporarily reduce pro-
duction of CSF and ease increased intracranial pressure include acetazolamide and mannitol. States of
hypothermia, hypocarbia, hypoxia, and hyperosmolality can also temporarily decrease CSF production.
Virchow–Robin spaces act as a duct for substances in the subarachnoid space to enter the brain. The
other answer choices are distractors.
Neurology
B&D Chapter 59

Question 41. D. Regression analysis is a method of predicting the value of one variable in relation to another variable
based on observed data. Probability is the likelihood that an event will occur. A probability of 0 means it
will not occur. A probability of 1 means it will definitely occur. Point prevalence is the number of people
with a disorder at a specific point in time divided by the total population at that point in time. Incidence
is the number of new cases of a disease over a given time divided by the total number of people at risk
during that time. Central tendency is a central value in a distribution around which other values are
arranged. Examples of central tendency are the mean, median, and mode. The mean is the average. The
median is the middle value in a series of values. The mode is the value that appears most frequently in
a series of measurements. Kappa is a variable used to indicate a constant value that does not change.
Sensitivity is the ability of a test to detect that which is being tested for.
Statistics
K&S Chapter 5

Question 42. D. Noncontrast head CT is the best immediate test of choice in the emergency room if a lobar hemorrhage
is suspected. MRI of the brain can add precision but in the hyperacute stage may not be able to reveal
acute bleeding as well as the CT scan. The most common location of intracranial hemorrhage is from
the putamen, which accounts for about 35% of cases. Lobar hemorrhage is second to putaminal hemor-
rhage in frequency and accounts for about one-quarter of cases. Hypertension remains the most frequent
cause of intracranial hemorrhage, but arteriovenous malformations, cerebral amyloid angiopathy, and
sympathomimetic agents can also account for quite a number of cases.

Emergency room management of suspected intracranial hemorrhage begins with stabilization of vital
signs and airway protection. Intubation is indicated if the level of consciousness drops to a Glasgow
Coma Scale score of 8 or less. CT scan of the head must be done next to determine the location and
size of the hemorrhage. Neurosurgical consultation may be indicated if the hemorrhage is large and
increased intracranial pressure is suspected.

Blood work for checking the coagulation panel is very important. If the patient is on oral or paren-
teral anticoagulation therapy, it is imperative to consider reversing anticoagulation by protamine sulfate
for those on heparin or with parenteral vitamin K or fresh frozen plasma for those patients on warfarin
therapy.

Bedside EEG is not indicated unless there is pervasive coma or seizures. Lumbar puncture is usually con-
traindicated if intracranial or lobar hemorrhage is suspected, because it can trigger uncal herniation and
eventual death if the patient is already in a state of increased intracranial pressure. The lumbar puncture
with CSF xanthochromia assay is useful only if a subarachnoid hemorrhage is suspected.
Neurology
B&D Chapter 51

Question 43. E. Outcomes in psychodynamic therapy have been found to be closely associated with the empathy pro-
vided by the therapist. The goal is for the therapist to be in tune with the patient’s internal state in an
empathetic way. When patients feel that therapists understand their internal world, they are more likely

102

Psychiatry Test Preparation and Review


Manual E-Book
Test Number Two

to accept interpretations made by the therapist. The other answer choices, although important, are not
the most important. Some of these distractors may have effects on the therapeutic relationship but will
not universally be the most important factor in the outcome of therapy.
Psychotherapy
K&S Chapter 28

Question 44. A. Melatonin is believed to be released principally by the pineal gland, and there is a feedback loop
between the pineal gland and the suprachiasmatic nucleus in the hypothalamus that helps with sleep
regulation. Melatonin is secreted predominantly at night, and levels peak between 3:00 am and 5:00 am
and decrease to lower levels during the day. Melatonin is a modulator of human circadian rhythm for
entrainment by the light–dark cycle.
Basic Neuroscience
B&D Chapter 68

Question 45. D. The MMPI is an objective psychological test. It is a self-report inventory used to find areas of pathologi-
cal personality structure. The other tests listed are projective tests. In projective tests the patient is given
a picture or incomplete piece of information and asked to fill in the details or complete the unfinished
task. The answers that the patient gives reveal aspects of his or her personality, thought content, thought
structure, and psychological makeup. In the Rorschach test the patient is shown ink blots and asked to
share what he or she thinks the ink blots look like. In the sentence completion test the patient is asked
to finish incomplete sentences. In the thematic apperception test the patient is shown a series of pictures
and asked to make up stories based on the pictures. In the draw a person test the patient is asked to
draw persons of the same and the opposite sex. The person of the same sex is thought to represent the
patient. Interpretations are made based on the details the patient puts into the drawings.
Psychological Theory and Psychometric Testing
K&S Chapter 5

Question 46. A. Subacute sclerosing panencephalitis (SSPE) is the result of a persistent and nonproductive viral infection
of the neurons and glia that is caused by the measles virus. It is a late complication of measles. Children
5 to 10 years of age are affected most often. Clinical manifestations include personality and cognitive
changes, myoclonic seizures, spasticity, choreoathetoid movements, and difficulty swallowing, eventually
leading to coma and death. The three tests of choice are lumbar puncture for CSF assay, EEG, and brain
biopsy. CSF reveals an elevated measles antibody titer, oligoclonal bands, absence of pleocytosis, nor-
mal glucose, and normal to elevated protein. Brain biopsy may reveal neuronal and glial nuclear and
cytoplasmic viral inclusion bodies. EEG reveals a characteristic pattern of periodic bursts of generalized
slow-wave complexes. There is no specific accepted treatment, but studies have revealed that intraven-
tricular interferon-α combined with oral isoprinosine can be helpful. Prognosis is generally poor, and
often death can be expected within 12 months if response to treatment is poor. There is no evidence that
SSPE results from measles vaccination in children.
Neurology
B&D Chapter 53

Question 47. E. ADHD is diagnosed by six or more symptoms of inattention or six or more symptoms of hyperactiv-
ity–impulsivity that persist for 6 months or more. Several inattentive or hyperactive–impulsive symptoms
should be present before age 12. Several symptoms of impairment must be present in more than one
setting to make the diagnosis. Symptoms of inattention involve failure to pay close attention to tasks,
failure to sustain attention, not listening, not following through on tasks, problems organizing tasks,
forgetfulness, and being easily distracted by extraneous stimuli. Symptoms of hyperactivity–impulsivity
include fidgeting, inability to remain seated when expected, running or climbing excessively, difficulty
playing quietly, acting as if driven by a motor, talking excessively, blurting out answers, difficulty await-
ing turn, and interrupting others.
Disruptive, Impulse Control, Conduct Disorders, and ADHD
K&S Chapter 31

103

Psychiatry Test Preparation and Review


Manual E-Book
Psychiatry Test Preparation & Review Manual

Question 48. E. Lesch–Nyhan syndrome is an X-linked recessive hereditary disorder of purine and pyrimidine metab-
olism. Hyperuricemia results from a deficiency in hypoxanthine–guanine phosphoribosyltransferase.
Clinical symptoms and signs of the syndrome include choreoathetosis, hyperreflexia, hypertonia, dysar-
thria, behavioral disturbances, cognitive impairment, and self-mutilatory behavior. Neurologic signs and
symptoms are probably a result of diminished dopamine concentrations in the CSF and basal ganglia.

Metachromatic leukodystrophy is an autosomal recessive disorder caused by arylsulfatase A deficiency.


There is an accumulation of excess sulfatides in the nervous system, which leads to progressive demy-
elination. The disorder localizes to chromosome 22.

Tay–Sachs is a recessive disorder localizing to chromosome 15. It is caused by a deficiency in hexosaminidase


A. The adult form presents as progressive weakness in the proximal muscles of the upper and lower extremi-
ties. Associated symptoms may involve spasticity, dysarthria, and cognitive and psychiatric impairment.

Krabbe’s disease, also called globoid cell leukodystrophy, is an autosomal recessive disease that local-
izes to chromosome 14. It is a result of a deficiency in the lysosomal enzyme galactocerebroside
β-galactosidase. Generalized central and peripheral demyelination is the hallmark of the disorder, as is
the presence of multinucleated macrophages (globoid cells) in cerebral white matter. Infantile arrest of
motor and cognitive development is noted, with seizures, hypertonicity, optic atrophy, and opisthotonic
posturing (extension of trunk and limbs with increased muscle tone) occurring. Stem cell transplantation
may reverse the neurologic deficits by providing the missing enzyme.

Gaucher’s disease is an autosomal recessive disorder resulting from β-glucosidase deficiency. It localizes
to chromosome 1. There are three identified types. Type I presents with the characteristic findings of
hematologic anomalies, hypersplenism, bone lesions, skin pigmentation, and ocular pingueculae (growth
on the conjunctiva).
Neurology
B&D Chapters 62 and 76

Question 49. B. To diagnose schizophrenia, active-phase symptoms must be present for a 1-month period only. It may be
diagnosed if other symptoms (i.e., negative) are present over a 6-month period, but active-phase symptoms
must be present for at least 1 month of those 6 months. All other answer choices are part of the character-
istic active-phase symptoms for schizophrenia. Social isolation is a bad prognostic sign for schizophrenia.
Good prognostic signs include late age of onset, acute onset, and the presence of an affective component.
Psychotic Disorders
K&S Chapter 7

Question 50. D. Botulinum toxin type A is FDA-approved for the treatment of cervical dystonia, blepharospasm, hemi-
facial spasm, strabismus, and axillary hyperhidrosis, and it’s approved cosmetically for hyperactive gla-
bellar lines of the procerus muscles of the forehead. Although not FDA-approved for spasticity related
to multiple sclerosis, it is an excellent treatment for that particular disorder. Botulinum toxin type A is
also used off-label for the treatment of migraine. The mechanism of action of botulinum toxin type A
is through blockade of neuromuscular transmission via blockade of presynaptic acetylcholine release.
Botulinum toxin type A has no place in the treatment of RLS.
Neurology
B&D Chapter 48

Question 51. D. This question is a clear description of a delusional disorder. In a delusional disorder, the patient usually
presents with a bizarre or nonbizarre delusion. There are no other psychotic symptoms present. His abil-
ity to function in his daily life is preserved. There is not a significant mood component to the disease.
The question says nothing about memory impairment; therefore dementia is not correct. The patient
does not meet criteria for active-phase schizophrenic symptoms; therefore schizophrenia and schizoaf-
fective disorder are ruled out. Brief psychotic disorder lasts less than a month, and this patient has had
symptoms for 5 months, so this answer choice is incorrect.
Psychotic Disorders
K&S Chapter 7

104

Psychiatry Test Preparation and Review


Manual E-Book
Test Number Two

Question 52. A. The clinical presentation described in this question is that of classic tension-type or muscle-contraction
headache. These headaches can result from both muscular and emotional tension and stress. These head-
aches are not well understood, and muscle tension does not fully explain their pathogenesis. Often,
treatment with rest, sleep, or simple analgesics is sufficient to bring relief. These headaches can begin
at any age, are most often bilateral, and are described as band-like around the head. Patients describe
squeezing, pressure, or burning types of pain. The pain can wax and wane throughout the day and can
be present, if left untreated, from a period of hours to days. This headache type is usually free of any
associated or autonomic symptoms, which helps differentiate it from migraine. Treatment of tension-
type headache usually begins with aspirin or acetaminophen. If headaches are more severe, combination
drugs, like acetaminophen–butalbital–caffeine (Fioricet), or nonsteroidal antiinflammatory agents, such
as naproxen or ibuprofen, may be helpful.

Migraine is differentiated from tension-type headache mainly by its stereotyped features and associated
autonomic symptoms. Migraine is generally unilateral, pulsatile, and accompanied by photophobia, pho-
nophobia, or osmophobia (hypersensitivity to odors). Other features can include nausea and vomiting,
anorexia, blurred vision, and light-headedness. The aura that can precede a migraine involves transient
visual, motor, or focal neurologic deficits or symptoms. The scintillating scotoma is the most common
visual manifestation of aura. The patient can perceive a shimmering arc or zigzag pattern of light in the
peripheral visual field that can gradually enlarge. Sensory aura can involve transient numbness and tin-
gling in the extremities that can last from seconds up to a half-hour’s duration. Motor aura is rare and
can manifest as a transient motor weakness or hemiparesis, in much the same way as the sensory defi-
cits. The patient may therefore misinterpret these symptoms as the onset of a stroke.

Basilar migraine usually presents with occipital headache and neurologic symptoms attributable to the
brainstem. The aura can involve visual and sensory phenomena, but these are often accompanied by ver-
tigo, dysarthria, tinnitus, and speech deficits. Loss of consciousness may occur if the reticular activating
system of the pons is involved. The patient will often experience severe occipital pain after awakening
from the aura. These attacks resemble stroke-like symptoms in the vertebrobasilar territory.

Paroxysmal hemicrania is a special headache entity that falls into the category of indomethacin-responsive
headache syndromes. This headache type begins early in life and is more common in women than in men
(2:1 ratio). The hemicrania can be episodic or chronic. Pain is unilateral and brief, occurring generally in
the periorbital or temporal areas. Attacks are frequent, usually five or more daily, and each lasts about 20
minutes. Each attack is accompanied by autonomic features, such as conjunctival injection, lacrimation,
ptosis, and rhinorrhea, ipsilateral to the side of the headache. Response to indomethacin is usually dra-
matic. The mechanism of indomethacin’s efficacy in this headache variety is not well understood.
Neurology
B&D Chapter 69

Question 53. E. In major depressive disorder, libido is decreased. Increased libido is often found in mania.
Depressive Disorders
K&S Chapter 8

Question 54. B. Generally speaking there is a higher prevalence of psychiatric disorders in women with polycystic ovar-
ian syndrome than in the general population. The disorder with the most elevated rates in polycystic
ovarian syndrome is depression. It is found to be the most significantly increased psychiatric condition
in several studies. The astute student should also keep in mind that Depakote, which we frequently use
in bipolar disorder, is known to cause polycystic ovarian syndrome, which leads to hair loss, hirsut-
ism, menstrual irregularities, insulin resistance, hyperinsulinemia, and the development of a metabolic
syndrome.
Depressive Disorders
K&S Ch. 29; BMC Medicine June 2010; Fertility and Sterility June 2007; Frontiers Endocrinology V5 2014.

Question 55. C. This is a case of SAD, which is a depression that sets in during the fall and winter and resolves during
the spring and summer. It is often characterized by hypersomnia, hyperphagia, and psychomotor slow-
ing. Treatment involves exposure to bright artificial light for 2 to 6 hours each day during the fall and

105

Psychiatry Test Preparation and Review


Manual E-Book
Psychiatry Test Preparation & Review Manual

winter months. It is thought to be related to abnormal melatonin metabolism. All other answer choices
given are distractors and are unrelated to the patient’s primary problem.
Depressive Disorders
K&S Chapter 8

Question 56. C. Carbidopa serves a simple function: it inhibits DOPA decarboxylase and thereby prevents the peripheral
metabolism of levodopa to dopamine before it can cross the blood–brain barrier. The dopamine ago-
nists such as ropinirole, pramipexole, pergolide, and bromocriptine are direct postsynaptic dopamine
receptor agonists indicated for Parkinson’s disease treatment. Selegiline is the selective MAO-B inhibitor
that blocks dopamine degradation by MAO-B and can potentiate the action of levodopa in the CNS.
Catechol-O-methyltransferase (COMT) inhibitors, such as tolcapone and entacapone, prevent periph-
eral degradation of levodopa and central metabolism of levodopa and dopamine, thereby increasing cen-
tral levodopa and dopamine levels. COMT inhibitors are usually taken simultaneously with doses of
levodopa–carbidopa. Anticholinergic agents, such as trihexyphenidyl and benztropine, can reduce tremor
in Parkinson’s disease by blockade of postsynaptic muscarinic receptors. These agents carry the dan-
ger of toxic side effects that include confusion, blurred vision, urinary retention, constipation, and dry
mouth.
Neurology
B&D Chapter 71

Question 57. E. This question includes common symptoms found in a panic attack. Others include trembling, choking
sensations, dizziness, fear of losing control, fear of death, paresthesias, chills, or hot flushes. Lifetime
prevalence of panic disorder is 1% to 4%. The patient in question does not present with the characteris-
tic signs and symptoms of a manic episode. Myxedema madness is a depressed and psychotic state found
in some patients with hypothyroidism. Mad Hatter syndrome presents as manic symptoms resulting
from chronic mercury intoxication. The patient describes no psychotic symptoms, so psychotic disorder
NOS is clearly the wrong choice. The prudent student will also keep in mind that the organic differen-
tial for panic disorder includes hyperthyroidism, hypothyroidism, pheochromocytoma, cardiac arrhyth-
mias, myocardial infarction, irritable bowel syndrome, migraines, angina, asthma, epilepsy, Huntington’s
disease, hyperparathyroidism, amphetamine intoxication, cocaine, marijuana, caffeine, nicotine, alcohol
(ETOH) withdrawal, mitral valve prolapse, and B12 deficiency.
Anxiety Disorders
K&S Chapter 9

Question 58. E. Apoptosis refers to programmed neuronal cell death. This concept is believed to account for the patho-
physiology of several neurodegenerative diseases. These diseases include spinal muscular atrophy,
Parkinson’s disease, amyotrophic lateral sclerosis, and Alzheimer’s dementia.
Neurology
B&D Chapter 60

Question 59. D. People with paranoid personality disorder externalize their own emotions and use the defense of pro-
jection. They attribute to others the impulses and thoughts they cannot accept in themselves. They are
characterized by long-standing suspiciousness and mistrust of others. Other defense mechanisms are dis-
cussed elsewhere in this volume.
Personality Disorders
K&S Chapter 22

Question 60. B. Sturge–Weber syndrome is a neurocutaneous disorder that is sporadic and not genetically inherited. The
hallmark is the presence of a facial cutaneous angioma (port-wine nevus), usually with a brain angioma
ipsilateral to the skin lesion. Other characteristics can include contralateral hemiparesis, mental retar-
dation, and homonymous hemianopia. Glaucoma is common and if untreated can lead to blindness.
Seizures develop in over 70% of Sturge–Weber patients and present most often as motor seizures or gen-
eralized tonic–clonic seizures. Treatment of seizures is achieved with anticonvulsant medication, which
if proven ineffective may lead to the consideration of surgical intervention, such as hemispherectomy to
excise the offending brain tissue.

106

Psychiatry Test Preparation and Review


Manual E-Book
Test Number Two

Von Hippel–Lindau syndrome (VHL) is a heritable neurocutaneous disorder that is transmitted with an
autosomal dominant pattern of inheritance. The VHL gene is a tumor-suppressor gene on chromosome
3. Prevalence is about 1 in 40,000 to 100,000. The predominant clinical presentation is characterized
by retinal and CNS hemangioblastomas and visceral cysts and tumors. Hemangioblastomas are slow-
growing vascular tumors that are benign but can bleed and cause local mass effect. The most common
CNS site of this tumor type is the cerebellum in about 50% of cases. Renal cysts occur in about half of
patients with VHL. Pheochromocytoma occurs in about 10% to 20% of patients. Frequent follow-up
and screening for tumors are the managements of choice for patients with VHL.

Ataxia–telangiectasia is a neurodegenerative neurocutaneous disorder that is inherited with an auto-


somal recessive pattern of inheritance. Prevalence is about 1 in 40,000 to 100,000. The hallmark of
the disorder is the development of early-onset ataxia in childhood. Ataxia tends to develop at around
12 months of age, when the child begins to walk. Children typically are wheelchair-bound by 12 years
of age. Telangiectases (small dilated blood vessels) tend to develop later, from about age 3 to 6 years,
and often affect the earlobes, nose, and sclerae. There is also a higher risk of lymphoma and leuke-
mia in patients with the disorder than in the general population at large. Abnormal eye movements are
common in children with the disorder. These include nystagmus, ocular motility impairment, and gaze
apraxia.

Fabry’s disease is an X-linked lysosomal storage disease resulting from deficiency in α-galactosidase
A. Stigmata of the disease include the development of asymptomatic red or purple papules that occur
around the umbilicus, hips, thighs, and scrotum area. Other characteristics include corneal deposits,
painful dysesthesias of the distal extremities, cerebral thrombosis or hemorrhage, and eventual vascular
narrowing from deposition of glycolipids in the arterial endothelium. Renal failure is a common cause of
death due to renal vascular compromise. Treatments are disappointing in Fabry’s disease. Enzyme reple-
tion does not help the clinical problems. Plasmapheresis is also not particularly effective. Renal trans-
plantation can delay systemic complications and alleviate renal failure but is not curative.

Tuberous sclerosis is discussed elsewhere in this volume in great detail.


Neurology
B&D Chapter 65

Question 61. A. The doctor who takes Medicare must accept Medicare’s maximum fee. This fee includes any appropriate
co-pays set forth in the patient’s policy. The physician cannot bill the patient or another party for the dif-
ference between what Medicare allows and what the physician wants to charge.
Public Policy
K&S Chapter 35

Question 62. E. Reflex sympathetic dystrophy is one of the so-called complex regional pain syndromes. It is the result of
regional pain and sensory changes after trauma or a noxious event. There may or may not be a diagnos-
able underlying nerve lesion to the problem. Soft tissue injury is the trigger in about 40% of cases, and
bony fracture is the trigger in about 25% of cases.

There are three stages to the disorder. Stage 1 (acute) is associated with pain that is out of proportion to
the initial injury. There is usually hypersensitivity to painful stimuli or physical contact with the extrem-
ity. Stage 2 (dystrophic) is associated with tissue edema and skin that is cool, cyanotic, hyperhidrotic,
with livedo reticularis. Pain is constant and increases with any physical contact to the affected extrem-
ity. Stage 3 (atrophic) is associated with paroxysmal pain and irreversible tissue damage. The skin can
become thin and shiny, and the fascia can become thickened or contractured. Treatments begin with
intense physical therapy, which can be helpful by improving mobility of the affected extremity. Steroids
may be helpful in certain cases. Phenoxybenzamine, which is a sympathetic blocking agent, can also be
useful, if tolerated. Some patients require invasive anesthesia by regional block, which can be helpful in
certain patients.
Neurology
B&D Chapter 29

107

Psychiatry Test Preparation and Review


Manual E-Book
Psychiatry Test Preparation & Review Manual

Question 63. B. In the situation in which you are presented with an unknown psychotic or paranoid patient, the first step
is to make sure that the patient and staff are safe. All other options are valid but would be taken after
an initial assessment of the patient’s safety has been made. When given a question such as this on an
examination, maintenance of safety comes first, and diagnosis and treatment come later. You would not
automatically give sedation to the patient if he or she were not agitated or dangerous, so answer choice
A is not the correct first move. Remember that past violence is the best predictor of future violence.
Management in Psychiatry
K&S Chapter 25

Question 64. B. Carbamazepine is an inducer of CYP 2C19 and 3A4. It can therefore induce the metabolism of any sub-
strate of the 2C19 and 3A4 systems. The 3A4 substrates that can be induced by carbamazepine include
erythromycin, clarithromycin, alprazolam, diazepam, midazolam, cyclosporine, indinavir, ritonavir, saqui-
navir, diltiazem, nifedipine, amlodipine, verapamil, atorvastatin, simvastatin, aripiprazole, buspirone, halo-
peridol, tamoxifen, trazodone, propranolol, zolpidem, zaleplon, methadone, estradiol (oral contraceptives),
progesterone, testosterone, and fentanyl, to name but a few. The 2C19 substrates that can be induced by
carbamazepine include amitriptyline, citalopram, clomipramine, imipramine, R-warfarin, propranolol, and
primidone. Carbamazepine can lower lamotrigine levels by induction of glucuronidation enzyme 1A4.
Clozapine is a substrate of CYP 1A2 and is not affected by carbamazepine.
Neurology Psychopharmacology
B&D Chapters 8, 52, and 67

Question 65. E. The standard of substituted judgment holds that a surrogate decision-maker will make decisions based
on what the patient would have wanted and implies that the decision-maker be familiar with the patient’s
values and attitudes. The best interest principle, which was the past but not current standard, states that a
decision-maker will decide which option would be in the patient’s best interests. Patients do have the right
to refuse treatment that they feel would lessen their quality of life. Advanced directives and living wills
are ways for patients to preserve their wishes in writing so that their desires are reflected in the decisions
that are made for them should they become incapacitated. The state will follow the course that preserves
human life should a suitable surrogate decision-maker not be present. Surrogate decision-makers can be
appointed by the patient, the court, or the hospital. In many cases this person is the patient’s next of kin.
Forensic Psychiatry
K&S Chapter 36

Question 66. D. Tobacco smoking is a potent inducer of CYP 1A2. As such, it can significantly lower levels of amitripty-
line, fluvoxamine, clozapine, olanzapine, haloperidol, and imipramine. Risperidone is a substrate of CYP
2D6, and its levels can be lowered by 2D6 inducers, such as dexamethasone and rifampin. Risperidone
levels can be significantly increased by 2D6 inhibitors, such as bupropion, citalopram, clomipramine,
doxepin, duloxetine, escitalopram, fluoxetine, paroxetine, sertraline, and perphenazine. Risperidone lev-
els are not affected by tobacco smoking.
Psychopharmacology
B&D Chapter 9

Question 67. E. One cannot treat what is assumed to be denial before actually knowing that it is denial. Often in medi-
cal settings things are not clearly explained to patients in language they can understand. As such, it is
first necessary to explain what is going on to the patient clearly and in language he or she can fol-
low. Confronting the patient’s denial forcefully, and aggressively removing his or her defenses against
overwhelming emotion, is potentially more harmful than helpful. Playing along with the denial can
lead to noncompliance and failed treatment outcomes and is a bad idea. Neuropsychological testing is
not necessary if a patient is in denial. If the patient had a neurocognitive deficit that precluded his or
her understanding of the material as presented it could be considered, but those were not the circum-
stances described in this question. Meeting the family is important, but not more important than making
sure the patient has had the situation explained properly. The patient is the primary person who has to
understand. Informing the family while the patient is left in the dark is not the best approach.
Psychological Theory and Psychometric Testing
K&S Chapter 5

108

Psychiatry Test Preparation and Review


Manual E-Book
Test Number Two

Question 68. A. This is a classic case of Sheehan’s syndrome, which is essentially a postpartum pituitary infarction or
apoplexy. The classic symptoms are those similar to a pituitary hemorrhage. The pituitary infarction can
cause chiasmal compression that can lead to bitemporal hemianopsia from bilateral medial optic nerve
compression. Hypotension can be caused by a preexisting adrenocorticotropic hormone deficiency. The
clinical picture can resemble a subarachnoid hemorrhage owing to the rupture of a berry aneurysm or
an arteriovenous malformation. This would account for the severe acute headache and cranial nerve
palsy. The syndrome can also result in meningeal irritation with positive Kernig’s and Brudzinski’s signs
and a stiff neck. The diagnosis can often be confirmed by a CT or MRI scan of the brain. Treatment is
generally supportive, and only in certain cases is it necessary to provide corticosteroid replacement or
conduct a surgical decompression.

Cushing’s disease is caused by endogenous overproduction of adrenocorticotropic hormone from the


anterior pituitary gland. Cushing’s syndrome is the result of exposure to either excessive exogenous
or excessive endogenous corticosteroids. The clinical picture is that of hypertension, truncal obe-
sity, impaired glucose tolerance or diabetes mellitus, menstrual irregularities, hirsutism, acne, purplish
abdominal striae, osteoporosis, thin skin with excessive bruising, and proximal myopathy. The diagnos-
tic test of choice is the dexamethasone suppression test.

Subarachnoid hemorrhage is most often the result of an intracranial arterial aneurysmal rupture. The
acute presentation very often consists of a sudden explosive headache (“the worst headache of my life”),
meningeal signs, nausea, vomiting, photophobia, and obtundation. Head CT scan is the imaging test of
choice because it reveals acute blood better than MRI. CT scans can reveal subarachnoid blood in only
about 90% to 95% of patients within 24 hours of the hemorrhage. This sensitivity drops to about 80%
after 72 hours. A negative CT scan should therefore be followed by a lumbar puncture with centrifug-
ing of CSF to look for characteristic xanthochromia (from lysed red blood cells). If either CT or lumbar
puncture is positive for subarachnoid hemorrhage, then cerebral angiography should be performed as
soon as possible to determine the location of the aneurysm and the best method of intervening. The
most popular interventions for intracranial aneurysms are endovascular coiling, microsurgical clipping,
and balloon embolization. Saccular or berry aneurysms are the most common kind of intracranial aneu-
rysms. Between 80% and 85% of aneurysms stem from the anterior cerebral circulation, most often
arising from the anterior communicating artery, the posterior communicating artery, or the trifurcation
of the MCA. Between 15% and 20% of aneurysms originate in the posterior circulation, most often
from the origin of the posterior inferior cerebellar artery or at the bifurcation of the basilar artery.

Bacterial meningitis presents in adults as a febrile illness involving headache, stiff neck, and signs of
cerebral dysfunction, which are present in over 85% of patients. Associated signs and symptoms
include nausea, vomiting, photophobia, and myalgia. Often Kernig’s and Brudzinski’s signs can be elic-
ited. Cerebral dysfunction can involve delirium, confusion, and a decreased level of consciousness that
ranges from lethargy to coma. Seizures can occur in about 40% of cases. Cranial nerve palsies can be
seen in 10% to 20% of patients. Increased intracranial pressure may result in the appearance of bilat-
eral VI nerve palsies. Lumbar puncture and blood cultures clinch the diagnosis. CSF opening pressure
is high (200 to 500 mm H2O) and protein is also high (100 to 500 mg/dL, normal = 15 to 45 mg/dL).
Decreased glucose and marked pleocytosis are also noted (100 to 10,000 WBC/mL, normal = <5), with
60% or more polymorphonuclear leukocytes. CSF cultures are positive in about 75% of cases. In the
United States, the predominant causative organism in children ages 2 to 18 years is Neisseria meningiti-
dis, and in adults it’s Streptococcus pneumoniae. Intravenous ampicillin, penicillin-G, and third-genera-
tion cephalosporins (ceftriaxone, cefotaxime, ceftazidime) are the usual agents of first-line treatment.

Familial hemiplegic migraine (FHM) is a rare autosomal dominant migraine subtype in which the aura is
accompanied by hemiplegia. FHM has been mapped to chromosome 19p13, which codes for the α1 sub-
unit of brain-specific voltage-gated P/Q-type calcium channel. There is some notion that because FHM
has symptoms similar to those of traditional migraine with aura, this implies that migraine with aura
may also be genetically linked to chromosome 19.
Neurology
B&D Chapters 42 and 53

109

Psychiatry Test Preparation and Review


Manual E-Book
Psychiatry Test Preparation & Review Manual

Question 69. C. The question stem accurately describes agoraphobia. The other answer choices are distractors.
Agonothete is the judge of games in ancient Greece. Agoniada is the bark of a South American shrub.
Agora is the market place in ancient Greece. Agouara is a South American wild dog or a crab-eating rac-
coon. Needless to say, the only one that will show up on the boards is agoraphobia.
Anxiety Disorders
K&S Chapter 9

Question 70. C. Normal-pressure hydrocephalus (NPH) presents with the triad of dementia, incontinence, and gait dis-
turbance. The cause in up to one-third of cases is undetermined. NPH can be caused by trauma, infec-
tion, or subarachnoid hemorrhage. Brain CT scan or MRI reveal enlargement of the third, fourth,
and lateral ventricles. NPH causes an apraxic, or magnetic gait, making it difficult for the patient to
raise his or her legs off the ground. The dementia is considered to be subcortical and results in slow-
ing of verbal and motor functioning, although the cortical functions remain intact. Abulia, apathy, and
depression are common in NPH. Urinary incontinence occurs early in the course of the illness, particu-
larly when there is prominent gait disturbance. Lumbar puncture reveals a normal opening pressure.
Serial lumbar punctures with drainage of CSF can improve the symptoms and support the diagnosis of
NPH. Ventriculoperitoneal shunting is the treatment of choice and is successful in up to 80% of cases.
Shunts fail about one-third of the time, and complications of shunting include subdural hematoma and
infection.

Diffuse Lewy-body disease, or dementia with Lewy bodies, is the second-most prevalent dementia after
Alzheimer’s disease. The symptomatic triad is that of dementia, parkinsonism, and visual hallucinations.
A hallmark of the disease is extreme sensitivity to dopamine receptor antagonists, which can result in
severe parkinsonism when treatment with neuroleptics is undertaken. Dopaminergic therapy is generally
not that helpful. Hallucinations are ideally treated with the newer antipsychotics, particularly quetiap-
ine, clozapine, and ziprasidone. Rivastigmine and donepezil may be useful cholinergic therapy to try to
improve cognition.

Wernicke’s encephalopathy presents with the acute triad of mental confusion, ophthalmoplegia, and gait
ataxia, predominantly in alcoholic patients. On autopsy, the neuropathologic hallmark is multiple small
hemorrhages in the periventricular gray matter, mainly around the aqueduct and the third and fourth
ventricles. An MRI of the brain often reveals abnormalities in the periaqueductal regions, bilateral mam-
millary bodies, and medial thalami.

Sydenham’s chorea is a result of rheumatic fever. It is extremely rare these days because of the widespread
availability of antistreptococcal therapy. The disorder is more frequent in girls generally between 5 and 15
years of age. Chorea begins insidiously over a period of weeks and can last up to 6 months. Behavioral
manifestations can include irritability, obsessive–compulsive traits, and restlessness. Enlargement of the
basal ganglia can be noted on MRI. Symptoms are generally self-limited, lasting up to about 6 months.
Anti-basal ganglia antibodies can be detected by Western blot testing, and these antibodies seem to account
for the mechanism of the disorder. Valproic acid is a useful therapy for the chorea.
Neurology
B&D Chapters 66 and 71

Question 71. E. Cyclothymic disorder does not involve psychotic symptoms, although these symptoms may be found in
bipolar disorder. Cyclothymia is a less severe form of bipolar, with alternation between hypomania and
moderate depression. Symptoms must exist for 2 years to make the diagnosis. It is equally common in
men and women. Substance use often coexists. The onset is usually insidious and occurs in late adoles-
cence or early adulthood. Lifetime prevalence of cyclothymia is 0.5% to 6.3%.
Bipolar Disorders
K&S Chapter 8

Question 72. B. Depression is not considered a stroke risk factor. Common risk factors for stroke include older age,
male gender, low socioeconomic status, diabetes mellitus, obesity, cigarette smoking, excessive alcohol
consumption, and family history. Other important risk factors are arterial hypertension, prior stroke or

110

Psychiatry Test Preparation and Review


Manual E-Book
Test Number Two

transient ischemic attack, asymptomatic carotid bruit, dyslipidemia, hyperhomocysteinemia, and oral
contraceptive use. Hereditary blood dyscrasias also elevate stroke risk, such as protein C or S deficiency,
antithrombin III deficiency, and factor V Leiden deficiency.
Neurology
B&D Chapter 51

Question 73. D. This question gives a classic description of the appearance of a patient in the manic phase of bipolar
disorder. They are often very bizarre, colorful, seductive, and erratically behaved. A depressed patient
would be apathetic and psychomotor-retarded, not hyperexcited. There was no mention made in the
question of psychosis or delusions, so brief psychotic disorder and delusional disorder are incorrect.
There is also no mention of abnormally perceived body image, so body dysmorphic disorder is incor-
rect as well. There is an equal prevalence of bipolar disorder in women and men. Bipolar I disorder
in women most often starts with depression. Lifetime prevalence of bipolar I disorder is up to 2.4%.
An additional fact to keep in mind is that bipolar patients have three times the rate of type 2 diabe-
tes compared with the general population, irrespective of what medications they are prescribed. This
is an important clinical point that argues for increased awareness on the part of those treating bipolar
patients.
Bipolar Disorders
K&S Chapter 8, Annals of Medicine, March 2013.

Question 74. A. Ataxia is indicative of a cerebellar lesion, and the cerebellum is perfused by the vertebrobasilar arterial
system. Symptoms indicative of a carotid territory transient ischemic attack or stroke would be transient
ipsilateral monocular blindness (amaurosis fugax), contralateral body weakness or sensory loss, aphasia
with dominant hemisphere involvement, and contralateral homonymous visual field deficits. Symptoms
that suggest a vertebrobasilar territory stroke or transient ischemic attack include bilateral, shifting, or
crossed weakness or sensory loss (ipsilateral face with contralateral body); bilateral or contralateral
homonymous visual field defects or binocular visual loss; and two or more of vertigo, diplopia, dyspha-
gia, dysarthria, and ataxia.
Neurology
B&D Chapter 51

Question 75. E. Suicidal ideation is part of the thought content of the depressed patient. It is possible to find it in the
manic bipolar patient but is more likely to occur during the depressed phase of the illness and is not part
of the thought process.
Diagnostic and Treatment Procedures in Psychiatry
K&S Chapter 8

Question 76. D. Essential tremor (ET) is one of the most common movement disorders. It is defined as a postural and
kinetic tremor of the forearms and hands (sometimes with other body parts) that gradually increases in
amplitude over time. ET is believed to be a monosymptomatic illness without other neurologic deficits.
As many as two-thirds of patients have a positive family history of ET. The disease is likely heteroge-
neous with an autosomal dominant pattern of inheritance.

The mainstay of treatment is the use of β-adrenergic blocking agents and primidone. Propranolol is the
β-blocker of choice. This helps reduce tremor amplitude in roughly half of patients. Primidone can also
be used. Benzodiazepine sedative–hypnotic agents, like clonazepam and lorazepam, are also frequently
used and can be helpful. Botulinum toxin type A can be injected intramuscularly for head and hand
tremor and can be effective in certain cases. Desipramine and the tricyclic antidepressants are not useful
in ET and may, in fact, worsen tremor. If oral therapy fails, surgical intervention can be contemplated if
symptoms are serious. Stereotactic thalamotomy has been shown to reduce contralateral tremor in as
many as 90% of cases. Thalamic deep brain stimulation has also been demonstrated to be effective in
treating ET and can improve symptoms in up to 80% of cases.
Neurology
B&D Chapter 71

111

Psychiatry Test Preparation and Review


Manual E-Book
Psychiatry Test Preparation & Review Manual

Question 77. B. To qualify for a diagnosis of major depressive disorder, symptoms must be present for at least 2 weeks.
Symptoms can include depressed mood, diminished interest in activities, weight loss or gain, insomnia
or hypersomnia, psychomotor retardation, fatigue, feelings of worthlessness, decreased concentration,
recurrent thoughts of death, and recurrent suicidal ideation. Lifetime prevalence of major depressive dis-
order is 5% to 17%.
Depressive Disorders
K&S Chapter 8

Question 78. B. The thymus gland is believed to play a role in the pathogenesis of myasthenia gravis (MG). MG is an
autoimmune disorder, and the thymus gland is involved in tolerance to self-antigens. Ten percent of
patients with MG have a thymic tumor, and 70% have cellular hyperplasia of the thymus indicative of
an active immune response. About 20% of patients with MG who develop symptoms between 30 and
60 years of age have a thymoma. The thymus contains myoid cells that express the AChR (acetylcholine
receptor) antigen, antigen-presenting cells, and immunocompetent T cells. The thymus is believed to pro-
duce AChR subunits that act as autoantigens in the sensitization of the patient against the AChR. Most
thymomas in MG patients are benign and are amenable to easy surgical resection, which may in certain
cases improve symptomatology. The other tumor types mentioned previously are not typically associated
with MG.
Neurology
B&D Chapter 78

Question 79. B. Prevalence for schizophrenia is higher in urban than in rural areas, as are morbidity and severity of pre-
sentation. The lifetime prevalence is about 1%. The male to female ratio is 1:1. Onset is usually between
15 and 35 years, with onset before 10 years and after 40 years being rare. Despite this fact, never forget
that schizophrenia does have a bimodal distribution in women, with up to 10% of cases presenting after
age 40. There is a higher incidence of cases in babies born in the winter and early spring.
Psychotic Disorders
K&S Chapter 7

Question 80. A. Friedrich’s ataxia is an autosomal recessive disease that localizes to chromosome 9. It is the result of
an unstable expansion of a trinucleotide repeat (GAA). Onset is typically noted in adolescence, with
gait ataxia, loss of lower-extremity proprioception, and absence of deep tendon reflexes, more often in
the lower extremities. There is also predominant CNS involvement with noted dysarthria, presence of
Babinski’s sign, and eye movement anomalies. The natural progression of the disease is toward a com-
plete loss of ambulation ability and ultimately death due to hypertrophic cardiomyopathy in about 50%
of cases. Death occurs on average by late in the fourth decade.

Myotonic dystrophy (MD) type I is an autosomal dominantly inherited disease with trinucleotide repeat
expansion of CTG that codes to chromosome 19. The incidence is 1 in 8000 live births. Classic symp-
toms and signs include ptosis, bifacial weakness, frontal baldness, and triangular drooping facies. Motor
weakness is greater distally than proximally. A classic sign is myotonia, which is the inability to relax a
contracted muscle or group of muscles. For example, a patient is unable to let go after shaking hands.
Percussion myotonia can also be demonstrated, particularly in the thenar and hypothenar hand muscles.
This is involuntary contraction of muscles after percussion with a reflex hammer. Fibrotic or infiltra-
tive cardiomyopathy is a frequent associated problem in MD. DNA testing by serum polymerase chain
reaction is the diagnostic modality of choice for MD. Electromyography reveals myopathic features
and myotonic discharges (“dive-bomber” sound after muscular relaxation). Perifascicular muscle fiber
atrophy is the classic histopathologic finding on muscle biopsy. Please note that the other three answer
choices are explained in questions elsewhere in this volume and are all diseases inherited by autosomal
dominant transmission.
Neurology
B&D Chapters 72 and 79

112

Psychiatry Test Preparation and Review


Manual E-Book
Test Number Two

Question 81. D. Disinhibited behavior is more characteristic of mania than it is of depression. All of the other symptoms
are somatic or sleep complaints that are frequently associated with depression. Patients with depres-
sion have disrupted rapid eye movement (REM) sleep, including shortened REM latency, increased per-
centage of REM sleep, and a shift in REM distribution from the last half to the first half of the night.
Acetylcholine is associated with the production of REM sleep.
Depressive Disorders
K&S Chapter 8

Question 82. E. It is believed that migraine maps to several regions on chromosome 19 because of the entity FHM that
has been associated with this gene locus. FHM is thought to be a channelopathy involving a brain-spe-
cific calcium channel α1 subunit gene that has been mapped to chromosome 19. Given the similarity
between FHM and typical migraine, researchers believe that chromosome 19 may well be the locus that
links both disorders.
Neurology
B&D Chapter 69

Question 83. A. Obsessions are part of thought content, as are delusions, ideas of reference, phobias, suicidal or homi-
cidal thoughts, depersonalization, derealization, and neologisms, to name a few. Thought process would
include word salad, flight of ideas, circumstantiality, tangentiality, clang associations, perseveration, and
goal-directed ideas.
Diagnostic and Treatment Procedures in Psychiatry
K&S Chapter 5

Question 84. B. CNS cysticercosis involves parenchymal invasion of the brain by the larval-stage pork tapeworm T.
solium. Cysticercosis is caused by ingestion of undercooked pork containing cysticerci. The infection
occurs most frequently in Central and South America, where poor hygiene and sanitation lead to unsani-
tary conditions. After ingestion of tainted pork, the tapeworm eggs hatch in the gastric tract, develop to
larval stage, and eventually penetrate the bowel to migrate to host tissue, most often in the CNS. Clinical
presentation ranges from epilepsy, focal neurologic deficits, hydrocephalus, cognitive decline, meningitis,
or myelopathy. Diagnosis is confirmed by brain MRI demonstrating the parasites in the brain and from
confirmatory serology and/or CSF enzyme-linked immunosorbent assay (ELISA) tests. ELISA on CSF is
more sensitive and specific than serologic tests. Treatment is with albendazole or praziquantel oral ther-
apy coupled with parenteral steroids. Trichinella spiralis infection causes trichinosis, which is also caused
by ingestion of poorly cooked pork or other game meats in endemic areas. Echinococcus granulosus are
the larval tapeworms that cause CNS echinococcosis. Humans generally acquire echinococci through
contact with dogs that are infected. Echinococcus granulosus tends to cause solitary CNS spherical cysts
without edema. Treatment can involve surgical resection of the cyst or conservative medical management
with albendazole if the cysts are not resectable. Leishmania major is a protozoan that causes leishmani-
asis. The vector is a sandfly bite. Visceral disease can be accompanied by inflammatory neuropathies that
resemble Guillain–Barré syndrome. Treatment is by parenteral administration of antimony or amphoteri-
cin B. Toxoplasma gondii is the intracellular protozoan responsible for toxoplasmosis infection. Vectors
include migratory birds and cats, and so the infection can be found worldwide. Toxoplasmosis rarely
occurs in immunocompetent patients but is more often seen in AIDS patients with low CD4 counts.
CNS lesions present as ring-enhancing on both CT and MRI. CSF titers may contain T. gondii DNA
detectable by polymerase chain reaction. Treatment is usually a combined regimen of pyrimethamine
and sulfadiazine with folinic acid. Ventricular shunting may be required if lesions are space-occupying
and cause hydrocephalus.
Neurology
B&D Chapter 53

113

Psychiatry Test Preparation and Review


Manual E-Book
Psychiatry Test Preparation & Review Manual

Question 85. C. The effects of alcohol can be seen in several laboratory tests. The GGT will be elevated in 80% of alco-
holic patients, and mean corpuscular volume is increased in 60% of alcoholic patients. Uric acid, tri-
glycerides, aspartate transaminase, and alanine transaminase can also be elevated. The other laboratory
values given in the question are unrelated to alcohol abuse.
Laboratory Tests in Psychiatry
K&S Chapter 20

Question 86. C. Alcoholic neuropathy is the most common neurologic manifestation of chronic alcoholism. Up to 75%
of alcoholic patients are diagnosed with this disorder. Most patients are chronic alcohol abusers between
40 and 60 years of age. This type of neuropathy is a mixed motor and sensory disorder. Symptoms usu-
ally begin gradually, symmetrically in the feet.

Alcoholic dementia refers to older patients with a lifelong history of heavy alcohol use who experience
an insidious decline in their cognitive functioning. Nearly 20% of older alcoholics have some form of
dementia, but this is complicated by the presence of other comorbidities, such as liver abnormalities,
head trauma, malnutrition, and stroke. Alcoholic dementia results in more of a predominance of fine
motor control and verbal fluency deficits than is seen in patients with dementia of the Alzheimer type.

Marchiafava–Bignami disease is a rare demyelinating disease preferentially affecting the corpus callo-
sum in chronic alcoholics. The exact etiology of the disorder is not well understood. It may be related to
nutritional factors or direct toxic effects of alcohol on the cerebral white matter, but this is unclear. The
most common neurologic manifestation is that of frontal lobe damage and dementia noted on neuro-
logic examination. The central portion of the corpus callosum is more often affected than the anterior or
posterior portions. Treatment is directed at nutritional support and alcoholic rehabilitation.

Alcoholic-nutritional cerebellar degeneration is seen more often in men than in women. The disorder
presents in long-standing alcoholics as unsteadiness in walking evolving over weeks to months. The most
common presentation on examination is truncal ataxia, exhibiting a wide-based gait and difficulty with
tandem walking. Pathologically, the disorder results from preferential atrophy of the superior and ante-
rior cerebellar vermis, with lesser involvement of the cerebellar hemispheres. Wernicke’s encephalopathy
is explained extensively elsewhere in this volume.
Neurology
B&D Chapters 57 and 76

Question 87. B. A cohort study is when a group from a well-known population is chosen and followed over a long period of
time. These studies give us estimates of risk based on suspected causative factors for a given disease. Case his-
tory studies look back on people with a given disease. Cross-sectional studies give information on the preva-
lence of a disease in a population at a given point in time. Retrospective studies are based on past data, as
opposed to prospective studies, which are based on observing things as they occur.
Statistics
K&S Chapter 5

Question 88. C. The Miller–Fisher syndrome is a variant of Guillain–Barré syndrome (GBS; acute inflammatory demy-
elinating polyneuropathy). The classic triad of symptoms is gait ataxia, areflexia, and ophthalmoplegia.
This accounts for about 5% of GBS cases. Motor strength is usually intact. Serum IgG antibodies to
the ganglioside GQ1b can be detected in serum early in the course of the Miller–Fisher variant of GBS.
F-wave latencies may be intact on electromyography with reduced or absent sensory nerve action poten-
tial (SNAP) amplitudes. CSF protein is elevated without pleocytosis about 1 week into the course of
the illness. Dementia, parkinsonism, and psychosis make up the triad of dementia with Lewy bodies.
The clinical picture is that of dementia, EPS, fluctuations, and visual hallucinations. Visual hallucinations
occur in as many as 80% of cases. Neuroleptic sensitivity is another diagnostic characteristic. The clas-
sic pathologic finding is the Lewy bodies, which are diffuse throughout the cortex and are eosinophilic
inclusions with a core halo on hematoxylin and eosin stain. Treatment with newer atypical antipsychot-
ics, particularly clozapine, is usually best, and cholinesterase inhibitors, such as donepezil, may also be
helpful. The triad of gait ataxia, urinary incontinence, and dementia is classic for NPH.
Neurology
B&D Chapter 76

114

Psychiatry Test Preparation and Review


Manual E-Book
Test Number Two

Question 89. C. Konrad Lorenz is known best for his work with imprinting. Imprinting is the phenomenon whereby
during a critical period of early development a young animal will attach to its parent, or whatever sur-
rogate is in the parent’s place. From then on the presence of that parent or surrogate will elicit a specific
behavior pattern even when the animal is much older. In Lorenz’s case newborn goslings imprinted on
him instead of their mother and followed him around as if he were their mother. This has correlates in
psychiatry because it is evidence of the link between early experiences and later behaviors. Lorenz also
studied aggression in animals and worked on the need for aggression in humans given the pressures of
natural selection. All the other answer choices in this question are distractors that have nothing to do
with the work of Lorenz.
Human Development
K&S Chapter 3

Question 90. B. This question depicts a classic case of acute inflammatory demyelinating polyneuropathy (or GBS). About
65% of patients report a prior “insult” before the onset of symptoms. This often takes the form of a gas-
trointestinal infection, an upper respiratory infection, surgery, or immunization a few weeks before symp-
toms appear. The most commonly identified organism responsible for prodromal infection is C. jejuni.
Campylobacter jejuni can be detected in stool cultures and serologic studies. The clinical presentation of GBS
can vary from case to case. Typically patients present with symmetrical lower-extremity weakness, with par-
esthesias, and possibly with sensory symptoms. The paralysis is usually an ascending paralysis, and the most
worrisome outcome is paralysis of the muscles of respiration, which can prove fatal in some cases. Deep ten-
don reflexes are usually greatly diminished or absent. Diagnostic testing reveals the classic nerve conduction
abnormalities of conduction block and prolonged F-wave latencies, which are pathognomonic of GBS. CSF
studies reveal cytoalbuminologic dissociation, or elevated protein with an acellular fluid. In certain cases,
nerve biopsy can reveal complement fixing antibodies to peripheral nerve myelin. Treatment should take place
in the hospital and in an intensive care unit setting if respiratory compromise is imminent. Two treatments of
choice are high-dose intravenous immunoglobulin administration and plasmapheresis.
Neurology
B&D Chapter 76

Question 91. A. Second messengers are molecules that work within the cell to carry on the message delivered by the
neurotransmitter on the cell surface. IP3, cGMP, Ca2+, cAMP, DAG (diacylglycerol), NO, and CO are
all common second-messenger molecules. Adenylyl cyclase is not a second messenger itself but rather is
the enzyme that makes cAMP from ATP. Adenylyl cyclase is turned on or off by G proteins, depending
on the need for cAMP. Binding cAMP to transcription factors regulates gene transcription, including
the machinery to make certain neurotransmitters. Calcium plays a number of roles within the cell, and
excess Ca2+ is linked to production of NO and cell death through excitotoxicity. One of the major func-
tions of IP3 is to cause the release of intracellular Ca2+ from the endoplasmic reticulum.
Basic Neuroscience
K&S Chapter 1

Question 92. A. The cortical dementias, such as Alzheimer’s disease, generally produce a gradual decline in cognitive
function with normal cognition speed and the presence of aphasia, dyspraxia, and agnosia. Depression is
less common in cortical dementia than in subcortical disease. Motor abnormalities are typically absent
in cortical dementia, unless the disease is in the terminal stages. Subcortical dementia, as exemplified in
Parkinson’s disease, typically presents with dysarthria and extrapyramidal motor abnormalities. Apathy
and depression are often present. Frontal memory impairment with recall aided by cues is often noted.
Speed of cognition in subcortical dementia is slow.
Neurocognitive Disorders
B&D Chapter 95

Question 93. B. This question focuses on Piaget’s stages of cognitive development. In the sensorimotor stage, children
respond to stimuli in the environment and learn during this process. They eventually develop object per-
manence (objects exist independent of the child’s awareness of their existence) and begin to understand
symbols (as in the use of words to express thoughts).

115

Psychiatry Test Preparation and Review


Manual E-Book
Psychiatry Test Preparation & Review Manual

During the preoperational stage, there is a sense that punishment for bad deeds is unavoidable (imma-
nent justice). There is a sense of egocentrism and phenomenalistic causality (the thought that events that
occur together cause one another). Phenomenalistic causality is the subject of this question. Animistic
thinking (inanimate objects are given thoughts and feelings) is also seen.

In the concrete operations stage, egocentric thought changes to operational thought, wherein another’s
point of view can be taken into consideration. In concrete operations children can put things in order
and group objects according to common characteristics. They develop the understanding of conservation
(a tall cup and a wide cup can both hold equal volumes of water) and reversibility (ice can change to
water and back to ice again).

During formal operations children can think abstractly, reason deductively, and define abstract concepts.
Latency is a distractor. It occurs in Freud’s model after the genital phase and before puberty.
Human Development
K&S Chapter 2

Question 94. C. T4 is the dermatome at the level of the nipples. T10 is the dermatome at the level of the umbilicus.
Other important dermatomes to remember include C2, back of head; C4, above collar bone; C6, thumb;
C7, middle fingers; C8, little finger; L1, groin; L2, lateral thigh; L3, medial thigh; L4, medial leg; L5, lat-
eral leg, big toe; S1, little toe, sole of foot; S5, perianal area.
Neurology
B&D Chapter 24

Question 95. D. The three receptor types associated with glutamate are AMPA, kainate, and NMDA. Acetylcholine is
associated with the nicotinic and muscarinic receptors. NE is associated with the α1, α2, and β recep-
tors. Serotonin is associated with the various 5-HT receptors. GABA is associated with the GABA recep-
tor. Opioids are associated with the μ and δ receptors. Dopamine is associated with the D1, D2, etc.,
receptors.
Basic Neuroscience
K&S Chapter 1

Question 96. B. Buspirone is a serotonin 1A partial agonist. It is indicated for the treatment of anxiety disorders, in
particular generalized anxiety disorder. It does have activity at the serotonin 2 and dopamine 2 receptor
sites, but its significance at those sites is not well understood. It may have mild dopamine 2 agonistic and
antagonistic effects, but this is not its predominant mechanism of action. Buspirone takes 2 to 3 weeks
to exert therapeutic effects. The initial dose is 15 mg daily in two or three divided doses. Therapeutic
effects are usually not seen until a dose of 30 mg or above is reached. The maximum approved daily
dose is 60 mg. Buspirone can increase blood levels of haloperidol. Buspirone cannot be used with MAO
inhibitors (MAOIs), and a 2-week washout needs to happen after an MAOI is stopped before buspirone
can be started. Buspirone levels can be increased by nefazodone, erythromycin, itraconazole, and grape-
fruit juice by their inhibition of CYP 3A4 in the liver.
Psychopharmacology
K&S Chapter 29

Question 97. E. Excitatory neurotransmitters open cation channels that depolarize the cell membrane and increase
the likelihood of generating an action potential. These neurotransmitters elicit excitatory postsynaptic
potentials.
Basic Neuroscience
K&S Chapter 1

Question 98. C. PCP is a potent NMDA receptor antagonist. The NMDA receptor is a subtype of the glutamate receptor.
PCP has calcium channel-binding properties and prevents the influx of calcium into neurons. PCP also
has dopaminergic properties that would seem to explain the reinforcing effects of the drug.

116

Psychiatry Test Preparation and Review


Manual E-Book
Test Number Two

Tolerance can occur with PCP, but it is generally held that PCP does not cause a physical dependence.
There is psychological dependence to the agent, and users can become dependent on its euphoric effects
in this way. PCP intoxication is characterized by maladaptive behavioral changes, such as violence,
impulsivity, belligerence, agitation, and impaired judgment. Symptoms of intoxication include nystag-
mus, hypertension, decreased pain responsiveness, ataxia, dysarthria, muscle rigidity, seizures, coma, and
hyperacusis. PCP can induce a delirium with agitated, bizarre, or violent behavior. It can also cause an
acute psychotic disorder with delusions and hallucinations that can persist for several weeks after inges-
tion of the drug. PCP can remain in the blood and urine for more than a week. Treatment of the behav-
ioral abnormalities related to PCP is best undertaken with benzodiazepines and dopamine antagonists.
Substance Abuse and Addictive Disorders
K&S Chapter 20

Question 99. C. Sublimation is one of the mature defenses. It is characterized by obtaining gratification through trans-
forming a socially unacceptable aim or object into an acceptable one. Sublimation allows instincts to
be channeled in an acceptable direction rather than blocked. All other choices are immature defenses.
Hypochondriasis is exemplified by exaggerating an illness for the purpose of evasion or regression.
Introjection is demonstrated by internalizing the qualities of an outside object. It is an important part of
development but can also be used as an unproductive defense. The classic example is identification with
an aggressor. Regression is characterized by attempting to return to an earlier libidinal phase of func-
tioning to avoid the tension and conflict of the current level of development. Passive aggression takes
the form of expressing anger toward others through passivity, masochism, and turning against the self.
Manifestations can include failure and procrastination.
Psychological Theory and Psychometric Testing
K&S Chapter 5

Question 100. E. Phenelzine is an MAOI and is not likely to worsen the movement disorder symptoms of Parkinson’s dis-
ease. The other four agents are antagonists of dopamine D2 receptors and can of course worsen symp-
toms of Parkinson’s disease and cause drug-induced parkinsonism. The pathophysiology involves D2
receptor antagonism in the caudate. Patients who are elderly and female are at greatest risk for neurolep-
tic-induced parkinsonism. More than half of patients exposed to neuroleptics on a long-term basis have
been noted to develop this unwanted adverse effect. Amoxapine (Asendin) is a dibenzoxazepine tetracy-
clic antidepressant that has strong D2-antagonistic properties because it is a chemical derivative of the
neuroleptic loxapine (Loxitane). Because of its unique structure and chemical properties, amoxapine can
also cause akathisia, dyskinesia, and, infrequently, neuroleptic malignant syndrome.
Psychopharmacology
K&S Chapter 29

Question 101. D. Anna Freud wrote The Ego and Mechanisms of Defense and was the first to give a comprehensive
study of the defense mechanisms. She maintained that all people, healthy or neurotic, use a number
of defense mechanisms. Sigmund Freud was the father of psychoanalysis whose work focused on the
importance of libido, aggression, and the Oedipal complex, among other concepts. Kohut was the father
of self-psychology. Fromm defined five character types common to Western culture. Jung went beyond
Freud’s work and founded the school of analytic psychology. It focused on the growth of the personality
through one’s experiences.
Psychological Theory and Psychometric Testing
K&S Chapter 4

Question 102. B. In the case of Dusky v United States, the U.S. Supreme Court determined that to have the competence
to stand trial, a criminal defendant must be able to have the ability to consult his or her lawyer with a
reasonable degree of rational understanding, and he or she must have a reasonable and rational under-
standing of the proceedings against him or her. The McGarry instrument is a clinical guide that identifies
13 areas of functioning that must be demonstrated by a criminal defendant to be declared competent to
stand trial. Answer choices A, C, D, and E are included in these 13 areas, as well as the ability to plan

117

Psychiatry Test Preparation and Review


Manual E-Book
Psychiatry Test Preparation & Review Manual

legal strategy, ability to appraise the roles of participants in courtroom procedure, capacity to challenge
prosecution witnesses realistically, capacity to testify relevantly, ability to appraise the likely outcome,
and understanding the possible penalties, among several others.
Forensic Psychiatry
K&S Chapter 36

Question 103. D. Schizoaffective disorder in DSM 5 requires that symptoms that meet criteria for a major mood epi-
sode be present for the majority of the total duration of the active and residual portions of the illness.
Schizoaffective may be specified “with catatonia” but this is not necessary to make the diagnosis. To
meet criteria for schizoaffective there needs to be a major mood episode concurrent with criteria for
schizophrenia. The diagnosis cannot be attributable to the effects of a substance or another medical con-
dition. Delusions or hallucinations must be present for 2 weeks or more in the absence of a major mood
episode during the lifetime duration of the illness.
Psychotic Disorders
K&S Chapter 7

Question 104. E. Studies have indicated that depression tends to be a chronic, relapsing disorder. Lifetime prevalence of
major depressive disorder is 5% to 17%. The percentage of patients who recover after repeated episodes
diminishes over time. About one-quarter of patients have a recurrence within the first 6 months after ini-
tial treatment. This figure rises to about 30% to 50% in the first 2 years, and even higher to about 50%
to 75% within 5 years. It has been proven that ongoing antidepressant prophylaxis helps lower relapse
rates. As a patient experiences more depressive episodes over time, the time between episodes decreases,
and the severity of the episodes worsens.
Depressive Disorders
K&S Chapter 8

Question 105. D. Dysthymic disorder is characterized by decreased mood over a period of 2 years with poor appetite or
overeating, sleep problems, fatigue, low self-esteem, poor concentration, and feelings of hopelessness.
Hallucinations are not considered part of dysthymia, although it is possible to have hallucinations as
part of a major depressive episode. Lifetime prevalence of dysthymic disorder is 3% to 6%.
Depressive Disorders
K&S Chapter 8

Question 106. D. Tarasoff v Regents of the University of California is the landmark case from 1976 in which the
California Supreme Court ruled that any psychotherapist who believes that a patient could injure or kill
someone must notify the potential victim, the victim’s relatives or friends, or the authorities. In 1982 the
same court issued a second ruling that broadened Tarasoff to include the duty to protect, not only to
warn, the intended victim.

The Durham Rule was determined by the ruling in the case of Durham v the United States in 1954 by
Judge Bazelon. This rule stipulates that a defendant cannot be found criminally responsible if the crimi-
nal act was the product of a mental illness or defect. In 1972 the District of Columbia Court of Appeals,
in the ruling United States v Brawner, discarded the Durham Rule.

In 1976 in the ruling of O’Connor v Donaldson, the U.S. Supreme Court ruled that harmless mentally ill
patients cannot be confined involuntarily without treatment if they can survive outside an institution.

Clites v State of Iowa was a landmark case pertaining to a ruling in favor of a patient and his family,
who sued for damages resulting from chronic neuroleptic exposure that resulted in tardive dyskinesia.
The appellate court ruled that the defendants deviated from the usual standards of care by failing to
conduct physical examinations and routine laboratory tests and failed to intervene at the first signs of
tardive dyskinesia.
Forensic Psychiatry
K&S Chapter 36

118

Psychiatry Test Preparation and Review


Manual E-Book
Test Number Two

Question 107. A. Mature defense mechanisms are considered healthy and adaptive throughout the life cycle. Mature
defenses include sublimation, altruism, anticipation, humor, and suppression. Answer choices B–E repre-
sent immature defenses often found during preadolescent years and in adult character disorders.
Personality Disorders
K&S Chapter 4

Question 108. B. Systemic lupus erythematosus is an autoimmune disorder. The vast majority of cases are women.
Symptoms can span multiple organ systems and include fever, arthritis, fatigue, muscle pain, weight
loss, rash, renal damage, pericarditis, pleuritis, photosensitivity, anemia, and leukopenia. Up to 90% of
patients have neuropsychiatric manifestations, the most common of which are cognitive dysfunction and
depression. When steroids are given, most symptoms resolve, with the exception of cognitive dysfunc-
tion, which can be difficult to treat. Steroids can also precipitate mania and psychosis in some patients.
The most common neuropsychiatric effect of steroids is hypomania/mania. What an important fact for
the prudent student to know!
Psychosomatics
K&S Ch. 21

Question 109. B. Fiduciary duty is the obligation to work in the patient’s best interests. Beneficence is an obligation to
help patients and relieve suffering. Nonmaleficence is the duty to do no harm. Altruism is putting the
needs of others before your own needs. Parens patriae is a doctrine that allows the state to intervene and
act as a surrogate parent for those who cannot take care of themselves.
Ethics
K&S Chapter 36

Question 110. C. Right–left disorientation is a result of a lesion to the dominant angular gyrus and is one of the symptoms
of Gerstmann’s syndrome. A large stroke in the right MCA territory can result in hemineglect, visual and
tactile extinction, impaired speech prosody (loss of musical and emotional inflection), anosognosia (not
knowing that you have a deficit or problem), and behavioral problems, such as delirium and confusion.
The patient may not recognize that the affected left arm and/or hand is his or her own and may have
limb apraxia. A contralateral homonymous hemianopia or inferior quadrantanopia can also be noted in
nondominant hemispheric strokes.
Neurology
B&D Chapter 51

Question 111. B. The practice described in the question is called “fee splitting” and is considered unethical. As a physician
you cannot receive financial compensation for referring patients to other doctors, nor can you pay for
such referrals. Such an arrangement puts the doctor’s interests (the financial incentive to refer) over the
best interests of the patient and leads to inappropriate referrals.
Ethics
K&S Chapter 36

Question 112. A. This question depicts a classic diabetic third nerve palsy. The differential diagnosis of greatest impor-
tance with an isolated third nerve palsy is diabetes (benign) versus that of an internal carotid artery
(ICA) aneurysm (potentially fatal). Aneurysms that most commonly affect third nerve functioning are
those originating at the ICA near the origin of the posterior communicating artery. A third nerve palsy
resulting from an aneurysm is usually associated with a dilated pupil and eyelid ptosis, as contrasted
with a diabetic third nerve palsy, which usually spares the pupillary function.
Neurology
B&D Chapter 51

119

Psychiatry Test Preparation and Review


Manual E-Book
Psychiatry Test Preparation & Review Manual

Question 113. A. Interpersonal psychotherapy was originally developed as a treatment for depression. It assumes that
some psychiatric illnesses occur in a social and interpersonal context and that their onset, response to
treatment, and outcomes are influenced by interpersonal relationships between the patient and others in
his or her life. Interpersonal psychotherapy usually lasts for up to 20 sessions over a 5-month period and
has three specific phases. In the initial phase the therapist reviews the patient’s illness, explains the ratio-
nale for interpersonal therapy, and conducts an interpersonal inventory. Symptoms are linked to one of
four interpersonal problem areas, including grief, interpersonal deficits, interpersonal role disputes, and
role transitions. Next is the intermediate phase, which often covers 10 sessions. This is where the work
of the therapy takes place. The therapist tries to link changes the patient makes in his or her interper-
sonal life with improvements the patient is seeing in his or her psychiatric symptoms. Last is the termi-
nation phase, which is the final five sessions. Termination is discussed, and the patient is encouraged to
review gains, work out plans for future improvements, and develop action plans should symptoms recur.
Cognitive distortions would be more associated with cognitive behavioral therapy than with interper-
sonal psychotherapy. A central part of the theory behind cognitive behavioral therapy is an emphasis on
the psychological significance of people’s beliefs about themselves, their world, and their future.
Psychotherapy
K&S Chapter 28

Question 114. E. Severe motor tics in Tourette’s syndrome are best treated by neuroleptics, in particular haloperidol and
pimozide. Recently the atypical neuroleptics have come to be used more readily because of their superior
safety profiles, in particular risperidone, quetiapine, olanzapine, ziprasidone, and clozapine. Fluphenazine,
molindone, and other conventional antipsychotics are also acceptable treatment choices. Clonidine is also
a frequently used and effective treatment of tics and is particularly favored by pediatric neurologists for its
excellent safety profile. Botulinum toxin type A can be effective for blepharospasm and eyelid motor tics
and is in fact FDA-approved for this indication. Protriptyline and the other antidepressants may be effec-
tive for associated obsessive–compulsive symptoms, but these agents are not useful for treatment of tics.
Disruptive, Impulse Control, Conduct Disorders, and ADHD
B&D Chapter 71

Question 115. E. Hoarding disorder is characterized by persistent difficulty discarding and parting with possessions
regardless of their actual value. These patients have a perceived need to save items and experience stress
associated with discarding them. This results in the accumulation of possessions that clutter active liv-
ing areas and substantially compromise their intended use. If the living area is not cluttered, it is only
because of the intervention of other people. It can be specified as “with excessive acquisition,” in which
the patient excessively acquires new items for which there is no space. It can also be categorized based
on level of insight. OCD consists of obsessions (recurrent intrusive negative thoughts or urges that
cause marked anxiety or distress) that lead to compulsions (repetitive behaviors or mental acts aimed
at preventing or reducing the anxiety or distress). Lifetime prevalence of OCD is 2% to 3%. Obsessive–
compulsive personality disorder consists of a pervasive pattern of preoccupation with orderliness, per-
fectionism, and mental and interpersonal control at the expense of flexibility, openness, and efficiency.
Shubo-kyofu is a Japanese cultural syndrome that is similar to body dysmorphic disorder and is charac-
terized by excessive fear of having a bodily deformity. Amphetamine use can lead to myocardial infarc-
tion, severe hypertension, cerebrovascular disease, and ischemic colitis. Psychological effects include
restlessness, dysphoria, insomnia, irritability, hostility, and confusion. Amphetamines can also induce
symptoms of anxiety disorders.
Obsessive–Compulsive and Related Disorders
K&S Chapter 10

Question 116. C. The MCA territory includes the optic radiations. A lesion of these tracts results in either a contralateral
homonymous hemianopsia or a contralateral inferior quadrantanopia (“pie on the floor”). Bitemporal
hemianopsia is of course caused by invasion of the optic chiasm, often by a sellar lesion, such as a pituitary
macroadenoma. Left monocular blindness would result from an ipsilateral central retinal artery occlusion.
Neurology
B&D Chapter 51

120

Psychiatry Test Preparation and Review


Manual E-Book
Test Number Two

Question 117. E. The process by which a neurotransmitter is released into the synaptic cleft is called exocytosis.
Neurotransmitters are synthesized in the presynaptic neuron, and both their synthesis and their release
are mediated by Ca2+ influx into the cell. Feedback receptors exist on the presynaptic membranes of
many cells, a good example being the α2 receptor on the noradrenergic neuron, which participates in a
negative feedback loop to stop the release of NE. Neurotransmitters, such as dopamine, NE, and sero-
tonin, will remain active until they diffuse out of the cleft or are removed by reuptake mechanisms.
Degradation of recycled neurotransmitters is done via MAOs, with MAO-A degrading NE and serotonin
and MAO-B degrading dopamine.
Basic Neuroscience
K&S Chapter 1

Question 118. B. Lesch–Nyhan syndrome is an X-linked recessive disorder. Hyperuricemia results from a deficit in hypo-
xanthine–guanine phosphoribosyltransferase. Clinical hallmarks of the disorder include cognitive
impairment, self-mutilatory behavior, hypotonia, hyperreflexia, dysarthria, developmental delay, and
choreoathetotic movements. Low dopamine concentrations in the basal ganglia and CSF may be a cause
of the movement disorder noted in Lesch–Nyhan syndrome.

Ornithine transcarbamylase deficiency is an X-linked inborn error of metabolism of the urea cycle that
causes hyperammonemia, encephalopathy, and respiratory alkalosis. Treatment with low-protein diet
and dietary arginine supplementation can be effective.

Carbamoyl phosphate synthase deficiency is an autosomal recessive inborn error of metabolism of the
urea cycle that causes hyperammonemia, encephalopathy, and respiratory alkalosis. There are two forms
of the disorder; type I is an early fatal form, and type II is a delayed-onset form, which may present only
later in childhood or early adulthood. Treatment with dietary arginine repletion may be helpful in cer-
tain cases.

Arginase deficiency results in hyperammonemia as well. The disorder is usually caused by a point muta-
tion or deletion on chromosome 6q23 in the gene that codes for ARG1. The incidence of the disorder is
less than 1 in 100,000.

Adenylosuccinase deficiency results in autism, growth retardation, and psychomotor delay. It is believed
to be an autosomal recessive cause of autism. Seizures may also arise as a result of this disorder.
Neurology
B&D Chapter 62

Question 119. D. This question focuses on Piaget’s stages of cognitive development. In the sensorimotor stage, children
respond to stimuli in the environment and learn during this process. They eventually develop object per-
manence (objects exist independent of the child’s awareness of their existence), and begin to understand
symbols (as in the use of words to express thoughts).

During the preoperational stage, there is a sense that punishment for bad deeds is unavoidable (imma-
nent justice). There is a sense of egocentrism and phenomenalistic causality (the thought that events that
occur together cause one another). Animistic thinking (inanimate objects are given thoughts and feel-
ings) is also seen.

In the concrete operations stage, egocentric thought changes to operational thought whereby another’s
point of view can be taken into consideration. In concrete operations children can put things in order
and group objects according to common characteristics. They develop the understanding of conservation
(a tall cup and a wide cup can both hold equal volumes of water) and reversibility (ice can change to
water and back to ice again.)

During formal operations children can think abstractly, reason deductively, and define abstract concepts.
Symbiosis is one of Mahler’s stages of separation–individuation. It is unrelated to Piaget.
Human Development
K&S Chapter 2

121

Psychiatry Test Preparation and Review


Manual E-Book
Psychiatry Test Preparation & Review Manual

Question 120. D. Posttraumatic dementia most often results in frontotemporal cognitive dysfunction from diffuse sub-
cortical axonal shear that disrupts cortical and subcortical circuitry. Slowing of mental processing and
difficulty with executive functioning, set-shifting, organization, and planning are the most frequent cog-
nitive deficits noted. Dementia pugilistica can occur in patients with repeated head trauma and is not
limited simply to boxers. Clinical features include severe memory and attentional deficits and extrapyra-
midal signs. Carriers of apo E4 (which resides on chromosome 19) are at increased risk of posttraumatic
dementia as well as Alzheimer’s disease.
Neurology
B&D Chapter 68

Question 121. E. Doxepin is the tricyclic with the most antihistaminic activity.
Psychopharmacology
K&S Chapter 29

Question 122. A. Bupropion is neither indicated nor effective in the treatment of OCD. The SSRIs have demonstrated
proven efficacy in treating OCD. Trials of 4 to 6 weeks’ duration are usually needed to produce results,
but often trials need to be extended to 8 to 16 weeks to achieve maximal therapeutic benefit. The stan-
dard of care is to start either an SSRI or clomipramine, which usually generates a response in 50% to
70% of cases. Four of the SSRIs have been FDA-approved for OCD: fluoxetine, fluvoxamine, parox-
etine, and sertraline. Higher doses may be needed to achieve a response. Clomipramine (Anafranil) is
the most serotonergic of the tricyclic and tetracyclic antidepressant agents, which are more noradren-
ergic in their nature. Clomipramine was the first agent specifically FDA-approved for OCD. It needs to
be titrated upward gradually over several weeks to minimize gastrointestinal and anticholinergic side
effects. The best results are achieved with a combination of drug and cognitive behavioral therapy.
Psychopharmacology
K&S Chapter 29

Question 123. E. Collaborative care is a systemic, evidence-based approach in which psychiatrists and primary care physi-
cians work together to deliver care for common mental health conditions in a primary care setting. By
working with primary care providers, the psychiatrist is able to affect a far larger patient population
than if they were working alone. Population-based care focuses on health promotion, disease prevention,
service access, and epidemiologic surveillance. Illness management is a framework to help patients col-
laborate with their providers, reduce relapses, and cope with symptoms. It aims to improve self-efficacy
and self-esteem. Tertiary prevention aims to reduce the effects of a disorder through rehabilitation and
chronic care management. (You will come across primary and secondary prevention elsewhere in this
book.) Universal interventions are medical interventions intended for the general public, like immuniza-
tions, public education, and resources for health promotion.
Public Policy
K&S Chapter 35

Question 124. E. Enuresis should first be treated with the bell-and-pad behavioral conditioning method before pharmaco-
therapy is instituted. The principle is simple: a bell awakens the child when the mattress becomes wet.
Tricyclic antidepressants, such as amitriptyline and imipramine, can reduce the frequency of enuresis in
about 60% of patients. Desmopressin (or DDAVP) is effective in about half of patients. Tricyclic antide-
pressants are to be given about 1 hour before bedtime. The response to therapy can be as rapid as a few
days. Desmopressin is administered intranasally in doses of 10 to 40 mg daily. Children who respond com-
pletely to any of these pharmacological agents should continue the therapy for several months to prevent
relapse. Olanzapine and the antipsychotic medications are not generally used in the treatment of enuresis.
Elimination Disorders
K&S Chapter 31

Question 125. D. This question stem is referring to Ca2+. During an action potential, the first ion channel to open is
the Na+ channel. This lets Na+ flow into the neuron. Next Ca2+ channels open, allowing more posi-
tively charged ions to enter and contribute to the action potential. Once inside, Ca2+ ions act as second

122

Psychiatry Test Preparation and Review


Manual E-Book
Test Number Two

messengers involved in protein–protein interactions and gene regulation. Calcium ions are critical to the
release of neurotransmitter and also activate the opening of potassium ion channels that then put a stop
to the action potential through the afterhyperpolarization of the membrane. With regard to receptors,
also keep in mind that the GABA receptor is a chloride ion channel.
Basic Neuroscience
K&S Chapter 1

Question 126. B. The most worrisome side effects of the tricyclic antidepressant agents are cardiac conduction abnor-
malities, because of course these side effects can lead to fatal cardiac arrhythmias if the medication is
taken in overdose. These agents can cause flattened T waves, tachycardia, prolonged QT intervals, and
depressed ST segments on ECGs. The tricyclics are also noted for causing orthostatic hypotension by
α1 adrenergic blockade, and they can of course cause sedation and the lowering of the seizure thresh-
old. The tricyclics can also cause anticholinergic side effects, which consist of dry mouth, constipation,
blurred vision, and urinary retention. In the male patient suffering from benign prostatic hypertrophy,
the anticholinergic load can lead to severe urinary retention and even anuria, which can be very prob-
lematic. Bethanechol 25 to 50 mg three to four times daily can reduce urinary hesitancy and retention.
Tricyclic antidepressants can actually help with migraine and neuropathic pain prophylaxis, particularly
at lower dosages. These are off-label uses for these agents.
Psychopharmacology
K&S Chapter 29

Question 127. D. Inhibitory neurotransmitters open chloride channels that hyperpolarize the membrane and decrease the
likelihood of an action potential being generated. They cause inhibitory postsynaptic potentials.
Basic Neuroscience
K&S Chapter 1

Question 128. C. This question addresses various types of behavior therapy for a simple phobia of spiders (arachnopho-
bia). Flooding (implosion) involves exposing the patient to the feared stimulus in vivo immediately, with-
out a gradual buildup, as would be expected in therapeutic graded exposure. The goal of flooding is
immediate exposure and response prevention until the patient can tolerate the anxiety and gain a sense
of mastery of the anxiety. Therapeutic graded exposure is similar to systematic desensitization, except
that relaxation techniques are not involved, and the technique is carried out in real situations. Progress is
graded, and at first the patient may be exposed to pictures of the spiders only, then later exposed to real
spiders themselves. Participant modeling involves the patient learning a new behavior by observation
first, then eventually by doing it, often accompanied by the therapist. This model can work particularly
well with agoraphobia. Aversion therapy is the use of a noxious stimulus or punishment to suppress an
undesired behavior. Using a bad-tasting nail polish to prevent nail-biting, or taking disulfiram to prevent
alcohol use would be excellent examples of aversion therapy. Assertiveness training teaches the patient
to develop social and interpersonal skills through responding appropriately in social or occupational
situations, expressing opinions acceptably, and achieving personal goals. Techniques such as role model-
ing, positive reinforcement, and desensitization can be used to develop assertiveness.
Psychotherapy
K&S Chapter 28

Question 129. C. Disclaimers such as “I know you don’t agree with this, but…” or “I don’t know why I think this, but…”
often precede emotionally charged material. They serve to soften the delivery of such material and
relieve anxiety on the part of the person making them. Whenever a disclaimer is made by the patient
in therapy, the therapist should listen closely to the material that follows because that material is often
a window into how the patient truly feels. The other answer choices in this question border on the
ridiculous.
Psychotherapy
K&S Chapter 28

123

Psychiatry Test Preparation and Review


Manual E-Book
Psychiatry Test Preparation & Review Manual

Question 130. D. This question depicts a patient with an acute delirium. The mainstay of treatment of delirium is to treat
the underlying cause if it is identifiable. The other goal of treatment is to give the patient environmental,
sensory, and physical support. Environmental support can be given by increasing the lighting level in the
room, providing a clock and calendar in the room showing the correct time and date, and dimming the
lights and closing the blinds only at night. Physical support can be provided to patients by the presence
of friends and relatives in the room or a constant caregiver figure. This can help prevent falls and other
physical accidents. Low-dose haloperidol in either oral or intramuscular form can be helpful to treat
psychotic symptoms or agitation. Benzodiazepines, particularly those of high potency and short duration
of action like flurazepam (Dalmane), should be avoided, because they can paradoxically agitate and fur-
ther confuse the delirious patient.
Neurocognitive Disorders
K&S Chapter 21

Question 131. D. Abreaction is an emotional release after recalling a painful event. It is part of psychodynamic therapy
and is a large part of what Freud thought brought about cure during psychoanalysis of conversion dis-
order patients during his early work. The other answer choices are part of cognitive behavioral therapy
(CBT). In CBT the false belief systems that underlie maladaptive behaviors and mood disturbance are
examined through techniques such as those listed in the question. Underlying assumptions that feed false
belief systems are also examined. The goal of therapy is to correct the underlying thoughts and assump-
tions and, in so doing, change the mood and behavior.
Psychotherapy
K&S Chapter 28

Question 132. A. Pinpoint pupils are a feature of opioid intoxication or usage and not withdrawal. Opioid withdrawal
consists of dysphoric mood, nausea or vomiting, muscle aches or twitches, lacrimation or rhinorrhea,
papillary dilatation, piloerection, sweating, diarrhea, yawning, fever, and insomnia. Associated symptoms
can include hypertension, tachycardia, and temperature dysregulation, such as hypothermia or hyper-
thermia. Other possible symptoms can include depression, irritability, weakness, and tremor.
Substance Abuse and Addictive Disorders
K&S Chapter 20

Question 133. A. The least sedating tricyclic antidepressants are desipramine and protriptyline. Moderately sedating tricy-
clics include imipramine, amoxapine, nortriptyline, and maprotiline. The most sedating include amitrip-
tyline, trimipramine, and doxepin.
Psychopharmacology
K&S Chapter 29

Question 134. B. Secondary gain refers to tangible advantages that a patient may get as a result of being unwell. Examples
of secondary gain include getting out of responsibility; getting money, drugs, or other financial entitle-
ments; or controlling other people’s behavior. Getting or seeking medical help, or enjoying playing the
sick role for its own sake, would be considered a form of primary gain.
Somatic Symptom Disorders
K&S Chapter 13

Question 135. C. In hypothyroidism, one would expect to find an increased TSH and a low free T4 level. Subclinical
cases of hypothyroidism will present with a high TSH and a normal T4 level. Other answer choices
are distractors and are not the most important laboratory values to look for when trying to diagnose
hypothyroidism.
Laboratory Tests in Psychiatry
K&S Chapter 5

Question 136. E. There are no absolute contraindications to ECT. Pregnancy is not a contraindication for ECT. Fetal
monitoring is considered important only if the pregnancy is high risk or complicated. Brain tumors
increase the risk of ECT, especially of brain edema and herniation after ECT. If the tumor is small,

124

Psychiatry Test Preparation and Review


Manual E-Book
Test Number Two

complications can be minimized by administration of dexamethasone before ECT and close monitoring
of blood pressure during the treatment. Patients with aneurysms, vascular malformations, or increased
intracranial pressure are at greater risk during ECT because of increased blood flow during the induc-
tion of the seizure. This risk can be decreased by careful control of blood pressure during the seizure.
Epilepsy and prior neuroleptic malignant syndrome are not problematic with the administration of ECT.
Recent myocardial infarction is another risk factor, but the risk decreases markedly 2 weeks after the
infarction and even further 3 months after the infarction. Hypertension, if controlled and stabilized with
antihypertensive medication, does not pose an increased risk during ECT.
Diagnostic and Treatment Procedures in Psychiatry
K&S Chapter 29

Question 137. E. Carbamazepine has many drug–drug interactions because of its effects on the cytochrome P450 sys-
tem. Of importance is the fact that it lowers concentrations of oral contraceptives, leading to break-
through bleeding and uncertain protection against pregnancy. It has a long list of other drugs with which
it interacts, including lithium, some antipsychotics, and certain cardiac medications. It should not be
combined with MAOIs. Combining clozapine with carbamazepine will increase the risk of bone marrow
suppression.
Psychopharmacology
K&S Chapter 29

Question 138. B. Alfred Binet developed the idea of mental age in 1905 as the average intellectual level of a particular age.
IQ is simply the quotient of mental age divided by chronological age multiplied by 100. An IQ of 100 is
therefore considered to be the average, that is, when mental age and chronological age are equal. An IQ
of 100 represents the 50th percentile in intellectual ability for the general population.
Psychological Theory and Psychometric Testing
K&S Chapter 5

Question 139. B. Open-ended questions are those that allow the patient to express what he or she is thinking and do not
direct the patient to speak about a specific topic that the doctor chooses. Ideally an interview should
begin with open-ended questions, and the questions become more specific and closed-ended as the inter-
view continues. Closed-ended questions can often be answered in one or few words. Open-ended ques-
tions often require the patient to explain and provide information.
Psychotherapy
K&S Chapter 5

Question 140. A. The Karolinska Institute has shown in numerous studies that diminished central serotonin plays a role in
suicidal behavior. This group was the first to demonstrate that low levels of CSF 5-HIAA are associated
with suicidal behavior. It has also been shown that low 5-HIAA levels predict future suicidal behavior
and that low 5-HIAA levels have been shown in the CSF of adolescents who kill themselves.
Depressive Disorders
K&S Chapter 8

Question 141. E. Symptoms of lithium toxicity include ataxia, tremor, nausea, vomiting, nephrotoxicity, muscle weakness,
convulsions, coma, lethargy, confusion, hyperreflexia, and nystagmus. Nonspecific T-wave changes can
also be seen at high lithium concentrations. Jaundice is a result of hepatic dysfunction. Lithium damages
the kidneys, not the liver. Lithium-induced polyuria is the result of lithium antagonism to the effects of
antidiuretic hormone.
Psychopharmacology
K&S Chapter 29

Question 142. B. OCD is believed to be a result of an imbalance in the serotonin system. Data show that serotonergic
drugs are more effective in relieving symptoms of OCD than drugs that work on other neurotransmit-
ter systems. Studies are unclear as to whether serotonin actually is involved in the cause of OCD. CSF
assays of 5-HIAA have reported variable findings in patients with OCD. In one study, CSF levels of

125

Psychiatry Test Preparation and Review


Manual E-Book
Psychiatry Test Preparation & Review Manual

5-HIAA decreased after treatment with clomipramine, leading researchers to the conclusion that the
serotonin system is involved in OCD symptom genesis.
Obsessive Compulsive and Related Disorders
K&S Chapter 10

Question 143. A. The patient in this question has schizoid personality disorder. He has a pervasive pattern of detachment
from social relationships and a restricted range of emotional expression. Other symptoms include taking
little to no pleasure in activities, lacking close friends or confidants, and showing emotional coldness or
detachment. Schizotypal patients have odd behaviors and beliefs, such as magical thinking. Narcissistic
patients are grandiose, need admiration, and lack empathy. This patient is socially isolated but has none
of the other criteria that one would expect to find with a major depressive disorder or dysthymic disor-
der, and as such they are not the correct answers.
Personality Disorders
K&S Chapter 22

Question 144. D. Vascular dementia, formerly known as multiinfarct dementia, is generally the result of one or more cere-
brovascular accidents that are the consequence of ischemic or embolic risk factors for cerebrovascular
events. Vascular dementia occurs more frequently in men, particularly those with known cerebrovascular
risk factors, which include smoking, hypertension, and diabetes. Symptoms are generally abrupt in onset,
and a pattern of stepwise decline in cognition and functioning can often be noted. There is often a pres-
ence of demonstrable focal neurologic deficits. Vascular dementia is generally poorly responsive to cho-
linergic repletion therapy by acetylcholinesterase inhibitors.
Neurocognitive Disorders
K&S Chapter 21

Question 145. C. This patient presents with symptoms of REM sleep behavior disorder, from which patients exhibit vocal-
izations and complex motor movements during REM sleep. They act out their dreams and don’t experi-
ence the normal absence of muscle tone found during normal REM sleep. Most commonly it occurs in
men over age 65. As they act out their dreams, they can hit, kick, thrash and scream, potentially harming
anyone else in the bed with them. When they wake up, they are not confused and often remember their
dreams. REM sleep behavior disorder often precedes cognitive symptoms in Lewy body disease. It is
associated with Lewy body disease in 50% to 80% of cases. Treatment with clonazepam at bedtime will
often reduce or abolish the REM sleep behavior episodes. The other answer choices, although fair game
for their own questions on a standardized examination, have no association with REM sleep behavior
disorder.
Sleep–Wake Disorders
K&S Chapter 10

Question 146. E. The psychiatrist is the member of the team who is held legally responsible for the team’s decisions. It
stems from the concept that the highest person in the hierarchy is responsible for the actions of those he
or she supervises. The psychiatrist is considered the head of this team. As such, the attending psychiatrist
is responsible for the actions of the residents he or she supervises and is responsible for the actions of the
team.
Forensic Psychiatry
K&S Chapter 36

Question 147. E. The tricyclic antidepressants have many cardiac side effects that are worsened in overdose. They act
as type 1A antiarrhythmics. As such, they can terminate ventricular fibrillation and increase collat-
eral blood supply to ischemic heart tissue. In overdose, they can be highly cardiotoxic and will cause
decreased myocardial contractility, tachycardia, hypotension, and increased myocardial irritability.
Also important to note is that nortriptyline is unique for its therapeutic window. Blood levels should be
obtained, and the therapeutic range is 50 to 150 ng/mL. Levels above 150 ng/mL may reduce its efficacy.
Psychopharmacology
K&S Chapter 29

126

Psychiatry Test Preparation and Review


Manual E-Book
Test Number Two

Question 148. C. Past suicidal behavior is the best predictor of future suicidal behavior. It is a better predictor than any
of the other answer choices given in this question. As such, one of the most important questions to ask
a patient who has suicidal thoughts is about his or her history of past suicide attempts. Other risk fac-
tors for suicide are as follows: men are more likely than women; divorced or single persons more likely
than married; elderly are less likely to attempt, but more likely to succeed when they do; whites are more
likely than other ethnic groups; and the higher the social status, the higher the risk.
Management in Psychiatry
K&S Chapter 23

Question 149. C. This question gives a description of someone who has schizotypal personality disorder. Schizotypal per-
sonality disorder is composed of a pervasive pattern of personal and social deficits characterized by ideas
of reference, odd beliefs or magical thinking, unusual perceptual experiences, paranoid ideation, inap-
propriate affect, eccentric appearance, lack of close friends, and excessive social anxiety that has a para-
noid flair.
Personality Disorders
K&S Chapter 22

Question 150. E. Imipramine is often used to treat childhood enuresis. Desmopressin is also useful in 50% of patients.
Improvement can range from cessation of enuresis to leaking less urine. Drugs should be given 1 hour
before bedtime, and results are usually evident within days. Patients who achieve full dryness should
take the medication for several months to prevent relapse.
Elimination Disorders
K&S Chapter 31

127

Psychiatry Test Preparation and Review


Manual E-Book
Psychiatry Test Preparation & Review Manual

THREE
Test Number Three
1. During which one of Piaget’s stages of development will a child be able to understand that a tall glass and a short wide
glass can contain the same volume of water, despite their different shapes?

A. 
Sensorimotor stage
B. 
Preoperational thought stage
C. 
Concrete operations stage
D. 
Formal thought stage
E. 
Anal stage

2. Which one of the following is not a characteristic of transient global amnesia?

A. 
Loss of personal information and identity
B. 
Reversible anterograde and retrograde memory loss
C. 
Inability to learn newly acquired information
D. 
Preservation of alertness without motor or sensory deficits
E. 
Men are affected more commonly than women

3. Which one of the following answer choices is not true regarding γ-aminobutyric acid (GABA)?

A. 
GABA is thought to suppress seizure activity
B. 
GABA is thought to exacerbate mania
C. 
GABA is thought to decrease anxiety
D. 
GABA-A activity is potentiated by topiramate
E. 
Gabapentin has no activity at the GABA receptors or transporter

4. An 8-year-old boy has been at sleep-away summer camp for 2 weeks and presents with a sudden onset of facial diplegia.
The most likely infectious organism that might have caused this symptom is:

A. 
Treponema pallidum
B. 
Borrelia burgdorferi
C. 
Leptospira interrogans
D. 
Rickettsia rickettsii
E. 
Yersinia pestis

Full test - and additional VIDEO vignettes - available online - see inside front cover for details.

128

Psychiatry Test Preparation and Review


Manual E-Book
Test Number Three

5. A middle-aged man has been referred to your office by a plastic surgeon. The man is seeking a face lift for his “exces-
sively large cheeks.” The surgeon has not been able to find anything abnormal about the man’s face or skin, and when he
comes to see you, you fail to see anything wrong either. The patient insists that his cheeks are grotesque and ruining his
whole appearance. When pressed, he admits that others may not consider his cheeks to be as bad as he does. His most
likely diagnosis is:

A. 
Malingering
B. 
Schizophrenia
C. 
Somatic symptom disorder
D. 
Conversion disorder (functional neurological symptom disorder)
E. 
Body dysmorphic disorder

6. Tourette’s syndrome often involves which one of the following psychiatric manifestations?

A. 
Generalized anxiety disorder
B. 
Social anxiety disorder
C. 
Panic disorder
D. 
Obsessive–compulsive disorder (OCD)
E. 
Psychotic disorder

7. A young woman comes to the emergency room with a 1-week history of pressured speech, decreased sleep, grandios-
ity, and loosening of associations. The patient feels that she is being monitored by a satellite, and she is seen talking to
herself when no one else is in the room. Which one of the following criteria must be met to diagnose this patient with
schizoaffective disorder instead of bipolar disorder?

A. 
Presence of mania
B. 
Psychotic symptoms in the absence of mood symptoms for a 1-week period
C. 
At least one prior episode of depression
D. 
Presence of psychotic symptoms during a manic episode
E. 
Psychotic symptoms in the absence of mood symptoms for a 2-week period

8. Which one of the following therapies would be best suited to a bipolar patient in a manic episode during pregnancy?

A. 
Haloperidol
B. 
Lithium
C. 
Aripiprazole
D. 
Divalproex sodium
E. 
Electroconvulsive therapy

9. You are called to evaluate a 60-year-old man with a history of depression. His family reports that he has not been him-
self for the past 5 days. On examination, he makes poor eye contact, is inattentive, mutters incoherently, keeps rearrang-
ing pieces of paper on his bed tray with no apparent logic, and drifts off to sleep while you are talking to him. What is
his most likely diagnosis?

A. 
Depression
B. 
Dementia
C. 
Systemic lupus erythematosus
D. 
Delirium
E. 
OCD

129

Psychiatry Test Preparation and Review


Manual E-Book
Psychiatry Test Preparation & Review Manual

10. Which one of the following pharmacologic agents would be most likely to cause extrapyramidal side effects and possible
tardive dyskinesia?

A. 
Hydroxyzine
B. 
Diphenhydramine
C. 
Metoclopramide
D. 
Ondansetron
E. 
Tizanidine

11. The most common symptom seen in patients with narcolepsy is:

A. 
Hypnopompic hallucinations
B. 
Sleep paralysis
C. 
Hypnagogic hallucinations
D. 
Sleep attacks
E. 
Cataplexy

12. The primary auditory cortex localizes to which one of the following brain regions?

A. 
Temporal lobe
B. 
Parietal lobe
C. 
Frontal lobe
D. 
Occipital lobe
E. 
Thalamus

13. Which one of the following neurotransmitters works as an adjunctive neurotransmitter for glutamate and as an indepen-
dent neurotransmitter with its own receptors?

A. 
GABA
B. 
Norepinephrine
C. 
Serotonin
D. 
Dopamine
E. 
Glycine

14. Which one of the following agents is not acceptable and useful for the treatment of postherpetic neuralgia?

A. 
Gabapentin
B. 
Lidoderm
C. 
Pregabalin
D. 
Carbamazepine
E. 
Phenytoin

15. A construction worker was standing very near a three-story scaffold that fell and missed crushing him by inches. Four
days later he reports feeling anxiety; a sense of numbing, detachment, and difficulty remembering the accident; and he
states that he feels like he’s in a daze. The most likely diagnosis is:

A. 
Generalized anxiety disorder
B. 
Major depression
C. 
Delirium

130

Psychiatry Test Preparation and Review


Manual E-Book
Test Number Three

D. 
Dissociative amnesia
E. 
Acute stress disorder

16. Trauma to which one of the following vessels or groups of vessels commonly causes epidural hematoma?

A. 
Middle meningeal artery
B. 
Meningeal bridging veins
C. 
Cavernous sinus
D. 
Basilar artery
E. 
Transverse sinus

17. A 26-year-old woman comes into the emergency room. She reports that she has been having mood swings that go from
depressed to elated to rageful in minutes to hours. She has been having paranoid feelings and vague auditory hallucina-
tions over the past week since breaking up with her boyfriend. On this past Monday, she cut her arms with a razor, but
only superficially. Her history reveals promiscuity, unstable relationships, and cocaine use. She now reports suicidal ide-
ation. Her most likely diagnosis is:

A. 
Bipolar disorder
B. 
Depression with psychotic features
C. 
Schizoid personality disorder
D. 
Borderline personality disorder
E. 
Schizotypal personality disorder

18. What is the characteristic electroencephalographic pattern noted in Creutzfeldt–Jakob disease?

A. 
Three-per-second spike and wave
B. 
Periodic sharp wave complexes
C. 
Temporal spikes
D. 
Generalized background slowing
E. 
Periodic lateralizing epileptiform discharges (PLEDs)

19. According to the Diagnostic and Statistical Manual of Mental Disorders 5 (DSM 5), catatonia may not be used as a
specifier to which one of the following diagnoses?

A. 
Major depressive disorder (MDD)
B. 
Bipolar disorder
C. 
Schizophreniform disorder
D. 
Hepatic encephalopathy
E. 
OCD

20. Which one of the following cerebrospinal fluid (CSF) findings is indicative of aseptic meningitis?

A. 
Variably increased lymphocytes, slightly decreased glucose, very high protein
B. 
Moderately increased lymphocytes, decreased glucose, mildly elevated protein
C. 
Highly increased neutrophils, decreased glucose, very high protein
D. 
Slightly increased lymphocytes, normal glucose, mildly elevated protein
E. 
Absent or few lymphocytes, normal glucose, very high protein

131

Psychiatry Test Preparation and Review


Manual E-Book
Psychiatry Test Preparation & Review Manual

21. A small child is in the park with her mother. As the two interact, the child goes off to play for a brief time. She then
returns to her mother, then goes off to play, then returns to her mother. The child continues this pattern, regularly check-
ing to see that her mother is still there. She would best fit into which one of Mahler’s stages of separation–individuation?

A. 
Normal autism
B. 
Symbiosis
C. 
Rapprochement
D. 
Practicing
E. 
Object constancy

22. What type of systemic poisoning results in the development


of characteristic Mees’ lines of the fingernails, as in the
photograph?

A. 
Mercury
B. 
Arsenic
C. 
Lead
D. 
Organophosphates
E. 
Ionizing radiation

23. 
The assumption that there is no significant difference
between two random samples of a population is called:

A. 
Correlation coefficient
B. 
Control group
C. 
Analysis of variance (ANOVA)
D. 
Regression analysis
E. 
Null hypothesis

24. Organophosphate poisoning by pesticide exposure causes neurologic deficits by which one of the following mechanisms?

A. 
Anticholinergic toxicity
B. 
Cholinergic toxicity
C. 
GABA blockade
D. 
Serotonergic toxicity
E. 
Dopaminergic toxicity

25. Emil Kraepelin used which one of the following terms in reference to schizophrenia?

A. 
Language delay
B. 
Dementia praecox
C. 
Split personality
D. 
Rebound hyperactivity
E. 
Downward drift

26. Which one of the following agents causes poisoning in humans that can result in a blue line at the gingival margin?

A. 
Manganese
B. 
Thallium
C. 
Arsenic
D. 
Mercury
E. 
Lead

132

Psychiatry Test Preparation and Review


Manual E-Book
Test Number Three

27. The police bring a man into the hospital who has been stealing satellite dishes off houses and setting them up in his
own yard because he feels that he has a chip in his head that allows him to talk directly to God. He states that God has
instructed him to do this as preparation for the second coming. When his wife is questioned about her husband’s behav-
ior, she responds that indeed God has been directly communicating with her husband, and that she has helped him steal
some of the larger satellite dishes. The wife’s condition can best be described as:

A. 
Schizoid personality disorder
B. 
MDD
C. 
Delusional disorder
D. 
Bipolar disorder
E. 
Substance-induced psychotic disorder

28. Truncal ataxia or instability can result most specifically from a lesion to the:

A. 
Cerebellar hemispheres
B. 
Cerebellar vermis
C. 
Cerebellopontine angle
D. 
Thalamus
E. 
Midbrain

29. Which one of the following neurotransmitters has large numbers of receptors in the spinal cord, is synthesized primarily
from serine, and has been the subject of research involving negative symptom reduction in schizophrenia?

A. 
GABA
B. 
Glycine
C. 
Serotonin
D. 
Dopamine
E. 
Glutamate

30. Which one of the following is not a contraindication to lumbar puncture?

A. 
Thrombocytopenia
B. 
Cerebral mass lesion
C. 
Suspected meningitis with obtundation
D. 
Recent head trauma
E. 
Papilledema

31. What is the difference between posttraumatic stress disorder and acute stress disorder?

A. 
The nature of the trauma
B. 
The symptoms that follow the trauma
C. 
The impairment resulting from the symptoms
D. 
The duration of the symptoms
E. 
The age of the patient

32. Which chromosomal abnormality is the most common cause of mental retardation?

A. 
Trisomy 21
B. 
Trisomy 18

133

Psychiatry Test Preparation and Review


Manual E-Book
Psychiatry Test Preparation & Review Manual

C. 
Cri-du-chat syndrome
D. 
Fragile X syndrome
E. 
Prader–Willi syndrome

33. Ending up in strange places with no recollection of how one got there or finding objects in one’s possession that one
doesn’t recall acquiring is most characteristic of:

A. 
Anxiety
B. 
Psychosis
C. 
Histrionic personality
D. 
Dissociation
E. 
Depression

34. Which one of the following child abuse injuries is the most likely to result in death or long-term sequelae?

A. 
Embolic stroke from multiple bone fractures
B. 
Subdural hematoma from head trauma
C. 
Skull fracture from head trauma
D. 
Cerebral hypoxia from choking
E. 
Seizures from head and brain trauma

35. A 59-year-old man comes into your office complaining of depression. His wife of 25 years died unexpectedly 5 weeks
before. Since then he has been crying, has had little appetite but has lost no weight, and reports difficulty sleeping. He
has been going out to dinner once each week with friends and says that it helps get his mind off of his wife’s death. He is
not suicidal. This is the first time in his life that he has had symptoms such as these. His most likely diagnosis is:

A. 
Bipolar disorder
B. 
MDD
C. 
Acute stress disorder
D. 
Bereavement
E. 
Dysthymic disorder

36. Which one of the following is not considered a lower motor neuron sign?

A. 
Hypotonia
B. 
Muscle atrophy
C. 
Fasciculations
D. 
Babinski’s sign
E. 
Hyporeflexia

37. Which of the following is not a common comorbidity associated with social phobia?

A. 
Drug abuse
B. 
Conversion disorder
C. 
Alcohol abuse
D. 
Other anxiety disorders
E. 
MDD

134

Psychiatry Test Preparation and Review


Manual E-Book
Test Number Three

38. Which one of the following therapeutics can eliminate benign paroxysmal positional vertigo?

A. 
Diazepam
B. 
Brandt–Daroff exercises
C. 
Meclizine
D. 
Metoclopramide
E. 
Gabapentin

39. Which one of the following is not a diagnostic criterion for kleptomania?

A. 
Recurrent failure to resist stealing objects
B. 
Decreased sense of tension immediately preceding the theft
C. 
Pleasure at the time of committing the theft
D. 
The theft is not done to express anger
E. 
The act is not in the context of an antisocial personality disorder

40. The persistent vegetative state is not characterized by which one of the following?

A. 
Preserved eye opening
B. 
Preserved response to noxious stimuli
C. 
Preserved eye tracking
D. 
Preserved swallowing
E. 
Preserved sleep–wake cycles

41. A Hispanic man comes into the emergency room complaining of headache, insomnia, fear, anger, and despair. Your dif-
ferential diagnosis is most likely to include which one of the following?

A. 
Schizophrenia
B. 
Schizoaffective disorder
C. 
Panic disorder
D. 
Ataque de nervios
E. 
Myoclonic sleep disorder

42. What is the most typical effect of depression on nocturnal sleep?

A. 
Decreased total sleep time
B. 
Initial insomnia
C. 
Middle insomnia
D. 
Early morning awakening
E. 
Sleep–wake cycle reversal

43. Which one of the following symptoms would not be expected in a patient with pyromania?

A. 
Deliberate and purposeful fire setting
B. 
Tension before the act
C. 
Fascination with fire
D. 
Pleasure when setting fires
E. 
Setting fires for monetary gain or as an expression of political ideology

135

Psychiatry Test Preparation and Review


Manual E-Book
Psychiatry Test Preparation & Review Manual

44. Apoptosis mediated by the N-methyl-d-aspartate (NMDA) receptor complex is most likely caused by elevated intracel-
lular levels of which one of the following ions?

A. 
Calcium
B. 
Magnesium
C. 
Sodium
D. 
Potassium
E. 
Chloride

45. A patient is brought into the emergency room after a fight with police. Upon examination the psychiatrist finds that the
patient has a history of several discrete assaultive acts. His aggression in these situations was out of proportion to what
one would consider normal. The patient has no other psychiatric disorder and no history of substance abuse. He has no
significant medical history. What is his most likely diagnosis?

A. 
Unspecified disruptive, impulse control, and conduct disorder
B. 
Pyromania
C. 
Mania
D. 
Temporal lobe epilepsy
E. 
Intermittent explosive disorder

46. A 45-year-old unconscious and barely breathing woman comes to the emergency room by ambulance. Paramedics found
an empty bottle of 90 tablets of 2 mg clonazepam on her dresser that was filled at the pharmacy the day before. One of
the first agents to administer to this patient in the acute setting is:

A. 
Naloxone
B. 
Flumazenil
C. 
Dimercaprol
D. 
Atropine
E. 
Asenapine

47. Which one of the following should not be considered in the differential diagnosis for intermittent explosive disorder?

A. 
Delirium
B. 
Dementia
C. 
Temporal lobe epilepsy
D. 
OCD
E. 
Substance intoxication

48. Which one of the following choices is not consistent with the collaborative care model?

A. 
Primary care physicians and psychiatrists need education on the collaborative care model
B. 
Primary care physicians can manage patients who have been stabilized on psychiatric medications
C. 
All psychiatric patients can be managed by the primary care physician with backup by the psychiatrist
D. 
Primary care physicians can initiate treatment for psychiatric disorders
E. 
Communication between primary care physician and psychiatrist is essential for the patient’s success

136

Psychiatry Test Preparation and Review


Manual E-Book
Test Number Three

49. Which one of the following psychiatric symptoms is not found with AIDS?

A. 
Progressive dementia
B. 
Personality changes
C. 
Heat intolerance
D. 
Depression
E. 
Loss of libido

50. Which one of the following structures is not part of Papez’ circuit?

A. 
Amygdala
B. 
Mammillary body
C. 
Fornix
D. 
Cingulate gyrus
E. 
Hippocampus

51. A patient comes to you reporting sexual difficulties. She wants to have sex but is unable to get sexually excited. She fails
to develop appropriate genital swelling and lubrication for intercourse. She denies any pain during intercourse. What is
the most appropriate diagnosis?

A. 
Substance/medication-induced sexual dysfunction
B. 
Female sexual interest/arousal disorder
C. 
Female orgasmic disorder
D. 
Genitopelvic pain/penetration disorder
E. 
Gender dysphoria

52. Which one of the following neurologic disorders is not believed to be caused by defects in the calcium channel system?

A. 
Lambert–Eaton myasthenic syndrome
B. 
Malignant hyperthermia
C. 
Hypokalemic periodic paralysis
D. 
Familial hemiplegic migraine
E. 
Benign familial neonatal convulsions

53. A young woman comes to her psychiatrist’s office for help. She feels that others are exploiting her but has no hard evi-
dence. She is preoccupied with the lack of loyalty that she feels all of her friends have. She reads hidden, demeaning con-
notations into the psychiatrist’s comments. She bears grudges and is unforgiving of slights. Her most likely diagnosis is:

A. 
Schizophrenia
B. 
Schizotypal personality disorder
C. 
Paranoid personality disorder
D. 
Schizoid personality disorder
E. 
Dementia

54. Which one of the following is not a clinical feature of phenylketonuria?

A. 
Sensorineural deafness
B. 
Infantile spasms
C. 
Microcephaly

137

Psychiatry Test Preparation and Review


Manual E-Book
Psychiatry Test Preparation & Review Manual

D. 
A characteristic “mousy” odor
E. 
Light hair and skin pigmentation

55. Which one of the following dopaminergic tracts or areas is responsible for the parkinsonian side effects of antipsychotic
medications?

A. 
Mesolimbic–mesocortical tract
B. 
Tuberoinfundibular tract
C. 
Nigrostriatal tract
D. 
Caudate neurons
E. 
Ventral striatum

56. Which one of the following disorders is correctly depicted in the follow-
ing image?

A. 
Lissencephaly
B. 
Schizencephaly
C. 
Dandy–Walker syndrome
D. 
Arnold–Chiari type I malformation
E. 
Arnold–Chiari type II malformation

57. A schizophrenic man on an inpatient psychiatric unit develops a fever of


102.3° F (39.1° C), a high white blood cell (WBC) count, unstable pulse
and blood pressure, and rigidity in the arms and legs. The most likely diagnosis is:

A. 
Meningitis
B. 
Serotonin-specific reuptake inhibitor withdrawal
C. 
Lithium toxicity
D. 
Neuroleptic malignant syndrome
E. 
Phencyclidine (PCP) use

58. Internal carotid artery stenosis should be treated surgically by endarterectomy if the stenosis is symptomatic and above:

A. 
50%
B. 
60%
C. 
70%
D. 
80%
E. 
90%

59. Which one of the following is not true regarding use of the tricyclic antidepressants?

A. 
Owing to their ability to prolong cardiac conduction time, their use in patients with conduction defects is
contraindicated
B. 
These agents should be discontinued before elective surgery because they may cause hypertensive episodes during
surgery
C. 
Some patients who experience orthostatic hypotension may respond to the use of fludrocortisone
D. 
Myoclonic twitches and tremors of the tongue and upper extremities are common in some patients on tricyclics
E. 
Amoxapine is the least likely of all the tricyclics to cause parkinsonian symptoms

138

Psychiatry Test Preparation and Review


Manual E-Book
Test Number Three

60. Poor outcome in Guillain–Barré syndrome is often associated with a preceding infection by which one of the following
pathogens?

A. 
Haemophilus influenzae
B. 
Streptococcus pneumoniae
C. 
Escherichia coli
D. 
Campylobacter jejuni
E. 
Clostridium difficile

61. Which one of the following is false regarding the Pearson correlation coefficient?

A. 
It spans from −1 to +1
B. 
A positive value means that one variable moves the other variable in the same direction
C. 
It can give information about cause and effect
D. 
It indicates the degree of relationship
E. 
A negative value means that one variable moves the other variable in the opposite direction

62. Which one of the following solid tumors metastasizes most frequently to the brain?

A. 
Breast
B. 
Colon
C. 
Prostate
D. 
Lung
E. 
Thyroid

63. A pervasive pattern of social inhibition, feelings of inadequacy, and hypersensitivity to negative evaluation is most char-
acteristic of:

A. 
Obsessive–compulsive personality disorder (OCPD)
B. 
Avoidant personality disorder
C. 
Schizoid personality disorder
D. 
Dependent personality disorder
E. 
Histrionic personality disorder

64. The most useful treatment for intractable post–lumbar puncture headache is:

A. 
Repeat lumbar puncture with blood patch
B. 
Bed rest for 2 weeks
C. 
Acetaminophen
D. 
Hydrocodone
E. 
Sumatriptan

65. Which one of the following baseline laboratory values would be the least important to obtain on a patient starting lith-
ium therapy?

A. 
Thyroid function tests
B. 
Electrocardiogram
C. 
WBC count
D. 
Renal function tests
E. 
Venereal Disease Research Laboratory (VDRL) test

139

Psychiatry Test Preparation and Review


Manual E-Book
Psychiatry Test Preparation & Review Manual

66. Compression of which one of the following peripheral nerves results in meralgia paresthetica?

A. 
Sciatic
B. 
Lateral femoral cutaneous
C. 
Obturator
D. 
Superior gluteal
E. 
Common peroneal

67. After a long day in the hospital, you visit your best friend from college who has recently had a baby. The child has been
spending more of her time asleep than awake and is not particularly aware of the environment. If your friend were to
ask you which of Mahler’s stages of separation–individuation the child fits into, you would confidently answer:

A. 
Normal autism
B. 
Symbiosis
C. 
Object constancy
D. 
Practicing
E. 
Differentiation

68. The presence of porphobilinogen and aminolevulinic acid in urine, blood, and stool is diagnostic for:

A. 
Adrenal insufficiency
B. 
Sheehan’s syndrome
C. 
Cushing’s syndrome
D. 
Acute intermittent porphyria
E. 
Systemic lupus erythematosus

69. Which one of the following brain structures’ dopaminergic neurons have been linked with Tourette’s syndrome and the
development of tics?

A. 
Caudate
B. 
Substantia nigra
C. 
Amygdaloid body
D. 
Frontal cortex
E. 
Hippocampus

70. A 35-year-old woman delivers her baby at 40 weeks’ gestation without complication. Seven days later she experiences
an acute onset of pancephalic headache, behavioral and personality changes, irritability, intermittent seizures, and diplo-
pia. The most likely diagnosis of her problem is:

A. 
Aseptic meningitis
B. 
Cerebral venous thrombosis
C. 
Complicated migraine
D. 
Bacterial meningoencephalitis
E. 
Pseudotumor cerebri

71. When should advanced directives be discussed with a patient?

A. 
At a time when the patient is competent
B. 
When admitted to the hospital

140

Psychiatry Test Preparation and Review


Manual E-Book
Test Number Three

C. 
When a question of capacity arises
D. 
When the patient is in pre-op
E. 
When in the outpatient office

72. The most common congenital viral infection in newborns is caused by which one of the following pathogens?

A. 
Cytomegalovirus (CMV)
B. 
Herpes simplex
C. 
Human immunodeficiency virus (HIV)
D. 
Rubella
E. 
Measles

73. A pervasive and excessive need to be taken care of that leads to submissive and clinging behavior, as well as fear of sepa-
ration, is characteristic of which one of the following?

A. 
OCPD
B. 
Avoidant personality disorder
C. 
Schizoid personality disorder
D. 
Dependent personality disorder
E. 
Histrionic personality disorder

74. The most frequent myopathy in patients over 50 years of age is:

A. 
Dermatomyositis
B. 
Polymyositis
C. 
Facioscapulohumeral dystrophy
D. 
Inclusion body myositis
E. 
Oculopharyngeal muscular dystrophy

75. Which one of the following is a behavioral effect of opioids?

A. 
Miosis
B. 
Increased arousal
C. 
Euphoria
D. 
Diarrhea
E. 
Tachycardia

76. Duchenne’s and Becker’s muscular dystrophies are both disorders linked to an absence or deficiency of which one of the
following muscle membrane proteins?

A. 
Dystrophin
B. 
Laminin
C. 
Dystroglycan
D. 
Spectrin
E. 
Merosin

141

Psychiatry Test Preparation and Review


Manual E-Book
Psychiatry Test Preparation & Review Manual

77. A 60-year-old woman is 63 lb (28 kg) overweight. She comes to her psychiatrist’s office with a complaint of increased
irritability, noting a fight with her husband over how much sugar he put in her coffee one morning. She is fatigued and
naps several times each day. She has no history of psychiatric problems, but she says that her husband now sleeps in the
living room because of her snoring. Which one of the following is the most likely cause of the patient’s symptoms?

A. 
Night terrors
B. 
MDD
C. 
Bipolar disorder
D. 
Narcolepsy
E. 
Sleep apnea

78. The rate-limiting enzyme in the dopamine synthetic pathway is:

A. 
DOPA decarboxylase
B. 
Tyrosine hydroxylase
Dopamine β-hydroxylase
C. 
D. 
Phenylethanolamine N-methyltransferase (PNMT)
E. 
Catechol-O-methyltransferase

79. Which one of the following is the therapeutic range for lithium?

A. 
6–12 mEq/L
B. 
0.5–1.9 mEq/L
C. 
0.6–1.2 mEq/L
D. 
2–4 mEq/L
E. 
>4 mEq/L

80. Alcohol, benzodiazepine sedative-hypnotic agents, and barbiturates all predominantly exert their clinical effects on the
brain at which one of the following receptor sites?

A. 
Cholinergic nicotinic
B. 
NMDA
C. 
Glycine
D. 
GABA-A
E. 
GABA-B

81. An elderly man comes into his doctor’s office with symptoms of dementia and notable loss of executive functioning.
Dysfunction of which one of the following brain regions would be most closely associated with the patient’s loss of
executive functioning?

A. 
Caudate nucleus
B. 
Putamen
C. 
Globus pallidus
D. 
Frontal lobes
E. 
Temporal lobes

142

Psychiatry Test Preparation and Review


Manual E-Book
Test Number Three

82. A patient who is not malingering but is believed to be producing the symptoms and signs of confusion or dementia
involuntarily or unconsciously and believes that the symptoms are real is thought to have which one of the following
conditions?

A. 
Conversion disorder (functional neurological symptom disorder)
B. 
Ganser’s syndrome
C. 
Capgras’ syndrome
D. 
Illness anxiety disorder
E. 
Folie-à-deux

83. A child in school fails test after test. No matter how hard he studies, he fails. Over time he views himself as destined to
fail and stops trying. Which one of the following theories best applies to this child’s situation?

A. 
The epigenetic principle
B. 
Industry theory
C. 
Cognition theory
D. 
Learned helplessness
E. 
Sensory deprivation

84. Entacapone and tolcapone exert their antiparkinsonian effects on which one of the following enzymes?

A. 
Catechol-O-methyltransferase
B. 
Monoamine oxidase type A
C. 
Monoamine oxidase type B
D. 
DOPA decarboxylase
Dopamine β-hydroxylase
E. 

85. Which one of the following could drastically increase lithium levels?

A. 
Citalopram
B. 
Carbamazepine
C. 
Sertraline
D. 
Ibuprofen
E. 
Acetaminophen

86. The neurologic examination finding of Argyll Robertson pupils occurs in a majority of patients with which one of the
following conditions:

A. 
Lyme disease
B. 
Multiple sclerosis
C. 
Tabes dorsalis
D. 
Bubonic plague
E. 
Intracerebral aneurysm

87. You are hired by an employer to do a psychiatric evaluation on one of their employees. Which one of the following cor-
rectly represents the limits of confidentiality in this situation?

A. 
Your evaluation of the patient is confidential
B. 
Your evaluation of the patient is not confidential

143

Psychiatry Test Preparation and Review


Manual E-Book
Psychiatry Test Preparation & Review Manual

C. 
The employer has a right to know the patient’s entire medical history
D. 
The patient can prevent you from disclosing any information obtained in this evaluation to the employer
E. 
The employer has a right to know only those elements of the patient’s condition that affect job performance

88. A 33-year-old man with known epilepsy has a 45-second generalized tonic–clonic seizure at a bus stop outdoors. He
is brought to the nearest emergency room, and once he is arousable and awake, is found to have a marked right-sided
hemiparesis. What is the best explanation of this occurrence?

A. 
Postictal stroke
B. 
Todd’s paralysis
C. 
Complicated postictal migraine
D. 
Conversion disorder
E. 
Transient ischemic attack

89. Which one of the following does not have active metabolites?

A. 
Oxazepam
B. 
Chlordiazepoxide
C. 
Triazolam
D. 
Clonazepam
E. 
Quazepam

90. The disease known as tropical spastic paraparesis is considered to be a _____ caused by the _____ virus.

A. 
Myelopathy; human T-lymphotropic virus type-1 (HTLV-1)
B. 
Neuropathy; HTLV-1
C. 
Myelopathy; herpes simplex
D. 
Neuropathy; herpes simplex
E. 
Neuropathy; HIV type 1

91. To control aggression in a child with intellectual disability, which one of the following would be most effective?

A. 
Clonazepam
B. 
Mirtazapine
C. 
Doxepin
D. 
Lithium
E. 
Naltrexone

92. Which one of the following diseases is not associated with expansion of trinucleotide repeat sequences?

A. 
Friedrich’s ataxia
B. 
Myotonic dystrophy
C. 
Multiple system atrophy
D. 
Fragile X syndrome
E. 
Huntington’s disease

144

Psychiatry Test Preparation and Review


Manual E-Book
Test Number Three

93. Which one of the following dopaminergic pathways or areas is associated with the antipsychotic effects of the antipsy-
chotic medications?

A. 
Nigrostriatal pathway
B. 
Mesolimbic–mesocortical pathway
C. 
Tuberoinfundibular pathway
D. 
Caudate nucleus
E. 
Amygdaloid body

94. The characteristic bedside neurologic diagnostic sign known as the “battle sign” is indicative of which one of the follow-
ing pathologies?

A. 
Basilar skull fracture
B. 
Frontal lobe damage
C. 
Increased intracranial pressure
D. 
Hypocalcemia
E. 
Impending transtentorial cerebral herniation

95. Of the following combinations of medications, which one would the knowledgeable psychiatrist most want to avoid?

A. 
Fluoxetine–lithium
B. 
Fluoxetine–phenelzine
C. 
Citalopram–valproic acid
D. 
Citalopram–aripiprazole
E. 
Mirtazapine–lamotrigine

96. Which one of the following agents is the best choice for treating attention deficit hyperactivity disorder in a patient with
Tourette’s syndrome?

A. 
Bupropion
B. 
Methylphenidate
C. 
Dextroamphetamine
D. 
Atomoxetine
E. 
Clonidine

97. A method of obtaining a prediction for the value of one variable in relation to another variable is called:

A. 
Correlation coefficient
B. 
Control group
C. 
ANOVA
D. 
Regression analysis
E. 
Null hypothesis

98. Which one of the following oral agents would be most beneficial in the treatment of limb spasticity related to multiple
sclerosis?

A. 
Clonazepam
B. 
Phenytoin
C. 
Lioresal

145

Psychiatry Test Preparation and Review


Manual E-Book
Psychiatry Test Preparation & Review Manual

D. 
Brexpiprazole
E. 
Cyproheptadine

99. A measurement of the direction and strength of the relationship between two variables is called:

A. 
Correlation coefficient
B. 
Control group
C. 
ANOVA
D. 
Regression analysis
E. 
Null hypothesis

100. A patient in the intensive care unit is delirious and agitated. The electroencephalogram demonstrates a characteristic
pattern of triphasic waves and some generalized background slowing. What other clinical bedside sign would you be
most likely to see in this patient on physical examination?

A. 
Herpetic skin vesicles
B. 
Penile chancre
C. 
Asterixis
D. 
Dupuytren’s contractures
E. 
Pulmonary rales

101. Which one of the following is not necessary to be found competent to stand trial?

A. 
Ability to understand the charges
B. 
Ability to consult with a lawyer
C. 
Ability to process information related to the case
D. 
Ability to communicate with an attorney
E. 
Ability to recognize the symptoms of their own mental illness

102. Which one of the following is not a potential side effect of valproic acid?

A. 
Weight gain
B. 
Alopecia
C. 
Hemorrhagic pancreatitis
D. 
Thrombocytosis
E. 
Liver failure

103. A patient is admitted to the hospital for suicidal behavior. The insurance company refuses to pay for the stay after
the third day, and the patient is still suicidal. Which one of the following answer choices is the most ethical way to
proceed?

A. 
Send the patient home because the insurance company refuses to pay
B. 
Secretly tell the patient to sign out against medical advice
C. 
Make the patient pay out of pocket, and if he or she can’t afford it, discharge him or her
D. 
Sue the insurance company
E. 
Continue to treat the patient as long as necessary, and file an appeal with the insurance company after discharge

146

Psychiatry Test Preparation and Review


Manual E-Book
Test Number Three

104. The mechanism by which clonidine can help alleviate the symptoms of opioid withdrawal is through mediation of:

A. 
Norepinephrine reuptake inhibition
α2 Adrenergic agonism
B. 
α1 Adrenergic antagonism
C. 
D. 
Dopamine antagonism
E. 
Serotonin antagonism

105. On a urine toxicology screen, how long can heroin be detected?

A. 
12 hours
B. 
48 hours
C. 
4 weeks
D. 
72 hours
E. 
8 days

106. An elderly hospitalized patient with vascular risk factors has a stroke. The patient’s behavior after the stroke is noted to
be unusually calm and markedly hypersexual. This presentation probably resulted from a stroke to the:

A. 
Hippocampi
B. 
Nucleus accumbens
C. 
Hypothalamus
D. 
Occipital lobes
E. 
Amygdaloid bodies

107. A psychiatrist is working on an inpatient unit, and one of her patients has a very resistant case of depression. She con-
siders the option of giving the patient electroconvulsive therapy (ECT). Which one of the following should make her
most concerned about giving this patient ECT?

A. 
Pregnancy
B. 
Past seizures
C. 
Family history of severe depression
D. 
Psychotic symptoms
E. 
Recent myocardial infarction

108. A 20-year-old man comes to your office with his mother because of behavioral problems. On examination you note
that he is verbally inappropriate, mildly intellectually disabled, very tall and somewhat obese, and has a small penis and
scrotum. His condition is most likely due to which one of the following?

A. 
Absence of an X chromosome (XO)
B. 
Presence of an extra X chromosome (XXY)
C. 
Trisomy 21
D. 
Deletion on the paternal chromosome 15
E. 
Trisomy 18

109. A psychiatrist is consulted on a medical unit because there is a patient with a substance abuse history who is in need of
pain control. Which one of the following answer choices would be the best way to treat this patient’s pain?

A. 
Large doses of opiates
B. 
A mixture of opiates and benzodiazepines

147

Psychiatry Test Preparation and Review


Manual E-Book
Psychiatry Test Preparation & Review Manual

C. 
Patient-controlled analgesia
D. 
No opiates of any kind
E. 
PRN (as needed) buprenorphine

110. A 35-year-old woman presents to your office with her husband. He tells you that she has experienced discrete episodes of
physical and verbal aggression directed toward other people and property for the past year. Her husband states that the
aggressiveness is not precipitated by any particular trigger and is completely unpredictable and intermittent. In between these
bouts of aggression, the patient is otherwise fine and leads a normal life as a wife and mother. The most plausible diagnosis is:

A. 
Borderline personality disorder
B. 
Intermittent explosive disorder
C. 
Antisocial personality disorder
D. 
Psychotic disorder due to temporal lobe seizures
E. 
Bipolar I disorder, with manic and psychotic episodes

111. While working on the ward of a state hospital, a psychiatrist comes across a patient with schizophrenia. The patient sits
in one spot for extended periods of time without changing position. This phenomenon is best described as:

A. 
Psychomotor retardation
B. 
Catalepsy
C. 
Cataplexy
D. 
Catatonia
E. 
Stereotypy

112. Habit reversal is a type of therapy indicated for:

A. 
Developmental coordination disorder
B. 
Trichotillomania
C. 
OCD
D. 
Intermittent explosive disorder
E. 
Gambling disorder

113. Harry Harlow conducted a series of experiments with rhesus monkeys. Some of his monkeys would stare vacantly into
space, engage in self-mutilation, and follow stereotyped behavior patterns. To which one of the following groups did
these monkeys belong?

A. 
Total isolation (no caretaker or peer bond)
B. 
Mother-only reared
C. 
Peer-only reared
D. 
Partial isolation (can see, hear, and smell other monkeys)
E. 
Separation (taken from caretaker after developing bond)

114. Sildenafil, vardenafil, and tadalafil all improve erectile dysfunction by which one of the following mechanisms?

A. 
Phosphodiesterase 5 inhibition
B. 
Calcium channel antagonism
C. 
Nitric oxide antagonism
α1 Adrenergic antagonism
D. 
E. 
Carbonic anhydrase inhibition

148

Psychiatry Test Preparation and Review


Manual E-Book
Test Number Three

115. Which one of the following medications is the most likely to cause parkinsonian symptoms?

A. 
Maprotiline
B. 
Amoxapine
C. 
Venlafaxine
D. 
Doxepin
E. 
Clomipramine

116. If you divide the incidence of a disease in those with risk factors by the incidence of the same disease in those without
risk factors, the result is called the:

A. 
Relative incidence
B. 
Attributable risk
C. 
Relative risk
D. 
Period incidence
E. 
Incidence risk

117. Cingulotomy is a treatment used for which one of the following choices?

A. 
Psychosis
B. 
Depression
C. 
OCD
D. 
Generalized anxiety disorder
E. 
Pedophilia

118. Which one of the following causative etiologic factors is not believed to contribute to the genesis of dissociative identity
disorder?

A. 
A traumatic life event
B. 
A vulnerability for the disorder
C. 
Environmental factors
D. 
Absence of external support
E. 
Prior viral infection or exposure

119. Which one of the following is considered first-line treatment for a mute catatonic patient who is brought into the emer-
gency room?

A. 
Haloperidol
B. 
Methylphenidate
C. 
Iloperidone
D. 
Lorazepam
E. 
Paroxetine

120. Which one of the following is not a feature of malingering?

A. 
Findings are compatible with self-inflicted injuries
B. 
Medical records may have been tampered with or altered
C. 
Family members are able to verify the consistency of symptoms
D. 
Symptoms are vague or ill defined
E. 
History and examination do not yield complaints or problems

149

Psychiatry Test Preparation and Review


Manual E-Book
Psychiatry Test Preparation & Review Manual

121. Which one of the following answer choices is most true concerning mutism?

A. 
It is a psychiatric disorder only
B. 
It is a neurological disorder only
C. 
It is a function of a high-energy environment
D. 
It is associated with both psychiatric and neurologic conditions
E. 
It is most frequently the result of head trauma

122. Therapy that is focused on the measurement of autonomic processes and teaching patients to gain voluntary control
over these physiological parameters through operant conditioning is called:

A. 
Stimulus–response therapy
B. 
Biofeedback
C. 
Relaxation training
D. 
Behavior therapy
E. 
Desensitization

123. A patient has overdosed on lithium after a fight with her boyfriend. Her lithium level is 2.8 mEq/L. She is exhibiting
severe signs of lithium toxicity. Which one of the following answer choices is the best treatment at this time?

A. 
Intravenous fluids
B. 
Celecoxib
C. 
Wait and watch
D. 
Hemodialysis
E. 
Gastric lavage

124. The most useful long-term treatment parameter for a noncompliant patient with schizophrenia and history of violence
would be:

A. 
Long-term state psychiatric hospitalization
B. 
Partial hospitalization
C. 
Day treatment program
D. 
Outpatient commitment program
E. 
Social skills training

125. Which one of the following is a dangerous combination?

A. 
Monoamine oxidase inhibitor (MAOI)–lorazepam
B. 
MAOI–acetaminophen
C. 
MAOI–meperidine
D. 
MAOI–ziprasidone
E. 
MAOI–loxapine

126. The psychiatrist’s right to maintain a patient’s secrecy in the face of a subpoena is known as:

A. 
Privilege
B. 
Confidentiality
C. 
Communication rights
D. 
Private rights
E. 
Clinical responsibility

150

Psychiatry Test Preparation and Review


Manual E-Book
Test Number Three

127. Which one of the following is not a requirement for treatment with clozapine?

A. 
Baseline WBC count before starting treatment
B. 
Two WBC counts during the first 7 days of treatment
C. 
Weekly WBC count during the first 6 months of treatment
D. 
WBC count every 2 weeks after the first 6 months of treatment
E. 
WBC count every week for 4 weeks after discontinuation of clozapine

128. Which one of the following is not a change in sleep architecture noted in patients over 65 years of age?

A. 
Lower percentage of N3 sleep
B. 
Less total rapid eye movement (REM) sleep
C. 
Fewer REM episodes
D. 
Increased awakening after sleep onset
E. 
Shorter REM episodes

129. Which one of the following will increase clozapine levels?

A. 
Red wine
B. 
Cimetidine
C. 
Cheese
D. 
Acetaminophen
E. 
Aripiprazole

130. The most widely abused recreational substance among U.S. high school students is:

A. 
Alcohol
B. 
Cocaine
C. 
Lysergic acid diethylamide (LSD)
D. 
Inhalants
E. 
Cannabis

131. Beck, in the theory supporting his cognitive triad, felt that “distorted negative thoughts” lead to:

A. 
Failure of “good enough mothering”
B. 
Transitional object development
C. 
Mania
D. 
Depression
E. 
Aggression toward the primary caregiver

132. The mechanism of action of the sleeping aid ramelteon involves which one of the following receptor systems?

A. 
Acetylcholine
B. 
GABA-A
C. 
Histamine
D. 
Melatonin
E. 
Norepinephrine

151

Psychiatry Test Preparation and Review


Manual E-Book
Psychiatry Test Preparation & Review Manual

133. What is alogia?

A. 
Poverty of movement
B. 
Poverty of emotion
C. 
Poverty of speech only
D. 
Poverty of thought content only
E. 
Poverty of speech and thought content

134. The rapid-cycling specifier in bipolar I disorder applies to patients who have had _____ mood disturbance episodes over
the previous _____.

A. 
Four; 6 months
B. 
Four; 12 months
C. 
Six; 24 months
D. 
Six; 12 months
E. 
Three; 12 months

135. Which one of the following best explains pathological gambling?

A. 
Primary reinforcement
B. 
Random reinforcement
C. 
Poor response to dexamethasone suppression testing
D. 
Continuous reinforcement
E. 
Cerebellar dysfunction

136. Late-onset schizophrenia is characterized by a more favorable prognosis and the onset of symptoms after age:

A. 
40 years
B. 
45 years
C. 
50 years
D. 
55 years
E. 
60 years

137. A group that does not receive treatment and is the standard for comparison is called the:

A. 
Correlation coefficient
B. 
Control group
C. 
ANOVA
D. 
Regression analysis
E. 
Null hypothesis

138. Bowlby’s stages of childhood attachment disorder, after a lengthy departure of the child’s mother, do not include which
one of the following?

A. 
Protest
B. 
Despair
C. 
Detachment
D. 
Denial of affection
E. 
Acceptance

152

Psychiatry Test Preparation and Review


Manual E-Book
Test Number Three

139. You have a patient who you think has both depression and attention deficit hyperactivity disorder. Which one of the fol-
lowing medications would be the best choice for this patient?

A. 
Fluoxetine
B. 
Vortioxetine
C. 
Bupropion
D. 
Venlafaxine
E. 
Imipramine

140. In restless legs syndrome the:

A. 
Urge to move legs worsens during periods of intense activity
B. 
Urge to move legs is relieved by rest
C. 
Urge to move legs can be accompanied by uncomfortable and unpleasant sensations in the legs
D. 
Urge to move legs is worst in the morning
E. 
Symptoms must persist for 1 month or more

141. During a session a therapist tells the patient, “I know you feel terrible right now, but things are going to get better with
the passage of time.” This type of statement is characteristic of which type of therapy?

A. 
Supportive psychotherapy
B. 
Psychodynamic psychotherapy
C. 
Psychoanalysis
D. 
Play therapy
E. 
Cognitive behavioral therapy

142. Which one of the following medications is not available as a long-acting injection?

A. 
Haloperidol
B. 
Olanzapine
C. 
Ziprasidone
D. 
Paliperidone
E. 
Risperidone

143. A patient is brought into the emergency room unconscious with signs of respiratory depression. The patient was found
on the bathroom floor and has written a suicide note saying that he wanted to die. An empty pill bottle that had con-
tained his mother’s prescription for morphine was found on the bathroom floor. Which one of the following would the
knowledgeable physician use to treat this patient?

A. 
Buprenorphine
B. 
Benztropine
C. 
Naloxone
D. 
Naltrexone
E. 
Bromocriptine

153

Psychiatry Test Preparation and Review


Manual E-Book
Psychiatry Test Preparation & Review Manual

144. Which one of the Kübler-Ross stages of reaction to impending death corresponds to a period in which a patient goes
through self-blame for his or her illness and asks “Why me?”

A. 
Shock and denial
B. 
Anger
C. 
Bargaining
D. 
Depression
E. 
Acceptance

145. Asking patients if they are suicidal will:

A. 
Increase the chance that they will kill themselves
B. 
Help them plan out their suicide
C. 
Scare the patients
D. 
Have no influence on whether patients will attempt suicide
E. 
Make patients refuse to speak to their therapist any further

146. The usual and accepted length of a period of grief after the death of a loved one can last up to:

A. 
3 months
B. 
6 months
C. 
9 months
D. 
12 months
E. 
24 months

147. In which one of the following groups has suicide increased dramatically over the past 40+ years?

A. 
Geriatrics
B. 
Married men
C. 
Married women
D. 
Adolescents
E. 
Chronic alcoholics

148. Which one of the following sleep-promoting agents has the longest half-life?

A. 
Ramelteon
B. 
Zolpidem
C. 
Zaleplon
D. 
Eszopiclone
E. 
Triazolam

149. What is the therapeutic range for valproic acid?

A. 
50–100 ng/mL
B. 
25–50 ng/mL
C. 
200–250 ng/mL
D. 
1000–1500 ng/mL
E. 
0.5–0.15 ng/mL

154

Psychiatry Test Preparation and Review


Manual E-Book
Test Number Three

150. A psychiatrist interviews a Japanese immigrant who was brought to the hospital by her family for depression. In the meeting
with her she does not endorse any significant symptoms. When speaking to the family, they state that she has been having
many of the symptoms that she previously denied. Which one of the following is the most likely explanation for this situation?

A. 
She is lying
B. 
She is psychotic and paranoid
C. 
Culture
D. 
Mental retardation
E. 
The family is lying

155

Psychiatry Test Preparation and Review


Manual E-Book
Psychiatry Test Preparation & Review Manual

Three Answer Key – Test Number Three


1. C 26. E 51. B 76. A 101. E 126. A
2. A 27. C 52. E 77. E 102. D 127. B
3. B 28. B 53. C 78. B 103. E 128. C
4. B 29. B 54. A 79. C 104. B 129. B
5. E 30. C 55. C 80. D 105. D 130. E
6. D 31. D 56. D 81. D 106. E 131. D
7. E 32. D 57. D 82. B 107. E 132. D
8. E 33. D 58. C 83. D 108. B 133. E
9. D 34. B 59. E 84. A 109. C 134. B
10. C 35. D 60. D 85. D 110. B 135. B
11. D 36. D 61. C 86. C 111. B 136. B
12. A 37. B 62. D 87. E 112. B 137. B
13. E 38. B 63. B 88. B 113. D 138. E
14. E 39. B 64. A 89. A 114. A 139. C
15. E 40. B 65. E 90. A 115. B 140. C
16. A 41. D 66. B 91. D 116. C 141. A
17. D 42. D 67. A 92. C 117. C 142. C
18. B 43. E 68. D 93. B 118. E 143. C
19. E 44. A 69. A 94. A 119. D 144. B
20. D 45. E 70. B 95. B 120. C 145. D
21. C 46. B 71. A 96. D 121. D 146. D
22. B 47. D 72. A 97. D 122. B 147. D
23. E 48. C 73. D 98. C 123. D 148. D
24. B 49. C 74. D 99. A 124. D 149. A
25. B 50. A 75. C 100. C 125. C 150. C

Full test - and additional VIDEO vignettes - available online - see inside front cover for details.

156

Psychiatry Test Preparation and Review


Manual E-Book
Test Number Three

Three
Explanations – Test Number Three
Question 1. C. This question focuses on Piaget’s stages of cognitive development. (Just in case you don’t have it mas-
tered after Test Two.) In the concrete operations stage, egocentric thought changes to operational
thought in which another’s point of view can be taken into consideration. In concrete operations, chil-
dren can put things in order and group objects according to common characteristics. They develop the
understanding of conservation (a tall cup and a wide cup can both hold equal volumes of water) and
reversibility (ice can change to water and back to ice again).
Human Development
K&S Chapter 2

Question 2. A. Transient global amnesia (TGA) presents with a reversible anterograde and retrograde memory loss. It is
accompanied by an inability to learn newly acquired information and total amnesia of events occurring
during the attacks. Patients remain awake and alert during attacks, without motor or sensory deficits.
Patients retain their personal information and identity and can carry on personal activities as usual. The
patient may ask the same question repeatedly. Affected patients are usually in their 50s or older. Attacks
are acute in onset and can last for several hours, but rarely longer than 12 hours. The true mechanism of
TGA is unknown. Onset often occurs after physical exertion, sexual exertion, or exposure to extremes
of temperature. The prognosis of patients with TGA is generally benign. There is no noted increased
risk for future ischemic attacks or stroke. Recurrence is uncommon, and there is no need for extensive
workup. No particular treatment is indicated.
Neurology
B&D Chapter 65

Question 3. B. GABA is the major inhibitory neurotransmitter of the central nervous system (CNS). It is thought to
decrease seizure activity, decrease mania, and lessen anxiety. It has three receptors, GABA-A, GABA-B,
and GABA-C. Topiramate works on the GABA-A receptor, potentiating its activity. Gabapentin
decreases seizure activity but does not work directly on the GABA receptors or transporter. Tiagabine is
an anticonvulsant that works by blocking the GABA transporter.
Psychopharmacology
K&S Chapter 29

Question 4. B. The case described in this question is a classic presentation of Lyme disease. The offending organism
in Lyme infection is the spirochete B. burgdorferi, whose vector is the deer tick: Ixodes dammini in
the eastern United States, Ixodes pacificus in the western United States, and Ixodes ricinus in Europe.
Lyme is the great mimicker of other neurologic conditions, and it can manifest in many different ways.
Early infection can appear as a meningitis, a unilateral or bilateral Bell’s palsy (as in this question), a
painful radiculoneuritis, optic neuritis, mononeuritis multiplex, or Guillain–Barré syndrome. After initial
infection, a classic Lyme rash, a painless expanding macular patch, can be noted (erythema chronicum

157

Psychiatry Test Preparation and Review


Manual E-Book
Psychiatry Test Preparation & Review Manual

migrans) in about two-thirds of patients. Diagnosis begins with initial enzyme-linked immunosorbent
assay (ELISA) serologic screening, which, if positive, can be confirmed by Western blot testing. CSF
Lyme antibody can also be titered by polymerase chain reaction. Treatments of first choice are parenteral
antibiotics: ceftriaxone or penicillin intravenously for 2 to 4 weeks. CSF persistence of antibody produc-
tion may continue for years after successful treatment and remission, and in isolation it does not indicate
active disease. Rocky Mountain spotted fever is caused by the tick-borne spirochete R. rickettsii. Various
tick species can carry this organism, and the disease can be seen all over the world. The condition starts
with fever, headache, muscle aches, and gastrointestinal symptoms about 2 to 14 days after the tick bite.
There is a rash that appears initially around the wrists and hands and spreads over days to the feet
and forearms. Systemic symptoms can also appear and include meningoencephalitis, renal failure, and
pulmonary edema. Retinal vasculitis may be seen on funduscopic examination. Diagnosis is confirmed
by direct immunofluorescence or immunoperoxidase skin biopsy staining. Treatment is undertaken with
oral or parenteral tetracycline or chloramphenicol and a switch to oral doxycycline for a total of about
10 to 14 days of therapy.

Yersinia pestis is an organism causing the plague, a zoonotic infection of wild rodents, transmitted by the
bites of infected fleas to human victims. Human infection can result in infectious lymphadenitis (bubonic
plague) or pneumonic, septicemic, or meningeal plague. Primary or secondary meningitis can occur from
Y. pestis infection. Diagnosis is confirmed by CSF Gram stain and culture. Treatment of primary infec-
tion is undertaken with intramuscular streptomycin twice daily for 10 days. Meningitis is treated with
intravenous chloramphenicol for at least 10 days.

Leptospirosis is caused by zoonotic infection from L. interrogans. This spirochete is transmitted to


humans by contact with urine from infected rodents or farm animals or through soil or water that con-
tains the infected urine. About 15% of patients develop a meningitic picture. Severe systemic symptoms
include jaundice, hemorrhage, and renal failure. Diagnosis can be confirmed by organism isolation from
blood or CSF. Severe disease is treated with at least a week-long course of parenteral penicillin G. Less
severe illness can be treated with a week-long course of oral doxycycline.

Treponema pallidum is the spirochete associated with syphilis, which is explained in depth elsewhere in
this volume.
Neurology
B&D Chapter 53

Question 5. E. This is a clear case of body dysmorphic disorder. It is characterized by an imagined belief that there
is a defect in part or all of the body. The belief does not approach delusional proportions. The patient
complains of the defect, and his perception is out of proportion to any minor physical abnormality that
exists. The person’s concern is grossly excessive, but when pressed he can admit that it may be excessive
(this is why it is not delusional). Treatment with serotonergic drugs and therapy is usually helpful; plas-
tic surgery is not.
Obsessive–Compulsive and Related Disorders
K&S Chapter 10

Question 6. D. Tourette’s syndrome (TS) is a tic disorder. Boys are affected more often than girls. The manifestations
begin during childhood. Early signs include cranial motor tics, including eye blinks, stretching of the
lower face, and head-shaking. Vocal tics can include throat clearing, grunting, coughing, sniffing, and
involuntary swearing (coprolalia). Both motor and vocal tics must be present to meet criteria for the
disorder. Symptoms tend to peak in severity during adolescence and often wane during adulthood.
Behavioral manifestations are common in TS and can include attention deficit hyperactivity disorder
(ADHD), OCD, and conduct disorder. Obsessions and rituals often revolve around counting and sym-
metry. The diagnosis is clinical and is based on the appropriate history and physical examination.

Evidence suggests that TS is hereditary. Concordance in monozygotic twins is greater than 85%.
Positron emission tomography studies suggest increased dopaminergic activity in the ventral striatum
and abnormal release and reuptake of dopamine as being probable pathophysiologic mechanisms.

158

Psychiatry Test Preparation and Review


Manual E-Book
Test Number Three

Treatment of TS is symptomatic. Tics can be treated with conventional or atypical neuroleptic agents.
Guanfacine and clonidine are useful for both tics and ADHD symptoms. OCD can be treated with the
selective serotonin reuptake inhibitors. The stimulants methylphenidate and dextroamphetamine must
be used carefully, because they can either alleviate or exacerbate tics. Atomoxetine, a nonstimulant
ADHD medication that is a norepinephrine reuptake inhibitor, is an excellent choice for ADHD in TS
because it does not exacerbate tics.
Disruptive, Impulse Control Disorders, Conduct Disorder, and ADHD
B&D Chapter 71

Question 7. E. To diagnose schizoaffective disorder (as opposed to bipolar), one needs a 2-week period in which the
patient has had psychotic symptoms in the absence of mood symptoms. The patient must also have had
periods of mania or depression during which psychotic symptoms are present. A bipolar patient may
have psychotic symptoms during periods of mania or depression, but these psychotic symptoms cease
when the mood disturbance resolves. To meet criteria in DSM 5, a major mood episode must be present
for the majority of the total duration of the active and residual portions of the illness.
Psychotic Disorders
K&S Chapter 7

Question 8. E. The safest choice of all the answers is of course ECT. There is no real contraindication to ECT in normal
pregnancy. If the pregnancy is high risk or complicated, fetal monitoring can be carried out during the
procedure. Haloperidol would be the next best choice and is of course a butyrophenone antipsychotic
agent. Haloperidol can pass into breast milk, so mothers should not breastfeed if they are taking this
drug. Haloperidol has been shown to cause teratogenicity in animals. Human studies are inadequate
in this regard, and the benefit needs to outweigh the risk before the drug is given to a pregnant patient.
The first trimester is the most vulnerable period of pregnancy for teratogenic fetal effects. Lithium is of
course contraindicated in pregnancy because of the risk of Ebstein’s anomaly of the tricuspid valves.
The risk of Ebstein’s anomaly is 1 in 1000. Lithium is also excreted into breast milk. Divalproex sodium
is also dangerous in pregnancy because of the first-trimester risk of fetal spina bifida and neural tube
defects in about 1% to 2% of those taking the drug in their first trimester. Folic acid supplementation (1
to 4 mg daily) taken during the first trimester of pregnancy reduces the risk of neural tube defects with
divalproex sodium. Aripiprazole has not been well studied in pregnancy. There are animal studies that
have revealed fetal abnormalities as a direct result of maternal exposure to aripiprazole. Another com-
monly tested fact with respect to psychiatric medications and pregnancy is that selective serotonin reup-
take inhibitors (SSRIs), most notably paroxetine, can increase the risk of pulmonary hypertension in the
infant.
Management in Psychiatry
K&S Chapter 30

Question 9. D. This is a case of delirium. Delirium presents as confusion, impaired consciousness (often fluctuating),
emotional lability, hallucinations or illusions, and irrational behavior. Its onset is rapid, and its course
fluctuates. Thinking is often disordered, and attention and awareness are often impaired. Causes are usu-
ally medical or organic in nature (metabolic imbalance, infection, intracranial bleed, etc.). The patient’s
increased age makes him more susceptible to a delirium. Dementia and depression are incorrect because
the time of onset of symptoms for these disorders is over weeks to months, not days. Systemic lupus ery-
thematosus (SLE) is a potential cause of delirium, but there are no details in this question that would point
you toward this as the best answer choice. SLE is an autoimmune disorder with manifestations across mul-
tiple organ systems, including fever, fatigue, weight loss, joint pain, photosensitivity, rashes, renal damage,
anemia, and leukopenia. Seventy to 90% of patients have neuropsychiatric manifestations such as head-
aches, seizures, stroke, depression, cognitive impairment, psychosis, and delirium. Cognitive dysfunction is
the most common neuropsychiatric symptom, with depression as the second most common. Ninety percent
of cases are female. And while on the topic of delirium… The prudent student would do well to keep in
mind that the best treatment for an agitated patient with delirium is low dose haloperidol.
Neurocognitive Disorders
K&S Chapter 21

159

Psychiatry Test Preparation and Review


Manual E-Book
Psychiatry Test Preparation & Review Manual

Question 10. C. Metoclopramide (Reglan) is a potent antiemetic agent that is a benzamide derivative, has phenothiazine-like
properties, and can cause extrapyramidal side effects (EPS) and akathisia. It is a potent antagonist of dopamine
type 2 (D2) receptors and blocks these receptors on the chemoreceptor trigger zone of the area postrema, which
prevents nausea and emesis. Because of its affinity for the D2 receptor, it has been known to cause EPS, espe-
cially at higher doses, and can also cause tardive dyskinesia after long-term use and discontinuation.

Ondansetron (Zofran) is a potent antiemetic, like metoclopramide, but its mechanism does not involve
D2, so it does not have the potential to cause EPS or tardive dyskinesia. It is a potent serotonin (5-HT) 3
antagonist and also works in the area postrema and probably on peripheral vagal nerve receptors.

Hydroxyzine (Atarax, Vistaril), like diphenhydramine (Benadryl), is an antihistamine that also has anal-
gesic and antiemetic properties. Is has no effect on dopamine receptors and therefore does not cause EPS
or tardive dyskinesia.

Tizanidine (Zanaflex) is a potent sedating muscle relaxant that works via α2 adrenergic agonism. It can
lower blood pressure, much like clonidine, which has this same mechanism of action. It does not affect
dopamine receptors and does not cause EPS or tardive dyskinesia.
Psychopharmacology
K&S Chapter 29

Question 11. D. Sleep attacks are the most common symptom of narcolepsy. Narcolepsy is also characterized by cataplexy (a
sudden loss of muscle tone) and hallucinations while falling asleep and waking up. Patients with narcolepsy
can have sleep paralysis in which they wake up and are often unable to move. Their sleep also characteristi-
cally goes into REM cycles when sleep begins. Treatments involve taking regular naps during the day, stimu-
lants, and antidepressants. Narcolepsy has been associated with low levels of CSF hypocretin (orexin).
Sleep–Wake Disorders
K&S Chapter 16

Question 12. A. The primary auditory cortex localizes to the superior temporal gyrus (Heschl’s gyrus) in both temporal
lobes. Cortical deafness can result from bilateral strokes to the temporal lobes that destroy Heschl’s gyri.
The thalamus is of course the relay station for much of the sensory input to the brain. The frontal lobes
are responsible for attention, concentration, set-shifting, organization and executive functioning, and
planning. The occipital lobes are the location of the calcarine cortex, which is responsible for interpret-
ing and processing visual input and stimuli.
Neurology
B&D Chapters 11 and 12

Question 13. E. Glycine is a neurotransmitter synthesized from serine. It is a necessary adjunctive neurotransmitter at the
NMDA receptor that binds with glutamate. It is also an independent inhibitory neurotransmitter with
its own receptors that open chloride ion channels. The activity of glycine on the NMDA receptor is an
area of research for schizophrenia, with some studies showing improvement in negative symptoms with
the use of glycine or glycine analogues. The highest concentrations of glycine receptors have been found
in the spinal cord. Mutations of this receptor lead to a rare condition called hyperekplexia, of which the
main symptom is an exaggerated startle response.
Basic Neuroscience
K&S Chapter 1

Question 14. E. Gabapentin and pregabalin are both U.S. Food and Drug Administration (FDA)-approved for the sys-
temic oral treatment of postherpetic neuralgia. The lidocaine transdermal patch (Lidoderm) is FDA-
approved for the topical and local treatment of postherpetic neuralgic pain. Carbamazepine and
oxcarbazepine are not FDA-approved, but both have been shown to be acceptable alternatives for the
treatment of trigeminal and postherpetic neuralgia. Phenytoin, an anticonvulsant agent, has no place in
the treatment of postherpetic neuralgia.
Neurology
B&D Chapters 44 and 67

160

Psychiatry Test Preparation and Review


Manual E-Book
Test Number Three

Question 15. E. This is a clear case of acute stress disorder. Acute stress disorder occurs after a person is exposed to
a traumatic event. The patient then feels anxiety, detachment, derealization, feelings of being “in a
daze,” dissociative amnesia, and numbing. Flashbacks and avoidance of stimuli can occur. The symp-
toms do not last longer than 4 weeks and occur within 4 weeks of the traumatic event (as opposed
to posttraumatic stress disorder [PTSD] in which symptoms must last more than 1 month). In disso-
ciative amnesia the patient has one or more episodes of inability to recall important personal infor-
mation, usually of a traumatic or stressful nature. The patient maintains intact memory for other
information. The prepared test-taker should be able to distinguish this from TGA, which is a revers-
ible anterograde and retrograde memory loss with retention of basic biographic information that usu-
ally occurs in elderly or middle-aged men, lasts several hours, and is likely to be related to a transient
ischemic attack.
Anxiety Disorders
K&S Chapter 11

Question 16. A. Epidural hematomas result most often from head trauma and skull fracture that cause a tear in the mid-
dle meningeal artery or one of its branches. They occur between the dura and the skull table and appear
as a lens-shaped hyperdensity on computed tomography scan of the brain. Most epidural hematomas are
localized to the temporal or parietal areas, but they can occur elsewhere in the brain as well.

Subdural hematoma is the most common intracranial hematoma, found in 20% to 25% of all traumatic
brain injury patients who are comatose. The subdural hematoma is believed to result from tearing of
the bridging veins over the cortical surface or from trauma to the venous sinuses or their tributaries.
Subdural hematoma is more common in patients over 60, particularly alcoholics, presumably because
the dura is tightly adhered to the skull table in this population. The other answer choices are distractors.
Neurology
B&D Chapter 50

Question 17. D. Although one could argue for bits and pieces of other diagnoses, when taking the whole picture into
consideration, this patient is a case of borderline personality disorder. Borderline personality disorder is
characterized by frantic efforts to avoid abandonment, unstable interpersonal relationships, disturbed
self-image, impulsive behavior, recurrent suicidality or self-mutilation, affective instability, chronic
feelings of emptiness, intense anger, and stress-related paranoia or severe dissociative symptoms. The
woman in the question clearly presents with many of these symptoms.
Personality Disorders
K&S Chapter 22

Question 18. B. Electroencephalograms of patients with Creutzfeldt–Jakob disease (CJD) classically demonstrate
periodic sharp-wave complexes. These high-amplitude sharp waveforms are present in up to 80% of
patients with CJD. This pattern can also be noted in subacute sclerosing panencephalitis, but with longer
interburst intervals.

The 3-per-second spike and wave pattern is indicative of absence seizures (petit mal epilepsy). Temporal
spikes would be indicative of a seizure focus in the temporal lobe, most often of the partial-complex
variety. Generalized background slowing can be noted in postictal states, coma, or delirium or after
anoxic brain injury. It usually indicates a decrease in the level of consciousness. PLEDs are characteristic
of herpes simplex encephalitis but can also be seen in acute hemispheric stroke, tumors, abscesses, and
meningitis.
Neurology
B&D Chapters 32 and 53

Question 19. E. DSM 5 allows for catatonia associated with another mental disorder (i.e., use of catatonia as a specifier),
catatonia due to another medical condition, or unspecified catatonia. Symptoms of catatonia include stu-
por, catalepsy, waxy flexibility, mutism, negativism, posturing, mannerisms, stereotypy, agitation, grimac-
ing, echolalia, and echopraxia. Catatonia can be used as a specifier with MDD, brief psychotic disorder,
schizophreniform disorder, schizophrenia, schizoaffective disorder, and bipolar disorder. It is not used

161

Psychiatry Test Preparation and Review


Manual E-Book
Psychiatry Test Preparation & Review Manual

as a specifier for OCD. The answer choice “hepatic encephalopathy” is an example of catatonia due to
a medical condition. One would document the diagnosis as “catatonic disorder due to hepatic encepha-
lopathy.” While on the topic of catatonia, also keep in mind that when treating a new onset catatonia the
first step is a trial of a benzodiazepine.
Psychotic Disorders
K&S Chapter 7

Question 20. D. Aseptic or viral meningitis tends to produce a CSF assay with mild-to-moderate lymphocytic pleocyto-
sis, normal glucose, and normal to mildly elevated protein. Gram stain and cultures would be negative.
A bacterial meningitis would produce a CSF as in answer choice C, with marked lymphocytosis, par-
ticularly polymorphonuclear neutrophils; markedly increased protein; and decreased glucose. A fungal
meningitis would produce a CSF as in answer choice B, with moderate lymphocytic pleocytosis, mildly
decreased glucose, and mildly increased protein. Tuberculous meningitis would produce a CSF with
mild lymphocytic pleocytosis, mildly decreased glucose, and markedly increased protein, as in answer
choice A. Herpetic meningitis would produce a CSF picture similar to that in aseptic meningitis but may
also reveal a predominance of red blood cells in the CSF. Answer choice E is a CSF assay indicative
of Guillain–Barré syndrome (acute inflammatory demyelinating polyneuropathy), which is character-
ized by a cytoalbuminologic dissociation with high protein (>55 mg/dL) and an absence of significant
pleocytosis.
Neurology
B&D Chapter 53

Question 21. C. This question focuses on Mahler’s stages of separation–individuation. The first stage is normal autism,
lasting from birth to 2 months. In this stage the baby spends more time asleep than awake. The next
stage is symbiosis, from 2 to 5 months. In this stage the baby is developing the ability to distinguish the
inner from the outer world. The child perceives itself as being part of a single entity with its mother. The
following stage is differentiation, from 5 to 10 months. Here the child is drawn further into the outside
world and begins to distinguish itself from its mother. Next is practicing. Practicing is from 10 to 18
months and is characterized by the baby’s ability to move independently and explore the outside world.
Practicing is followed by rapprochement between 18 and 24 months. In rapprochement the child’s inde-
pendence vacillates with its need for its mother. The child moves away from the mother, then quickly
returns for reassurance. Mahler’s last stage is object constancy, from 2 to 5 years. In this stage the child
understands the permanence of other people, even when they are not present. It would be wise to know
all of Mahler’s stages well. You never know where you might see them again!
Human Development
K&S Chapter 31

Question 22. B. Arsenic poisoning has both central and


peripheral nervous system manifestations.
Systemic manifestations include nausea,
vomiting, diarrhea, hypotension, tachycar-
dia, and vasomotor collapse that can lead
to death. Stupor or encephalopathy may
develop. Peripherally, arsenic causes a dis-
tal axonal neuropathy. Symptoms of the
neuropathy may develop 2 to 3 weeks after
initial exposure. Skin manifestations may
develop with more chronic exposure, par-
ticularly keratosis, melanosis, malignancies,
and characteristic Mees’ lines of the fingernails. Mees’ lines are white transverse striations of the finger-
nails that occur about 3 to 6 weeks after initial arsenic exposure. Mees’ lines can also be noted in thal-
lium poisoning and from chemotherapy exposure.

Lead poisoning occurs in those who work with metal, in soldering, in battery manufacturing, and in smelt-
ing factories. Children tend to develop an acute encephalopathy and adults a polyneuropathy. Children can

162

Psychiatry Test Preparation and Review


Manual E-Book
Test Number Three

become exposed to lead by ingestion of old paint that contains lead. They develop an acute gastrointestinal
illness and ultimately behavioral manifestations with confusion, drowsiness, generalized seizures, and intra-
cranial hypertension. In adults, the lead neuropathy manifests predominantly as a motor neuropathy, which
presents as bilateral wrist drop and/or foot drop. A rare sign of lead poisoning is the appearance of a blue line
at the gingival margin in patients with poor oral hygiene. Adults sometimes develop a gastrointestinal illness
and a hypochromic, microcytic anemia. Lead encephalopathy is managed supportively. Systemic corticoste-
roids can be given to reduce brain edema, and chelating agents like dimercaprol (British anti-Lewisite) are
also prescribed.

Mercury poisoning can occur from mercury vapor inhalation in the making of batteries, electronics
manufacturing, and in the past, the hat-making industry. Clinical manifestations include personality
changes (“mad as a hatter”), irritability, insomnia, drowsiness, confusion, and stupor. Other systemic
symptoms and signs include intention tremors (“hatter’s shakes”), proteinuria, glycosuria, hyperhidrosis,
and muscle weakness. Chronic exposure can lead to visual field deficits, sensory disturbances, progres-
sive ataxia, tremor, and cognitive impairment. Treatment using chelating agents does not always increase
rate or extent of recovery.

Organophosphate poisoning can occur from exposure to pesticides, herbicides, and flame retardants.
Organophosphates inhibit acetylcholinesterase by phosphorylation and result in cholinergic toxicity.
Clinical manifestations include salivation, lacrimation, nausea, bronchospasm, headache, weakness, and
in severe cases, bradycardia, tremor, diarrhea, pulmonary edema, cyanosis, and convulsions. Coma can
ensue, and death can result from respiratory or cardiac failure. The treatment of choice is pralidoxime
and atropine. Pralidoxime helps restore acetylcholinesterase, and atropine helps counteract muscarinic
adverse effects.

Ionizing radiation is a risk factor for CNS tumor formation, particularly meningiomas and nerve sheath
tumors. Electromagnetic radiation is a likely risk factor for both leukemia and brain tumor.
Poisoning
B&D Chapters 52 and 58

Question 23. E. This question covers some of the statistical terms that could be fair game for an examination. Learn them
well. A control group is a group that does not receive treatment and is a standard of comparison. ANOVA
is a set of statistical procedures that compares two groups and determines if the differences are due to
experimental influence or chance. Regression analysis is a method of using data to predict the value of one
variable in relation to another. The null hypothesis is the assumption that there are no differences between
two samples of a population. When the null hypothesis is rejected, differences between the groups are not
attributable to chance alone. Correlation coefficient will be covered in detail in future questions.
Statistics
K&S Chapter 5

Question 24. B. Organophosphates inhibit acetylcholinesterase and cause cholinergic toxicity. Organophosphates are
found predominantly in pesticides and herbicides, and poisoning generally occurs in agriculture work-
ers who are exposed after spraying in fields. Pralidoxime and atropine are the agents of choice to treat
organophosphate toxicity. The other answer choices are simply distractors. For a more detailed explana-
tion of the organophosphates, see Question 22.
Poisoning
B&D Chapter 58

Question 25. B. Emil Kraepelin used the term dementia praecox to describe schizophrenia. It referred to the early onset
of memory loss or decreased cognitive function often seen in patients with schizophrenia. Patients with
dementia praecox were found to have a long, deteriorating course subject to hallucinations and delu-
sions. It was Eugen Bleuler who coined the term schizophrenia. The term schizophrenia is often miscon-
strued to mean split personality, which in modern times is referred to as dissociative identity disorder.
Other answer choices given are unrelated distractors.
History of Psychiatry
K&S Chapter 7

163

Psychiatry Test Preparation and Review


Manual E-Book
Psychiatry Test Preparation & Review Manual

Question 26. E. Lead poisoning can cause a rare appearance of a blue line at the gingival margin in patients with poor
oral hygiene. Lead can also cause a microcytic, hypochromic anemia. A more detailed explanation of
lead poisoning appears in Question 22.

Manganese classically causes neurotoxicity after months or years of exposure. Manganese miners are
particularly at risk because of their prolonged inhalation of the toxin. Parkinsonism and motor symp-
toms can develop from manganese poisoning and usually follow initial manifestations of behavioral
changes, headache, and cognitive disturbances (“manganese madness”). A characteristic gait (walking on
the toes with spine erect and elbows flexed) called the “cock-walk” can emerge. The condition is gener-
ally poorly responsive to l-DOPA therapy.

Thallium poisoning results in a severe neuropathy and CNS degeneration. A chronic, progressive sensory
polyneuropathy can develop. Thallium causes potassium depletion, which can result in cardiac abnor-
malities, such as sinus tachycardia, T-wave changes, and U waves. Alopecia can develop 2 to 4 weeks
after initial exposure. Treatment with intravenous potassium chloride and oral potassium ferric ferrocya-
nide, as well as hemodialysis and forced diuresis, can help in recovery from acute thallium intoxication.
Arsenic and mercury poisonings are discussed in more detail in Question 22.
Poisoning
B&D Chapter 58

Question 27. C. DSM 5 has removed the DSM 4 diagnosis of shared psychotic disorder (folie-à-deux) in which a delu-
sion develops in an individual who is in a close relationship with someone who already has an estab-
lished delusion. In DSM 5, if criteria are met the wife would be diagnosed with delusional disorder. If
criteria are not met but shared beliefs are present, the diagnosis should be other specified schizophrenia
spectrum and other psychotic disorder.
Psychotic Disorders
K&S Chapter 7

Question 28. B. Ataxia results from cerebellar lesions. Midline cerebellar vermian lesions cause truncal ataxia. Head tremor
and truncal instability leading to oscillation of the head and trunk in a seated or standing posture (tituba-
tion) may result from lesions of the cerebellar vermis. In lateralized lesions to the cerebellar hemisphere,
signs and symptoms occur ipsilateral to the lesion. Cerebellar hemispheric lesions would be expected to
cause ipsilateral limb ataxia and/or dysmetria of either the arm or the leg, or both, depending on the loca-
tion of the lesion. Dysdiadochokinesis can result from a cerebellar hemispheric lesion. This refers to a defi-
cit in the ability to perform smooth, rapid, alternating movements of the hands or feet. A disturbance in
both rhythm and amplitude of these alternating movements can be noted with cerebellar lesions.

Lesions arising in the cerebellopontine angle area can result in cranial neuropathy, particularly to nerves
V, VII, and VIII. Manifestations include ipsilateral peripheral facial palsy (Bell’s palsy, VII nerve palsy);
ipsilateral facial numbness and weakness of masseter muscles (V nerve palsy); and ipsilateral hearing
loss, tinnitus, and vertigo (VIII nerve palsy).

Thalamic lesions can cause any number of deficits. Lacunar thalamic infarcts can lead to a pure sen-
sory stroke (contralateral to the lesion), or to a sensorimotor stroke if the lesion also invades the
internal capsule. Thalamic lesions can cause a rare disorder of central thalamic pain (again contralat-
eral to the lesion and usually in the extremities and/or the face) known as the thalamic syndrome of
Dejerine–Roussy.

Strokes or lesions to the midbrain can cause a variety of symptoms and syndromes. One such classic
presentation is Parinaud’s syndrome, which can result from a midbrain lesion arising from ischemia to
the posterior cerebral artery (PCA) penetrating branches. This manifests with supranuclear paresis of eye
elevation, eyelid retraction, skew deviation of the eyes, defective convergence and convergence–retrac-
tion nystagmus, and light-near dissociation. Another midbrain stroke syndrome is Weber’s syndrome,
also arising from ischemia in the PCA territory. There is a contralateral hemiplegia of the face, arm, and
leg and ipsilateral oculomotor (III nerve) paresis with a dilated fixed pupil.
Neurology
B&D Chapters 20 and 51

164

Psychiatry Test Preparation and Review


Manual E-Book
Test Number Three

Question 29. B. Here is another chance to understand glycine if you did not master it after Question 13. Glycine is a
neurotransmitter synthesized from serine. It is a necessary adjunctive neurotransmitter at the NMDA
receptor that binds with glutamate. It is also an independent inhibitory neurotransmitter with its own
receptors that open chloride ion channels. The activity of glycine on the NMDA receptor is an area of
research for schizophrenia, with some studies showing improvement in negative symptoms with the use
of glycine or glycine analogues. The highest concentrations of glycine receptors have been found in the
spinal cord. Mutations of this receptor lead to a rare condition called hyperekplexia, of which the main
symptom is an exaggerated startle response.
Basic Neuroscience
K&S Chapter 1

Question 30. C. Suspected meningitis with obtundation is not a contraindication to performing a lumbar puncture (LP).
All of the other answer choices are indeed absolute or relative contraindications to LP. A localized infec-
tion at the level of the puncture would be a contraindication to performing the procedure, owing to a
risk of the LP needle seeding the infection into a meningitis. Thrombocytopenia may lead to excessive
and uncontrollable bleeding if LP is performed. Fresh-frozen plasma or platelet transfusion may need
to be given before the procedure if the LP is deemed to be essential to further diagnosis and manage-
ment of the patient. Cerebral mass lesion is generally an absolute contraindication to LP because of the
risk of cerebral or cerebellar herniation after the procedure. Transtentorial herniation can arise if an LP
suddenly releases elevated CSF pressure, forcing the medial temporal lobe downward, and causing the
midbrain or the cerebellum to compress the cervicomedullary junction through the foramen magnum.
Papilledema is a sign of increased intracranial pressure and a possible mass lesion. Funduscopic exami-
nation must be conducted before LP is performed. Head trauma is also a contraindication to LP, because
it too may lead to a herniation syndrome due to increased intracranial pressure.
Neurology
B&D Chapter 31

Question 31. D. In case Question 15 didn’t solidify acute stress disorder and PTSD in your mind, take advantage of this
question to clarify your understanding. Acute stress disorder occurs when a person is exposed to a trau-
matic event. The patient then feels anxiety, detachment, derealization, feelings of being “in a daze,” dis-
sociative amnesia, and numbing. Flashbacks and avoidance of stimuli can occur. The symptoms do not
last longer than 4 weeks and occur within 4 weeks of the traumatic event (as opposed to PTSD, of which
symptoms must last more than 1 month).
Anxiety Disorders
K&S Chapter 11

Question 32. D. Fragile X syndrome is believed to be the most frequent cause of inherited intellectual disability. The
syndrome is characterized by moderate to severe intellectual disability, macro-orchidism, prominent jaw,
large ears, and jocular high-pitched speech. Hyperactivity and inattention are characteristic in affected
males with fragile X syndrome. The chromosomal anomaly lies at Xq28.

Down’s syndrome results generally from chromosomal nondisjunction that leads to a trisomy 21. This is
most often due to advanced maternal age. Manifestations include infantile hypotonia, hyperlaxity of the
joints, brachycephaly, flattened occiput, MR, upslanting palpebral fissures, flattened nasal bridge, epican-
thal folds, small ears, hypoplastic teeth, short neck, lenticular cataracts, speckling of the iris (Brushfield’s
spots), brachydactyly, simian creases, and congenital cardiac anomalies (in 30% to 40% of cases). Down’s
syndrome patients acquire Alzheimer-like dementia much earlier than the general population at large.

Trisomy 18 occurs in about 1 in 6000 live births. Fifty percent of infants do not survive past the first
week of life. Manifestations include low-set ears, small jaw, hypoplastic fingernails, intellectual disability,
cryptorchidism, congenital heart disease (patent ductus arteriosus, atrial septal defect, ventricular septal
defect), microcephaly, and renal anomalies, such as polycystic kidneys.

Cri-du-chat syndrome is a hereditary congenital intellectual disability syndrome caused by a deletion


at the short arm of chromosome 5p15.2. It occurs in 1 in 20,000 to 50,000 live births. Manifestations
include severe intellectual disability, microcephaly, round face, hypertelorism (increased distance between
the eyes), micrognathia, epicanthal folds, hypotonia, and low-set ears. Newborns present with a cat-like

165

Psychiatry Test Preparation and Review


Manual E-Book
Psychiatry Test Preparation & Review Manual

high-pitched cry that is considered to be diagnostic of the disorder. A majority of these patients do not
live past early childhood.

Prader–Willi syndrome is considered to be an autosomal dominant disorder resulting in a deletion to


chromosome 15q11–q13. Clinical stigmata include decreased fetal activity, obesity, intellectual disability,
hypotonia, short stature, hypogonadism, and small hands and feet. High caloric intake leads to diabetes
and cardiac failure in many patients, and many Prader–Willi syndrome patients do not survive past 25
to 30 years of age.
Neurodevelopmental Disorders
B&D Chapter 61

Question 33. D. Dissociation is a disturbance in which a person fails to recall important information. There are a num-
ber of dissociative disorders, including dissociative amnesia, dissociative fugue, and dissociative identity
disorder. In all of these situations the patient’s lack of recall is in excess of what could be explained by
ordinary forgetfulness. Used as a defense mechanism, the term dissociation represents an unconscious
process involving the segregation of mental or behavioral processes from the rest of the person’s psycho-
logical activity. It can involve the separation of an idea from its emotional tone, as one sees in conversion
disorder.
Dissociative Disorders
K&S Chapter 12

Question 34. B. Shaken-baby syndrome from child abuse can lead to death from intracranial, subarachnoid, and sub-
dural hemorrhages. These result from vascular shearing and tearing due to the violent back-and-forth
movement that results in cerebral trauma. Retinal hemorrhages are often noted as well in these cases.
Hemorrhages can lead to seizures and ischemic cerebral infarction due to vasospasm of intracranial ves-
sels. Ninety-five percent of severe intracranial injuries in children 1 year of age or younger are due to
child abuse. The other answer choices are of less pathological importance in cases of child abuse.
Neurology
B&D Chapter 51

Question 35. D. This is a case of normal bereavement. Crying, weight loss, decreased libido, withdrawal, insomnia, irri-
tability, and poor concentration and attention can all be part of normal bereavement. Keys to normal
bereavement are that suicidality is rare, the bereavement improves with social contacts, and it lacks
global feelings of worthlessness. In depression, one finds anger and ambivalence toward the deceased,
suicidality is common, and social contacts do not help; thus the person isolates. In addition, others find
the depressed person irritating or annoying, whereas the bereavement patient evokes sympathy from
others. In depression the patient may feel that he or she is worthless, which is not the case in bereave-
ment. With respect to the other answer choices, these symptoms have not been going on long enough for
dysthymic disorder. There are no anxiety symptoms described to argue for acute stress disorder. There is
no mania described, and the patient does not meet criteria for major depression.
Depressive Disorders
K&S Chapter 34

Question 36. D. Lower motor neuron signs include hypotonia, muscle atrophy, fasciculations, hyporeflexia, flaccidity,
muscle cramps, and marked motor weakness. These are considered to arise from lesions that are distal
(i.e., peripheral) to the anterior horn cells where the upper and lower motor neurons synapse. Upper
motor neuron signs include hyperreflexia, spasticity, Babinski’s sign, clonus, pseudobulbar palsy, loss of
dexterity, and mild motor weakness. Both upper and lower motor neuron signs can be demonstrated in
amyotrophic lateral sclerosis (Lou Gehrig’s disease), a disease that carries both sets of characteristics.
Poliomyelitis is a classic disease of the lower motor neurons, and it carries the manifestations of lower
motor neuron disease, as noted previously.
Neurology
B&D Chapter 76

166

Psychiatry Test Preparation and Review


Manual E-Book
Test Number Three

Question 37. B. Disorders that are frequently comorbid with social phobia include other anxiety disorders, affective dis-
orders, and substance abuse disorders. About one-third of patients with social phobia will meet criteria
for MDD. There is no significant comorbidity with the somatoform disorders in general, and conversion
disorder in particular.
Anxiety Disorders
K&S Chapter 9

Question 38. B. Brandt–Daroff exercises can be extremely helpful in the treatment of benign paroxysmal positional ver-
tigo (BPPV). The exercises involve rapidly lying down on one side from a seated position and remaining
there for about 30 seconds before sitting up for 30 seconds and repeating the maneuver in the oppo-
site direction. Patients are supposed to complete 20 full repetitions twice daily. Most patients see relief
within a week, but it may take 3 months or more to achieve complete symptom remission. Once in
remission, most patients are cured of the disorder.

BPPV is believed to be a result of sludge deposition or otoliths in the utricle of the posterior semicircu-
lar canal of the inner ear. When the patient moves his or her head, the movement of the material in the
semicircular canal irritates the hair cells in the inner ear and triggers a severe acute onset of rotational
vertigo with nausea, possible vomiting, and characteristic rotatory nystagmus. Diagnosis is clinical and
is suggested by the history. Symptoms can usually be evoked acutely by the Dix–Hallpike maneuver, in
which the patient’s head is rapidly lowered to the bed by the examiner. The head is then turned with one
ear down to the bed. The examiner notes the reproduction of vertigo and nystagmus, which are gener-
ally pathognomonic for BPPV. The fast phase of the nystagmus is noted in the direction of the lower
ear (the ear with the vestibular problem) when the patient looks toward the affected side. In primary
gaze, the fast rotational phase is vertical and upward, rotating toward the affected (lower) ear. Other
treatments include meclizine, an antiemetic agent that helps with nausea and vomiting but not generally
with the vertigo. Metoclopramide (Reglan) is also an antiemetic agent that can help in a similar way to
meclizine. Diazepam can lessen anxiety that can be associated with severe symptoms. Gabapentin is not
indicated, nor is it particularly useful, for BPPV.
Neurology
B&D Chapters 37

Question 39. B. All of this question’s answer choices are criteria for diagnosis of kleptomania, except that, before the
theft, the patient has an increased sense of tension. Kleptomania is found within the larger heading of
impulse control disorders. In kleptomania the patient engages in repeated stealing of objects that he or
she does not need. An important part of the disorder is the sense of tension before the act and the sense
of pleasure or relief afterward.
Disruptive, Impulse Control, Conduct Disorders, and ADHD
K&S Chapter 19

Question 40. B. The persistent vegetative state usually follows from a period of coma and is believed to signal the onset
of severe cerebral cortical damage, often due to brain anoxia, trauma, or both. A period of 1 month of
coma needs to elapse before the patient can be said to be in a persistent vegetative state. The condition
is characterized by absence of cognitive function and lack of awareness of the surrounding environment,
despite a preserved sleep–wake cycle. Spontaneous movements can be noted, and eye opening can be
preserved, but the patient does not speak and cannot obey commands. Eye tracking and swallowing
can also be preserved. There is no response to noxious stimulation in the persistent vegetative state.
Permanent irreversible brain damage is believed to set in after about 12 months into a persistent vegeta-
tive state that follows brain trauma and usually about 3 or more months after anoxic brain injury.
Neurology
B&D Chapter 5

Question 41. D. Ataque de nervios is a culture-bound anxiety syndrome associated with those from Latin American cul-
tures. Its symptoms include headache, insomnia, anorexia, fear, anger, despair, and diarrhea.

167

Psychiatry Test Preparation and Review


Manual E-Book
Psychiatry Test Preparation & Review Manual

The question says nothing about psychosis, so schizophrenia and schizoaffective disorder are incorrect. This case
does not meet criteria for panic disorder. Myoclonic sleep disorder is not even remotely related to the symptoms
given. Add in the fact that the patient is Hispanic, and the answer choice is clear: ataque de nervios.
Cultural Issues in Psychiatry
K&S Chapter 3

Question 42. D. Depression classically disrupts REM sleep patterns. The most typical effect of depression on sleep, how-
ever, is early morning awakening. Depression can also shorten REM latency (1 hour or less). Depression
can increase the total percentage of REM sleep and flip the predominance of REM from the early morn-
ing near awakening to the beginning of the night.
Depressive Disorders
K&S Chapter 8

Question 43. E. In pyromania the patient sets fires repeatedly because of the tension before the act and the relief after.
There is also a fascination with fire and its various uses. If the patient is setting fires for gain, such as
money or to make a political statement, then it is not a case of pyromania. One cannot make the diagno-
sis in the presence of conduct disorder, mania, or antisocial personality disorder. Pyromania is included
in the impulse control disorders.
Disruptive, Impulse Control, Conduct Disorders, and ADHD
K&S Chapter 19

Question 44. A. Apoptosis is the phenomenon of programmed cell death. Apoptosis can be triggered by exposure to antigens,
exposure to corticosteroids, or withdrawal of growth factors and cytokines. NMDA receptor channels, when
opened, lead to calcium influx into neuronal cells, and this triggers the apoptotic cascade known as hyper-
excitability and excitotoxicity that leads to neuronal compromise and demise. This mechanism of cellular
death is not well understood but is believed to be implicit in the mediation of such neurologic conditions as
epilepsy, stroke, and neurodegenerative diseases like Alzheimer’s dementia and amyotrophic lateral sclerosis.
Basic Neuroscience
B&D Chapters 41 and 60

Question 45. E. This is a case of intermittent explosive disorder. The patient has a history of violent outbursts that are
out of proportion to the severity of the situation in which they occur. He has no other signs or symptoms
that would suggest another Axis I or II diagnosis. He has no medical history that would suggest that he
could have a seizure disorder or that would otherwise explain his behavior.
Disruptive, Impulse Control, Conduct Disorders, and ADHD
K&S Chapter 19

Question 46. B. The treatment of choice for acute benzodiazepine overdose is flumazenil (Romazicon). Flumazenil is
administered intravenously and has a short half-life of 7 to 15 minutes. The initial dosage in suspected
benzodiazepine overdose is 0.2 mg IV over 30 seconds. An additional 0.3 mg can be given for more effi-
cacy. Further doses of 0.5 mg can be given to a maximum total of 3 mg. The most common serious side
effect of flumazenil administration is the onset of seizures, particularly in those patients who are depen-
dent on benzodiazepines or in those who have ingested large quantities of benzodiazepines.

Naloxone (Narcan) is administered in cases of acute opioid intoxication or overdose. Dimercaprol


(British anti-Lewisite) is a chelating agent that is administered in cases of acute lead poisoning and lead
encephalopathy. Atropine is a potent anticholinergic agent that is administered in cases of organophos-
phate or herbicide poisoning. Epinephrine injection is administered in cases of allergic anaphylactic
shock. Asenapine is a second-generation antipsychotic sold in the U.S. under the brand name Saphris,
indicated for schizophrenia and bipolar disorder. It is notable for only coming in a sublingual tablet
which must be placed under the tongue and allowed to dissolve. It has nothing to do with benzodiaz-
epine overdose and is a distractor in this question.
Psychopharmacology
K&S Chapter 29

168

Psychiatry Test Preparation and Review


Manual E-Book
Test Number Three

Question 47. D. All of the answer choices in this question are potential considerations for the differential diagnosis of
intermittent explosive disorder except OCD. Patients with OCD are not particularly likely to become
intermittently violent and destructive. Other things to include in the differential would be personality
change from a medical condition, oppositional defiant disorder, antisocial personality disorder, mania,
malingering, and schizophrenia.
Disruptive, Impulse Control, Conduct Disorders, and ADHD
K&S Chapter 19

Question 48. C. The collaborative care model involves consultation and collaborative management between primary care
physician and psychiatrist, with the goal of maximizing treatment in the primary care setting. For it to
work, both primary care physician and psychiatrist must be educated in the model. The goal is to meet
the patient’s psychiatric needs in the primary care setting. Primary care physicians can initiate care for
some disorders. With in-house psychiatric consultation, primary care physicians can better care for a
large number of patients. Some patients are too ill for primary care management and must be referred
to the psychiatrist for traditional care. For the system to work, communication between primary care
physician and psychiatrist is essential. In more severely ill patients a care manager can also be involved
to assist communication between providers and ensure continuity between primary care, inpatient and
outpatient psychiatry, substance abuse treatment, and residential settings.
Management in Psychiatry
MGH Comprehensive Clinical Psychiatry, Chapter 59

Question 49. C. Heat intolerance is a symptom of hyperthyroidism, not AIDS. The other symptoms in this question, such
as progressive dementia, personality changes, depression, and loss of libido, are all worth considering in
a patient with AIDS. More than 60% of AIDS patients have neuropsychiatric symptoms. They can also
show impaired memory and decreased concentration and may have seizures.
Neurocognitive Disorders
K&S Chapter 21

Question 50. A. Papez’ circuit connects the hippocampus with the thalamus, hypothalamus, and cortex. The pathway
includes all of the answer choices noted, plus the mammillothalamic tract and anterior nucleus of the
thalamus. The amygdala is not considered part of Papez’ circuit.
Basic Neuroscience
B&D Chapter 6

Question 51. B. Female sexual interest/arousal disorder involves lack of sexual interest/arousal in response to appropriate
sexual/erotic cues. It may manifest as an inability to attain an adequate lubrication–swelling response found
in normal sexual excitement. In female orgasmic disorder, there is a delay in or absence of orgasm approxi-
mately 75% to 100% of the time. Vaginismus is involuntary muscle spasm of the outer third of the vagina,
which interferes with sexual intercourse. Treatment of vaginismus involves systematic desensitization to teach
the patient to control relaxation of the musculature involved. Dyspareunia is recurrent genital pain associated
with intercourse in either a man or a woman. Vaginismus and dyspareunia are now covered in the DSM 5
diagnosis of genitopelvic pain/penetration disorder. There is no mention of substance abuse or medications in
the question, so substance/medication-induced sexual dysfunction would not be the best answer choice.
Sexual Dysfunction
K&S Chapter 17

Question 52. E. Benign familial neonatal convulsions are rare, autosomal, dominant hereditary disorders resulting from
a defect in voltage-gated potassium channels. Generalized tonic–clonic seizures occur after about the
third day of life and disappear spontaneously in most cases in a few weeks to months.

Lambert–Eaton myasthenic syndrome is a paraneoplastic autoimmune disorder affecting P/Q-type volt-


age-gated calcium channels at the motor neuron terminal. Malignant hyperthermia, hypokalemic peri-
odic paralysis, and familial hemiplegic migraine are all genetic disorders involving gene mutations that
result in abnormal voltage-gated calcium channels.
Neurology
B&D Chapter 64

169

Psychiatry Test Preparation and Review


Manual E-Book
Psychiatry Test Preparation & Review Manual

Question 53. C. The patient in this question has paranoid personality disorder. Symptoms not listed in this question
include reluctance to confide in others for fear that the information will be used maliciously against one,
perceiving attacks on one’s character that others do not see, and having recurrent suspicions regarding
the fidelity of sexual partners. The patient in the question does not present with the usual grouping of
positive and negative symptoms used to diagnose schizophrenia. Although there is a great deal of suspi-
ciousness present, she does not present with frank psychosis. There is an absence of bizarre beliefs on a
number of subjects, magical thinking, and excessive social anxiety, as would be expected in schizotypal
personality disorder. She does not exhibit ego-syntonic social isolation, as would be expected in schizoid
personality disorder. There is no suggestion of memory loss, as would be noted in dementia.
Personality Disorders
K&S Chapter 22

Question 54. A. Phenylketonuria (PKU) is an autosomal recessive heritable inborn error of metabolism resulting from
a deficiency in phenylalanine hydroxylase that can cause intellectual disability. The prevalence of the
disorder is about 1 in 10,000 to 20,000 live births. Phenylalanine levels increase dramatically, because
owing to the missing enzyme, it cannot be converted to tyrosine. The defect localizes to chromosome
12. Classic clinical features include microcephaly, a characteristic “mousy” odor, infantile spasms, and
light hair and skin pigmentation. Sensorineural deafness is not a feature of PKU. Diagnosis can be estab-
lished by measurement of blood levels of phenylalanine, which are elevated. The treatment of choice is a
phenylalanine-free diet, which must begin during gestation to prevent intellectual disability.
Neurodevelopmental Disorders
B&D Chapter 62

Question 55. C. The nigrostriatal tract projects from the substantia nigra to the corpus striatum. When the D2 receptors
in this tract are blocked, parkinsonian side effects emerge. This tract degenerates in Parkinson’s disease.
Choices A, B, and C are all tracts involved in some way with antipsychotic medications. The antipsy-
chotic medications affect both positive and negative symptoms of schizophrenia through the mesolim-
bic–mesocortical tract. They also lead to increased prolactin, amenorrhea, and galactorrhea through the
tuberoinfundibular tract. The caudate neurons have many D2 receptors as well, and they regulate motor
activity. With blockade of the caudate D2 receptors bradykinesia develops. With their overstimulation,
tics and extraneous motor movements develop.
Neurocognitive Disorders
K&S Chapter 1

Question 56. D. The photo depicts a classic Arnold–Chiari type I malforma-


tion. The Chiari type I malformation presents as a descent
of the cerebellar tonsils below the level of the foramen
magnum with or without forward displacement of the
medulla. This manifestation is believed to be caused by a
low intracranial pressure state. The defect is often accom-
panied by syringomyelia, syringobulbia, and hydrocepha-
lus. Common clinical features include headache, cranial
neuropathies, and visual disturbances. Some cases present
with motor and sensory complaints, particularly myelopa-
thy with a “shawl” distribution pattern of sensory deficit
over the shoulders due to the syrinx. Surgical decompres-
sion of the brainstem may be needed if the patient is highly
symptomatic. Chiari type II malformation has features similar to those of type I, with caudal displacement
of the medulla and fourth ventricle and the addition of a lumbar myelomeningocele.

Lissencephaly, also known as agyria, is a disorder of early neuroblast migration. It results from the
developmental failure of the gyri of the cerebral cortex. The cortex remains smooth and lacks the con-
volutions that are typical of normal neuronal migration and development. Schizencephaly is a disorder
associated with defective genetic expression of the EMX2 gene. It results in clefts in the cerebral hemi-
spheres. There are two types of schizencephaly: closed lip (the edges of the cleft are closed) and open lip

170

Psychiatry Test Preparation and Review


Manual E-Book
Test Number Three

(the edges of the cleft are open). Schizencephaly can be symmetrical or asymmetrical. Dandy–Walker
syndrome presents as a cystic enlargement or pouching of the fourth ventricle. In addition, the posterior
portion of the cerebellar vermis is hypoplastic or aplastic. Noncommunicating hydrocephalus invariably
results from this anomaly. Intellectual disability and spastic diplegia are frequent clinical manifestations.
Neurology
B&D Chapter 60

Question 57. D. Neuroleptic malignant syndrome is a life-threatening condition resulting from the use of antipsychotic
medications. Its symptoms are muscular rigidity, dystonia, akinesia, obtundation, or agitation. It also
involves autonomic instability, such as fever, sweating, unstable blood pressure, or unstable heart rate.
Patients are given supportive medical treatment, and medications such as dantrolene and bromocrip-
tine may be used. Mortality is around 10% to 20%. The syndrome is a result of dopamine blockade,
and some postulate it may be the result of a precipitous withdrawal of dopamine receptor stimulation.
The other answer choices would not present with the grouping of symptoms listed, and the fact that the
patient has schizophrenia and is in a psychiatric inpatient unit should point the test-taker in the direc-
tion of a side effect of antipsychotics.
Psychopharmacology
K&S Chapter 29

Question 58. C. About 15% of ischemic strokes are caused by extracranial internal carotid artery (ICA) stenosis.
Endarterectomy, surgical removal of the atherosclerotic plaque, has been deemed to be indicated and
useful in symptomatic ICA stenosis of greater than 70%. Symptomatic stenosis implies the prior occur-
rence of ipsilateral ischemic events, such as transient ischemic attacks, amaurosis fugax, or completed
nondisabling carotid territory stroke within the past 6 months. Asymptomatic ICA stenosis is a more dif-
ficult and controversial issue with respect to its optimal surgical management.
Neurology
B&D Chapter 51

Question 59. E. Amoxapine is the most likely of the tricyclic antidepressants to cause parkinsonian symptoms. It can
also cause akathisia or dyskinesia. This is because its metabolites have dopamine-blocking activity. It can
even cause neuroleptic malignant syndrome in rare cases. Tricyclics are contraindicated in patients with
cardiac conduction deficits. They should be stopped before elective surgery because of the risk of hyper-
tension when tricyclics are given concomitant with anesthetics. Fludrocortisone can help some patients
with orthostatic hypotension. Myoclonic twitches and tremors can be seen in patients on desipramine
and protriptyline.
Psychopharmacology
K&S Chapter

Question 60. D. Campylobacter jejuni is the most common bacterial infection that precedes the onset of Guillain–Barré
syndrome (acute inflammatory demyelinating polyneuropathy [AIDP]). It accounts for about 20% to
40% of all cases of AIDP. Campylobacter jejuni causes an acute enteric and systemic illness, and the
onset of AIDP occurs about 2 to 3 weeks after this initial diarrheal condition. AIDP tends to follow
in about 1 in 1000 to 2000 cases of known C. jejuni infections. Campylobacter jejuni infection can be
diagnosed by stool or blood cultures and is treated with a week-long course of oral erythromycin.

AIDP is preceded by an upper respiratory or gastrointestinal infection, a surgical intervention, or an


immunization about 1 to 4 weeks before the illness onset in about two-thirds of cases. Campylobacter
jejuni infection has been linked to a worse prognosis because it tends to be associated with the more
severe axonal form of AIDP.
Neurology
B&D Chapters 53 and 76

Question 61. C. The correlation coefficient is a measurement of the direction and strength of the relationship between
two variables. The Pearson correlation coefficient is on a scale from −1 to +1. A positive correlation
means that one variable moves the other in the same direction. A negative value means that one moves

171

Psychiatry Test Preparation and Review


Manual E-Book
Psychiatry Test Preparation & Review Manual

the other in the opposite direction. A correlation close to −1 or +1 shows a strong relationship. A cor-
relation close to 0 shows a weak relationship. Correlation coefficients indicate the degree of relationship
only; they say nothing about cause and effect.
Statistics
K&S Chapter 5

Question 62. D. Brain metastases are the most common form of neurologic complication from systemic cancer, and they
account for about 20% to 40% of all CNS tumors. Lung cancer is the most common cause of metasta-
ses to the brain and accounts for about two-thirds of all such metastases. In particular, non-small-cell
lung carcinoma accounts for about two-thirds of all metastases to the brain originating from the lung.
Breast cancer leads to about 15% to 20% of patients with brain metastases. Melanoma, gastrointestinal
carcinoma, and renal cell carcinoma each account for about 5% to 10% of metastatic brain tumors.
Neurology
B&D Chapter 52

Question 63. B. This question gives a description of avoidant personality disorder. These patients avoid interpersonal con-
tact for fear of criticism. They are unwilling to get involved with others without assurance of being liked.
They show restraint in interpersonal relationships for fear of being shamed or ridiculed. They are inhib-
ited interpersonally because of fears of inadequacy. They see themselves as inferior to others. Of the other
choices, the only one that comes close is schizoid personality disorder, but in schizoid personality disor-
der the person doesn’t care that he is isolated, and he has a blunted affect. OCPD presents as a perva-
sive pattern of preoccupation with orderliness, perfectionism, and control. This preoccupation comes at
the expense of openness, efficiency, and flexibility. The OCPD patient’s perfectionism interferes with task
completion. These patients are inflexible regarding moral and ethical issues. They devote time to work at
the expense of leisure activities. They are reluctant to delegate tasks to others. They are characteristically
rigid and stubborn. OCPD patients often cannot discard old or worn-out objects, even when they have no
value. Patients with histrionic personality disorder show a pattern of excessive emotionality and attention
seeking characterized by inappropriate sexually seductive or provocative behavior, use of physical appear-
ance to draw attention to self, self-dramatization, theatricality, and exaggerated expression of emotion.
Personality Disorders
K&S Chapter 22

Question 64. A. A post-LP headache is a frequent adverse event resulting from diagnostic or therapeutic LP. The head-
ache is considered to be due to low CSF pressure because the needle bevel leaves an open dural tear from
shearing as it is pulled out of the patient. This dural tear results in a chronic CSF leak from the puncture
site and an intractable headache that is noted particularly when the patient is upright. The headache is
generally relieved when the patient lies recumbent. Loss of CSF results in a traction of the brain on sen-
sory nerves and bridging veins, which causes pain.

The pain often resolves in several days if the patient lies recumbent and receives adequate hydration. A
blood patch is often immediately curative of the post-LP headache and involves the gentle injection of
10 to 20 mL of the patient’s own blood into the epidural space at the same site of the LP. Bed rest for
2 weeks is far too long a period to wait before undertaking a blood patch. Acetaminophen and hydro-
codone may be helpful but are not curative therapy for post-LP headache. Sumatriptan is indicated for
migraine only and not for post-LP headache.
Neurology
B&D Chapter 69

Question 65. E. Patients starting lithium should obtain baseline thyroid function tests, electrolytes, WBC count, renal
function tests, and a baseline electrocardiogram (ECG). Why? Because lithium can cause renal damage,
hypothyroidism, increased WBC count, and ECG changes (T-wave flattening or inversion). Low sodium
can lead to toxic lithium levels. VDRL is a distractor that is unrelated to starting a patient on lithium.
Laboratory Tests in Psychiatry
K&S Chapter 29

172

Psychiatry Test Preparation and Review


Manual E-Book
Test Number Three

Question 66. B. Compression of the lateral femoral cutaneous nerve as it passes beneath the inguinal ligament results in a
painful sensory syndrome known as meralgia paresthetica. Predisposing factors include obesity, pregnancy,
and the wearing of pants that are tight at the waist or a heavy belt, such as those worn by workmen carry-
ing heavy tools. Clinically there is pain and sensory loss (numbness) to the lateral thigh. Motor deficits are
absent. Treatment is conservative and involves weight loss (if that is the cause), wearing looser clothing, or
the use of tricyclic antidepressants or anticonvulsants to treat the neuropathic pain. Surgery is not generally
needed in these cases. The other peripheral nerves listed as answer choices are simply distractors.
Neurology
B&D Chapter 30

Question 67. A. Your new friend Mahler is back again! If you didn’t get this right, it’s time to memorize her stages!
Mahler’s first stage is normal autism, lasting from birth to 2 months. In this stage the baby spends more
time asleep than awake. For more information on Mahler’s stages, see Question 21.
Human Development
K&S Chapter 31

Question 68. D. Acute intermittent porphyria is a disorder of heme synthesis leading to the accumulation of porphy-
rins and their metabolites in urine, blood, and stool. Patients present with abdominal pain, peripheral
neuropathy, and altered mental status. Psychiatric symptoms include anxiety, depression, psychosis, and
delirium. Barbiturates and anticonvulsants can precipitate symptoms.
Neurocognitive Disorders
K&S Ch. 21

Question 69. A. The caudate nucleus neurons have many D2 receptors. They regulate motor activity by determin-
ing which motor acts get carried out. With blockade of the caudate D2 receptors, bradykinesia devel-
ops from excessive dampening of motor activity. With caudate D2 receptor overstimulation, tics and
extraneous motor movements develop. This is important to understanding the role of the caudate in
Parkinson’s, Tourette’s and tics.
Basic Neuroscience
K&S Chapter 1

Question 70. B. Cerebral venous thrombosis (CVT) generally occurs 1 day to 4 weeks postpartum, with a peak in inci-
dence about 7 to 14 days after delivery. Clinical features include a puerperal headache that worsens over
days, seizures, neurologic deficits, and behavior or personality changes. The condition is believed to be due
to the hypercoagulable state induced by pregnancy with likely decreased protein S activity and to a possible
presence of circulating antiphospholipid antibodies during the puerperal period. Brain magnetic resonance
imaging (MRI) with MRI venography is the imaging modality of choice to clinch the diagnosis. Heparin anti-
coagulation may be helpful. Thrombolysis by either intravenous infusion or invasive venographic approach
has also proved to be useful treatment, especially in severe cases in which prognosis is deemed to be poor.

Meningitis and meningoencephalitis are certainly in the differential diagnosis when considering CVT,
but both would be expected to be accompanied by high fever and possibly obtundation. A migraine,
even when complicated, is rarely accompanied by seizures. Pseudotumor cerebri does not cause seizures
and usually presents with intermittent visual obscurations and papilledema rather than diplopia.
Neurology
B&D Chapter 81

Question 71. A. Patients must make decisions regarding advanced directives at a time when they are competent to do
so. At the time of admission to the hospital they may or may not be competent. They may or may not
be competent immediately before an operation or in the office. Competence is a legal decision made by
the courts that a patient has sufficient ability to manage his or her own affairs. Capacity is a medical
decision made by a psychiatrist that says whether at a given point in time the patient is thinking clearly
enough to make certain medical decisions for him- or herself. To make an advanced directive, one would
want the patient to both have capacity and be competent at the time of the decision.
Forensic Psychiatry
K&S Chapter 36

173

Psychiatry Test Preparation and Review


Manual E-Book
Psychiatry Test Preparation & Review Manual

Question 72. A. CMV is the most common congenital viral infection in newborns and is the result of either a primary
maternal infection or a viral reactivation in the mother. Most affected newborns are asymptomatic and
most develop normally. Less than 10% of patients with CMV have complications such as jaundice, hep-
atosplenomegaly, microcephaly, chorioretinitis, ataxia, and seizures. The mortality rate is about 20%
to 30% in symptomatic newborns. The other neonatal infections noted in this question occur less fre-
quently than CMV.
Neurology
B&D Chapter 80

Question 73. D. Patients with dependent personality disorder have trouble making decisions without excessive amounts
of advice from others. They need others to assume responsibility for most areas of their lives. They do
not express disagreement with others for fear of disapproval. They feel helpless and uncomfortable when
alone. They urgently seek new relationships when a prior one ends and can be unrealistically preoc-
cupied with fears of being left alone to care for themselves. None of the other answer choices fit these
parameters.
Personality Disorders
K&S Chapter 22

Question 74. D. Inclusion body myositis is the most common myopathy in those over 50 years of age, and it rarely
occurs in those younger than 50 years of age. Men are more frequently affected than women. The dis-
order affects strength in the distal muscles of the arms and legs. Wrist and finger flexors and quadri-
ceps muscles are preferentially weak. There are no muscle pains noted in the disorder. The disorder is
generally chronic, progressive, and poorly responsive to corticosteroid or immunosuppressive therapies.
Muscle biopsy helps confirm the diagnosis in about 80% or more of cases. It classically reveals endo-
mysial inflammation, macrophage invasion of muscle, rimmed vacuoles, and characteristic inclusion
bodies in the nuclei.

Facioscapulohumeral dystrophy (FSHD) is an autosomal dominant inherited disease of muscle. The


genetic anomaly localizes to chromosome 4 in most cases. The phenomenon of anticipation is noted in
FSHD, which implies that as successive generations acquire the disease, the onset of the condition occurs
earlier and the disease becomes more severe. Clinical features include weakness of orofacial muscles,
leaving a patient unable to pucker or whistle. Shoulder muscles are weak, and winging of the scapula
can be noted when the arms are outstretched. Biceps and triceps are often weak, as are the hip flexors
and quadriceps. DNA studies establish the diagnosis. Treatment is supportive because the condition is
irreversible.

Oculopharyngeal muscular dystrophy is an autosomal dominant inherited disease of muscle. The disor-
der localizes to chromosome 14. The disease most often begins in the fifth or sixth decade and presents
initially with eye muscle weakness and ptosis. Difficulty swallowing soon follows, and swallowing may
become impossible. Death can occur from starvation if nutritional support is not given. DNA testing
proves the diagnosis. Treatment is supportive.

Dermatomyositis and polymyositis are discussed at length in other questions in this volume.
Neurology
B&D Chapter 79

Question 75. C. Euphoria is a behavioral effect of opioids, as are drowsiness, decreased sex drive, hypoactivity, and per-
sonality changes. Miosis is a physical effect of opioid use. Increased arousal is not an effect of opioid
intoxication, drowsiness is. Diarrhea can come from withdrawal, but opioids themselves cause constipa-
tion. Bradycardia is also a physical effect of opioids, and as such tachycardia is incorrect.
Substance Abuse and Addictive Disorders
K&S Chapter 20

Question 76. A. Duchenne’s and Becker’s muscular dystrophies are X-linked inherited disorders of muscle. The gene
locus is Xp21 on the short arm of the X chromosome. This abnormality results in a deficiency in dystro-
phin, a structural muscle membrane protein located in muscle fibers. These two disorders are explained

174

Psychiatry Test Preparation and Review


Manual E-Book
Test Number Three

in greater detail as part of other questions in this volume. The other answer choices are all muscle pro-
teins that work together with dystrophin to stabilize the muscle membrane, but they are not implicated
in the pathophysiology of the two muscular dystrophies mentioned in this question.
Neurology
B&D Chapter 79

Question 77. E. Sleep apnea is a disorder in which there is a cessation of airflow in and out of the lungs during sleep.
These stoppages of airflow must last for 10 seconds or more. In central sleep apnea, both respiratory
effort and airflow stop. In obstructive sleep apnea, air stops flowing, but respiratory effort increases.
It is considered pathological if patients have five or more apneic episodes per hour or 30 or more epi-
sodes per night. Sleep apnea can lead to cardiovascular changes, including arrhythmia and blood pres-
sure changes. Long-standing sleep apnea can lead to pulmonary hypertension. The characteristic pattern
of sleep apnea involves an older person who reports tiredness or inability to stay awake during the
daytime. It can be associated with depression, irritability, and daytime sleepiness. Bed partners often
report loud snoring. Patients may also awaken during the night as a result of the cessation of breath-
ing. Patients suspected of sleep apnea should undergo sleep studies and should be treated with a CPAP
(continuous positive airway pressure) machine. Losing weight helps many people, and for some, surgery
is the appropriate option to remove the obstruction in the airway. Now, looking at the test question, the
patient is not dropping into “sleep attacks” while in the middle of activities, and as such narcolepsy is
incorrect. There is no substantial evidence of mania given. The patient does not meet criteria for major
depression. Behaviors associated with sleep terrors are not described. The snoring that drove her hus-
band out of the bedroom should immediately point you in the direction of sleep apnea. The fact that she
is overweight clinches the diagnosis.
Sleep–Wake Disorders
K&S Chapter 16

Question 78. B. Tyrosine hydroxylase is the rate-limiting enzyme in the dopamine synthetic pathway. Dopamine synthe-
sis occurs as follows: l-tyrosine is converted to l-DOPA by tyrosine hydroxylase. DOPA decarboxylase
converts l-DOPA to dopamine. Once dopamine is extruded into the synaptic cleft, its termination of
action is carried out by monoamine oxidase catechol-O-methyltransferase, as is its reuptake into the
presynaptic bouton where it is initially synthesized. Norepinephrine synthesis occurs when dopamine
β-hydroxylase converts dopamine to norepinephrine. Norepinephrine in turn is converted to epinephrine
by PNMT.
Basic Neuroscience
B&D Chapter 71

Question 79. C. The therapeutic range for lithium is 0.6 to 1.2 mEq/L. Toxic levels are 2 mEq/L or higher. Lethal levels
are 4.0 mEq/L or higher. Bonus Fact: For a patient who needs regular dialysis and for whom lithium is
the best medication…give a dose of lithium immediately after the dialysis.
Laboratory Tests in Psychiatry
K&S Chapter 29

Question 80. D. GABA-A is a complex receptor with multiple binding sites. GABA is found throughout the central and
peripheral nervous systems and is the predominant inhibitory neurotransmitter in the brain. When the
GABA receptor is occupied by an agonist, there is a rapid influx of negatively charged chloride ions
through the postsynaptic cellular membrane. This results in fast inhibitory postsynaptic potentials. The
GABA-A receptor is believed to be responsible for the clinical effects of benzodiazepines, barbiturates,
and alcohol. Only sodium oxybate (γ-hydroxybutyrate; Xyrem), which is a “date-rape” drug that is
FDA-approved for narcolepsy and cataplexy, and Lioresal, a potent antispasticity agent, act in the CNS
by agonism of the GABA-B receptor. The other receptor types mentioned in this question are simply
distractors.
Neurology
Basic Neuroscience
B&D Chapter 71

175

Psychiatry Test Preparation and Review


Manual E-Book
Psychiatry Test Preparation & Review Manual

Question 81. D. It is the frontal lobes that determine how the brain acts on information. The relatively large size of the
human frontal lobes is what distinguishes our brains from those of other primates. It is in the frontal
lobes that executive functioning takes place, and injury of the frontal lobes leads to impairment in moti-
vation, attention, and sequencing of actions. A “frontal lobe syndrome” exists and consists of slowed
thinking, poor judgment, decreased curiosity, social withdrawal, and irritability. Patients with frontal
lobe dysfunction may have normal IQ, because IQ has been found to be mostly a parietal lobe function.
Basic Neuroscience
K&S Chapter 1

Question 82. B. Ganser’s syndrome, considered to be a dissociative disorder, is the voluntary production of symptoms
that involve giving approximate answers or talking past the point. This syndrome is often associated
with other psychopathies, such as conversion, perceptual disturbances, and dissociative symptoms, like
amnesia and fugue. Males and prisoners are most commonly affected. The major contributory factor is
the presence of a severe personality disorder. Recovery is most often sudden, and patients claim amnesia
of the symptoms. It is believed to be a variant of malingering, with possible secondary gain.

Conversion disorder (functional neurological symptom disorder) is one of the somatic symptom
and related disorders characterized by the presence of one or more neurologic symptoms that are not
explained by any known neurologic or medical disorder. The symptoms are judged to be caused by psy-
chological factors, and the illness is often preceded by conflicts or other stressors. Capgras’ syndrome
is a specific type of systematized delusion in which the patient mistakenly feels that a familiar person is
an unfamiliar imposter. Illness anxiety disorder is one of the somatic symptom and related disorders in
which the patient misinterprets bodily symptoms and functions and becomes preoccupied with the fear
of contracting or having a serious disease even after reassurance to the contrary is given by a physician.
Folie-à-deux is a psychotic disorder that involves the transfer of delusions from a patient to another per-
son who has a close relationship with the patient. The associated person’s delusion is similar in content
to that of the patient. Folie-a-deux was removed as a separate diagnosis in DSM 5 and would now fall
under “other specified schizophrenia spectrum and other psychotic disorders”.
Somatic Symptom Disorders
K&S Chapter 12

Question 83. D. Learned helplessness is a behavioral model for depression developed by Martin Seligman. He took dogs and
gave them electric shocks from which they could not escape. Eventually they gave up and stopped trying to
escape. In time this spread to other areas of functioning, until they were always helpless and apathetic. This
behavioral pattern has also been seen in humans with repeated setbacks or failures in their lives, as the ques-
tion stem demonstrates. Industry is part of Erikson’s stages and is irrelevant to the question. Cognition is the
process of obtaining, learning, and using intellectual knowledge. This has some relation to the test taking
but is not the explanation for the child’s behavior. Sensory deprivation is removing a person or animal from
external stimuli of any kind. Again, it is unrelated to the question. The epigenetic principle states that devel-
opment occurs in sequential, clearly defined stages. This is clearly unrelated to the question.
Psychological Theory and Psychometric Testing
K&S Chapter 3

Question 84. A. Entacapone (Comtan) and tolcapone (Tasmar) are catechol-O-methyltransferase (COMT) inhibitors.
COMT inhibitors block peripheral degradation of peripheral levodopa and central degradation of
l-DOPA and dopamine, thereby increasing l-DOPA and dopamine levels centrally. The COMT inhibitor
is generally given concomitant with each dose of carbidopa–levodopa (Sinemet) throughout the day to
improve parkinsonian symptoms.

Selegiline (Eldepryl) is the classic monoamine oxidase type-B (MAO-B) inhibitor. It has some poten-
tiating effects on dopamine and prevents MAO-B-dependent dopamine degradation. Phenelzine
(Nardil) and tranylcypromine (Parnate) are nonspecific MAO inhibitors that affect both MAO-A and
MAO-B. There are no specific pharmacologic agents that act upon DOPA decarboxylase or dopamine
β-hydroxylase. These two answer choices are simply distractors.
Neurology
B&D Chapter 71

176

Psychiatry Test Preparation and Review


Manual E-Book
Test Number Three

Question 85. D. The commonly used drug ibuprofen can drastically increase lithium levels. Many diuretics can increase
lithium levels, particularly thiazide diuretics, as can angiotensin-converting enzyme inhibitors and
other nonsteroidal anti-inflammatory drugs such as naproxen. Keep in mind, however, that the diuretic
amiloride can actually be used to treat lithium induced nephrogenic diabetes insipidus. Aspirin will not
affect lithium levels. Lithium combined with anticonvulsants can increase neurotoxic effects.
Laboratory Tests in Psychiatry
K&S Chapter 29

Question 86. C. Argyll Robertson pupils are a characteristic of late syphilis, particularly in either general paresis or tabes
dorsalis. Argyll Robertson pupils are small, irregular pupils that constrict to accommodation but not
to light. Tabes dorsalis is the spinal form of syphilis and develops about 15 to 20 years after the initial
infection. The clinical triad is that of sensory ataxia, lightning pains, and urinary incontinence. Lower
extremity deep tendon reflexes are absent. There is impaired proprioception with a positive Romberg’s
sign. Ninety percent of patients have pupillary abnormalities, and about one-half have Argyll Robertson
pupils. Another classic characteristic is the presence of Charcot’s (neuropathic) joints.

The pupillary abnormality seen in optic neuritis or multiple sclerosis is called the Marcus Gunn pupil (rela-
tive afferent pupillary defect). It can be observed with the swinging flashlight test. When the light is applied
to the normal eye, we see brisk bilateral constriction. When the light is applied to the affected eye, we see
a lesser constriction with an initial relative dilation. Lyme disease and bubonic plague do not present with
characteristic pupillary abnormalities. Intracerebral aneurysm, particularly of the posterior communicating
artery, presents with a complete third nerve palsy that involves the pupil, which is fixed and slightly dilated.
Neurology
B&D Chapter 53

Question 87. E. In a situation in which you are hired by an employer to evaluate an employee, it is not presumed that
your evaluation will remain confidential. It is understood that you will disclose information from the
evaluation to the patient’s employer. The patient should be made aware of this at the beginning of the
interview. The patient does not have a right to expect the entire evaluation to be confidential, but he or
she does have the right to refuse to cooperate with the evaluation. Despite this, it is not as straightfor-
ward as answer choices A or B. The employer does not have a right to elements of the patient’s history
that do not have bearing upon his or her work performance. You should attempt to maintain confidenti-
ality to the extent that it is possible. That would involve disclosing only the information that is truly nec-
essary and relevant in the situation. For example, details about the patient’s sexuality that have nothing
to do with his or her work performance should not be reported to the employer. Medical diagnoses that
have no bearing on the patient’s work performance should not be reported to the employer.
Ethics
K&S Chapter 36

Question 88. B. Todd’s paralysis is a brief period of transient hemiparesis or hemiplegia after a seizure. The symptoms
usually dissipate within 48 hours, and treatment is expectant and supportive. The weakness is generally
contralateral to the side of the brain with the epileptic focus. The condition may also affect speech and
vision, but again the deficits are temporary. Certain studies have pointed to Todd’s paralysis being the
result of arteriovenous shunting that leads to transient cerebral ischemia after an ictal event. The other
answer choices are distractors.
Neurology
B&D Chapter 67

Question 89. A. The 3-hydroxy benzodiazepines are directly metabolized by glucuronidation and have no active metabo-
lites. The 3-hydroxy benzodiazepines include oxazepam, lorazepam, and temazepam. Know these three
well, because they are often the subject of questions on standardized examinations. Some of the longest
half-lives are found with the 2-keto benzodiazepines (chlordiazepoxide, diazepam, prazepam) because
they have multiple active metabolites that can keep working in the body from 30 to more than 200
hours in patients who are slow metabolizers.
Psychopharmacology
K&S Chapter 29

177

Psychiatry Test Preparation and Review


Manual E-Book
Psychiatry Test Preparation & Review Manual

Question 90. A. Tropical spastic paraparesis is a chronic progressive myelopathy associated with infection by HTLV-1. The
condition affects men more often than women, and the onset is usually after 30 years of age. Diagnosis is
confirmed by CSF polymerase chain reaction and detection of HTLV-1 antibodies. Clinical features include
upper motor neuron weakness, bladder disturbance, and variable sensory loss. By 10 years out, 60% to
70% of patients cannot walk. Treatment can be undertaken with corticosteroids, interferon-α, or plasma-
pheresis but has proven only minimally beneficial.
Neurology
B&D Chapter 74

Question 91. D. Lithium is often used to treat aggression in patients with schizophrenia, prisoners, those with conduct
disorder, and intellectually disabled. It is less useful in aggression associated with head trauma and epi-
lepsy. Other drugs used for aggression include anticonvulsants and antipsychotics.
Psychopharmacology
K&S Chapter 29

Question 92. C. The neurodegenerative disorders that are associated with expansion of genetic trinucleotide repeat
sequences are fragile X syndrome; myotonic dystrophy; Huntington’s disease; X-linked spinobulbar
muscular atrophy; dentatorubral-pallidoluysian atrophy; spinocerebellar atrophies types 1, 2, 3, 6, and
7; and Friedrich’s ataxia. Multiple system atrophy is a Parkinson’s plus syndrome that is not associated
with an expansion of trinucleotide repeat sequences.
Neurology
B&D Chapter 40

Question 93. B. The mesolimbic–mesocortical pathway projects from the ventral tegmental area to many areas of the
cortex and limbic system. This is the tract that is thought to mediate the antipsychotic effects of the anti-
psychotic medications. The nigrostriatal pathway is associated with parkinsonian effects of the antipsy-
chotics. It should be noted that the caudate is innervated by dopaminergic neurons from the substantia
nigra pars compacta and thus plays a role in Parkinson’s disease as well. The caudate is also associated
with Tourettes Syndrome and tics (see Question 69). The tuberoinfundibular pathway is associated with
prolactin increase and lactation from antipsychotics.
Basic Neuroscience
K&S Chapter 1

Question 94. A. The “battle sign” is a hematoma overlying the mastoid that results from a basilar skull fracture extend-
ing into the mastoid portion of the temporal bone. The lesion is not usually visible until 2 to 3 days
after the trauma. Frontal lobe damage would be expected to yield classic frontal lobe signs on examina-
tion, such as a Myerson’s sign, rooting reflex, snout reflex, palmomental reflex, and grasp reflex. These
signal extensive damage to the frontal lobe (or lobes if the sign is bilateral). Increased intracranial pres-
sure presents with obtundation of level of consciousness, papilledema, and signs of brainstem compro-
mise. Hypocalcemia that is chronic may result in the clinical observation of a Chvostek sign. The sign is
positive when the cheek is tapped with the examiner’s finger and the corner of the mouth involuntarily
contracts. Impending cerebral herniation usually presents with all of the signs of increased intracranial
pressure noted previously. Systemic hypertension and respiratory compromise are also often noted.
Neurology
B&D Chapters 5 and 56

Question 95. B. Fluoxetine and phenelzine should not be combined, because one is an SSRI and the other is an MAOI.
SSRIs should not be combined with MAOIs because of the possibility of causing a fatal serotonin syn-
drome. Using an SSRI with selegiline, which selectively inhibits MAO-B at low doses, is tolerated by
some patients. But the general rule to remember is not to mix SSRIs and MAOIs. If you give one fol-
lowed by the other, there must be a washout period in between. It would be a good idea for the well-
prepared test-taker to know the symptoms of a serotonin syndrome and how to distinguish it from
neuroleptic malignant syndrome. Both conditions are covered elsewhere in this volume.
Psychopharmacology
K&S Chapter 29

178

Psychiatry Test Preparation and Review


Manual E-Book
Test Number Three

Question 96. D. Atomoxetine (Strattera) is FDA-approved for symptoms of ADHD in both adults and children. Its mech-
anism of action is by norepinephrine reuptake inhibition. It is the most useful choice when trying to treat
both tics and ADHD, because it does not worsen the tic condition and may in fact help the tic symptoms.
Bupropion (Wellbutrin) is not specifically FDA-approved for ADHD, and its use in the disorder has been
shown to have some limited benefit. Because results are mixed and it does not have an FDA approval for
ADHD, it would not be the best choice in this question. The amphetamine-like stimulant medications
methylphenidate (Ritalin, Metadate, Concerta) and dextroamphetamine (Dexedrine), although approved,
well-studied, and efficacious in ADHD, have been known to worsen underlying tics in concomitant
Tourette’s syndrome. Although clonidine can be used in ADHD with tics, it would not be considered
first-line treatment after the stimulants; therefore Atomoxetine would be the best answer choice.
Psychopharmacology
Psychopharmacology
B&D Chapter 61

Question 97. D. More statistics! Regression analysis is a method of using data to predict the value of one variable in rela-
tion to another. The other distractors are explained elsewhere in this volume.
Statistics
K&S Chapter 5

Question 98. C. Lioresal (baclofen) is one of the most potent of the oral muscle-relaxing agents and treats spasticity highly
effectively. Its full mechanism of action is not well understood, but it is believed to have predominant
effect as a GABA-B agonist. It can cause muscular weakness and difficulty with weight-bearing because
of its potency. Other first-line agents to treat symptomatic spasticity include gabapentin, diazepam, cloni-
dine, tizanidine, and dantrolene. Second-line agents include intrathecal Lioresal, Marinol, chlorpromazine,
cyproheptadine, phenytoin, and phenobarbital. Brexpiprazole is a second-generation antipsychotic sold in
the U.S. under the brand name Rexulti. It is indicated for schizophrenia and adjunct treatment of depres-
sion. It is a distractor in this question and has nothing to do with treatment of spasticity.
Neurology
B&D Chapter 48

Question 99. A. Just in case Question 61 wasn’t enough, here it is again. Correlation coefficient is a measurement of the
direction and strength of the relationship between two variables. The Pearson correlation coefficient is
on a scale from −1 to +1. A positive correlation means that one variable moves the other in the same
direction. A negative value means that one moves the other in the opposite direction.
Statistics
K&S Chapter 5

Question 100. C. Triphasic waves on electroencephalogram are characteristic of hepatic or metabolic encephalopathy.
Of course, hepatic encephalopathy is often accompanied by asterixis, particularly when the condition
is severe. Asterixis is a sudden loss of postural tone and manifests as a flapping tremor of the hands. It
is exhibited when the arms are fully extended. Herpetic skin vesicles would of course be expected with
herpes simplex virus infections. Penile chancre can be demonstrated in cases of syphilis. Dupuytren’s
contractures are a form of benign progressive fibroproliferative disease of the palmar fascia of unknown
etiology. The condition is seen more often in men than in women and is associated with alcoholism,
hand trauma, and diabetes. Pulmonary rales would be an expected sign in congestive heart failure.
Neurology
B&D Chapter 56

Question 101. E. Competence to stand trial was established by the court case Durskey v United States. This case determined
that the defendant must have sufficient present ability to consult with his or her lawyer with a reasonable
degree of rational understanding and must have rational as well as factual understanding of the proceed-
ings against him or her. Concretely this translates into the ability to understand the charges and legal issues
relevant to their current situation. The defendant must be able to process this factual understanding and
participate appropriately. Participation typically entails the ability to communicate and work with an attor-
ney and to make decisions after receiving legal advice. Should there be a deficit in one of these areas, that

179

Psychiatry Test Preparation and Review


Manual E-Book
Psychiatry Test Preparation & Review Manual

deficit must be related to symptoms of a mental disorder for the defendant to be found incompetent to
stand trial. It is not required that the patient can recognize the symptoms of his or her mental illness.
Forensic Psychiatry
K&S Chapter 36

Question 102. D. Common side effects of valproic acid include weight gain, tremor, thinning of the hair, and ankle swell-
ing. Other noted adverse effects are gastrointestinal distress, sedation, pancreatitis, bone marrow sup-
pression (pancytopenia), and hepatotoxicity. The agent is teratogenic, and the most worrisome fetotoxic
effect is the neural tube defect (spina bifida), which is dangerous during neurogenesis in the first trimes-
ter of gestation.
Psychopharmacology
B&D Chapter 67

Question 103. E. It is the doctor’s ethical obligation to treat the patient in question regardless of ability to pay. It is unethi-
cal to allow a suicidal patient to leave the hospital because of a dispute with the insurance company.
Should the patient go and commit suicide, the doctor, not the insurance company, is liable. After the
patient is treated and released, the issue can be taken up more aggressively with the insurance company.
As a physician, your first and most important obligation is to the patient, not to the insurance company
or to your wallet.
Ethics
K&S Chapter 36

Question 104. B. Clonidine (Catapres) is a presynaptic α2-receptor agonist. It is FDA-approved as an antihypertensive


agent. It acts by reducing the amount of norepinephrine that is released from the synaptic bouton. This
effect decreases sympathetic tone and bodily arousal and activation. The agent diminishes the autonomic
symptoms associated with opioid withdrawal, such as tachycardia, hypertension, sweating, and lacri-
mation. Atomoxetine (Strattera), FDA-approved for ADHD in children and adults, is a norepinephrine
reuptake inhibitor. The neuroleptic medications, both conventional and atypical, can cause α1 adrenergic
antagonism, thereby causing orthostatic hypotension. Dopamine type 2 antagonism is the putative anti-
psychotic mechanism of all of the neuroleptic agents, both conventional and atypical. Serotonin antago-
nism is what makes an atypical antipsychotic atypical. It is, in fact, the ratio of D2 to 5-HT 2 blockade
that reduces the EPS of the atypical neuroleptics.
Psychopharmacology
K&S Chapter 29

Question 105. D. Heroin can be detected on a urine toxicology screen for 36 to 72 hours. Alcohol can be detected for up
to 12 hours. Amphetamines can be detected for up to 48 hours. Cannabis can be detected for up to 4
weeks. PCP can be detected for up to 8 days.
Substance Abuse and Addictive Disorders
K&S Chapter 5

Question 106. E. The Klüver–Bucy syndrome results from bilateral destruction of the amygdaloid bodies and the infe-
rior temporal cortex. Clinical features include hypersexuality, placidity, and hyperorality. One of the
causes of the syndrome is Pick’s disease (frontotemporal dementia). Other causes include stroke and
Alzheimer’s dementia. Other associated features include visual agnosia (psychic blindness), hyperpha-
gia, and prosopagnosia (the inability to recognize faces). The rest of the answer choices are simply
distractors.
Neurocognitive Disorders
K&S Chapter 21

Question 107. E. Patients at high risk during ECT include the following: those with space-occupying lesions in the CNS,
those with increased intracranial pressure, those at risk for cerebral bleed, those who have had a recent
myocardial infarction, and those with uncontrolled hypertension. There are no absolute contraindica-
tions for ECT, but patients who fall into any of the before-mentioned categories should be screened

180

Psychiatry Test Preparation and Review


Manual E-Book
Test Number Three

carefully and decisions made on a case-by-case basis depending on risks, benefits, and ability to control
risk factors.
Diagnostic and Treatment Procedures in Psychiatry
K&S Chapter 30

Question 108. B. Klinefelter’s syndrome results from the presence of an extra X chromosome (XXY triploidy). It is noted
in about 1 in 700 men. Clinical features include small dysfunctional testes, intellectual disability, and
pear-shaped stature. Testosterone replenishment may offset some of the stigmata of the condition.
Turner’s syndrome is the absence of an X chromosome in a genetic female (XO). Characteristics include
short stature and lack of pubertal sexual development. Other clinical features include a webbed neck
and heart and kidney anomalies. Trisomy 21 is of course classic Down’s syndrome, which is explained in
detail in another question in this volume. Deletion on the paternal chromosome 15 results in the Prader–
Willi syndrome. The prevalence is about 1 in 12,000 to 15,000. Clinical features include profound intel-
lectual disability, hypogonadism, hypotonia, behavioral disinhibition, rapid and excessive weight gain,
and facial dysmorphism. Trisomy 18 is described elsewhere in this volume.
Neurodevelopmental Disorders
B&D Chapter 40

Question 109. C. Patient-controlled analgesia has proven to be an extremely good way of treating pain. Patients who con-
trol their own dosing end up using less pain medication than those who have to ask for the medication
and wait for the doctor to write an order. They also have far better pain control. Some patients, par-
ticularly cancer patients, may need large and escalating doses of medication to control their pain. Under
these circumstances, this should not be viewed as addiction, but as a necessary part of the treatment of
their illness. Cancer patients have been shown to wean themselves off the pain medication once the pain
decreases. The use of pain control in a drug addict with a painful medical illness is as important as it is
in a nonaddict with the same illness. They may need higher doses to control their pain, but the doctor
has as much of an obligation to manage their pain as he or she does to manage the pain of the nonad-
dict. The other answer choices do not address the issue of using as little medication as necessary while
obtaining the best pain control. This is why patient-controlled analgesia is the best choice.
Somatic Symptom Disorders
K&S Chapter 13

Question 110. B. This vignette points to a diagnosis of intermittent explosive disorder. Intermittent explosive disor-
der manifests as discrete episodes of failure to resist aggressive impulses that lead to extreme physical
aggression directed toward people and/or property. The degree of aggression is completely out of pro-
portion to any particular psychosocial stressor that may trigger such an episode.

Episodes are unpredictable and often arise without cause or particular trigger and remit as spontane-
ously as they begin. There are no signs or symptoms of aggressivity noted in between these discrete epi-
sodes. The disorder is more common in men than in women. Predisposing psychosocial factors include
an underprivileged or tempestuous childhood, childhood abuse, and early frustration and deprivation.
Biological predisposing factors are believed to be decreased cerebral serotonergic transmission, low CSF
levels of 5-hydroxyindoleacetic acid, and high CSF levels of testosterone in men. There is strong comor-
bidity with fire setting, substance use, and the eating disorders.

Personality disorders, such as borderline and antisocial, are distinguished from intermittent explosive
disorder by a pervasive pattern of maladaptive behavior that would be expected to occur in between epi-
sodes and affect the patient’s life adversely in more areas of functioning. Aggressive patients with bipo-
lar mania would be expected to present with evidence of manic symptoms, including elevated/irritable
mood, increased energy, rapid pressured speech, sleeplessness, racing thoughts, distractability, increased
goal-directed behavior, and perhaps even psychosis. Temporal lobe seizures are a remote possibility and
can certainly result in aggression, most often interictally, but there is no evidence of this presented in this
question.

181

Psychiatry Test Preparation and Review


Manual E-Book
Psychiatry Test Preparation & Review Manual

Treatment of intermittent explosive disorder can be undertaken with mood stabilizers, such as lithium,
carbamazepine, divalproex sodium, and gabapentin. SSRIs and tricyclic antidepressants can also be
effective in reducing aggression. Avoid use of benzodiazepines because they can disinhibit the patient,
leading to a further loss of behavioral control.
Disruptive, Impulse Control, Conduct Disorders, and ADHD
K&S Chapter 19

Question 111. B. Catalepsy is an immobile position that is constantly maintained. Cataplexy is temporary loss of muscle tone
precipitated by an emotional state. Psychomotor retardation is decreased motor and cognitive activity often seen
with depression. Catatonia is markedly slowed motor activity to the point of immobility and unawareness of
surroundings. It can include stupor, waxy flexibility, mutism, negativism, posturing, mannerisms, stereotypy, agi-
tation, grimacing, echolalia, or echopraxia. Stereotypy is a repetitive fixed pattern of movement or speech.
Psychotic Disorders
K&S Chapter 7

Question 112. B. Habit reversal is a type of therapy used to treat trichotillomania and tics found in Tourette’s syndrome.
It involves awareness training in which the patient’s awareness of the tic or hair-pulling behavior is
increased. It then entails competing-response training in which the patient is helped to develop a compet-
ing response to the urge to pull or perform the tic. It also involves strategies used to decrease overall ten-
sion. Habit reversal is not indicated for any of the other answer choices given.
Obsessive–Compulsive and Related Disorders
K&S Chapter 31

Question 113. D. The description in this question is of partial isolation monkeys. Those totally isolated from other monkeys
were very fearful, unable to copulate, and unable to raise young. Mother-only raised monkeys failed to
copulate, did not leave the mother to explore, and were afraid of their peers. The peer-only raised mon-
keys were easily frightened, timid, had little playfulness, and grasped other monkeys in a clinging manner.
In monkeys separated from their mothers, there was an initial protest stage followed by despair. Many of
these behavior patterns can be correlated with human behaviors that are seen in our patients.
Psychological Theory and Psychometric Testing
K&S Chapter 5

Question 114. A. Sildenafil, vardenafil, and tadalafil are nonselective phosphodiesterase 5 (PDE5) inhibitors. They also
have some agonistic effects on nitric oxide (NO). PDE5 blockade causes arterial smooth muscle dilata-
tion and facilitates cavernosal blood filling, which potentiates penile erection. These agents cannot be
used with nitrates because the combined effect of NO agonism with nitrates can cause significant vaso-
dilatation and precipitous lowering of the blood pressure than can result in diminished cardiac perfusion
and myocardial infarction. The other answer choices are simply distractors.
Psychopharmacology
K&S Chapter 29

Question 115. B. Amoxapine is one of the tricyclic antidepressants. What sets it apart from the others is that one of its
metabolites has dopamine-blocking activity. Because of this, amoxapine has the potential to cause par-
kinsonian symptoms, akathisia, and even neuroleptic malignant syndrome. None of the other drugs
listed have dopamine-blocking activity.
Psychopharmacology
K&S Chapter 29

Question 116. C. The relative risk of an illness is the ratio of the incidence of the condition in those with risk factors to
the incidence of the condition in those without risk factors. Attributable risk refers to the absolute inci-
dence of the illness in patients exposed to the condition that can be attributed to the exposure. The other
answer choices are nonsense distractors and are not true biostatistical terms.
Statistics
K&S Chapter 5

182

Psychiatry Test Preparation and Review


Manual E-Book
Test Number Three

Question 117. C. Cingulotomy is a surgical treatment for OCD. It is successful in treating about 30% of otherwise treat-
ment-resistant patients. Some patients who fail medication, and then subsequently fail surgery, will respond
to medication after surgery. Complications of cingulotomy include seizures, which are then managed with
anticonvulsants. The other disorders listed in this question do not have surgical treatments.
Diagnostic and Treatment Procedures in Psychiatry
K&S Chapter 10

Question 118. E. Dissociative identity disorder (DID), also known as multiple personality disorder, is a chronic dissocia-
tive disorder. The origins of the disorder are believed to stem from early childhood trauma, most often
sexual or physical. The hallmark of the disorder is the presence of two or more distinct identities or
personality states that recurrently take over the person’s behavior. There is also a presence of dissocia-
tive amnesia, with a noted inability to recall important personal information that is too extensive to be
explained solely by forgetfulness.

The true cause of DID is unknown. Some research points to a possible connection between DID and epi-
lepsy, with some patients having abnormal electroencephalograms. The absence of external support, partic-
ularly from parents, siblings, relatives, and significant others, seems to play a pivotal role in the genesis of
the disorder. The patient’s lack of stress-coping mechanisms is also a likely contributory factor. The differ-
ential diagnosis includes borderline personality disorder, rapidly cycling bipolar disorder, and schizophre-
nia. The disorder can start at almost any age, and an early age of onset is predictive of a worse prognosis.

Treatment is focused on insight-oriented psychotherapy. Hypnotherapy may also be helpful.


Antipsychotic medications are often unhelpful. Antidepressant and anxiolytic medications can be useful
in addition to psychotherapy. Anticonvulsant mood stabilizers have shown some efficacy in certain stud-
ies. Viral exposure or infection has nothing to do with the etiology of DID.
Dissociative Disorders
K&S Chapter 12

Question 119. D. The first-line therapy for the catatonic patient is intramuscular lorazepam. Many patients will respond
to this treatment and will come out of their catatonic state. They can then be given subsequent treatment
with antipsychotic drugs for the underlying psychotic disorder that is the most likely cause of the catato-
nia. Antidepressants and stimulants are not indicated for catatonia.
Psychotic Disorders
K&S Chapter 7

Question 120. C. Malingering is diagnosed in the presence of intentional production of symptoms that are exaggerated
and either physical or psychological in nature. These symptoms are motivated by secondary gain and
incentives to avoid responsibility or danger, or to obtain compensation or some other benefit that is
material or monetary. Symptoms are vague, ill-defined, overdramatized, and do not conform to known
clinical conditions. Patients seek secondary gain, most often in the form of drugs, money, or the avoid-
ance of work or jail. History and examination typically do not reveal complaints from the patient. The
patient is often uncooperative and refuses to accept a good prognosis or clean bill of health. Findings
can be compatible with self-inflicted injuries. Medical records may have been tampered with or altered.
Family members are usually unable to verify the consistency of symptoms.
Somatic Symptom Disorders
K&S Chapter 25

Question 121. D. Mutism refers to a patient who is voiceless but has no abnormalities in the structures that produce
speech. Mutism is common in catatonia. It can be seen in conversion disorder. There is also a diagno-
sis known as selective mutism for children who consistently fail to speak in social situations despite
speaking in other situations. Other causes of mutism include MR, pervasive developmental disorder, and
expressive language disorders. It can also be a component of buccofacial apraxia, locked-in syndrome,
and a persistent vegetative state.
Psychotic Disorders
K&S Chapters 7 and 31

183

Psychiatry Test Preparation and Review


Manual E-Book
Psychiatry Test Preparation & Review Manual

Question 122. B. Biofeedback is a therapy in which instruments are used to measure autonomic parameters in patients,
who are provided with “real-time” feedback from the instrumentation about their bodily physiologic
processes. This feedback enables patients to control their own physiologic functions and alter them in
positive ways to alleviate symptoms using operant conditioning techniques. Feedback is provided to the
patient by measuring physiologic parameters, such as heart rate, blood pressure, galvanic skin response,
and skin temperature. The measurement is translated into a visual or auditory output signal that patients
can rely on to gauge their responses. Patients can alter the tone by using guided imagery, breathing tech-
niques, cognitive techniques, and other relaxation techniques. The modality is useful for anxiety disor-
ders, migraine, and tension-type headache in particular.

Stimulus–response therapy is a nonsense distractor, because there is no such thing. Relaxation training is
a form of behavior therapy that basically encompasses techniques such as meditation and yoga to help
patients dispel anxiety by tapping into their own physiologic parameters, such as heart rate and breath-
ing rate. Guided mental imagery also helps patients enter a relaxed state of mind. Behavior therapy is the
global term used to describe various therapeutic modalities that employ either operant or classical con-
ditioning techniques to help patients overcome their fears, phobias, and anxieties. Flooding, systematic
desensitization, and aversion therapy are all examples of behavior therapy. Desensitization refers to the
technique that helps patients gradually overcome their fears, phobias, and anxieties by graded exposure
to the very stimulus that is the source of their fears. The patient is exposed to more and more anxiety-
provoking stimuli, but relaxation training helps the patient cope with his or her maladaptive responses
and eventually ideally respond to the stimulus without it evoking anxiety.
Diagnostic and Treatment Procedures in Psychiatry
K&S Chapter 28

Question 123. D. Lithium toxicity is a medical emergency and can result in permanent neuronal damage and death.
Toxicity occurs at lithium levels above 2.5 mEq/L. Treatment includes discontinuation of lithium and
vigorous hydration. If the level is above 4 mEq/L, or the patient shows serious signs of lithium toxicity
(nephrotoxicity, convulsions, coma), the patient must have hemodialysis. Hemodialysis can be repeated
every 6 to 10 hours until the level is no longer toxic and the patient’s symptoms remit. If the patient in
question was not showing serious signs of toxicity, laboratory tests to assess the situation could be sent,
a neurological examination done, ECG obtained, gastric lavage performed, activated charcoal given, and
vigorous hydration used. The patient could then be monitored and given time to clear the lithium.
Laboratory Tests in Psychiatry
Management in Psychiatry
K&S Chapter 29

Question 124. D. The most useful long-term treatment parameter for the noncompliant patient with schizophrenia who
has a history of violence would be the use of an outpatient commitment program. Certain states have
such laws in place but others do not. Treating clinicians can petition the court to place refractory, poten-
tially dangerous patients on this status. A judge mandates the patient’s cooperativeness, and the patient
is compelled to report for outpatient follow-up. If the patient is noncompliant, a psychiatrist can order
that the patient be picked up against his or her wishes and brought in for psychiatric evaluation to an
emergency room, where the patient can be held for a period of time. Certain states use an alternative to
outpatient commitment called conservatorship. This modality involves the court appointing a conserva-
tor, often a family member, to look after the patient and make decisions on the patient’s behalf, including
placing the patient in involuntary hospitalization if it is deemed necessary.

Partial hospitalization is quite similar to inpatient hospitalization, except the patient sleeps at home. This
modality is often a good transition between inpatient and outpatient services, and patients can stay in such
a program over extended periods of time if necessary. Patients have a case manager assigned to them in
partial programs. The case manager helps coordinate and facilitate the patient’s care and helps make the
patient’s transition to the less restrictive setting easier. Day treatment programs are somewhat less intense
and less structured than partial hospital programs. Patients again sleep at home, but they participate in
facility-based care 5 or more days a week. Case management and social work also play pivotal roles in the
day treatment program setting.

184

Psychiatry Test Preparation and Review


Manual E-Book
Test Number Three

Social skills training refers to a program that is dedicated to helping low-functioning patients, such as
those with schizophrenia, to gain key skills that will enable them to function more independently in the
community. The focus is on improving patients’ interactions with other people in their environment.
Public Policy
K&S Chapters 28 and 36

Question 125. C. MAOIs have several serious drug–drug interactions that must be kept in mind. Meperidine (Demerol)
can never be given with an MAOI, because this combination has led to death in several patients (com-
mon examination question!). MAOIs should never be used with anesthetics (no spinal anesthetics and
no anesthetics containing epinephrine; lidocaine is OK). MAOIs should not be combined with asthma
medication or over-the-counter drugs that contain dextromethorphan (cold and flu medications). They
cannot be given with sympathomimetics (epinephrine, amphetamines, cocaine). They cannot be given
with SSRIs or clomipramine, because this will precipitate a serotonin syndrome. There are also many
food restrictions with MAOIs, which will, no doubt, be the subject of another question.
Psychopharmacology
K&S Chapter 29

Question 126. A. Privilege refers to the psychiatrist’s right to maintain a patient’s secrecy or confidentiality even in the face
of a subpoena. This implies that the right of privilege belongs to the patient, not the psychiatrist, and
therefore the patient can waive the right. There are many exceptions to medical privilege, and many phy-
sicians are not aware that they do not legally enjoy the same privilege that exists between husband and
wife, priest and parishioner, and a client and an attorney.

Confidentiality is the professional obligation of the physician to maintain secrecy regarding all informa-
tion given to him or her by the patient. A psychiatrist may be asked to appear in court and testify by
subpoena and thereby be forced to break a patient’s confidentiality. A patient may release the clinician
from the obligation of confidentiality by signing a consent to release information. Each release pertains
to a specific matter or piece of information and may need to be reobtained for subsequent disclosures.

Communication rights refers to the patient’s right to free and open communication with the outside
world by either telephone or mail while hospitalized. Private rights refers to the patient’s right to privacy.
In a hospital setting, this applies to patients having private toileting and bathing space, secure storage
space for personal effects, and adequate personal floor space per person. Patients also have the right to
wear their own clothing and carry their own money if they desire to do so. Certain restrictions to this
right may apply based on the danger presented to self or others.

Clinical responsibility is not a forensic term per se, but it refers to the responsibility of the physician to
the patient to provide the patient with the best care possible in any clinical setting, irrespective of the
patient’s financial, racial, or personal status.
Forensic Psychiatry
K&S Chapter 36

Question 127. B. Patients being started on clozapine should have a baseline WBC count with differential before treatment.
A WBC count with differential is taken every week during treatment for the first 6 months, then every
2 weeks thereafter. Monthly monitoring of WBC count may be done after 12 months of therapy on clo-
zapine. When treatment is stopped, WBC counts should be taken every week for 4 weeks. It is not a part
of standard monitoring to take two WBC counts in 1 week.
Laboratory Tests in Psychiatry
K&S Chapter 29

Question 128. C. Patients over 65 years of age undergo sleep problems that affect both REM sleep and non-rapid eye
movement (NREM) sleep. There are more REM episodes noted. REM episodes are shorter in duration.
There is less total REM sleep. In NREM sleep there is a decreased amplitude of delta waves. There is a
lower percentage of N3 sleep. There is a higher percentage of N1 and N2 sleep. The elderly experience
increased awakening after sleep onset.
Sleep–Wake Disorders
K&S Chapter 16

185

Psychiatry Test Preparation and Review


Manual E-Book
Psychiatry Test Preparation & Review Manual

Question 129. B. Clozapine (Clozaril) has several drug–drug interactions that are noteworthy. Cimetidine, SSRIs, tricy-
clics, valproic acid, and erythromycin will all increase Clozaril levels. Phenytoin and carbamazepine
will decrease clozapine levels. Clozapine should not be combined with any medication that can cause
agranulocytosis (carbamazepine, propylthiouracil, sulfonamides, and captopril). CNS depressants (alco-
hol, benzodiazepines, and tricyclics) cause even more depression when combined with clozapine. The
combination of lithium and Clozaril can increase neuroleptic malignant syndrome, seizures, confusion,
and movement disorders. Other answer choices are distractors. The foods given should be avoided when
taking MAOIs. Acetaminophen and aripiprazole have no interaction.
Psychopharmacology
K&S Chapter 29

Question 130. E. Cannabis is the most widely abused recreational drug among U.S. high school students. Cannabis use
has been demonstrated to lead to future cocaine abuse in adolescents. About 35% of high school seniors
reported using cannabis. Alcohol is also a pervasive problem among high school teens, but only in about
10% to 20% of students surveyed. Over 85% of high school seniors have reported that they have tried
alcohol at some time. About 15% of adolescents have reported using inhalants. Fewer than 2% of high
school students report having used cocaine. About 9% of high school seniors have reported trying LSD
at some time.
Substance Abuse and Addictive Disorders
K&S Chapter 20

Question 131. D. Aaron Beck is the originator of cognitive therapy. It is based on the theory that affect and behavior are
determined by the way in which patients structure the world. Patients have assumptions, on which they
base cognitions, which lead to affect and behavior. In depression specifically, Beck feels that there is a
triad consisting of the following:
1. Depressed people see themselves as defective, inadequate, and worthless.
2. Depressed people experience the world as negative and self-defeating.
3. Depressed people have an expectation of continued hardship and failure.
It is this triad of distorted negative thoughts that Beck feels leads to depression. The goal of cognitive
therapy is to help test the cognitions and develop more productive alternatives. We want to help the
patient find more benign explanations for symptoms of distress to reduce catastrophizing, overgener-
alizing, selective abstraction, and dichotomous thinking, as well as other cognitive distortions. Other
answer choices given are distractors. Good enough mothering and transitional object are terms associ-
ated with the child development theory of Winnicott. Mania is not associated with Beck’s cognitive triad.
Aggression toward the primary caregiver has nothing to do with Beck.
Psychotherapy
K&S Chapter 28

Question 132. D. Ramelteon (Rozerem) is a novel sleeping agent that was FDA-approved in 2005 for insomnia with sleep-
onset difficulties. It has a unique mechanism of action: it is a melatonin agonist. It works by stimulating
melatonin type 1 and type 2 receptors in the suprachiasmatic nucleus of the hypothalamus. It has no
addictive or abuse potential, because it is not a GABA-A agonist and has no activity at the benzodiaz-
epine receptor whatsoever. It has no effects at histamine, acetylcholine, dopamine, serotonin, or nor-
epinephrine receptors. The dosage is the same for all patients: one 8-mg tablet at bedtime. Recall that
zolpidem, zaleplon, and eszopiclone are all benzodiazepine receptor agonists and as such have sedative–
hypnotic effects and potential for tolerance, withdrawal, and abuse.
Psychopharmacology
K&S Chapter 29

Question 133. E. Alogia is a lack of speech that results from a mental deficiency or dementia. Poverty of movement is
called akinesia. Poverty of emotion is described using the term “flat affect.”
Diagnostic and Treatment Procedures in Psychiatry
K&S Chapter 5

186

Psychiatry Test Preparation and Review


Manual E-Book
Test Number Three

Question 134. B. To meet criteria for the rapid cycling specifier in bipolar disorder, the patient must present with at least
four mood episodes over the past 12 months. The mood episodes must meet criteria for a major depres-
sive, manic, mixed, or hypomanic episode. Female patients are more likely than men to have rapid
cycling bipolar disorder. There is no evidence to suggest that rapid cycling is a heritable phenomenon in
bipolar disorder. It is therefore likely to be a result of external factors, such as stress or medication.
Bipolar Disorders
K&S Chapter 8

Question 135. B. Random reinforcement is seen with the gambler. In random reinforcement the reward is given only a
fraction of the time at random intervals. The money from a slot machine is won at random times. This
keeps the gambler guessing and trying to anticipate when he or she will win. This is a very good way to
maintain a behavior. Continuous reinforcement is when every action is rewarded and is the best way to
teach a new behavior. Primary reinforcers are independent of previous learning; for example, the need to
eat is biological and not based on previous learning. Secondary reinforcers are based on previous learn-
ing, such as rewarding a child with a present when he or she does something well. The dexamethasone
suppression test is an experimental measure associated with depression and is unrelated to pathological
gambling. Cerebellar dysfunction would lead to ataxia and gait disturbance, which again is unrelated to
pathological gambling.
Psychological Theory and Psychometric Testing
K&S Chapter 5

Question 136. B. Late-onset schizophrenia is noted more often in women than in men. The prognosis seems to be more
favorable when the onset is late. There is a tendency to see more paranoia in these late-onset schizo-
phrenic patients. Schizophrenia is considered late onset when symptoms begin after 45 years of age. It is
clinically identical to schizophrenia that has normal onset.
Psychotic Disorders
K&S Chapter 7

Question 137. B. One might think that this one was too easy, but it’s just the kind of question that could show up on an
examination. A control group is a group in a study that does not receive treatment and is used as a stan-
dard of comparison.
Statistics
K&S Chapter 5

Question 138. E. Bowlby and Robertson identified three essential stages of separation response among children. The first
stage is that of protest. The child protests the mother’s departure by crying, calling out, and searching
for her. The second stage is despair and pain. The child loses faith that the mother will return. The third
stage is detachment and denial of affection to the mother figure upon her return. These phases are noted
universally in children who go through separation by loss of parents to death, through divorce, or by
going off to boarding school. Acceptance is not one of Bowlby’s stages of the separation response. It is
the fifth and final stage of Kübler-Ross’ stages of reaction to impending death.
Human Development
K&S Chapter 2

Question 139. C. Bupropion has been shown in some studies to be efficacious for the treatment of ADHD in children and
adults. However, studies are mixed as to how helpful it is and it does not have an FDA approval for
ADHD. It is also a very good antidepressant, which is its most common use. It has also been found very
useful in patients who do not respond to an SSRI. When bupropion is added to SSRIs, up to 70% of these
cases improve, because both drugs together hit more neurotransmitter systems than either drug alone.
The other drugs listed have no proven usefulness in ADHD. Of note, vortioxetine is a serotonin modula-
tor sold in the U.S. under the brand name Trintellix, indicated for MDD. It works via serotonin reuptake
inhibition, antagonizing the 5HT3 receptor and agonizing the 5HT1A receptor. It has no use in ADHD.
Psychopharmacology
K&S Chapter 29

187

Psychiatry Test Preparation and Review


Manual E-Book
Psychiatry Test Preparation & Review Manual

Question 140. C. In restless legs syndrome there is an urge to move the legs accompanied by an uncomfortable and
unpleasant sensation, which begins or worsens during periods of rest, is partially or totally relieved by
movement, and is worse in the evening or at night. In some cases it occurs only in the evening or at
night. The symptoms must persist for at least 3 months.
Sleep–Wake Disorders
K&S Chapter 16

Question 141. A. This question contains a supportive statement that acknowledges how difficult things are for the patient
currently but gives her hope for the future. It is characteristic of supportive psychotherapy. In psycho-
dynamic therapy and psychoanalysis, the therapist would wish to remain more neutral and not make
statements that expressed opinion or boosted the patient’s mood. The question does not involve playing
with children, so play therapy is incorrect. Cognitive behavioral therapy would involve looking at the
patient’s assumptions and cognitions and seeing how they affect the patient’s mood. That is not happen-
ing in this question stem.

Supportive psychotherapy seeks to stabilize the self and the patient’s ability to cope. Defenses are
strengthened. Symptom relief is sought. Neutrality is suspended. Direction by the therapist is encour-
aged. Free association is not part of this technique. Supportive psychotherapy can be used with those
who have severe character pathology or psychosis or are in the midst of an acute crisis or a physical
illness.
Psychotherapy
K&S Chapter 28

Question 142. C. Ziprasidone is available as a short-acting injection but not as a long-acting decanoate. Haloperidol is
available as a long-acting decanoate administered every 4 weeks. Olanzapine is available as a long-act-
ing injection sold in the U.S. under the brand name Zyprexa Relprevv. It can be dosed every 2 weeks
or every 4 weeks. It requires specific monitoring postinjection, due to the potential for a postinjection
delirium/sedation syndrome. Paliperidone is available in two long-acting forms that are sold in the
U.S. under the brand names Invega Sustenna and Invega Trinza. Invega Sustenna can be dosed every 4
weeks, and Invega Trinza can be dosed every 3 months. Risperidone is available in a long-acting injec-
tion given every 2 weeks that is branded Risperidone Consta in the U.S. Other notable long-acting injec-
tions include fluphenazine decanoate, which can be dosed every 3 to 6 weeks, and aripiprazole lauroxil,
branded Aristada in the U.S. Aripiprazole lauroxil can be dosed once per month.
Psychopharmacology
K&S Chapter 29

Question 143. C. Naloxone is an opioid antagonist. An opioid overdose is a medical emergency. Respiratory depression
ensues, leading to coma and shock. Naloxone is given intravenously and can be repeated four or five
times in the first 30 minutes. Care must be used because the half-life is short, and the patient can relapse
back into coma. Plus, severe withdrawal can ensue from the use of an opioid antagonist.

Naltrexone is a longer-acting antagonist, with a half-life of 72 hours. It is a preventative measure for


those with opioid addiction. It is used for blocking the euphoric effects of opioid use; thus it decreases
craving. Because of its long half-life, it is not the best choice for an emergency.

Buprenorphine is a mixed opioid agonist–antagonist. It is used in place of morphine to keep people off
heroin and is not used in emergencies.

Benztropine is also known as Cogentin and is an anticholinergic medication used to mitigate the side
effects of antipsychotic drugs. It has nothing to do with opium overdose.

Bromocriptine is a mixed dopamine agonist–antagonist approved in the United States for treatment of
Parkinson’s disease.
Psychopharmacology
K&S Chapter 20

188

Psychiatry Test Preparation and Review


Manual E-Book
Test Number Three

Question 144. B. Elisabeth Kübler-Ross developed a comprehensive paradigm to classify the stages of a person’s reactions
to impending death. Stage 1 is that of shock and denial. Upon learning the news that they are dying,
people are initially in a state of shock and may deny that the diagnosis is correct. Stage 2 is that of anger.
During this stage patients get frustrated, angry, and irritable about their condition. They often ask: “Why
me?” They typically undergo a lot of self-blame about their illness. Stage 3 is that of bargaining. Patients
may try to negotiate or bargain with doctors, friends, family, and even God to alleviate their illness in
exchange for good deeds or fulfillment of certain pledges. Stage 4 is that of depression. During this stage,
patients demonstrate frank signs and symptoms of depression, including hopelessness, suicidal ideation,
social withdrawal, and sleep problems. If the symptoms are severe enough to qualify as an MDD, the
patient should be treated with an antidepressant. Stage 5 is that of acceptance. Patients acknowledge and
come to terms with the inevitability of their death during this stage. Patients can begin to talk about fac-
ing the unknown without fear and with resolution.
Psychological Theory and Psychometric Testing
K&S Chapter 34

Question 145. D. Asking patients about suicide will have no effect on whether they are actually suicidal. Talking about
suicide will not make patients suicidal. Most patients, when asked, are relieved to have permission to
speak about something they have already thought about but were uncomfortable talking about with
friends or family. The question is not frightening to them, nor will the mention of the word suicide help
them plan their death.
Diagnostic and Treatment Procedures in Psychiatry
K&S Chapter 23

Question 146. D. Patients can display their grief over the death of loved ones in different ways and with different intensity.
It is generally believed that a period of grief or mourning typically lasts about 6 months to 1 year. Some
symptoms and signs of mourning may persist for a longer period, even up to 2 years or more. In most
cases, the acute symptoms of grief improve over a period of about 1 to 2 months, after which time the
individual returns to a more normal level of functioning.
Depressive Disorders
K&S Chapter 34

Question 147. D. The suicide rate for adolescents has quadrupled since 1950. Suicide accounts for 12% of deaths in the
adolescent age group. The suicide rate has gone up more in this group than in any other group over the
same time period.
Depressive Disorders
K&S Chapter 31

Question 148. D. Ramelteon (Rozerem) is a melatonin agonist and has a short half-life ranging from 1 to 2.5 hours.
Ramelteon has an active metabolite, M-II, that has a half-life of about 5 hours. Zolpidem (Ambien) has
a half-life of about 2.5 hours, but the duration of action can range from 1 to 4.5 hours. Zolpidem has
no active metabolite. Zaleplon (Sonata) is a benzodiazepine receptor agonist that has the shortest half-
life of all these agents at about 1 hour. It is therefore very useful for the treatment of middle insomnia.
Eszopiclone (Lunesta) has the longest half-life of all these sleeping agents at about 6 hours. It is there-
fore, at least theoretically, the one most likely to cause next-day drowsiness. Triazolam (Halcion) is a
benzodiazepine sedative–hypnotic agent with high potency and a short half-life ranging in duration from
2 to 4 hours.
Psychopharmacology
K&S Chapter 29

Question 149. A. The therapeutic range for valproic acid is 50 to 100 ng/mL. At levels as high as 125 ng/mL, side effects,
including thrombocytopenia, may occur. Liver function tests should be obtained at the start of treatment
and every 6 to 12 months thereafter, and valproic acid levels should be checked periodically.
Laboratory Tests in Psychiatry
K&S Chapter 5

189

Psychiatry Test Preparation and Review


Manual E-Book
Psychiatry Test Preparation & Review Manual

Question 150. C. This is a question based on an understanding of the importance of culture. Different cultures view health
and sickness differently. They view the medical system differently, and these differences must be taken
into account when dealing with a multicultural population. In Japanese culture, it is customary to mini-
mize distress in front of an authority figure. This is the explanation for the patient’s behavior in this
question. If the doctor involved was unaware of cultural issues, the patient’s behavior could be misinter-
preted as psychosis or malingering, or the family could be seen as manipulative.
Cultural Issues in Psychiatry
K&S Chapter 3

190

Psychiatry Test Preparation and Review


Manual E-Book
Test Number Four

FOUR Test Number Four


1. Which one of the following is false regarding Freud’s theories of human development?

A. 
During development, sexual energy shifts to different areas of the body that are usually associated with eroticism
B. 
The anal phase is from 1 to 3 years of age
C. 
Latency is marked by a sharp increase in sexual interest
D. 
Freud thought that resolution of his stages was essential to normal adult functioning
E. 
The phallic stage is from 3 to 5 years of age

2. Ischemia to which one of the following arterial territories is responsible for the phenomenon known as amaurosis
fugax?

A. 
Carotid
B. 
Vertebrobasilar
C. 
Lenticulostriate
D. 
Anterior cerebral
E. 
Middle cerebral

3. Which one of the following is true regarding norepinephrine (NE) and/or the locus ceruleus?

A. 
NE is synthesized in the locus ceruleus
B. 
Dopamine is synthesized in the locus ceruleus, NE in the dorsal raphe nuclei
C. 
Acetylcholine is synthesized with NE in the substantia nigra
γ-Aminobutyric acid (GABA) is synthesized in the locus ceruleus
D. 
E. 
The locus ceruleus is the site of formation of serotonin

4. A 75-year-old man with a recent history of influenza vaccination presents to the emergency room with an acute onset
of paraparesis and urinary incontinence. He reported that his symptoms began a week earlier with dull, progressive low
back pain that soon resulted in bilateral leg weakness. The most likely diagnosis in this case is:

A. 
Spinal cord metastases
B. 
Acute spinal cord compression
C. 
Vacuolar myelopathy
D. 
Transverse myelitis
E. 
Acute disseminated encephalomyelitis

Full test - and additional VIDEO vignettes - available online - see inside front cover for details.

191

Psychiatry Test Preparation and Review


Manual E-Book
Psychiatry Test Preparation & Review Manual

5. A doctor in a certain hospital makes a diagnosis for a particular patient. That diagnosis is considered reliable if:

A. 
It is accurate
B. 
Many different doctors in different locations would agree upon the same diagnosis
C. 
The disorder has features characteristic enough to distinguish it from other disorders
D. 
The disorder allows doctors to predict the clinical course and treatment response
E. 
The diagnosis is based on an understanding of the underlying pathophysiology and has biological markers

6. Which one of the following sets of symptoms and signs correctly identifies Horner’s syndrome?

A. 
Ptosis, miosis, sweating
B. 
Ptosis, mydriasis, sweating
C. 
Ptosis, miosis, anhidrosis
D. 
Ptosis, mydriasis, anhidrosis
E. 
Lid-lag, miosis, anhidrosis

7. A patient presents with the following symptoms: tremor, halitosis, dry mouth, tachycardia, hypertension, fever, euphoria,
alertness, agitation, paranoia, hallucinations, and irritability. Which one of the following substances is the most likely
cause?

A. 
Amphetamines
B. 
Opioids
C. 
Alcohol
D. 
Barbiturates
E. 
Benzodiazepines

8. The inability to recognize familiar faces is known as:

A. 
Anosognosia
B. 
Simultanagnosia
C. 
Aprosodia
D. 
Prosopagnosia
E. 
Astereognosis

9. Which one of the following is not a potential effect of phencyclidine (PCP) use?

A. 
Paranoia
B. 
Nystagmus
C. 
Catatonia
D. 
Convulsions
E. 
Hypotension

10. A characteristic of a facial nerve (Bell’s) palsy that clearly distinguishes it from a stroke-related facial paresis is:

A. 
The presence of miosis
B. 
The involvement of the whole face
C. 
Anhidrosis
D. 
Rapid recovery of motor functioning
E. 
A postinfectious onset

192

Psychiatry Test Preparation and Review


Manual E-Book
Test Number Four

11. Object permanence develops during which one of Piaget’s developmental stages?

A. 
Sensorimotor
B. 
Preoperational thought
C. 
Concrete operations
D. 
Formal thought
E. 
Rapprochement

12. Which one of the following is not a classic feature of syringomyelia?

A. 
Spasticity
B. 
Muscular atrophy
C. 
Fasciculations
D. 
Loss of temperature and pain sensation
E. 
Preservation of proprioception

13. A patient comes into the emergency room and admits to sniffing glue daily for the past 8 months. Which one of the fol-
lowing is not your concern as this patient’s physician?

A. 
Liver damage
B. 
Permanent brain damage
C. 
Kidney damage
D. 
Myocardial damage
E. 
Urinary retention

14. One of the most common causes of the movement disorder of opsoclonus–myoclonus in the infant is:

A. 
Neonatal seizures
B. 
Craniopharyngioma
C. 
Prematurity
D. 
Neuroblastoma
E. 
Meningioma

15. In the emergency room, a physician examines a female patient who has recently been diagnosed with a social phobia.
Which one of the following answer choices would most likely be the greatest fear for this patient?

A. 
Having to take responsibility for planning a dinner with her husband
B. 
Being in a relationship with a new boyfriend
C. 
Going to a state fair and being around thousands of people
D. 
Being scrutinized by others
E. 
Competing for a new position that just opened up in her company

16. What is the psychiatrist’s role in the collaborative care model?

A. 
Direct consultation with the patient
B. 
Cognitive behavioral therapy supervision
C. 
Family therapy
D. 
Administrative oversight
E. 
Indirect consultation with the primary care physician, care managers, and other mental health professionals

193

Psychiatry Test Preparation and Review


Manual E-Book
Psychiatry Test Preparation & Review Manual

17. Which one of the following statements is not true regarding body dysmorphic disorder?

A. 
It is a preoccupation with an imagined defect in appearance
B. 
It is most commonly associated with a comorbid mood disorder
C. 
The most common concerns involve facial flaws
D. 
Treatment with surgical, medical, dental, or dermatological care is usually successful
E. 
If a slight physical anomaly is present, the concern is markedly excessive

18. Which one of the following primary brain tumors is the most common in patients over 60 years of age?

A. 
Anaplastic astrocytoma
B. 
Glioblastoma multiforme
C. 
Meningioma
D. 
Ependymoma
E. 
Acoustic neuroma

19. Patients with obsessive–compulsive disorder are noted to have anomalies of which one of the following brain regions?

A. 
Corpus callosum
B. 
Striatum
C. 
Hippocampus
D. 
Caudate nucleus
E. 
Cerebellum

20. Anton’s syndrome results from a stroke that localizes to the:

A. 
Frontal lobes
B. 
Temporal lobes
C. 
Parietal lobes
D. 
Occipital lobes
E. 
Cerebellar hemispheres

21. Which one of the following should not be considered a predictive factor for violence?

A. 
Alcohol intoxication
B. 
Recent acts of violence
C. 
Command auditory hallucinations
D. 
High socioeconomic status
E. 
Menacing other people

22. Riluzole, the only FDA-approved agent for the treatment of amyotrophic lateral sclerosis, affects which one of the fol-
lowing neurotransmitters?

A. 
Glutamate
B. 
GABA
C. 
Acetylcholine
D. 
Dopamine
E. 
NE

194

Psychiatry Test Preparation and Review


Manual E-Book
Test Number Four

23. A 29-year-old patient is admitted to the neurology unit for evaluation of seizures. Workup is negative, and there has not
been any seizure activity captured on electroencephalogram during his seizure episodes. Neurological examination is
negative. The patient is noted to be very sad, and a psychiatric consult is called. Nurses have noted conflict between the
patient and his wife during her visits. As the consultant, which one of the following would be the highest on your list of
differential diagnoses?

A. 
Obsessive–compulsive disorder
B. 
Conversion disorder (functional neurological symptom disorder)
C. 
Somatic symptom disorder
D. 
Social phobia
E. 
Panic disorder

24. What is the most common central nervous system (CNS) cancer noted in patients with advanced acquired immunodefi-
ciency syndrome (AIDS)?

A. 
Glioblastoma multiforme
B. 
Lymphoma
C. 
Meningioma
D. 
High-grade brain-stem glioma
E. 
Epidermoid tumor

25. The highest rate of synapse formation in the brain takes place during which one of the following time periods?

A. 
Adolescence
B. 
Weeks 32 to 35 of gestation
C. 
Weeks 13 to 26 of gestation
D. 
Within the first 6 weeks of gestation
E. 
As a toddler

26. Which one of the following is not a presenting feature of AIDS dementia complex?

A. 
Poor attention and concentration
B. 
Slowness of thinking
C. 
Personality changes
D. 
Apathy
E. 
Hemiparesis

27. Which one of the following is not a medical complication of weight loss in eating disorders?

A. 
Cachexia
B. 
Loss of cardiac muscle
C. 
Delayed gastric emptying
D. 
Lanugo
E. 
Increased bone density

28. The most frequent opportunistic CNS infection in the AIDS patient is:

A. 
CNS toxoplasmosis
B. 
Cryptococcal meningitis

195

Psychiatry Test Preparation and Review


Manual E-Book
Psychiatry Test Preparation & Review Manual

C. 
Herpes meningitis
D. 
Cytomegalovirus encephalitis
E. 
Neurosyphilis

29. Which one of the following is not a medical complication of purging seen in eating disorders?

A. 
Electrolyte abnormalities
B. 
Salivary gland inflammation
C. 
Erosion of dental enamel
D. 
Hyperkalemia
E. 
Seizures

30. A 45-year-old man with end-stage AIDS and CD4 count of 50/μL presents to the emergency room with a complaint of
rapidly progressive onset of gait difficulty, spasticity, leg weakness, sphincter dysfunction, and loss of proprioception to
both feet and legs. The most likely diagnosis in this case is:

A. 
Progressive multifocal leukoencephalopathy
B. 
Distal sensory polyneuropathy
C. 
Vacuolar myelopathy
D. 
Neurosyphilis
E. 
Human T-lymphotropic virus type 1 (HTLV-1) myelopathy

31. 
Which one of the following medical conditions should not be considered when evaluating patients with anxiety
disorders?

A. 
Carcinoid syndrome
B. 
Hyperventilation syndrome
C. 
Hypoglycemia
D. 
Hyperthyroidism
E. 
Central serous chorioretinopathy

32. A 45-year-old man with end-stage AIDS and CD4 count of 0/μL presents
to the emergency room with a complaint of progressive onset over the past
few weeks of ataxia, visual field deficits, altered mental status, aphasia, and
fluctuating sensory deficits. T2-weighted brain magnetic resonance imaging
(MRI) reveals the image shown here. The most likely diagnosis in this case is:

A. 
CNS toxoplasmosis
B. 
Lymphoma
C. 
Neurosyphilis
D. 
AIDS dementia complex
E. 
Progressive multifocal leukoencephalopathy

33. 
When diagnosing a patient with social phobia, which diagnosis is most
important for the treating psychiatrist to keep in mind?

A. 
Paranoid personality disorder
B. 
Schizoid personality disorder

196

Psychiatry Test Preparation and Review


Manual E-Book
Test Number Four

C. 
Obsessive–compulsive personality disorder
D. 
Avoidant personality disorder
E. 
Borderline personality disorder

34. Which one of the following is not a prion disease?

A. 
Kuru
B. 
Gerstmann–Sträussler–Scheinker syndrome
C. 
Fatal familial insomnia
D. 
Devic’s syndrome
E. 
Creutzfeldt–Jakob disease

35. A psychiatrist is covering the emergency room, and a patient comes in who has a previous diagnosis of bipolar II disor-
der. Based on this diagnosis, which one of the following symptoms would the knowledgeable psychiatrist expect to see
in this patient over time?

A. 
Psychotic features
B. 
Manic episodes that do not respond to treatment with mood stabilizers
C. 
Rapid cycling between severe depression and mania
D. 
Recurrent manic episodes in the absence of depression
E. 
Recurrence of both depressive and hypomanic episodes

36. The classic triad of headache, ipsilateral Horner’s syndrome, and contralateral hemiparesis is generally due to:

A. 
Carotid artery occlusion
B. 
Giant cell arteritis
C. 
Wallenberg’s syndrome
D. 
Pontine hemorrhage
E. 
Cerebral aneurysmal rupture

37. A set of statistical procedures designed to compare two or more groups of observations and determine whether the dif-
ferences are due to chance or experimental difference is called:

A. 
Correlation coefficient
B. 
Control group
C. 
Analysis of variance
D. 
Regression analysis
E. 
Null hypothesis

38. A 55-year-old woman presents to the emergency room with a sudden and acute onset of right-sided painless complete
facial paralysis involving both the upper and the lower parts of the face. The symptoms began earlier that day and
were initially accompanied by a mild ache behind the ear, which resolved. The organism most likely responsible for
this condition is:

A. 
Borrelia burgdorferi
B. 
Herpes simplex virus
C. 
Epstein–Barr virus
D. 
West Nile virus
E. 
Varicella zoster virus

197

Psychiatry Test Preparation and Review


Manual E-Book
Psychiatry Test Preparation & Review Manual

39. A psychiatrist is asked to evaluate a child who does not make appropriate eye contact, fails to respond to the social cues
of others, lacks the ability for spontaneous make-believe play, and, on close examination, has a delay in language devel-
opment. What is this child’s most likely diagnosis?

A. 
Schizophrenia
B. 
Avoidant personality disorder
C. 
Schizoid personality disorder
D. 
Conduct disorder
E. 
Autism spectrum disorder

40. A veterinary radiologist presents to the emergency room with a 4-week history of headache, vague fever, and paresthe-
sias in the fingers and toes. His temperature is 103.5° F (39.7° C). He complains of difficulty swallowing with pharyn-
geal spasms for the past 3 days. The most likely diagnosis is:

A. 
West Nile virus infection
B. 
Tetanus infection
C. 
Acute botulism
D. 
Rabies
E. 
Epstein–Barr virus infection

41. Which one of the following receptor subtypes is associated with the neurotransmitter glutamate?

A. 
Nicotinic
B. 
Muscarinic
α1
C. 
α-Amino-3-hydroxy-5-methyl-4-isoxazolepropionic acid (AMPA)
D. 
E. 
GABA

42. A 60-year-old woman presents to the emergency room with a progressive downward course over the past 6 months
characterized by behavioral disinhibition, emotional lability, severe naming and word-finding difficulties, hyperorality,
stubbornness, inability to plan, and poor judgment. Autopsy of this patient’s brain would most likely reveal:

A. 
Hirano bodies
B. 
Pick’s inclusion bodies and gliosis
C. 
Lewy bodies
D. 
Neurofibrillary plaques and tangles
E. 
Severe white matter demyelination

43. Your friend had a baby who is now 8 months old. You and she talk about the child and note its temperament. At this
point in the child’s development you tell your friend that the child’s temperament is most likely a function of:

A. 
Biological factors
B. 
The parents’ culture
C. 
The grandmother’s influence on weekends
D. 
The baby’s birth month
E. 
The influence of the child’s siblings

198

Psychiatry Test Preparation and Review


Manual E-Book
Test Number Four

44. The neuropathological hallmark of idiopathic Parkinson’s disease is:

A. 
Brain-stem Lewy bodies
B. 
Hirano bodies
C. 
Amyloid plaques
D. 
Mesial temporal sclerosis
E. 
Caudate nucleus atrophy

45. A patient presents to your office with obsessive–compulsive disorder (OCD). She has been tried on several selective
serotonin reuptake inhibitors (SSRIs) but has shown little improvement. You decide to try a tricyclic. Which tricyclic has
been shown to have significantly better efficacy in treating OCD than the others?

A. 
Desipramine
B. 
Doxepin
C. 
Amitriptyline
D. 
Clomipramine
E. 
Nortriptyline

46. Which one of the following is not a possible symptom of fibromyalgia?

A. 
Headache
B. 
Psychosis
C. 
Depression
D. 
Sleep disturbance
E. 
Paresthesias

47. Pancreatic cancer is most often associated with which one of the following psychiatric disorders?

A. 
Psychosis
B. 
Anxiety
C. 
Depression
D. 
Impulse control disorders
E. 
Bulimia

48. A 75-year-old man with known history of prostate cancer presents to the emergency room with an acute onset of bilat-
eral leg weakness, leg spasticity, sensory loss to pain and temperature below the waist, and acute bladder and bowel
incontinence. The first test of choice to perform in the emergency room setting is:

A. 
Noncontrast head computed tomography (CT)
B. 
Brain MRI
C. 
Screening spine MRI
D. 
Spinal x-rays
E. 
HTLV-1 antibody titer

199

Psychiatry Test Preparation and Review


Manual E-Book
Psychiatry Test Preparation & Review Manual

49. Which one of the following symptoms would a psychiatrist look for in a child to make the diagnosis of conduct disorder
rather than depression, attention deficit hyperactivity disorder (ADHD), or bipolar disorder?

A. 
Irritable mood
B. 
Difficulty organizing tasks
C. 
Excessive activity
D. 
Starting fights with other children
E. 
Sleep disturbance

50. Subacute combined degeneration of the spinal cord is a result of deficiency in which one of the following:

A. 
Vitamin B12
B. 
Vitamin B1
C. 
Vitamin B6
D. 
Niacin
E. 
Folic acid

51. When patients who have been a victim of childhood incest become adults, which one of the following disorders are they
most prone to developing?

A. 
Anorexia
B. 
Bipolar disorder
C. 
Social phobia
D. 
Major depression
E. 
Conversion disorder (functional neurological symptom disorder)

52. A patient presents to the emergency room with an acute onset of pure right hemiparesis that, on examination, is noted
to affect the face, arm, and leg equally. There is no sensory deficit, and no cortical signs are noted. This stroke most
likely localizes to the:

A. 
Left thalamus and internal capsule
B. 
Left internal capsule
C. 
Right basis pontis
D. 
Right medulla
E. 
Left midbrain

53. A psychiatrist keeps a medical chart and a separate set of psychotherapy notes for one of her patients. The patient’s
records are subpoenaed by a court. Which of the following is true?

A. 
The medical chart only must be turned over to the court
B. 
Both the medical chart and the psychotherapy notes must be turned over to the court
C. 
Only the psychotherapy notes should be turned over to the court
D. 
Neither record should be turned over unless a HIPAA (Health Insurance Portability and Accountability Act) release
is signed by the patient
E. 
The psychotherapy notes should be turned over to the court only if specifically subpoenaed

200

Psychiatry Test Preparation and Review


Manual E-Book
Test Number Four

54. Which one of the following is not a symptom or sign of Parkinson’s disease?

A. 
Bradykinesia
B. 
Loss of postural reflexes
C. 
Tremor
D. 
Choreoathetosis
E. 
Rigidity

55. A psychiatrist is asked to evaluate an 8-year-old girl. She does not want to go to school and refuses to do her homework.
Her teacher reports that she will not read out loud in class. She likes to go and spend the weekends at friends’ houses or
go on overnight trips with her grandparents. Her IQ is average. What diagnosis should the informed psychiatrist most
strongly consider?

A. 
ADHD
B. 
Conduct disorder
C. 
Separation-anxiety disorder
D. 
Autism spectrum disorder
E. 
Specific learning disorder with impairment in reading

56. Which one of the following is not a feature of botulism toxin poisoning?

A. 
Myoclonus
B. 
Dysphagia
C. 
Diplopia
D. 
Nausea
E. 
Urinary retention

57. A couple brings their son in to see a psychiatrist. The child fights with his mother and father and is rude and dismissive
toward them. He states that he wants to leave, and when the doctor tells him he has to stay, he yells, curses, cries, and
rolls around on the floor. His teacher tells the psychiatrist that his work at school is good but that he gets very nasty
with her when she tells him to do a particular task, and he often refuses to cooperate with her. The child’s most likely
diagnosis is:

A. 
ADHD
B. 
Bipolar disorder
C. 
Conduct disorder
D. 
Oppositional defiant disorder
E. 
Separation-anxiety disorder

58. Bálint’s syndrome is the result of a lesion to which one of the following areas?

A. 
Frontal lobe
B. 
Temporal lobe
C. 
Parietal lobe
D. 
Occipital lobe
E. 
Bilateral parietal–occipital lobes

201

Psychiatry Test Preparation and Review


Manual E-Book
Psychiatry Test Preparation & Review Manual

59. A psychiatrist evaluates a 7-year-old patient who is brought in by his parents because of complaints they have been
receiving from school. The child has been sexually provocative with other children, sexualizes play activities, and openly
displays sexual behavior. The most likely cause of this behavior is:

A. 
Normal development
B. 
Early-onset puberty
C. 
Traumatic brain injury
D. 
Sexual abuse
E. 
Psychosis with sexual delusion

60. Which one of the following symptoms is a feature of Anton’s syndrome?

A. 
Aphasia
B. 
Prosopagnosia
C. 
Apraxia
D. 
Confabulation
E. 
Hiccups

61. Which one of the following personality disorders is least associated with violent behavior?

A. 
Borderline personality disorder
B. 
Histrionic personality disorder
C. 
Narcissistic personality disorder
D. 
Dependent personality disorder
E. 
Antisocial personality disorder

62. Inhalant intoxication (sniffing glue) causes which one of the following neurologic conditions?

A. 
Myopathy
B. 
Neuropathy
C. 
Myelopathy
D. 
Denervation of muscle
E. 
Seizures

63. A 30-year-old man presents to the emergency room with a complaint that there is a cockroach living in his rectum. He
says he knows that there is a hole in the side of his rectum that was created by the roach. He has no prior psychiatric
history but states that he has felt the roach crawling around in his rectum for the past 18 months. He is a lawyer with a
busy successful practice, and he says he has no problems at work. During previous trips to the doctor for this complaint,
he was examined and told that there was no cockroach in his rectum. He feels that they just did not examine him prop-
erly, otherwise the roach would have been found. Examination and blood work are all normal. His most likely diagnosis
is:

A. 
Conversion disorder (functional neurological symptom disorder)
B. 
Schizophrenia
C. 
Depression with psychotic features
D. 
Delusional disorder somatic type
E. 
Illness anxiety disorder

202

Psychiatry Test Preparation and Review


Manual E-Book
Test Number Four

64. Which one of the following substances of abuse is the most likely to lower the seizure threshold during intoxication?

A. 
Morphine
B. 
PCP
C. 
Cocaine
D. 
Cannabis
E. 
Alcohol

65. Five patients are brought into the emergency room on a Friday evening. Of the five, which one is most likely to kill him-
or herself successfully?

A. 
Bob, who has schizophrenia
B. 
Carol, who has alcoholism
C. 
Dave, who has intellectual developmental disorder
D. 
Sally, who has borderline personality disorder
E. 
Mark, who has major depressive disorder

66. Meige’s syndrome comprises which one of the following sets of symptoms?

A. 
Hemifacial spasm and seizures
B. 
Hemifacial spasm and cervical dystonia
C. 
Blepharospasm and ptosis
D. 
Blepharospasm and oromandibular dystonia
E. 
Lid apraxia and myokymia

67. A physician who has reason to believe a patient may kill or injure another person must notify the potential victim,
authorities, or the victim’s family or friends. This is the result of which one of the following answer choices?

A. 
Durham rule
B. 
M’Naghten rule
C. 
Ford v Wainwright
D. 
Tarasoff rule
E. 
Respondeat superior

68. The phenomenon of scanning speech results from a lesion to the:

A. 
Cerebellum
B. 
Thalamus
C. 
Frontal lobes
D. 
Midbrain
E. 
Dominant temporal lobe

69. A friend just gave birth to a healthy baby boy 5 days ago. Now your friend is crying and irritable and has been dys-
phoric over the past 2 days. Which one of the following is the most likely diagnosis based on the information given?

A. 
Postpartum depression
B. 
Postpartum blues
C. 
Postpartum psychosis
D. 
Postpartum bipolar disorder
E. 
Specific phobia of being a parent

203

Psychiatry Test Preparation and Review


Manual E-Book
Psychiatry Test Preparation & Review Manual

70. Which one of the following is a syndrome of near muteness with normal reading, writing, and comprehension?

A. 
Aphasia
B. 
Apraxia
C. 
Agnosia
D. 
Aphemia
E. 
Abulia

71. Which one of the following is not true regarding the N-methyl-d-aspartate (NMDA) receptor?

A. 
The NMDA receptor has been linked with learning and memory
B. 
The NMDA receptor allows for the passage of potassium only
C. 
The NMDA receptor opens only when it has bound two molecules of glutamate and one molecule of glycine
D. 
The NMDA receptor can be blocked by physiological concentrations of magnesium
E. 
The NMDA receptor can be blocked by PCP

72. Which one of the following refers to a state of unresponsiveness from which arousal occurs only with vigorous and
repeated stimulation?

A. 
Alertness
B. 
Lethargy
C. 
Stupor
D. 
Coma
E. 
Persistent vegetative state

73. Of the following disorders, which one has the best prognosis?

A. 
Somatic symptom disorder
B. 
Body dysmorphic disorder
C. 
Somatic symptom disorder with predominant pain
D. 
Illness anxiety disorder
E. 
Conversion disorder (functional neurological symptom disorder)

74. Which one of the following statements is true?

A. 
All patients with acute Guillain–Barré syndrome should be hospitalized in an intensive care unit in case of respira-
tory compromise
B. 
All comatose patients require a head CT scan before a lumbar puncture is performed
C. 
A positive grasp reflex is always a sign of frontal lobe damage
D. 
Cerebellar hemispheric lesions produce deficits that are contralateral to the lesion
E. 
Bell’s palsy is most often caused by B. burgdorferi infection

75. Which one of the following choices is considered unethical by the American Psychiatric Association?

A. 
Closing a practice and finding follow-up care for your patients
B. 
Refusing to discuss a patient’s case with her family unless she gives you permission
C. 
Charging a colleague rent to sublet office space from you
D. 
A patient wills you his or her estate after death, and you accept and use the money for a new car
E. 
You charge a fee to supervise another psychiatrist

204

Psychiatry Test Preparation and Review


Manual E-Book
Test Number Four

76. Emotional memory localizes to the:

A. 
Amygdala
B. 
Hippocampus
C. 
Primary auditory cortex
D. 
Nucleus basalis of Meynert
E. 
Pons

77. Of the following medications, which one is the least likely to exacerbate urinary retention in a patient with BPH?

A. 
Amitriptyline
B. 
Imipramine
C. 
Desipramine
D. 
Nortriptyline
E. 
Maprotiline

78. By what age should a child have a six-word vocabulary, be able to self-feed, and be able to walk up steps with its hand
being held?

A. 
6 months
B. 
9 months
C. 
12 months
D. 
18 months
E. 
24 months

79. A 30-year-old male is brought to the emergency room after being arrested for exposing his genitals to women on the
train. He states that he has impulses to expose himself that he can’t control, and that he finds the whole experience very
sexually exciting. Which one of the following medications would be an appropriate treatment for this patient?

A. 
Medroxyprogesterone acetate
B. 
Lorazepam
C. 
Ziprasidone
D. 
Duloxetine
E. 
Chlorpromazine

80. A 45-year-old woman presents to the emergency room with an acute left hemiparesis of the arm and leg. You ask her
to lift her normal right leg while you put your hand under her paretic leg. You note that while lifting her good leg, she
pushes her affected leg downward on the bed with normal strength. You suspect a hysterical or psychogenic disorder.
This phenomenon is termed:

A. 
Hoffman’s sign
B. 
Hoover’s sign
C. 
Lasègue’s sign
D. 
Romberg sign
E. 
Gegenhalten

205

Psychiatry Test Preparation and Review


Manual E-Book
Psychiatry Test Preparation & Review Manual

81. Clomipramine is not used for which one of the following?

A. 
Depression
B. 
Obsessive–compulsive disorder
C. 
Panic disorder
D. 
Premature ejaculation
E. 
Command auditory hallucinations

82. The only true emergency in neurology that requires immediate MRI and evaluation in the emergency room is:

A. 
Acute suspected early hemispheric stroke
B. 
Acute suspected myasthenia gravis
C. 
Acute suspected spinal cord compression
D. 
Acute suspected Guillain–Barré syndrome
E. 
Acute suspected subarachnoid hemorrhage

83. Which one of the following terms refers to the state’s right to intervene and act as a surrogate parent for those who can-
not care for themselves?

A. 
Actus reus
B. 
Mens rea
C. 
Parens patriae
D. 
Durable power
E. 
Respondeat superior

84. A female patient suffers from central obesity, hypertension, hirsutism, amenorrhea, acne, hyperpigmentation, striae,
insomnia, depression, and anxiety. These symptoms are most consistent with which one of the following?

A. 
Cushing’s disease
B. 
HIV
C. 
Addison’s disease
D. 
Psoriasis
E. 
Arsenic poisoning

85. Which one of the following answer choices is measured by the trail-making test?

A. 
Memory
B. 
Language
C. 
Social learning
D. 
Psychosis
E. 
Executive function

86. The principal mechanism of action of the Alzheimer’s disease agent memantine involves which one of the following
receptors?

A. 
Acetylcholine
B. 
Dopamine
C. 
NMDA
D. 
Glycine
E. 
GABA

206

Psychiatry Test Preparation and Review


Manual E-Book
Test Number Four

87. Which one of the GABA receptors is thought to be the site of action of the benzodiazepines?

A. 
GABA-A
B. 
GABA-B
C. 
GABA-C
D. 
GABA-D
E. 
GABA-E

88. Vagal nerve stimulation is FDA-approved for which one of the following indications?

A. 
Refractory epilepsy
B. 
Bipolar mania
C. 
Schizophrenia
D. 
Intermittent explosive disorder
E. 
Obsessive–compulsive disorder

89. Which one of the following statements is true regarding carbamazepine?

A. 
Carbamazepine is approved in the United States for treatment of temporal lobe epilepsy and general epilepsy but
not trigeminal neuralgia
B. 
Carbamazepine is metabolized by the kidneys
C. 
Carbamazepine can be associated with a transient increase in the white blood cell count
D. 
Carbamazepine has been shown to be as effective as the benzodiazepines in some studies for management of alco-
hol withdrawal
E. 
A benign pruritic rash occurs in 60% to 70% of patients treated with carbamazepine

90. Auscultation of the head that reveals a bruit would probably be indicative of which one of the following?

A. 
Brain tumor
B. 
Venous sinus thrombosis
C. 
Temporal arteritis
D. 
Intracranial aneurysm
E. 
Arteriovenous malformation

91. Which one of the following tests is considered to be projective?

A. 
Halstead–Reitan battery
B. 
Stanford–Binet test
C. 
Wechsler–Bellevue test
D. 
Draw a person test
E. 
Minnesota Multiphasic Personality Inventory (MMPI)

92. A 47-year-old man presents to the emergency room with intermittent headaches and
periodic drop attacks. His brain MRI on T1-weighted imaging reveals the scan shown
here. The most likely diagnosis is:

A. 
Choroid plexus papilloma
B. 
Colloid cyst of the third ventricle
C. 
Ependymoma
D. 
Pineal region germinoma
E. 
Pituitary macroadenoma

207

Psychiatry Test Preparation and Review


Manual E-Book
Psychiatry Test Preparation & Review Manual

93. Which one of the following drugs is both an opioid agonist and antagonist?

A. 
Aripiprazole
B. 
Naltrexone
C. 
Buprenorphine
D. 
Methadone
E. 
Gabapentin

94. Hallervorden–Spatz syndrome is a neurodegenerative disorder that results from lesions localizing to the:

A. 
Frontal lobes
B. 
Parietal lobes
C. 
Occipital lobes
D. 
Basal ganglia
E. 
Hippocampus

95. A patient presents to the emergency room because of alcohol withdrawal. He and his family describe a history of
alcohol-induced blackouts. Which one of the following memory problems is most consistent with alcohol-induced
blackouts?

A. 
Making up details of how he got to work 3 days ago
B. 
Retrograde amnesia
C. 
Anterograde amnesia
D. 
Loss of memories from his daughter’s birthday 5 years ago
E. 
Inability to tell you who the current president is

96. Strict vegetarians who ingest no meat products can suffer from deficits in proprioception and vibration sensation due to
lesions that localize to the:

A. 
Posterior spinal cord
B. 
Central spinal cord
C. 
Anterior spinal cord
D. 
Thalamus
E. 
Peripheral sensory nerves

97. A psychiatrist is treating a bipolar patient with carbamazepine. After being started on the drug, he has therapeutic serum
levels. Three months later the patient starts to become hypomanic, and the psychiatrist decides to check a level. The level
comes out below therapeutic range. Both the patient and his family reassure the psychiatrist that he has been taking the
medication regularly. What should the psychiatrist do with this patient?

A. 
Confront him, because he and his family are lying
B. 
Stop the carbamazepine and put him on divalproex sodium
C. 
Increase the dose of carbamazepine and take follow-up serum levels
D. 
Add a high dose of a selective serotonin reuptake inhibitor
E. 
Hospitalize the patient

208

Psychiatry Test Preparation and Review


Manual E-Book
Test Number Four

98. Patients exposed to isoniazid (INH) for tuberculosis treatment can develop a sensory polyneuropathy as a result of a
deficiency in which one of the following?

A. 
Vitamin B6
B. 
Vitamin B12
C. 
Niacin
D. 
Thiamine
E. 
Vitamin A

99. What is the best way to handle suicidal patients with borderline personality disorder?

A. 
Take the threats seriously and take whatever steps are necessary to protect these patients
B. 
Do not discuss suicide with them
C. 
Isolate these patients from friends and family
D. 
Make these patients promise not to hurt themselves (contract for safety)
E. 
Give these patients benzodiazepine prescriptions to calm them down

100. The West Nile virus is considered to belong to which one of the following viral families?

A. 
Arenaviruses
B. 
Arboviruses
C. 
Filoviruses
D. 
Papovaviruses
E. 
Retroviruses

101. A patient is on lithium, risperidone, and a daily aspirin. He comes to his session confused and unsteady on his feet. He
states that he has the flu, based on recent gastrointestinal upset and fever. Which one of the following should the psy-
chiatrist do with this patient first?

A. 
Refer him to an internist
B. 
Get a lithium level
C. 
Review the patient’s recent diet
D. 
Send stool for ova and parasites
E. 
Obtain a complete blood count

102. Which one of the following neurotransmitters localizes predominantly to the basal forebrain and is responsible for
memory, attention, and executive functioning?

A. 
Serotonin
B. 
NE
C. 
GABA
D. 
Glycine
E. 
Acetylcholine

103. Which one of the following drugs does not act by blocking the reuptake of NE into the presynaptic neuron?

A. 
Imipramine
B. 
Venlafaxine
C. 
Bupropion
D. 
Nefazodone
E. 
Mirtazapine

209

Psychiatry Test Preparation and Review


Manual E-Book
Psychiatry Test Preparation & Review Manual

104. What is the mechanism of action of the hallucinogen PCP?

A. 
Dopamine antagonism
B. 
Serotonin antagonism
C. 
NE antagonism
D. 
NMDA antagonism
E. 
Acetylcholinesterase inhibition

105. A physician needs to give a benzodiazepine to someone with impaired liver function. Which one of the following would
be the best choice of medication in this situation?

A. 
Diazepam
B. 
Oxazepam
C. 
Clonazepam
D. 
Prazepam
E. 
Estazolam

106. Which one of the following fungal organisms can cause vertebrobasilar strokes by invasion of vessel walls and tends to
colonize in the paranasal sinuses and cause a hypersensitivity pneumonitis?

A. 
Histoplasma
B. 
Candida albicans
C. 
Aspergillus
D. 
Cryptococcus neoformans
E. 
Pseudallescheria boydii

107. Which one of the following side effects is most likely to develop in a patient started on fluoxetine?

A. 
Loss of consciousness
B. 
Shuffling gait
C. 
Headache
D. 
High blood pressure
E. 
Blurred vision

108. The GABA-A receptor (the most predominant GABA receptor) is which one of the following:

A. 
A sodium channel
B. 
A chloride channel
C. 
A calcium channel
D. 
A potassium channel
E. 
A magnesium channel

109. A psychiatrist made a minor error in her last session with a patient. The patient comes to her for psychodynamic psy-
chotherapy. The best approach is to:

A. 
Interpret the patient’s reaction
B. 
Ignore the mistake
C. 
Give a long but clear explanation of her reasoning
D. 
Briefly acknowledge that she made a mistake
E. 
Profusely apologize

210

Psychiatry Test Preparation and Review


Manual E-Book
Test Number Four

110. A failure to develop a cohesive self-awareness is known as:

A. 
Entrapment
B. 
Climacterium
C. 
Identity diffusion
D. 
Activity-dependent modulation
E. 
All-or-none phenomenon

111. What is the most important step in treating separation-anxiety disorder in an 11-year-old?

A. 
Give methylphenidate
B. 
Give risperidone
C. 
Rapidly send the child back to school
D. 
Thorough psychoanalysis of the mother
E. 
High-dose benzodiazepine treatment

112. Why do doctors use naltrexone for alcohol abuse?

A. 
It is almost 100% effective
B. 
It blocks the effects of alcohol at the GABA receptor
C. 
It alters dopamine levels to decrease pleasure from drinking
D. 
It has been shown to decrease craving and decrease alcohol consumption
E. 
It is better than behavioral modification in treating alcohol abuse

113. A 30-year-old man on imipramine complains of difficulty urinating and impotence. What should his doctor do for him?

A. 
Increase the dose of imipramine
B. 
Tell the patient to decrease fluid intake
C. 
Tell the patient to stop all sexual activity
D. 
Prescribe bethanechol
E. 
Prescribe melatonin

114. Which one of the following statements about mood disorders is true?

A. 
Major depression is more common in men than in women
B. 
Bipolar disorder has equal prevalence for men and women
C. 
Higher socioeconomic status leads to increased depression
D. 
There is a correlation between the hyposecretion of cortisol and depression
E. 
About 90% of those with major depressive disorder receive specific treatment

115. A mother brings a 26-month-old child into the doctor’s office. The child has not spoken any words yet. How should the
doctor proceed?

A. 
Speech therapy
B. 
Audiometry
C. 
Sensory evoked potentials
D. 
Tell the mother to give it more time
E. 
Chromosomal analysis

211

Psychiatry Test Preparation and Review


Manual E-Book
Psychiatry Test Preparation & Review Manual

116. The assisted recall of information by a person in the same external environment in which the information was originally
acquired is known as:

A. 
Classical conditioning
B. 
Social learning
C. 
Partial recovery
D. 
Respondent conditioning
E. 
State-dependent learning

117. A 29-year-old man comes into the hospital with complaint of confusion, ataxia, disorientation, and dysarthria. He has
the smell of alcohol on his breath. Which one of the following is the best first step for the physician to take?

A. 
Phone the patient’s primary care physician
B. 
Speak with the patient’s family
C. 
Give intravenous thiamine
D. 
Sedate the patient with haloperidol
E. 
Give the patient an anticonvulsant

118. Niacin deficiency (pellagra) results in which one of the following classic triads of symptoms?

A. 
Gastritis, neuropathy, stroke
B. 
Dementia, dermatitis, diarrhea
C. 
Neuropathy, ataxia, dementia
D. 
Neuropathy, retinopathy, areflexia
E. 
Neuropathy, spasticity, encephalopathy

119. A psychiatrist is doing psychodynamic psychotherapy with a patient. The patient is usually on time but missed a session
last Tuesday. When he comes back, how should the psychiatrist approach this issue?

A. 
Do not mention the missed appointment
B. 
Refuse to treat the patient anymore
C. 
“You missed your appointment Tuesday. I was wondering what happened.”
D. 
“I’m glad you didn’t show on Tuesday. I spent the time with a patient I like better than you.”
E. 
“I’m going to charge you twice the normal fee because you missed your appointment last Tuesday.”

120. Which one of the following is not a sign of cannabis intoxication?

A. 
Conjunctival injection
B. 
Increased appetite
C. 
Dry mouth
D. 
Bradycardia
E. 
Orthostatic hypotension

121. A patient comes to his psychiatrist’s office with complaints consistent with akathisia. Which one of the following would
be the best treatment?

A. 
Bupropion
B. 
Amoxapine
C. 
Vitamin B6
D. 
Captopril
E. 
Propranolol

212

Psychiatry Test Preparation and Review


Manual E-Book
Test Number Four

122. What is the mechanism by which clonidine is effective in reducing symptoms of opiate withdrawal?

A. 
Indirect dopamine blockade
B. 
Serotonin increase in the locus ceruleus
Agonist activity at α2 adrenergic receptors
C. 
D. 
Generation of the metabolite trichloroethanol
E. 
Decreased free T4

123. Blockade of muscarinic cholinergic receptors will not lead to which one of the following?

A. 
Difficulty urinating
B. 
Improvement in Alzheimer’s symptoms
C. 
Dry mouth
D. 
Blurred vision
E. 
Delirium

124. Which one of the following laboratory tests is most likely to pick up alcohol abuse?

γ-Glutamyl transferase (GGT)


A. 
B. 
Mean corpuscular volume (MCV)
C. 
Uric acid
D. 
Serum glutamic–oxaloacetic transaminase (SGOT)
E. 
Serum glutamic–pyruvic transaminase (SGPT)

125. A psychiatrist is asked by a primary care physician to treat a patient with Tourette’s disorder. He is suffering from sev-
eral motor tics. Which one of the following would be the best medication to give him?

A. 
Propranolol
B. 
Pimozide
C. 
Paroxetine
D. 
Pindolol
E. 
Piroxicam

126. On which chromosome is the gene for amyloid precursor protein found?

A. 
Chromosome 19
B. 
Chromosome 20
C. 
Chromosome 21
D. 
Chromosome 4
E. 
Chromosome 13

127. Which one of the following answer choices is central to Kohut’s theories of self psychology?

A. 
The theory of Oedipal conflict
B. 
The concept of the good enough mother
C. 
The paranoid–schizoid position
D. 
The necessity for parental mirroring and empathic responsiveness to the child
E. 
The importance of the depressive position

213

Psychiatry Test Preparation and Review


Manual E-Book
Psychiatry Test Preparation & Review Manual

128. A patient with bipolar disorder is on carbamazepine. He goes to his primary care physician and is placed on erythromy-
cin. What should be expected to happen?

A. 
Carbamazepine levels will go down
B. 
Erythromycin levels will go down
C. 
No interaction of any kind
D. 
Carbamazepine levels will go up
E. 
Erythromycin levels will go up

129. A patient with bipolar disorder gives birth to a child with spina bifida and hypospadias. What is the most likely cause
for the child’s defects?

A. 
Genetics
B. 
Intrauterine infection
C. 
Haloperidol use during pregnancy
D. 
Valproic acid use during pregnancy
E. 
Lithium use during pregnancy

130. A patient presents with decreased energy, increased appetite, weight gain, increased sleep, decreased mood, lack of inter-
est in usual activities, and social withdrawal. Which one of the following medications would be the best choice to treat
him?

A. 
Citalopram
B. 
Lithium
C. 
Desipramine
D. 
Phenelzine
E. 
Venlafaxine

131. A patient states that he was given an antidepressant in the past but does not remember the name. He does remember
having his blood pressure checked regularly by the psychiatrist because of the antidepressant. Which one of the follow-
ing was the patient most likely taking?

A. 
Mirtazapine
B. 
Paroxetine
C. 
Venlafaxine
D. 
Citalopram
E. 
Fluoxetine

132. Which one of the following is the focus of interpersonal therapy?

A. 
Anxiety management
B. 
Belief systems
C. 
Faulty cognitions
D. 
Social interactions
E. 
Transference

214

Psychiatry Test Preparation and Review


Manual E-Book
Test Number Four

133. Which one of the following is not an aspect of experiments carried out by Nikolaas Tinbergen?

A. 
Quantifying the power of certain stimuli in eliciting specific behavior
B. 
Displacement activities
C. 
Innate releasing mechanisms
D. 
Autism
E. 
Imprinting

134. Which one of the following structures is most critical to the formation of memory?

A. 
Right frontal lobe
B. 
Right parietal lobe
C. 
Thalamus
D. 
Cerebellum
E. 
Hippocampus

135. Which one of the following is not a rating scale used for mood disorders?

A. 
Beck depression inventory
B. 
Zung self-rating scale
C. 
Carroll rating scale
D. 
Montgomery–Åsberg scale
E. 
Brief psychiatric rating scale (BPRS)

136. Which one of the following is not a characteristic of sleep terrors (non–rapid eye movement sleep arousal disorder)?

A. 
Awakening from sleep and screaming
B. 
Autonomic arousal
C. 
Recall of a detailed dream
D. 
Sweating
E. 
Unresponsiveness to attempts to comfort the person during the episode

137. Thioridazine is most often associated with which one of the following side effects?

A.
Hematuria
B. 
Delayed orgasm
C. 
Retrograde ejaculation
D. 
Priapism
E. 
Hypospadias

138. What is the mechanism of action of donepezil?

A. 
Dopamine blockade
B. 
Serotonin reuptake inhibition
C. 
Acetylcholinesterase inhibition
D. 
Increasing GABA activity
Prevention of β-amyloid deposition
E. 

215

Psychiatry Test Preparation and Review


Manual E-Book
Psychiatry Test Preparation & Review Manual

139. Depression may present differently in different cultures. How would the knowledgeable psychiatrist predict depression
would present in a 37-year-old Chinese immigrant?

A. 
Concern with mood symptoms
B. 
Somatic complaints
C. 
Hysteria
D. 
Self-mutilation
E. 
Paranoia

140. 
Which one of the following neurotransmitters is most involved in the effects of methylenedioxymethamphetamine
(ecstasy)?

A. 
Serotonin
B. 
NE
C. 
GABA
D. 
Glycine
E. 
Acetylcholine

141. The growth of child guidance clinics in the United States in the early 1900s led to:

A. 
The development of the first medications for ADHD
B. 
The development of sewage systems in major U.S. cities
C. 
The development of child psychiatry as a profession
D. 
Freud’s three essays on the theories of sexuality
E. 
The Ryan White Care Act

142. The most common neurologic manifestation of neurosarcoidosis is:

A. 
Cranial neuropathy
B. 
Cauda equina syndrome
C. 
Peripheral neuropathy
D. 
Meningoencephalitis
E. 
Uveitis

143. Trazodone is most often associated with which one of the following side effects?

A. 
Hematuria
B. 
Delayed orgasm
C. 
Retrograde ejaculation
D. 
Priapism
E. 
Hypospadias

144. Which one of the following answer choices is true regarding aggression?

A. 
High levels of cerebrospinal fluid (CSF) serotonin are associated with increased aggression
B. 
Serotonin is unrelated to aggression
C. 
Low levels of CSF serotonin are associated with increased aggression
D. 
Low levels of CSF serotonin are associated with decreased aggression
E. 
Low levels of dopamine are associated with increased aggression

216

Psychiatry Test Preparation and Review


Manual E-Book
Test Number Four

145. How does mirtazapine work?

A. 
Serotonin reuptake inhibition
B. 
NE reuptake inhibition
α2 Adrenergic receptor antagonism
C. 
D. 
Partial dopamine antagonism
E. 
Decreasing breakdown of serotonin in the synaptic cleft

146. Which one of the following answer choices is most consistent with sleep changes in the elderly?

A. 
Increased rapid eye movement (REM) sleep only
B. 
Increased slow-wave sleep only
C. 
Increased REM and slow-wave sleep
D. 
Decreased REM and slow-wave sleep
E. 
Decreased slow-wave sleep only

147. Which one of the following is most consistent with delirium?

A. 
Slow onset
B. 
Impaired remote memory
C. 
Fragmented sleep
D. 
Impaired immediate memory
E. 
Normal alertness

148. Which one of the following is not an aspect of Kleine–Levin syndrome?

A. 
Irritability
B. 
Voracious eating
C. 
Loss of libido
D. 
Incoherent speech
E. 
Hypersomnia

149. David works 17 hours per day. He does not have many friends because he feels that they interfere with his work sched-
ule. He believes that he is a moral person and harshly criticizes those whom he finds to be unethical. He often starts
projects but fails to complete them because he cannot do them perfectly. His family describes him as stubborn and
cheap, because he will never throw anything out. David’s most likely diagnosis is:

A. 
Generalized anxiety disorder
B. 
Obsessive–compulsive disorder
C. 
Obsessive–compulsive personality disorder
D. 
Schizoid personality disorder
E. 
Avoidant personality disorder

150. Which one of the following is an important technique of cognitive behavioral therapy?

A. 
Maintaining therapeutic neutrality
B. 
Offering interpretations of patients’ unconscious wishes
C. 
Abreaction
D. 
Working through unresolved conflict
E. 
Finding and testing automatic thoughts

217

Psychiatry Test Preparation and Review


Manual E-Book
Psychiatry Test Preparation and Review Manual

Four
Answer Key – Test Number Four
1. C 26. E 51. D 76. A 101. B 126. C
2. A 27. E 52. B 77. C 102. E 127. D
3. A 28. A 53. A 78. D 103. E 128. D
4. D 29. D 54. D 79. A 104. D 129. D
5. B 30. C 55. E 80. B 105. B 130. D
6. C 31. E 56. A 81. E 106. C 131. C
7. A 32. E 57. D 82. C 107. C 132. D
8. D 33. D 58. E 83. C 108. B 133. E
9. E 34. D 59. D 84. A 109. D 134. E
10. B 35. E 60. D 85. E 110. C 135. E
11. A 36. A 61. D 86. C 111. C 136. C
12. A 37. C 62. B 87. A 112. D 137. C
13. E 38. B 63. D 88. A 113. D 138. C
14. D 39. E 64. C 89. D 114. B 139. B
15. D 40. D 65. E 90. E 115. B 140. A
16. E 41. D 66. D 91. D 116. E 141. C
17. D 42. B 67. D 92. B 117. C 142. A
18. B 43. A 68. A 93. C 118. B 143. D
19. D 44. A 69. B 94. D 119. C 144. C
20. D 45. D 70. D 95. C 120. D 145. C
21. D 46. B 71. B 96. A 121. E 146. D
22. A 47. C 72. C 97. C 122. C 147. D
23. B 48. C 73. E 98. A 123. B 148. C
24. B 49. D 74. B 99. A 124. A 149. C
25. E 50. A 75. D 100. B 125. B 150. E

218

Psychiatry Test Preparation and Review


Manual E-Book
Test Number Four

Four
Explanations – Test Number Four
Question 1. C. Freud described a series of stages through which children pass as a part of normal development. These
stages correspond to shifts of sexual energy from one erotic body part to the next.

The first phase is the oral phase from birth to 1 year. In the oral phase the infant’s needs and expression
are centered in the mouth, lips, and tongue. Tension is relieved through oral gratification. Those who do
not complete the oral phase successfully can be very dependent in adulthood. Successful resolution of the
oral stage allows the adult to both give and receive without excessive dependency or envy.

The next is the anal stage from ages 1 to 3 years. In the anal stage the child develops control of its anal
sphincter. This phase is marked by an increase in aggressive and libidinal drives. Control of feces gives
the child independence, and the child struggles with the parent over separation. Successful completion of
this stage leads to a sense of independence from the parent. Failure to complete this stage leads to obses-
sive–compulsive neuroses.

The phallic stage is from ages 3 to 5 years. In the phallic stage there is a focus on sexual interests and
excitement in the genital area. The goal of this stage is to focus erotic interests in the genital area and lay
the groundwork for gender identity. Poor resolution of this stage leads to the neuroses often associated
with poor resolution of the Oedipal complex. Successful resolution leads to a clear sense of sexual iden-
tity, curiosity without embarrassment, initiative without guilt, and mastery over things both internal and
external.

The phallic stage is followed by latency, during which there is a decrease in sexual interest and energy.
Latency lasts from age 5 years until puberty. It is a period of consolidating and integrating previous
development in psychosexual functioning and developing adaptive patterns of functioning.

At puberty there is an increase in sexual energy. This time is described as the genital stage. It lasts from
about ages 11 to 13 years until adulthood. In this stage, libidinal drives are intensified. There is a regres-
sion in personality organization, allowing for resolution of prior conflicts and the solidification of the
adult personality. The goal is the ultimate separation from the parents and the development of noninces-
tuous object relations. Failure to complete this stage can lead to multiple complex outcomes. Freud felt
that one must pass successfully through all of these stages to develop normal functioning as an adult.
Human Development
K&S Chapter 4

Question 2. A. Amaurosis fugax is a symptom of carotid artery territory ischemia. It presents as a sudden onset of tran-
sient loss of vision that manifests as a curtain, shade, or veil, usually over the central visual field. The
duration of visual loss is generally brief, lasting about 1 to 5 minutes, and only infrequently exceeds 30
minutes in duration. When the episode concludes, vision is generally returned to normal. In some cases,

219

Psychiatry Test Preparation and Review


Manual E-Book
Psychiatry Test Preparation & Review Manual

there is permanent visual loss due to retinal infarction. Amaurosis fugax is the only feature that can dis-
tinguish a middle cerebral artery syndrome from a carotid artery syndrome.

Vertebrobasilar territory ischemia would be expected to affect the cerebellum and/or brain stem. Classic
posterior circulation ischemic symptoms include ataxia, nystagmus, vertigo, dysarthria, and dysphagia.
An ipsilateral Horner’s syndrome can occur if the descending oculosympathetic fibers are disrupted.
Crossed weakness (ipsilateral facial paresis, contralateral limb paresis) is indicative of brain-stem
involvement above the level of the area of decussation of the pyramids in the medulla.

Lenticulostriate territory ischemia affects the small penetrating branching arteries off the middle cerebral
artery that feed the striatum. Ischemia to this territory can produce lacunar infarcts of the internal cap-
sule that result in a pure contralateral motor hemiparesis.

Anterior cerebral artery ischemia would be expected to produce a contralateral hemiparesis of the leg
preferentially, because the cortical homuncular representation of the leg is situated parasagittally in the
postcentral motor cortex.

Middle cerebral artery territory ischemia can take many different forms. If the lesion is in the dominant
hemisphere, aphasia may result. Nondominant ischemia can result in hemineglect, anosognosia, visual
and tactile extinction, aprosody of speech, and contralateral limb apraxia.
Neurology
B&D Chapter 65

Question 3. A. NE is made in the locus ceruleus. Serotonin is made in the dorsal raphe nuclei. Dopamine is made in the
substantia nigra. Acetylcholine is made in the nucleus basalis of Meynert.
Basic Neuroscience
K&S Chapter 1

Question 4. D. The onset of back pain, followed by leg weakness and urinary incontinence, is a classic manifestation
of bilateral spinal cord pathology. In this case, the correct syndrome is a transverse myelitis, because the
deficits are closely preceded by an influenza vaccination. Transverse myelitis is a segmental inflamma-
tory syndrome of the bilateral spinal cord. It is believed to be immunologic in origin and often follows
an infection or vaccination, or is the direct result of demyelination due to multiple sclerosis (MS). Up to
40% of cases have no identifiable origin. The classic presentation is the rapid onset of bilateral leg weak-
ness that presents with a clear-cut sensory level below the level of the lesion. Pain and temperature sen-
sation are usually affected, but often proprioception and vibration sensation are spared. Urinary and/or
bowel incontinence are common findings. MRI of the spine is the imaging modality of choice. If both the
spine and the brain show demyelination on MRI, the likelihood of the myelitis being the first manifesta-
tion of MS is greater than 50%. If the condition is proven to be a result of inflammation or demyelin-
ation, then high-dose intravenous steroids are the initial acute treatment of choice.

Spinal cord metastases causing an acute cord compression would be expected to cause a presentation
similar to that of myelitis, as noted previously, but one would expect the history to reveal some sort
of primary cancer, such as that of the lung or prostate, that would precede the onset of spinal cord
symptoms. Only about 15% of primary CNS tumors originate in the spinal cord. Emergent MRI must
be performed to characterize the lesion. Treatment of acute cord compression due to metastatic lesions
involves intravenous steroids, radiation therapy, and possible surgical decompression.

Vacuolar myelopathy is found almost uniquely in AIDS patients and is similar to the condition noted
in vitamin B12 deficiency. The clinical picture is similar to that of myelitis or cord compression, but the
timeline is usually much slower and more progressive, evolving over many months. The pathophysiol-
ogy of AIDS-related vacuolar myelopathy may be due to viral release of cytokines that are neurotoxic
in nature or abnormalities in vitamin B12 utilization. Vitamin B12 levels are often normal in these cases.
Antiretroviral therapy may not reverse the symptoms.

220

Psychiatry Test Preparation and Review


Manual E-Book
Test Number Four

Acute disseminated encephalomyelitis (ADEM) is a demyelinating syndrome that follows a systemic


infection or vaccination. It differs from transverse myelitis, because ADEM involves the whole CNS and
is not simply localized to a segment of the spinal cord. ADEM usually results in multifocal signs and
symptoms that include an encephalopathy. Brain stem and cerebral involvement are noted, as are symp-
toms localizing to the spinal cord.
Neurology
B&D Chapters 55 and 56

Question 5. B. If different doctors can look at the same case and make the same diagnosis, the diagnosis is said to be
reliable. In other words, it is consistent. Accuracy of the diagnosis is called validity. Descriptive validity
means that the disorder has features that are characteristic enough to separate it from other disorders.
Predictive validity means that the diagnosis allows the doctor to predict clinical course and treatment
response. Construct validity means the diagnosis is based on underlying pathophysiology and the use of
biologic markers to confirm the disease.
Statistics
K&S Chapter 5

Question 6. C. Horner’s syndrome results from an interruption to the sympathetic fibers supplying the pupil, upper
eyelid, facial sweat glands, and facial blood vessels. The classic symptom triad is that of ptosis, mio-
sis, and anhidrosis. Horner’s syndrome can be seen as part of the lateral medullary stroke syndrome
(Wallenberg’s syndrome), carotid occlusion or dissection, high spinal cord lesions, neoplasms like
Pancoast’s tumor that affect the cervical ganglia, or intracranial hemorrhage. If the condition affects
only the eye and not the sweat glands, the lesion usually localizes to the territory of the internal carotid
artery. The other answers to this question need no explanation, because they are simply nonsense
distractors.
Neurology
B&D Chapter 77

Question 7. A. This question lists side effects that may be found with amphetamines. All of the answer choices except
choice A are CNS depressants and would be predicted to have effects opposite to many of those listed
in the question, although there may be some similarities with certain symptoms. Amphetamine is a
sympathomimetic.
Substance Abuse and Addictive Disorders
K&S Chapter 20

Question 8. D. Prosopagnosia is the inability to recognize familiar faces. It is associated classically with bilateral occipi-
tal–temporal lesions. It is often associated with agraphia and achromatopsia (inability to recognize col-
ors and hues). It is almost always associated with visual field deficits. Anosognosia, the denial of disease
or hemiparesis, is seen with lesions of the nondominant hemisphere. Simultanagnosia is the inability to
perceive a scene with multiple parts to it. The patient is only able to see and recognize individual ele-
ments of a multipart scene and cannot interpret the overall picture. Astereognosis refers to the inability
to recognize and identify items by weight, texture, and form alone when the items are held in the hand.
It is a form of tactile agnosia. Aprosodia is a deficit in the emotional aspect of expressive or receptive
speech.
Neurology
B&D Chapters 11, 12, and 51

Question 9. E. With PCP use, hypertension is often seen, not hypotension. Other symptoms include paranoia, nystag-
mus, catatonia, convulsions, hallucinations, mood lability, loosening of associations, violence, mydriasis,
ataxia, and tachycardia. Acute management consists of haloperidol every 2 to 4 hours until the patient is
calm. PCP can be detected in the urine up to 8 days after ingestion.
Substance Abuse and Addictive Disorders
K&S Chapter 20

221

Psychiatry Test Preparation and Review


Manual E-Book
Psychiatry Test Preparation & Review Manual

Question 10. B. Bell’s palsy (VII nerve palsy) is a severe, acute, unilateral, complete facial paresis that evolves over 24 to
48 hours. The condition is often accompanied by pain behind the ear. Taste impairment and hyperacusis
are frequent associated symptoms. The incidence is about 20 in 100,000. The syndrome is believed to be
of viral etiology, and herpes simplex virus is thought to be the most frequent viral pathogen responsible
for the condition. About 80% of patients improve completely within 3 months. Incomplete paralysis at
onset is a better prognostic sign than complete paralysis. Treatment with acyclovir and prednisone is
often administered but remains controversial because of the etiological uncertainty of the disorder. The
paralysis of Bell’s palsy involves the entire face, whereas the facial paresis of a hemispheric stroke spares
the upper third of the face (the brow and upper eyelid).
Neurology
B&D Chapter 70

Question 11. A. This question focuses on Piaget’s stages of cognitive development. Object permanence develops during
the sensorimotor stage. There are several other questions on Piaget in this book. If you did not get this
question correct, go back and review those questions and answer explanations. Know Piaget well!
Human Development
K&S Chapter 2

Question 12. A. Syringomyelia refers to the constellation of signs and symptoms produced by a syrinx, a cavitation of the
central spinal cord. The cavitation may be contiguous with a dilated central spinal canal, or it may be
separate from a central canal. Most syringes occur in the cervical spinal cord. Most of those that develop
from a central spinal canal are associated with an Arnold–Chiari type I or type II malformation. The
classic presentation is that of a dissociated suspended sensory deficit usually in a cape or shawl pattern
over the arms and upper trunk. There is impairment of pain and temperature sensation and preserved
light touch, vibration, and proprioception. These sensory deficits are combined with lower motor neuron
signs (flaccidity, muscular atrophy, fasciculations) at or about the level of the lesion, as well as spinal
long tract signs below the level of the lesion. Pain often accompanies a syrinx and can manifest as head-
ache, neck pain, radicular pain, and segmental dysesthesia. MRI of the spine is the imaging modality of
choice to best evaluate a syrinx.
Neurology
B&D Chapter 73

Question 13. E. Urinary retention is a side effect of the belladonna alkaloids, but not of glue and benzene products. All
of the other answer choices are possibilities for a patient sniffing glue daily for more than 6 months. In
addition to those contained in the question, side effects include slurred speech, ataxia, hallucinations,
and tachycardia with ventricular fibrillation. This grouping of substances includes glue, benzene, gaso-
line, paint thinner, lighter fluid, and aerosols.
Substance Abuse and Addictive Disorders
K&S Chapter 20

Question 14. D. Opsoclonus–myoclonus is a paraneoplastic movement disorder that is also called “dancing eyes–dancing
feet” syndrome. It is seen most often in infants ages 6 to 18 months and in 50% or more of cases is asso-
ciated with an infantile neuroblastoma. In adults, lung cancer is a common cause of this disorder. It can
also be noted as a result of postviral encephalitis, MS, thalamic hemorrhage, and hyperosmolar coma.
The condition presents as multifocal myoclonus and rapid dancing movements of the eyes. It is caused
by lesions to the pause cells in the pons. Steroids or adrenocorticotropic hormone is an effective treat-
ment for this particular type of myoclonus. The other conditions noted in this question are distractors
that are not related to opsoclonus–myoclonus.
Neurology
B&D Chapter 52

Question 15. D. Social phobia is characterized by a fear of one or more social or performance situations in which the
person is exposed to unfamiliar people or possible scrutiny by others. The individual is afraid of acting
in a way that would be embarrassing. Exposure to the situation almost always causes anxiety, and the

222

Psychiatry Test Preparation and Review


Manual E-Book
Test Number Four

person is aware that the fear is excessive. In this question, fear of scrutiny is the most definitive symptom
of social phobia. The other choices could provoke anxiety in someone with some type of anxiety disor-
der but are neither necessarily limited to, nor considered major diagnostic criteria for social phobia.
Anxiety Disorders
K&S Chapter 9

Question 16. E. The main role of the psychiatrist in the collaborative care model is to indirectly consult on patients. The
psychiatrist usually does not meet the patient directly but rather provides caseload consultation to the
primary care physician, case manager, and other mental health professionals. These other professionals
provide the hands-on care. The primary care doctor is the head of the team. It is not the psychiatrist’s
job to do therapy supervision, administration, or provide therapy directly. Some patients may be too ill
to be managed in this model and may need referral for a psychiatrist to directly treat them. However, the
goal of the collaborative care model is for a much larger population of patients to be managed, with the
psychiatrist working with and through other team members rather than seeing the patients directly.
Management in Psychiatry
MGH Comprehensive Clinical Psychiatry, Chapter 59

Question 17. D. All of the answer choices given are true except for D. In cases of body dysmorphic disorder, medical,
dental, or surgical treatment fails to solve the patient’s preoccupation. The disorder consists of a preoc-
cupation with an imagined defect in appearance. If a slight physical anomaly is present the concern is
markedly excessive. The cause of this disorder is unknown, onset is usually slow, and marked impair-
ment in functioning occurs. There is an association with mood disorders, and in some studies as many as
50% of patients benefited from treatment with a serotonin-selective reuptake inhibitor. The body part of
concern may change over the course of the disorder. Differential diagnosis should include anorexia, gen-
der identity disorder, and conditions that can result in brain damage, such as neglect syndromes.
Somatic Symptom Disorders
K&S Chapter 10

Question 18. B. Glioblastoma multiforme is the most common primary brain tumor in about 50% or more of patients
over 60 years of age. Average survival is about 1 year after diagnosis, with radiation therapy. Anaplastic
astrocytoma has a bimodal peak in incidence in the first and third decades. Ependymomas make up
about 5% of all brain tumors. They are the third most common CNS tumor in children. Meningiomas
make up about 20% to 25% of all brain tumors. They are more likely to occur after 50 years of age
and occur about twice as frequently in females as in males. About 80% of meningiomas turn out to be
benign. Acoustic neuromas are considered to be schwannomas (nerve sheath tumors). The peak inci-
dence of acoustic neuromas is in the fourth and fifth decades.
Neurology
B&D Chapter 52

Question 19. D. Neuroimaging studies in obsessive–compulsive disorder (OCD) patients have demonstrated abnormali-
ties in the caudate, thalamus, and orbitofrontal cortex. Functional neuroimaging by positron-emission
tomography scanning has demonstrated increased metabolism in the basal ganglia (predominantly in
the caudate), the frontal lobes, and the cingulate gyrus. CT and MRI of the brains of patients with OCD
have demonstrated bilaterally smaller caudate nuclei than those seen in normal controls.
Laboratory Tests in Psychiatry
K&S Chapter 10

Question 20. D. Anton’s syndrome is considered to be an agnosia characterized by cortical blindness. The lesion localizes
to the bilateral occipital lobes, usually due to strokes, particularly in the calcarine cortex (Brodmann’s
area 17) and visual association cortex. The hallmark of the syndrome is that patients deny that they are
blind and they confabulate. Patients may also suffer from visual hallucinations.
Neurology
B&D Chapter 14

223

Psychiatry Test Preparation and Review


Manual E-Book
Psychiatry Test Preparation & Review Manual

Question 21. D. Signs of impending violence, or predictive factors for violence, include alcohol and drug intoxication,
recent acts of violence, command auditory hallucinations, paranoia, menacing behavior, psychomotor
agitation, carrying weapons, frontal lobe disease, catatonic excitement, certain manic or agitated depres-
sive episodes, violent ideation, male gender, ages 15 to 24 years, low socioeconomic status, and few
social supports.
Management in Psychiatry
K&S Chapter 26

Question 22. A. Riluzole (Rilutek) is a glutamate antagonist and is FDA-approved in the treatment of amyotrophic lat-
eral sclerosis (ALS). It has demonstrated mild-to-moderate improvement in the survival rate of ALS. It
may also help patients remain in a milder disease state for a longer period of time. Side effects include
gastrointestinal upset, dizziness, fatigue, and liver enzyme elevation. The other answer choices are
distractors.
Neurology
B&D Chapter 74

Question 23. B. Conversion disorder (functional neurological symptom disorder) consists of one or more neurologic
symptoms that cannot be explained by a medical condition. The symptoms are thought to be uncon-
sciously produced in response to psychological conflict. Pseudoseizures can be common symptoms of
conversion disorder. Conversion disorder can be associated with antisocial, histrionic, and dependent
personality disorders. In OCD a patient has intrusive unwanted thoughts that cause him or her to repeat
a ritual or action to remove the anxiety associated with that thought. Nothing like that is described in
this question. Somatic symptom disorder presents as a patient who has a number of somatic complaints
that cannot be otherwise explained by a known medical condition. These somatic symptoms disrupt
daily life, and the patient has excessive thoughts, feelings, or behaviors related to them that are mani-
fested by high levels of anxiety or excessive time and energy devoted to the symptoms. That is not the
case in this question. Social phobia presents as a patient who has a fear of being in social situations or
meeting new people. That has nothing to do with this question. Panic disorder presents as a patient who
has recurrent and unexpected panic attacks. This patient is having pseudoseizures, not panic attacks.
Somatic Symptom Disorders
K&S Chapters 9 and 13

Question 24. B. Primary CNS lymphoma occurs in AIDS patients in about 5% of cases. These lymphomas are most
often of B-cell origin. Patients can present with a gradual onset of any of the following symptoms: head-
ache, aphasia, hemiparesis, altered mental status, behavioral changes, and ataxia. Constitutional symp-
toms like fever and weight loss are generally absent. Diagnosis is established by brain MRI. Polymerase
chain reaction (PCR) testing on CSF revealing Epstein–Barr virus DNA helps corroborate the diagnosis.
Treatment with antiretroviral polypharmacy can help slow progression. None of the other tumor types
mentioned in the answer choices are particularly associated with AIDS.
Neurology
B&D Chapter 53

Question 25. E. The development of the CNS is a very important area of study, because abnormal development is impli-
cated in several clinical conditions, including schizophrenia. The central and peripheral nervous systems
arise from the neural tube. The neural tube gives rise to the ectoderm, which becomes the peripheral
nervous system, whereas the neural tube itself becomes the CNS. The second trimester of gestation is the
peak of neuronal proliferation, with 250,000 neurons born each minute. Migration of neurons, guided
by glial cells, peaks during the first 6 months of gestation. Synapse formation occurs at a high rate from
the second trimester through age 10 but peaks around 2 years (toddler period), with as many as 30 mil-
lion synapses forming per second. The nervous system also actively myelinates its axons starting prena-
tally, continuing through childhood, and finishing in the third decade of life.
Basic Neuroscience
K&S Chapter 13

224

Psychiatry Test Preparation and Review


Manual E-Book
Test Number Four

Question 26. E. AIDS dementia complex is a late complication of AIDS, particularly when the human immunodeficiency
virus (HIV) infection is untreated and CD4 cell counts are low. Symptoms include poor attention and
concentration, bradyphrenia (slowness of thought), forgetfulness, poor balance, uncoordination, person-
ality changes, apathy, and depression. Focal neurologic deficits such as hemiparesis or aphasia are usu-
ally absent. Treatment involves administration of antiretroviral therapy.
Neurology
B&D Chapter 53

Question 27. E. All of the answer choices given are correct, with the exception of increased bone density. Anorexic
patients tend to suffer from osteoporosis. They also have cachexia, loss of muscle mass, reduced thyroid
metabolism, loss of cardiac muscle, arrhythmias, delayed gastric emptying, bloating, abdominal pain,
amenorrhea, lanugo (fine, baby-like hair), and abnormal taste sensation.
Feeding and Eating Disorders
K&S Chapter 15

Question 28. A. CNS toxoplasmosis is the most frequent CNS opportunistic infection in the AIDS patient. It is noted
in 10% or more of AIDS patients. The infection is caused by the parasite Toxoplasma gondii. Cerebral
toxoplasmosis in the AIDS patient results most often from a resurgence of a previously acquired infec-
tion. The infection usually occurs in late-stage AIDS, when CD4 cell counts are less than 200/μL. CNS
toxoplasmosis presents clinically with headache and focal neurologic deficits with or without fever. Other
possible manifestations include aphasia, seizures, and hemiparesis. In cases with significant progression,
patients can develop confusion and lethargy that lead to coma. The diagnosis is made on the basis of CT
or MRI of the brain revealing a single lesion or multiple ring-enhancing lesions. Therapy is undertaken
with pyrimethamine and sulfadiazine, or with clindamycin in sulfa-allergic patients. The other answer
choices occur less frequently in AIDS patients. Cryptococcal meningitis, another important opportunistic
infection in AIDS, and the other three answer choices are explained elsewhere in this volume.
Neurology
B&D Chapter 53

Question 29. D. All choices given are correct, except for hyperkalemia. In purging, it is common to see a hypokalemic,
hypochloremic alkalosis. In addition, one may also see hypomagnesemia, pancreatic inflammation,
increased serum amylase, esophageal erosion, and bowel dysfunction.
Feeding and Eating Disorders
K&S Chapter 15

Question 30. C. HIV-associated vacuolar myelopathy is the most common cause of spinal cord pathology in AIDS
patients and is seen in one-quarter to one-half of patients on autopsy. The disorder usually occurs in
late-stage AIDS. The clinical picture is that of spasticity, gait instability, lower extremity weakness, loss
of proprioception and vibration sensation, and sphincter dysfunction. Neurologic examination reveals a
spastic paraparesis, hyperreflexia, and Babinski’s signs. It is unusual to note a clear-cut sensory level on
the trunk. Vitamin B12 levels are usually normal. Viral neurotoxic cytokines may contribute to the patho-
physiology of the disease.

Distal sensory polyneuropathy is the most common peripheral nerve syndrome that complicates AIDS.
It is seen in about one-third of AIDS patients. The clinical presentation is that of diminished ankle jerk
reflexes; decreased pain, temperature, and vibration sensation; and possible paresthesia or numbness of
the feet. The disorder is usually symmetrical. The condition frequently presents with lower extremity
neuropathic pain. Treatment involves antiretroviral therapy and management of the pain with tricyclic
antidepressants, anticonvulsants, or even narcotic analgesics if needed.

Neurosyphilis, progressive multifocal leukoencephalopathy, and HTLV-1 myelopathy are all explained
elsewhere in this volume.
Neurology
B&D Chapter 53

225

Psychiatry Test Preparation and Review


Manual E-Book
Psychiatry Test Preparation & Review Manual

Question 31. E. Central serous chorioretinopathy is a disease leading to detachment of the retina and has nothing to
do with anxiety. Carcinoid syndrome can mimic anxiety disorders and is accompanied by hypertension
and elevated urinary 5-hydroxyindoleacetic acid (5-HIAA). Hyperthyroidism presents with anxiety in
the context of elevated T3 and T4 and exophthalmos. Hypoglycemia presents with anxiety and fasting
blood sugar under 50 mg/dL. Signs and symptoms of diabetes may also be present with hypoglycemia
(polyuria, polydipsia, and polyphagia). Hyperventilation syndrome presents with a history of rapid deep
respirations, circumoral pallor, and anxiety. It responds well to breathing into a paper bag.
Anxiety Disorders
K&S Chapter 9

Question 32. E. Progressive multifocal leukoencephalopathy (PML) is a demyelinating disorder affecting AIDS patients with
low CD4 counts. It results from an opportunistic infection by
the JC virus, a form of human papilloma virus. It occurs in
about 5% of patients with AIDS. Demyelination occurs pref-
erentially in the subcortical white matter of the parietal and/
or occipital lobes. Clinical presentation can involve hemipa-
resis, aphasia, sensory deficits, ataxia, and visual field deficits.
Mental status may deteriorate progressively over time. MRI
often reveals multiple or coalesced nonenhancing white matter
lesions in the parietal or occipital lobes. CSF assay for the JC
virus DNA by PCR can help confirm the diagnosis. The other
answer choices in this question are distractors and are explained
elsewhere in this volume.
Neurology
B&D Chapter 53

Question 33. D. The distinction between social phobia and avoidant person-
ality disorder can sometimes be confused. In a situation in
which the patient is afraid of almost all social situations, then
avoidant personality disorder should be considered. Social phobia presents with a fear of one or more
social situations. Avoidant personality disorder is defined by a pervasive pattern of social inhibition, feel-
ings of inadequacy, and hypersensitivity to negative evaluation. This can be shown by avoiding activities
involving interpersonal contact, unwillingness to get involved with people unless certain of being liked,
showing fear in intimate relationships for fear of ridicule, preoccupation with criticism or rejection in
social situations, viewing self as socially inept, and being inhibited in new social situations or afraid to take
risks for fear of embarrassment.
Personality Disorders
K&S Chapter 22

Question 34. D. Prion diseases are also known as transmissible spongiform encephalopathies. Prions are proteinaceous
and infectious particles that are unlike bacteria or viruses. Prions are unique in that they can be passed
on through heredity, acquired by infection, or acquired spontaneously. Kuru is the prion disease endemic
to the cannibalistic Fore people. The clinical presentation is that of progressive cerebellar ataxia. The
progressive disease course generally leads to death in about 12 months from initial onset.

Gerstmann–Sträussler–Scheinker syndrome is an inherited form of prion disease. Symptoms begin to


manifest in about the third or fourth decade. The disease is the slowest of the spongiform encephalopa-
thies and can progress over several years. The clinical presentation depends on which mutation in the
PRNP gene is acquired by the patient. The syndrome varies in nature and presents with any combina-
tion of ataxia, Parkinson’s plus-like symptoms, or progressive dementia.

Fatal familial insomnia occurs between ages 35 and 60 years. It presents with progressive insomnia and
sympathetic autonomic hyperactivity, such as hyperhidrosis, tachycardia, hyperthermia, or hypertension.
Mild cognitive impairment is usually noted. Other symptoms can include ataxia, tremor, myoclonus,
confusion, or hallucinations.

226

Psychiatry Test Preparation and Review


Manual E-Book
Test Number Four

Creutzfeldt–Jakob disease can be sporadic, iatrogenic, or familial. CSF evaluation of 14-3-3 and Tau
proteins is the diagnostic test of choice to confirm the disease. The classic sporadic disease usually affects
those between 50 and 75 years of age. The downward clinical course is rapid and progresses toward
death over 6 months to 2 years. The disease has three stages. Stage 1 presents with neuropsychiatric
symptoms that can include fatigue, sleep disturbance, memory and concentration deficits, and per-
sonality changes. Stage 2 presents with generalized cognitive deficits, as well as significant psychiatric
symptoms, such as psychosis and hallucinosis. The third and final stage, which precedes death, is charac-
terized by profound and severe dementia, with myoclonus and choreoathetosis. Treatment of the spongi-
form encephalopathies is palliative and symptomatic. There is no known cure for these diseases.

Devic’s disease, also known as neuromyelitis optica, is believed to be a variant of MS. The presentation
is that of cervical myelopathy and bilateral optic neuropathy. In certain patients, the two conditions
occur simultaneously, but in others, they occur at separate intervals, pointing more directly to a diagno-
sis of MS. Devic’s disease is a disease of demyelination and has nothing to do with prions.
Neurology
B&D Chapters 53 and 54

Question 35. E. Bipolar II disorder is characterized by at least one major depressive episode and at least one hypomanic
episode during the patient’s lifetime. There are no full manic episodes in bipolar II disorder. If criteria for a
manic episode are met, then the correct diagnosis is bipolar I disorder. Psychotic features can be found in
bipolar I disorder during mania or depression but in bipolar II will occur only associated with depression,
as full mania is not present in bipolar II. Lifetime prevalence of bipolar II disorder is 0.3% to 4.8%.
Bipolar Disorders
K&S Chapter 8

Question 36. A. Carotid artery occlusion presents with the triad of headache, ipsilateral Horner’s syndrome, and contralat-
eral hemiparesis. A resultant cerebral infarct can increase intracranial pressure, thereby causing headache.
The Horner’s syndrome itself can result in headache because of interruption of ascending sympathetic tracts
from the superior cervical ganglion to the intracranial vessels and dura, which are pain-sensitive structures.

Pontine hemorrhage is a neurologic emergency, because it can lead to rapid death if the bleed is large.
This hemorrhage type accounts for about 5% of all intracranial hemorrhages. Large bleeds in the basal
tegmentum of the pons result in a clinical presentation characterized by quadriplegia, coma, decerebrate
posturing, respiratory rhythm anomalies, pinpoint reactive pupils, hyperthermia, horizontal ophthalmo-
plegia, and ocular bobbing. The likely etiology of these bleeds is the rupture of small tegmental pontine
penetrating arteries that originate from the trunk of the basilar artery.

Cerebral aneurysmal rupture results in a sudden explosive headache (“the worst headache of my life”),
obtundation, nausea, vomiting, meningeal signs, neck pain, and photophobia. Aneurysms are discussed
in further detail elsewhere in this volume, as are giant cell arteritis and Wallenberg’s syndrome.
Neurology
B&D Chapters 51 and 69

Question 37. C. Analysis of variance is a set of statistical procedures that compares two groups and determines if the dif-
ferences are due to experimental influence or chance. A control group is a group that does not receive
treatment and is a standard of comparison. Regression analysis is a method of using data to predict
the value of one variable in relation to another. The null hypothesis is the assumption that there are
no differences between two samples of a population. When the null hypothesis is rejected, differences
between the groups are not attributable to chance alone. The correlation coefficient is a measurement of
the direction and strength of the relationship between two variables.
Statistics
K&S Chapter 5

Question 38. B. The condition described in this question is of course that of a Bell’s palsy (VII nerve palsy). Bell’s palsy
is characterized by a sudden, severe, unilateral facial paralysis involving both the upper and the lower
parts of the face. In serious cases, the brow droops, with widening of the palpebral fissure, and the eyelid

227

Psychiatry Test Preparation and Review


Manual E-Book
Psychiatry Test Preparation & Review Manual

cannot close completely. When the patient tries to close the affected eye with effort, the lid remains partially
opened, and the globe turns up and out. This sign is known as Bell’s phenomenon. The incidence is about
20 per 100,000, with a peak in the third decade. The disorder is believed to be viral in origin and is thought
to be most often due to occult herpes simplex infection. The facial paresis is often preceded by pain behind
the ipsilateral ear, which may resolve. The facial paresis results soon thereafter and may evolve over 1 to 2
days, with maximal deficit within the first 72 hours. About 85% of patients recover fully within 3 months.

Borrelia burgdorferi, the spirochete responsible for Lyme disease, can result in a unilateral or even bilat-
eral Bell’s palsy. This is often accompanied by an aseptic meningitis, which when combined with facial
diplegia is termed Bannwarth’s syndrome. Facial diplegia from Lyme disease is much less common than
idiopathic Bell’s palsy, and the history should point to deer tick exposure in the outdoor setting at some
time directly preceding the onset of symptoms.

Epstein–Barr virus, West Nile virus, and varicella zoster virus infections do not usually result in a Bell’s
palsy. Other conditions that can cause a VIIth nerve palsy include Guillain–Barré syndrome, sarcoidosis,
and facial nerve tumors or metastases.
Neurology
B&D Chapter 70

Question 39. E. This question gives a clear description of autism spectrum disorder. In autism spectrum disorder the child
displays a lack of ability to use and read nonverbal gestures, failure to develop appropriate peer relation-
ships, lack of sharing enjoyment with other people, and lack of emotional reciprocity. The child may also
show communication deficits, such as delays in language development, inability to sustain conversation,
repetitive or idiosyncratic use of language, and lack of make-believe play. He or she exhibits repetitive or
stereotyped patterns of behavior, such as preoccupation with areas of interest that are abnormal in either
intensity or focus, adherence to inflexible ineffective routines, repetitive motor mannerisms, or persistent
preoccupation with parts of objects. Some of the before-mentioned symptoms appear before 3 years of age.
Neurodevelopmental and Pervasive Developmental Disorders
K&S Chapter 31

Question 40. D. This question points to a diagnosis of rabies virus infection. Rabies virus is transmitted to humans most
often by wild animals, such as bats, foxes, and raccoons, or by nonimmunized dogs. The incubation
period is about 1 to 2 months. There is a prodromal period characterized by paresthesias, headache,
and fever, which can then progress to generalized neurologic compromise, coma, and death. During the
progressive phase of the illness, 80% or more of patients exhibit hydrophobia, which manifests as pha-
ryngeal and nuchal spasms that are triggered by swallowing, smells, tastes, or sounds. These spasms can
last up to several minutes in duration. The condition usually progresses to an encephalitis that is often
accompanied by high fever that can rise up to 107° F, as well as to autonomic hyperactivity, seizures,
agitation, and psychosis. Treatment begins with postexposure prophylaxis with rabies vaccine and anti-
rabies immunoglobulin in patients who have never received immunization.

West Nile virus infection is an arbovirus infection that is endemic to many parts of the world.
Mosquitoes are frequent vectors of transmission of the virus to humans. Most infections in humans are
asymptomatic. In about 20% of affected patients, the infection presents as a febrile condition after an
incubation period of a few days to up to 2 weeks. Only 1 in 150 patients go on to develop a meningitis
or encephalitis picture. The infection may progress to the development of a demyelinating or axonal
neuropathy. Diagnosis is made by detection of IgM antibodies in CSF or IgM and IgG antibodies in
serum. Treatment is generally supportive. Patients with serious symptoms may respond to intravenous
administration of anti-West Nile virus immunoglobulin.

Acute tetanus results from infection with the bacterium Clostridium tetani. This bacterium releases teta-
nospasmin, also known as tetanus toxin, which blocks GABA and glycine in the inhibitory interneurons
in the spinal cord and causes the characteristic muscle contractions that are seen in tetanus. The bacte-
rial spores can live for years in dust and soil, and when an open wound is inoculated with dirt that con-
tains the spores, they can release the toxin that causes the disease. The toxin is taken up into the anterior
horn cells by retrograde axonal transport. Trismus, or lockjaw, is a primary symptom of the disorder

228

Psychiatry Test Preparation and Review


Manual E-Book
Test Number Four

in over 75% of cases. Risus sardonicus, a sustained involuntary grimace resulting from uncontrollable
facial muscle spasm, is another characteristic of tetanus. Laryngospasm can lead to respiratory compro-
mise and death if the disorder is untreated. Cardiac arrest can also result from dysautonomia. Treatment
begins with airway protection and intubation if there is respiratory compromise. Tetanus immune globu-
lin neutralizes blood-borne toxin. Prevention of the disease by vaccination with anti-tetanus antibody
vaccine after contraction of a dirty open wound is the cornerstone of disease management.

Clostridium botulinum is a bacterium that secretes a potent neurotoxin that blocks acetylcholine release
at the neuromuscular junction and thus prevents neuromuscular transmission. The bacterium infects
humans via its presence in tainted food or via wound contamination from dirt or soil containing the
organism. Classic symptoms of botulism include dysphagia, dysarthria, ptosis, and diplopia. These symp-
toms rapidly progress to limb paralysis, and eventually to paralysis of respiratory muscles that can lead
to death if the condition is untreated. Gastrointestinal symptoms of nausea, vomiting, and diarrhea often
present with neurologic compromise after a 12- to 36-hour incubation period subsequent to ingestion
of the toxin. Infants who consume unpasteurized honey may ingest spores and can present with weak
cry, lethargy, floppiness, poor suck, and constipation. Diagnosis can be established by wound or stool
culture. Electromyography and nerve conduction studies can reveal characteristic anomalies compatible
with presynaptic neuromuscular blockade. Treatment is supportive, particularly with respect to airway
protection, and trivalent equine antitoxin administration can reverse the effects of circulating toxin.

Epstein–Barr virus (EBV) infection can be asymptomatic, or it can present as infectious mononucleosis,
with splenomegaly, pharyngitis, and cervical lymphadenopathy. Less than 1% of EBV infections present
with neurologic manifestations, which can include meningitis, transverse myelitis, sensory polyneuropa-
thy, or Guillain–Barré syndrome. EBV can be detected in the CSF of patients with AIDS-related primary
CNS lymphoma.
Neurology
B&D Chapter 53

Question 41. D. The three receptor types associated with glutamate are AMPA, kainate, and NMDA. Acetylcholine is
associated with the nicotinic and muscarinic receptors. NE is associated with the α1, α2, and β receptors.
Serotonin is associated with the various 5-HT receptors. GABA is associated with the GABA receptor.
Opioids are associated with the μ and δ opioid receptors. Dopamine is associated with the D1, D2, D3,
and D4 receptors.
Basic Neuroscience
K&S Chapter 1

Question 42. B. This vignette depicts a patient with classic manifestations of Pick’s disease. Pick’s disease is one of the
so-called frontotemporal dementias. The frontal and temporal lobes are preferentially affected, and path-
ological studies reveal localized “knife-edge” atrophy of these lobes together with ballooned cells and
intraneuronal inclusions, termed “Pick’s inclusion bodies.” These inclusions, plus the finding of swollen
neurons and gliosis, have come to be termed Pick-type histology.

The clinical presentation can manifest either with predominant behavioral disturbance or with progres-
sive language disturbance. In the first type, behavioral changes occur with poor judgment, inability to
reason or plan, and impulsivity. Patients can display hyperorality and hypersexuality (similar to that in
Klüver–Bucy syndrome), with both behavioral disinhibition and a lack of motivation.

Treatment is aimed at management of depression and behavioral problems. Antidepressant therapy with
a serotonin-selective reuptake inhibitor medication can be useful. Atypical antipsychotic agents may help
with impulsivity and behavioral anomalies. The condition is invariably irreversible.

Hirano bodies are eosinophilic neuronal inclusions that can be seen in certain neurodegenerative dis-
eases, such as Creutzfeldt–Jakob disease and Alzheimer’s dementia. Lewy bodies are cytoplasmic
inclusions found in the substantia nigra and cortex of many patients with idiopathic Parkinson’s dis-
ease and in the cerebral cortex of patients with Alzheimer’s dementia and diffuse Lewy body disease.
Neurofibrillary plaques and tangles are the neuropathologic hallmark of Alzheimer’s dementia. Severe

229

Psychiatry Test Preparation and Review


Manual E-Book
Psychiatry Test Preparation & Review Manual

white matter demyelination can occur in several distinct diseases, particularly in MS, Binswanger’s dis-
ease, ADEM, and PML.
Neurology/Neurocognitive Disorders
B&D Chapter 66

Question 43. A. It is thought that a child’s temperament at an age as young as 8 months is a function of its genes. At such a
young age, environmental factors play a small role in the child’s temperament. As the child gets older, envi-
ronmental factors will play a larger and larger role. Temperament has been examined by researchers and
divided into several components. These include activity level, rhythmicity (of hunger, feeding, elimination,
etc.), approach or withdrawal, adaptability, intensity of reaction, responsiveness, mood, distractibility, and
attention span. There is a genetic component to why these vary among individuals, as well as the influential
component of parents, consequences of the child’s behavior, and other environmental influences.
Psychological Theory and Psychometric Testing
K&S Chapter 2

Question 44. A. The neuropathologic hallmark of idiopathic Parkinson’s disease is the finding of characteristic Lewy
bodies in surviving neurons in the pars reticulata of the substantia nigra in the midbrain. The substantia
nigra itself is depigmented and pale and demonstrates gliosis and neuronal loss. Lewy bodies have a pale
halo and an eosinophilic center.

Hirano bodies are eosinophilic neuronal inclusions that can be seen in certain neurodegenerative dis-
eases, such as Creutzfeldt–Jakob disease and Alzheimer’s dementia.

Amyloid and neuritic plaques are noted in Alzheimer’s dementia. Amyloid may also be found in the
cerebrovascular wall. Mesial temporal sclerosis is the pathologic hallmark of brain tissue in those with
temporal lobe epilepsy. The entorhinal cortex and hippocampi are characterized by neuronal loss and
gliosis. Atrophy of the head of the caudate nucleus is found in Huntington’s disease, but it does not cor-
relate with the severity of the disease.
Neurology
B&D Chapters 66, 67, and 71

Question 45. D. Clomipramine has been shown to be particularly useful in treating OCD in comparison to the other tri-
cyclics. Its efficacy compared with the SSRIs for OCD is equal or better, depending on the study.
Obsessive Compulsive and Related Disorders
K&S Chapter 10

Question 46. B. Fibromyalgia does not present with psychosis. Fibromyalgia is a syndrome characterized by diffuse mus-
cular and soft-tissue pain with multiple tender “trigger” points. The patient must have tenderness to pal-
pation at specific tender points. Other clinical manifestations can include weakness, paresthesias, sleep
disturbance, mood disturbance, headache, fatigue, muscle or joint stiffness, and dizziness. Neurologic
examination and all laboratory and clinical investigations are usually within normal limits. There is no
objective evidence of muscle inflammation. The cause of the disorder is not known. Treatment involves a
good physician–patient relationship, exercise, avoiding a sedentary lifestyle, and possibly the use of tricy-
clic antidepressants. Pregabalin is also frequently used in treatment of fibromyalgia.
Psychosomatics
B&D Chapter 73

Question 47. C. Pancreatic cancer has been associated with a high rate of depression. Patients present with apathy, decreased
energy, and anhedonia. It should be a consideration in the clinician’s mind whenever seeing middle-aged
depressed patients. Cancer in general brings about many psychological reactions in patients. Fear of death,
fear of abandonment and disfigurement, loss of independence, denial, anxiety, guilt, financial worries, and
disruption in relationships all play a role. Of patients with cancer, 50% often have comorbid psychiatric diag-
noses, with adjustment disorder, major depressive disorder, and delirium being the most common.
Depressive Disorders
K&S Chapter 13

230

Psychiatry Test Preparation and Review


Manual E-Book
Test Number Four

Question 48. C. This vignette depicts a classic neurologic emergency: acute epidural spinal cord compression (ESCC).
In this case, the causative agent is likely to be a spinal metastasis from the patient’s prostate cancer.
Metastases from primary carcinoma of the breast, lung, and prostate each make up 20% of the causes
of ESCC. Back pain is the most frequent presenting feature. The back pain is usually progressive and
increases with tumor growth over a period of up to 2 months before the onset of neurologic deficits.
Motor weakness is noted in about 80% of cases, and it predicts the posttreatment outcome much of the
time. Sensory deficits are noted in about 75% of cases. Metastatic ESCC causes a spastic paraparesis or
paraplegia with a sensory level that is usually several levels below the actual lesion. Bowel and bladder
incontinence, urinary retention, or constipation are observed in a majority of patients. Sagittal screening
MRI of the entire spine is the initial test of choice. Treatment is undertaken acutely with parenteral cor-
ticosteroids and subacutely with radiation therapy. Decompressive laminectomy or radiotherapy can be
used to treat. Chemotherapy is generally ineffective, because metastases causative of ESCC are usually
not chemosensitive.
Neurology
B&D Chapter 52

Question 49. D. It is important to be able to distinguish conduct disorder from other childhood psychiatric diagnoses. In
conduct disorder the patient shows a pattern whereby the rights of others and societal rules are violated.
This presents as bullying other children, using weapons, physical fighting, cruelty to animals, stealing,
fire setting, destroying property, truancy, or running away from home. Patients with ADHD can have
many behavioral problems, but those problems are a result of inattention, hyperactivity, and impulsiv-
ity. ADHD does not show a pattern in which the child is intentionally or malevolently being violent or
destructive. Any such acts happen as a result of hyperactivity and poor impulse control. In depression
children can become very irritable, withdraw, and not wish to socialize. They may even act out as a
result of how badly they are feeling. However, this is different from a long-standing pattern of actively
trying to carry out violence or do property damage regardless of mood state. In bipolar disorder children
may break rules and have behavioral difficulties during manic and depressive episodes. However, there
will be a clear cycling pattern to their moods (and other symptoms) that corresponds to the times when
their behaviors become problematic. One of the other important distinctions to make is between con-
duct disorder and oppositional defiant disorder (ODD). In ODD there is a pattern of negativistic, hostile,
or defiant behavior directed at adults or authority figures. In conduct disorder the negative behavior is
directed at all others regardless of whether they are authority figures or not. ODD behaviors are there-
fore more targeted and have less of a wide-ranging destructive nature than those of conduct disorder.
Disruptive, Impulse Control, Conduct Disorders, and ADHD
K&S Chapter 31

Question 50. A. Subacute combined degeneration is the result of a deficiency in vitamin B12 (cobalamin). Cobalamin defi-
ciency manifests as macrocytic anemia, atrophic glossitis, and neurologic deficits. Neurologic symptoms
include lesions to the lateral and posterior columns of the spinal cord (subacute combined degeneration),
peripheral neuropathy, optic atrophy, and brain lesions. Spinal cord symptoms present as posterior col-
umn deficits, which can manifest as upper motor neuron limb weakness, spasticity, and Babinski’s signs.
Peripheral neuropathy can present as paresthesias and large-fiber sensory impairment (loss of proprio-
ception and vibration sensation). Cerebral symptoms present as behavioral changes, forgetfulness, and
in severe cases, dementia and stupor. Sensory ataxia is demonstrated by a positive Romberg sign. There
can also be a diffuse hyperreflexia with absent ankle jerk reflexes. Deficiencies of vitamins B6, niacin, and
folic acid are explained elsewhere in this volume.
Neurology
B&D Chapters 57 and 76

Question 51. D. Victims of childhood incest often develop depressive feelings mixed with guilt and shame and a feeling
of permanent damage. Teens that undergo sexual abuse often show poor impulse control, self-destruc-
tive behavior, and suicidal behavior. Adults abused as children often have posttraumatic stress disor-
der and dissociative disorders. Incest is most often perpetrated by fathers, stepfathers, or older siblings.
It is defined as sexual relations between two people who are related or who society has deemed inap-
propriate to be sexually involved. Most reported cases involve father–daughter incest. In addition to

231

Psychiatry Test Preparation and Review


Manual E-Book
Psychiatry Test Preparation & Review Manual

depression, children involved in incest may present to the doctor with complaints of abdominal pain,
genital irritation, separation-anxiety disorder, phobias, or school problems.
Trauma and Stress Related Disorders
K&S Chapter 31

Question 52. B. This question presents the classic lacunar stroke syndrome called pure motor hemiparesis. This lacunar
syndrome classically localizes to the contralateral internal capsule. Two other possible locations of such
a lacunar stroke are the contralateral basis pontis or corona radiata. The pure motor stroke presents
without cortical deficits. There are no sensory or visual deficits. Aphasia, agnosia, and apraxia are all
absent in a pure motor stroke. If both the thalamus and the internal capsule are involved, one would
expect to see a combined sensorimotor stroke, which manifests not only as a contralateral motor hemi-
paresis but also as a contralateral hemisensory loss. The lesion localizes to ischemia in the territory of
the small penetrating lenticulostriate arteries that originate off the proximal middle cerebral artery.
Neurology
B&D Chapter 51

Question 53. A. Only the medical chart should be turned over to the court. The law treats the medical chart separately
from psychotherapy notes. The medical chart is the official record of the patient’s care. The patient has
a right to see it, and it can be subpoenaed into a legal proceeding. Psychotherapy notes belong to the
therapist. They are the therapist’s own notes to him- or herself that serve as a reminder of the details of
the sessions. They are not considered part of the medical record and are not part of the official record of
care. As such, they cannot be subpoenaed. They should not be turned over in this situation. Although it
is always a good idea to get signed permission from the patient to release records, when presented with a
subpoena from a court you do not have the right to withhold the records from the court. However, you
should not release anything more than is specifically requested.
Forensic Psychiatry
K&S Chapter 36

Question 54. D. Choreoathetosis is not a feature of Parkinson’s disease (PD). The classic cardinal clinical features of idio-
pathic PD are akinesia (or bradykinesia), rigidity, resting tremor, and loss of postural reflexes. Tremor is
an oscillating 3 to 5 Hz, seen more in the hands, in which it resembles pill-rolling, but also in the face,
chin, and lower extremities. Bradykinesia refers to slowness of voluntary movements that make it dif-
ficult for the patient to dress, eat, and maintain personal hygiene. Bradykinesia becomes most evident
when a patient attempts to perform rapid alternating movements. Rigidity is often manifested as a cog-
wheeling when the extremities are passively mobilized. Loss of postural reflexes is also a hallmark of
idiopathic PD and is responsible for the many falls sustained by patients with the disease. Other associ-
ated clinical features include masked facies, decreased blink, micrographia, seborrhea, weight loss, con-
stipation, and dysphagia. Gait is classically altered, and patients walk with “petits pas” (small steps) and
a shuffling gait. When they turn around, they turn “en bloc” (without swinging the hips and shoulders,
much like a robot). Other autonomic features include orthostatic hypotension, sweating disorder, and
urinary frequency. Other symptoms include sleep disturbance, restless legs, fatigue, anxiety, depression,
and cognitive and behavioral disturbances. These associated features are not present in every case and
are seen less frequently than the cardinal symptoms.
Neurology
B&D Chapter 71

Question 55. E. Specific learning disorder with impairment in reading is characterized by reading achievement being sub-
stantially below what is expected for the child’s age, intelligence, and education. It interferes with aca-
demic achievement and activities of daily living that require reading skills. In the question given, the
child’s difficulties are all in situations in which she may be asked to read. When children refuse to go
to school, separation-anxiety disorder should also be considered. In this case, however, the child likes
to stay with friends and grandparents, so separation from her parents is not the issue. Children with
specific learning disorder with impairment in reading become depressed and demoralized. The diagno-
sis must be made with a standardized reading test, and autism spectrum disorder, ADHD, and mental

232

Psychiatry Test Preparation and Review


Manual E-Book
Test Number Four

retardation must be ruled out. Treatment consists of understanding the child’s deficits and developing
an educational program to remedy them. The child must be given coping strategies such that he or she
is not overwhelmed and discouraged. Coexisting emotional and behavioral problems should also receive
treatment.
Neurodevelopmental and Pervasive Developmental Disorders
K&S Chapter 31

Question 56. A. Myoclonus is not typically a symptom of botulism toxin poisoning. Botulism results from the ingestion of C.
botulinum, a bacterium that extrudes an exotoxin into the circulation and that blocks acetylcholine release
at the neuromuscular junction. The bacterium infects humans via its presence in tainted food or via wound
contamination from dirt or soil containing the organism. Classic symptoms of botulism include dysphagia,
dysarthria, ptosis, diplopia, and urinary retention. Other systemic symptoms include dry mouth and lethargy.
Pupillary reflexes are usually impaired. These symptoms rapidly progress to limb paralysis and eventually to
paralysis of respiratory muscles that can lead to death if the condition is untreated. Gastrointestinal symptoms
of nausea, vomiting, and diarrhea often present with neurologic compromise after a 12- to 36-hour incubation
period after ingestion of the toxin. Infants who consume unpasteurized honey may ingest spores and can pres-
ent with weak cry, lethargy, floppiness, poor suck, and constipation. Diagnosis can be established by wound or
stool culture. Electromyography and nerve conduction studies can reveal characteristic anomalies compatible
with presynaptic neuromuscular blockade. Treatment is supportive, particularly with respect to airway protec-
tion, and trivalent equine antitoxin administration can help reverse the effects of circulating toxin.
Neurology
B&D Chapter 78

Question 57. D. This question gives a clear case of ODD. In ODD a child shows a pattern of negativistic, hostile, and
defiant behavior directed at adults or authority figures. This behavior may include temper tantrums,
arguing with adults, actively defying adults’ requests or rules, deliberately annoying people, blaming oth-
ers for one’s own mistakes or misbehavior, being easily annoyed by others, being angry and resentful,
or being spiteful or vindictive. The child in question should not meet criteria for conduct disorder to be
diagnosed with ODD. The primary treatment for ODD is therapy for the child and parental training to
give parents management skills. Often behavioral therapy will be used to reinforce good behavior while
ignoring or not reinforcing bad behavior.
Disruptive, Impulse Control, Conduct Disorders, and ADHD
K&S Chapter 19

Question 58. E. Bálint’s syndrome is a rare stroke syndrome resulting from ischemic lesions to the bilateral parietal-
occipital lobes or the occipital lobes alone. The condition is often a complication of vascular dementia
and can occur after a series of strokes. It can sometimes be the result of a “top of the basilar syndrome.”
The top of the basilar syndrome results from basilary artery occlusion that is often embolic in origin.
This results in infarction to the midbrain, thalamus, and parts of the temporal and occipital lobes. Top
of the basilar syndrome presents with delirium, peduncular hallucinosis (brain-stem-induced visual hal-
lucinations), obtundation, and memory deficits. There may be a gaze palsy, skew deviation of the eyes,
ocular bobbing, impaired convergence, or convergence–retraction nystagmus.

Bálint’s syndrome presents with ocular apraxia (the inability to scan extrapersonal space appropriately with
the eyes), optic ataxia (dysmetric or saccadic jerks that can impede vision and ocular focus), and deficits in
visual attention. Simultanagnosia may also be noted, which is an inability to perceive a scene with multiple
parts to it. The patient is able to see and recognize only individual elements of a multipart scene and can-
not interpret the overall picture. Visual fields are generally normal. Bálint’s syndrome can be accompanied
by a complete or partial Gerstmann’s syndrome if the dominant parietal lobe is affected in the area of the
angular gyrus (Gerstmann’s syndrome is explained elsewhere in this volume). Bálint’s syndrome may also
result in visuoconstructional apraxia (the inability to put things in their proper place or order). The other
answer choices are explained in other questions in this volume, and examination candidates are strongly rec-
ommended to review these lobar syndromes, as they appear frequently on standardized tests.
Neurology
B&D Chapters 51 and 66

233

Psychiatry Test Preparation and Review


Manual E-Book
Psychiatry Test Preparation & Review Manual

Question 59. D. Although there are no specific behaviors that prove that sexual abuse has taken place, children that have
been abused often behave in certain patterns. If very young children have detailed knowledge of sexual
acts, they have usually witnessed or participated in sexual behavior. They express their sexual knowledge
through play and may initiate sexual behavior with other children. Abused children can also be very
aggressive. Other signs of sexual abuse include bruising, pain or itching of the genitals, genital or rectal
bleeding, recurrent urinary tract infections, or the presence of sexually transmitted disease.
Trauma and Stress Related Disorders
K&S Chapter 31

Question 60. D. Anton’s syndrome is considered to be an agnosia characterized by cortical blindness. The lesion localizes
to the bilateral occipital lobes, usually owing to strokes, particularly in the calcarine cortex (Brodmann’s
area 17) and visual association cortex. The arterial territory involved is that of the bilateral posterior
cerebral arteries. The hallmark of the syndrome is that patients are unaware or deny that they are blind
and they confabulate. Patients may also suffer from visual hallucinations.

Prosopagnosia is the inability to recognize familiar faces. It is associated classically with bilateral occip-
ital-temporal lesions. It is often associated with agraphia and achromatopsia (inability to recognize col-
ors and hues). It is almost always associated with visual field deficits. Aphasia is a disorder of speech
and language that can be expressive, receptive, or both. Aphasia occurs subsequent to lesions to the
dominant hemisphere. Apraxia is the inability to perform simple motor tasks that have been previously
learned. Motor apraxia can result from lacunar infarcts to the internal capsule or pons or from non-
dominant hemispheric strokes. Hiccups are often the result of phrenic nerve irritation. This is frequently
caused by medication side effects, such as DOPA repletion therapy in PD.
Neurology
B&D Chapters 14 and 66

Question 61. D. Dependent personality disorder (DPD) is the least likely of the answer choices given to present with vio-
lence. Patients with DPD subordinate their needs to those of others, get others to assume responsibil-
ity for major areas of their life, lack self-confidence, and are uncomfortable when alone. They have an
intense need to be taken care of that leads them to be clingy and submissive and fear separation. They
cannot make decisions without an excessive amount of advice from others. Because of their submissive-
ness, patients with DPD are less likely to become violent than patients with other disorders. People with
the other personality disorders listed can be aggressive, outrageous, and at odds with others. Some patients
with these disorders may have a clear history of violence. Treatment of DPD consists of therapy to modify
the patient’s interpersonal interactions and medication to deal with comorbid anxiety and depression.
Personality Disorders
K&S Chapter 27

Question 62. B. Chronic abuse of n-hexane or other hydrocarbon inhalants (glue, paint thinner) can lead to distal sen-
sorimotor polyneuropathy. The first manifestation is usually numbness and distal paresthesia, which is
subsequently followed by distal motor weakness. If inhalants continue to be abused, the motor weakness
can worsen and can spread to involve the proximal muscles of both the arms and the legs. At low to
moderate doses, the inhalants cause dysarthria, uncoordination, euphoria, and relaxation. Higher doses
and more chronic exposure can lead to hallucinosis, psychosis, and seizures, as well as more systemic
end organ damage, such as renal failure, bone marrow suppression, and hepatic failure.
Substance Abuse and Addictive Disorders
B&D Chapter 20

Question 63. D. Delusional disorder is characterized by a fixed false belief that is either bizarre or nonbizarre (i.e., may or may
not be possible, such as being poisoned, infected, followed, or having a disease versus being impregnated by
snakes or believing you are Jesus Christ). It is differentiated from schizophrenia by the lack of positive symp-
toms (no auditory or visual hallucinations, no thought disorganization). Tactile or olfactory hallucinations
may be present. Apart from the direct effect of the delusion, the patient does not have a marked impairment
in functioning. In the case of somatic delusions, the person believes he or she has some physical defect or
medical condition. Treatment consists of antipsychotics in addition to psychotherapy.

234

Psychiatry Test Preparation and Review


Manual E-Book
Test Number Four

The other answer choices do not fit the question stem. The patient does not have a neurologic deficit, so
conversion disorder (functional neurological symptom disorder) is incorrect. He does not have auditory
or visual hallucinations and does not have significant negative symptoms and functional impairment, so
schizophrenia is unlikely. There is no mention of depressive symptoms. This would not be considered
illness anxiety disorder, because illness anxiety disorder presents with an unreasonable obsession with a
perceived medical problem that could be real. This case clearly presents a delusion that has no possible
basis in reality, which would push it out of the realm of illness anxiety disorder.
Psychotic Disorders
K&S Chapter 7

Question 64. C. Stimulants such as cocaine typically lower the seizure threshold when abused to intoxication. In moder-
ate doses, cocaine and the stimulants can produce wakefulness, alertness, mood elevation, diminished
appetite, and increased performance in certain tasks. The stimulants can also result in psychosis and
paranoia. There is also an increased risk of myocardial infarction with cocaine use. Systemic problems
such as dehydration, rhabdomyolysis, and hyperthermia can also be noted. The most common neuro-
logic adverse effect of cocaine and the stimulants is headache. In certain cases, the stimulants can pro-
duce myoclonus, encephalopathy, and seizures. Cocaine is the stimulant that is most likely to induce
seizures. Smoking and intravenous administration of cocaine are more likely to induce seizures than
intranasal use. Ischemic and hemorrhagic strokes can also result from cocaine use. Cocaine is the most
significant cause of drug-induced stroke and accounts for about 50% of all cases. The mechanism of
cocaine-induced stroke is believed to involve vasoconstriction, acute hypertension, and vasospasm.

Opioid ingestion and intoxication can produce a state of euphoria or dysphoria. Other possible adverse
effects include hallucinations, dry mouth, nausea, vomiting, constipation, and urinary retention.
Examination usually reveals pupillary constriction during acute intoxication. Autonomic disturbance,
such as hypotension and hypothermia, can also be noted. Seizures are rare with opioid intoxication.
Overdose can result in coma and respiratory compromise. Opioids do not usually cause seizures or
lower the seizure threshold.

PCP intoxication produces hallucinosis, dysphoria, and paranoia. Agitation, catatonia, and bizarre
behavior are also common. At higher doses, PCP use can lead to stupor and coma. In overdose, rhabdo-
myolysis may result from agitation and dysautonomia, such as fever, hypertension, and sweating. PCP
can lower the seizure threshold, but less frequently than cocaine.

Tetrahydrocannabinol (THC), the primary active ingredient in marijuana, causes euphoria, deperson-
alization, and relaxation. Other effects of the drug include sleepiness, paranoia, and anxiety. High doses
of THC can result in hallucinosis, panic, and even paranoia. Seizures are not generally noted with THC
intoxication.

Alcohol has intoxicating effects that can cause sedation, memory impairment, uncoordination, dysar-
thria, euphoria, dysphoria, sleepiness, and acute confusion. Alcohol intoxication does not lower the sei-
zure threshold; it, in fact, is protective against seizures because of its agonistic effects at the GABA-A
receptor. Alcohol withdrawal can lower the seizure threshold because of rapid desaturation of the
GABA-A receptor, in much the same way as the benzodiazepines.
Substance Abuse and Addictive Disorders
B&D Chapter 58

Question 65. E. Mark, who has major depressive disorder, has the highest likelihood of completing suicide. About 95%
of those who attempt or commit suicide have some form of psychiatric disorder. Major depressive dis-
order accounts for 80% of suicides, schizophrenia accounts for 10%, and dementia or delirium for 5%.
Of those who have problems with alcohol, 15% attempt suicide, but the numbers are less than for those
with depression. Personality disorders may contribute to suicide attempts, but numbers do not surpass
those for depression, and often the two overlap. Mental retardation is not a significant risk factor for
suicide. Always keep in mind that the best predictor of future suicidal behavior is past suicidal behavior.
Management in Psychiatry
K&S Chapter 23

235

Psychiatry Test Preparation and Review


Manual E-Book
Psychiatry Test Preparation & Review Manual

Question 66. D. Meige’s syndrome denotes the combination of blepharospasm (involuntary eyelid blinking) and oroman-
dibular dystonia. Patients have involuntary contraction of the eyelids and of the lower facial muscles
in the area of the jaw, tongue, or neck. The treatment of choice is chemodenervation of the hyperac-
tive muscles by botulinum toxin type A injection. The tongue requires careful injection in these cases,
because it can fall back and occlude the airway if it is weakened too severely. The other answers are sim-
ply distractors.
Neurology
B&D Chapter 16

Question 67. D. The Tarasoff rule came as a result of the Tarasoff court case. In this case it was decided that a physician
or therapist who has reason to believe that a patient might injure or kill someone must notify the poten-
tial victim, the victim’s family or friends, and the authorities.

The Durham rule is no longer used, but it said that an accused is not responsible if his or her unlawful
actions were the result of mental disease or defect.

The M’Naghten rule comes from British law and states that a patient is not guilty by reason of insan-
ity if he or she has a mental disease such that he or she was unaware of the nature, quality, and conse-
quences of his or her actions and was incapable of realizing that his or her actions were wrong.

Ford v Wainwright was a case that sustained the need for a patient to be competent to be executed. Also
worthy of note is that psychiatrists are ethically bound not to participate in state-mandated executions
in any way.

Respondeat superior is a legal concept stating that a person at the top of a hierarchy is responsible for
the actions of those at the bottom of the hierarchy.
Forensic Psychiatry
K&S Chapter 36

Question 68. A. Scanning speech is essentially a form of dysphasia that causes a cerebellar “dysmetria” of the speech
pattern. It is also known as ataxic dysarthria. Lesions of the cerebellum, such as with strokes, degenera-
tion, or tumor, can cause this condition. Speech rhythm is usually irregular and choppy. This can also be
accompanied by a slow, labored speech pattern.
Neurology
B&D Chapter 12

Question 69. B. As many as 40% of mothers may experience mood or cognitive symptoms during the postpartum
period. Postpartum blues (or maternity blues) is a normal state of sadness, dysphoria, tearfulness, and
dependence, which may last for several days and is the result of hormonal changes and the stress of
being a new mother.

Postpartum depression is more severe and involves neurovegetative signs and symptoms of depression
and potential suicidality.

Postpartum psychosis can involve hallucinations and delusions, as well as thoughts of infanticide.

There is no specific phobia regarding being a parent recognized by the Diagnostic and Statistical Manual
of Mental Disorders. The fear of the responsibility associated with parenthood may play a part in post-
partum blues or postpartum depression but is not a distinct entity.
Depressive Disorders
K&S Chapter 27

Question 70. D. Aphemia is a motor speech disorder characterized by near muteness along with normal reading, writing,
and comprehension. Aphemia is likely to result from lesions to the primary motor cortex or Broca’s area.
Patients are first mute, then they become able to speak with hesitancy and phonemic substitutions.

Aphasia is simply any acquired disorder of language. Aphasias can take multiple forms, including
expressive, receptive, conduction, global, or transcortical varieties.

236

Psychiatry Test Preparation and Review


Manual E-Book
Test Number Four

Agnosia is the inability to recognize and identify objects. Agnosias imply that the patient is able to see,
hear, or touch the object to be identified and that the patient does not have a sensory or perceptual defi-
cit that could impair perception of the object.

Apraxia is the inability to perform a skilled, learned, purposeful motor behavior in the absence of other
deficits that may impair motor functioning. Abulia resembles akinetic mutism. Patients display severe
apathy, with affective blunting, amotivation, and immobility. There is an absence of spontaneous speech
and movement. Patients retain awareness of their environment.
Neurology
B&D Chapters 5 and 12

Question 71. B. The NMDA receptor is one of the best known glutamate receptors. It has been found to play a role in
learning and memory, as well as in psychopathology. The other glutamate receptors are known as the non-
NMDA glutamate receptors. The NMDA receptor allows sodium, potassium, and calcium to pass through.
It opens when bound by two glutamate molecules and one glycine molecule at the same time. The receptor
can be blocked by physiological concentrations of magnesium and bound by PCP and PCP-like substances.
Basic Neuroscience
K&S Chapter 1

Question 72. C. Stupor refers to a state of unresponsiveness from which arousal occurs only with vigorous and repeated
stimulation. It is on the continuum between alertness and coma. Alert patients are awake and in a nor-
mal state of arousal. Lethargic patients are sleepy but awake and arousable to alertness with stimu-
lation. Coma implies a state of unarousable unresponsiveness. The persistent vegetative state follows
coma. Patients lose cognitive functioning but retain vegetative, or autonomic, functioning (such as car-
diac function, respiration, and blood pressure maintenance).
Neurology
B&D Chapter 5

Question 73. E. Of the various somatoform disorders, the one with the best prognosis is conversion disorder (functional
neurological symptom disorder). The somatic symptom and related disorders are characterized by physi-
cal symptoms suggestive of a medical condition but are not fully explained by a medical condition or
substance abuse. The symptoms are not intentionally produced, as in factitious disorders or malingering.
Somatic symptom disorder has a poor prognosis. It can present with or without predominant pain. In
illness anxiety disorder the patient is falsely convinced that he or she has a serious disease, often based
on the misinterpretation of bodily symptoms or functions. Illness anxiety disorder has a fair to good
prognosis. Conversion disorder (functional neurological symptom disorder) is the development of one or
more neurologic deficits that cannot be explained by a known medical disorder. Psychological factors are
often associated with the onset of the deficit. The prognosis is excellent. Somewhere from 90% to 100%
of patients with conversion disorder (functional neurological symptom disorder) are in remission within
less than a month. Body dysmorphic disorder is characterized by the false belief or exaggerated percep-
tion that a body part is defective. The prognosis is usually poor, and the disease is chronic.
Somatic Symptom Disorders
K&S Chapter 13

Question 74. B. All of these answer choices are at least partly untrue, except for answer choice B. Comatose patients
require a head CT scan before lumbar puncture (LP) to assess if there is the presence of a mass lesion
or acute hemorrhage that could result in transtentorial or cerebellar herniation if an LP were to be
performed.

Not every patient with Guillain–Barré syndrome (GBS) should be hospitalized in an intensive care unit.
Mild forms of the disease can be monitored in a regular inpatient setting, and bedside respiratory peak-
flow monitoring can be assessed frequently to screen for acute respiratory compromise that could lead to
death. Patients with GBS who deteriorate rapidly or show signs of poor oxygenation can be transferred
to an intensive care or pulmonary unit for close monitoring, and intubation can be undertaken if deemed
clinically necessary.

237

Psychiatry Test Preparation and Review


Manual E-Book
Psychiatry Test Preparation & Review Manual

A positive grasp reflex is usually a pathological sign in an adult. It is a normal infantile reflex that is
present at birth and usually disappears by 6 months of age. Persistence or redevelopment of the reflex
in adulthood would be indicative of frontal lobe pathology, and the grasp reflex is considered one of the
frontal-release signs.

Cerebellar hemispheric lesions produce movement deficits that affect the side of the body ipsilateral to
the lesion. This is due to the double-crossing of pathways. Ascending cerebellocortical tracts decussate
in the midbrain and proceed to the contralateral cortex. Descending corticospinal tracts (the pyramidal
tracts) decussate in the medulla to project to the contralateral body.

Bell’s palsy is believed to be the result of a herpes simplex infection that affects the Gasserian ganglion.
Borrelia burgdorferi, the spirochete responsible for Lyme infection, can indeed cause facial diplegia, but
is a much rarer infection than that caused by the herpes simplex virus.
Neurology
B&D Chapter 76

Question 75. D. The American Psychiatric Association considers it unethical for a psychiatrist to accept a patient’s estate
after death. It is considered an exploitation of the therapeutic relationship. It is acceptable to accept
a token bequest that you were unaware was in the will when the patient was alive. All of the other
acts listed are ethical. Psychiatrists cannot participate in executions (often asked on examinations).
Abandoning patients without arranging for follow-up care is unethical. It is unethical to release infor-
mation about a patient to another party without the patient’s permission. It is unethical to pay another
doctor for sending you referrals. You can rent to other doctors space in your office. And it is ethical to
charge a fee for supervision.
Ethics
K&S Chapter 36

Question 76. A. The amygdala is necessary for the recall of emotional contexts of specific events and the experience of
fear, pleasure, or other emotions associated with these events. Declarative or episodic memory (also
known as short-term memory) requires the intact functioning of the hippocampus and parahippocampal
areas (nucleus basalis of Meynert) of the medial temporal lobe for storage and retrieval of information.
The other answers are distractors.
Basic Neuroscience
B&D Chapter 6

Question 77. C. Desipramine is the least anticholinergic of the tricyclic antidepressants (TCAs). Anticholinergic side
effects are common with patients on TCAs, but tolerance develops over time. Amitriptyline, imipramine,
trimipramine, and doxepin are the most anticholinergic. Amoxapine, nortriptyline, and maprotiline are
less so. Anticholinergic side effects include dry mouth, blurred vision, constipation, and urinary reten-
tion. Because of this last side effect, desipramine would be the best choice for someone with prostatic
hypertrophy. One should always keep in mind the potential for severe anticholinergic effects to lead to a
delirium.
Psychopharmacology
K&S Chapter 29

Question 78. D. Children are expected to reach developmental milestones at the appropriate age. Standardized examina-
tion candidates should memorize these milestones, as questions concerning this material come up on
many different examinations. Here are the milestones that need to be remembered:
• 2 months of age: cooing, smiling with social contact, holding head up 45 degrees
• 4
 months of age: laughing/squealing, sustaining social contact, grasping objects, weight bearing on
legs
• 6
 months of age: imitating speech sounds, single syllables, prefers mother, enjoys mirror, transferring
objects hand to hand, raking grasp, sitting up with support

238

Psychiatry Test Preparation and Review


Manual E-Book
Test Number Four

• 8
 months of age: jabbering, playing peek-a-boo, playing patty-cake, waving bye-bye, sitting without
support, creeping or crawling
• 1
 2 months of age: speech specific to “dada/mama,” playing simple ball games, able to adjust body to
dressing, standing alone, able to use thumb–finger pincer grasp
• 1
 4 months of age: one- to two-word vocabulary, indicating desires by pointing, hugging parents,
walking alone, stooping, and recovering
• 1
 8 months of age: six-word vocabulary, able to feed self, walking up stairs while hand is held, imitat-
ing scribbling
• 2
 4 months of age: combining words, 250-word vocabulary, helping to undress, listening to picture
stories, running well, making circular scribbles, copying a horizontal line
• 3
 0 months of age: knows full name, refers to self as “I,” pretending in play, helping put things away,
climbing stairs with alternating feet, copying a vertical line
• 3
 6 months of age: counting three objects correctly, knowing age and sex, helping in dressing, riding a
tricycle, standing briefly on one foot, copying a circle
• 4
 8 months of age: telling a story, counting four objects, playing with other children, using toilet
alone, hopping on one foot, using scissors to cut out pictures, copying a square and a cross
• 6
 0 months of age: naming four colors, counting 10 objects, asking about word meanings, domestic
role playing, skipping, copying a triangle.
Human Development
B&D Chapter 7

Question 79. A. This question is a case of exhibitionistic disorder. Exhibitionistic disorder is a paraphilia in which
patients have a recurrent urge to expose their genitals to strangers. Sexual arousal is brought about
by the event. Cases are almost always men exposing themselves to women. Medroxyprogesterone ace-
tate has been shown to be helpful in some cases and can be useful in any sexual disorder in which the
patients are extremely hypersexual to the point of being out of control or dangerous. Other drugs such
as antipsychotics and antidepressants have not been shown to be particularly useful in such cases. Some
patients may improve with the sexual side effects of a serotonin-selective reuptake inhibitor, but it would
not be the first choice for treatment.
Sexual Dysfunctions
K&S Chapter 17

Question 80. B. The clinical vignette described in this question involves a classic presentation of Hoover’s sign. Hoover’s
sign is positive when a patient suspected of a hysterical or psychogenic hemiparesis does not give effort
in the contralateral (unaffected) lower extremity when asked to push down on the bed with the paretic
(affected) lower extremity. The examiner places a hand under the patient’s heel on the unaffected side
to feel if the patient is pushing down toward the bed in an attempt to give a full effort at raising the
affected leg. In a real hemiparesis the patient would be expected to make every effort to brace himself
with the unaffected leg while trying to raise the paretic leg.

Hoffmann’s sign is equivalent to Babinski’s sign, but it is noted in the upper extremity. The sign is posi-
tive when a flick of the distal phalange of the index or middle finger results in an adduction of the ipsi-
lateral thumb. The sign, when present, indicates contralateral corticospinal tract damage that affects the
upper extremity.

Lasègue’s sign is present when straight-leg raising in a recumbent position results in reproduction of pain
or paresthesia in the sciatic distribution. The sign can be either ipsilateral or crossed. The positive sign
points to sciatic nerve compression due to mechanical interruption of the nerve trajectory, most often
due to an intervertebral disk bulge or herniation at the lumbar or sacral level.

Romberg’s sign is positive when a patient is asked to stand up straight with eyes closed and subsequently
loses his or her balance. The presence of Romberg’s sign usually signals a deficit localizing either to the
posterior columns of the spinal cord (i.e., loss of proprioception and/or vibration sensation in the legs),
to cerebellar pathology, or both.

239

Psychiatry Test Preparation and Review


Manual E-Book
Psychiatry Test Preparation & Review Manual

Gegenhalten refers to “clasp-knife” type rigidity that can be observed in the extremities in several types
of disorders that can include stroke, MS, and catatonia.
Neurology
B&D Chapter 22

Question 81. E. Clomipramine is one of the TCAs. It has been useful in the treatment of patients with premature ejacula-
tion, depression, panic, OCD, phobias, and pain disorder. It has no use in treating psychosis. It carries
with it the cardiac, autonomic, neurologic, sedative, and anticholinergic effects found with many of the
TCAs. All TCAs should be avoided during pregnancy.
Psychopharmacology
K&S Chapter 29

Question 82. C. The only true emergency in neurology that requires immediate MRI imaging is acute ESCC (epidural
spinal cord compression). If MRI is unavailable, the alternate imaging modality of choice is a spinal
myelogram. The cause of spinal cord compression can be an intervertebral disk, metastatic carcinoma,
or epidural hematoma. Metastases from primary carcinoma of the breast, lung, and prostate each make
up 20% of the causes of ESCC. Back pain is the most frequent presenting feature in over 80% of cases.
The back pain is usually progressive and increases with tumor growth before the onset of neurologi-
cal deficits. Motor weakness is noted in about 80% of cases, and it predicts the posttreatment outcome
much of the time. Sensory deficits are noted in about 75% of cases. Metastatic ESCC causes a spastic
paraparesis or paraplegia with a sensory level that is usually several levels below the actual lesion. Bowel
and bladder incontinence, urinary retention, or constipation are observed in a majority of patients.
Sagittal screening MRI of the entire spine is the initial test of choice, if available, because only MRI and
myelography can immediately identify and characterize the nature of the lesion and guide treatment.
Treatment is undertaken acutely with parenteral corticosteroids and subacutely with radiation therapy.
Decompressive laminectomy can be performed, but radiotherapy has of late produced results as good as
a surgical approach. Chemotherapy is generally ineffective, because metastases causative of ESCC are
usually not chemosensitive. The other answers are clearly distractors and are discussed individually else-
where in this volume.
Neurology
B&D Chapter 52

Question 83. C. The legal concept of parens patriae allows the state to intervene and act as a surrogate parent for those
who are unable to care for themselves or who may harm themselves.

Actus reus means voluntary conduct. Mens rea means evil intent. Durable power refers to durable power
of attorney, in which patients select who they want to make decisions for them should they become
incompetent to make those decisions for themselves. Respondeat superior is the concept that a person at
the top of a hierarchy is responsible for the actions of those at the bottom of the hierarchy.
Forensic Psychiatry
K&S Chapter 36

Question 84. A. This patient’s symptoms are characteristic of Cushing’s disease. Cushing’s disease is caused by pituitary
hypersecretion of ACTH leading to excess glucocorticoids. It can also be the result of tumors of the
adrenal glands. Symptoms include central obesity, hirsutism, glucose intolerance, menstrual irregularities,
hyperpigmentation, striae, hypertension, and easy bruising. Patients with Cushing’s disease are prone to
depression, anxiety, and agitation. They can also develop paranoia and memory problems. Treatment
can involve pituitary irradiation, chemotherapy, or surgery. If the adrenal glands are removed, the
patient will need glucocorticoid and mineralocorticoid replacement therapy.
Somatic Symptom Disorders
K&S Ch. 13

240

Psychiatry Test Preparation and Review


Manual E-Book
Test Number Four

Question 85. E. The trail-making test is a measure of executive function. The second part of the test consists of drawing
lines between a series of letters and numbers in the correct order. For example, the patient would con-
nect A-1-B-2-C-3, etc. The trail-making test is given as part of the Halstead–Reitan test battery.
Psychological Theory and Psychometric Testing
K&S Chapter 5

Question 86. C. Memantine (Namenda) is an NMDA antagonist that is FDA-approved for the treatment of moderate
to severe Alzheimer’s dementia. Its mechanism of action is completely different from that of donepezil
(Aricept), rivastigmine (Exelon), and galantamine (Reminyl), which are inhibitors of acetylcholinester-
ase that are FDA-approved for treatment of mild to moderate Alzheimer’s dementia. There is now evi-
dence to suggest that concomitant therapy with memantine and one of the cholinesterase inhibitors has
measurable clinical advantages over therapy with either class of agent on its own. Neither of these two
classes of agent attacks the true pathophysiology of Alzheimer’s dementia, which is the formation of
neurofibrillary plaques and tangles in the brain that lead to neuronal cell death. Memantine may reverse
the process of apoptosis (preprogrammed cell death) that is believed to be intrinsic to the pathologi-
cal basis of cell death in Alzheimer’s dementia. Glutamate and NMDA hyperstimulation may lead to
premature cell death by promoting calcium influx into neurons, and this in turn leads to progression of
Alzheimer’s dementia. Memantine may slow disease progress by modulating and lessening the adverse
effects of glutamate on the brain. The other answer choices are distractors and need no particular
explanation.
Psychopharmacology
B&D Chapter 66, K&S Chapter 29

Question 87. A. It is the GABA-A receptor that has binding sites for the benzodiazepines. There are three types of GABA
receptors, GABA-A, -B, and -C. There are no GABA-D or GABA-E receptors. The benzodiazepines
increase the affinity of the GABA-A receptor for GABA. The GABA-A receptor is a chloride ion channel.
Psychopharmacology
K&S Chapter 1

Question 88. A. Vagal nerve stimulation has been FDA-approved since 1997 for adjunctive treatment of refractory par-
tial complex epilepsy in patients over 12 years of age. It also obtained FDA approval for treatment of
intractable major depressive disorder. The modality involves the invasive implantation of an electric
stimulating device inside the chest cavity. The stimulator is connected to a wire that wraps around the
left vagal nerve (the right vagal nerve is not clinically important in the management of epilepsy). The
stimulator is programmed to cycle on for 30 seconds and off for 5 minutes throughout the day. Seizure
frequency is typically reduced by about 50%. Side effects can include hoarseness from irritation of the
adjacent recurrent laryngeal nerve and throat tingling and/or coughing during actual stimulation.
Neurology
B&D Chapter 67

Question 89. D. It is true that some studies have shown the efficacy of using carbamazepine in the treatment of alcohol
withdrawal to be equal to that of the benzodiazepines. Carbamazepine is approved in the United States
for temporal lobe and generalized epilepsy and for trigeminal neuralgia. It is metabolized by the liver
and excreted by the kidneys. It causes its own autoinduction by hepatic enzymes, which makes more
medication necessary over time to achieve the same blood levels. It also affects the metabolism of several
other drugs. Carbamazepine has been associated with a transient decrease in white blood cell count and
has been associated with an inhibition of colony-stimulating factor in the bone marrow. It has also been
associated with severe blood dyscrasias, such as aplastic anemia and agranulocytosis. A benign rash has
been found in 10% to 15% of patients on carbamazepine, and a small percentage of these patients go
on to develop serious rashes, such as Stevens–Johnson syndrome, exfoliative dermatitis, erythema multi-
forme, or toxic epidermal necrolysis.
Psychopharmacology
K&S Chapter 29

241

Psychiatry Test Preparation and Review


Manual E-Book
Psychiatry Test Preparation & Review Manual

Question 90. E. Auscultation of the head that reveals a bruit is a classic hallmark of an arteriovenous malformation
(AVM). Seizures are the presenting symptom of AVMs in about one-quarter to two-thirds of cases.
Headaches occur in anywhere from 5% to 35% of cases. Hemorrhage from ruptured AVM tends to be
intracerebral in location rather than subarachnoid, as in the case of a ruptured aneurysm. The other dis-
tractors are discussed elsewhere in this volume.
Neurology
B&D Chapter 51

Question 91. D. The draw a person test is a projective test. It is administered by telling the patient to draw a person.
Then the patient is asked to draw a person of the sex opposite that of the first drawing. The assump-
tion is that the drawing of the patient’s gender is representative of the self in the environment. The level
of detail is also correlated with intelligence in children. The Halstead–Reitan battery is used to find the
location and effects of certain brain lesions. It is not projective. The Stanford–Binet test is an intelligence
test. The Wechsler–Bellevue test is a memory test. The MMPI is a self-report inventory used to assess
personality traits.
Psychological Theory and Psychometric Testing
K&S Chapter 5

Question 92. B. The image depicts a classic colloid cyst of the third ventricle. This is a
rare condition that presents in middle-aged adults. The cyst is a round,
well-circumscribed lesion that is situated in the anterior aspect of the
third ventricle. It appears as a “button nose” right in the middle of
the ventricular system, and this MR image is a typical example of its
appearance. Clinical presentation is that of intermittent headaches that
result from increased intracranial pressure because of the ball–valve
blockage of the passage of CSF in the ventricular system. The block-
age can even lead to brief intermittent drop attacks in certain patients.
These cysts are composed of high concentrations of cholesterol and
protein that give them their characteristic hyperintense appearance, par-
ticularly on T2-weighted brain MRI.
Neurology
B&D Chapter 52

Question 93. C. Buprenorphine is a mixed opioid agonist/antagonist. It is used for the treatment of heroin addiction as
an alternative to methadone. Aripiprazole is a mixed dopamine agonist/antagonist. Naltrexone is an opi-
oid antagonist. Methadone is an opioid agonist. Gabapentin is an anticonvulsant and is thus unrelated
to the opioid receptors.
Psychopharmacology
K&S Chapter 20

Question 94. D. Hallervorden–Spatz syndrome is a rare autosomal recessive disease of childhood onset that presents with
the combination of dementia and parkinsonism. The classic neuropathologic hallmark of the disorder is
a rusty brown discoloration of the medial globus pallidus and pars reticulata of the substantia nigra on
autopsy. The discoloration is due to the accumulation of iron in the basal ganglia. The disorder is caused
by an enzymatic deficiency in cysteine dioxygenase. This leads to increased levels of cysteine in the brain,
which chelates iron. Free iron is deposited in the basal ganglia and leads to free radical formation, neu-
ronal demise, and ultimately death. Other features of the syndrome include optic atrophy, pigmentary
retinopathy, psychomotor retardation, and clinical signs of corticospinal tract damage.
Neurology
B&D Chapter 66

Question 95. C. Anterograde amnesia is most often associated with alcohol abuse. Keep the Wernicke–Korsakoff syn-
drome in mind. Wernicke’s encephalopathy is an acute disorder characterized by ataxia, vestibular
dysfunction, confusion, and eye movement abnormalities. Korsakoff’s syndrome is a chronic amnestic

242

Psychiatry Test Preparation and Review


Manual E-Book
Test Number Four

syndrome that can follow Wernicke’s. It presents with impaired recent memory and anterograde
amnesia. The patient may or may not confabulate as well. Long-term memory is usually not affected.
Treatment for both of these conditions is thiamine administration. If not treated early, permanent dam-
age can take place, and Korsakoff’s syndrome can become permanent.
Substance Abuse and Addictive Disorders
K&S Chapter 20

Question 96. A. Subacute combined degeneration is the result of a deficiency in vitamin B12 (cobalamin). Cobalamin defi-
ciency manifests as macrocytic anemia, atrophic glossitis, and neurologic deficits. Neurologic symptoms
include lesions to the lateral and posterior columns of the spinal cord (subacute combined degeneration),
peripheral neuropathy, optic atrophy, and brain lesions. Spinal cord symptoms present as posterior col-
umn deficits, which can manifest as upper motor neuron limb weakness, spasticity, and Babinski’s signs.
Peripheral neuropathy can present as paresthesias and large fiber sensory impairment (loss of proprio-
ception and vibration sensation). Cerebral symptoms present as behavioral changes, forgetfulness, and in
severe cases, dementia and stupor. Sensory ataxia is demonstrated by a positive Romberg sign. There can
also be a diffuse hyperreflexia with absent ankle jerk reflexes. Treatment is undertaken with parenteral
B12 injections intramuscularly.
Neurology
B&D Chapter 76

Question 97. C. The appropriate action to take in the case presented is to increase the carbamazepine dose and take
follow-up levels. This is because carbamazepine has an autoinduction phenomenon whereby it causes
the induction of the hepatic enzymes that break it down. Thus, after starting at one dose, the enzymes
are induced and break down more carbamazepine, thereby decreasing the serum level. The answer? Give
more carbamazepine to bring serum levels back up. The informed clinician knows that this phenomenon
will happen and as such knows the patient is being honest. There is no need to switch to another medi-
cation if the carbamazepine was keeping the patient stable. Adding a serotonin-selective reuptake inhibi-
tor would be a good way to flip the patient into overt mania, but not a good way to solve this problem.
The patient in question has yet to meet criteria for hospitalization, so that would just be overly aggres-
sive and unwarranted.

While we’re on the topic of carbamazepine, keep in mind that giving ketoconazole to a patient on car-
bamazepine will both significantly increase carbamazepine levels and lower ketoconazole levels. The
patient will appear over-sedated from the carbamazepine or have carbamazepine associated side effects,
and the fungal infection will not be effectively treated. What a valuable little fact for the prudent student
to know!
Psychopharmacology
K&S Chapter 29

Question 98. A. INH exposure can cause a vitamin B6 (pyridoxine) deficiency. Most normal adults consume adequate
amounts of vitamin B6 in their diets (1.5 to 2 mg daily). Hydralazine and penicillamine can also cause
drug-induced vitamin B6 deficiency. These drugs interfere with vitamin B6 coenzyme activity. Vitamin B6
deficiency results in a distal sensorimotor peripheral neuropathy. Patients can develop distal paresthesias,
sensory loss, and motor weakness after 6 months of INH therapy if not on vitamin B6 supplementation.
Pyridoxine supplementation needs to accompany INH therapy to avoid this untimely deficiency.

Vitamin A deficiency is rare and can occur with malabsorption syndromes like sprue and biliary atre-
sia. The earliest manifestation is night blindness. Hypervitaminosis A is associated with pseudotumor
cerebri, which manifests as headache and papilledema. The other answer choices are distractors that are
explained elsewhere in this volume.
Neurology
B&D Chapters 57 and 76

243

Psychiatry Test Preparation and Review


Manual E-Book
Psychiatry Test Preparation & Review Manual

Question 99. A. The answer to this question is undoubtedly choice A. Although borderline patients make frequent sui-
cidal gestures, they are extremely impulsive. As such it is the psychiatrist’s job to assume that any gesture
could be potentially life-threatening and to take the threat seriously. Steps must be taken to protect these
patients. Choices B and C are ridiculous and dangerous. Talking about suicide does not increase the risk
that patients will try to harm themselves. Isolating them will offer them less support and increase their
chances of harming themselves. Choice D is inappropriate because these patients cannot be relied on to
keep promises when they are in an impulsive, emotionally labile state. Giving these patients a benzodi-
azepine could potentially disinhibit them further and make it more likely that they would try to harm
themselves.
Management in Psychiatry
K&S Chapter 23

Question 100. B. West Nile virus infection is an arbovirus (a form of flavivirus) infection that is endemic to many parts
of the world. Mosquitoes are frequent vectors of transmission of the virus to humans. Most infections
in humans are asymptomatic. In about 20% of affected patients, the infection presents as a febrile con-
dition after an incubation period of a few days to up to 2 weeks. Only 1 in 150 patients goes on to
develop a meningitis or encephalitis picture. The infection may progress to the development of a demy-
elinating or axonal neuropathy. Diagnosis is made by detection of IgM antibodies in CSF or IgM and
IgG antibodies in serum. Treatment is generally supportive. Patients with serious symptoms may respond
to intravenous administration of anti–West Nile virus immunoglobulin.

Arenaviruses are rodent-borne and infect humans when a person comes into contact with infected
rodent fecal matter. Lymphocytic choriomeningitis virus and Lassa fever virus are two examples of
arenaviruses.

Filoviruses are represented by the Ebola and Marburg viruses. The reservoir of these viruses is not
known. Infection can cause a severe hemorrhagic encephalitis with myositis and muscle pain. Treatment
is supportive. Body fluids of these patients are highly contagious.

Retroviruses are represented by the well-known HIV, HTLV-1, and HTLV-2. The JC virus is a papovavi-
rus and is the causative pathogen of PML, which can develop in patients with advanced AIDS with low
CD4 cell counts. JC virus can be detected in CSF by PCR amplification of its DNA.
Neurology
B&D Chapter 53

Question 101. B. You, being the skillful physician that you are, would order a lithium level on this patient! Why? Because
the side effects of lithium intoxication include gastrointestinal upset such as nausea, vomiting, and diar-
rhea. Lithium can cause tremor, nephrogenic diabetes insipidus, acne, muscular weakness, hypothy-
roidism, weight gain, edema, leukocytosis, psoriasis, hair loss, and cardiac dysrhythmias. When toxic it
can also cause ataxia, slowed thinking, impaired memory, impaired consciousness, seizures, and death.
Though not essential to answer this question, let’s take a minute to review diabetes insipidus. There
are two types of diabetes insipidus, neurogenic and nephrogenic. In neurogenic diabetes insipidus the
hypothalamus is deficient in producing antidiuretic hormone. In nephrogenic diabetes insipidus the
kidneys cannot properly respond to antidiuretic hormone. In diabetes insipidus we see increased urine
output, decreased urine osmolality, and increased serum sodium. Let’s compare this with a case of psy-
chogenic polydipsia, in which urine output is increased, urine osmolality is decreased, and serum sodium
is decreased. The other similar situation that the prudent student should be ready to recognize is SIADH
(syndrome of inappropriate antidiuretic hormone secretion), in which urine output is decreased, urine
osmolality is increased, and serum sodium is decreased. When the diabetes insipidus is a result of lith-
ium, the use of amiloride with a hydrochlorothiazide can decrease polyuria and sometimes allow the
patient to remain on lithium.
Psychopharmacology
K&S Chapter 29

244

Psychiatry Test Preparation and Review


Manual E-Book
Test Number Four

Question 102. E. This question asks the examination-taker to correlate the neuroanatomical location of a certain type of
neurons with the function of the neurotransmitter particular to those neurons. The best answer to this
question is choice E, acetylcholine. The basal forebrain is the location of the nucleus basalis of Meynert,
which is the structure containing a high density of cholinergic neurons. These neurons project to the
limbic system and the cerebral cortex. Alzheimer’s disease is a result of cholinergic neuronal demise pre-
dominantly in the nucleus basalis of Meynert.

Acetylcholine is synthesized from acetyl-coenzyme A and choline by the enzyme choline acetyltransfer-
ase in the synaptic nerve terminal. Acetylcholine is then stored in vesicles in the synaptic bouton. Once
released into the synapse, it is inactivated and metabolized by acetylcholinesterase and the resultant cho-
line is taken back up into the presynaptic terminal for reutilization.

Acetylcholine is responsible for maintaining short-term memory, attention, executive functioning, and
novelty seeking, which are mediated through the nucleus basalis of Meynert. In Alzheimer’s demen-
tia, acetylcholine is depleted, and memory and executive functioning are compromised as a result. The
Alzheimer’s agents donepezil, rivastigmine, and galantamine are all acetylcholinesterase inhibitors and
can increase levels of circulating acetylcholine in the nucleus basalis and throughout the brain, thereby
improving symptoms of dementia to a limited extent. The other neurotransmitters offered in this ques-
tion are distractors. Each one is explained in other questions in this volume.
Basic Neuroscience
K&S Chapter 3

Question 103. E. The TCAs venlafaxine, bupropion, and nefazodone block the reuptake of NE (and serotonin in some
cases) into the presynaptic neuron. This leads to more NE in the synaptic cleft. Mirtazapine works by
blocking presynaptic α2 receptors, which stops feedback inhibition on the release of NE into the synap-
tic cleft. This results in more NE released into the synapse.
Psychopharmacology
K&S Chapter 29

Question 104. D. PCP exerts its hallucinogenic effects by antagonism of NMDA receptors, which in turn prevents the
influx of calcium ions into neurons. PCP also activates ventral tegmental dopamine, which results in the
reinforcing qualities of the drug. Tolerance to the physiologic effects of PCP can occur in humans, but
dependence and physiologic withdrawal do not usually occur. PCP intoxication produces hallucinosis,
dysphoria, and paranoia. Agitation, catatonia, and bizarre behavior are also common. At higher doses,
PCP use can lead to stupor and coma. In overdose, rhabdomyolysis may result from agitation and dys-
autonomia such as fever, hypertension, and sweating. The other answer choices are simply distractors
that need no explanation.
Substance Abuse and Addictive Disorders
K&S Chapter 20

Question 105. B. The three best benzodiazepines for patients with liver dysfunction are temazepam, oxazepam, and loraz-
epam. This is an important little fact for the well-prepared test-taker to know. They have short half-lives
and do not have active metabolites. Other benzodiazepines are less desirable in patients with hepatic
dysfunction.
Psychopharmacology
K&S Chapter 29

Question 106. C. Aspergillus is a fungus that colonizes in the paranasal sinuses and can cause a hypersensitivity pneumo-
nitis. Infection can originate from the lungs in immunocompromised patients. The fungus has a predi-
lection for invading the posterior circulation and can cause vertebrobasilar strokes. The fungus causes
a cerebral vasculitis by invasion of vessel walls. Sinus infection can extend to the brain by contiguous
infiltration. Spinal cord compression can result from pulmonary aspergillosis that extends to the thoracic
vertebrae through the epidural space.

245

Psychiatry Test Preparation and Review


Manual E-Book
Psychiatry Test Preparation & Review Manual

Histoplasma is a fungus that can cause an influenza-like infection with erythematous skin lesions and
liver function abnormalities. In fewer than 20% of cases, there is a development of neurologic manifes-
tations in the form of cerebritis, basilar meningitis, or CNS granuloma. Cerebral abscess is also a pos-
sible neurologic complication of histoplasmosis infection in about 40% of cases. Meningeal symptoms
and signs of headache, fever, and neck stiffness can also be noted in the neurologic form of the infection.

Candida albicans is one of the most common fungal organisms found in the human body. Neurologic
infection is rare, except in immunocompromised hosts. In patients who are immunocompromised, can-
didal infection can manifest in the form of intracranial abscesses, vasculitis, and small-vessel throm-
bosis. Candida can form mycotic intracerebral aneurysms that can rupture and cause parenchymal
hemorrhage.

Pseudallescheria boydii is an uncommon fungal pathogen that can infect immunosuppressed patients.
Clinical presentation is typically that of meningitis or multiple brain abscesses.

Cryptococcus neoformans infection is discussed in detail elsewhere in this volume.


Neurology
B&D Chapter 53

Question 107. C. Adverse effects of fluoxetine that set it apart from other serotonin-selective reuptake inhibitors include
headache, anxiety, and respiratory complaints. Other side effects include nausea, diarrhea, and insomnia.
High blood pressure is found with patients on venlafaxine. Blurred vision can occur from anticholinergic
medications. Shuffling gait can occur as a result of parkinsonian side effects of antipsychotics. Loss of
consciousness occurs with sedatives.
Psychopharmacology
K&S Chapter 29

Question 108. B. The GABA-A receptor (the most predominant GABA receptor) is a chloride channel. Such a useful little
fact for the prudent student to know!
Basic Neuroscience
K&S Chapter 1

Question 109. D. After making an error in psychodynamic psychotherapy, the best way to proceed is to briefly acknowl-
edge that a mistake was made and move on, focusing on the patient and his or her problems.
Interpreting the patient’s reaction can be seen as dismissive. It does not address the fact that the therapist
made a mistake, not the patient. Ignoring the mistake will contaminate the patient’s transference toward
the therapist, potentially making him or her angry. Giving a long but clear explanation puts too much
emphasis on the mistake. The emphasis should be on the patient and his or her behavior, not the thera-
pist and hers. To profusely apologize is the wrong approach, because it is an overreaction to a minor
mistake. The goal is to acknowledge the mistake and move on, spending as little time focusing on the
therapist and her actions and more time on the patient and his or her actions. Mistakes are a normal
part of therapy, because the therapist is human. Dealing with them in a way that maintains boundaries
and therapeutic neutrality will be in the best interest of the patient and the therapy.
Psychotherapy
K&S Chapter 28

Question 110. C. Identity diffusion is the failure to develop a cohesive self or self-awareness. Do not bother looking the
other answer choices up. They are unrelated distractors, some of which are ludicrously unrelated to the
question.
Psychological Theory and Psychometric Testing
K&S Chapter 2

Question 111. C. Treatment of children with separation-anxiety disorder should be multimodal. It should involve indi-
vidual therapy for the child, medication to reduce anxiety, family therapy and education, and return to
school, which is graded if necessary (i.e., start with 1 hour per day, then increase to 2 hours, then to 3

246

Psychiatry Test Preparation and Review


Manual E-Book
Test Number Four

hours, etc.). The parental education should focus on giving the child consistent support but maintaining
clear boundaries about the child’s avoidant behaviors toward anxiety-provoking situations.
Anxiety Disorders
K&S Chapter 31

Question 112. D. Naltrexone is an opioid antagonist that is often used as an adjunctive agent for alcohol abuse, because it
decreases craving and alcohol consumption. It is nowhere near 100% effective, and its success is very much
dependent on the patient’s desire to stop drinking and the success of concurrent behavioral modification. It is
not better than behavioral modification. Naltrexone has nothing to do with dopamine or the GABA receptor.
Psychopharmacology
K&S Chapter 29

Question 113. D. The anticholinergic activity of many psychiatric drugs (including TCAs such as imipramine) can cause
urinary hesitancy, dribbling, and urinary retention. These side effects occur especially in older men who
have enlarged prostates. Treatment usually consists of bethanechol 10 to 50 mg three or four times daily.
Psychopharmacology
K&S Chapter 29

Question 114. B. The most important take-home point from this question is that bipolar I disorder has equal prevalence
for men and women. Major depression is more common in women than in men. There is no correla-
tion between socioeconomic status and frequency of depression. There is a correlation between hyperse-
cretion (not hyposecretion) of cortisol and increased depression. Only about 50% of those with major
depressive disorder receive specific treatment.
Bipolar Disorders
K&S Chapter 8

Question 115. B. A child who is not speaking should first have his or her hearing checked. Phonological disorders are
characterized by a child’s inability to make age-appropriate speech sounds. The child cannot be diag-
nosed if the deficits are being caused by a structural or neurologic problem; therefore these things must
first be ruled out. Phonological disorder may present as substitutions of one sound for another or omis-
sions, such as leaving the final consonant off words. The treatment of choice is speech therapy, and
recovery can be spontaneous in some children. Speech therapy is indicated if the child cannot be under-
stood, if the child is over 8 years of age, when self-image and peer relationships are being affected, when
many consonants are misarticulated, and when the child is frequently omitting parts of words.
Diagnostic and Treatment Procedures in Psychiatry
K&S Chapter 31

Question 116. E. State-dependent learning is the facilitated recall of information in the same internal state or environment in
which the information was originally obtained. An example of this is when someone learns a behavior when
intoxicated with a drug. Without the drug he or she cannot recall the behavior. When the drug is given to him
or her again, he or she remembers the behavior. The other answer choices relate to learning and conditioning.
The most important of them have been covered in their own separate questions. Others are just distractors.
Psychological Theory and Psychometric Testing
K&S Chapter 2

Question 117. C. This is a case of Wernicke’s encephalopathy. Wernicke’s and its partner, Korsakoff’s amnesia, are the result
of thiamine deficiency often found in alcoholics. Wernicke’s is an acute neurologic disorder characterized
by ataxia, confusion, vestibular dysfunction, and eye movement impairment. Wernicke’s encephalopathy
is reversible with treatment, but if it progresses into Korsakoff’s amnesia, damage may be irreversible.
Korsakoff’s presents as impaired recent memory and anterograde amnesia. The treatment for these syn-
dromes is thiamine, first intravenously in the case of an acute Wernicke’s, and then orally for as long as 3
to 12 months in the case of Korsakoff’s amnesia. Although other answer choices are good ideas, the pri-
mary goal is to prevent further brain damage by getting thiamine into the patient immediately.
Substance Abuse and Addictive Disorders
K&S Chapter 20

247

Psychiatry Test Preparation and Review


Manual E-Book
Psychiatry Test Preparation & Review Manual

Question 118. B. Niacin is an essential nutrient also called nicotinic acid. Niacin deficiency, termed pellagra, occurs in
individuals who consume corn as their main carbohydrate staple. Corn lacks niacin and tryptophan
(which can be converted in the body to niacin). Bread is now niacin-fortified, which has diminished the
widespread problem of pellagra in most countries.

Pellagra causes the classic triad of the three D’s: dementia, dermatitis, and diarrhea. Gastrointestinal
problems present as diarrhea, anorexia, and abdominal discomfort. Skin manifestations present as a
hyperkeratotic rash over much of the body. The neurologic manifestations can include depression, mem-
ory impairment, apathy, and irritability. A confusional state may result and may lead to stupor or coma.
Oral doses of nicotinic acid can reverse the symptoms of pellagra.

The triad of neuropathy, retinopathy, and areflexia can result from a deficiency in vitamin E
(α-tocopherol). Other manifestations can include ataxia, loss of proprioception and vibration sensation,
nystagmus, and external ophthalmoplegia.

The triad of neuropathy, ataxia, and dementia can result from vitamin B12 (cobalamin) deficiency.
Symptoms present as paresthesias of the hands and feet, weakness, gait disturbance, depression, confu-
sion, psychosis, peripheral neuropathy, and loss of position and vibration sensation. Myelopathic symp-
toms, such as spastic paraparesis, can occur, as can visual disturbances, manifested by optic atrophy and
visual loss. Treatment is undertaken with parenteral administration of vitamin B12.

Folate deficiency can result in a clinical picture that is similar in presentation to that of B12 deficiency.
Elevated serum homocysteine is a surrogate marker for low folate levels. The deficiency can lead to neu-
ropathy and/or spasticity due to spinal cord involvement. Treatment is initially undertaken with oral
folate. Answer choice A is a nonsense distractor and needs no explanation.
Neurology
B&D Chapter 57

Question 119. C. The best way to address a missed therapy session is to use neutral questioning to help explore why it
happened. Ignoring the missed appointment is a mistake that can be misunderstood by the patient as the
therapist not caring if he shows or not. Getting angry at the patient or punishing the patient is the wrong
approach. It will only make the patient angry and less likely to reveal the emotional reasons for why the
session was missed and what meaning that has for the therapeutic relationship. In therapy, the patient’s
treatment of the therapist is a reflection of how he or she treats others in life as well. As such, aspects of
the patient’s behavior, such as missed appointments, should be noted and explored.
Psychotherapy
K&S Chapter 5

Question 120. D. Tachycardia, not bradycardia, is a symptom of cannabis intoxication. All other answer choices are also
symptoms. Orthostatic hypotension is usually seen only with high doses of cannabis.
Substance Abuse and Addictive Disorders
K&S Chapter 20

Question 121. E. Akathisia is a subjective feeling of muscular tension caused by antipsychotic medication, which can
cause restlessness, pacing, or an inability to stand still. Treatment consists of a β adrenergic receptor
antagonist, such as propranolol. Other choices include anticholinergic medications, such as benztropine.
Benzodiazepines can be useful in some cases. While on the topic of propranolol, keep in mind that beta
blockers may be used in patients with aggression. In particular, there have been randomized placebo-
controlled studies supporting the use of propranolol for aggression in patients who have suffered a trau-
matic brain injury. This little fact has shown up on standardized examinations before, so it is worth
remembering.
Psychopharmacology
K&S Chapter 29

248

Psychiatry Test Preparation and Review


Manual E-Book
Test Number Four

Question 122. C. Clonidine works by agonist activity at presynaptic α2 receptors. This leads to a decrease in the amount
of neurotransmitter released into the synaptic cleft, leading to decreased sympathetic tone and decreased
arousal. In the case of opioid withdrawal, the action of clonidine on the locus ceruleus is thought to
be particularly important to the decrease in autonomic symptoms associated with withdrawal. Other
answer choices are unrelated distractors.
Psychopharmacology
K&S Chapter 29

Question 123. B. Blockade of muscarinic cholinergic receptors is a common side effect of many drugs. Blockade leads to
blurred vision, dry mouth, constipation, and difficulty urinating. When there is excessive blockade of this
receptor, a patient can develop confusion and delirium. Alzheimer’s disease has been postulated to be, in
part, the result of too little cholinergic activity. As such, drugs like donepezil block the enzyme acetylcho-
linesterase (which breaks down acetylcholine) and thereby increase cholinergic activity. This has been
shown to be useful in the treatment of dementia. This is the opposite of blockade of muscarinic choliner-
gic receptors, which, one would postulate, would worsen the symptoms of Alzheimer’s.
Psychopharmacology
K&S Chapter 1

Question 124. A. Although all of the laboratory tests involved in this question can be elevated in alcohol abuse, the most
likely test to pick up alcohol abuse is for GGT. It is elevated in 80% of those with alcohol-related disor-
ders. The other tests are elevated at lower rates than 80% and as such will not pick up as many alcohol
disorders as the GGT test.
Laboratory Tests in Psychiatry
K&S Chapter 20

Question 125. B. Pimozide (Orap) is a dopamine receptor antagonist that has been approved in the United States for the
treatment of Tourette’s disorder. Haloperidol is also widely used for this indication. In Europe, pimozide
is used as an antipsychotic medication for treatment of schizophrenia.
Psychopharmacology
K&S Chapter 29

Question 126. C. The gene for amyloid precursor protein is found on chromosome 21. Amyloid precursor protein is
broken down to form β-amyloid protein, which is a major component of senile plaques in Alzheimer’s
disease.
Neurocognitive Disorders
K&S Chapter 21

Question 127. D. Heinz Kohut developed the school of self psychology. Central to his theories of personality development
is the idea that when parents mirror a child’s behavior, this functions as a form of empathy, which is nec-
essary for personality development and the formation of healthy self-esteem. When this parental empa-
thy is lacking, the sense of self does not develop properly, and personality disorders develop. Patients
then need others to fulfill functions that the self would normally handle. Oedipal conflict is a classic
part of Freud’s theories in which, greatly simplified, a child competes with the parent of the same sex
for the attention of the parent of the opposite sex. The concept of the good enough mother came from
Winnicott. He describes a holding environment that develops in which the good enough mother allows
the child’s true self to develop. He also gave us the concept of the transitional object. The paranoid–
schizoid position and the depressive position are found in the work of Melanie Klein. The paranoid–
schizoid position is a view of the world from the perspective of the infant, in which the whole world
is split into good and bad elements. The depressive position occurs when the infant is able to view the
mother ambivalently as having both positive and negative aspects.
Human Development
K&S Chapter 4

249

Psychiatry Test Preparation and Review


Manual E-Book
Psychiatry Test Preparation & Review Manual

Question 128. D. When erythromycin and carbamazepine are given together, the carbamazepine levels are increased.
Psychopharmacology
K&S Chapter 29

Question 129. D. Valproic acid should be avoided during pregnancy because of its propensity to cause neural tube defects
in the developing fetus. It should not be used by nursing mothers, as it is excreted in breast milk. Should
the continuation of valproic acid in pregnancy be an absolute necessity, the risk of neural tube defects
can be reduced by giving the patient 1 to 4 mg of folic acid per day. However, switching to another
medication is the best choice.
Psychopharmacology
K&S Chapter 29

Question 130. D. Increased appetite, weight gain, and increased sleep make this question a case of atypical depression.
The treatment of choice for atypical depression is the monoamine oxidase inhibitors (MAOIs). As such,
phenelzine is the answer, because it is the only MAOI listed.
Psychopharmacology
K&S Chapter 29

Question 131. C. Venlafaxine carries the potential side effect of increasing blood pressure. For this reason, a baseline
blood pressure should be taken for anyone starting venlafaxine, and regular monitoring of blood pres-
sure is a good idea. Increased blood pressure has been found particularly with doses over 300 mg per
day, and lower doses have shown less hypertension. As such, caution must be used when giving this drug
to anyone with preexisting hypertension.
Psychopharmacology
K&S Chapter 29

Question 132. D. Interpersonal therapy was developed to treat depression. It focuses on interpersonal behavior and social
interaction. Patients are taught to rate their interactions with others and become aware of their own
behavior. The therapist may give direct advice, help make decisions, and clarify conflicts.
Psychotherapy
K&S Chapter 28

Question 133. E. Imprinting is the work of Konrad Lorenz. Imprinting implies that an animal has a critical period during
which it is sensitive to certain stimuli that elicit a specific behavioral response. For example, the baby goose
has a period during which it “imprints” on its mother and learns to follow her wherever she goes. If a per-
son was the first moving object the goose saw, it would imprint on that person and follow that person as
if he or she were the mother. Nikolas Tinbergen conducted experiments both on animal behavior and on
humans. He worked on measuring the power of certain stimuli to elicit specific behaviors from animals.
He studied displacement activities whereby during times when the urges to fight or flee would be equal,
the animal would do some other activity to diffuse the tension. Humans also do this in times of stress.
He described innate release mechanisms whereby animals have a specific response that is triggered by a
releaser. A releaser is an environmental stimulus that prompts the specific response. Tinbergen also worked
with human autistic children. He observed the behavior of both autistic children and normal children and
postulated that certain stimuli that are comforting to a normal child arouse fear in the autistic child. He
postulates that this is part of what leads to the behavioral pattern found in autistic children.
Psychological Theory and Psychometric Testing
K&S Chapter 3

Question 134. E. The hippocampus is one of the most important structures in the formation of memory. Other areas
important to memory include some of the diencephalic nuclei and the basal forebrain. The amygdala
also plays a role by rating the emotional content of memories, thereby leading to stronger recall of more
emotionally charged memories.
Basic Neuroscience
K&S Chapter 1

250

Psychiatry Test Preparation and Review


Manual E-Book
Test Number Four

Question 135. E. This is a difficult question unless you know all of the involved scales. It is easier if you know that the
BPRS is a scale used for schizophrenia and psychosis. This is the take-home point. The other scales listed
are all mood disorder scales. Will the Montgomery–Åsberg scale end up on a standardized test near you?
Probably not. But the BPRS probably will, so remember it. It is a good idea to be familiar with the most
common psychiatric rating scales. We will not print the full scales in this text, but it is worth your time
to be familiar with them.
Psychological Theory and Psychometric Testing
K&S Chapter 5

Question 136. C. Sleep terrors (non-REM sleep arousal disorder) are characterized by recurrent episodes of awakening
and screaming during the first third of the night. The patient has intense fear, autonomic arousal, sweat-
ing, tachycardia, and rapid breathing. The patient is unresponsive to efforts of others to comfort him or
her. There are no dreams recalled, and there is amnesia for the event. Small doses of diazepam are often
useful to stop the episodes.
Sleep–Wake Disorders
K&S Chapter 16

Question 137. C. Thioridazine (Mellaril) is one of the older typical antipsychotics that is not used as frequently since
the advent of the atypicals. It is very sedating, causes orthostatic hypotension, has anticholinergic side
effects, and has low rates of extrapyramidal symptoms. One of its more notable side effects is retrograde
ejaculation, in addition to impotence. Patients on thioridazine can be told that retrograde ejaculation is
not dangerous, but they will produce milky-white urine after orgasm.
Psychopharmacology
K&S Chapter 29

Question 138. C. Donepezil is an acetylcholinesterase inhibitor. All other answer choices are distractors. Donepezil is
used to treat mild to moderate Alzheimer’s disease. By blocking acetylcholinesterase, the drug leads
to increased acetylcholine in the synaptic cleft, which has been proven to slow decline in Alzheimer’s
disease.
Psychopharmacology
K&S Chapter 29

Question 139. B. Different cultures may present with culture-bound psychiatric syndromes, or different aspects of a certain
disorder may be more prevalent in one culture than in another. With regard to depression, Chinese cul-
ture often presents with more somatic complaints and less focus on mood symptoms. Very often Chinese
patients will come to the primary care physician or emergency room with somatic symptoms that are
somewhat nonspecific and are found to be driven by an underlying depressive disorder. Cultures have dif-
ferences in how they view many things, including definitions of what constitutes health and sickness.
Cultural Issues in Psychiatry
K&S Chapter 3

Question 140. A. The amphetamines in general exert their effects through the dopaminergic system. However, ecstasy is
a “designer amphetamine” that acts through both the dopaminergic and the serotonergic systems. Had
dopamine been given as an answer choice, it would also have been acceptable. The other neurotransmit-
ters listed are unrelated to ecstasy.
Substance Abuse and Addictive Disorders
K&S Chapter 20

Question 141. C. The field of child psychiatry developed out of the growth of child guidance centers in the early 1900s.
Other answer choices are just distractors. Choice A happened after the early 1900s. Choice B happened
before the early 1900s. Choice E took place in 1996 and is a bill relating to HIV testing. Choice D is
unrelated to the United States, because Freud lived in Vienna.
History of Psychiatry
K&S Chapter 31

251

Psychiatry Test Preparation and Review


Manual E-Book
Psychiatry Test Preparation & Review Manual

Question 142. A. Sarcoidosis is a granulomatous disease that affects multiple organ systems. Neurologic manifestations
occur in about 5% of sarcoidosis patients. Up to 20% of neurologic manifestations present as a peripheral
neuropathy. Cranial neuropathies, and in particular facial nerve palsy, constitute the most common mani-
festation of neurosarcoidosis, occurring in up to 75% of cases. The treatment of choice is systemic cortico-
steroid or immunosuppressive therapy. The diagnosis is established clinically but can often be confirmed by
muscle biopsy or an elevated level of angiotensin-converting enzyme in the CSF. Other possible neurologic
manifestations include cauda equina syndrome, mononeuropathy multiplex, peripheral sensorimotor poly-
neuropathy, diffuse meningoencephalitis, uveitis, and polyradiculoneuropathy resembling GBS.
Neurology
B&D Chapters 49 and 76

Question 143. D. Trazodone is an antidepressant that is frequently used for insomnia. It is associated with priapism, which
is a prolonged erection in the absence of sexual stimulation. Priapism is a potential medical emergency that
can be treated by intracavernosal injection of epinephrine. Untreated priapism can also lead to impotence.
Patients who start to develop priapism on trazodone should be switched to another medication.
Psychopharmacology
K&S Chapter 29

Question 144. C. Low levels of CSF serotonin are associated with increased aggression. Increased levels of dopamine are
associated with increased aggression.
Basic Neuroscience
K&S Chapter 1

Question 145. C. Mirtazapine is an antidepressant medication that works by antagonism of presynaptic α2 adrenergic recep-
tors leading to potentiation of serotonergic and noradrenergic neurotransmission. Mirtazapine is sedating,
particularly at low doses, which is good for depressed patients with insomnia. It lacks the anticholinergic
side effects of the tricyclics and lacks the anxiogenic side effects of the serotonin-selective reuptake inhibi-
tors. Mirtazapine is also notable for its lack of sexual side effects. Other psychiatric medications notable
for their lack of sexual side effects include bupropion, buspirone, donepezil, galantamine, and nefazodone.
Psychopharmacology
K&S Chapter 29

Question 146. D. Sleep changes characteristic of the elderly include both decrease in REM sleep and decrease in slow-
wave sleep. This is a useful little fact for the test-taker to remember.
Sleep–Wake Disorders
K&S Chapter 16

Question 147. D. In delirium we see a rapid onset of symptoms. The duration is hours to weeks, and attention fluctuates.
Recent and immediate memory are impaired. Speech can be incoherent and may be either speeded up or
slowed down. There is frequent disruption of the sleep–wake cycle. Thoughts can be very disorganized,
and alertness and awareness can be reduced. By comparison, the onset of dementia is often slow, span-
ning months to years. Attention is preserved, and memory impairment usually involves remote memory.
There can be word-finding difficulties, but speech remains coherent. The sleep–wake cycle is fragmented.
Thoughts are impoverished, and awareness and alertness are normal.
Neurocognitive Disorders
K&S Chapter 21

Question 148. C. Kleine–Levin syndrome is a rare condition. (But not so rare on standardized tests!) It is marked by peri-
ods of hypersomnia with periods of normal sleep in between. During the periods of excessive sleep the
patients wake up and experience apathy, irritability, confusion, voracious eating, loss of sexual inhibi-
tions, disorientation, delusions, hallucinations, memory impairment, incoherent speech, excitation, and
depression. The onset of the illness usually hits between 10 and 20 years of age, and it goes away by the
time the patient is in his or her 40s.
Disruptive, Impulse Control, Conduct Disorders, and ADHD
K&S Chapter 15

252

Psychiatry Test Preparation and Review


Manual E-Book
Test Number Four

Question 149. C. David has obsessive–compulsive personality disorder (OCPD). OCPD presents as a pervasive pattern of
preoccupation with orderliness, perfectionism, and control. This preoccupation comes at the expense of
openness, efficiency, and flexibility. The OCPD patient’s perfectionism interferes with task completion.
These patients are inflexible regarding moral and ethical issues. They devote time to work at the expense
of leisure activities. They are reluctant to delegate tasks to others. They are characteristically rigid and
stubborn. OCPD patients often cannot discard old or worn-out objects even when they have no value.
They will not give tasks to others without reassurance that the tasks will be done their way. They are
miserly in spending and view money as something to be hoarded for catastrophes. The most common
defense mechanisms of OCPD are isolation of affect, undoing, reaction formation, intellectualization,
and rationalization. David is not presenting with prominent anxiety symptoms, so generalized anxiety
disorder is incorrect. He does not have obsessive thoughts and compulsions to stop those thoughts, so
OCD is incorrect. If David had schizoid personality disorder we would see that he had no friends or
close contacts and that this does not bother him. In this question David has few friends because he
spends all of his time working. If David had avoidant personality disorder he would have a pattern of
social inhibition, feelings of inadequacy, and hypersensitivity to negative evaluation. That pattern is not
described in this question.
Personality Disorders
K&S Chapter 22

Question 150. E. Cognitive behavioral therapy is founded on the principle that people make assumptions that affect
their thoughts. Their thoughts then affect their mood. The goal of the therapy is therefore to uncover
assumptions and thoughts that may be both faulty and automatic and determine how they contribute
to changes in mood. Then the therapy aims to correct these faulty thoughts and stop them from being
automatic.

The other answer choices are all appropriate pieces of psychodynamic therapy and psychoanalysis. They
are not, however, part of cognitive behavioral therapy.
Psychotherapy
K&S Chapter 28

253

Psychiatry Test Preparation and Review


Manual E-Book
Psychiatry Test Preparation & Review Manual

FIVE
Test Number Five
1. Blocking the H1 receptor will lead to which of the following?

A. 
Weight gain
B. 
Dry mouth
C. 
Orthostatic hypotension
D. 
Elevated prolactin
E. 
Urinary retention

2. Needle electromyographic studies of muscles of patients with myasthenia gravis would be expected to reveal:

A. 
Delayed F-responses
B. 
Slow nerve conduction velocities
C. 
Decrementing response to repetitive stimulation
D. 
Incrementing response to repetitive stimulation
E. 
Increased motor latencies

3. While working in the emergency room you evaluate a patient with confusion, myoclonus, diarrhea, hypotension, tachy-
cardia, and a normal creatine phosphokinase (CPK) level. The patient is not speaking. His family tells you that he was
recently started on a new medication by his psychiatrist for treatment of bipolar disorder. They don’t know the name of
the medication or what kind of medication it was. Using your astute clinical skills and significant psychiatric knowledge
you narrow the diagnosis down to either neuroleptic malignant syndrome (NMS) or serotonin syndrome. But alas, you
must pick one diagnosis for the medical record. Which one will it be and why?

A. 
NMS because the patient is confused
B. 
NMS because rigidity is more commonly part of serotonin syndrome
C. 
Serotonin syndrome because it commonly presents with hypotension
D. 
NMS because it usually presents with normal CPK
E. 
Serotonin syndrome because it more commonly presents with myoclonus and gastrointestinal (GI) symptoms

Full test - and additional VIDEO vignettes - available online - see inside front cover for details.

254

Psychiatry Test Preparation and Review


Manual E-Book
Test Number Five

4. Which of the following magnetic resonance imaging (MRI) anomalies is most closely associated with patients diagnosed
with Huntington’s disease?

A. 
Caudate head atrophy
B. 
Cerebellar atrophy
C. 
Generalized cortical atrophy
D. 
Putamen atrophy
E. 
Communicating hydrocephalus

5. Which of the following must be present to meet Diagnostic and Statistical Manual of Mental Disorders 5 (DSM 5) crite-
ria for somatic symptom disorder?

A. 
Preoccupation with acquiring a serious illness
B. 
Two sexual symptoms
C. 
Three pseudoneurological symptoms
D. 
Maladaptive thoughts, feelings, or behaviors in response to the symptoms
E. 
The illness must begin after age 30

6. A 65-year-old woman is brought to the emergency room from a nursing home. She is noted to have cognitive impair-
ment, delusions, masked facies, shuffling gait, cogwheel rigidity of the extremities, visual hallucinations, and agitation.
She is medicated by the emergency room physician with 5 mg of intramuscular haloperidol but experiences acute wors-
ening of her muscular rigidity. The most likely diagnosis is:

A. 
Frontotemporal dementia
B. 
Dementia with Lewy bodies
C. 
Alzheimer’s dementia
D. 
Normal-pressure hydrocephalus
E. 
Vascular dementia

7. Which one of the following is not an inhibitor of cytochrome P450 (CYP) 1A2?

A. 
Amiodarone
B. 
Tobacco
C. 
Cimetidine
D. 
Fluvoxamine
E. 
Grapefruit juice

8. Brain metastasis is most commonly due to which of the following?

A. 
Lung carcinoma
B. 
Breast carcinoma
C. 
Melanoma
D. 
Prostate carcinoma
E. 
Colorectal carcinoma

255

Psychiatry Test Preparation and Review


Manual E-Book
Psychiatry Test Preparation & Review Manual

9. The metabolite of which one of the following tricyclic antidepressants has potent dopamine-blocking ability that can
lead to antipsychotic-like side effects?

A. 
Amoxapine
B. 
Clomipramine
C. 
Desipramine
D. 
Trimipramine
E. 
Imipramine

10. 
The computed tomography (CT) scan of the brain shown here depicts
which one of the following pathologies?

A. 
Epidural hematoma
B. 
Left middle cerebral artery infarction
C. 
Subdural hematoma
D. 
Sagittal sinus thrombosis
E. 
Uncal herniation

11. When considering schizophreniform disorder and brief psychotic disorder, which one of the following statements is
false?

A. 
A clear stressor is not needed for the diagnosis of brief psychotic disorder
B. 
In schizophreniform disorder the patient returns to baseline functioning after the episode resolves
C. 
Both conditions can present with hallucinations
D. 
Schizophreniform disorder is ruled out when there is a medical condition present that could be causing psychotic
symptoms
E. 
The duration for brief psychotic disorder is at least 1 month but less than 6 months

12. Which one of the following does not pertain to Parkinson’s disease?

A. 
Resting 4- to 6-Hz tremor
B. 
Considered a synucleinopathy
C. 
Loss of pigment in the pars compacta of the substantia nigra
D. 
Increase in carbohydrate-deficient transferrin (CDT)
E. 
Impaired dopamine synthesis in the nigrostriatal tract

256

Psychiatry Test Preparation and Review


Manual E-Book
Test Number Five

13. You meet a new patient who describes episodes of distractibility, racing thoughts, increased goal-directed activity, and
elevated mood. These episodes last for 4 days, and she is having one currently. She also describes past episodes lasting 3
to 4 weeks in which her mood is depressed and she experiences loss of appetite, fatigue, poor concentration, and suicidal
thoughts. Which of the following is the most accurate diagnosis?

A. 
Bipolar I disorder
B. 
Bipolar II disorder
C. 
Methylenedioxymethamphetamine (MDMA; ecstasy) intoxication
D. 
Schizoaffective disorder bipolar type
E. 
Cyclothymic disorder

14. Which one of the following does not pertain to Wilson’s disease?

A. 
Dementia
B. 
Intention tremor, 4 to 6 Hz
C. 
Kayser–Fleischer corneal rings
D. 
Chronic hepatic dysfunction, cirrhosis
E. 
Early onset, ages 15 to 45 years

15. Which one of the following statements is false concerning development during the school-age years (ages 5 to 12)?

A. 
Children develop the ability to empathize with others during these years
B. 
Children have a basic grasp of grammar and syntax and can understand word play
C. 
Children learn social cues, rules, and expectations
D. 
Children become motivated by desire for approval and positive feedback
E. 
Children have developed the capacity for abstract thinking

16. Which one of the following movement disorders is not associated with dementia?

A. 
Parkinson’s disease
B. 
Wilson’s disease
C. 
Diffuse Lewy body disease
D. 
Huntington’s disease
E. 
Primary torsion dystonia (dystonia musculorum deformans)

17. What percentage of the general population has a diagnosable personality disorder?

A. 
5%
B. 
10%
C. 
25%
D. 
50%
E. 
70%

18. Which one of the following central nervous system (CNS) infections is most likely to produce ring-enhancing lesions on
brain neuroimaging studies?

A. 
Tuberculosis
B. 
CNS human immunodeficiency virus (HIV)-related lymphoma
C. 
Cryptococcal meningitis
D. 
Progressive multifocal leukoencephalopathy
E. 
Acquired immunodeficiency syndrome (AIDS)-related dementia complex (ARDC)

257

Psychiatry Test Preparation and Review


Manual E-Book
Psychiatry Test Preparation & Review Manual

19. Which one of the following would not correlate with the brain imaging finding of thin gyri, wide sulci, and enlarged
ventricles (especially the third ventricle)?

A. 
Alzheimer’s disease
B. 
Chronic alcoholism or drug abuse
C. 
A lacunar infarct of the subcortical white matter
D. 
Schizophrenia
E. 
Trisomy 21 (Down’s syndrome)

20. A left-sided middle cerebral artery territory stroke is most likely to produce which of the following?

A. 
Impaired naming and repetition
B. 
Preserved language functioning
C. 
Right hemiparesis with leg weakness worse than the face and arm
D. 
Receptive, fluent aphasia
E. 
Speech paraphasias

21. Dr. Smith, a psychiatrist, has been having sexual fantasies about one of his psychotherapy patients. He tells you this and
gives you several reasons the prohibition of sex with patients does not apply to this situation. Which of the following
would be the most ethical advice you could give him?

A. 
He should continue to discuss the situation with colleagues to better define his sexual boundaries
B. 
He should see the patient more frequently to use the strong feelings between them as a motivational tool for posi-
tive change in therapy
C. 
He should share his fantasies with the patient
D. 
He should transfer the patient’s care to another psychiatrist
E. 
He should stock his desk with condoms so that if they have sex in his office she does not become pregnant

22. Alexia without agraphia is attributable to a stroke in which of the following arterial territories?

A. 
Middle cerebral
B. 
Posterior cerebral
C. 
Posterior communicating
D. 
Vertebral
E. 
Basilar

23. You are studying two variables, a binary predictor variable and a continuous outcome variable. You want to know if the
relationship between those two variables is due to chance alone. Which of the following tests would you use?

A. 
Analysis of variance (ANOVA)
χ2 test
B. 
C. 
T test
D. 
Negative predictive power
E. 
Predictive validity

258

Psychiatry Test Preparation and Review


Manual E-Book
Test Number Five

24. Which of the following childhood disorders is accompanied by pectus excavatum, ocular lens dislocation, Marfan’s hab-
itus, mental retardation, and scoliosis?

A. 
Homocystinuria
B. 
Phenylketonuria
C. 
Tay–Sachs disease
D. 
Niemann–Pick disease
E. 
Metachromatic leukodystrophy

25. Which one of the following neural imaging findings has been associated with posttraumatic stress disorder?

A. 
Increased volume in the anterior cingulate cortex
B. 
Increased dopaminergic neurotransmission
C. 
Increased size of the lateral ventricles
D. 
Decreased hippocampal volume
E. 
Decreased metabolic activity in the prefrontal cortex

26. Niemann–Pick disease is due to a deficiency in which one of the following?

A. 
Arylsulfatase-A
B. 
Sphingomyelinase
C. 
Hexosaminidase-A
D. 
Hypoxanthine–guanine phosphoribosyltransferase (HPRT)
E. 
Phenylalanine hydroxylase

27. A patient in treatment for bipolar disorder reports severe mid-upper abdominal pain radiating to the back, nausea,
anorexia, fever, and evidence of acute abdominal pain. Pain is worse after eating and upon lying down. She is taking
citalopram and valproic acid. Which of the following tests would be most useful in making a diagnosis?

A. 
Complete blood count
B. 
Basic metabolic panel
C. 
Liver function tests
D. 
Amylase
E. 
Prolactin level

28. The most common inherited cause of infantile intellectual disability is:

A. 
Arylsulfatase-A deficiency
B. 
Hexosaminidase-A deficiency
C. 
Trisomy 21
D. 
Sphingomyelinase deficiency
E. 
Trinucleotide repeat disorder causing long face, long ears, and macro-orchidism

29. Which one of the following is not a feature of trisomy 21?

A. 
Single palmar crease
B. 
Short stature
C. 
Brushfield spots on the iris
D. 
Normal penis with small, firm testes and sterility
E. 
Increased risk of dementia

259

Psychiatry Test Preparation and Review


Manual E-Book
Psychiatry Test Preparation & Review Manual

30. A child is brought to the clinic with the following characteristics: elfin facies, short stature, hypoplastic teeth, mild men-
tal retardation, friendly personality, and gifted musically. This disorder is due to:

A. 
Trisomy 21
B. 
Autosomal microdeletion in chromosome 7q
C. 
Deletion in chromosome 15q
D. 
Deletion 45XO
E. 
47 XXY

31. Where should electrodes be placed in electroconvulsive therapy (ECT) to minimize cognitive impairment?

A. 
Anterior bilateral placement
B. 
Right unilateral placement
C. 
Left unilateral placement
D. 
Posterior bilateral placement
E. 
Midline placement

32. Which one of the following is not a neurologic cause of autistic symptoms?

A. 
Rett’s syndrome
B. 
Fragile X syndrome
C. 
Marfan’s syndrome
D. 
Angelman’s syndrome
E. 
Tuberous sclerosis

33. A patient suffering from depression is referred for ECT. Before ECT, which of the following medications should be
discontinued?

A. 
Lithium
B. 
Olanzapine
C. 
Imipramine
D. 
Thioridazine
E. 
Selegiline

34. Which one of the following is not a feature of tuberous sclerosis?

A. 
Cutaneous and conjunctival telangiectasias
B. 
Mental retardation
C. 
Seizures
D. 
Shagreen patches
E. 
Ash leaf spots

35. Which one of the following is considered a mature defense mechanism?

A. 
Sublimation
B. 
Displacement
C. 
Repression
D. 
Hypochondriasis
E. 
Introjection

260

Psychiatry Test Preparation and Review


Manual E-Book
Test Number Five

36. Hemianesthesia after a stroke with gradual return of sensory function and significant pain is due to a lesion of:

A. 
The corona radiata
B. 
The spinothalamic tract
C. 
The spinoreticular tract
D. 
The thalamus
E. 
The internal capsule

37. Robert is arrested for assault and held in prison. Because of disorganized behavior, a psychiatric evaluation is conducted
by a forensic psychiatrist to determine Robert’s fitness to stand trial for the charges. Robert reports that 1 month earlier
he was evaluated in a local emergency room for hearing voices. What is the difference between the evaluation done by
the forensic psychiatrist and the evaluation done by the emergency room (ER) psychiatrist?

A. 
Only the ER evaluation requires a full medication history
B. 
Only the forensic evaluation contains details on Robert’s memory
C. 
The forensic psychiatrist does not have a doctor–patient relationship with Robert
D. 
The ER evaluation will be the only one to contain recommendations
E. 
Only the ER evaluation contains details on Robert’s memory

38. Periodic sharp-wave complexes seen on electroencephalography (EEG) are characteristic of which of the following?

A. 
Alzheimer’s disease
B. 
Parkinson’s disease
C. 
ARDC
D. 
Creutzfeldt–Jakob disease
E. 
Delirium

39. The symptoms of this disorder include dementia, incontinence, and gait apraxia:

A. 
Normal-pressure hydrocephalus
B. 
Diffuse Lewy body disease
C. 
Alzheimer’s disease
D. 
Progressive supranuclear palsy
E. 
Wernicke–Korsakoff syndrome

40. An intracranial bleed that causes brief loss of consciousness, followed by an initial lucid period and then further deterio-
ration thereafter, is a result of damage to:

A. 
Bridging veins
B. 
Middle meningeal artery
C. 
Aneurysm of the posterior communicating artery
D. 
Reticular activating system
E. 
Frontal lobes

261

Psychiatry Test Preparation and Review


Manual E-Book
Psychiatry Test Preparation & Review Manual

41. A psychiatrist and a psychologist are sharing care for the same patient. In addition, the patient is also seeing a nurse in
their clinic. There is a negative outcome in the case, and the patient decides to sue for malpractice. Who is ultimately
held responsible?

A. 
The psychologist
B. 
The psychiatrist
C. 
The organization that owns the clinic
D. 
The nurse
E. 
The patient

42. An HIV-positive patient develops incoordination, lack of attention and motivation, and memory loss that are gradual in
onset. Cerebrospinal fluid (CSF) is normal on lumbar puncture. MRI of the brain reveals confluent white matter changes
on T2 sequencing, no mass effect, and no gadolinium enhancement. The most likely diagnosis is:

A. 
Progressive multifocal leukoencephalopathy
B. 
CNS lymphoma
C. 
CNS toxoplasmosis
D. 
Cryptococcal meningitis
E. 
HIV-related dementia

43. Which one of the following is not a substrate for CYP 2D6?

A. 
Aripiprazole
B. 
Bupropion
C. 
Codeine
D. 
Duloxetine
E. 
Haloperidol

44. 
Which of the following disorders characteristically presents with festinating gait, postural instability, and truncal
rigidity?

A. 
Alcoholic cerebellar atrophy
B. 
Alzheimer-type dementia
C. 
Normal-pressure hydrocephalus
D. 
Parkinson’s disease
E. 
Tropical human T-lymphotropic virus type-1 (HTLV-1) paraparesis

45. Billy and Susan are out at a bar. They spend the evening drinking heavily. They each drink 10 beers. As the evening
comes to an end, they both vomit and pass out on the floor. An ambulance is called, and they are brought into the ER.
Susan is found to have a higher blood alcohol level than Billy. Why is this so?

A. 
Susan drank more
B. 
Susan has a higher body water content than Billy
C. 
Billy metabolizes more alcohol in his gastric mucosa
D. 
Susan urinates less frequently than Billy
E. 
Susan is morbidly obese

262

Psychiatry Test Preparation and Review


Manual E-Book
Test Number Five

46. A 70-year-old patient presents with severe left hemicranial headache of acute onset with left eye visual loss. He also
presents with aches, pains, and stiffness in the extremities for several weeks. MRI of the brain reveals T2 periventricular
white matter hyperintensities. Serum sedimentation rate is 110 mm/h. Which of the following is the next most important
step in management?

A. 
Obtain a temporal artery biopsy
B. 
Begin treatment with high-dose prednisone
C. 
Begin treatment with broad-spectrum intravenous antibiotics
D. 
Obtain a magnetic resonance arteriogram
E. 
Begin treatment with intravenous acyclovir

47. Which of the following conditions is not a potential effect of low folate levels?

A. 
Fatigue
B. 
Panic attacks
C. 
Agitation
D. 
Delirium
E. 
Dementia

48. Which of the following agents is most appropriate for acute abortive therapy of migraine headache?

A. 
Naratriptan
B. 
Divalproex sodium
C. 
Topiramate
D. 
Desipramine
E. 
Gabapentin

49. A depressed patient comes to you with brittle hair, weight gain, muscle weakness, dry skin, constipation, and cold intol-
erance. Which is the most appropriate next step in management?

A. 
Start sertraline
B. 
Refer the patient for a head CT
C. 
Check thyroid-stimulating hormone (TSH) level
D. 
Check serum glucose
E. 
Prescribe sertraline plus Colace

50. A patient reports a history of sudden onset of severe headache, vomiting, collapse, and preservation of consciousness.
On examination there are no lateralizing neurologic signs, but there is some neck stiffness noted. This clinical picture is
most characteristic of:

A. 
Epidural hemorrhage
B. 
Subdural hemorrhage
C. 
Acute venous sinus thrombosis
D. 
Intracerebral hemorrhage
E. 
Subarachnoid hemorrhage

263

Psychiatry Test Preparation and Review


Manual E-Book
Psychiatry Test Preparation & Review Manual

51. Internuclear ophthalmoplegia is a characteristic finding in patients with which of the following?

A. 
Amyotrophic lateral sclerosis
B. 
Myasthenia gravis
C. 
Multiple sclerosis
D. 
Huntington’s chorea
E. 
Progressive supranuclear palsy

52. Salt cravings, fatigue, anorexia, hyperpigmentation, low serum cortisol, and hypotension are characteristic symptoms of:

A. 
Cushing’s disease
B. 
Diabetes mellitus
C. 
Chronic fatigue syndrome
D. 
Mercury poisoning
E. 
Addison’s disease

53. Which of the following is not one of the DSM signs of cannabis intoxication?

A. 
Conjunctival injection
B. 
Diuresis
C. 
Increased appetite
D. 
Dry mouth
E. 
Tachycardia

54. A 30-year-old patient presents with depression and personality changes over several years. His family reports ongo-
ing irritability, impulsivity, and odd social behaviors. On examination, you note mild hyperreflexia, mild bradykinesia,
increased eye blinking, and tongue impersistence. You note he is very fidgety. His father died of dementia in his 50s. The
most likely diagnosis in this case is:

A. 
Early-onset Alzheimer’s disease
B. 
Wilson’s disease
C. 
Huntington’s disease
D. 
Corticobasal degeneration
E. 
Frontotemporal dementia

55. Caution should be exercised when prescribing which one of the following to a patient on carbamazepine because the
combination will increase carbamazepine levels?

A. 
Cyclosporine
B. 
Doxycycline
C. 
Erythromycin
D. 
Phenobarbital
E. 
Theophylline

56. Which of the following disorders is known to be precipitated by chiropractic adjustments?

A. 
Lumbosacral subluxation
B. 
Brachial plexitis
C. 
Vertebral artery dissection
D. 
Sacroiliac dislocation
E. 
Anterior spinal artery occlusion

264

Psychiatry Test Preparation and Review


Manual E-Book
Test Number Five

57. A patient presents with acute onset of right facial paresis involving the forehead and lower face equally. The paresis was
preceded by right ear and mastoid pain and loud noises and low-pitch sounds ipsilaterally. This presentation is most
likely to be:

A. 
Bell’s palsy
B. 
Trigeminal neuralgia
C. 
Facial dystonia
D. 
Acute lateral medullary infarct
E. 
Left-sided Horner’s syndrome

58. The pharmacologic treatment of choice for postherpetic neuralgia is:

A. 
Topiramate
B. 
Duloxetine
C. 
Carbamazepine
D. 
Desipramine
E. 
Gabapentin

59. In which brain area can characteristic lesions be seen that are associated with coma due to carbon monoxide poisoning?

A. 
Cingulate gyrus
B. 
Globus pallidus
C. 
Corona radiata
D. 
Corpus callosum
E. 
Thalamus

60. Persistent limb rigidity and spasms, also known as stiff-person syndrome, is often related to:

A. 
High titers of JC virus antigen
B. 
High titers of HTLV-1 virus antigen
C. 
Circulating lupus anticoagulant
D. 
Autoantibodies against glutamic acid decarboxylase
E. 
Autoantibodies against nicotinic receptors

61. Why does mirtazapine decrease nausea?

A. 
5-HT 2 antagonism
B. 
5-HT 3 antagonism
C. 
5-HT 2 agonism
α2 Antagonism
D. 
α2 Agonism
E. 

265

Psychiatry Test Preparation and Review


Manual E-Book
Psychiatry Test Preparation & Review Manual

62. A 6-year-old girl presents with 6 months of headache, vomiting, diplopia,


gait unsteadiness, and falls. T1-weighted sagittal brain MRI reveals the scan
shown here. The most likely diagnosis is:

A. 
Glioblastoma
B. 
Neuroblastoma
C. 
Arachnoid cyst
D. 
Medulloblastoma
E. 
Hemangioblastoma

63. Which one of the following happens in 50% of families after the death of a child?

A. 
One of the remaining children commits suicide
B. 
One parent dies
C. 
Neglect of the remaining children
D. 
Parental divorce
E. 
Physical abuse of surviving children

64. Which area of the brain, when damaged minimally, can induce unconsciousness?

A. 
Limbic system
B. 
Reticular formation
C. 
Internal capsule
D. 
Thalamus
E. 
Cerebral cortex

65. Which of the following is most likely to be found on neuroimaging of schizophrenics?

A. 
Increased metabolic activity in the frontal lobes
B. 
Increased size of the lateral ventricles
C. 
Increased cerebral asymmetry
D. 
Increased size of the fourth ventricle
E. 
Decreased size of the thalamus

66. A patient suffers an injurious fall down a flight of stairs. Neurologic examination reveals right biceps weakness, absent
right biceps reflex, and right shoulder and arm pain with pain radiating to the right thumb. The likely diagnosis is:

A. 
C-6 radiculopathy
B. 
Biceps tendon tear
C. 
Radial neuropathy
D. 
Ulnar neuropathy
E. 
Cervical myelopathy

266

Psychiatry Test Preparation and Review


Manual E-Book
Test Number Five

67. Which of the following is not a common symptom of narcolepsy?

A. 
Sleep attacks
B. 
Sleep-onset rapid eye movement (REM) period
C. 
Catalepsy
D. 
Sleep paralysis
E. 
Cataplexy

68. A deficiency of which vitamin is associated with subacute combined degeneration of the posterior columns of the spinal cord?

A. 
Vitamin E
B. 
Vitamin D
C. 
Vitamin B12
D. 
Vitamin B6
E. 
Vitamin A

69. Which of the following neurologic signs would be more indicative of vascular dementia than of a dementia of the
Alzheimer type?

A. 
Anomic aphasia
B. 
Constructional apraxia
C. 
Anosognosia
D. 
Extensor plantar responses
E. 
Short-term memory loss

70. 
A 75-year-old man displays paranoid ideation and suspiciousness. He is
unable to dress himself or manage his activities of daily living, such as cook-
ing or cleaning. He does not have memory loss or difficulty in identifying
people or objects, driving a car, or using a telephone or a pen. Head CT scan
reveals the image shown here. The likely diagnosis is:

A. 
Pick’s disease
B. 
Late-onset schizophrenia
C. 
Lewy body dementia
D. 
Alzheimer’s dementia
E. 
Medication-induced psychosis with parkinsonism

71. Meredith is a 27-year-old school teacher who has bipolar disorder. She has been doing well on lamotrigine for the past
few years. She begins a new relationship, and her new boyfriend tells her to stop taking her medication because he “does
not believe in pills.” She stops the lamotrigine and becomes hypomanic. Which of the following would be the best next
step to take in this situation?

A. 
Refer her for group therapy
B. 
Involuntary hospitalization
C. 
Start cognitive behavioral therapy
D. 
Psychoeducation for both patient and boyfriend
E. 
Vocational training

267

Psychiatry Test Preparation and Review


Manual E-Book
Psychiatry Test Preparation & Review Manual

72. Which is the most reliable CSF finding in patients in the chronic progressive phase of multiple sclerosis?

A. 
Myelin basic protein concentration
B. 
Total protein concentration
C. 
Presence of oligoclonal bands
D. 
Acellularity of the CSF
E. 
Immunoglobulin G concentration

73. Mirtazapine works via action on which of the following receptors?

A. 
Dopamine type 2 (D2)
α2
B. 
C. 
Glutamate
α1
D. 
γ-Aminobutyric acid (GABA)
E. 

74. The most useful treatment for acute cluster headaches in men is:

A. 
Indomethacin
B. 
Oxygen therapy with a triptan medication
C. 
Immediate-release oxycodone with acetaminophen (Percocet)
D. 
Butalbital and acetaminophen (Fioricet)
E. 
Methysergide

75. Which of the following personality traits most increases the risk of cardiac disease in men?

A. 
Neuroticism
B. 
Impulsivity
C. 
Hostility
D. 
Recklessness
E. 
Sadness

76. Which of the following combinations of risk factors is most important to control when considering the prevention of
lacunar strokes?

A. 
Hypertension and hyperlipidemia
B. 
Hypertension and obesity
C. 
Hyperlipidemia and obesity
D. 
Hypertension and tobacco smoking behavior
E. 
Hyperlipidemia and diabetes

268

Psychiatry Test Preparation and Review


Manual E-Book
Test Number Five

77. A 1-day-old neonate is found to have the MRI of the brain shown here.
The most likely diagnosis is:

A. 
Arnold–Chiari type I malformation
B. 
Arnold–Chiari type II malformation
C. 
Dandy–Walker malformation
D. 
Arachnoid cyst
E. 
Anencephaly

78. A 5-year-old boy is evaluated and found to have developmental delay, aggressivity, temper tantrums, and overeating
behavior. The boy had hypotonia as an infant. The most likely diagnosis is:

A. 
Prader–Willi syndrome
B. 
Fragile X syndrome
C. 
Trisomy 21
D. 
Williams’ syndrome
E. 
Angelman’s syndrome

79. Which of the following syndromes is characterized by a rapidly progressing dementia with exaggerated startle response
and violent unprovoked myoclonus?

A. 
Progressive multifocal leukoencephalopathy
B. 
Subacute sclerosing panencephalitis
C. 
Spongiform encephalopathy
D. 
Cytomegalovirus encephalitis
E. 
Herpes encephalitis

80. Which one of the following complications of HIV disease is caused by a CNS infection by JC virus?

A. 
CNS lymphoma
B. 
CNS toxoplasmosis
C. 
Cryptococcal meningitis
D. 
Progressive multifocal leukoencephalopathy
E. 
Neurocysticercosis

81. 
Which of the following is the most common complication of idiopathic intracranial hypertension (pseudotumor
cerebri)?

A. 
Exophthalmos
B. 
Visual impairment
C. 
Gait disorder
D. 
Uncal herniation
E. 
Hypovitaminosis A

269

Psychiatry Test Preparation and Review


Manual E-Book
Psychiatry Test Preparation & Review Manual

82. Which of the following is achieved during Piaget’s developmental stage of concrete operations?

A. 
Conservation
B. 
Animistic thinking
C. 
Deductive reasoning
D. 
Object permanence
E. 
Egocentricity

83. Male sexual behavior in the seventh decade is characterized by which of the following?

A. 
Decreased time to ejaculation
B. 
Decreased time to erection
C. 
No change in sex drive
D. 
No change in penile turgidity
E. 
Absence of ejaculation upon orgasm

84. Infantile secure attachment is associated with which of the following outcomes?

A. 
Higher intellectual ability in adolescence and adulthood
B. 
Educational success throughout childhood and adolescence
C. 
Physical coordination and later athletic ability
D. 
Emotional and social competence
E. 
Easy temperament in childhood

85. A young girl performs in a school play and revels in the praise and pride displayed to her by her mother. In Kohut’s
theory of self-psychology, this child is experiencing:

A. 
Idealization
B. 
Good enough mothering
C. 
Mastery
D. 
Primary narcissism
E. 
Mirroring

86. According to Margaret Mahler’s stages of separation–individuation, the period from 10 to 18 months of age, during
which the infant’s ability to move autonomously increases its exploration of the outer world, is known as:

A. 
Object constancy
B. 
Rapprochement
C. 
Practicing
D. 
Differentiation
E. 
Symbiosis

87. Which of the following is an example of declarative memory?

A. 
Priming
B. 
Retention and recall of facts
C. 
Skills and habits
D. 
Simple classical conditioning
E. 
Nonassociative learning

270

Psychiatry Test Preparation and Review


Manual E-Book
Test Number Five

88. The “addictive” nature of gambling behavior and the disorder of pathological gambling can best be explained by theo-
ries of:

A. 
Operant conditioning
B. 
Classical conditioning
C. 
Drive and neurotic impulses
D. 
Self psychology and pathological narcissism
E. 
Habituation and sensitization

89. A patient with depression tells the therapist about a variety of unpleasant personal circumstances that have occurred in
the past week, worries about friends being sad and withdrawn, and a reluctance to watch the news on television because
“the only news on television is bad news.” In the theory of cognitive psychology, this type of behavior is best formulated
as:

A. 
Learned helplessness
B. 
Selective attention bias
C. 
Catastrophizing
D. 
Automatic negative thoughts
E. 
Information overload

90. A clinical test that detects 96% of patients with a certain disease but also produces many false positives in patients with-
out the disease is deemed to have:

A. 
High sensitivity and high specificity
B. 
Low sensitivity and high specificity
C. 
Low sensitivity and low specificity
D. 
High sensitivity and low specificity
E. 
Low overall clinical utility

91. Which of the following classes of psychotropic medications has the best documented treatment effects for agitation in
the demented patient?

A. 
Antipsychotics
B. 
Mood stabilizers
C. 
Benzodiazepines
D. 
Selective serotonin reuptake inhibitor (SSRI) antidepressants
E. 
Tricyclic antidepressants

92. A television news program reports that a famous actress has been seen in public exhibiting verbal outbursts and unex-
plained violent behavior directed at people and property. A well-known psychiatrist appears on the program and, when
asked about the actress, states she is probably suffering from bipolar disorder and possibly substance abuse. Which of
the following best qualifies the ethics of the psychiatrist’s on-air affirmations?

A. 
It is ethical because the actress is a public figure
B. 
It is ethical unless there is a conflict of interest involved
C. 
It is unethical because the psychiatrist has never examined the actress
D. 
It is unethical because the psychiatrist was paid to appear on-air and make such statements
E. 
It is ethical because the psychiatrist is acting under the constitutional rights of freedom of speech and freedom of
the press

271

Psychiatry Test Preparation and Review


Manual E-Book
Psychiatry Test Preparation & Review Manual

93. Transcranial magnetic stimulation was cleared by the FDA in 2008 for in-office treatment of major depressive disorder.
The area of the brain that is treated with electromagnetic stimulus application is first mapped by finding the motor
threshold at the area on the motor strip corresponding to the thumb and fingers. After the discovery of the best area of
motor threshold by this mapping, the ongoing treatment is applied to approximately which of the following brain areas?

A. 
Left cingulate gyrus
B. 
Left parietal lobe
C. 
Left dorsolateral prefrontal cortex
D. 
Right parietal lobe
E. 
Right dorsolateral prefrontal cortex

94. Transcranial magnetic stimulation (NeuroStar TMS Therapy) was cleared by the FDA in 2008 for in-office treatment of
major depressive disorder. Which one of the following correctly classifies the type of patient history for which this treat-
ment is indicated?

A. 
For patients who failed to achieve satisfactory benefit from one antidepressant medication at an adequate dose and
duration in the current episode
B. 
For patients who failed to achieve satisfactory benefit from two antidepressant medications at an adequate dose and
duration in the current episode
C. 
For patients who failed to achieve satisfactory benefit from three antidepressant medications at an adequate dose
and duration in the current episode
D. 
For patients who failed to achieve satisfactory benefit from at least one antidepressant medication with at least one
augmenting agent (second antidepressant or antipsychotic agent) at adequate doses and duration in the current
episode
E. 
For patients who failed to achieve satisfactory benefit from at least one antidepressant agent at an adequate dose
and duration in the current episode or who could not tolerate the side effects of antidepressant agents or who did
not wish to take antidepressant agents

95. A patient comes to you 3 weeks after witnessing a child hit by a car. Since then she has been having feelings of detach-
ment and feels “in a daze.” She has recurrent thoughts and dreams about the event. She describes depersonalization.
She has avoided discussing the event with friends and family, and her sleep has been poor. What is the most appropriate
diagnosis?

A. 
Posttraumatic stress disorder
B. 
Major depressive disorder
C. 
Acute stress disorder
D. 
Panic disorder
E. 
Primary insomnia

96. Which one of the following properties of atypical antipsychotics is hypothesized to be responsible for these medications
being less likely to produce extrapyramidal side effects?

A. 
Decreased binding to D2 receptors
B. 
Rapid dissociation from D2 receptors
C. 
Affinity for muscarinic cholinergic receptors
D. 
Binding to multiple dopamine receptors other than D2 receptors
E. 
Higher affinity for D1 receptors

272

Psychiatry Test Preparation and Review


Manual E-Book
Test Number Five

97. Which of the following choices is a risk factor for developing conversion disorder (functional neurological symptom
disorder)?

A. 
High socioeconomic status
B. 
Little education
C. 
High intelligence
D. 
Urban setting
E. 
Lack of military service

98. 
Neurotoxicity due to MDMA is associated with deficits in neurons that produce which of the following
neurotransmitters?

A. 
Dopamine
B. 
Norepinephrine
C. 
Acetylcholine
D. 
Serotonin
E. 
Glutamate

99. A high score on the Minnesota Multiphasic Personality Inventory (MMPI) infrequency scale (scale F) is most consistent
with which one of the following diagnoses?

A. 
Major depressive disorder
B. 
Histrionic personality disorder
C. 
Malingering
D. 
Obsessive–compulsive disorder
E. 
Hypochondriasis

100. When triiodothyronine is added to a tricyclic antidepressant for augmentation, the clinician needs to be cautious if:

A. 
The patient is a woman over 50 years of age
B. 
The patient has subclinical hypothyroidism
C. 
The patient has a history of hypertension
D. 
The patient has a history of cognitive deficits with depression
E. 
The patient relapsed while on a previously effective tricyclic antidepressant

101. One of your patients has been taking zolpidem for sleep recently and started having sleep-walking episodes on the medi-
cation. She finds these episodes to be disturbing. She asks you about taking eszopiclone instead. Which of the following
possible responses to her question is most accurate?

A. 
Eszopiclone is a bad choice because she will develop tolerance over time
B. 
Eszopiclone can also cause sleep walking
C. 
Eszopiclone works as a melatonin receptor agonist
D. 
Eszopiclone is used to treat both sleep and depression
E. 
Unlike with zolpidem, hallucinations do not occur with eszopiclone

273

Psychiatry Test Preparation and Review


Manual E-Book
Psychiatry Test Preparation & Review Manual

102. As part of doing blood work for an alcoholic patient you decide to order a GGT (γ-glutamyl transpeptidase). How long
after the patient stopped drinking would you expect the GGT to return to normal?

A. 
2 days
B. 
3 weeks
C. 
10 days
D. 
8 weeks
E. 
5 days

103. A schizophrenic patient asks his doctor to bill his insurance company as if he had an adjustment disorder instead of
schizophrenia because he is concerned about the stigma that schizophrenics face. What should the doctor do?

A. 
Agree to bill for an adjustment disorder because the stigma is real
B. 
Kick the patient out of his or her practice and refuse to see him again
C. 
Explore the reasons behind the request and explain why this is something the doctor is not comfortable doing
D. 
Suggest the doctor bills the visit as psychotic disorder NOS (not otherwise specified) because, though it is not accu-
rate, it is closer to the truth than adjustment disorder
E. 
Agree to bill as an adjustment disorder as long as the patient spends time exploring his feelings surrounding the
request and his feelings about the diagnosis

104. Which of the following is not one of the psychiatric symptoms that can result from adrenal insufficiency?

A. 
Delirium
B. 
Psychosis
C. 
Mania
D. 
Depression
E. 
Irritability

105. Which of the following is true about administering different mood stabilizers together?

A. 
Valproic acid will increase lamotrigine levels
B. 
Lamotrigine will increase valproic acid levels
C. 
Valproic acid will decrease lamotrigine levels
D. 
Valproic acid will decrease carbamazepine levels
E. 
Carbamazepine will increase lithium levels

106. Which of the following neurochemical changes would you expect to see in a patient with increased aggression?

A. 
Increased serotonin
B. 
Decreased dopamine
C. 
Decreased testosterone
D. 
Decreased GABA
E. 
Decreased acetylcholine

107. Which one of the following statements is false concerning motor development during infancy?

A. 
The cerebellum is not fully formed until 1 year of age
B. 
Myelination of peripheral nerves is not complete until 2 years of age
C. 
The grasp and tonic neck reflexes begin to develop between 2 and 6 months
D. 
Fine pincer grasp develops around 9 to 12 months
E. 
The ability to sit without support occurs at 6 months

274

Psychiatry Test Preparation and Review


Manual E-Book
Test Number Five

108. Which of the following is a risk factor for neuroleptic-induced tardive dyskinesia?

A. 
Anxiety disorder
B. 
Migraine headaches
C. 
Lower socioeconomic class
D. 
Advanced age
E. 
Male gender

109. Which of the following statements about benzodiazepines is correct?

A. 
Clonazepam has a rapid rate of absorption
B. 
Alprazolam has a half-life of 2.5 hours
C. 
Diazepam has a half-life of 200 hours
D. 
Lorazepam is considered a long-acting benzodiazepine
E. 
Clonazepam is considered a short-acting benzodiazepine

110. Which of the following anxiety disorders has equal rates in both males and females?

A. 
Panic disorder
B. 
Generalized anxiety disorder
C. 
Social phobia
D. 
Obsessive–compulsive disorder
E. 
Specific phobia

111. 
Which of the following is part of the correct schedule for monitoring white blood cell (WBC) count when using
clozapine?

A. 
Weekly WBC for the first 6 weeks and every other week thereafter
B. 
Daily WBC for the first week and weekly for the next 6 months
C. 
Weekly WBC for the first 6 months and monthly thereafter
D. 
Daily WBC for the first week after discontinuation of clozapine
E. 
Weekly for the first 6 months, then biweekly for 6 months, and monthly thereafter

112. The following choices are potential side effects of lithium except:

A. 
Hair loss
B. 
Tremor
C. 
Confusion
D. 
Neural tube defects
E. 
Seizures

113. Which of the following is most correct regarding management of alcohol withdrawal symptoms?

A. 
Intravenous thiamine and dextrose should be given for the first 9 days
B. 
If alprazolam is used, it should be given twice a day
C. 
Disulfiram should be given during withdrawal
D. 
One should choose clonazepam over lorazepam in a hepatically impaired patient
E. 
If lorazepam is used, it should be given four times a day

275

Psychiatry Test Preparation and Review


Manual E-Book
Psychiatry Test Preparation & Review Manual

114. Which of the following terms refers to spastic contractions of discrete muscle groups such as the neck, tongue, eyes, or
back?

A. 
Akathisia
B. 
Tardive dyskinesia
C. 
Acute dystonia
D. 
Blepharospasm
E. 
Tardive dystonia

115. Which one of the following is not an inducer of CYP 3A4?

A. 
Carbamazepine
B. 
Oxcarbazepine
C. 
Phenytoin
D. 
Fluoxetine
E. 
Rifampin

116. Suzie goes to the ER with a pain in her right leg. Before initiating treatment, the physician obtains informed consent.
This is an example of which of the following ethical principles?

A. 
Competence
B. 
Justice
C. 
Autonomy
D. 
Parens patriae
E. 
Nonmaleficence

117. The average number of ECT treatments needed to treat depression is:

A. 
1 to 3
B. 
6 to 12
C. 
15 to 20
D. 
25 to 30
E. 
35 to 45

118. Which of the following is not included in the SCID (Structured Clinical Interview for DSM)?

A. 
Mood episodes
B. 
Anxiety disorders
C. 
Functional impairment
D. 
Eating disorders
E. 
Mood disorders differential

119. Which of the following is true regarding self-disclosure in psychotherapy?

A. 
It is always prohibited
B. 
It is acceptable if it helps the therapist feel more comfortable
C. 
It is acceptable for therapists to share their real feelings as long as they don’t lie
D. 
It is acceptable if it is done solely for the benefit of the patient
E. 
It is never prohibited

276

Psychiatry Test Preparation and Review


Manual E-Book
Test Number Five

120. A 67-year-old male carries a diagnosis of bipolar disorder and is being treated with carbamazepine. His family brings
him to the ER because he has become confused and disoriented over the past 2 days. He is afebrile and has no neuro-
logic deficits. Although you could reasonably order any of the following tests to evaluate his condition, which of the fol-
lowing is most important that you check given the patient’s history?

A. 
Head CT
B. 
Chest x-ray
C. 
Urinalysis
D. 
Basic metabolic panel
E. 
TSH

121. 
Which of the following medications would be the best choice for a patient with psychosis and impaired hepatic
function?

A. 
Olanzapine
B. 
Quetiapine
C. 
Paliperidone
D. 
Ziprasidone
E. 
Risperidone

122. A general consensus among experienced clinicians and researchers is known as:

A. 
Face validity
B. 
Descriptive validity
C. 
Predictive validity
D. 
Construct validity
E. 
Positive predictive power

123. Which one of the following statements is false concerning desvenlafaxine extended-release tablets (Pristiq)?

A. 
Desvenlafaxine may cause orthostasis and hyponatremia in the elderly
B. 
Dosage should be adjusted for patients with renal disease
C. 
Monitoring must be conducted for increase in suicidality during treatment, especially in children and teens
D. 
Taking desvenlafaxine in combination with a monoamine oxidase inhibitor (MAOI) may precipitate a serotonin
syndrome
E. 
Desvenlafaxine has no effect on blood pressure, unlike venlafaxine

124. Which of the following is not consistent with mild to moderate lithium intoxication?

A. 
Dizziness
B. 
Dry mouth
C. 
Vomiting
D. 
Muscle fasciculations
E. 
Lithium level 1.5 to 2.0

277

Psychiatry Test Preparation and Review


Manual E-Book
Psychiatry Test Preparation & Review Manual

125. Which of the following changes can be seen in the sleep of elderly human subjects?

A. 
Gradual phase retardation
B. 
Increased REM sleep
C. 
Decreased N1 sleep
D. 
Increased sleep efficiency
E. 
Decreased N3 sleep

126. Which one of the following statements is false concerning atomoxetine (Strattera)?

A. 
Atomoxetine works as a norepinephrine reuptake inhibitor
B. 
Full results are seen within 2 weeks of initiating treatment with atomoxetine
C. 
Once-daily dosing of atomoxetine works well for most patients
D. 
Most common side effects include dizziness and reduced appetite
E. 
Atomoxetine carries a black box warning for suicidality

127. Which of the following famous therapists suggested that empathic failures in the mother lead to developmental arrest in
the child at a stage when the child needs others to help perform self-object functions?

A. 
Melanie Klein
B. 
Heinz Kohut
C. 
Jacques Lacan
D. 
Adolph Meyer
E. 
B.F. Skinner

128. 
Naltrexone extended-release injectable suspension (Vivitrol) is indicated for treatment of which of the following
disorders?

A. 
Major depressive disorder
B. 
Generalized anxiety disorder
C. 
Psychosis
D. 
Alcohol dependence
E. 
Cocaine withdrawal

129. Which of the following rating scales is used to evaluate severity of depression?

A. 
CAPS
B. 
HAM-D
C. 
PANSS
D. 
BPRS
E. 
CAGE

130. Which of the following medications is a serotonin and norepinephrine reuptake inhibitor, lacks significant antihistaminic
effects, can cause withdrawal symptoms if stopped abruptly, and has an indication for neuropathic pain?

A. 
Mirtazapine
B. 
Quetiapine
C. 
Nefazodone
D. 
Paroxetine
E. 
Duloxetine

278

Psychiatry Test Preparation and Review


Manual E-Book
Test Number Five

131. Which one of the following foods can be consumed when taking an MAOI?

A. 
Aged cheese
B. 
Cured meats
C. 
Smoked fish
D. 
Vodka
E. 
Caviar

132. A 19-year-old male is brought into the ER after his family noticed an acute change in his behavior. He is aggressive,
assaultive, and belligerent. On examination, it is noted that he has nystagmus, slurred speech, a perioral rash, and a
tremor. His family reports that he became out of control while in the garage and kicked a hole through the garage door.
He has no prior psychiatric history. What is his most likely diagnosis?

A. 
Bipolar disorder
B. 
Psychotic disorder NOS
C. 
Panic disorder
D. 
Inhalant intoxication
E. 
Antisocial personality disorder

133. Which one of the following is not a clinical feature of narcolepsy?

A. 
Cataplexy
B. 
Increased REM latency
C. 
Hypnagogic hallucinations
D. 
Associated with human leukocyte antigen (HLA)-DR2 in most cases
E. 
Sleep paralysis

134. A diagnosis of intellectual disability (intellectual development disorder) may not be based on which one of the following?

A. 
Deficits in reasoning
B. 
Deficits in adaptive functioning
C. 
Deficits in social-emotional reciprocity
D. 
Onset during childhood development
E. 
Deficits in learning from experience

135. Which of the following statements is correct concerning clozapine?

A. 
Smoking will not affect clozapine levels
B. 
Risk of agranulocytosis is dose-related
C. 
Clozapine carries a high risk of prolactin elevation
D. 
Doses of clozapine greater than 600 mg/day carry a much higher risk of seizures than lower doses
E. 
Adding lithium to clozapine lowers the chances of NMS

136. Which one of the following is false concerning delusional disorder?

A. 
Nonbizarre delusions may be present
B. 
The patient meets criterion A for schizophrenia
C. 
There is a preservation of functioning
D. 
“Erotomanic” is a subtype
E. 
“Somatic” is a subtype

279

Psychiatry Test Preparation and Review


Manual E-Book
Psychiatry Test Preparation & Review Manual

137. Which of the following is the most significant risk factor for posttraumatic stress disorder and the best predictor of
symptom development after exposure to trauma?

A. 
A family history of anxiety or depression
B. 
The availability of psychiatric care immediately after the trauma
C. 
Low socioeconomic status
D. 
Being single, divorced, or widowed
E. 
The nature, severity, and duration of exposure to the trauma

138. Which of the following correctly describes the likelihood of patients with schizophrenia committing homicide compared
with the general population at large?

A. 
Half as likely
B. 
Equally likely
C. 
Twice as likely
D. 
Five times as likely
E. 
Ten times as likely

139. A new patient comes to you asking for help with depression. She states that she has had a very low mood recently and
has been feeling overwhelmed. She has a 2-month-old daughter at home, and keeping up with a new baby has been hard
for her. She reports constant fatigue, listlessness, and weight loss. She states that her libido is down, and she is very upset
because she could not breastfeed her baby owing to lack of milk production. Her concentration is normal. She bursts
into tears and states that she doesn’t think she can be a good mother. Which of the following would be the most appro-
priate next step in the management of this case?

A. 
Start psychotherapy only
B. 
Start an antidepressant
C. 
Start a benzodiazepine
D. 
Send patient for endocrine blood work
E. 
Report the case to child protective services

140. Which of the following choices about avoidant personality disorder is most accurate?

A. 
It is often misdiagnosed as paranoid personality disorder
B. 
These patients tend to be indifferent to praise or criticism
C. 
Affective instability is a key component of the disorder
D. 
These patients avoid social interaction out of fear of shame or ridicule
E. 
These patients are unable to discard worthless objects

141. Which one of the following is not a possible psychiatric manifestation of acute intermittent porphyria?

A. 
Anxiety
B. 
Dementia
C. 
Depression
D. 
Psychosis
E. 
Delirium

280

Psychiatry Test Preparation and Review


Manual E-Book
Test Number Five

142. The prevalence of Alzheimer’s disease in those patients over age 85 years is:

A. 
Less than 5%
B. 
5% to 10%
C. 
11% to 20%
D. 
21% to 40%
E. 
41% to 50%

143. Which of the following statements concerning psychiatrists and suicide is most accurate?

A. 
The law imposes liability on the psychiatrist whenever a patient commits suicide
B. 
A psychiatrist is expected to foresee all possible harm that may come to a patient
C. 
The law views suicide as predictable in all cases
D. 
No Harm contracts are dependable legal protection for the psychiatrist
E. 
Psychiatrists must document risk assessment and appropriate interventions to meet the standard of care when treat-
ing a suicidal patient

144. Which of the following is the correct criterion for a covered disability, according to the Americans with Disabilities Act
(ADA)?

A. 
It is due to a substance-related disorder
B. 
It is due to a general medical condition
C. 
It seriously affects social functioning at work
D. 
It substantially limits one or more major life activities
E. 
It causes significant impairment in the workplace

145. Which one of the following is false concerning the homeless mentally ill?

A. 
The numbers of homeless mentally ill continue to grow
B. 
A significant number of homeless mentally ill are also dependent on alcohol or other substances of abuse
C. 
Street dwellers usually have schizophrenia, substance abuse, or both
D. 
Episodically homeless have personality, substance abuse, or mood disorders
E. 
Traditional mental health systems are often very effective at treating the homeless once they are enrolled in
treatment

146. Which one of the following situations is considered ethical behavior for a psychiatrist?

A. 
Having sex with a current patient
B. 
Noting romantic feelings between therapist and patient, then discontinuing therapy, and then beginning a sexual
relationship
C. 
Transferring the case to another doctor, then waiting for 1 year, and then beginning a sexual relationship with the
former patient
D. 
Taking on a new patient with whom you had been sexually involved several years ago but are not currently
E. 
Never having sex with a patient or ex-patient, ever, under any circumstances

281

Psychiatry Test Preparation and Review


Manual E-Book
Psychiatry Test Preparation & Review Manual

147. Which of the following medications is a partial nicotine agonist that decreases craving and withdrawal in patients trying
to stop smoking?

A. 
Bupropion
B. 
Buspirone
C. 
Nicotine gum
D. 
Nicotine nasal spray
E. 
Varenicline

148. The treatment of choice for organophosphate insecticide exposure and poisoning is:

A. 
Pralidoxime and atropine
B. 
Flumazenil
C. 
Intravenous fluids and ventilation
D. 
Naloxone
E. 
Dimercaprol or penicillamine

149. Which one of the following is false regarding HIPAA?

A. 
Authorization must be obtained for release of health information except for routine uses, such as provider payment
or health care operations
B. 
Patients must have access to written notice of their privacy rights
C. 
Patients have a right to know how their protected information is kept and disclosed
D. 
Patients have a right to copies of their medical record
E. 
Patients may control who receives their information but not how the information is communicated

150. One of your patients asks you for information about mirtazapine. If you made all of the following statements to the
patient, which one would be considered factually inaccurate?

A. 
Mirtazapine blocks serotonin reuptake
B. 
Mirtazapine causes minimal sexual dysfunction
C. 
Mirtazapine can be very sedating
D. 
Mirtazapine is a potent H1 receptor antagonist
E. 
Mirtazapine can cause weight gain

282

Psychiatry Test Preparation and Review


Manual E-Book
Test Number Five

Answer Key – Test Number Five


Five
1. A 26. B 51. C 76. D 101. B 126. B
2. C 27. D 52. E 77. C 102. D 127. B
3. E 28. E 53. B 78. A 103. C 128. D
4. A 29. D 54. C 79. C 104. C 129. B
5. D 30. B 55. C 80. D 105. A 130. E
6. B 31. B 56. C 81. B 106. D 131. D
7. B 32. C 57. A 82. A 107. C 132. D
8. A 33. A 58. E 83. C 108. D 133. B
9. A 34. A 59. B 84. D 109. A 134. C
10. C 35. A 60. D 85. E 110. D 135. D
11. E 36. D 61. B 86. C 111. E 136. B
12. D 37. C 62. D 87. B 112. D 137. E
13. B 38. D 63. D 88. A 113. E 138. B
14. B 39. A 64. B 89. B 114. C 139. D
15. E 40. B 65. B 90. D 115. D 140. D
16. E 41. B 66. A 91. A 116. C 141. B
17. B 42. E 67. C 92. C 117. B 142. D
18. A 43. B 68. C 93. C 118. C 143. E
19. C 44. D 69. D 94. A 119. D 144. D
20. A 45. C 70. A 95. C 120. D 145. E
21. D 46. B 71. D 96. B 121. C 146. E
22. B 47. B 72. C 97. B 122. A 147. E
23. C 48. A 73. B 98. D 123. E 148. A
24. A 49. C 74. B 99. C 124. D 149. E
25. D 50. E 75. C 100. C 125. E 150. A

283

Psychiatry Test Preparation and Review


Manual E-Book
Psychiatry Test Preparation and Review Manual

Five
Explanations – Test Number Five
Question 1. A. Blocking the H1 receptor leads to weight gain and sedation. You should also be familiar with the effects
of blocking some other receptors. Blocking acetylcholine receptors leads to dry mouth, constipation,
blurry vision, urinary retention, and cognitive dysfunction. Blocking α1 adrenergic receptors leads to
orthostatic hypotension and drowsiness. Blocking dopamine receptors can lead to extrapyramidal syn-
drome (EPS) and elevated prolactin.
Basic Neuroscience
K&S Chapter 1

Question 2. C. The diagnostic evaluation of myasthenia gravis (MG) is multifaceted. Timed endurance tasks, such
as prolonged upgaze or holding outstretched arms in abduction, are useful evidence of fatigability.
Weakness improvement upon edrophonium chloride (Tensilon) injection is a useful sign. The detection
of acetylcholine receptor antibodies (AChR-ab) in serum is another useful corroborative test. The ice
test is useful for ptosis. An ice pack is applied to the affected upper eyelid for 5 minutes. A positive test
is the improvement of ptosis by > 2 mm or more. This transient improvement in ptosis is due to the cold
decreasing the acetylcholinesterase breakdown of acetylcholine at the neuromuscular junction.

Repetitive nerve stimulation by needle electromyography (EMG) may demonstrate a decrementing


response: a reduction of the compound muscle action potential, potentiated after 1 minute of exercise,
consistent with a polysynaptic transmission defect. This decremental response on EMG creates a charac-
teristic “dive bomber” sound. The sensitivity of repetitive stimulation is about 75%. The demonstration
of increased “jitter” by single-fiber EMG is the most sensitive test for MG. The sensitivity ranges from
80% in ocular myasthenia to close to 100% in moderate generalized MG. Using AChR-ab, repetitive
stimulation, and single-fiber EMG, identify all MG cases.
Neurology
B&D Chapter 109

Question 3. E. This question may be a bit odd, but it covers an important point nonetheless. It is asking you to com-
pare NMS and serotonin syndrome. This can be a very tricky task, as they both present with many of
the same symptoms. Both present with mental status changes, autonomic instability, diaphoresis, and
mutism. Both can have elevated CPK levels, but high CPK is more common in NMS because of the
muscular rigidity. To make the decision between the two, the key point is that serotonin syndrome pres-
ents with myoclonus, hyperreflexia, and GI symptoms, whereas NMS presents with muscle rigidity. Now
some may argue that serotonin syndrome can present with rigidity also…and they’re right. But when we
take the whole clinical picture into account, the myoclonus, GI symptoms, and normal CPK would push
the decision in the direction of serotonin syndrome over NMS. In the real world, it is hoped that the
patient’s family members or doctor could tell you what the new medication was, which would greatly
influence your diagnosis.
Psychopharmacology
K&S Chapter 29

284

Psychiatry Test Preparation and Review


Manual E-Book
Test Number Five

Question 4. A. Classically, MRI studies of the brains of patients with Huntington’s disease (HD) reveal caudate and
cerebral atrophy. In addition to taking a fastidious family history, blood should be drawn and DNA test-
ing conducted for the Huntington’s genetic anomalies. HD is caused by a mutation on chromosome 4
causing excessive CAG repeats. If there are more than 36 repeats, we get Huntington’s disease; if there
are more than 60 repeats, we get juvenile Huntington’s. These repeats are unstable and tend to increase
in successive generations, leading to “anticipation,” in which symptoms occur at earlier ages in succes-
sive generations. These CAG repeats cause intracellular accumulation of the huntingtin protein, which
leads to excitotoxicity and cell death.

The atrophy of the caudate alters the appearance and configuration of the frontal horns of the lateral
ventricles (“bat-wing” ventricles). Also, in general the ventricles are diffusely enlarged. Further discus-
sion of HD can be found in other questions in this volume. Bonus fact: Other illnesses with trinucleotide
repeats include myotoic dystrophy and fragile X.
Neurology
B&D Chapter 71

Question 5. D. For somatic symptom disorder you need one or more somatic symptoms that are distressing or result in
significant disruption of daily life. The patient has excessive thoughts, feelings, or behaviors related to
the symptoms as manifested by disproportionate thoughts about the seriousness of the symptoms, high
level of anxiety about the symptoms, or excessive time and energy devoted to the symptoms. The symp-
toms and excessive thoughts should be present for at least 6 months. There is no DSM criterion for age
of onset; therefore, answer choice E is incorrect. Symptoms may present over a period of several years.
The symptoms either cannot be explained by a medical condition or are out of proportion to the symp-
toms/impairment one would expect given the medical condition. The symptoms are not intentionally
produced or feigned. More women get the illness than men, and it often presents as a dramatic patient
who is very vague about the details of the symptoms. Answer choice A, preoccupation with acquiring a
serious illness, is more consistent with illness anxiety disorder.
Somatic Symptom Disorders
K&S Chapter 13

Question 6. B. The core features of dementia with Lewy bodies (DLB) include visual hallucinations, fluctuating cog-
nition, and parkinsonism. Two features are required to diagnose clinically probable DLB, and one is
required to diagnose clinically possible DLB. REM sleep behavior disorder and neuroleptic sensitivity,
as well as low uptake in basal ganglia on fluorodopa positron-emission tomography (PET) and single-
photon emission CT scans, are now considered “suggestive” clinical criteria. In general, the clinical pic-
ture of mild parkinsonism with more severe dementia and marked visual hallucinations and paranoid
delusions should trigger consideration of the diagnosis of DLB. Keep in mind that patients with DLB are
particularly sensitive to EPS when treated with antipsychotics.

Parkinsonian features of DLB may respond well to l-DOPA therapy, but usually only temporarily and
sometimes with the undesirable side effect of causing agitated delirium or hallucinations. The best treat-
ment for psychosis in LBD and Parkinson’s is clozapine, due to its low potential to cause EPS. Some
patients with DLB also suffer from orthostatic hypotension. There is no definitive diagnostic test for
DLB, although PET scans sometimes reveal reduced activity in the posterior parietal cortical regions.
Neurocognitive Disorders
B&D Chapter 95

Question 7. B. All of the choices listed are inhibitors of CYP 1A2 except tobacco, which is an inducer. Other inducers
of 1A2 include charbroiled meats and cruciferous vegetables.
Psychopharmacology
K&S Chapter 1

Question 8. A. Intracranial metastatic carcinoma is more frequently seen than primary brain tumors. Most carcinomas
metastasize to the brain via hematogenous spread. About one-third of brain metastases originate in the
lung, and about half that number originate in the breast. The third most frequent source is melanoma,

285

Psychiatry Test Preparation and Review


Manual E-Book
Psychiatry Test Preparation & Review Manual

followed by the GI tract, in particular the colon and the rectum. This is followed in frequency by cancers
of the kidney. The remainder are accounted for by cancers of the thyroid, liver, gallbladder, testicle, pan-
creas, uterus, and ovaries.
Neurology
B&D Chapter 52F

Question 9. A. Amoxapine has a 7-hydroxy metabolite that has potent dopamine-blocking activity. This can lead to
antipsychotic-like side effects that result from the drug’s use.
Psychopharmacology
K&S Chapter 29

Question 10. C. Most subdural hematomas occur over the convexity of the
cerebral hemispheres. Bleeding into the subdural space is usu-
ally a result of tearing of one or more of the bridging veins
that cross the space to reach the venous sinuses. In 50% to
80% of patients there is a history of head trauma. It is more
common in alcoholics, the elderly, and in patients on warfa-
rin or with coagulopathies. Headache due to subdural hema-
toma can be caused by chronic expansion of the lesion. Other
symptoms that can be caused by chronic subdural hema-
toma include cognitive deficits, personality changes, subacute
dementia, dizziness, and excessive sleepiness. Headache is the
single most common symptom.

The photo attached to this question most likely represents a


subacute subdural hematoma. An acute bleed would initially
be hyperdense but gradually become more isodense after
a period of 1 or more weeks. The fluid then becomes more
hypodense (with respect to the cortex) over a period of 2 to 6
weeks.
Neurology
B&D Chapter 69

Question 11. E. This question compares schizophreniform disorder with brief psychotic disorder to see if you know
some of the key differences. Both disorders present with schizophrenia-like symptoms (i.e., criteria A
symptoms). In both disorders the patient returns to baseline functioning after the disturbance resolves.
In brief psychotic disorder there is often a stressor that precedes the episode, but there does not need to
be one. In making the diagnosis the DSM allows us to describe the episode as “with marked stressor,”
“without marked stressor,” or “with postpartum onset.” The key point to keep in mind is the time differ-
ence. Brief psychotic disorder lasts from 1 day to 1 month. Schizophreniform disorder lasts from 1 to 6
months. After 6 months the psychosis can be called schizophrenia (assuming schizophrenia criteria are
met). Either diagnosis is ruled out if there is a medical condition present that could be causing the symp-
toms. In that case, one would diagnose psychotic disorder due to another medical condition.
Psychotic Disorders
K&S Chapter 7

Question 12. D. The core features of Parkinson’s disease (PD) are resting tremor, rigidity, postural instability, and hypo-
kinesia/bradykinesia. The classic parkinsonian tremor is seen in the initial presentation of about 70% of
patients with the disease. The four- to six-per-second characteristic “pill-rolling” tremor of thumb and
fingers is seen in about half of the patients with PD. The tremor is typically seen when the hands are still
(hence the term resting tremor).

The most replicable finding in idiopathic PD is loss of pigmented cells in the substantia nigra and other
pigmented dopaminergic tracts. Recall that there must be a loss of at least 80% of dopaminergic cell
bodies before the clinical symptoms and signs of PD begin to manifest. Mutations in specific genes have
been conclusively shown to cause PD. These genes code for α-synuclein, parkin, and leucine-rich repeat

286

Psychiatry Test Preparation and Review


Manual E-Book
Test Number Five

kinase 2. Recent studies also suggest a role for the GBA gene coding for glucocerebrosidase. In most
cases, people with these mutations will develop PD. In terms of pathophysiology, PD is considered a
synucleinopathy because of an abnormal accumulation of α-synuclein protein in the brain in the form
of Lewy bodies. A classic Lewy body is an eosinophilic cytoplasmic inclusion that consists of a dense
core surrounded by a halo of radiating fibrils, the primary structural component of which is α-synuclein.
These are found pervasively in the substantia nigra of patients with PD. CDT is a test used to detect
heavy levels of alcohol consumption. It begins to increase after heavy alcohol use of about four to seven
drinks per day for 1 week or more. It is not a good screening test but is useful for monitoring for absti-
nence or increased alcohol consumption. It has nothing to do with PD and is simply a distractor in this
question. While on the topic of Parkinson’s, please keep in mind that evidence supports the use of ECT
in cases of treatment for refractory Parkinson’s disease.
Neurology
B&D Chapter 71

Question 13. B. The answer to this question is bipolar II disorder because the patient meets criteria for hypomania as
well as major depressive episodes. The important learning point here is to distinguish mania from hypo-
mania. To qualify for mania, one needs symptoms for at least a week. If the core symptom is euphoria,
three additional symptoms are needed. If the core symptom is irritability, four additional symptoms are
needed. Additional symptoms can include grandiosity, decreased need for sleep, pressured speech, flight
of ideas, distractibility, increased goal-directed activity, or excessive involvement in pleasurable activities.
One must demonstrate impairment in social or occupational functioning, need for hospitalization, or
presence of psychosis as part of the picture of mania.

Hypomania lasts for at least 4 days. The same criteria for symptoms apply, but the patient does not
experience disturbance in social or occupational functioning, require hospitalization, or become psy-
chotic as part of the symptom picture.

To qualify for cyclothymic disorder a patient must have 2 years with periods of hypomania and depres-
sive symptoms that do not meet criteria for major depressive disorder (MDD). MDMA (ecstasy) intox-
ication does not present with the picture described. Ecstasy is a serotonin reuptake inhibitor. At low
doses it causes feelings of closeness and empathy. At high doses it causes anxiety and paranoia. Other
symptoms that point toward ecstasy use would be bruxism and hyperthermia. When hyperthermia
induced by ecstasy is combined with increased physical activity, it can lead to death. Schizoaffective dis-
order is ruled out by the absence of any psychotic symptoms in the patient’s description. Bipolar I disor-
der is incorrect because the patient does not meet criteria for manic episodes.
Bipolar Disorders
K&S Chapter 8

Question 14. B. Wilson’s disease (hepatolenticular degeneration) is a rare disorder. It is an autosomal recessive disorder
of copper metabolism localizing to mutations on chromosome 13, which lead to mitochondria dysfunc-
tion. Many patients present in childhood with signs and symptoms of liver dysfunction that span the
spectrum from cirrhosis to fulminant liver failure. Neurologic signs and symptoms can be seen in up
to half of patients with Wilson’s disease. These CNS manifestations can include parkinsonism, postural
and kinetic tremor, ataxia, titubation, chorea, seizures, dysarthria, and dystonia. The tremor in Wilson’s
is often described as “wing-beating,” because the patients flap their arms in a manner that mimics bird
wings. Dementia may be present and is most often mild. Psychiatric manifestations can also be seen and
include mood and personality disorders, behavioral changes, and psychosis. When neurologic manifes-
tations are seen, ophthalmologic examination almost inevitably demonstrates the presence of Kayser–
Fleischer rings due to copper deposition. Treatment is classically accomplished by copper chelation with
d-penicillamine. Note that answer B is correct because the classic 4- to 6-Hz intention tremor is gener-
ally associated with idiopathic PD and not Wilson’s disease.
Neurology
B&D Chapter 71

287

Psychiatry Test Preparation and Review


Manual E-Book
Psychiatry Test Preparation & Review Manual

Question 15. E. The school-age years (ages 5 to 12) include many important milestones. All of those listed are correct
with the exception of choice E. The ability to think abstractly usually develops during adolescence and
coincides with the stage that Piaget coined “formal operations.” In this stage the person can manipulate
data and emotions in the environment in a constructive manner using his or her own experience and a
hypothetical or “what if” framework. It is not a skill that usually develops during the school-age years.
Human Development
K&S Chapter 2

Question 16. E. This answer refers to what is currently termed childhood-onset generalized primary dystonia. This
disease entity was formerly known as Oppenheim’s dystonia or dystonia musculorum deformans.
It is commonly seen in individuals of Ashkenazi Jewish decent. Early symptoms are characterized by
an action-induced dystonia of the leg or the arm. Dystonic movements may spread to the trunk and
other limbs, and the condition generalizes over about 5 years. Cognition is generally intact. Parkinson’s,
Wilson’s, Lewy body disease, and Huntington’s are all associated with a dementing process. These disor-
ders are described in other questions in this volume.
Neurology
B&D Chapter 71

Question 17. B. Of the general population, 10% to 15% has a diagnosable personality disorder.
Personality Disorders
K&S Chapter 22

Question 18. A. Approximately 1% of tuberculosis (TB) infections are complicated by neurologic disease, such as intra-
cerebral tuberculoma, tuberculous meningitis, or tuberculous spine involvement with myelopathy (Pott’s
disease). Tuberculous meningitis usually follows a subacute course, with low-grade fever, headache, and
intermittent nausea and vomiting. This is followed by more severe headache, neck stiffness, altered men-
tation, and cranial nerve palsies. The diagnosis is made by identifying tubercle bacilli on CSF acid-fast
bacilli (AFB) smear or culture. CSF examination reveals normal or elevated opening pressure, elevated
protein (80 to 400 mg/dL), low glucose (<40 mg/dL), and pleocytosis (averaging 200 to 400 WBC/μL
with lymphocytic predominance). Tuberculomas, the parenchymal form of TB, are single or multiple
brain or spinal cord lesions. CT or MRI may show low- or high-intensity lesions with ring enhancement.
The other answer choices, B through E, are among the most common CNS manifestations of HIV dis-
ease. These diseases do not classically present with ring-enhancing lesions on neuroimaging. Though not
an answer choice in this question, toxoplasmosis is also noted to cause ring-enhancing lesions on MRI
or CT in HIV patients. Be sure to read up on these other AIDS-related disorders, because they do appear
frequently on standardized examinations.
Neurology
B&D Chapter 53C

Question 19. C. The finding of thin gyri, wide sulci, and enlarged ventricles on neuroimaging studies is typically indica-
tive of cortical atrophy. With normal aging, there is mild to moderate progressive enlargement of the
ventricles, sulci, and cisternal spaces. In neurodegenerative disorders, such as Alzheimer’s disease, atro-
phy is excessive and premature. Chronic alcoholism and drug abuse can cause early cerebral atrophy.
More typically though, chronic alcohol use causes cerebellar atrophy, which can result in truncal ataxia
due to the preferential involvement of the vermis over the cerebellar hemispheres. The classic symptoms
of cerebellar atrophy due to alcohol use are gait unsteadiness and a wide-based gait. Brain imaging of
patients with schizophrenia can also reveal generalized cortical atrophy with enlargement of the ven-
tricular spaces. Trisomy 21 (Down’s syndrome) leads to an early onset of Alzheimer-like changes in the
brain. Trisomy 21 leads to increased levels of amyloid precursor protein (APP) because it is expressed
on chromosome 21. Increase in amyloid deposition occurs as a result of the increase in APP. A single
lacunar infarct of the subcortical white matter would not be expected to be accompanied by generalized
cortical atrophy.
Neurology
B&D Chapter 66

288

Psychiatry Test Preparation and Review


Manual E-Book
Test Number Five

Question 20. A. Middle cerebral artery (MCA) infarction is one of the most common types of strokes. The clinical pic-
ture varies depending on location.

A stem occlusion typically produces a large hemispheric infarction with contralateral hemiplegia, con-
jugate eye deviation toward the side of the infarct, hemianesthesia, and homonymous hemianopia.
Associated global aphasia occurs if the dominant hemisphere is involved. Hemineglect occurs in cases of
nondominant hemispheric involvement.

A Broca-type aphasia is more common in upper division MCA infarcts because of the preferential
involvement of the anterior branches of the upper division when occluded. Answer choice A is the best
response to this question because a perisylvian infarct would almost certainly impair naming and rep-
etition, producing a perisylvian aphasia. Answer B is incorrect because a dominant hemisphere infarct
in the MCA territory would almost certainly involve some kind of language functioning impairment.
Answer choice C is incorrect because the left MCA territory stroke would cause a right hemiparesis with
face and arm weakness that is worse than the leg. This is so, because the anterior cerebral artery irrigates
the part of the motor strip responsible for lower-extremity movement. Answers D and E are features of
a Wernicke-type aphasia. Wernicke’s area is indeed within the MCA territory, particularly the inferior
branch, but is less likely to be affected than Broca’s area.
Neurology
B&D Chapters 12A and 51A

Question 21. D. There is only one ethical answer here, and that is to transfer the patient’s care to another psychiatrist.
There are no exceptions to the rule prohibiting sex between psychiatrist and patient. Regardless of cir-
cumstances, how the doctor processes the matter, or what the doctor tells the patient, having a sexual
relationship is prohibited. The doctor can no longer serve a neutral therapeutic role in this case. It must
be transferred to another psychiatrist.
Ethics
K&S Chapter 36

Question 22. B. Pure alexia without agraphia is classically caused by a stroke in the left posterior cerebral artery terri-
tory, particularly in the splenium of the corpus callosum. Patients can write, but they cannot read their
own writing. Speech, auditory comprehension, and repetition are intact. Naming, particularly for colors,
may be deficient. Associated signs may include right hemianopia or superior quadrantanopia, short-term
memory loss, and the absence of motor and sensory signs.
Neurology
B&D Chapter 12A

Question 23. C. A binary variable has two possible values, such as yes or no, positive or negative, male or female. A con-
tinuous variable will fall somewhere on a range, such as age, height, or weight. Independent variables
(predictor variables) are contributing factors to a result or predictors of a certain outcome within an
experimental study. They are the variables that are manipulated by the experimenter. Dependent vari-
ables (outcome variables) are the outcomes that the independent variables contributed to or predicted.
Dependent variables are the variables that are not manipulated by the experimenter.

To answer this type of question, first ask yourself two more questions. How many variables are there?
Are they binary or continuous? Then consider the following:

The χ2 test is used for one binary predictor variable and one binary outcome variable. The T test is used
for one binary predictor variable and one continuous outcome variable. The ANOVA is used for two or
more binary predictor variables and one continuous outcome variable. Correlation is used for one con-
tinuous predictor variable and one continuous outcome variable. Regression analysis is used with two or
more continuous or binary variables and one continuous outcome variable.
Statistics
K&S Chapter 5

289

Psychiatry Test Preparation and Review


Manual E-Book
Psychiatry Test Preparation & Review Manual

Question 24. A. Homocystinuria is an inborn error of amino acid metabolism. Three types of enzymatic deficiencies
are possible: homocysteine methyltransferase, methylene tetrahydrofolate reductase, and cystathione-
β-synthetase. The accumulation of homocysteine in the blood leads to endothelial injury and prema-
ture atherosclerosis. Patients can present with marfanoid habitus, livedo reticularis, malar flush, ectopia
lentis, glaucoma, myopia, optic atrophy, mental retardation, spasticity, seizures, psychiatric problems,
osteoporosis, and a high likelihood of intracranial arterial or venous thrombosis. Death can result from
myocardial infarction, stroke, or pulmonary embolism. Raised plasma homocysteine levels may be an
independent risk factor for coronary artery disease, cerebrovascular disease, or peripheral artery occlu-
sive disease. Elevated homocysteine levels can effectively be lowered by administration of folic acid,
sometimes needing the addition of pyridoxine (vitamin B6) and vitamin B12.

Phenylketonuria is an aminoacidopathy resulting from a deficiency of phenylalanine hydroxylase. The


pattern of inheritance is autosomal recessive. Symptoms may include delayed mental and social skills,
head size significantly below normal, hyperactivity, jerking movements of the arms or legs, mental retar-
dation, seizures, skin rashes, tremors, and unusual positioning of hands. If the condition is untreated or
foods containing phenylalanine are not avoided, a “mousy” or “musty” odor may be detected on the
breath and skin and in urine. The unusual odor is due to a buildup of phenylalanine substances in the
body.

Tay–Sachs disease is the infantile form of hexosaminidase-A deficiency. The pattern of inheritance is
autosomal recessive. It is caused by a mutation in the HEXA gene on chromosome 15. Infants with the
disorder appear to develop normally for the first 6 months after birth. Then, as nerve cells become dis-
tended with gangliosides, a relentless deterioration of mental and physical abilities occurs and progresses
inexorably. The child becomes blind, deaf, and unable to swallow and develops atrophy and paralysis.
Death usually occurs before age 4. The disease has no known treatment or cure.

Metachromatic leukodystrophy (MLD) is yet another lysosomal storage disease of infancy. This disor-
der results from a deficiency in arylsulfatase-A. Again, its pattern of inheritance is autosomal recessive.
Affected children begin having difficulty walking after the first year of life, usually at 15 to 24 months.
Symptoms include muscle wasting and weakness, muscle rigidity, developmental delays, progressive loss
of vision leading to blindness, convulsions, impaired swallowing, paralysis, and dementia. Children may
become comatose. Untreated, most children die by age 5, often much sooner. There is no cure for MLD
and no standard treatment.
Neurology
B&D Chapters 51A and 62

Question 25. D. Patients with posttraumatic stress disorder (PTSD) have demonstrated decreased hippocampal volume
on neuroimaging studies. Some have postulated that this is a result of cortisol release in response to
the trauma; however, there is still some debate as to whether their decreased hippocampal volume is a
result of the trauma, or whether patients who have decreased hippocampal volume at baseline are more
prone to developing PTSD when exposed to trauma. Studies of PTSD subjects have also found smaller
anterior cingulate cortex volumes, which have been associated with more severe symptoms in some stud-
ies, suggesting that this may be a vulnerability factor to PTSD. Dopaminergic neurotransmission is not
measured by standard neuroimaging so is not a possible correct answer. Increased size of the lateral
ventricles and decreased metabolic activity in the prefrontal cortex are neuroimaging findings associated
with schizophrenia.
Trauma-and Stress-Related Disorders
K&S Chapter 11

Question 26. B. Niemann–Pick disease is a sphingomyelinase deficiency that is categorized as a lysosomal storage disease
of infancy. The pattern of inheritance is autosomal recessive and is linked to a mutation in the SMPD1
gene. Symptoms are related to the organs in which sphingomyelin accumulates. Enlargement of the liver
and spleen (hepatosplenomegaly) may cause reduced appetite, abdominal distension, and pain, as well as
thrombocytopenia secondary to splenomegaly. Sphingomyelin accumulation in the CNS (including the
cerebellum) results in unsteady gait (ataxia), slurring of speech (dysarthria), and discoordinated swal-
lowing (dysphagia). Basal ganglia dysfunction causes abnormal posturing of the limbs, trunk, and face

290

Psychiatry Test Preparation and Review


Manual E-Book
Test Number Five

(dystonia), and upper brain-stem disease results in impaired voluntary rapid eye movements (supranu-
clear gaze palsy). Current and future treatment research focuses on enzyme replacement therapy and
gene therapy.

Hexosaminidase-A deficiency is Tay–Sachs disease. Arylsulfatase-A deficiency is MLD. Phenylalanine


deficiency is phenylketonuria. HPRT deficiency results in hyperuricemia and Lesch–Nyhan syndrome.
Lesch–Nyhan syndrome, unlike the other diseases noted in this question, is an X-linked recessive disease.
It is a disorder of purine and pyrimidine metabolism. These diseases of inborn errors in metabolism are
all explained further in other questions in this volume.
Neurology
B&D Chapter 62

Question 27. D. Depakote has the potential to cause pancreatitis, which will lead to an increased serum amylase and the
symptoms described in this question. Another concern with valproic acid would be hepatitis, for which
one would want to look at liver function tests, but hepatitis would most likely present with right upper
quadrant pain, nausea, vomiting, diarrhea, fever, Coca-Cola urine, and jaundice.
Psychopharmacology/Laboratory Tests in Psychiatry
K&S Chapter 29

Question 28. E. The disease described in answer choice E is that of fragile X syndrome. Fragile X syndrome is the most
common inherited cause of infantile intellectual disability and the most common identifiable cause of
autism symptoms. Of course, this question is constructed to try to trick you by not giving you the dis-
ease name but forcing you to extrapolate the disease name from its presenting symptoms. Fragile X
results in a spectrum of intellectual disability ranging from mild to severe and in physical characteristics
that include an elongated face, large or protruding ears, and large testes (macro-orchidism); behavioral
characteristics, such as stereotypical movements (e.g., hand-flapping); and social anxiety.

Fragile X syndrome is associated with the expansion of the CGG trinucleotide repeat affecting the frag-
ile X mental retardation 1 (FMR1) gene on the X chromosome, resulting in a failure to express the frag-
ile X mental retardation protein (FMRP), which is required for normal neural development. Depending
on the length of the CGG repeat, an allele may be classified as normal (unaffected by the syndrome), a
premutation (at risk of fragile X associated disorders), or a full mutation (usually affected by the syn-
drome). Those with more than 200 repeats tend to be affected by the syndrome. A definitive diagnosis of
fragile X syndrome is made through genetic testing to determine the number of CGG repeats. The mode
of transmission is X-linked dominant. Fragile X ranks as the most common cause of inherited intellec-
tual disability.

There is no drug treatment that has shown benefit specifically for fragile X syndrome. However, medica-
tions are commonly used to treat symptoms of attention deficit and hyperactivity, anxiety, and aggres-
sion. Supportive management is important in optimizing functioning in individuals with fragile X
syndrome and may involve speech therapy, occupational therapy, and individualized educational and
behavioral programs. Fragile X can be found in both males and females.
Neurology
B&D Chapter 40

Question 29. D. The signs and symptoms of Down’s syndrome include the following: microgenia (abnormally small
chin), oblique eye fissures with epicanthic skin folds on the inner corner of the eyes (formerly known
as a mongoloid fold), muscle hypotonia (poor muscle tone), a flat nasal bridge, a single palmar fold, a
protruding tongue (due to a small oral cavity and an enlarged tongue near the tonsils) or macroglossia,
“face is flat and broad,” a short neck, white spots on the iris known as Brushfield spots, excessive joint
laxity including atlantoaxial instability, excessive space between the large toe and the second toe, a single
flexion furrow of the fifth finger, and short fingers. They tend to have a single palmar crease (also called
a “simian crease” for its resemblance to that in apes and monkeys). Growth parameters, such as height,
weight, and head circumference, are smaller in children with DS than in typical individuals of the same
age. Adults with DS tend to have short stature and bowed legs. Individuals with DS are also at increased
risk for obesity as they age. Individuals with trisomy 21 often develop early-onset Alzheimer-type

291

Psychiatry Test Preparation and Review


Manual E-Book
Psychiatry Test Preparation & Review Manual

dementia and die young from its complications. This occurs because the amyloid precursor protein gene
is expressed on chromosome 21, which these patients possess in triplicate. Stunted growth and mental
retardation are seen in virtually 100% of trisomy 21 patients. Small genitalia are seen in at least 75% of
cases, and in most cases these individuals are sterile.
Neurodevelopmental Disorders
B&D Chapters 40 and 66

Question 30. B. Answer choice B refers to Williams’ syndrome. The syndrome is a result of an autosomal microdeletion
on chromosome 7q11. Features of the disorder include cardiac valvular stenosis, hypotonia, hyperacusis,
short stature and “elfin” facies, low nasal bridge, unusual ease with strangers and pleasant demeanor,
developmental delay, and strong language skills. The disorder has no known cure and is strongly associ-
ated with attention deficit hyperactivity disorder.

The distractors in this question are described in greater detail in other questions in this volume. Trisomy
21 is, of course, Down’s syndrome. Deletion in chromosome 15q is Angelman’s syndrome. Deletion of
45 XO refers to Turner’s syndrome. Klinefelter’s syndrome is a result of an extra sex chromosome, 47
XXY.
Neurodevelopmental Disorders
B&D Chapter 7

Question 31. B. Right unilateral placement of the electrodes in ECT has been shown to minimize cognitive impairment
and memory deficits. If unilateral electrode placement fails to improve the patient’s symptoms after four
to six treatments, the placement may be switched to bilateral, which can be more effective but carries a
higher risk of side effects.
Diagnostic and Treatment Procedures in Psychiatry
K&S Chapter 30

Question 32. C. Marfan’s syndrome is an inherited autosomal dominant connective tissue disease associated with defects
in fibrillin. Features include arachnodactyly, joint laxity, extreme limb length, pectus carinatum or exca-
vatum, aortic valvular insufficiency, and subluxation of the lens. Marfan’s syndrome is also associated
with dilatation of the aortic root and coarctation of the aorta, mitral valve prolapse, and mitral annulus
calcification with regurgitation. Dissection of the ascending aorta is also a possible occurrence, following
from dilatation of the aortic root. Intracranial aneurysms or carotid artery dissection are also possible.
It is recommended that patients with Marfan’s syndrome undergo annual echocardiogram screening and
avoid contact sports.

The other answer choices are all inherited disorders that present with either mental retardation or autis-
tic features, or both, and they are explained in the answer material of other questions in this volume.
Marfan’s syndrome is not associated with either mental retardation or autism.
Neurology
B&D Chapter 51A

Question 33. A. Lithium can prolong seizure activity during ECT and should be discontinued. Antipsychotics are fine
during ECT, with the exception of clozapine, which causes late-appearing seizures during ECT. Tricyclic
antidepressants (TCAs) and MAOIs are fine to continue during ECT.
Diagnostic and Treatment Procedures in Psychiatry
K&S Chapter 29

Question 34. A. The major features of tuberous sclerosis include facial angiofibromas or forehead plaque, nontraumatic
nail fibroma, hypomelanotic macules (ash leaf spots), shagreen patch, multiple retinal hamartomas,
cortical tuber, subependymal nodule, subependymal giant astrocytoma, cardiac rhabdomyoma, lymph-
angiomyomatosis, and renal angiomyolipoma. The predominant neurologic manifestations of tuber-
ous sclerosis are seizures, mental retardation, and behavioral abnormalities. Seizures of various types
occur in 80% to 90% of patients with tuberous sclerosis. Tuberous sclerosis is autosomal dominant
and comes from mutations on chromosomes 9 and 16. The TSC1 gene codes for Hamartin, and the
TSC2 gene codes for Tuberin. These proteins control cell size and growth. Cutaneous and conjunctival

292

Psychiatry Test Preparation and Review


Manual E-Book
Test Number Five

telangiectasias are seen in the disorder known as ataxia–telangiectasia (AT). This is a neurodegenerative
disorder that begins in childhood as slowly progressing ataxia. Later, other features develop, in particu-
lar, telangiectasias (small dilated blood vessels), deficiency of IgA and IgE, and delayed sexual matura-
tion. The telangiectasias typically involve the earlobes, sclera, bridge of the nose, and less commonly,
eyelids, neck, and antecubital and popliteal fossae. AT is an autosomal recessive neurocutaneous dis-
order affecting both sexes equally. Nearly all patients with AT have elevated α-fetoprotein levels. The
gene associated with AT is on chromosome 11. Many patients develop leukemia, lymphoma, or diabetes.
Therapies involve antimicrobials and intravenous immune globulin (IVIG).
Neurology
B&D Chapter 65

Question 35. A. Sublimation is a mature defense mechanism. Displacement and repression are neurotic defense mecha-
nisms. Hypochondriasis and introjection are immature defense mechanisms. For your examination, you
must not only know what each defense mechanism is but also be able to say which are mature and
which are immature.
Psychological Theory and Psychometric Testing
K&S Chapter 4

Question 36. D. Thalamic lesions are usually responsible for sensory deficits of origin as well as significant pain. The
thalamus receives its blood supply from the thalamoperforate branches of the posterior cerebral arteries.
In some individuals, both thalami are supplied by one posterior cerebral artery, thus unilateral arterial
occlusion may result in bilateral thalamic infarction. Thalamic pain syndrome sometimes results after
thalamic sensory stroke. The pain is typically spontaneous and localized to the distal arm and leg and is
exacerbated by contact and stress. It can be experienced as an intense burning, freezing, or prickly sensa-
tion (like being stuck repetitively with needles).
Neurology
B&D Chapter 28

Question 37. C. When a forensic psychiatrist is evaluating a case, he or she is doing so for a third party, such as the
court. As such, there is no assumption of confidentiality the way there normally would be with a psy-
chiatrist. The forensic psychiatrist should tell the patient this at the beginning of the interview. The infor-
mation collected is expected to be shared with a third party and is not expected to remain confidential.
If there is no confidentiality and no therapeutic alliance, there is no doctor–patient relationship. Both
evaluations should include a medication history and basic elements of the mental status examination
and mini mental status examination, such as memory, as would any complete psychiatric examination,
regardless of setting. Both evaluations will contain recommendations. ER recommendations will focus
on the next step in treatment, whereas forensic recommendations will focus on the particular legal issues
being decided and the effect of the patient’s illness upon those issues.
Forensic Psychiatry
K&S Chapter 36

Question 38. D. The typical EEG findings in Creutzfeldt–Jakob disease (CJD) are periodic sharp-wave complexes. These
characteristic EEG abnormalities can be seen in about 80% of patients with CJD.

EEG findings in Alzheimer’s dementia are dependent on timing. Early in the disease course, the EEG
may be normal or show an alpha rhythm at or just below normal. As the disease progresses, general-
ized slowing ensues. Delirium is most often characterized by generalized slow-wave activity on EEG. If
triphasic waves are seen, these are indicative of a metabolic cause in an unresponsive patient. Previously,
it was thought that these triphasic waves were diagnostic of hepatic encephalopathy, but other metabolic
causes such as uremia, hypoxia, hyponatremia, and hyperosmolarity can also produce this EEG abnor-
mality. There are no particular classic EEG abnormalities associated directly with either Parkinson’s dis-
ease or HIV-related dementia.
Neurology
B&D Chapters 32A and 66

293

Psychiatry Test Preparation and Review


Manual E-Book
Psychiatry Test Preparation & Review Manual

Question 39. A. The classic triad of dementia, urinary incontinence, and gait apraxia is the clinical picture of normal-
pressure hydrocephalus (NPH). The gait problems resemble those in Parkinson’s disease, though they
abate after gait is initiated. Cognitively, patients can present with deficits in speed of mental processing,
decreased memory functioning, poor abstraction, and difficulty with set-shifting tasks. Other common
features include depression, hypersomnia, abulia, and apathy. The gait disturbance and incontinence are
believed to be a result of pressure on, and distortion of, descending white matter pathways. The cogni-
tive symptoms are thought to be due to pressure on anterior cortical structures. Neuroimaging studies
usually show ventricular enlargement of the lateral, third, and fourth ventricles. Treatment of NPH is
placement of a ventricular peritoneal shunt. The other distractors in answers B through E are explained
in detail in other questions in this volume.
Neurology
B&D Chapter 66

Question 40. B. The clinical picture here is that of the epidural hematoma. Notice that nowhere in this question is the
phrase “epidural hematoma” mentioned. You are asked to extrapolate this from the data given. Epidural
hematomas are located between the inner table of the skull and the dura. They typically occur when a
skull fracture tears the middle meningeal artery or one of its branches. Epidural hematomas have a lens-
shaped appearance and smooth inner border, because as they enlarge, they strip the dura away from the
inner table of the skull. The classic clinical presentation is as described in this question: a brief loss of
consciousness followed by an intercurrent period of lucidity, and then a later deterioration. Recall that
damage to the bridging veins usually causes a subdural hematoma, which is often seen in alcoholics who
have a head injury that goes undetected. An intracranial aneurysmal rupture would present as a sub-
arachnoid hemorrhage and “the worst headache of my life” scenario. Damage to the reticular activating
system (in the brain stem) would probably cause unconsciousness and, if protracted, coma. Frontal lobe
damage from traumatic brain injury, for example, can result in loss of executive functioning and plan-
ning ability, impulsivity, inappropriate behavior, and mood swings.
Neurology
B&D Chapter 50B

Question 41. B. In shared care or split treatment the psychiatrist retains full responsibility for the patient’s care. This
does not absolve the responsibility of the other health professionals involved, as they can also be held
accountable. However, split treatment situations require that the psychiatrist remain fully informed of
the patient’s clinical condition and the treatment the patient is receiving from other providers, because
the psychiatrist is ultimately responsible for the patient’s care.
Forensic Psychiatry
K&S Chapter 36

Question 42. E. This is a great question because it takes you through the most common CNS manifestations of AIDS.
The best answer here is HIV-related dementia (ARDC). ARDC is the most common complication of
late untreated HIV disease. Early symptoms usually consist of problems with attention and concentra-
tion. Many patients have a slowness of thought (bradyphrenia). Complex tasks may become difficult to
accomplish and may take longer owing to these problems. Social withdrawal, apathy, depression, and
fatigue are all commonly seen in ARDC. There are generally psychomotor deficits noted as well. These
can include gait incoordination and rigidity and slowness of gait. Fine and skilled hand and finger move-
ments are affected early in the course of ARDC.

Diffuse cerebral atrophy is almost always seen on brain MRI and CT imaging in ARDC patients. In
some patients, MRI may show nonspecific white matter abnormalities. CSF examination can be used
to exclude other infectious diagnoses, such as TB, syphilis, CMV, and cryptococcus. Please note that the
other four answer choices, A through D, are explained in other questions in this volume. Do study them,
because they can and do appear on standardized examinations.
Neurology
B&D Chapter 53A

294

Psychiatry Test Preparation and Review


Manual E-Book
Test Number Five

Question 43. B. The CYP 2D6 enzyme has a long list of substrates. All of the choices listed in the question are substrates
except bupropion, which is an inhibitor. Other inhibitors include citalopram, duloxetine (this is both
a substrate and an inhibitor), escitalopram, fluoxetine, methadone, paroxetine, sertraline, and TCAs.
Bupropion is a substrate of 2B6.
Psychopharmacology
K&S Chapter 1

Question 44. D. This is a simple question really, but some will find it tricky because it asks about the less common symp-
toms and signs of PD. You can never study PD enough for standardized examinations in psychiatry.
This topic comes up time and time again on neurology questions, because there is a strong overlap with
behavioral and psychiatric presentation in this disorder. We’ve discussed the disorder in many questions
in this book, but we want to give you some of the essential bullet points now:

• T he pathology of PD presents as depigmentation and neuronal loss in the substantia nigra and pres-
ence of Lewy bodies and pale bodies.
• The pathologic hallmark is dopaminergic underactivity in the striatum.
• 60% to 85% of striatal dopamine and nigral neurons must be lost before symptoms of PD arise.
• The prevalence is 100 to 200 per 100,000 population.
• Cardinal features are resting tremor (4 to 7 Hz), rigidity, postural instability, and bradykinesia.
• Tremor is called “pill-rolling,” because it resembles the rolling of the fingertips over a pill surface.
• Festination, or a festinating gait, refers to the tendency of the patient to propulse involuntarily while
standing or walking, or to retropulse and fall.
• Manifestations are usually asymmetrical early in the disease and become bilateral later on in the course.
• Levodopa–carbidopa is the mainstay of therapy. Other important antiparkinsonian agents include
dopamine agonists, MAOIs, and anticholinergics.
Remember that alcoholic cerebellar atrophy presents with truncal ataxia for the most part, because the
cerebellar vermis is preferentially affected. The vermis is responsible for truncal balance and smooth
movement around the midline.
Neurology
B&D Chapter 71

Question 45. C. Compared with men, women have less alcohol dehydrogenase in their gut. As such, men metabolize more
alcohol in their gut, leading to lower blood alcohol levels. Women have a lower body water content com-
pared with men so that the alcohol is distributed in less water and is therefore more concentrated. The ques-
tion said they both drank the same amount, so choice A is wrong. There is no evidence that Susan urinates
more than Billy or that it can be assumed that a woman would urinate more than a man. The question stem
says nothing about their weight, so we can’t make any assumptions. If you chose answer choices A, D, or E,
you made assumptions about data that were not provided in the question stem. This is a common way that
people get questions wrong. Never insert data into the question that the test writer didn’t give you.
Substance Abuse and Addictive Disorders
K&S Chapter 20

Question 46. B. Here we have another question that asks you to jump directly to appropriate management from symp-
tom presentation only. The question bypasses asking you what the disease entity is, because that would
clearly be too simple: temporal arteritis. Temporal arteritis (TA) is a vasculitis found in the elderly
patient. If left undiagnosed and untreated, it can frequently lead to permanent blindness or ischemic
stroke. Headache is the most common symptom. More than half of the patients with TA present con-
comitantly with polymyalgia rheumatica, which consists of aching in proximal and axial joints, proxi-
mal myalgias, and often significant morning stiffness. Jaw claudication (jaw pain on chewing) is a
common associated symptom and when present is almost pathognomonic for the diagnosis. Amaurosis
fugax (temporary loss of vision as if a curtain is coming down over the eye) is one of the most omi-
nous symptoms, as it may evolve to partial or total blindness if left untreated. The most often recog-
nized laboratory abnormality in TA is the erythrocyte sedimentation rate, which is elevated in most cases

295

Psychiatry Test Preparation and Review


Manual E-Book
Psychiatry Test Preparation & Review Manual

(60 mm +). Diagnosis is then confirmed by temporal artery biopsy. Nevertheless, it is essential to treat
early, even before biopsy is done or results are known, with high-dose oral prednisone to avoid the onset
of irreversible visual loss. Prednisone should be initiated at 40 to 60 mg daily and maintained for at least
1 month before beginning a cautious taper.
Neurology
B&D Chapter 69

Question 47. B. Low levels of folate can lead to fatigue, agitation, delirium, dementia, psychosis, and paranoia. It is seen
commonly in alcoholics. Low folate does not cause panic attacks.
Laboratory Tests in Psychiatry
K&S Chapter 24

Question 48. A. This is a simple question. Answer choices B through E are all preventative (prophylactic) treatments of
migraines. Divalproex sodium and topiramate are both in fact FDA-approved for preventative therapy
of migraine headache. Desipramine is of course a TCA that is used off-label for neuropathic pain. It
is also effective in the prevention of migraine, and this is an off-label use as well. Gabapentin is an
anticonvulsant agent, FDA-approved for seizure and postherpetic neuralgia. It too is used off-label for
the prevention of migraine. The triptans, including sumatriptan, naratriptan, eletriptan, and rizatriptan,
are abortive agents for migraines. Their mechanism of action is as agonists at the 5-HT 1B and 5-HT
1D receptors. These agents modulate the excitability of cells in the trigeminal nucleus caudalis, which
receives input from the trigeminal nerve.
Neurology
B&D Chapter 69

Question 49. C. This patient is clearly hypothyroid. If you miss that fact and treat the depression and not the thyroid,
you will get the question wrong and mistreat the patient. Symptoms of hypothyroidism include depres-
sion, weakness, stiffness, poor appetite, constipation, menstrual irregularities, coarse hair, slowed speech,
apathy, impaired memory, hallucinations, or delusions.

On a related topic, sleep apnea is another medical illness that can present similarly to depression. A case
of a patient with daytime fatigue and mood disturbance who is significantly overweight should lead you
to consider a sleep study.

These are very common standardized test questions. Make sure you get them right when they show up
on an examination near you!
Somatic Symptom Disorders
K&S Chapter 13

Question 50. E. Rupture of an intracranial aneurysm or arteriovenous malformation (AVM) results in a subarachnoid hemor-
rhage, which may extend into the brain parenchyma. The headache associated with this bleed is usually sud-
den in onset and explosive. It is often described by patients as “the worst headache of my life.” The headache
is often accompanied by vomiting. Hydrocephalus may develop, resulting from intraventricular blood that
distorts the midline structures. Noncontrast head CT confirms the diagnosis by revealing blood in the sub-
arachnoid cisterns or in the parenchyma. If CT confirms the presence of blood in the subarachnoid spaces,
lumbar puncture should be avoided to prevent a possible herniation or further bleeding. Subarachnoid hem-
orrhage generally indicates the need for cerebral angiography. Note that the most frequent cause of subarach-
noid hemorrhage is rupture of an aneurysm arising from one of the arteries that form the circle of Willis.
Neurology
B&D Chapters 49, 51C, and 69

Question 51. C. Damage to the medial longitudinal fasciculus between the third cranial nerve (oculomotor) and the sixth
cranial nerve (abducens nerve) nuclei interrupts transmission to the ipsilateral medial rectus muscle. Because
those nuclei themselves remain intact, the pupils and eyelids are normal in both eyes. Adduction of the ipsi-
lateral eye is impaired. The nystagmus-like movement is in fact overshoot dysmetria. Upward-beating and
torsional nystagmus are often present. The classic cause of this lesion is a demyelinating lesion of the brain

296

Psychiatry Test Preparation and Review


Manual E-Book
Test Number Five

stem in multiple sclerosis. There are many other possible causes of internuclear ophthalmoplegia (INO). These
include pontine stroke, intrinsic tumor, drug intoxication, and chemotherapy with radiation therapy. INO can
be caused by MG and progressive supranuclear palsy, but certainly less often than in multiple sclerosis.
Neurology
B&D Chapter 36

Question 52. E. Addison’s disease (adrenal insufficiency) symptoms include fatigue, hyperpigmentation, anorexia and
weight loss, low blood pressure, nausea, salt craving, and joint and muscle pain. There are primary,
secondary, and tertiary causes. Primary adrenal insufficiency is autoimmune. Secondary is the result of
panhypopituitaryism. Tertiary can be caused by high-dose glucocorticoid therapy. Laboratory work
reveals hyponatremia, hyperkalemia, and low serum cortisol. Psychiatric symptoms include depres-
sion and delirium. Diagnosis is made based on a poor cortisol response to an adrenocorticotropic hor-
mone (ACTH) stimulation test. Treatment involves replacement of cortisol. Other answer choices are
addressed elsewhere in this volume.
Psychosomatics
K&S Ch. 21

Question 53. B. Signs of cannabis intoxication include conjunctival injection, increased appetite, dry mouth, and tachy-
cardia. Impaired motor coordination, euphoria, anxiety, sensation of slowed time, impaired judgment,
and social withdrawal may also be found. Diuresis is not one of the side effects.
Substance Abuse and Addictive Disorders
K&S Chapter 20

Question 54. C. Huntington’s disease (HD) is an autosomal dominant neurodegenerative disorder characterized by progres-
sive movement disorder associated with psychiatric and cognitive decline and culminating in a terminal state
of dementia and immobility. Prevalence is about 10 per 100,000. HD usually begins between ages 30 and 55
years. Dementia and chorea usually appear within a year of each other. The first neuropsychiatric symptom
to develop is often depression. A change in the ability to generate saccadic eye movements and their speed is
often seen early in the disease. In normal saccades both eyes rapidly rotate or jump to look from one fixed
object to another. Abnormalities can include blinks, head toss or thrust, slow acceleration, catch-ups, and
under or over shooting. The motor disorder usually begins with clumsiness and fidgetiness that evolves into
chorea. In addition to chorea, HD patients have bradykinesia and motor impersistence, with difficulty sus-
taining ongoing movement. The gait often resembles a marionette, lurching, swaying, dipping, and bobbing.
Dysarthria and dysphagia progressively impair communication and nutrition. Mean survival is 17 years. Most
patients wind up wheelchair- or bedbound and perish from complications that include pneumonia and head
injury. HD is dominantly inherited and caused by an unstable expanded CAG trinucleotide repeat on chromo-
some 4. Essentially all patients with 40 or more CAG repeats will develop the clinical illness. The direct DNA
test for the CAG repeat expansion in the huntingtin (HTT) gene is highly sensitive and specific. The pathology
of HD includes prominent neuronal loss and gliosis in the caudate nucleus and putamen along with regional
and more diffuse atrophy. No treatment has yet been proven to improve disease progression. The use of tet-
rabenazine has been found to be effective in reducing chorea in HD patients and is now FDA-approved and
available in the United States for this labeled use. As a bonus fact, keep in mind that abnormal saccades and
pursuits can be found in HD, schizophrenia, first-degree relatives of schizophrenics, and concussions.
Neurology
B&D Chapter 71

Question 55. C. Carbamazepine will interact with all of the drugs listed in this question. Erythromycin will increase car-
bamazepine levels. Phenobarbital and theophylline will decrease carbamazepine levels. Carbamazepine
will decrease doxycycline and cyclosporine levels.
Psychopharmacology
K&S Chapter 29

Question 56. C. Extracranial vertebral artery dissection presents with high cervical pain sometimes radiating to the
occipital region, and with posterior circulation transient ischemic attacks or stroke. Vertebral artery
dissection can occur spontaneously, but most commonly, it is due to trauma, which can be dramatic

297

Psychiatry Test Preparation and Review


Manual E-Book
Psychiatry Test Preparation & Review Manual

(cervical fractures and dislocation) or subtle (abrupt turning of the neck, chiropractic manipulation, or
simple hyperextension of the neck, such as sometimes used for shaving under the chin). Anticoagulation
is the mainstay of treatment for extracranial vertebral dissection when there is ischemic symptomatol-
ogy. The results are usually favorable, unless the patient already has a fixed deficit.

Pain in the arm can result from damage to the brachial plexus. Infiltrative or inflammatory lesions of
the brachial plexus produce severe pain radiating down the arm and spreading into the shoulder region.
Causes of brachial plexopathy include traction and heavy impact. Sporting injuries in bicycling, football,
snowmobiling, skiing, and horseback riding can all cause plexus injury. Rucksack paralysis is another
form of brachial plexus injury from the traction and pressure exerted on the brachial plexus by heavy
backpack straps to the shoulders. Other plexus injuries can be caused by metastatic disease or radiation-
induced injury in the cancer patient.

Sacroiliac joint syndrome is a major source of low back pain. Pain can present on one side of the lower
back and radiate down to the hip or thigh. Pain often worsens upon walking up stairs. Single leg stand-
ing test often exacerbates the pain. Nonsteroidal antiinflammatory drugs (NSAIDs) are the first-line ther-
apy in patients with joint inflammation. Sacroiliac joint corticosteroid injection can provide temporary
pain relief.

Anterior spinal artery syndrome results in spinal cord infarction and has been seen more frequently
because of increased numbers of invasive surgical procedures, such as vascular and thoracoabdominal
surgeries. The anterior horns and anterolateral tracts are affected (corticospinal tract involvement below
the level of the lesion), resulting in loss of bowel and bladder control and sexual dysfunction. A sensory
deficit develops, sparing the posterior columns and involving only the spinothalamic tracts. Spinal shock
with areflexia is expected initially, followed by later onset of spasticity.
Neurology
B&D Chapters 24, 29, 44, and 46

Question 57. A. Bell’s palsy is a self-limited facial nerve palsy of acute or subacute onset. Pain often accompanies the facial
weakness, with impaired lacrimation, taste changes, and hyperacusis also being seen. MRI of the internal
auditory canal often demonstrates enhancement of the facial nerve, most often at the geniculate ganglion.
Eighty-five percent of patients recover normal facial function within 3 weeks. Bell’s palsy occurs with
increased frequency during pregnancy, and pregnancy is associated with a poorer recovery rate. Residual
abnormalities after Bell’s palsy occur in 12% of patients, persistent severe facial weakness in 4%, and
contraction and twitching of the upper and lower facial muscles in 17%. Aberrant regeneration involv-
ing the lacrimal gland may lead to tearing with facial muscle contraction (syndrome of “crocodile tears”),
particularly during eating. Bell’s palsy can be recurrent, but in these cases other causes such as Lyme dis-
ease or sarcoidosis must be considered. Bilateral facial palsy is also common with acute inflammatory
demyelinating polyneuropathy. Bell’s palsy is considered to be idiopathic, but herpes simplex virus and
varicella zoster virus reactivation in the geniculate ganglion have been implicated in its pathogenesis.
Early administration of corticosteroids and acyclovir is common practice and may enhance recovery.

The classic triad of Horner’s syndrome is that of ipsilateral ptosis, miosis, and facial anhidrosis. Lateral
medullary syndrome (Wallenberg’s syndrome) typically results from occlusion of the posterior inferior
cerebellar artery and produces sensory loss on the ipsilateral face, from trigeminal involvement, plus loss
of pain and temperature sensation from the contralateral body, from damage to the ascending spinotha-
lamic tract. Motor findings include ipsilateral cerebellar ataxia, bulbar weakness resulting in dysarthria
and dysphagia, and Horner’s syndrome.
Neurology
B&D Chapters 16, 28, and 70

Question 58. E. The pain of postherpetic neuralgia (PHN) is described as continuous deep aching, burning, sharp stab-
bing, and shooting, and it is triggered by light touch over the affected dermatomes. It is often debilitating
and difficult to treat. Tricyclics (amitriptyline or desipramine), SSRIs (sertraline or nefazodone), anticonvul-
sants (carbamazepine and gabapentin), oral opioids (oxycodone), and topical capsaicin cream or lidocaine
patches are helpful in about 50% of patients. Note that gabapentin is the best answer to this question
because it is specifically FDA-approved for PHN, and carbamazepine and desipramine are used off-label.

298

Psychiatry Test Preparation and Review


Manual E-Book
Test Number Five

PHN occurs in anywhere from 8% to 70% of patients after herpes zoster (the shingles, which is varicella
zoster virus reactivation of the dorsal root ganglion). Herpes zoster is characterized by sharp or burning
radicular pain, sometimes associated with fever, malaise, and rash. The cutaneous eruption begins as an
erythematous maculopapular rash and progresses to grouped clear vesicles that continue to form for 3
to 5 days. These become pustules by 3 to 4 days and form crusts by 10 days. Pain usually disappears as
the vesicles fade.
Neurology
B&D Chapter 75

Question 59. B. Carbon monoxide exposure occurs mainly in miners, gas workers, and garage employees. Other expo-
sures occur with poorly ventilated home-heating systems, stoves, and suicide attempts. The neurotoxic
effects of carbon monoxide relate to intracellular hypoxia. Carbon monoxide binds to hemoglobin with
high affinity to form carboxyhemoglobin. Acute toxicity leads to headache, disturbances of conscious-
ness, and other behavioral changes. Motor abnormalities can occur with pyramidal and extrapyramidal
deficits. Seizures may occur, and focal cortical deficits sometimes develop. Pathologic examination shows
hypoxic and ischemic damage in the cerebral cortex as well as in the hippocampus, cerebellar cortex,
and basal ganglia. Lesions are also present diffusely in the cerebral white matter.
Neurology
B&D Chapter 58

Question 60. D. In approximately 80% of patients with stiff-person syndrome, the disorder develops as a nonparaneo-
plastic phenomenon. It is often diagnosed by finding anti-GAD antibodies (glutamic acid decarboxylase)
in the serum and CSF. Characteristic of stiff-person syndrome is fluctuating rigidity of the axial muscu-
lature with superimposed spasms. Muscle stiffness primarily affects the lower trunk and legs, but it can
extend to the arms, shoulders, and neck. The paraneoplastic form of stiff-person syndrome is usually
associated with breast and lung cancers and Hodgkin’s disease. Treatment of the tumor and the use of
corticosteroids may improve paraneoplastic stiff-person syndrome. Intravenous immunoglobulin is use-
ful in patients with nonparaneoplastic stiff-person syndrome. Because of the muscle stiffness involved it
can be misdiagnosed as catatonia or a dystonic reaction. GABA-enhancing agents, such as benzodiaz-
epines, gabapentin, and baclofen, provide symptomatic relief.
Neurology
B&D Chapter 52G

Question 61. B. Mirtazapine has 5-HT 3 receptor antagonism, which is thought to be responsible for its antinausea
effect. This same receptor antagonism is responsible for the antinausea properties of ondansetron.
Psychopharmacology
K&S Chapter 29

Question 62. D. The clinical manifestations of a medulloblastoma, located in


the fourth ventricle, are the result of increased intracranial
pressure (ICP) and obstructive hydrocephalus. Headache
is the most common initial symptom and usually precedes
diagnosis by 4 to 8 weeks. Intractable nausea and vom-
iting occur frequently, characteristically in the morning.
Personality changes, namely irritability, are an early feature.
Lethargy, diplopia, head tilt, and truncal ataxia are other
features that can help lead to diagnosis. Common signs on
physical examination are papilledema, ataxia, dysmetria,
and cranial nerve involvement. Abducens nerve palsy, sec-
ondary to increased ICP, is a cause of diplopia and head tilt.
Torticollis can be a sign of cerebellar tonsil herniation.
Neurology
B&D Chapter 52E

299

Psychiatry Test Preparation and Review


Manual E-Book
Psychiatry Test Preparation & Review Manual

Question 63. D. In 50% of families in which a child dies, parental divorce follows.
Depressive Disorders
K&S Chapter 8

Question 64. B. For the individual to be rendered unconscious by a traumatic brain lesion, either the reticular activat-
ing system or diffuse injury to both hemispheres must be involved. The functioning of the awake mind
requires the ascending inputs referred to as the reticular activating system, with its way stations in the
brain stem and thalamus, as well as an intact cerebral cortex. Bilateral lesions of the brain stem or the
thalamus produce coma. Very diffuse lesions of the hemispheres produce an “awake” patient who shows
no responsiveness to the environment in a persistent vegetative state. Patients with very slight responses
to environmental stimuli are said to be in a minimally conscious state.
Neurology
B&D Chapter 6

Question 65. B. Neuroimaging of schizophrenics demonstrates increased size of the lateral ventricles. PET scan and func-
tional MRI of schizophrenics will show decreased metabolic activity in the frontal lobes. Know these
facts for your examination.
Laboratory Tests in Psychiatry
K&S Chapter 7

Question 66. A. Symptoms of cervical radiculopathy often appear suddenly. Acute trauma is often the cause of nerve
root contusion or disk herniation that results in cervical radiculopathy. In patients younger than age 45,
disk herniation is likely to be the cause. With increasing age, degenerative changes resulting in neural
foraminal stenosis are more likely to be the cause. Pain is usually in the neck, with radiation to an arm.
Headache may also be a symptom. The C5, C6, and C7 roots are the ones most commonly involved in
cervical spondylosis, because they are at the level of greatest mobility. Lesions at the C4–C5 level affect
the C5 root, causing pain, paresthesias, and sometimes loss of sensation over the shoulder, with weak-
ness of the deltoid, biceps, and brachioradialis muscles. The biceps and supinator reflexes may be lost.
Lesions at the C5–C6 level affect the C6 root and cause paresthesias in the thumb or lateral distal fore-
arm and weakness in the brachioradialis, biceps, or triceps. The biceps and brachioradialis reflexes may
be diminished or inverted. Lesions at the level of C6–C7 affect the C7 root and cause paresthesias, usu-
ally in the index, middle, or ring fingers, and weakness in C7-innervated muscles, such as the triceps and
pronators. The triceps tendon reflex may be diminished. Remember that myelopathy (compromise of the
spinal cord) generally causes hyperreflexia and not diminished or absent reflexes, particularly early on in
the course of the condition.

Typical findings in cervical myelopathy are a combination of leg spasticity, upper extremity weakness or
clumsiness, and sensory changes in the arms, legs, or trunk. Some patients experience leg or trunk pares-
thesia induced by neck flexion (Lhermitte’s sign).
Neurology
B&D Chapter 73

Question 67. C. Narcolepsy consists of irresistible attacks of sleep that occur daily for at least 3 months. They involve
cataplexy (loss of muscle tone during attacks often associated with strong emotion) and REM sleep dur-
ing episodes often leading to hypnopompic or hypnagogic hallucinations and sleep paralysis. Catalepsy
is a condition in which the patients maintain the body position into which they are placed. Similarly,
in waxy flexibility the patients maintain the body position into which they were placed with slight
resistance to movement, giving it a waxy feel. Both catalepsy and waxy flexibility are seen in catatonic
schizophrenia.
Sleep–Wake Disorders
K&S Chapter 16

Question 68. C. Subacute combined degeneration refers to the combination of spinal cord and peripheral nerve pathol-
ogy associated with vitamin B12 (cobalamin) deficiency. Patients often complain of unsteady gait and dis-
tal paresthesias. The neurologic examination may demonstrate evidence of posterior column, pyramidal

300

Psychiatry Test Preparation and Review


Manual E-Book
Test Number Five

tract, and peripheral nerve involvement. Patients also develop a characteristic megaloblastic anemia.
Personality change, cognitive dysfunction, mania, depression, and psychosis have all been reported.
Dementia is often comorbid with cobalamin deficiency, and it is considered one of the correctable
causes of dementia because B12 injections can reverse the cognitive impairment. Cognitive impairment is
more likely to improve when it is mild and of short duration. To confirm diagnosis, a serum B12 level is
checked. In equivocal cases, checking for elevated homocysteine and methylmalonic acid levels corrobo-
rates the diagnosis, as they both tend to be elevated when B12 is low.
Neurology
B&D Chapter 9

Question 69. D. Extensor plantar responses (Babinski’s sign) are indicative of a lesion of the corticospinal tracts, that is,
upper motor neuron involvement. The other four answers are clearly cardinal features of a dementia
of the Alzheimer type. Alzheimer’s dementia does not typically involve lesions to the descending motor
tracts. Vascular dementia can present with aphasia, apraxia, agnosia, or memory loss. Vascular dementia
typically includes a history of clinical strokes with focal neurologic signs and symptoms and “stepwise”
cognitive decline. Epidemiological studies find that the prevalence of vascular dementia ranges from 3%
to 21%. In autopsy studies of late-life dementia, 15% to 20% of the patients are reported to have had
vascular dementia.
Neurology
B&D Chapter 66

Question 70. A. This scan and the question vignette are indicative of a fronto-
temporal dementia (FTD), one of which is Pick’s disease. The
scan demonstrates classic preferential atrophy of the frontal
and temporal lobes with occipitoparietal sparing. This kind
of imaging finding is known as “knife-edge” atrophy of the
frontal and temporal lobes and is characterized histologically
by ballooned cells and intraneuronal inclusions (Pick cells
and Pick bodies). Criteria focus on loss of insight, easy dis-
tractibility, reduced concern or empathy for others, emotional
lability or withdrawal, impulsiveness, poor self-care, perse-
veration, features of the Klüver–Bucy syndrome, and dimin-
ished verbal output. There is a second type of presentation of
FTD that is characterized by early and progressive change in
language with problems of expression of language or severe
naming difficulty, and problems with word meaning. Males
and females are affected equally. Mean duration of illness is
approximately 8 years, with a range from 2 to 15 years. FTD makes up 10% to 15% of the neurodegen-
erative dementias.
Neurology
B&D Chapter 66

Question 71. D. In this situation the patient has been successfully treated with medication for bipolar disorder and has
stopped it due to the influence of her new boyfriend. The best approach is to educate the patient and her
boyfriend regarding the symptoms of bipolar disorder, its treatment, and its course. We should especially
take the opportunity to reeducate the patient, because ultimately she is responsible for the management
of her illness. Those unfamiliar with mental illness often fail to understand its potential effect and fail
to recognize symptoms as they arise. Group therapy and CBT are both legitimate treatments but will
not address the specific problem in this case. Involuntary hospitalization is not warranted, as there is no
indication in the question that the patient is a danger to self or others. Vocational training has no rel-
evance to the situation at all, so is a terrible choice.
Management in Psychiatry
K&S Ch. 8

301

Psychiatry Test Preparation and Review


Manual E-Book
Psychiatry Test Preparation & Review Manual

Question 72. C. CSF findings on their own cannot make or exclude the diagnosis of multiple sclerosis (MS). The CSF is
grossly normal in MS, being clear, colorless, and under normal pressure. Total leukocyte count is normal
in two-thirds of patients. CSF protein (or albumin) level is normal in the majority of patients with MS.
Albumin levels are elevated in only 20% to 30% of patients. A common finding in MS is an elevation of
CSF immunoglobulin levels. The increase is predominantly IgG, but the synthesis of IgA and IgM is also
increased. Oligoclonal bands (OCBs) have been found in 85% to 95% of patients with clinically definite
MS. As such, they would be the most reliable CSF finding. The presence of OCBs in monosymptomatic
patients predicts a significantly higher rate of progression to MS than the absence of bands: 25% versus
9% at 3-year follow-up. The pattern of banding remains relatively consistent in individual patients dur-
ing the disease course, though bands may be added over time. It should be kept in mind however that a
variety of inflammatory CNS diseases can present with oligoclonal bands, so they alone are not sufficient
to make a diagnosis of MS.
Neurology
B&D Chapter 54

Question 73. B. Mirtazapine works via α2 antagonism on the presynaptic neuron. This blocks a feedback loop, causing
more serotonin and norepinephrine to be released into the synaptic cleft.
Psychopharmacology
K&S Chapter 29

Question 74. B. Cluster headache is classified by at least five repeated attacks that are severe unilateral, orbital, supra-
orbital, and/or temporal pain lasting 15 minutes to 3 hours. The headache must be accompanied by
one of the following: ipsilateral conjunctival injection or lacrimation, ipsilateral nasal congestion and/
or rhinorrhea, ipsilateral eyelid edema, ipsilateral forehead and facial swelling, ipsilateral miosis and/or
ptosis, or a sense of restlessness or agitation. Attacks have a frequency from one every other day to eight
per day. Male to female ratio 8:1. It is the only headache syndrome more common in men than women.
Onset typically in the 20s and 30s. Periodicity is a cardinal feature of cluster headache. The first cluster
of attacks tends to last on average for 6 to 12 weeks, and this is followed by a remission period that lasts
from months to years. Headaches tend to occur in spring and fall.

The pain is strictly unilateral and almost always remains on the same side of the head from cluster to
cluster. The pain is felt generally in the retro-orbital area with radiation to the eye, temple, and teeth. It
is often described as being boring or stabbing in nature, like a hot poker going through the eye or pok-
ing the eye. Lying down tends to exacerbate the pain. Other precipitants include alcohol and REM sleep.
Headaches often peak in early morning during REM sleep.

Acute symptomatic therapy is best accomplished by administering oxygen, subcutaneous sumatriptan,


and subcutaneous or intramuscular dihydroergotamine. Oxygen delivered via a nonrebreathing face
mask can be dramatically effective for aborting a cluster attack. Methysergide is used as maintenance
prevention against cluster attacks. Typically, NSAIDs, butalbital combination preparations, and oral nar-
cotics are not particularly effective in aborting cluster attacks.
Neurology
B&D Chapter 69

Question 75. C. Numerous studies have pointed to hostility as increasing risk of cardiac disease in men. In several stud-
ies, hostile men were much more likely to suffer cardiac events. Women did not see the same increase.
Some studies even argue that hostility is a better predictor of coronary heart disease in older men than
a variety of other sociodemographic and physiological risk factors, including smoking, drinking, high
caloric intake, and high levels of LDL cholesterol. Hostility is also linked to poorer outcomes in those
who already have cardiac disease.
Somatic Symptom Disorders
Health Psychology, Vol 21, No 6; Journal of the American College of Cardiology Mar. 2009;
Psychosomatic Medicine May 1983.

302

Psychiatry Test Preparation and Review


Manual E-Book
Test Number Five

Question 76. D. Lacunar infarcts are small ischemic infarctions in the deep regions of the brain or brain stem. Lacunes
usually occur in patients with long-standing arterial hypertension, current cigarette smoking, and diabe-
tes mellitus. The most frequent sites of involvement are the putamen, basis pontis, thalamus, posterior
limb of the internal capsule, and caudate nucleus. Multiple lacunar infarcts are strongly associated with
arterial hypertension and diabetes mellitus. Studies indicate that arterial hypertension leads to micro-
vascular changes characterized by fibrinoid angiopathy, lipohyalinosis, and microaneurysm formation.
Control of hypertension, prevention of microangiopathy, and use of platelet antiaggregants are essential
in the management of patients with lacunar infarcts.
Neurology
B&D Chapter 51A

Question 77. C. With the Dandy–Walker malformation, various parts of the


cerebellum develop abnormally, resulting in malformations
that can be seen on imaging. The central part of the cerebel-
lum (the vermis) is absent or very small and may be abnor-
mally positioned. The right and left sides of the cerebellum
may be small as well. In affected individuals, the fourth ven-
tricle and the posterior fossa are abnormally large. These
abnormalities often result in problems with movement, coor-
dination, intellect, mood, and other neurologic functions.

In the majority of individuals with Dandy–Walker malforma-


tion, signs and symptoms caused by abnormal brain develop-
ment are present at birth or develop within the first year of
life. Some children have hydrocephalus that may cause mac-
rocephaly. Up to half of affected individuals have intellectual
disability that ranges from mild to severe, and those with normal intelligence may have learning disabili-
ties. Children with Dandy–Walker malformation often have delayed development, particularly a delay in
motor skills, such as crawling, walking, and coordinating movements. People with Dandy–Walker mal-
formation may experience muscle stiffness and partial paralysis of the lower limbs (spastic paraplegia),
and they may also have seizures.

Chiari malformations involve a displacement of the lower cerebellum through the foramen magnum that
compresses the spinomedullary junction. The simple form is termed Chiari type I malformation. Type II
involves an additional downward displacement of the lower medulla and is a constant feature of lumbo-
sacral meningomyelocele (sac-like protrusion of spinal cord, nerve roots, and membranes overlying the
sacral spine). Chiari type III malformation is actually a cervical spina bifida with cerebellar encephalo-
cele (sac-like protrusion of brain tissue and membranes through openings in the skull).

Anencephaly is a failure of the entire upper end of the neural tube to form. The fetus will be born lack-
ing much of the brain, or the brain tissue present is consumed by a major, often cavitary malformation.
Almost all of these babies die before birth or within a few days of birth. The prenatal diagnosis of anen-
cephaly is by examination of amniotic fluid for elevation of α-fetoprotein and confirmation is by sono-
graphic imaging as early as 12 weeks’ gestation.
Neurology
B&D Chapter 60

Question 78. A. Prader–Willi syndrome (PWS) is a genetically defined mental retardation syndrome. It is caused by a
deletion on chromosome 15. It is a good example of “genetic imprinting”: if the individual inherits
the deletion from a maternal chromosome 15, the result is Angelman’s syndrome (“happy puppet syn-
drome”); if the mutation is on a paternal chromosome 15, the result is PWS. The mean IQ range in
PWS is 70, with a spectrum ranging from profound mental retardation to average intelligence. Language
function is impaired by oromotor dysfunction. Executive functioning is impaired. There is a high likeli-
hood of autism in PWS patients. Social functioning is often impaired, and patients often have attention
deficit hyperactivity disorder (ADHD). Psychosis and aggressive behaviors can be seen in young adults

303

Psychiatry Test Preparation and Review


Manual E-Book
Psychiatry Test Preparation & Review Manual

with PWS. Hypothalamic dysfunction in PWS results in overeating behavior leading to truncal obesity
in many patients. Patients have hypogonadism and micropenis. Prader–Willi is remembered with the
three Hs (hyperphagia, hypomentia, and hypogonadism). Growth hormone helps increase height, reduce
weight, and improve muscle tone. Bariatric surgery is not helpful. Angelman’s syndrome is typically
maternally inherited. It presents with severe intellectual disability, microcephalus, paroxysms of laugh-
ter, hyperactivity and autistic features, seizures, rigidity, and jerky movements (hence the term “happy
puppet”). The other answer choices to this question are also genetically defined mental retardation syn-
dromes. Explanations of these can be found in other questions in this volume.
Neurodevelopmental Disorders
B&D Chapter 61

Question 79. C. CJD is a rapidly progressive spongiform degeneration of the brain secondary to the accumulation of
misfolded prion proteins. Three other human prion diseases have been described: kuru (in New Guinea),
Gerstmann–Sträussler–Scheinker syndrome, and fatal familial insomnia. Because these diseases are
believed to be caused by a “proteinaceous infectious particle,” they are often referred to as prion dis-
eases. Do read up on these other prion diseases; they do appear once in a while on board-style examina-
tions. CJD affects men and women equally and typically manifests in late middle age. Peak incidence is
in the 55- to 70-year age group. Symptoms can appear gradually over a period of weeks or more sud-
denly. Approximately one-third of patients exhibit mental deterioration, one-third exhibit physical dis-
abilities (especially incoordination), and another third exhibit mixed mental and physical abnormalities.
Memory loss, accompanied by either cerebellar or visual–oculomotor signs, is frequent at the disease
onset. As the illness evolves, these symptoms are joined by pyramidal and extrapyramidal signs and a
variety of involuntary movements, especially myoclonus. A small number of patients present with sleep
and autonomic disorders that mimic familial fatal insomnia. Progressive mental deterioration terminates
in mutism and global dementia. Death commonly occurs within 6 months of disease onset. Three labora-
tory tests are used to aid the diagnosis: EEG, CSF analysis, and brain MRI. In its most pathognomonic
form the EEG reveals periodic sharp-wave complexes. The CSF is analyzed in two ways: for 14-3-3 pro-
tein kinase inhibitor and for tau protein. The presence of elevated levels of either one of these two mark-
ers has a sensitivity and specificity for CJD in the 90% range. Finally, in sporadic disease, the MRI may
reveal a symmetrical or sometimes unilateral hyperintense signal in the basal ganglia. The other distrac-
tors are discussed elsewhere in other questions in this volume. Please do study them as well; they do pop
up on standardized tests a great deal.
Neurology
B&D Chapter 66

Question 80. D. The JC virus is a polyomavirus and is ubiquitous; up to 85% of individuals have antibodies to the
virus by age 9 years. The JC virus is the etiological agent of progressive multifocal leukoencephalopa-
thy (PML). The kidneys are thought to be the site of latent JC viral persistence. Reactivation of the
virus associated with immunosuppression results in hematogenous spread of the virus to the brain. PML
occurs in patients with impaired cell-mediated immunity, such as those with AIDS, chronic lymphocytic
leukemia, Hodgkin’s disease, other lymphomas, sarcoidosis, organ transplantation, and lupus. The JC
virus preferentially infects myelin-producing oligodendrocytes, resulting in cell lysis and demyelination.
Focal neurologic deficits, seizure activity, and cognitive impairment characterized by memory impair-
ment, psychomotor retardation, and inattentiveness characterize the clinical presentation of PML. There
is no proven therapy for PML. The average length of survival from the onset of symptoms to death is 1
to 4 months.

The etiologic agent responsible for CNS lymphoma is Epstein–Barr virus (EBV). EBV is detectable from
a majority of primary CNS lymphomas in AIDS patients and to a lesser degree from primary CNS lym-
phomas in immunocompetent individuals. The clinical presentation is that of progressive personality
changes, seizures, and signs of increased intracranial pressure. The other answer choices to this question
are discussed in greater detail elsewhere in this volume. Do study all of these CNS manifestations of
HIV, because you are likely to come upon these questions on your board-style examination.
Neurology
B&D Chapter 53B

304

Psychiatry Test Preparation and Review


Manual E-Book
Test Number Five

Question 81. B. The major morbidity associated with idiopathic intracranial hypertension (aka pseudotumor cerebri) is
visual loss related to optic nerve dysfunction. Patients are usually young, obese females with menstrual
irregularities who may complain of headache, transient visual obscurations (lasting seconds), or double
vision. Almost uniformly, patients have papilledema. Typically, visual acuity and color are preserved, but
visual field defects are present in more than 90% of patients. LP shows increased pressure (>200 mm
H2O) and low CSF protein (40 mg%). CT and MRI show “swollen brain” and “slit-like” ventricles.
MRV (magnetic resonance venogram) may show thrombosis of intracranial venous sinus. Treatment
involves reducing ICP. Acetazolamide is an inhibitor of carbonic anhydrase that lowers CSF production
and pressure. Some patients undergo repeated lumbar punctures with CSF removal to maintain lower
CSF pressures. If vision is threatened, surgery is usually contemplated. Lumboperitoneal shunting is the
procedure of choice.
Neurology
B&D Chapter 59

Question 82. A. Here is yet another look at Piaget’s theory of development. This material comes up very frequently on
standardized tests, so it is imperative to study and memorize. The developmental stage of concrete oper-
ations refers to ages 7 to 11 years. During this period, children are able to act on real, concrete, and
perceivable objects and events that surround them. Egocentric thought (characteristic of the stage of
preoperational thought) is replaced by operational thought. Children gain the ability to see things from
other people’s perspectives. Conservation and reversibility are the two abilities that children demonstrate
in the stage of concrete operations. Conservation refers to the ability to recognize that although the
shape of objects may change, the objects still maintain or conserve characteristics that enable them to
be recognized as the same. Reversibility is the ability to understand the relation between things, that one
thing can turn into another and then back again.
Human Development
K&S Chapter 2

Question 83. C. Men do not lose their sex drive as they age, unless there are specific physical and/or psychological fac-
tors underlying this problem. As is the case with women, lack of desire in men can be of either physical
or psychological origin.

Physical causes:
• A lcoholism: quite common. Tobacco use: frequent, and its role is underestimated in loss of sexual
drive
• Abuse of drugs, such as cocaine
• Obesity: quite common; slimming down will often help
• Anemia: unusual, unless the man has been bleeding for any reason
• Hyperprolactinemia
• Prescribed drugs: particularly Proscar, a tablet used for prostate problems
• Low testosterone level: contrary to what many people think, this is rare, except in cases in which
some injury or illness has affected the testicles
• Any major disease such as diabetes.
Psychological causes:
•  epression: very common
D
• Stress and overwork
• Hang-ups from childhood
• Latent homosexuality
• Serious relationship problems with the partner.
Sexual desire has been described as a “longing for sexual union.” It associates with certain behaviors
that are more linked to arousal and states of fear, concern, and enhanced attention to others and sexual
cue displays, such as lip biting and touching. In keeping with the correlation between sexual desire and
arousal, sexual desire is mediated by gonadal estrogens and androgens.

305

Psychiatry Test Preparation and Review


Manual E-Book
Psychiatry Test Preparation & Review Manual

Enhanced focus, concern, and attention toward the desired other have been associated not only with
increased arousal by means of testosterone but also with elevated concentrations of central dopamine
and norepinephrine and decreased levels of central serotonin. Other forms of physiological arousal asso-
ciated with enhanced levels of dopamine include increased energy, exhilaration, euphoria, sleeplessness,
loss of appetite, trembling, pounding heartbeat, and accelerated breathing. This same increased arousal
is also a feature of attraction and is the suggested cause of feelings of exhilaration, ecstasy, intrusive
thinking about the love object, regarding the desired other as unique, and a craving for emotional union
with this partner or potential partner.
Sexual Disorders
K&S Chapter 17

Question 84. D. Attachment theory was first set forth by John Bowlby, a British psychoanalyst. Attachment is defined
as the emotional tone between children and caregivers, manifested by the infant’s seeking and clinging
closely to the caregiver, such as the mother. It is well accepted that a person’s psychological health and
sense of well-being depend to a great extent on the quality of relationships and attachments to others,
particularly those attachments that were laid down in childhood with caregivers. People with a secure
attachment style are highly invested in relationships and tend to behave without much possessiveness or
fear of rejection.
Human Development
K&S Chapter 2

Question 85. E. Heinz Kohut was best known for his development of self psychology. Kohut rejected Freud’s structural
theory of the id, ego, and superego. He then developed his ideas around what he called the tripartite
(three-part) self. According to Kohut, this three-part self can develop only when the needs of one’s “self
states,” including one’s sense of worth and well-being, are met in relationships with others. In contrast
to traditional psychoanalysis, which focuses on drives (instinctual motivations of sex and aggression),
internal conflicts, and fantasies, self psychology thus placed a great deal of emphasis on the vicissitudes
of relationships. Kohut demonstrated his interest in how we develop our “sense of self,” using narcissism
as a model. A narcissistic person will be able to suppress feelings of low self-esteem. By talking highly of
him- or herself, the person can eliminate his or her sense of worthlessness.

Kohut initially proposed a bipolar self, composed of two systems of narcissistic perfection: (1) a system
of ambitions and (2) a system of ideals. Kohut called the pole of ambitions the narcissistic self (later, the
grandiose self), whereas the pole of ideals was designated the idealized parental imago. According to
Kohut, these poles of the self represented natural progressions in the psychic life of infants and toddlers.

Kohut argued that when the child’s ambitions and exhibitionistic strivings were chronically frustrated,
arrests in the grandiose self led to the preservation of a false, expansive sense of self that could manifest
outwardly in the visible grandiosity of the frank narcissist or remain hidden from view, unless discov-
ered in a narcissistic therapeutic transference (or self-object transference) that would expose these primi-
tive grandiose fantasies and strivings. Kohut termed this form of transference a mirror transference. In
this transference, the strivings of the grandiose self are mobilized, and the patient attempts to use the
therapist to gratify these strivings (this is essentially what is displayed between child and mother in this
examination question).

Kohut proposed that arrests in the pole of ideals occurred when the child suffered chronic and excessive
disappointment over the failings of early idealized figures. Deficits in the pole of ideals were associated
with the development of an idealizing transference to the therapist, who becomes associated with the
patient’s primitive fantasies of omnipotent parental perfection.

Kohut believed that narcissistic injuries were inevitable and, in any case, necessary to temper ambitions
and ideals with realism through the experience of more manageable frustrations and disappointments.
It was the chronicity and lack of recovery from these injuries (arising from a number of possible causes)
that he regarded as central to the preservation of primitive self systems untempered by realism.
Human Development
K&S Chapter 4

306

Psychiatry Test Preparation and Review


Manual E-Book
Test Number Five

Question 86. C. Margaret Mahler was a pioneer in object-relations psychoanalysis. Her six stages of separation–­
individuation come up on almost every standardized examination in psychiatry. Here is the list in correct
order:

1. 
Normal autism (birth to 2 months): Sleep periods predominate over arousal periods. This stage
hearkens back to fetal and intrauterine life for the infant.
2. 
Symbiosis (2 to 5 months): Mother–infant is perceived as a single fused entity. Developing perceptual
abilities gradually enable infants to distinguish the inner from the outer world.
3. 
Differentiation (5 to 10 months): Distinctness from the mother is appreciated. Progressive neurologic
development and increased alertness draw the infant’s attention away from the self to the outer world.
4. 
Practicing (10 to 18 months): The ability to move autonomously increases the child’s exploration of
the outer world.
5. 
Rapprochement (18 to 24 months): Children move away from their mothers and come back for reas-
surance. As they slowly realize their helplessness and dependence, the need for independence alter-
nates with the need for closeness.
6. 
Object constancy (2 to 5 years): Children gradually comprehend and are reassured by the perma-
nence of mother and other important people, even when not in their presence.
Human Development
K&S Chapter 31

Question 87. B. Memory can be divided into three system categories: episodic, semantic, and procedural. Episodic mem-
ory is mediated by the medial temporal lobes, anterior thalamic nuclei, mammillary bodies, fornix, and
prefrontal cortex. It involves explicit, declarative awareness. An example of episodic memory would be
remembering what you ate for a meal a day earlier or what you did on your last vacation. Semantic
memory is mediated by the inferolateral temporal lobes. It too involves explicit, declarative awareness.
Examples of semantic memory include knowing who the first president of the United States was or
knowing the difference between a bus and a train. Procedural memory is mediated by the basal ganglia,
cerebellum, and supplementary motor area. It involves either implicit or explicit, nondeclarative aware-
ness. Examples of procedural memory include operating a motor vehicle (explicit) or learning how to
type your telephone number without thinking about it (implicit).
Neurocognitive Disorders
K&S Chapter 1

Question 88. A. Burrhus Frederic “B.F.” Skinner was an American behaviorist, author, inventor, social philosopher, and
poet. Skinner invented the operant conditioning chamber, innovated his own philosophy of science
called radical behaviorism, and founded his own school of experimental research psychology: the experi-
mental analysis of behavior. Skinner’s theory of learning and behavior is known as operant conditioning.
In operant conditioning, an animal is active and behaves in a way that produces a reward. Thus, learning
occurs as a consequence of action. Gambling behavior occurs in response to the reinforcement schedule
known as a variable-interval schedule. Reinforcement in this schedule occurs after variable intervals (of
time). As such, the response rate does not change between reinforcements. The animal responds at a
steady rate to get the reward when it is available.
Human Development
K&S Chapter 2

Question 89. B. Selective perception (or attention) is a broad term to identify the tendency all people exhibit to “see
things” based on their particular frame of reference. Selective perception may refer to any number of
cognitive biases in psychology related to the way expectations affect perception. For instance, several
studies have shown that students who were told they were consuming alcoholic beverages (which in fact
were nonalcoholic) perceived themselves as being “drunk,” exhibited fewer physiological symptoms of
social stress, and drove a simulated car in a manner similar to that of other subjects who had actually
consumed alcohol. The result is somewhat similar to the placebo effect. A cognitive bias is a pattern of
deviation in judgment that occurs in particular situations leading to perceptual distortion, inaccurate
judgment, illogical interpretation, or what is broadly called irrationality.

Learned helplessness means a condition of a human or an animal that has learned to behave help-
lessly, even when the opportunity is restored for it to help itself by avoiding an unpleasant or harmful

307

Psychiatry Test Preparation and Review


Manual E-Book
Psychiatry Test Preparation & Review Manual

circumstance to which it has been subjected. Learned helplessness theory is the view that clinical depres-
sion and related mental illnesses may result from a perceived absence of control over the outcome of
a situation. Organisms that have been ineffective and less sensitive in determining the consequences of
their behavior are defined as having acquired learned helplessness. Martin Seligman’s experiments and
theory of learned helplessness began as an extension of his interest in depression. In the learned helpless-
ness experiment, an animal is repeatedly hurt by an adverse stimulus that it cannot escape. Eventually
the animal will stop trying to avoid the pain and behave as if it is utterly helpless to change the situation.
Finally, when opportunities to escape are presented, this learned helplessness prevents any action. The
only coping mechanism the animal uses is to be stoic and put up with the discomfort, not expending
energy getting worked up about the adverse stimulus.

Cognitive distortions are exaggerated and irrational thoughts identified in cognitive therapy and its vari-
ants, and they in theory perpetuate some psychological disorders. Eliminating these distortions and neg-
ative thoughts is said to improve mood and discourage maladies such as depression and chronic anxiety.
The process of learning to refute these distortions is called cognitive restructuring. Magnifying or mini-
mizing a memory or situation such that it no longer corresponds to objective reality is one of many types
of cognitive distortion. It is common enough in the normal population to popularize idioms such as
“make a mountain out of a molehill.” In depressed clients, often the positive characteristics of other
people are exaggerated and negative characteristics are understated. There is one subtype of magnifica-
tion, catastrophizing, which is the inability to foresee anything other than the worst possible outcome,
however unlikely, or experiencing a situation as unbearable or impossible when it is just uncomfortable.
Psychological Theory and Psychometric Testing
K&S Chapters 2 and 28

Question 90. D. Sensitivity and specificity are statistical measures of the performance of a binary classification test, also
known in statistics as classification function. Sensitivity (also called recall rate in some fields) measures
the proportion of actual positives that are correctly identified as such (e.g., the percentage of sick people
who are correctly identified as having the condition). Specificity measures the proportion of negatives
that are correctly identified (e.g., the percentage of healthy people who are correctly identified as not
having the condition). These two measures are closely related to the concepts of type I and type II errors.
A perfect predictor would be described as having 100% sensitivity (i.e., predicts all people from the sick
group as sick) and 100% specificity (i.e., does not predict anyone from the healthy group as sick); how-
ever, theoretically any predictor will possess a minimum error bound known as the Bayes error rate. For
any test, there is usually a trade-off between the measures. The sensitivity of a test is the proportion of
people who have the disease who test positive for it. It is calculated by taking the number of true posi-
tives and dividing by the number of true positives plus the number of false negatives. It is the probability
of a positive test given that the patient is ill. If a test has high sensitivity, then a negative result would
suggest the absence of disease.

Specificity relates to the ability of the test to identify negative results. Consider the example of the medi-
cal test used to identify a disease. The specificity of a test is defined as the proportion of patients who do
not have the disease who will test negative for it. Specificity is calculated by taking the number of true
negatives and dividing by the number of true negatives plus the number of false positives. It is the prob-
ability of a negative test given that the patient is well. If a test has high specificity, a positive result from
the test means a high probability of the presence of disease.
Statistics
K&S Chapter 5

Question 91. A. Atypical antipsychotics are the most thoroughly studied class of medications for patients with demen-
tia who are agitated, and they are the most common drugs used in clinical practice. They are better
tolerated than typical neuroleptic agents, with less risk of causing EPS. In the absence of contraindi-
cations, such as serious extrapyramidal dysfunction (e.g., EPS, parkinsonism), an atypical neuroleptic
agent should be initiated at the lowest effective dosage and titrated weekly. Tremor, rigidity, dystonia,
and dyskinesia are identified in a significant number of patients at baseline and may be exacerbated
by the use of atypical antipsychotics, particularly when these agents are taken at higher dosages.
Physicians must use caution when increasing dosages and observe the patient closely for the emergence

308

Psychiatry Test Preparation and Review


Manual E-Book
Test Number Five

of EPS. Although the use of the conventional antipsychotic agent haloperidol (Haldol) is discouraged
in long-term care facilities, it is widely used in the management of delirium and acute agitation in
other settings. Haloperidol has been used with acceptable side effects in the management of behavior
disorders of dementia. If used, it should be prescribed at low dosages and for short periods (typically
days), after which the patient should be switched to another agent, such as an atypical antipsychotic.
Benzodiazepines should not be considered first-line therapy for management of chronic behavior dis-
orders of dementia, even in patients with prominent anxiety. Chronic benzodiazepine use may worsen
the behavior abnormality because of the amnestic and disinhibitory effects of these drugs. In clinical
practice, benzodiazepine use should be limited to management of acute symptoms that are unresponsive
to redirection or other agents. Anticonvulsant agents typically are used when psychotic behaviors result
in aggressive behavior. Increasing evidence supports the use of divalproex (Depakote) or carbamazepine
(Tegretol). These drugs are recommended as second-line agents in patients with inadequate response to
antipsychotic agents. TCAs should be avoided in demented patients, and in particular those with agita-
tion, because of these drugs’ anticholinergic effects, which can worsen both the agitation and the under-
lying dementia.
Psychopharmacology
K&S Chapters 21 and 29

Question 92. C. Psychiatrists are held to ethical principles and standards by the American Psychiatric Association that
are higher than those other physicians are held to by the American Medical Association. On occasion,
psychiatrists are asked to render an opinion about individuals who are public figures or individuals who
have revealed information about themselves through the public media. Psychiatrists are permitted to
share their expertise about psychiatric issues in general with the public. However, it is unethical for psy-
chiatrists to offer a professional opinion about a specific individual unless they have examined the indi-
vidual and been granted proper authorization for such a statement.
Ethics
K&S Chapter 36

Question 93. C. Transcranial magnetic stimulation (TMS) is a noninvasive method to cause depolarization or hyperpo-
larization in the neurons of the brain. TMS uses electromagnetic induction to induce weak electric cur-
rents using a rapidly changing magnetic field; this can cause activity in specific or general parts of the
brain with minimal discomfort, allowing the brain’s functioning and interconnections to be studied. A
variant of TMS, repetitive TMS (rTMS), has been tested as a treatment tool for various neurologic and
psychiatric disorders, including migraines, strokes, PD, dystonia, tinnitus, depression, and auditory hal-
lucinations. The treatment protocol involves the application of electromagnetic stimulation to the stan-
dardized treatment location, which is over the left prefrontal cortex, determined by moving the TMS coil
5 cm anterior to the motor threshold location along a left superior oblique plane with a rotation point
about the tip of the patient’s nose. Treatment for depression involves 20 to 30 sessions of about 40 min-
utes each over a 4- to 6-week period.
Diagnostic and Treatment Procedures in Psychiatry
K&S Chapter 30

Question 94. A. NeuroStar TMS Therapy is indicated for the treatment of major depressive disorder in adult patients
who have failed to achieve satisfactory improvement from one prior antidepressant medication at or
above the minimal effective dose and duration in the current episode. NeuroStar TMS Therapy nonin-
vasively stimulates the left prefrontal cortex of the brain to treat the symptoms of major depression. It
requires 4 to 6 weeks of treatment. TMS is a noninvasive method to cause depolarization or hyperpolar-
ization in the neurons of the brain. TMS uses electromagnetic induction to induce weak electric currents
using a rapidly changing magnetic field; this can cause activity in specific or general parts of the brain
with minimal discomfort, allowing the functioning and interconnections of the brain to be studied. A
variant of TMS, rTMS, has been tested as a treatment tool for various neurologic and psychiatric dis-
orders, including migraines, strokes, PD, dystonia, tinnitus, depression, and auditory hallucinations. In
2008, Neuronetics, Inc., a privately held company in the United States, received FDA clearance for its
NeuroStar TMS Therapy device for the in-office treatment of major depressive disorder.
Diagnostic and Treatment Procedures in Psychiatry
K&S Chapter 30

309

Psychiatry Test Preparation and Review


Manual E-Book
Psychiatry Test Preparation & Review Manual

Question 95. C. This patient has acute stress disorder. Acute stress disorder lasts from 2 days to 4 weeks, whereas PTSD
must last for 4 weeks or more. Major symptom clusters for both disorders include intrusion symptoms,
avoidance symptoms, negative alterations in cognition and mood, and alterations in arousal and reactivity.
Anxiety Disorders
K&S Chapter 11

Question 96. B. All antipsychotics work on the dopamine system, but all vary in regard to the affinity for the dopamine
receptors. There are five types of dopamine receptors in humans. There are the “D1-like” group, which
are types 1 and 5, which are similar in structure and drug sensitivity. The “D2-like” group includes
dopamine receptors 2, 3, and 4, and they have a very similar structure but very different sensitivities to
antipsychotic drugs.

The D1-like receptors have been found not to be clinically relevant in therapeutic action.

It has been shown that D2 receptor blockade is necessary for antipsychotic action. All antipsychotics
block D2 receptors to some degree, but the affinity of the antipsychotics varies from drug to drug, and it
has been hypothesized that it is the varying in affinities that causes a change in effectiveness.

One theory for how atypicals work is the “fast-off” theory. This theory of antipsychotic action is that
AAPs have low affinities for the D2 receptor and bind only loosely to the receptor and are rapidly
released. In fact, the AAPs bind more loosely to the D2 receptor than dopamine itself. The AAPs effec-
tively interfere with the phasic release of endogenous dopamine. The AAPs transiently bind and rapidly
dissociate from the D2 receptor to allow normal dopamine transmission. It is this transient binding that
keeps prolactin levels normal, spares cognition, and obviates EPS.
Psychopharmacology
K&S Chapter 29

Question 97. B. Conversion disorder (functional neurological symptom disorder) is highest among rural populations;
those with little education, low intelligence, and low socioeconomic status; and military personnel who
have had combat exposure.
Somatic Symptom Disorders
K&S Chapter 13

Question 98. D. Serotonin is involved in the mechanism of action of two major substances of abuse: LSD and MDMA
(ecstasy). The serotonin system is the major site of action of LSD, but exactly how it exerts its hallucinogenic
effects is not well understood. MDMA has dual effects: blocking the reuptake of serotonin and inducing
the massive release of the serotonin contents of serotonergic neurons. In animals, it is well understood that
MDMA produces selective, long-lasting damage to serotonergic nerve terminals. Users of MDMA show dif-
ferences in neuroendocrine responses to serotonergic probes, and studies of former MDMA users show global
and regional decreases in serotonin transporter binding, as measured by positron emission tomography.
Substance Abuse and Addictive Disorders
K&S Chapter 20

Question 99. C. The MMPI has several validity and clinical scales. There are three validity scales, the lie scale (L scale),
the infrequency scale (F scale), and the suppressor scale (K scale). The infrequency scale is useful in iden-
tifying malingering, illiteracy, confusion, psychosis, and panic. The lie scale focuses on socially desirable
behaviors that are rarely practiced to test if the patient is being honest in answering the questions. The
suppressor scale is used to decrease false positives and false negatives. There are 10 clinical scales, which
focus on specific symptom clusters.
Psychological Theory and Psychometric Testing
K&S Chapter 5

Question 100. C. Thyroid hormone can be used in psychiatry on its own or as an augmenting agent for depressive or
bipolar disorder. Liothyronine (Cytomel) is the synthetic oral replacement of endogenous T3 (triiodothy-
ronine). Several controlled trials have demonstrated that liothyronine can convert about 50% of antide-
pressant nonresponders into responders. The dosage of liothyronine is 12.5 to 25 μg a day added to the

310

Psychiatry Test Preparation and Review


Manual E-Book
Test Number Five

patient’s antidepressant regimen. Adverse events occur infrequently with liothyronine when given at this
dosage. The most common adverse effects are weight loss, palpitations, transient headache, abdominal
cramps, diarrhea, sweating, tachycardia, and increased blood pressure. Thyroid hormones should not
be taken by patients with cardiac disease, angina, or hypertension. The hormones are contraindicated in
thyrotoxicosis and uncorrected adrenal insufficiency and in patients with acute myocardial infarctions.
Thyroid hormones can potentiate the effects of warfarin. They can increase the insulin requirements of
diabetic patients and the digitalis requirements of patients with cardiac disease. Coadministration of thy-
roid hormones with SSRIs, tricyclics, lithium, or carbamazepine can mildly lower serum thyroxine and
raise serum thyrotropin levels. Thus, close serum monitoring is warranted in these patients, who may
require an increase in dosage of or initiation of thyroid hormone supplementation.
Psychopharmacology
K&S Chapter 29

Question 101. B. Eszopiclone is probably best known to most of us as Lunesta. It is a non-benzodiazepine hypnotic medi-
cation. It is indicated only for sleep (not depression or anything else, for that matter). It does not lead
to tolerance over time. Both zolpidem (Ambien) and eszopiclone can lead to hallucinations, sleep walk-
ing, or other abnormal behaviors. So given this patient’s sleep walking, eszopiclone may not be the best
alternative. As for choice C, the sleep medication famous for being a melatonin agonist is ramelteon
(Rozerem). I’m sure you will come across it again elsewhere in this book!
Psychopharmacology
K&S Chapter 29

Question 102. D. GGT is the most sensitive marker for alcohol abuse. In can be elevated in as many as three-quarters of
patients with alcohol dependence. It can also be increased by obesity, fatty liver, and medications, as well
as other types of liver disease. It usually takes up to 8 weeks for the GGT to return to normal after ces-
sation of alcohol consumption.
Laboratory Tests in Psychiatry
K&S Chapter 20

Question 103. C. You cannot lie to the insurance company. That would be simply unethical. The important therapeutic
work to do is to understand the request and why the patient is making it and help him explore his feel-
ings about it. Giving a different false diagnosis is no more ethical than calling it an adjustment disorder,
and kicking the patient out of your office is abdication of your responsibility as a therapist to help the
patient cope with his illness.
Ethics
K&S Chapter 36

Question 104. C. Adrenal insufficiency (aka Addison’s disease) is a condition in which there is decreased production of min-
eralocorticoids, glucocorticoids, and sex hormones by the adrenal gland. There are primary, secondary, and
tertiary causes. Primary comes from direct damage to the adrenal gland. Secondary comes from pituitary
disease. Tertiary comes from malfunction of the hypothalamus. This condition is fair game for a psychiatry
examination, because it can present with psychiatric symptoms. Most commonly these include depression,
apathy, and irritability, but psychosis and delirium are also possible. Mania is not a common presentation.
Patients may also experience fatigue, weight loss, hyperpigmentation, hypotension, nausea, vomiting, salt
craving, dizziness, and joint and muscle pain. Laboratory work will show hyponatremia and hyperkalemia.
The diagnosis is made when finding low serum cortisol. There will be increased adrenocorticotropic hor-
mone (ACTH) in primary adrenal insufficiency and a poor cortisol response to an ACTH stimulation test.
Treatment consists of intravenous (IV) hydrocortisone in acute cases and treatment with prednisone or oral
hydrocortisone in chronic cases. Dexamethasone can also be given.
Somatic Symptom Disorders
K&S Chapter 13

Question 105. A. When lamotrigine and valproic acid are given together, lamotrigine levels are elevated, and valproic acid
levels are decreased. When lamotrigine and carbamazepine are given together, lamotrigine levels are

311

Psychiatry Test Preparation and Review


Manual E-Book
Psychiatry Test Preparation & Review Manual

decreased. Valproic acid will increase carbamazepine levels. Because lithium is cleared renally, it will not
affect the levels of lamotrigine, valproic acid, or carbamazepine.
Psychopharmacology
K&S Chapter 29

Question 106. D. This question touches on the neurochemical changes that we associate with aggression. In aggressive
patients we would expect to find increased dopamine, decreased serotonin, decreased GABA, increased
testosterone, and increased acetylcholine.
Basic Neuroscience
K&S Chapter 1

Question 107. C. This question tests your knowledge of motor development during infancy. Choice C is incorrect because
the grasp and tonic neck reflexes begin to recede between 2 and 6 months. All other choices are correct
and are good markers to keep in mind to measure normal development. Motor development should be
looked at in conjunction with cognitive and social development when evaluating a young child for psy-
chiatric reasons.
Human Development
K&S Chapter 2

Question 108. D. Tardive dyskinesia (TD) is a delayed effect of antipsychotics. It is rarely seen until after 6 months of
treatment. The disorder manifests as abnormal, involuntary, irregular choreoathetoid movements of the
muscles of the trunk, limbs, and head. Perioral movements are the most common and manifest as dart-
ing, twisting, and protruding movements of the tongue, as well as chewing, lateral jaw movements, lip
puckering, and facial grimacing. In severe cases, torticollis, retrocollis, pelvic thrusting, and trunk twist-
ing can be seen. Dyskinesia disappears during sleep and is exacerbated by stress and anxiety. TD devel-
ops in about 10% to 20% of patients treated for more than a year. About 20% to 40% of patients
with long-term psychiatric hospitalization have TD. Women are more likely to develop TD than men.
Children, patients over 50 years of age, and those with brain damage or mood disorders are also at
greater risk. Between 5% and 40% of all cases of TD remit, and between 50% and 90% of all mild
cases of TD remit. TD is less likely to remit in elderly patients than in young patients, however.
Management in Psychiatry
K&S Chapter 29

Question 109. A. This question tests some important facts about commonly used benzodiazepines. Let’s review the
data. Clonazepam has a rapid rate of absorption, has a half-life of 34 hours, and is considered long-
acting. Alprazolam has a medium rate of absorption, has a half-life of 12 hours, and is considered short-
acting. Diazepam has a rapid rate of absorption, has a half-life of 100 hours, and is considered long-acting.
Lorazepam has a medium rate of absorption, has a half-life of 15 hours, and is considered short-acting.
Psychopharmacology
K&S Chapter 29

Question 110. D. For most anxiety disorders, the rates are higher for women than for men.

The only anxiety disorder with equal rates between men and women is obsessive–compulsive disorder.
Anxiety Disorders
K&S Chapter 9

Question 111. E. Appropriate monitoring of WBC count for patients on clozapine is as follows. First get a baseline WBC
before the drug is started. Then monitor WBC weekly for the first 6 months, then biweekly for the next
6 months, and then monthly thereafter. After discontinuation of clozapine, monitor weekly WBC for 4
consecutive weeks.
Laboratory Tests in Psychiatry
K&S Chapter 29

312

Psychiatry Test Preparation and Review


Manual E-Book
Test Number Five

Question 112. D. Common side effects of lithium include sedation, confusion, tremor, hair loss, nephrogenic diabetes
insipidus, polyuria, polydipsia, acne, weight gain, nausea, diarrhea, and hypothyroidism. Lithium tox-
icity can lead to coma, seizures, and death. Neural tube defects are caused in fetuses when the mother
takes Depakote. When pregnant women take lithium, it causes Ebstein’s anomaly, which is a cardiac
malformation of the tricuspid valve. Risk of Ebstein’s anomaly is greatest when lithium is taken during
the first trimester.
Psychopharmacology
K&S Chapter 29

Question 113. E. This question looks at management of alcohol withdrawal. During alcohol withdrawal it is preferable to
use long-acting benzodiazepines such as chlordiazepoxide. As such, alprazolam, which is short-acting, is
not a good choice. It would also have to be dosed more frequently than twice a day to actually cover the
patient for 24 hours. Intravenous thiamine and dextrose should be given for the first 3 days, and then
switch to oral thiamine. Some argue that after the first IV dose, the doses for the remainder of the 3 days
can be given intramuscularly and then switched to orally. Regardless of which method you advocate,
giving the medications IV for 9 days is clearly wrong. Disulfiram is an aversive treatment used to stop
a patient who is currently detoxed and sober from drinking again. It will make the patient sick if he or
she has a drink while on the medication. It is not used to manage withdrawal. In a hepatically impaired
patient one would choose lorazepam over clonazepam. This can be remembered by the phrase “tolerated
by our liver,” or TOL, which stands for temazepam, oxazepam, lorazepam—the three benzodiazepines
safe to use in hepatic impairment. When using lorazepam for alcohol withdrawal, it should be dosed
four times a day for even coverage and then tapered off over 3 days.
Substance Abuse and Addictive Disorders
K&S Chapter 20

Question 114. C. Acute dystonia is a rapid-onset spastic contraction of discrete muscle groups, such as the neck, eyes,
back, or tongue. Akathisia is a sensation of restlessness and an irresistible urge to move parts of the
body. Tardive dyskinesia is a choreoathetoid movement of the tongue, mouth, face, extremities, or trunk
that is involuntary and irregular. Blepharospasm is an involuntary spasm of the muscles surrounding the
eye. Tardive dystonia is a slow, sustained twisting movement of the limbs, trunk, and neck. It usually
occurs late in onset after treatment with antipsychotics.
Psychopharmacology
K&S Chapter 20

Question 115. D. All choices listed are inducers of CYP 3A4, except fluoxetine, which is an inhibitor. Other inhibitors
include calcium channel blockers, cimetidine, grapefruit juice, and antifungals.
Psychopharmacology
K&S Chapter 29

Question 116. C. Asking the patient to consent to treatment is an example of respecting a patient’s autonomy. Autonomy
is the belief that patients have a right to control what happens to their own bodies and make decisions
freely and without coercion. All of the other choices are legal or ethical terms, which are the subjects of
their own questions throughout this text. Pay attention to them the next time you see them.
Ethics
K&S Chapter 36

Question 117. B. On average, depressed patients require 6 to 12 ECT treatments to treat depression. Some cases may
require as many as 30 treatments, but these are the exception rather than the rule. Treatments are often
given 3 times per week, with one seizure per treatment. The use of more than one seizure per episode has
no proven advantages. The preference is for unilateral electrode placement, because it lessens memory
impairment from the procedure. If a patient fails to improve after six unilateral treatments, then bilateral
electrode placement should be considered.
Diagnostic and Treatment Procedures in Psychiatry
K&S Chapter 29

313

Psychiatry Test Preparation and Review


Manual E-Book
Psychiatry Test Preparation & Review Manual

Question 118. C. The SCID does not include functional impairment. However, functional impairment is covered in the
Schedule for Clinical Assessment in Schedule Neuropsychiatry (SCAN) , and the SCAN is thought to
give a broader assessment of psychosocial function than the SCID. The SCID covers the following topics:
general overview (demographics and medical, psychiatric services, and medication use histories), mood
episodes, psychotic symptoms, psychotic disorders differential, mood disorders differential, substance
use, anxiety disorders, somatic disorders, eating disorders, and adjustment disorders.
Diagnostic and Treatment Procedures in Psychiatry
K&S Chapter 5

Question 119. D. Self-disclosure by the therapist is acceptable only when it is done solely for the benefit of the patient.
It must be the patient’s needs that drive the disclosure. Therapists must be very careful not to disclose
things for their own benefit. Whether the disclosures are true is irrelevant. The focus of therapy should
always be on the patient. That’s what matters.
Psychotherapy
K&S Chapter 5

Question 120. D. To boil this question down to its most basic facts, you have an elderly patient who is looking delirious
and is on carbamazepine. Though there are a myriad of possible explanations for why an elderly patient
can be delirious, the astute psychiatrist knows that carbamazepine has a vasopressin-like effect, which
can lead to hyponatremia. Therefore one of the first things you would want to look for in a confused
patient on carbamazepine is the basic metabolic panel or, more specifically, the sodium level.
Psychopharmacology
K&S Chapter 29

Question 121. C. Of the medications listed, all are metabolized by the liver except paliperidone (Invega). Paliperidone is
80% excreted through the kidney, so you must be careful in using it in patients with renal impairment. It
is a good choice for patients with hepatic impairment, however. Paliperidone is the active metabolite of
risperidone. Risperidone is metabolized through the liver.
Psychopharmacology
K&S Chapter 29

Question 122. A. Validity is the degree to which an instrument measures what it is intended to measure. There are different
types of validity. Face validity means a diagnosis is based on a general consensus among experienced clini-
cians and researchers. Descriptive validity means that a diagnosis is based on characteristic features that
distinguish it from other disorders. Predictive validity means that a diagnosis will allow clinicians to accu-
rately predict treatment response and clinical course. Construct validity means that a diagnosis is based on
an understanding of the underlying pathophysiology. Positive predictive power is the ability of a positive
test result to predict disease. It is calculated as true positives divided by true positives plus false positives.
Statistics
K&S Chapter 5

Question 123. E. Desvenlafaxine extended-release tablets are approved for treatment of MDD in adults. Desvenlafaxine
is a serotonin and norepinephrine reuptake inhibitor (SNRI). The usual dosage is 50 mg by mouth once
a day. Care must be taken with the dose in renally compromised patients and in the elderly. Side effects
include orthostasis, hyponatremia (especially in the elderly), and hypertension, much like venlafaxine.
There is a black box warning to monitor for suicidality, especially in children and teens, as there is with
all of the SSRIs and SNRIs.
Psychopharmacology
Physicians Desk Reference, 64th edition, 2010, page 3564

Question 124. D. Muscle fasciculations would not be expected to be seen with mild to moderate lithium toxicity. Let’s review
severity and symptoms of lithium toxicity. Mild to moderate lithium toxicity is 1.5 to 2 mEq/L. Symptoms
include vomiting, abdominal pain, ataxia, dizziness, slurred speech, dry mouth, and nystagmus. Moderate
to severe lithium toxicity is 2.0 to 2.5 mEq/L. Symptoms include anorexia, muscle fasciculations, clonic

314

Psychiatry Test Preparation and Review


Manual E-Book
Test Number Five

limb movements, hyperactive deep tendon reflexes, choreoathetoid movements, convulsions, delirium, stu-
por, coma, and circulatory failure (low blood pressure, arrhythmia). Severe lithium toxicity is greater than
2.5 mEq/L. Symptoms include generalized convulsions, oliguria, renal failure, and death.
Psychopharmacology
K&S Chapter 29

Question 125. E. A large proportion of the elderly population complains of difficulty in initiating and maintaining sleep, early
morning awakening, unrefreshing sleep, and daytime sleepiness. Many resort to napping. Polysomnographic
investigations have supported these findings and have revealed a gradual decrease in N3 sleep (delta sleep;
aka slow-wave sleep) and an increase in N1 sleep. There may also be a decreased amount of REM sleep
in the elderly. Additionally, sleep efficiency progressively decreases, and the propensity for daytime napping
increases, although it is still a matter of investigation as to whether the elderly are truly sleepier during the
day. There also appears to be a progressive advance in the sleep/wake times with aging, related to a gradual
phase advance in the internal biological clock; this may explain the common complaint of seniors that they
fall asleep early in the evening, yet awaken much earlier than desired in the morning and cannot fall back
to sleep easily. Because aging is also associated with an increased risk for medical and psychiatric disorders,
which can also disrupt sleep, it may be difficult to ascertain, in any individual case, whether the sleep-related
alterations are a “normal” consequence of the aging process itself or secondary to other disorders.
Sleep–Wake Disorders
K&S Chapter 16

Question 126. B. Atomoxetine is a norepinephrine reuptake inhibitor approved for the treatment of ADHD in both chil-
dren and adults (ages 6 years and above). It may take up to 10 weeks after starting treatment to reach
optimal effect. Once-daily dosing works well for most patients. Most common side effects include dizzi-
ness, reduced appetite, and dyspepsia. Similar to the SSRIs, atomoxetine carries a black box warning for
suicidality. It is metabolized primarily by the liver and reports suggest that combination of atomoxetine
with stimulants is well tolerated and effective.
Psychopharmacology
K&S Chapter 29

Question 127. B. The theory that empathic failures in the mother lead to developmental arrest in the child at a stage when
the child needs others to help perform self-object functions is the work of Heinz Kohut. He specifically
applied this theory to narcissism and later expanded it to other pathology as well. He viewed the devel-
opment of self-esteem and self-cohesion as more important than sexuality or aggression. Object rela-
tions theory is best represented by the work of Melanie Klein. Object relations theory is known for the
schizoid, paranoid, and depressive positions, as well as tension between the true and the false self. Other
famous therapists listed in this question are the subject of their own questions elsewhere in this text.
Psychological Theory and Psychometric Testing
K&S Chapter 4

Question 128. D. Vivitrol (naltrexone extended-release injectable suspension) is a long-acting injection of naltrexone used
to control cravings in alcohol dependence. Naltrexone pills are often used to control alcohol craving in
those trying to maintain sobriety, and this injectable form of naltrexone aims to increase compliance and
lower rates of relapse. It is given as a 380-mg intramuscular injection every 4 weeks. Vivitrol was FDA-
approved for opioid relapse prevention in 2010.
Psychopharmacology
www.vivitrol.com

Question 129. B. The HAM-D (Hamilton depression rating scale) is used to evaluate depression. The CAPS (Clinician-
Administered PTSD Scale) includes items to help make the diagnosis of PTSD, rate its severity, and
determine its effect on social and occupational functioning. The Positive and Negative Symptom Scale
(PANSS) rates severity of psychosis. The Brief Psychiatric Rating Scale (BPRS) rates severity of psychosis.
The CAGE is a questionnaire used to evaluate alcohol abuse.
Diagnostic and Treatment Procedures in Psychiatry
K&S Chapter 5

315

Psychiatry Test Preparation and Review


Manual E-Book
Psychiatry Test Preparation & Review Manual

Question 130. E. The medication described in the question stem is duloxetine (Cymbalta). It inhibits reuptake of both
serotonin and norepinephrine. It has an indication for neuropathic pain. It lacks significant choliner-
gic, antihistaminic, and α adrenergic effects. The side effect profile is similar to that of the SSRIs; how-
ever, it is worth noting that abrupt discontinuation will give clear withdrawal symptoms (yes, paroxetine
is known for this as well), and unlike the other SNRI venlafaxine, hypertension is not common with
duloxetine. Of note, duloxetine is also FDA-approved for treatment of fibromyalgia.
Psychopharmacology
K&S Chapter 29

Question 131. D. Patients taking an MAOI have dietary restrictions to avoid hypertensive crisis. All of the foods listed in
the question are restricted except for vodka. Small amounts of clear alcohol may be used, but patients
should avoid tap beer, red wines, some white wines, and aged sherry.
Psychopharmacology
K&S Chapter 29

Question 132. D. This is a case of inhalant intoxication. This patient has been sniffing something out in the garage. The
perioral rash should be a big clue. When inhalants contact the skin it dries out, leading to small cracks,
which allow bacteria to enter. A dermatitis can develop, which looks like a nonspecific contact hyper-
activity reaction or perioral eczema. It is known as a “huffer’s rash.” Criteria for inhalant intoxication
include recent exposure to volatile inhalants, followed by change in behavior. The patient often becomes
agitated, belligerent, assaultive, or apathetic. If bad enough, this state can progress into a delirium or
coma or be fatal. The patient also must have 2 of a possible 13 neurologic signs. These signs include diz-
ziness, nystagmus, incoordination, slurred speech, unsteady gait, lethargy, depressed reflexes, psychomo-
tor retardation, tremor, muscle weakness, blurry vision, stupor, and euphoria.
Substance Abuse and Addictive Disorders
K&S Chapter 20

Question 133. B. Narcolepsy is a sleep disorder characterized by excessive daytime sleepiness with sleep attacks, as well
as the intrusion of REM sleep into wakefulness. The onset is usually in adolescents and young adults.
The classic sleep attack is an irresistible desire to fall asleep in inappropriate circumstances. These spells
last for a few minutes to as long as 20 to 30 minutes. There are wide variations in frequency of attacks.
Attacks generally persist throughout the patient’s lifetime. Sudden loss of tone in all voluntary muscles
except the respiratory and ocular muscles characterizes cataplexy. The attacks are often triggered by
emotional factors, such as laughter, rage, or anger. They may be complete or partial. Most commonly,
patients may momentarily have head nodding, sagging of the jaw, buckling of the knees, dropping of
objects from the hands, dysarthria, or loss of voice, but sometimes they may slump or fall forward to
the ground. The duration is usually a few seconds to a minute or two. There is decreased REM latency
during attacks. In approximately 25% to 50% of patients, sleep paralysis is noted. The sudden, apparent
paralysis of one or both sides of the body or one limb occurs either during sleep onset (hypnagogic) or
on awakening (hypnopompic) in the morning. The patient is unable to move or speak and is frightened,
although he or she retains consciousness. In 20% to 40% of narcoleptic patients, hypnagogic hallucina-
tions occur either at sleep onset or on awakening in the morning and generally appear months to years
after the onset of sleep attacks. In 30% of patients, three of the four major manifestations of the narco-
leptic tetrad (sleep attacks, cataplexy, sleep paralysis, and hypnagogic hallucinations) occur together, and
in about 10% of cases, all four major features occur together. Disturbed night sleep is commonly noted
in 70% to 80% of patients. In patients with narcolepsy, HLA typing may be performed, because most
of the patients with narcolepsy show positivity for HLA DQB1*0602. The most specific finding for nar-
colepsy with cataplexy is low or absent CSF hypocretin (orexin). Narcolepsy-cataplexy patients show a
degeneration of hypocretin-synthesizing cells in the hypothalamus. Treatment of narcolepsy can include
stimulants, such as methylphenidate or amphetamines, modafinil (Provigil) or armodafinil (Nuvigil), or
Oxybate (Xyrem).
Sleep–Wake Disorders
K&S Chapter 16

316

Psychiatry Test Preparation and Review


Manual E-Book
Test Number Five

Question 134. C. Intellectual disability (intellectual developmental disorder) has its onset during the developmental period and
includes both intellectual and adaptive functioning deficits in conceptual, social, and practical domains. There
are deficits in intellectual functioning, such as reasoning, problem-solving, abstract thinking, learning from
experience, and academic learning. There are deficits in adaptive functioning, which result in failure to meet
developmental and sociocultural standards for personal independence and social responsibility. These deficits
limit functioning in communication, social participation, and independent living across multiple environments.
Deficits in social/emotional reciprocity are more common in autism spectrum disorder. Patients with autism
spectrum disorder have social communication and social interaction deficits across multiple contexts. This can
take the form of failure of normal back-and-forth conversation, failure to initiate or respond to social inter-
actions, poorly integrated verbal and nonverbal communication, abnormalities of eye contact and body lan-
guage, or deficits in understanding nonverbal communication. They tend to have restricted repetitive patterns
of behavior, interests, or activities. This can be manifested by repetitive motor movements, inflexible adherence
to routines, highly restricted fixated interests that are abnormal in intensity or focus, and hyper- or hyporeac-
tivity to sensory input. Severity in autism spectrum disorder is based on social communication impairments
and restricted, repetitive patterns of behavior. Onset of the disorder is during the developmental period.
Neurodevelopmental and Pervasive Developmental Disorders
K&S Chapter 31

Question 135. D. This question covers some important facts about clozapine. Doses greater than 600 mg/day substantially
increase the chance of developing seizures compared with lower doses. Clozapine is very anticholinergic
and antihistaminic and has a high likelihood of leading to weight gain and metabolic syndrome. NMS
occurs rarely, but adding lithium to clozapine increases the chances of developing NMS. Agranulocytosis
can occur at any dose and is not dose related. Because of its effect on the cytochrome P450 system,
smoking can decrease clozapine levels. Clozapine can cause sialorrhea as well as cardiomyopathy.
Clozapine is considered the best treatment for psychosis in Parkinson’s disease, due to its low likelihood
of causing EPS and exacerbating parkinsonian symptoms.
Psychopharmacology
K&S Chapter 29

Question 136. B. In delusional disorder the patient has a bizarre or nonbizarre delusion of at least 1 month’s duration.
The patient does not have any other psychotic symptoms that would classify him or her as schizo-
phrenic. As such, criterion A is not met and there is a relative preservation of function. In some cases
the patient may have olfactory or tactile hallucinations, but these are directly linked in some way to the
delusional theme. The DSM 5 specifies several subtypes. These subtypes are erotomanic, grandiose, jeal-
ous, persecutory, somatic, mixed, and unspecified.
Psychotic Disorders
K&S Chapter 7

Question 137. E. Most people do not experience PTSD symptoms even when faced with severe trauma. The lifetime
prevalence of PTSD is about 6.7%, as per the National Comorbidity Study. As per that same study,
about 60% of males and 50% of females had experienced some significant trauma. Evidence points to a
“dose–response” relationship between the degree of trauma and the likelihood of symptoms. The subjec-
tive meaning of the trauma to the individual is also extremely important. The predisposing vulnerability
factors in PTSD are as follows:

1. Presence of childhood trauma


2. Borderline, paranoid, dependent, or antisocial personality disorder traits
3. Inadequate family or peer supports
4. Female gender
5. Genetic predisposition to mental illness
6. Recent life stressors
7. Perception of an external locus of control to the trauma (natural cause) as opposed to an internal one
(human cause)
8. Recent alcohol abuse
Anxiety Disorders
K&S Chapter 11

317

Psychiatry Test Preparation and Review


Manual E-Book
Psychiatry Test Preparation & Review Manual

Question 138. B. Schizophrenic patients are no more likely to commit homicide than anyone in the general population at
large. When a schizophrenic patient does commit murder, it may be for unpredictable or bizarre reasons
due to delusions and/or hallucinations. Predictors of future homicidal behavior include a prior history
of violence, dangerous behavior while hospitalized, and delusions or hallucinations involving this kind
of violence. By contrast, schizophrenic patients are more prone to suicide attempts and completed sui-
cide than the general population at large. Suicide attempts are made by 20% to 50% of schizophrenic
patients. Long-term rates of suicide among schizophrenic patients are estimated at 10% to 13%. This
reflects about a 20-fold increase in the suicide rate over the general population. The most important pre-
dictor of suicide in schizophrenic patients is the presence of a major depressive episode.
Psychotic Disorders/Public Policy
K&S Chapter 7

Question 139. D. This is a tricky question that can easily fool you if you are not familiar with the symptoms of Sheehan’s
syndrome. Sheehan’s syndrome is a postpartum pituitary necrosis that can lead to chronic symptoms,
which can easily be mistaken for depression and misdiagnosed. Symptoms of Sheehan’s syndrome
include failure to lactate, hypotension, weight loss, loss of secondary sex characteristics, scant men-
ses, constant fatigue, and diminished libido. These symptoms can develop months or even years after
giving birth. Diagnosis of Sheehan’s syndrome is made by first looking for history of failure to make
breast milk or failure to resume menstruation after giving birth. If suspected, pituitary hormone levels
are checked, followed by MRI to examine the pituitary. Important to keep in mind is that DSM criteria
for postpartum depression states that its onset must be within 4 weeks of the birth. When we consider
this patient in light of our knowledge of Sheehan’s, it is clear that an endocrine workup to rule out the
disease must be our first step, especially considering her failure to produce breast milk. To immediately
treat the situation as if she has a major depressive disorder or anxiety disorder from the stress of the new
baby would be missing a very important diagnostic rule out. There is no evidence of child abuse given
here, and as such, child protective services should not be called.
Somatic Symptom Disorders
K&S Chapter 21

Question 140. D. This question focuses on avoidant personality disorder. Patients with this disorder tend to avoid inter-
personal interaction out of fear of shame and ridicule. They are hypersensitive to negative evaluation
by others. They are unwilling to get involved with others without certainty of being liked. They are
inhibited interpersonally because of fears of inadequacy. They see themselves as inferior to others. They
are often misdiagnosed as dependent personality disorder and in some cases are very hard to distin-
guish from generalized social phobia. Other answer choices reflect characteristics of other personality
disorders:
• “These patients tend to be indifferent to praise or criticism”—schizoid.
• “Affective instability is a key component of the disorder”—borderline.
• “These patients are unable to discard worthless objects”—obsessive–compulsive personality disorder.
Personality Disorders
K&S Chapter 22

Question 141. B. Acute intermittent porphyria is one of the medical illnesses that is fair game for a psychiatry examina-
tion because of the prominence of psychiatric symptoms in its presentation. It is a disorder of heme
synthesis, which leads to a buildup of porphyria. This leads to the classic triad of abdominal pain,
peripheral neuropathy, and psychiatric disturbance. Psychiatric symptoms may come in the form of
delirium, psychosis, depression, or anxiety (sorry, no dementia). Medical symptoms include seizures due
to hyponatremia, dehydration due to nausea and vomiting, renal damage, and dermatologic manifesta-
tions. The diagnosis is confirmed by looking for metabolites of porphyrins (porphobilinogen) in urine,
blood, and stool. It is treated by limiting exacerbating medications, hydration, adequate nutrition, anti-
hypertensive agents, and intravenous heme preparations. This is the illness thought responsible for the
“madness” of King George III.
Somatic Symptom Disorders
K&S Chapter 21

318

Psychiatry Test Preparation and Review


Manual E-Book
Test Number Five

Question 142. D. Most studies point to a prevalence of Alzheimer’s disease of about 20% to 40% in the 85 years and over
age range. Other statistics of note include the fact that 50% to 60% of patients that are demented have
Alzheimer’s disease. The second most common type of dementia is vascular dementia, which accounts
for approximately 15% to 30% of all dementia cases. Alzheimer’s disease is slightly more prevalent in
women than in men throughout the life cycle, about 1.3 to 1. Vascular dementia is more prevalent in
men than women and is most common in the 60 to 70 years age range. About 10% to 15% of patients
have comorbid Alzheimer’s disease and vascular dementia.
Neurocognitive Disorders/Public Policy
K&S Chapter 21

Question 143. E. It is important to understand legal issues surrounding the suicidal patient. The law does not impose
liability on the psychiatrist whenever a patient commits suicide. It asks whether the suicide was foresee-
able and how a reasonable psychiatrist would have addressed the likelihood of harm. All potential harm
is not viewed as foreseeable or predictable. The standard of care calls for documentation of risk assess-
ment and implementation of appropriate precautions to protect the patient. No Harm contracts between
therapist and patient do not protect the therapist from liability in the event of a suicide.
Forensic Psychiatry
K&S Chapter 36

Question 144. D. The definition of “disability” under the ADA reflects the intent of Congress to prohibit the specific forms of
discrimination that persons with disabilities face. Although individuals with disabilities may experience the
types of discrimination that confront other groups, they also may encounter unique forms of discrimina-
tion because of the nature of their disabilities and the effect that their present, past, or perceived conditions
have on other persons. The purpose of the ADA is to eliminate discrimination that confronts individuals
with disabilities. Because the definition of the term “disability” under the ADA is tailored to the purpose of
eliminating discrimination prohibited by the ADA, it may differ from the definition of “disability” in other
laws drafted for other purposes. For example, the definition of a “disabled veteran” is not the same as the
definition of an individual with a disability under the ADA. Similarly, an individual might be eligible for
disability retirement but not be an individual with a disability under the ADA. Conversely, a person who
meets the ADA definition of “disability” might not meet the requirements for disability retirement.
In the statutory definition, with respect to an individual, the term “disability” means:
(A) a physical or mental impairment that substantially limits one or more of the major life activities of
such individuals;
(B) a record of such an impairment; or
(C) being regarded as having such an impairment.
A person must meet the requirements of at least one of these three criteria to be an individual with a dis-
ability under the Act.
Public Policy
K&S Chapter 35

Question 145. E. The homeless who are mentally ill are a significant public policy concern. Their population continues to
grow, with large numbers of homeless people carrying a psychiatric diagnosis. Many of them are also
dependent on substances. Street dwellers tend to be schizophrenic with substance abuse. Episodically
homeless tend to have personality disorders, substance abuse, and mood disorders. These of course are
generalizations, and exceptions can be found. Traditional mental health delivery systems often fail to
successfully treat the homeless owing to a variety of factors. Successful treatment for this population is
often very nonconventional, including street outreach programs.
Public Policy
K&S Chapter 35

Question 146. E. This is an easy one. No sex with patients or former patients…ever, under any circumstances. No former
sexual partners as current patients either.
Ethics
K&S Chapter 36

319

Psychiatry Test Preparation and Review


Manual E-Book
Psychiatry Test Preparation & Review Manual

Question 147. E. Varenicline (Chantix) is a partial nicotine agonist that decreases withdrawal and cravings and blocks the
reinforcing effects of nicotine. Whereas nicotine replacement and bupropion have been shown to double
quit rates, varenicline triples quit rates. Take note of its warning for behavioral changes such as hostil-
ity, agitation, depression, and suicidal thoughts either during treatment or after stopping treatment. The
most common side effect is nausea.
Psychopharmacology
K&S Chapter 20

Question 148. A. Treatment of organophosphate poisoning involves intravenous administration of pralidoxime together
with atropine given subcutaneously every 30 minutes until sweating and salivation are controlled.
Pralidoxime accelerates reactivation of the inhibited acetylcholinesterase, and atropine is effective in
counteracting muscarinic effects, although it has no effect on the nicotinic effects, such as neuromuscular
cholinergic blockade with weakness or respiratory depression. It is important to ensure adequate venti-
latory support before atropine is given. The dose of pralidoxime can be repeated if no obvious benefit
occurs, but in refractory cases it may need to be given by intravenous infusion, the dose being titrated
against clinical response. Functional recovery may take approximately 1 week, although acetylcholines-
terase levels take longer to reach normal levels. Measurement of paraoxonase status may be worthwhile
as a biomarker of susceptibility to acute organophosphate toxicity; this liver and serum enzyme hydro-
lyzes a number of organophosphate compounds and may have a role in modulating their toxicity.

Chelation therapy with either water-soluble derivatives of dimercaprol (dimercaptosuccinate or


2,3-dimercaptopropane sulfonate [DMPS]) or penicillamine is effective in controlling the systemic effects
of acute arsenic poisoning and may prevent the development of neuropathy if it is started within hours
of ingestion. Lead encephalopathy is managed supportively, but corticosteroids are given to treat cere-
bral edema, and chelating agents (dimercaprol or DMPS) are prescribed also.
Poisoning
B&D Chapter 58

Question 149. E. HIPAA stands for Health Insurance Portability and Accountability Act. It contains rules to protect the
transmission and confidentiality of patient information. The privacy rule is one that is very relevant to
psychiatric practice. It applies to all protected health information, whether on paper, electronic, or spoken.
Some of the important points to keep in mind are as follows: patients have a right to a written copy of
their privacy rights. They also have a right to know how their protected information will be stored and
disclosed. Patients have a right to a copy of their medical record. This does not include psychotherapy
notes. Authorization must be obtained from patients for release of their health information. Exclusions to
this rule involve routine uses, such as treatment, obtaining payment from the insurance company, or health
care operations. Patients do have the right to dictate how their information is communicated. For example,
they can tell a doctor not to call their home phone number and to call only their cell phone to protect their
privacy, or they can dictate a specific address to which all confidential information must be sent.
Forensic Psychiatry
K&S Chapter 36

Question 150. A. To answer this question correctly, you needed to know the mechanism of action of mirtazapine.
Mirtazapine works through antagonism at presynaptic α2 receptors, thereby potentiating the actions of
serotonin and norepinephrine. It does not block reuptake of serotonin or norepinephrine. It is a potent
H1 antagonist, which explains why it is sedating and causes increased appetite and weight gain. It has
minimal sexual side effects.
Psychopharmacology
K&S Chapter 29

320

Psychiatry Test Preparation and Review


Manual E-Book
Test Number Six

SIX
Test Number Six
1. Which of the following cytochrome P450 (CYP) enzymes is responsible for the metabolism of trazodone?

A. 
CYP 2D6
B. 
CYP 1A2
C. 
CYP 2C19
D. 
CYP 3A4
E. 
CYP 2C9

2. A young man presents to the emergency room with a complaint of several days of progressive lower extremity weak-
ness and numbness bilaterally. History reveals he had a flu-like illness for about 10 days before coming to the hospital.
Examination confirms the lower extremity weakness, as well as loss of sensation to all sensory modalities below the
middle of the thorax. Deep tendon reflexes are brisker below the waist than above, and plantar reflexes are extensor
bilaterally. The patient complains of recent episodes of urinary incontinence. The rest of the neurologic and physical
examinations are unremarkable. A lumbar puncture reveals 23 mononuclear cells, a protein level of 37 mg/dL, and nor-
mal glucose. The most likely diagnosis is:

A. 
Spinal epidural abscess
B. 
Anterior spinal artery infarction
C. 
Acute transverse myelitis
D. 
Acute inflammatory demyelinating polyneuropathy
E. 
Spinal metastasis

3. Which of the following medications has U.S. Food and Drug Administration (FDA) approval for treatment of bipolar
maintenance, thereby treating both mania and depression?

A. 
Topiramate
B. 
Carbamazepine
C. 
Lamotrigine
D. 
Valproic acid
E. 
Gabapentin

Full test - and additional VIDEO vignettes - available online - see inside front cover for details.

321

Psychiatry Test Preparation and Review


Manual E-Book
Psychiatry Test Preparation & Review Manual

4. Which one of the follow sleep stages correlates with childhood somnambulism as measured by an electroencephalogram
(EEG)?

A. 
Rapid eye movement (REM) sleep
B. 
Stage II
C. 
Stage III
D. 
Stage IV
E. 
Stage I

5. You are teaching a medical student about weight gain with psychiatric medications and begin to discuss the public
health ramifications of obesity-related illnesses in the United States. Which of the following is true concerning obesity in
the United States?

A. 
Obesity is more common in men
B. 
Obesity is more common in the elderly
C. 
Obesity rates are declining
D. 
Obesity is more common in minority populations
E. 
Obesity rates are lowest in middle-aged adults

6. A 60-year-old man with diabetes mellitus is seen by an outpatient psychiatrist because the family states he is not making
any sense. When asked what is wrong, the man states: “Thot blegging at bremull fee gelking.” This expression has nor-
mal intonation, but no one can understand it. The man is able to respond to other questions with similar expressions,
but he cannot execute any instructions. The most likely diagnosis is:

A. 
Wernicke’s aphasia
B. 
Bálint’s syndrome
C. 
Gerstmann’s syndrome
D. 
Schizophrenic word salad
E. 
Conduction aphasia

7. Which of the following would the American Psychiatric Association consider unethical?

A. 
Accepting a cup of coffee from a patient
B. 
Charging a patient for missed visits
C. 
Selling your used car to a patient
D. 
Releasing information about a patient to his or her insurance company
E. 
Maintaining a separate set of psychotherapy notes in addition to the medical record

8. A 75-year-old woman develops a sudden onset of right foot and leg paralysis. Her right arm, hand, and face are only
very slightly affected. There is no visual field deficit or aphasia. Over the ensuing weeks, she is found to have lack of
spontaneity, abulia, and loss of bladder control. Which of the following vascular territories would be implicated in this
stroke?

A. 
Right anterior choroidal artery
B. 
Left anterior cerebral artery
C. 
Small subcortical vessels
D. 
Deep penetrating vessels supplying the frontal lobes
E. 
Superior frontal branch of the right middle cerebral artery

322

Psychiatry Test Preparation and Review


Manual E-Book
Test Number Six

9. Which of the following medications is excreted unchanged in the urine?

A. 
Sertraline
B. 
Olanzapine
C. 
Risperidone
D. 
Lamotrigine
E. 
Gabapentin

10. A 55-year-old woman was found wandering around the local bus terminal in a confused and disoriented state. She could
recall her name but not how she got to the bus terminal or why she was there. Her daughter went to pick her up at the
local hospital thereafter and tried to reorient her mother many times. Nevertheless, her mother kept asking repetitive
questions, even about information she had just been given about her location. Neurologic examination was normal, as
were brain magnetic resonance imaging (MRI) and EEG. Within 12 hours the woman’s symptoms subsided entirely. The
most likely diagnosis of her problem was:

A. 
Attention-seeking behavior
B. 
An acute dissociative state
C. 
An epileptic fugue state
D. 
Alcohol intoxication
E. 
Transient global amnesia

11. Giving a patient nefazodone and trazodone at the same time will lead to which of the following?

A. 
Decreased trazodone levels
B. 
Decreased nefazodone levels
C. 
Increased trazodone levels
D. 
Increased nefazodone levels
E. 
Induction of CYP 3A4

12. Which of the following procedures would confirm the diagnosis of nonepileptic seizure?

A. 
Hypnotic suggestion
B. 
Videotelemetry
C. 
MRI brain scan
D. 
Observation for a 24-hour period
E. 
EEG between episodes

13. One of your patients on neuroleptic medication complains of symptoms of akathisia. Which of the following is the most
appropriate treatment?

A. 
Benztropine
B. 
Bromocriptine
C. 
Methylenedioxypyrovalerone
D. 
Propranolol
E. 
Clonidine

323

Psychiatry Test Preparation and Review


Manual E-Book
Psychiatry Test Preparation & Review Manual

14. Lancinating facial pain that is sometimes triggered by minor sensory stimuli is best treated initially with which of the
following drugs?

A. 
Lioresal
B. 
Transdermal lidocaine patch
C. 
Eletriptan
D. 
Carbamazepine
E. 
Capsaicin

15. In the management of patients with acute ischemic stroke, administration of which of the following agents within 48
hours of the onset of stroke has been shown to have a beneficial effect in reducing the risk of recurrent stroke, disability,
and death?

A. 
Aspirin
B. 
Heparin
C. 
Recombinant tissue plasminogen activator (t-PA)
D. 
Enoxaparin
E. 
Clopidogrel

16. A 35-year-old woman experiences numbness in both hands


during her second trimester of pregnancy. The numbness
involves the thumb, forefinger, and middle finger of both
hands. She reports that the dorsal part of the hand is not
affected. She often awakens in the morning with aching
arms, from the shoulder area to the hands. The most likely
diagnosis is:

A. 
Ulnar entrapment neuropathies
B. 
Brachial plexopathy
C. 
Median neuropathies at the wrist
D. 
Thoracic outlet syndrome
E. 
C6 radiculopathies due to cervical spondylosis

17. Which of the following anxiety disorders is most common?

A. 
Panic disorder
B. 
Obsessive–compulsive disorder
C. 
Posttraumatic stress disorder
D. 
Specific phobia
E. 
Generalized anxiety disorder

18. The association of skin tumors, Lisch nodules of the iris, and abundant café-au-lait spots are diagnostic of a mutation in
which of the following genes?

A. 
NACP (α-synuclein gene)
B. 
NF1 (neurofibromatosis 1 gene)
C. 
ATM (ataxia telangiectasia gene)
D. 
TSC1 (tuberous sclerosis gene 1)
E. 
NF2 (neurofibromatosis 2 gene)

324

Psychiatry Test Preparation and Review


Manual E-Book
Test Number Six

19. Culture correlates most with which of the following factors?

A. 
Race
B. 
Age
C. 
Gender
D. 
Nationality
E. 
Ethnicity

20. A 30-year-old patient presents to the emergency room 2 weeks after starting a crash fad diet to lose weight. The chief
complaint is abdominal and extremity pain. The patient is anxious, tachycardic, disoriented, and pale on examination
and seems to be responding to hallucinations. The family history from collaterals reveals that the patient’s father suffers
similar episodes at times. The most likely diagnosis is:

A. 
Acute intermittent porphyria
B. 
Huntington’s disease
C. 
Hemophilia B
D. 
Bipolar I disorder
E. 
Anorexia nervosa with hallucinogen intoxication

21. Which one of the following interactions might one expect when hydrochlorothiazide is prescribed to a patient taking
psychiatric medication?

A. 
Decreased clozapine level
B. 
Decreased valproic acid level
C. 
Increased lithium level
D. 
Increased clozapine level
E. 
Decreased lithium level

22. Which of the following agents has been shown to reduce the accumulation of plaques and disability in patients with
relapsing remitting multiple sclerosis?

A. 
Prednisone
Interferon β-1a
B. 
C. 
Tizanidine
D. 
Lioresal
E. 
Intravenous gamma globulin

23. A bulimic patient admits to you that she has been using large quantities of ipecac on a daily basis. Which of the follow-
ing medical issues should you be most concerned with?

A. 
Cardiomyopathy
B. 
Shortening of QTc interval
C. 
Leukocytosis
D. 
Increased serum amylase
E. 
Infection

325

Psychiatry Test Preparation and Review


Manual E-Book
Psychiatry Test Preparation & Review Manual

24. The major clinical features of Binswanger’s disease include a gait disorder, a pseudobulbar state, and:

A. 
Urinary incontinence
B. 
Aphasia
C. 
Constructional apraxia
D. 
Dementia
E. 
Diplopia

25. Psychological debriefing following a traumatic event:

A. 
Is considered first-line treatment for acute stress disorder
B. 
Has been proven more effective than selective serotonin reuptake inhibitors (SSRIs) alone
C. 
Is controversial
D. 
Has more data for efficacy than eye movement desensitization and reprocessing
E. 
Is the treatment of choice for posttraumatic stress disorder

26. A 32-year-old patient was referred for neurologic evaluation owing to hand shaking. The history reveals that the shak-
ing is present only when using the hands and is particularly bad when writing or bringing a cup to the mouth while
drinking. Stress worsens the shaking. The shaking improves when the patient drinks a glass of wine. The patient’s older
sibling also suffers from a similar disorder. From this history, the most likely diagnosis is:

A. 
Alcohol-withdrawal tremor
B. 
Wilson’s disease
C. 
Spinocerebellar ataxia
D. 
Early-onset Parkinson’s disease
E. 
Essential tremor

27. At what age can children use language to tell a story, share ideas, and discuss alternatives?

A. 
10 months
B. 
20 months
C. 
30 months
D. 
50 months
E. 
60 months

28. A palsy of the sixth cranial nerve is associated with which one of the following alcohol-related disorders?

A. 
Cerebellar degeneration
B. 
Wernicke’s encephalopathy
C. 
Marchiafava–Bignami syndrome
D. 
Alcohol-related delirium
E. 
Korsakoff’s syndrome

29. Which of the following tests would be most helpful in trying to understand deficits in a patient with right hemisphere disease?

A. 
Boston diagnostic aphasia examination
B. 
Sentence completion test
C. 
Word association technique
D. 
Judgment of line orientation test
E. 
Thematic apperception test

326

Psychiatry Test Preparation and Review


Manual E-Book
Test Number Six

30. 
A 68-year-old patient presents to an outpatient neurolo-
gist with marked gait abnormality, slow movements, and
asymmetric rigidity of the upper extremities. Eye examina-
tion reveals mild impairment in both upward and down-
ward voluntary vertical gaze. Further history reveals that
levodopa therapy improved the slowness and rigidity only
modestly. Several years later, the patient is found to have
impairment in voluntary horizontal as well as vertical gaze.
Oculocephalic reflexes were normal. The patient’s most
likely diagnosis is:

A. 
Pseudobulbar palsy
B. 
Olivopontocerebellar atrophy
C. 
Striatonigral degeneration
D. 
Progressive supranuclear palsy
E. 
Dentatorubral degeneration

31. All of the following are mature defenses except:

A. 
Humor
B. 
Introjection
C. 
Altruism
D. 
Anticipation
E. 
Suppression

32. A 50-year-old patient consults a neurologist for a gradually


progressive and intermittent involuntary turning of the neck
to the left. At times the patient complains of pain in the ster-
nocleidomastoid muscle and head tremor. These symptoms
are exacerbated by anxiety and stress. The most appropriate
treatment for this condition is:

A. 
Divalproex sodium
B. 
Olanzapine
C. 
Botulinum toxin
D. 
Levodopa/carbidopa
E. 
Benztropine

33. 
All of the following are possible causes for anorgasmia
except:

A. 
Alcohol
B. 
Tricyclic antidepressants (TCAs)
C. 
Diabetes
D. 
Spinal cord damage
E. 
Methylphenidate

327

Psychiatry Test Preparation and Review


Manual E-Book
Psychiatry Test Preparation & Review Manual

34. A 16-year-old patient is reported to have episodes of muscle jerks that involve the whole body and absence episodes dur-
ing which the patient seems dazed for a few seconds. The EEG shows bursts of 4- to 6-Hz polyspike and wave activity.
The most likely diagnosis in this case is:

A. 
Juvenile myoclonic epilepsy
B. 
Nonepileptic seizures
C. 
Cataplexy
D. 
Petit-mal epilepsy
E. 
Rolandic epilepsy

35. Interaction with the enzyme cytochrome P450 (CYP) 2D6 has what bearing on the use of tricyclic antidepressants?

A. 
None; tricyclics do not interact with CYP 2D6
B. 
Those with decreased CYP 2D6 activity will have lower than expected plasma drug concentrations
C. 
Giving a patient on a tricyclic a CYP 2D6 inhibitor could cause a drop in the plasma concentration of the drug
D. 
Cimetidine can cause an increase in tricyclic levels as a result of its interaction with CYP 2D6
E. 
Concomitant use of drugs that inhibit CYP 2D6 with tricyclics may necessitate higher than usual prescribed doses
of either drug to obtain the same levels

36. Which one of the following is a key feature in the diagnosis of irritable bowel syndrome?

A. 
Headaches
B. 
Mucus in stool, but no blood
C. 
Striae
D. 
Hyperpigmentation
E. 
Salt craving

37. Which of the following famous therapists developed the “epigenetic principle”?

A. 
Erik Erikson
B. 
Melanie Klein
C. 
B.F. Skinner
D. 
Harry Stack Sullivan
E. 
Heinz Kohut

38. A 60-year-old patient with hypertension and diabetes consults a neurologist for left periorbital pain and diplopia.
Examination reveals paralysis of abduction of the left eye. Which of the following is the most probable diagnosis?

A. 
Left pontine stroke
B. 
Pseudotumor cerebri
C. 
Diabetic sixth nerve palsy
D. 
Myasthenia gravis
E. 
Optic neuritis

328

Psychiatry Test Preparation and Review


Manual E-Book
Test Number Six

39. Which one of the following is an appropriate Diagnostic and Statistical Manual of Mental Disorders, fifth edition (DSM
5), specifier used for substance-induced anxiety disorder?

A. 
With generalized anxiety
B. 
With panic attacks
C. 
With obsessive–compulsive symptoms
D. 
With onset during withdrawal
E. 
With delayed onset

40. A 35-year-old patient presents with headache, fever, seizures, confusion, stupor, and coma that evolve over a period of a
week. The EEG demonstrates periodic lateralized high-voltage sharp waves emanating from the left temporal region, as
well as slow-wave complexes repeating at 2- to 3-second intervals. The brain computed tomography (CT) scan shows a
low-density lesion in the left temporal lobe. The most likely diagnosis in this case is:

A. 
Listeria meningitis
B. 
Human immunodeficiency virus (HIV) encephalitis
C. 
St. Louis encephalitis
D. 
Herpes simplex encephalitis
E. 
Cryptococcal meningitis

41. Which of the following is the most important factor in distinguishing somatic symptom disorder with predominant pain
from chronic pain syndromes?

A. 
The pain is severe enough to warrant clinical attention
B. 
The patient suffers disability from the pain
C. 
No physical lesion is found on imaging studies
D. 
Substance abuse and character pathology may complicate the picture
E. 
The pain is attributed to psychological factors

42. A 65-year-old man presents with 3 months of progressive limb weakness, worse on the right than the left. He has trou-
ble swallowing liquids. Neurologic examination reveals normal cranial nerve function but extensor neck muscle weak-
ness. He has motor weakness in both distal and proximal muscles, which include the quadriceps, dorsal foot flexors, and
especially the extensor pollicis longus. The upper-extremity motor examination is significant for wrist and finger flexor
weakness. Deep tendon reflexes are preserved and normal throughout. Muscle tone, coordination, and gait are normal.
The rest of the physical examination is normal. Laboratory values reveal mildly elevated serum creatine kinase. Which
one of the following is the most likely diagnosis?

A. 
Myasthenia gravis
B. 
Inclusion body myositis
C. 
Polymyalgia rheumatica
D. 
Dermatomyositis
E. 
Polymyositis

43. Which of the following brain pathways starts in the ventral tegmental area and projects to the frontal cortex?

A. 
Tuberoinfundibular pathway
B. 
Nigrostriatal pathway
C. 
Mesolimbic pathway
D. 
Mesocortical pathway
E. 
Ventral amygdalofugal pathway

329

Psychiatry Test Preparation and Review


Manual E-Book
Psychiatry Test Preparation & Review Manual

44. Which one of the following mitochondrial disorders is the most common?

A. 
Leber’s hereditary optic neuropathy
B. 
Myoclonus epilepsy with ragged-red fibers (MERRF)
C. 
Mitochondrial encephalopathy, myopathy, lactic acidosis, and stroke-like episodes (MELAS)
D. 
Kearns–Sayre syndrome (KSS)
E. 
Maternally inherited myopathy with cardiomyopathy

45. Yves has a preoccupation with sexual fantasies involving being beaten, bound, and humiliated. Which of the following
terms best describes him?

A. 
Voyeuristic disorder
B. 
Transvestic fetishism
C. 
Fetishistic disorder
D. 
Frotteuristic disorder
E. 
Sexual masochism

46. Botulinum toxin is FDA-approved for a number of movement disorders, including cervical dystonia and blepharospasm.
Its mechanism of action at the neuromuscular junction involves:

A. 
Inhibition of acetylcholine release from presynaptic terminals
B. 
Blockade of voltage-dependent calcium channels
C. 
Inhibition of acetylcholine synthesis at the neuromuscular junction
D. 
Inhibition of acetylcholinesterase
E. 
Blockade of the nicotinic acetylcholine receptors in muscle

47. All of the following are symptoms of tobacco withdrawal except:

A. 
Depressed mood
B. 
Bradycardia
C. 
Somnolence
D. 
Increased appetite
E. 
Irritability

48. A 58-year-old patient with chronic atrial fibrillation develops aphasia and right hemiparesis acutely while watching tele-
vision at home one evening at 8:00 pm. In the emergency room, the neurologic examination reveals moderate right
hemiparesis and an expressive aphasia. A head computed tomography (CT) scan done at 9:30 pm reveals no acute
lesion. The most appropriate therapy in the emergency room is:

A. 
Aspirin
B. 
Clopidogrel
C. 
Intravenous heparin
D. 
Low-molecular-weight heparin
E. 
Intravenous t-PA

330

Psychiatry Test Preparation and Review


Manual E-Book
Test Number Six

49. A man in his 30s is convinced that he has Lou Gehrig’s disease. Despite negative evaluations by several physicians, he
is still convinced that he has it. He comes to you convinced that the proper diagnosis has been missed. Your workup is
negative, but he remains convinced that he has the disease, despite your efforts at reassurance. What is the most appro-
priate diagnosis?

A. 
Somatic symptom disorder
B. 
Factitious disorder
C. 
Malingering
D. 
Illness anxiety disorder
E. 
Lou Gehrig’s disease

50. A 70-year-old woman complains of leg stiffness while ambulating and lower extremity spasms while trying to sleep.
Neurologic examination reveals a markedly stiff-legged gait, with leg adduction while walking. Lower extremity tone is
increased with a spastic catch. Knee-jerk reflexes are hyperactive, and ankle-jerk reflexes elicit clonus. Romberg testing
reveals sway when the patient closes her eyes. The most likely diagnosis in this case is:

A. 
Parkinson’s disease
B. 
Restless legs syndrome
C. 
Cervical spondylosis
D. 
Alcoholic cerebellar degeneration
E. 
Senile gait disorder

51. A child is in the doctor’s office for an evaluation. His mother is waiting outside in the waiting area. The child is aware
that his mother still exists, even though she is not present in the room. For this to be true, the child must have reached
which one of Mahler’s stages of separation–individuation?

A. 
Normal autism
B. 
Practicing
C. 
Differentiation
D. 
Symbiosis
E. 
Object constancy

52. A 60-year-old patient with hypertension complains of subacute painless visual loss of the right eye. Neurologic examina-
tion is normal, apart from the blindness and a right afferent pupillary defect. A brain MRI scan reveals several periven-
tricular white matter hyperintensities on T2 image sequencing. There are no lesions of the corpus callosum noted. The
most likely diagnosis in this case is:

A. 
Optic neuritis
B. 
Temporal arteritis
C. 
Closed-angle glaucoma
D. 
Retinal vein occlusion
E. 
Ischemic optic neuropathy

53. Intoxication with which one of the following can cause nystagmus?

A. 
Heroin
B. 
Lysergic acid diethylamide (LSD)
C. 
Methadone
D. 
Nicotine
E. 
Phencyclidine

331

Psychiatry Test Preparation and Review


Manual E-Book
Psychiatry Test Preparation & Review Manual

54. According to the American Academy of Neurology, an essential diagnostic criterion for the declaration of brain death
before organ donation for transplantation requires:

A. 
An MRI scan with diffuse cortical damage
B. 
A positive apnea test
C. 
Neurologic consultation
D. 
Two flat or isoelectric EEGs in 24 hours
E. 
Absence of deep tendon reflexes

55. When considering the CYP system, which one of the following statements is false?

A. 
CYP 2D6 and CYP 3A4 are two important enzymes psychiatrists must understand
B. 
Fluoxetine is a potent inhibitor of CYP 2D6
C. 
A CYP inhibitor will decrease the plasma levels of a drug over time
D. 
Genetic polymorphisms in the CYP 2D6 gene can make some patients rapid metabolizers
E. 
Paroxetine is a potent inhibitor of CYP 2D6

56. A 47-year-old patient complains of gradual progressive weakness over the past 3 to 4 months, particularly in the right
upper extremity. Neurologic examination demonstrates muscular atrophy of the right forearm and intrinsic muscles of
the right hand. Deep tendon reflexes are brisk, and plantar responses are extensor. Electrodiagnostic studies reveal wide-
spread fasciculations, fibrillations, and positive sharp waves. This clinical picture most likely represents a diagnosis of:

A. 
Multiple sclerosis
B. 
Cervical spondylosis
C. 
Chronic inflammatory demyelinating polyneuropathy
D. 
Amyotrophic lateral sclerosis
E. 
Myasthenia gravis

57. Which of the following is the most likely age for a patient on neuroleptics to develop tardive dyskinesia?

A. 
18 years
B. 
25 years
C. 
35 years
D. 
45 years
E. 
65 years

58. A 62-year-old patient presents with a first seizure with focal onset and secondary generalization. The most likely cause
of this seizure is:

A. 
Subarachnoid hemorrhage
B. 
Glioblastoma multiforme
C. 
Temporal lobe epilepsy
D. 
Alcohol withdrawal
E. 
Temporal arteritis

332

Psychiatry Test Preparation and Review


Manual E-Book
Test Number Six

59. Interactions with which one of the following medications will increase lamotrigine levels?

A. 
Oral contraceptives
B. 
Valproic acid
C. 
Carbamazepine
D. 
Phenytoin
E. 
Phenobarbital

60. Which of the following brain imaging techniques measures neuronal glucose metabolism?

A. 
EEG
B. 
Magnetic resonance spectroscopy (MRS)
C. 
Single-photon emission computerized tomography (SPECT)
D. 
Positron emission tomography (PET)
E. 
Functional MRI (fMRI)

61. Which one of the following inhibits the abnormal involuntary movements associated with Huntington’s disease?

A. 
Haloperidol
B. 
Stereotactic thalamotomy
C. 
Pramipexole
D. 
Lioresal
E. 
Benztropine

62. Which one of the following disorders presents with a gait disturbance characterized by involuntary acceleration?

A. 
Huntington’s disease
B. 
Alcohol intoxication
C. 
Parkinson’s disease
D. 
Astasia–abasia
E. 
Guillain–Barré syndrome

63. The classic triad of symptoms present in meningitis includes fever, neck stiffness, and which of the following?

A. 
Tachycardia
B. 
Alteration of sensorium
C. 
Vomiting
D. 
Syncope
E. 
Headaches

64. Which one of the following is not usually seen in Guillain–Barré syndrome?

A. 
Cerebrospinal fluid (CSF) pleocytosis
B. 
Elevated CSF protein
C. 
Areflexia
D. 
Motor loss with paresthesia
E. 
Ascending paralysis

333

Psychiatry Test Preparation and Review


Manual E-Book
Psychiatry Test Preparation & Review Manual

65. Which type of intracranial hemorrhage frequently requires surgical intervention to improve the outcome?

A. 
Pontine hemorrhage
B. 
Thalamic hemorrhage
C. 
Putaminal hemorrhage
D. 
Temporal lobe hemorrhage
E. 
Cerebellar hemorrhage

66. Stiff-person syndrome, causing persistent limb rigidity and spasms, is often correlated with the presence of:

A. 
High titers of JC virus antigen
B. 
Circulating lupus anticoagulant
C. 
Autoantibodies against glutamic acid decarboxylase
D. 
Elevated methylmalonic acid levels
E. 
Elevated syphilis (Treponema pallidum) titers

67. Gustatory special sensory auras localize to which area of the brain?

A. 
Cingulate gyrus
B. 
Dorsolateral prefrontal cortex
C. 
Occipital neocortex
D. 
Insular cortex
E. 
Parietal cortex

68. A man presents with sudden-onset left neck pain, left eyelid droop, left pupillary miosis, left facial anhidrosis, right
hemiparesis, and expressive aphasia. The most likely cause is:

A. 
Left internal carotid artery dissection
B. 
Left vertebral artery dissection
C. 
Middle cerebral artery cerebral vascular accident
D. 
Posterior inferior cerebellar artery lesion
E. 
Demyelination

69. Grandpa Joe began having a tremor last year. He began to shuffle when he walked. His handwriting became smaller.
Seven months ago he was diagnosed with Parkinson’s disease and has been seeing a neurologist. In the past few months
he has begun spending every weekend at the casino. Before this he had never gambled. He’s talking about taking out a
reverse mortgage on the house and has been going through huge sums of money gambling. He is planning a trip to Las
Vegas next month and is talking about selling his car to have money to gamble with. What is the most likely explanation
for his behavior?

A. 
He has developed mania late in life
B. 
He was recently started on dopamine agonist therapy
C. 
He is exhibiting the narcissistic defense of distortion
D. 
He is grappling with his own mortality and is in Kubler-Ross’s stage of bargaining.
E. 
He is demonstrating Erickson’s stage of generativity vs stagnation

334

Psychiatry Test Preparation and Review


Manual E-Book
Test Number Six

70. Which of the following benzodiazepines is metabolized by glucuronidation?

A. 
Lorazepam
B. 
Diazepam
C. 
Triazolam
E. 
Clonazepam
F. 
Alprazolam

71. Which of the following is the most common side effect of SSRIs early in treatment, leading to discontinuation by
patients?

A. 
Sleeplessness
B. 
Sexual dysfunction
C. 
Gastrointestinal distress
D. 
Headache
E. 
Agitation/anxiety

72. Anterior capsulotomy and/or cingulotomy have been demonstrated to be effective in cases of treatment failure with
which of the following severe disorders?

A. 
Tourette’s syndrome
B. 
Schizophrenia
C. 
Bipolar disorder
D. 
Antisocial personality disorder
E. 
Obsessive–compulsive disorder

73. A patient on lithium for bipolar disorder reports feeling slow, dull, and unable to concentrate, despite an excellent
response in terms of mood stability. The most likely reason for this is that the patient is:

A. 
Somaticizing
B. 
Accurately reporting side effects
C. 
Abusing other substances
D. 
Rationalizing noncompliance
E. 
Undervaluing normal mood

74. Which of the following choices accounts for about half of all completed suicides?

A. 
Schizophrenia
B. 
Personality disorders
C. 
Alcohol abuse
D. 
Major depression and bipolar disorder
E. 
Organic brain syndromes

75. Which of the following is not one of the therapist’s tasks in a group therapy setting?

A. 
Formulate goals
B. 
Decide on an open vs closed group
C. 
Maintenance of a therapeutic environment
D. 
Choose frequency and length of group meetings
E. 
Suppression of catharsis

335

Psychiatry Test Preparation and Review


Manual E-Book
Psychiatry Test Preparation & Review Manual

76. Which of the following blood tests is the most important to monitor in a patient taking carbamazepine?

A. 
Creatinine
B. 
Fasting glucose
C. 
Complete blood count
D. 
Lipid profile
E. 
Thyroid function tests (thyroid-stimulating hormone and free T4)

77. A patient does not want her psychiatrist to testify in court because she believes that what she discussed with her doctor
is confidential. The legal principle most applicable to this situation is:

A. 
Autonomy
B. 
Mens rea
C. 
Respondeat superior
D. 
Capacity
E. 
Privilege

78. Which of the following agents is contraindicated in patients taking clozapine?

A. 
Carbamazepine
B. 
Fluoxetine
C. 
Clonazepam
D. 
Lithium
E. 
Nortriptyline

79. Which one of the following cognitive-enhancing agents is an N-methyl-d-aspartate (NMDA) receptor antagonist?

A. 
Galantamine
B. 
Donepezil
C. 
Rivastigmine
D. 
Memantine
E. 
Tacrine

80. Which one of the following is not a symptom of organophosphate exposure or poisoning?

A. 
Lacrimation
B. 
Salivation
C. 
Diarrhea
D. 
Vomiting
E. 
Mydriasis

81. 
Clostridium tetani causes trismus and dysphagia by:

A. 
Activating glutamate receptors
Inhibiting γ-aminobutyric acid (GABA) and glycine release in the brain and spinal cord
B. 
C. 
Blocking acetylcholine release at the neuromuscular junction
D. 
Blocking acetylcholinesterase in cholinergic synapses
E. 
Blocking calcium-mediated muscular contraction

336

Psychiatry Test Preparation and Review


Manual E-Book
Test Number Six

82. Low-voltage, mixed-frequency background with sleep spindles and K complexes is typically seen on EEGs during which
phase of sleep?

A. 
N1
B. 
N2
C. 
N3
D. 
N4
E. 
REM Sleep

83. In normal young adults, what percentage of total sleep time is spent in REM sleep?

A. 
5%
B. 
0%
C. 
20%
D. 
25%
E. 
35%

84. 
Emergency room treatment of patients with monoamine oxidase inhibitor (MAOI)-related hypertensive crisis may
involve administration of which of the following agents?

A. 
Diazepam
B. 
Hydrochlorothiazide
C. 
Phentolamine
D. 
Bromocriptine
E. 
Dantrolene

85. The emergency room treatment of an acute serotonin syndrome involves stopping the offending agent(s) and then:

A. 
Administering dantrolene
B. 
Hemodialysis
C. 
Administering bromocriptine
D. 
Supporting vital signs and functions
E. 
Administering oral or intramuscular lorazepam

86. Which of the following factors is most closely associated with child abuse and neglect?

A. 
Low socioeconomic status
B. 
Child with behavioral problems
C. 
Single-parent household
D. 
Large family size
E. 
Female child under 12 years of age

337

Psychiatry Test Preparation and Review


Manual E-Book
Psychiatry Test Preparation & Review Manual

87. A patient is doing individual therapy in an interpersonal psychotherapy (IPT) framework for depression. In the first five
sessions, the patient and therapist come to identify the patient’s unresolved grief at the recent death of his or her spouse
as the area of greatest concern, and this is related to the patient’s current feelings of depression. Which of the following
will be the focus of the next phase in the patient’s treatment?

A. 
Identifying and testing the validity of the patient’s maladaptive assumptions
B. 
Finding new activities and relationships to offset the patient’s loss
C. 
Providing the patient with a corrective emotional experience in the treatment
D. 
Resolving guilt about conflicted feelings toward the patient’s spouse
E. 
Reframing the patient’s catastrophizing and fortune-telling over the spouse’s death

88. When dialectical behavior therapy is used to treat patients with borderline personality disorder, the word “dialectical”
refers to therapeutic strategies focused on:

A. 
Alleviating psychic tension between the id and its fantasies and the superego and its responsibilities
B. 
Going over traumatic events and analyzing their effect on the patient
C. 
Reframing cognitive distortions and finding underlying mechanisms to negative thoughts
D. 
Role-playing that seeks to improve the patient’s interpersonal relationships
E. 
Seeking for synthesis between seemingly contradictory ideas and emotions

89. A 35-year-old patient presents with a long-standing history of orderliness and alphabetizing and color-coding of all
his belongings. This behavior can take him up to 2 hours every day to accomplish, and he is often late for work in the
mornings owing to his inability to stop this behavior. The patient denies obsessive thoughts and cannot explain the
ongoing need to alphabetize, color-code, and order everything. The most probable diagnosis in this case is:

A. 
Impulse control disorder NOS (not otherwise specified)
B. 
Obsessive–compulsive personality disorder
C. 
Body dysmorphic disorder
D. 
Anxiety disorder NOS
E. 
Obsessive–compulsive disorder

90. Which one of the following would not be considered a case of pedophilia?

A. 
A 60-year-old elementary school teacher has clandestine sex with a 12-year-old neighbor
B. 
A 20-year-old female camp counselor has clandestine sex with a 17-year-old male camper
C. 
A 25-year-old man has clandestine sex with his 13-year-old female cousin
D. 
A grandfather has clandestine sex with his 9-year-old granddaughter
E. 
A married couple, in their 30s, who are middle-school teachers at the same school, have a ménage-à-trois with an
11-year-old male student at their home

91. An 11-year-old elementary school student is given a trial of methylphenidate for attention deficit hyperactivity disorder
(ADHD), inattentive type. The dose is titrated appropriately by the psychiatrist, but the student continues to complain of
the core symptoms of ADHD. The next medication to be tried should be:

A. 
Guanfacine
B. 
Clonidine
C. 
Bupropion
D. 
Dextroamphetamine
E. 
Venlafaxine

338

Psychiatry Test Preparation and Review


Manual E-Book
Test Number Six

92. A 45-year-old woman is raped on an inner-city street late one night. She tells no one about the incident. The next day,
her coworkers note that she is absent from work, and a week later she is found 50 miles away in a nearby town mum-
bling incoherently to herself and wandering the streets. When taken to the local hospital emergency room, she is initially
unable to recall her name, her address, her birth date, or her telephone number. This presentation is most compatible
with a diagnosis of:

A. 
Dissociative amnesia
B. 
Dissociative fugue
C. 
Dissociative identity disorder
D. 
Posttraumatic stress disorder
E. 
Acute stress disorder

93. Which of the following categories of disorders classically has a female-to-male ratio of about 10 to 1?

A. 
Substance abuse disorders
B. 
Sexual disorders
C. 
Eating disorders
D. 
Dissociative disorders
E. 
Personality disorders

94. The diagnosis of encopresis is seen to occur most frequently in children with which of the following in their history?

A. 
Sexual abuse
B. 
ADHD
C. 
Conduct disorder
D. 
Oppositional defiant disorder
E. 
Learning disorders

95. A 25-year-old female comes to the psychiatrist complaining of irritability, poor concentration, and poor sleep. Which
one of the following should not be included in the differential diagnosis?

A. 
Generalized anxiety disorder
B. 
Major depressive disorder
C. 
Premenstrual dysphoric disorder
D. 
Posttraumatic stress disorder
E. 
Social anxiety disorder

96. The impulse control disorder of chronic pathological gambling is believed to be associated with which of the following
biological markers?

A. 
Decreased uptake in the brain on fluorodopa PET scans
B. 
Decreased dopamine levels in the nucleus accumbens
C. 
Increased serotonin and norepinephrine activity in the raphe and locus ceruleus
D. 
Decreased plasma 3-methoxy-4-hydroxyphenylglycol (MHPG) levels and decreased platelet MAO activity
E. 
Decreased glutamate activity at NMDA receptors in the brain

339

Psychiatry Test Preparation and Review


Manual E-Book
Psychiatry Test Preparation & Review Manual

97. Which one of the following is not typically associated with juvenile conduct disorder?

A. 
Children with a history of abuse or domestic violence victimization
B. 
Children of underemployed parents
C. 
Children with an onset of serious violations of rules before 15 years of age
Children with low plasma dopamine β-hydroxylase levels
D. 
E. 
Children with greater right frontal EEG activity at rest

98. The main factor differentiating binge eating disorder from bulimia nervosa is:

A. 
Lack of control over eating
B. 
Must occur once per week for 3 months
C. 
Consuming a large amount of food in a 2-hour period
D. 
Feelings of embarrassment
E. 
Absence of inappropriate compensatory behaviors

99. Which of the following famous psychoanalysts is responsible for the concepts of the “collective unconscious” and
“archetypes”?

A. 
Erich Fromm
B. 
Kurt Goldstein
C. 
Edith Jacobson
D. 
Carl Jung
E. 
Otto Kernberg

100. A 5-year-old child presents with confusion, increased salivation, lacrimation, fasciculations, miosis, tachycardia, and
hypertension. Which of the following poisons can cause these manifestations?

A. 
Opium
B. 
Strychnine
C. 
Botulism
D. 
Hexane
E. 
Organophosphate insecticide

101. Which of the following benzodiazepines is phase II metabolized but not phase I metabolized?

A. 
Diazepam
B. 
Clonazepam
C. 
Lorazepam
D. 
Alprazolam
E. 
Triazolam

102. In which pregnancy category are benzodiazepines?

A. 
Category A
B. 
Category B
C. 
Category C
D. 
Category D
E. 
Category E

340

Psychiatry Test Preparation and Review


Manual E-Book
Test Number Six

103. Which of the following is the most common cause of malpractice claims against psychiatrists?

A. 
Sexual assault
B. 
Improper termination
C. 
Suicide or attempted suicide
D. 
Boundary violations
E. 
Medication error

104. Which of the following is the most appropriate treatment for premature ejaculation?

A. 
SSRIs
B. 
Yohimbine
C. 
Testosterone
D. 
Alprostadil
E. 
Sildenafil

105. Which of the following disorders is not more common in males?

A. 
Autism
B. 
Obsessive–compulsive disorder
C. 
ADHD
D. 
Exhibitionism
E. 
Pathological gambling

106. For a socially harmful act to be considered a crime, it must contain which of the following?

A. 
Parens patriae
B. 
Mens rea
C. 
Respondeat superior
D. 
Substituted judgment
E. 
Justice

107. A patient with severe panic disorder comes to you for help. Which of the following is the most effective treatment you
can offer her?

A. 
Psychodynamic psychotherapy
B. 
An SSRI
C. 
Cognitive behavioral therapy
D. 
A TCA
E. 
Cognitive behavioral therapy plus an SSRI

108. 
Mahler’s stage that is characterized by a baby considering herself a fused entity with her mother but developing
increased ability to differentiate between the inner and the outer world is called:

A. 
Normal autism
B. 
Symbiosis
C. 
Differentiation
D. 
Rapprochement
E. 
Object constancy

341

Psychiatry Test Preparation and Review


Manual E-Book
Psychiatry Test Preparation & Review Manual

109. You start a patient in your practice on an SSRI for treatment of panic disorder. Which of the following medications can
you add that will be effective, is FDA-approved for panic disorder, and will lead to a more rapid response?

A. 
Buspirone
B. 
Gabapentin
C. 
Propranolol
D. 
Topiramate
E. 
Clonazepam

110. An elderly bipolar patient is brought into the emergency room. He was found comatose at home by his son. His serum
sodium is 115 mmol/L. Which of the following medications is the most likely cause of his sodium imbalance?

A. 
Lithium
B. 
Olanzapine
C. 
Carbamazepine
D. 
Quetiapine
E. 
Topiramate

111. Which of the following is not part of the recommended monitoring for patients taking risperidone?

A. 
Weight
B. 
Blood pressure
C. 
Glucose
D. 
Thyroid function tests
E. 
Lipid profile

112. A psychotherapist you work with recently lost his teenage daughter in a car accident. He proceeded to tell all of his
patients about this recent loss. This revelation to his patients is problematic because:

A. 
He should not reveal personal information to a patient
B. 
The only personal information he can reveal is his educational history
C. 
Openly sharing his grief will help his patients understand him
D. 
It is driven by his personal needs rather than the needs of the patient
E. 
He should reveal this information only to patients who have also lost children

113. Which of the following statistical measures would be used to quantify the degree of agreement between two raters in a
study?

A. 
Point prevalence
B. 
Period prevalence
C. 
Lifetime prevalence
D. 
Kappa
E. 
Correlation coefficient

342

Psychiatry Test Preparation and Review


Manual E-Book
Test Number Six

114. A female therapist is doing psychodynamic psychotherapy with a 29-year-old woman. She has been discussing issues
surrounding her marriage and her relationship with her mother, who was distant emotionally when the patient was
growing up. The patient becomes angry with the therapist when she requests therapy sessions three times per week and
the therapist states that she cannot accommodate that with her schedule. She accuses the therapist of disliking her. This
is an example of which of the following?

A. 
Rejection by the therapist
B. 
Delusion of an erotomanic nature
C. 
Transference on the part of the patient
D. 
A precursor to stalking by the patient
E. 
Displacement of anger onto the therapist

115. Which of the following is not an indication for neurologic imaging in a psychiatric patient?

A. 
New-onset psychosis
B. 
Acute mental status change with neurologic abnormalities
C. 
Delirium of unknown etiology
D. 
Acute mental status change in a 65-year-old patient
E. 
Mania

116. 
Which of the following types of therapy is considered the “gold standard” for treatment of obsessive–compulsive
disorder?

A. 
Exposure with response prevention
B. 
Motivational enhancement therapy
C. 
Dialectical behavior therapy
D. 
Dynamic psychotherapy
E. 
Psychoanalysis

117. A 35-year-old woman with bipolar disorder gives birth to a child with an abnormally formed tricuspid valve. Which of
the following medications was she most likely taking?

A. 
Haloperidol
B. 
Lithium
C. 
Valproic acid
D. 
Lamotrigine
E. 
Carbamazepine

118. Which of the following is not one of the psychological issues of pregnancy?

A. 
Pregnancy as a means of self-realization
B. 
Fear of inadequate mothering
C. 
Projection of hope onto the child to be
D. 
Absence of desire for sexual activity
E. 
Unconscious ambivalence about the effect on the dyadic relationship

343

Psychiatry Test Preparation and Review


Manual E-Book
Psychiatry Test Preparation & Review Manual

119. The son of a 95-year-old woman with severe dementia asks that his mother be deemed incompetent. This decision must
be made by:

A. 
The son
B. 
The psychiatrist
C. 
The internist
D. 
The hospital administrator
E. 
The court

120. In which of the following tests is a patient presented with geometric figures for about 10 seconds and then asked to
draw them from memory?

A. 
Benton Visual Retention Test
B. 
Wechsler Memory Scale
C. 
Bender Visual-Motor Gestalt Test
D. 
Wisconsin Card Sorting Test
E. 
Mini mental status examination

121. Which of the following defense mechanisms is considered immature?

A. 
Anticipation
B. 
Schizoid fantasy
C. 
Altruism
D. 
Humor
E. 
Suppression

122. Lithium has been associated with which one of the following?

A. 
Neutropenia
B. 
Hyperparathyroidism
C. 
Pancreatitis
D. 
Hepatic failure
E. 
Eosinophilic colitis

123. You are screening Thomas for brief psychodynamic psychotherapy. Which of the following is a sign that he is an appro-
priate candidate?

A. 
He sticks his hand down his pants and smears feces on your office chair
B. 
He is fighting with both of his parents
C. 
He takes sertraline every day
D. 
He has one friend who he talks to twice per year
E. 
He is able to identify and discuss his feelings

124. Which of the following is most associated with object relations theory?

A. 
Melanie Klein
B. 
Carl Jung
C. 
Jean Baker Miller
D. 
Anna Freud
E. 
Sigmund Freud

344

Psychiatry Test Preparation and Review


Manual E-Book
Test Number Six

125. Joe Lehobo is cachectic and addicted to crack. Dr. Medpusher wants to convince Joe to stop using crack, but Joe is
ambivalent and denies that crack use is a problem for him. How should Dr. Medpusher proceed?

A. 
Prescribe fluoxetine
B. 
Dialectical behavior therapy
C. 
Motivational enhancement therapy
D. 
Electroconvulsive therapy
E. 
Involuntary hospitalization

126. Jim is a pediatrician in a local hospital. He is trying to raise money to build a new treatment center for children with
developmental disabilities. He asks you to solicit your patients for contributions. You should agree to solicit funds from:

A. 
Only former patients
B. 
No patients
C. 
Only wealthy patients, because this is a good cause and they have money to give
D. 
Only patients with intact reality testing
E. 
Elderly patients who can leave a donation to the hospital in their will

127. An explanatory statement that links a feeling to its unconscious meaning is known as:

A. 
Confrontation
B. 
Clarification
C. 
Interpretation
D. 
Empathic validation
E. 
Affirmation

128. A psychiatrist routinely receives free dinners, play tickets, and short trips from a pharmaceutical representative. Which
of the following statements concerning this situation is correct?

A. 
There is no evidence that gifts influence physicians’ behavior
B. 
This situation represents a conflict of interest for the psychiatrist
C. 
This situation is ethical if no single gift is worth more than $500
D. 
Self-regulation by physicians is the most effective way to deal with this issue
E. 
Whereas doctors are not influenced by this behavior, support staff such as nurses, physicians’ assistants, and secretaries are

129. You have to interview a patient using an interpreter. Which of the following is the best way to proceed?

A. 
Speak in English but very loudly
B. 
Look at the patient while speaking
C. 
Look at the interpreter while speaking
D. 
Address all questions to the interpreter and let the interpreter ask the patient
E. 
Make sure you have not only an interpreter but also the patient’s family in the room so that they can all translate

130. In which of the following theories does human development move through predetermined steps and stages wherein each
stage has its own characteristics and needs that must be negotiated before moving forward?

A. 
Levinson’s developmental theory
B. 
The epigenetic view of development
C. 
Vaillant theory of happy childhood
D. 
Neurodevelopmental theory
E. 
Normality as process

345

Psychiatry Test Preparation and Review


Manual E-Book
Psychiatry Test Preparation & Review Manual

131. Which of the following tests is not considered routine monitoring for patients taking lithium?

A. 
Complete blood count
B. 
Serum electrolytes
C. 
24-hour urine for creatinine and protein
D. 
Blood urea nitrogen and creatinine
E. 
Thyroid function tests

131. A patient has a panic attack and then worries significantly about having more attacks. How long must the patient worry
to meet DSM 5 criteria for panic disorder?

A. 
1 week
B. 
2 weeks
C. 
1 month
D. 
3 months
E. 
6 months

133. How long must a 42-year-old man have excessive anxiety to meet DSM 5 criteria for generalized anxiety disorder?

A. 
6 weeks
B. 
2 weeks
C. 
6 months
D. 
2 months
E. 
5 days

134. Which of the following drugs is known for a severe hyperthermic syndrome that can progress to disseminated intravas-
cular coagulation, rhabdomyolysis, and liver and kidney failure, as well as death?

A. 
Heroin
γ-Hydroxybutyrate
B. 
C. 
Flunitrazepam
D. 
Methylenedioxymethamphetamine
E. 
Ketamine

135. Giving a patient cognitive behavioral therapy (CBT) plus an SSRI for social anxiety disorder will most likely result in
which of the following?

A. 
Better results than an SSRI alone
B. 
Better results than CBT alone
C. 
Poor results for refractory cases
D. 
No initial benefit over monotherapy with either an SSRI or CBT in most cases
E. 
Poorer response than psychodynamic psychotherapy alone

136. Which of the following is least likely to cause weight gain?

A. 
Clozapine
B. 
Olanzapine
C. 
Valproic acid
D. 
Lamotrigine
E. 
Lithium

346

Psychiatry Test Preparation and Review


Manual E-Book
Test Number Six

137. Which of the following is not a component of skills training for patients with addictions?

A. 
Changing beliefs
B. 
Stress management techniques
C. 
Increasing assertiveness
D. 
Examining unconscious symbolism behind fears
E. 
Improving interpersonal communication

138. Which of the following is considered incorrect concerning seclusion and restraint?

A. 
Seclusion and restraint are considered ethical when no less restrictive alternative is available
B. 
A nurse observing a seclusion may extend the seclusion time if she feels the patient is still agitated
C. 
A written order from a physician is needed to seclude a patient
D. 
Seclusion orders are time-limited
E. 
The patient’s condition while in seclusion should be regularly reviewed and documented

139. Which of the following daily methadone doses is the best to block cravings and result in reduction of subsequent drug
use for the average patient?

A. 
10 mg
B. 
20 mg
C. 
40 mg
D. 
80 mg
E. 
100 mg

140. A homeless man comes to your emergency room with a long history of alcohol use. He has been living in a local park
and prostituting himself to other men for money to buy alcohol. Because the weather has been warm, he has been bath-
ing in local ponds and streams. He has been drinking excessively lately and requests detoxification. You must decide
whether to send him to inpatient or outpatient detoxification. Which of the following factors will affect your decision
most?

A. 
Length of time living in the park
B. 
Condom use
C. 
Social supports
D. 
Source of alcohol
E. 
History of delirium tremens

141. A 23-year-old male with severe obsessions, compulsions, and tics comes to you for evaluation. Which of the following
would be the best medication combination for this patient?

A. 
Citalopram plus lorazepam
B. 
Paroxetine plus buspirone
C. 
Sertraline plus naltrexone
D. 
Fluoxetine plus alprazolam
E. 
Fluvoxamine plus haloperidol

347

Psychiatry Test Preparation and Review


Manual E-Book
Psychiatry Test Preparation & Review Manual

142. Which of the following choices does not present with psychotic symptoms?

A. 
Major depressive disorder
B. 
Hypomanic episodes
C. 
Delirium
D. 
Bipolar mixed episodes
E. 
Manic episodes

143. A patient comes to you and reports recurrent hypomanic episodes but denies any depressive symptoms. What is the
diagnosis?

A. 
Bipolar I disorder
B. 
Bipolar II disorder
C. 
Cyclothymic disorder
D. 
Substance-induced mood disorder
E. 
Other specified bipolar and related disorder

144. 
Which of the following medical conditions can present with panic attacks, elevated blood pressure, flushing, and
tremulousness?

A. 
Crohn’s disease
B. 
Raynaud’s phenomenon
C. 
Hypothyroidism
D. 
Hyperparathyroidism
E. 
Pheochromocytoma

145. Which of the following could be expected for a patient with social phobia compared with the general population?

A. 
Increased number of friendships
B. 
Higher level of education
C. 
Lower rates of suicide
D. 
Poorer marital function
E. 
Increased success in career advancement

146. Regularly scheduled primary care appointments are a crucial and standard part of treating which one of the following
diagnoses?

A. 
Panic disorder
B. 
Major depressive disorder
C. 
Illness anxiety disorder
D. 
Posttraumatic stress disorder
E. 
Delusional disorder somatic type

147. Which of the following factors does not increase the chances that an elderly patient will commit suicide?

A. 
Alcohol dependence
B. 
Being widowed
C. 
Good physical health
D. 
Being unemployed
E. 
Social isolation

348

Psychiatry Test Preparation and Review


Manual E-Book
Test Number Six

148. You have a patient on clozapine who develops a white blood cell (WBC) count between 2000 and 3000 with an abso-
lute neutrophil count (ANC) of 700. What is your next step in management?

A. 
Continue clozapine and repeat absolute neutrophil count (ANC)
B. 
Stop clozapine; you may rechallenge if ANC count increases in the future
C. 
Continue clozapine; draw twice-weekly ANC
D. 
Continue clozapine; draw weekly ANC for the next 4 weeks
E. 
Stop clozapine; you may not rechallenge this patient in the future

149. Prolongation of the QTc interval above which of the following is an indication to stop a QTc-prolonging neuroleptic?

A. 
100 milliseconds
B. 
200 milliseconds
C. 
300 milliseconds
D. 
400 milliseconds
E. 
500 milliseconds

150. You are doing a shift in the emergency room, and a patient comes in complaining of depressed mood. On further ques-
tioning you discover that he is also suffering from fatigue and cognitive inefficiency. He provides you with a list of
medications prescribed by his primary care physician. Of the following medications, which one could be responsible for
exacerbating the patient’s symptoms?

A. 
Interferon
B. 
Ranitidine
C. 
Verapamil
D. 
Modafinil
E. 
Amoxicillin

349

Psychiatry Test Preparation and Review


Manual E-Book
Psychiatry Test Preparation & Review Manual

Six
Answer Key – Test Number Six
1. D 26. E 51. E 76. C 101. C 126. B
2. C 27. E 52. E 77. E 102. D 127. C
3. C 28. B 53. E 78. A 103. C 128. B
4. D 29. D 54. B 79. D 104. A 129. B
5. D 30. D 55. C 80. E 105. B 130. B
6. A 31. B 56. D 81. B 106. B 131. C
7. C 32. C 57. E 82. B 107. E 132. C
8. B 33. E 58. B 83. D 108. B 133. C
9. E 34. A 59. B 84. C 109. E 134. D
10. E 35. D 60. D 85. D 110. C 135. D
11. C 36. B 61. A 86. A 111. D 136. D
12. B 37. A 62. C 87. B 112. D 137. D
13. D 38. C 63. E 88. E 113. D 138. B
14. D 39. D 64. A 89. E 114. C 139. D
15. A 40. D 65. E 90. B 115. E 140. E
16. C 41. E 66. C 91. D 116. A 141. E
17. D 42. B 67. D 92. B 117. B 142. B
18. B 43. D 68. A 93. C 118. D 143. E
19. E 44. C 69. B 94. A 119. E 144. E
20. A 45. E 70. A 95. E 120. A 145. D
21. C 46. A 71. C 96. D 121. B 146. C
22. B 47. C 72. E 97. C 122. B 147. C
23. A 48. E 73. B 98. E 123. E 148. B
24. D 49. D 74. D 99. D 124. A 149. E
25. C 50. C 75. E 100. E 125. C 150. A

350

Psychiatry Test Preparation and Review


Manual E-Book
Test Number Six

Explanations – Test Number Six


Six
Question 1. D. Trazodone is metabolized by the CYP 3A4 enzyme.
Psychopharmacology
K&S Chapter 29

Question 2. C. Transverse myelitis (TM) is defined as the development of isolated spinal cord dysfunction over hours or
days in patients in whom no evidence exists of a compressive lesion. TM is presumed to be autoimmune
in origin. Patients present with motor and sensory deficits below the lesion, usually in the form of a
paraplegia. The abnormalities are typically bilateral but may be asymmetrical. MRI may show enhance-
ment in the spinal cord. CSF analysis is nondiagnostic. TM is a clinical diagnosis. The primary differen-
tial diagnosis is between multiple sclerosis and neuromyelitis optica. Only about 7% of patients with
TM go on to develop multiple sclerosis.

Anterior spinal artery infarction usually causes paraparesis and spinothalamic sensory loss below the
level of the lesion. Dorsal column function is preserved. Spinal metastasis can cause a central cord defi-
cit, but this is unusual in a young patient such as the one presented in this question!
Neurology
B&D Chapter 80

Question 3. C. Lamotrigine has FDA approval for bipolar maintenance, and its efficacy has been demonstrated for both
mania and depression. Topiramate is approved for seizures and migraines but is used off-label for mania.
Carbamazepine is approved for treatment of acute mania. Valproic acid has been demonstrably effective
for acute mania. Gabapentin is approved for seizures, postherpetic neuralgia, and neuropathic pain.
Psychopharmacology
K&S Chapter 29

Question 4. D. Somnambulism (sleepwalking) is a parasomnia. It is common in children between ages 5 and 12.
Sometimes it persists in adulthood or rarely begins in adults. Sleepwalking begins with the abrupt onset
of motor activity arising out of slow-wave sleep (stage IV) during the first one-third of sleep. Episodes
generally last less than 10 minutes. There is a high incidence of positive family history. Injuries and
violent activity have been reported during sleepwalking episodes, but generally individuals can negoti-
ate their way around the room. Rarely, the occurrence of homicide has been reported, and sometimes
abnormal sexual behavior occurs. Sleep deprivation, fatigue, concurrent illness, and sedatives may act
as precipitating factors. Treatment involves taking appropriate precautions to prevent wandering and
troublesome behaviors. Medications that may improve the condition include imipramine and benzodiaz-
epines. Remember, also, that sleep terrors occur during the same stage of sleep as sleepwalking (stage IV,
also called slow-wave sleep).
Sleep–Wake Disorders
B&D Chapter 68

351

Psychiatry Test Preparation and Review


Manual E-Book
Psychiatry Test Preparation & Review Manual

Question 5. D. Rates of obesity have been increasing over recent decades. Some estimates predict that 85% of the U.S.
population will be overweight by 2030. Obesity rates in the U.S. are highest in middle-age adults (not the
elderly). Rates are higher for minority populations and low socioeconomic groups. Obesity rates are higher
for women than for men. Females are disproportionately affected by both obesity and extreme obesity.
Public Policy
Pharmacoeconomics July 2018; www.cdc.gov

Question 6. A. In Wernicke’s aphasia, expressive speech is fluent, but comprehension is impaired. Speech is devoid of
meaning, containing verbal paraphasias, neologisms, and jargon productions. Naming in Wernicke’s
aphasia is deficient. Auditory comprehension is impaired. Repetition is impaired. Reading comprehen-
sion usually is impaired. Writing also is impaired, but in a manner quite different from that of Broca’s
aphasia. The patient usually has no hemiparesis and can grasp the pen and write easily, unlike Broca’s,
which can involve motor deficits.

In 1909 Bálint described a syndrome in which patients act blind yet can describe small details of objects
in central vision. The disorder is usually associated with bilateral hemisphere lesions, often involving the
parietal and frontal lobes. Bálint’s syndrome involves a triad of deficits: (1) psychic paralysis of gaze,
also called ocular apraxia, or difficulty directing the eyes away from central fixation; (2) optic ataxia, or
incoordination of extremity movement under visual control; and (3) simultanagnosia, or loss of ability
to perceive more than one item at a time.

Anomic aphasia refers to aphasic syndromes in which naming is the principal deficit. Spontaneous
speech is normal, except for the pauses and circumlocutions produced by the inability to name.
Comprehension, repetition, reading, and writing are intact, except for the same word-finding difficulty in
written productions. Isolated, severe anomia may indicate focal left hemisphere pathology. The angular
gyrus is the purported site of lesions producing anomic aphasia, but lesions there usually produce other
deficits as well, including alexia and the four elements of Gerstmann’s syndrome: agraphia, right–left dis-
orientation, acalculia, and finger agnosia (inability to identify fingers).

Conduction aphasia is an uncommon but theoretically important syndrome that can be recognized by its
striking deficit of repetition. Most patients have relatively normal spontaneous speech. Naming may be
impaired, but auditory comprehension is preserved. Repetition may be disturbed to seemingly ridiculous
extremes, such that a patient who is capable of self-expression at a sentence level and can comprehend
conversation may be unable to repeat even single words. Associated deficits include hemianopia in some
patients; right-sided sensory loss may be present, but right hemiparesis usually is mild or absent. The
lesions of conduction aphasia usually involve either the superior temporal or the inferior parietal region.
Conduction aphasia has been advanced as a classical disconnection syndrome. The arcuate fasciculus, a
white matter tract traveling from the deep temporal lobe, around the sylvian fissure, to the frontal lobe,
is the site of disconnection.

Most psychotic patients speak in an easily understood, grammatically appropriate manner, but their
behavior and speech content are abnormal. Only rarely do schizophrenics speak in “clang association”
or “word salad” speech. Sudden onset of fluent, paraphasic speech in a middle-aged or elderly patient
should always be suspected of representing a left hemisphere lesion with aphasia.
Neurology
B&D Chapter 12A

Question 7. C. Accepting a small gift from a patient is acceptable under certain circumstances. The details of the case
are important in determining whether to accept the gift. Exploitation involves using the therapeutic rela-
tionship for personal gain, such as hiring a patient or going into business with a patient. You cannot
have any business interactions with patients aside from their paying you for treatment. Charging for
missed visits is considered ethical. Releasing information to the patient’s insurance company is ethical.
You should release only as much information as is necessary to process the claim or preapprove the visit,
etc. It is ethical to keep a separate set of psychotherapy notes for your therapy patients that are not part
of the medical record and to which the patient is not entitled access.
Ethics
K&S Chapter 36

352

Psychiatry Test Preparation and Review


Manual E-Book
Test Number Six

Question 8. B. Anterior cerebral artery (ACA) territory infarctions are uncommon. They occur in patients with vaso-
spasm after subarachnoid hemorrhage caused by ACA or anterior communicating artery aneurysm. The
characteristics of ACA infarction vary according to the site of involvement and the extent of collateral
blood flow. Contralateral weakness involving primarily the lower extremity is characteristic. Other
characteristics include abulia, pseudobulbar palsy, apathy, mutism, personality changes, and urinary
incontinence.

The anterior choroidal artery syndrome is often characterized by hemiparesis, hemisensory loss, and
hemianopia. The visual field defect with anterior choroidal artery syndrome infarcts is characterized by
a homonymous defect in the superior and inferior visual fields that spares the horizontal meridian. With
bilateral infarctions in the anterior choroidal artery syndrome territory, there can be pseudobulbar mut-
ism and a variety of other features, including facial diplegia, hemisensory loss, lethargy, neglect, and
affect changes.

Other answer choices are discussed in detail elsewhere in this volume.


Neurology
B&D Chapter 51A

Question 9. E. Of the choices given, only gabapentin is excreted unchanged in the urine. It has no interaction with the
CYP system and is not metabolized by the liver. The other choices are first metabolized in the liver, and
their metabolites are subsequently excreted in the urine and/or feces.
Psychopharmacology
K&S Chapter 29

Question 10. E. Transient amnesia is a temporary version of amnestic syndrome. The most striking example of transient
amnesia is the syndrome of transient global amnesia, lasting from several to 24 hours. In this syndrome,
an otherwise cognitively intact individual suddenly loses memory for recent events, asks repetitive ques-
tions about his or her environment, and sometimes confabulates. During the episode, the patient has
both anterograde and retrograde amnesia. As recovery occurs, however, the retrograde portion “shrinks”
to a short period, leaving a permanent gap in memory of the brief retrograde amnesia before the epi-
sode and the period of no learning during the episode. The syndrome is of unknown cause but can be
closely imitated by disorders of known etiology, such as partial complex seizures, migraine, and possibly
transient ischemia of the hippocampus on one or both sides. Studies do not prove an ischemic etiology
for transient global amnesia; rather, they indicate transient dysfunction in the hippocampus or its con-
nections. Drug intoxication, alcoholic “blackouts,” and minor head injuries can also produce transient
amnesia.
Neurology
B&D Chapter 6

Question 11. C. Nefazodone inhibits CYP 3A4 thereby increasing trazodone levels. Also, always keep in mind that grape-
fruit juice is also a 3A4 inhibitor. This is such a useful fact for the prudent student to know!
Psychopharmacology
K&S Chapter 29

Question 12. B. Psychogenic nonepileptic seizures are episodes that resemble epileptic seizures but do not have a neuro-
logic origin; rather, they are somatic manifestations of psychological distress. Inpatient video-EEG moni-
toring is the gold standard for the diagnosis of nonepileptic seizures. Definitive diagnosis is made when a
patient is observed having typical seizures without accompanying EEG abnormalities. From 5% to 10%
of outpatient epilepsy patients and 20% to 40% of inpatient epilepsy patients have psychogenic nonepi-
leptic seizures. Such patients inevitably have comorbid psychiatric illnesses, most commonly depression,
posttraumatic stress disorder, other somatoform and dissociative disorders, and character pathology,
especially borderline personality disorder or traits. Many patients have a history of sexual or physi-
cal abuse. Up to 40% of patients with nonepileptic seizures also have true epileptic seizures. Between
75% and 85% of patients with psychogenic nonepileptic seizures are women. Treatment involves dis-
continuation of antiepileptic drugs in patients without concurrent epilepsy and referral for appropriate

353

Psychiatry Test Preparation and Review


Manual E-Book
Psychiatry Test Preparation & Review Manual

psychiatric care. As an auxiliary investigation of suspected psychogenic seizures, plasma prolactin con-
centrations may provide additional supportive data. Plasma prolactin concentrations frequently are ele-
vated after tonic–clonic seizures and less frequently after complex partial seizures. Serum prolactin levels
almost invariably are normal after psychogenic seizures, although such a finding does not exclude the
diagnosis of true epileptic seizures. Elevated prolactin levels, however, also may be present after syncope
and with the use of drugs such as antidepressants, estrogens, bromocriptine, ergots, phenothiazines, and
antiepileptic drugs. Although a number of procedures are employed to help distinguish epileptic from
nonepileptic seizures, none of these procedures have both high sensitivity and high specificity. No proce-
dure attains the reliability of EEG-video monitoring, which remains the standard diagnostic method for
distinguishing between the two.
Neurology
B&D Chapter 2

Question 13. D. β-Blockers are first-line treatment for akathisia. Benzodiazepines may also help in some cases.
Anticholinergics like benztropine are not helpful. Bromocriptine is a mixed dopamine agonist/antago-
nist useful in neuroleptic malignant syndrome, Parkinson’s disease, elevated prolactin, and cocaine
withdrawal. Methylenedioxypyrovalerone is a stimulant chemical found in “bath salts.” It acts as a nor-
epinephrine dopamine reuptake inhibitor and has four times the potency of Ritalin. It can lead to anxi-
ety, paranoia, delusions, and hallucinatory excited delirium, as well as suicide, violence, or homicide. It
is a street drug of abuse and has no role in the treatment of akathisia. Clonidine is an α1 agonist that is
useful in treating hypertension, aggression, ADHD, and opiate withdrawal. Always remember that you
should not give beta blockers to patients with asthma due to airway constriction.
Psychopharmacology
K&S Chapter 29

Question 14. D. A characteristic of trigeminal neuralgia (TN), tic douloureux, consists of paroxysmal lancinating attacks
of severe facial pain. Both genders experience the disorder, with a slight female predominance, and it
is most common after age 50. Characteristics of classic TN include an abrupt onset and termination
of unilateral brief electric-shock-like pain. Pain often is limited to the distribution of one or two (com-
monly the second and third) divisions of the trigeminal nerve. Trivial stimuli, including washing, shav-
ing, smoking, talking, and brushing the teeth (trigger factors), can evoke the pain. Most affected patients
are symptom-free between attacks, and clinical examination is usually normal. Attacks of TN occur in
clusters, and remissions can last for months. The cause of the pain attacks is unknown. Compression of
the trigeminal nerve by benign tumors and vascular anomalies may play a role in the development of
clinical symptoms.

Carbamazepine is the drug of first choice for treatment of TN. Carbamazepine monotherapy provides
initial symptom control in as many as 80% of the patients. Of those initially responding, approxi-
mately 75% will continue to have long-term control of pain attacks. Controlled studies demonstrate
that baclofen and lamotrigine are superior to placebo for treatment of TN. In the experience of many
clinicians, baclofen is just as effective as carbamazepine and often better tolerated. Baclofen could be
an alternate choice for initial drug therapy. Other medications that may be effective include oral gaba-
pentin, clonazepam, oxcarbazepine, topiramate, and phenytoin. If a patient is not satisfied with single-
medication therapy, adding another oral medication may offer additional benefit. Intravenous lidocaine
or phenytoin may be effective for some severe refractory cases of TN. These treatments carry additional
risks, however, and require close cardiovascular monitoring. Opioid analgesics are not effective for TN.
Neurology
B&D Chapter 44

Question 15. A. Aspirin, the oldest and most commonly used nonprescription drug in the world, is the standard medi-
cal therapy for prevention of stroke in patients with transient cerebral ischemia, as well as for reducing
the risk for recurrent stroke and postoperative strokes after carotid endarterectomy. Aspirin is effective,
inexpensive, and safe if started within 48 hours of acute ischemic stroke. Meta-analyses have shown
that aspirin reduces the combined risk for stroke, myocardial infarction (MI), and vascular death by
approximately 25%. The mechanism of action of aspirin is the irreversible inhibition of platelet function

354

Psychiatry Test Preparation and Review


Manual E-Book
Test Number Six

by inactivation of cyclo-oxygenase. The antiaggregant effect is seen within 1 hour after administration.
Aspirin is also antiinflammatory and antioxidant and may increase fibrinolytic activity up to 4 hours
after administration. The main side effect of aspirin is gastric discomfort. Gastrointestinal hemorrhage
occurs in 1% to 5% of cases.

Evidence from several clinical studies favors the use of platelet antiaggregants as the first line of therapy
in patients at high risk for stroke. Results of primary prevention trials do not support the use of aspirin
for primary stroke prevention. However, aspirin at 81 mg every other day was effective in primary pre-
vention of stroke in older women.

Oral anticoagulation with warfarin is indicated for primary and secondary prevention of stroke in
patients with nonventricular atrial fibrillation. Ticlopidine and clopidogrel have antiplatelet effects.
Ticlopidine reduces the relative risk for death or nonfatal stroke by 12% in comparison with aspirin.
Ticlopidine has more side effects than aspirin, including diarrhea, nausea, dyspepsia, and rash. The
Clopidogrel versus Aspirin in Patients at Risk of Ischemic Events study (CAPRIE) showed that clopi-
dogrel was modestly more effective than aspirin in reducing the combined risk for ischemic stroke, MI,
and vascular death in patients with atherosclerotic vascular disease. Clopidogrel is a platelet adenosine
diphosphate receptor antagonist. Overall, the tolerability of clopidogrel was excellent, with no increased
incidence of neutropenia and a lower incidence of gastrointestinal hemorrhage and peptic, gastric, or
duodenal ulcers compared with aspirin. Despite this evidence, aspirin is still the recommended treat-
ment of choice 48 hours following an ischemic cerebrovascular event. As an added bonus to this ques-
tion, keep in mind that when you are anticoagulating a patient with warfarin, the patient should not be
given vitamin E or ginkgo biloba. Combining these medications increases risk for bleeding and should be
avoided. This is a scenario which has been known to appear on standardized examinations as well as in
everyday clinical practice, so the prudent student should definitely keep it in mind.
Neurology
B&D Chapter 51A, PDR for Herbal Medicines 4th edition

Question 16. C. Carpal tunnel syndrome is by far the most


common entrapment neuropathy. Because
the transverse carpal ligament is an unyield-
ing fibrous structure forming the roof of the
tunnel, tenosynovitis or arthritis in this area
often produces pressure on the median nerve.
Symptoms consist of nocturnal pain and par-
esthesias, most often confined to the thumb,
index, and middle fingers. Patients complain
of tingling numbness and burning sensations,
often awakening them from sleep. Referred
pain may radiate to the forearm and even
as high as the shoulder. Symptoms are often
worse after excessive use of the hand or wrist.
As such, repetitive stress injuries, from typ- Thenar atrophy in chronic bilateral carpal tunnel syndrome.
ing, using a screwdriver, or prolonged driving,
may aggravate the condition. Objective sensory changes may be found in the distribution of the median
nerve, most often experienced as impaired two-point discrimination, pinprick and light touch sensa-
tion, or occasionally hyperesthesia in the thumb and index fingers with sparing of the thenar eminence.
Thenar (abductor pollicis brevis muscle) weakness and atrophy may be present with prolonged entrap-
ment (see photo).

The syndrome is frequently bilateral and usually of greater intensity in the dominant hand. A positive
Tinel’s sign, in which percussion of the nerve at the carpal tunnel causes paresthesias in the distal dis-
tribution of the median nerve, is present in approximately 60% of affected patients but is not specific
for carpal tunnel syndrome. Flexing the patient’s hand at the wrist for 1 minute (Phalen’s maneuver) or
hyperextension of the wrist (reversed Phalen’s maneuver) can reproduce the symptoms.

355

Psychiatry Test Preparation and Review


Manual E-Book
Psychiatry Test Preparation & Review Manual

Approximately one in five pregnant women reports nocturnal hand paresthesias, primarily during the
last trimester, often associated with peripheral edema. This irritation can be expected to disappear spon-
taneously within weeks after giving birth. During pregnancy, conservative therapy is indicated. Splinting
of the wrist in the neutral position is helpful. Additionally, some physicians inject corticosteroids into
the carpal tunnel. When hand muscles supplied by the median nerve weaken, surgical decompression is
indicated.

Although it is frequently (mis)diagnosed, neurogenic thoracic outlet syndrome is a rare entity. Most
patients are women, with a mean age at onset of 32 years. Pain is usually the first symptom, with either
aching noted on the inner side of the arm or soreness felt diffusely throughout the limb. Tingling sensa-
tions accompany pain and are felt along the inner side of the forearm and in the hand.

Arm pain and weakness are the cardinal manifestations of idiopathic brachial plexopathy. It occurs in
all age groups, especially in those engaged in vigorous athletic activities. In some patients the plexopathy
follows an upper respiratory tract infection, a flu-like illness, an immunization, surgery, or psychologi-
cal stress, or it occurs postpartum. The illness begins with the abrupt onset of intense pain described as
sharp, stabbing, throbbing, or aching and located in a variety of sites, including the shoulder, scapular
area, trapezius ridge, upper arm, forearm, and hand. The pain may last from hours to many weeks, and
then it gradually abates. Lessening of pain is associated with the appearance of weakness. Weakness
may progress for 2 to 3 weeks after the onset of pain. The natural history of brachial plexus neuropathy
is benign; improvement occurs in the vast majority of patients, even in those with considerable muscle
atrophy.
Neurology
B&D Chapters 75, 76, and 81

Question 17. D. Specific phobia is the most common anxiety disorder. Lifetime prevalence of social phobia is around
10%. It is the most common mental disorder among women and the second most common among men
(after substance abuse). This fact takes many psychiatrists by surprise, however, because most patients
with specific phobia do not seek medical attention.
Anxiety Disorders
K&S Chapter 9

Question 18. B. Neurofibromatosis (NF) is actually two separate diseases, each caused by a different gene. NF type 1
(NF1), or von Recklinghausen’s disease, is the most common of the neurocutaneous syndromes, occur-
ring in approximately 1 in 3000 people. The clinical triad of NF 1 is café-au-lait spots, neurofibromas,
and Lisch nodules. NF type 2 (NF2) is characterized by bilateral acoustic Neuromas (vestibular schwan-
nomas) and often is associated with other brain or spinal cord tumors. Acoustic neuromas can steadily
impair hearing until deafness ensues. NF2 does not lead to behavioral, learning, or cognitive impair-
ments the way NF1 does. Inheritance for both is an autosomal dominant pattern, but approximately
half of NF1 cases result from a spontaneous mutation. A mutation of the NF1 gene on chromosome
17 causes NF1. Remember this with the mnemonic “von Recklinghausen contains 17 letters.” The NF1
gene product, neurofibromin, functions to inhibit cell proliferation. A mutation of the NF2 gene on chro-
mosome 22 causes NF2. Remember this with the pneumonic “chromosome 22 carries NF2.” The NF2
protein product is schwannomin or merlin. The NF2 gene suppresses tumor function. Dysfunction of
the NF2 gene accounts for the occurrence of multiple CNS tumors in patients with NF2. The diagnostic
criteria for NF1 and NF2 are listed in the following box.

Lisch nodules are pigmented iris hamartomas that are pathognomonic for NF1. Lisch nodules do not
cause symptoms; their significance lies in their implications for the diagnosis of NF1. Optic nerve glio-
mas are the most common CNS tumors caused by NF1. Patients with NF2 have few cutaneous lesions.
To meet criteria for NF2, patients must go on to develop bilateral schwannomas of the auditory nerve
(acoustic neuromas).

356

Psychiatry Test Preparation and Review


Manual E-Book
Test Number Six

Neurofibromatosis Type 1 (any two or more)


• Six or more café-au-lait lesions more than 5 mm in diameter before puberty and more than 15 mm
in diameter afterward
• Freckling in the axillary or inguinal areas
• Optic glioma
• Two or more neurofibromas or one plexiform neurofibroma
• A first-degree relative with NF type 1
• Two or more Lisch nodules
• A characteristic bony lesion (sphenoid dysplasia, thinning of the cortex of long bones, with or
without pseudarthrosis)
Neurofibromatosis Type 2
Bilateral VIIIth nerve tumor (shown by MRI, CT, or histological confirmation)

• A first-degree relative with NF type 2 and a unilateral VIIIth nerve tumor
• A first-degree relative with NF type 2 and any two of the following lesions: neurofibroma,
­meningioma, schwannoma, glioma, or juvenile posterior subcapsular lenticular opacity

Neurology
B&D Chapter 65

Question 19. E. Culture correlates most with ethnicity. People can be of the same race, age, gender, or nationality and
have very different cultures.
Cultural Issues in Psychiatry
K&S Chapter 3

Question 20. A. The porphyrias are caused by enzymatic defects in the heme biosynthetic pathway. Acute intermit-
tent porphyria (AIP) may give rise to acute episodes of potentially fatal symptoms, such as abdomi-
nal pain, delirium, psychosis, neuropathy, and autonomic instability. AIP follows an autosomal
dominant pattern of inheritance and is due to a mutation in the gene for porphobilinogen deami-
nase. The disease is characterized by attacks that may last days to weeks, with relatively normal
function between attacks. The episodic nature, clinical variability, and unusual features may cause
symptoms to be attributed to somatization, conversion, or other psychiatric conditions. Attacks may
be spontaneous but are typically precipitated by a variety of factors, such as infection, alcohol use,
pregnancy, anesthesia, and numerous medications that include antidepressants, anticonvulsants, and
oral contraceptives.

Porphyric attacks usually manifest with a triad consisting of abdominal pain, peripheral neuropathy,
and neuropsychiatric symptoms. Seizures may also occur. Abdominal pain is the most common symp-
tom. During attacks, excessive porphyrins can color the urine red. A variety of cognitive and behavioral
changes can occur, including anxiety, restlessness, insomnia, depression, mania, hallucinations, delusions,
confusion, catatonia, and psychosis. The diagnosis can be confirmed during an acute attack of AIP by
measuring urine porphobilinogens. Acute attacks are treated with avoidance of precipitating factors
(e.g., medications), intravenous hemin, intravenous glucose, and pain control.
Neurology
B&D Chapters 9 and 62

357

Psychiatry Test Preparation and Review


Manual E-Book
Psychiatry Test Preparation & Review Manual

Question 21. C. Lithium is cleared through the kidneys. It is reabsorbed in the proximal tubules with sodium. When
a thiazide diuretic, such as hydrochlorothiazide, is added, it leads to sodium depletion, which in turn
causes the reabsorption of sodium and lithium in the proximal tubules. This increased lithium reabsorp-
tion added to the lower fluid levels as a result of the diuretic can lead to an increased lithium level and
lithium toxicity. Valproic acid and clozapine are metabolized through the liver and would not be affected
by a thiazide diuretic.
Psychopharmacology
K&S Chapter 29

Question 22. B. The first medication approved by the FDA for use in multiple sclerosis (MS) was recombinant interferon
β-1b (Betaseron). It has been proven to reduce the accumulation of plaques and reduce disability in
patients with relapsing remitting MS. Interferon β-1b is administered subcutaneously every other day by
self-injection. Side effects include influenza-like symptoms that usually diminish over weeks to months,
depression, and reactions at the injection site.

Glatiramer acetate (Copaxone) is a synthetic polypeptide administered by daily injection. Patients receiv-
ing glatiramer acetate have a 29% reduction in relapse rate over 2 years. Tizanidine and Lioresal are
muscle-relaxing agents that are used to relieve some of the spasticity caused by MS. These are not disease-
modifying agents. Prednisone, an oral corticosteroid, is given after an acute MS exacerbation to reduce
the disease burden of an acute attack. Acute attacks are typically treated with intravenous corticosteroids.
Indications for treatment of a relapse include functionally disabling symptoms with objective evidence of
neurologic impairment. Thus, mild sensory attacks are typically not treated. In the past, corticotropin and
oral prednisone were primarily used. More recently, treatment with short courses of intravenous methyl-
prednisolone for 3 to 7 days, with or without a short prednisone taper, has commonly been used.
Neurology
B&D Chapter 54

Question 23. A. The most likely effects of ipecac abuse are cardiomyopathy, enlarged heart, increased QTc interval,
increased creatine kinase–MB, decreased ejection fraction, tricuspid or mitral valve insufficiency, dys-
rhythmia, low WBC, and increased liver function tests. One would not expect pancreatitis (leading to an
increase in amylase) or infection.
Feeding and Eating Disorders/Laboratory Tests in Psychiatry
K&S Chapter 15

Question 24. D. Subcortical arteriosclerotic encephalopathy (SAE), also known as Binswanger’s disease, is a form of sub-
cortical vascular dementia. Originally described in younger patients with hypertension (ages 50 to 65), this
disorder is characterized by a gradual (or stepwise) progressive dementia associated with frequent clinical
strokes, motor and sensory deficits and seizures, multiple subcortical infarcts, white matter lesions (WML),
and ventricular enlargement. MRI can document that white matter abnormalities accompany the subcorti-
cal lacunes in a majority of cases. Therefore, the term used to describe this entity is subcortical ischemic
vascular dementia, which incorporates subcortical WMLs and lacunar infarctions under the same subtype.

Gradual onset of cognitive difficulties is the first sign in more than half of the cases. Memory deficits,
apathy, and slowed thinking are prominent. Acute episodes of neurologic dysfunction may occur at the
onset or during the course; weakness, discoordination, and slurred speech are typical. Frequent falls
often are reported. In some instances, these variable neurologic signs and symptoms are absent, and the
patient experiences only slowly progressing mental deterioration; such cases initially may be diagnosed
as neurodegenerative disease, such as Alzheimer’s disease.
Neurology
B&D Chapter 66

Question 25. C. Psychotherapeutic treatment for posttraumatic stress disorder (PTSD) includes behavior therapy, cogni-
tive therapy, and exposure therapy. Eye movement desensitization and reprocessing is a therapy in which
the patient focuses on the lateral movement of the clinician’s finger while maintaining a mental image of
the trauma experience. It has significant research data supporting its efficacy, and some evidence suggests

358

Psychiatry Test Preparation and Review


Manual E-Book
Test Number Six

it is possibly more effective than other treatments for PTSD. Psychological debriefing uses interviews,
groups, and education to prevent the development of symptoms. Evidence of its effectiveness is unclear,
and it is still considered controversial. Some data suggest that it could actually exacerbate PTSD symp-
toms or impede recovery in some individuals. It is not considered first line or the treatment of choice for
acute stress disorder or PTSD.
Trauma- and Stress-Related Disorders
K&S Chapter 11

Question 26. E. Essential tremor (ET) is one of the most common movement disorders. The prevalence increases steadily
with age, occurring in up to 10% of patients older than age 60 years. In its purest form, ET is a mono-
symptomatic illness characterized by gradually increasing amplitude postural and kinetic tremor of the
forearms and hands (with or without involvement of other body parts) in the absence of endogenous or
exogenous triggers or other neurologic signs.

The typical patient becomes aware of a barely perceptible postural or action tremor, usually in the dis-
tal arms and hands. The head and lower limbs are less commonly affected. Head tremor (titubation) is
milder than limb tremor and is predominantly of a side-to-side, “no–no” type. Handwriting is particu-
larly troublesome. A striking improvement after ingestion of a small amount of alcohol is seen in 50%
of patients and may be helpful in diagnosis. Over time, the tremor worsens, causing increasing func-
tional disability. ET is thought to be a monosymptomatic illness without changes in cognition, strength,
coordination, or muscle tone, and the results of the neurologic examination are usually normal.

As many as two-thirds of patients give a positive family history of tremor. Patients with mild ET
whose main source of disability is tremor during meals and whose tremors respond to alcohol often
benefit from a cocktail before meals. The two most commonly used pharmacological treatments are
β-adrenergic blockers and primidone.

Stereotactic thalamotomy has been reported to suppress contralateral tremor. The effect appears to be
long-lasting. Thalamic deep brain stimulation has shown very good efficacy in controlling ET and can be
performed safely with long-lasting benefits. Deep brain stimulation should be considered for cognitively
intact, otherwise healthy patients with disabling, medication-resistant tremor.
Neurology
B&D Chapter 75

Question 27. E. In the “communication stage,” from 55 months onward, a child can use language to tell a story, share
ideas, and discuss alternatives. The child can understand concepts such as number, speed, time, space,
and left vs right. Speech at this stage is 100% intelligible.
Human Development
K&S Chapter 2

Question 28. B. Wernicke’s encephalopathy is due to thiamine deficiency. Although the most common clinical setting for
this disorder is chronic alcoholism, a large number of cases occur in other conditions, with the only pre-
requisite being a poor nutritional state. Wernicke’s presents with a triad of confusion, ophthalmoplegia,
and ataxia. The confusional state develops over days or weeks and is characterized by inattention, apa-
thy, disorientation, and memory loss. Ophthalmoplegia, when present, may involve horizontal nystag-
mus on lateral gaze, and many also have vertical nystagmus on upgaze. Truncal ataxia is common, but
limb ataxia is not, which are findings similar to those seen in alcoholic cerebellar degeneration.

Lesions of the nuclei of cranial nerves III, VI, and VIII are responsible for the eye findings. The truncal
ataxia is probably caused by the vestibular dysfunction and involvement of the superior cerebellar vermis.

Wernicke’s encephalopathy is a clinical diagnosis, although MRI may show signal abnormalities in the
periaqueductal regions, medial thalami, and bilateral mammillary bodies.

Patients suspected of Wernicke’s encephalopathy should receive thiamine before administration of glu-
cose to avoid precipitation of acute symptom worsening. Thiamine is the only treatment known to alter
the outcome.

359

Psychiatry Test Preparation and Review


Manual E-Book
Psychiatry Test Preparation & Review Manual

If left untreated, Wernicke’s encephalopathy is progressive. The mortality, even with thiamine treatment,
was 10% to 20% in early studies.

In Korsakoff’s syndrome, memory is impaired out of proportion to other cognitive functions. The
memory impairment is characterized by the presence of both anterograde and retrograde amnesia.
Confabulation can be a prominent feature, especially in the early stages.

Despite treatment with thiamine, improvement in memory function is slow and may be incomplete.

Marchiafava and Bignami described a syndrome of selective demyelination of the corpus callosum in
alcoholics who indulged in large quantities of wine. The disease seems to affect severe and chronic alco-
holics in their middle or late adult life. Because of the background history of alcohol abuse, a nutritional
cause has been invoked, but no nutritional factor has been identified. A toxic cause, such as direct tox-
icity of ethanol or other constituents, seems equally plausible. The neurologic presentation is that of
a variable combination of mental and motor slowing, personality and behavior changes, incontinence,
dysarthria, seizures, and hemiparesis. Occasionally patients present with coma. The most common is a
frontal lobe or dementing syndrome. Sucking, grasping, and gegenhalten (hypertonia characterized by
involuntary resistance during passive movement) may be prominent. Pathologically, there is selective
involvement of the central portion of the corpus callosum. There also may be symmetrical involvement
of other white matter tracts. Treatment should be directed at nutritional support and rehabilitation from
alcoholism. In those patients who recovered, it is not clear whether improvement was a result of vitamin
supplementation or merely a reflection of the disease’s natural history.
Neurology
B&D Chapter 57

Question 29. D. The judgment of line orientation test is useful in detecting right hemisphere disease. This test involves
matching lines of the same slope. A patient with right hemisphere disease will have a very hard time
with this, whereas a patient with only left hemisphere disease can complete the task without difficulty.
The Boston diagnostic aphasia examination is just that—an examination for aphasia. The sentence
completion test is used to test the patient’s associations with areas of interest by having the patient
complete sentences. Responses that are particularly emotional or are repeated are noted by the exam-
iner. The thematic apperception test has patients create a story based on pictures they are presented
with. The stories the patients generate tell the examiner about their thoughts, feelings, organization,
assumptions, etc.
Psychological Theory and Psychometric Testing
K&S Chapter 5

Question 30. D. Progressive supranuclear palsy (PSP) is a


progressive illness characterized by vertical
supranuclear ophthalmoplegia, axial rigid-
ity, pseudobulbar palsy, and mild dementia.
PSP typically begins in the sixth to seventh
decades of life with gait disorder and fall-
ing. Patients develop an akinetic-rigid state
with symmetrical signs and prominent axial
rigidity. In contrast to the flexed posture of
patients with Parkinson’s disease, those with
PSP may have an extended trunk or retrocol-
lic neck posture (drawing back the head). A
characteristic facial appearance with a wide-
eyed stare, furrowing of the forehead, and
deepening of other facial creases allows expe-
rienced clinicians to make an instant diagno- Typical facial expression of a patient with progressive
sis. Frontal lobe features are common. There supranuclear palsy, illustrating worried or surprised appearance,
is striking executive dysfunction early in the with furrowed brow and fixed expression of lower face.

360

Psychiatry Test Preparation and Review


Manual E-Book
Test Number Six

disease course; concrete thought, difficulty shifting set, decreased verbal fluency, and personality changes,
such as impulsivity and poor judgment, are nearly universal.

Dopaminergic agents, particularly levodopa (LD), may provide temporary improvement in bradykinesia,
but LD usually does not improve dysarthria, gait, or balance problems. The prognosis of PSP is poor,
with serious effect on quality of life and a median duration of survival of approximately 8 years.

Pseudobulbar palsy develops when there is disease involvement of the corticobulbar tracts that involve
speech control, mastication, and swallowing. Spontaneous or unmotivated crying and laughter uniquely
characterize pseudobulbar palsy. This is also termed emotional lability, hyperemotionality, labile affect,
or emotional incontinence and is often a source of great embarrassment to the patient.
Neurology
B&D Chapters 71 and 74

Question 31. B. Of all the answer choices, only introjection is an immature defense. All other choices in this question
are mature. Other immature defenses include acting out, blocking, hypochondriasis, regression, passive–
aggressive behavior, somatization, projection, and schizoid fantasy. We will not take the space to define
all 12 defenses listed in this question and answer explanation, but suffice it to say they are all fair game
for a standardized examination, and you should take the time to learn any that are unfamiliar to you.
You may also see questions on several of them elsewhere throughout this book.
Psychological Theory and Psychometric Testing
K&S Chapter 4

Question 32. C. Cervical dystonia is the most frequently diag-


nosed form of focal dystonia. Patients with
cervical dystonia present with neck pain,
difficulty maintaining a normal head posi-
tion, and sometimes tremor. Dystonic tremor
is usually an irregular oscillatory move-
ment that stops when the patient is allowed
to place the head or limb in the position of
the dystonic pulling, the so-called null point.
There is a directional preponderance to dys-
tonia movements, which is usually main-
tained throughout the course of the disease.
Many studies have suggested that focal and
segmental dystonia might have a genetic
basis. The locus has been mapped to chromo-
some 18. Botulinum toxin injections are very
helpful in the treatment of focal and seg-
mental dystonia. Botulinum toxin is injected
subcutaneously over the facial muscles or A patient with cervical dystonia (torticollis) with head rotation
directly into larger deeper muscles that to the right demonstrating marked hypertrophy of the left
underlie pain and inappropriate movement sternocleidomastoid muscle.
in other focal dystonias. Many clinicians use
electromyography (EMG) to help guide toxin injection. Botulinum toxin injections have been proven
effective in the treatment of blepharospasm and other facial dystonias, as well as cervical dystonia.
Clinical experience suggests they are useful in the treatment of oromandibular, laryngeal, truncal, and
limb dystonia. Overall, more than 75% of treated patients report moderate to marked improvement in
dystonic pain or posture. The procedure is generally well tolerated, with excessive weakness of injected
muscles or occasionally neighboring muscles the most often reported side effect.
Neurology
B&D Chapter 75

361

Psychiatry Test Preparation and Review


Manual E-Book
Psychiatry Test Preparation & Review Manual

Question 33. E. This question is really about orgasmic disorder. To properly diagnose orgasmic disorder, you must rule
out possible medical causes, including medications. Some common causes of anorgasmia include alcohol,
marijuana, SSRIs, TCAs, benzodiazepines, diabetes, spinal cord damage, hormones, pelvic injury, cardiac
problems, liver disease, or kidney disease.
Orgasmic disorder is defined as a recurrent delay in or absence of orgasm approximately 75% to 100%
of the time. It must cause marked distress. DSM 5 divides the diagnosis into female orgasmic disorder
and delayed ejaculation. It occurs more frequently in women than in men. The best treatment for female
orgasmic disorder is directed masturbation training consisting of several weekly therapy sessions involving
graded exposure to genital stimulation.
Sexual Dysfunction
K&S Chapter 17

Question 34. A. Juvenile myoclonic epilepsy (JME) tends to onset between 12 and 18 years. The characteristic feature is
sudden, mild to moderate myoclonic jerks of the shoulders and arms that usually occur after awakening.
Generalized tonic–clonic seizures develop in 90% of cases, and approximately one-third of the patients
have absence seizures. Myoclonic seizures precede generalized tonic–clonic seizures in approximately
one-half of the patients. Initially, myoclonic jerks are mild and explained as nervousness, clumsiness, or
tics. They may be unrecognized as seizures until a generalized tonic–clonic seizure brings the patient to
medical attention. Even though JME manifests as a clinically well-defined disorder with some variation,
different genetic and pathophysiological mechanisms seem to be responsible for the condition.
Valproic acid controls the seizures, but lifetime treatment is necessary. Lamotrigine, levetiracetam, topira-
mate, and zonisamide may be considered in patients who do not tolerate valproic acid. Ethosuximide is
useful to treat uncontrolled absence seizures.
Benign childhood epilepsy with centrotemporal spikes (BECTS) is also referred to as benign rolandic
epilepsy. Seizures begin between 3 and 13 years of age, with a peak onset at 9 to 10 years of age. The
typical seizure characteristics are somatosensory disturbance of the mouth, preservation of conscious-
ness, excessive pooling of saliva and tonic or tonic–clonic activity of the face, and speech arrest when the
dominant hemisphere is affected. The somatosensory or motor activity may spread to the arm. Secondary
generalization may also occur. Seizures typically occur shortly after sleep onset, just before awakening, and
with naps. Headache and vomiting may be associated as well. Seizures usually stop in adolescence, and the
outcome is favorable. Some children with BECTS have cognitive or behavioral problems, particularly dif-
ficulty with sustained attention, reading, and language processing. Because the overall prognosis for benign
childhood epilepsy is good, treatment is not required after the first or even the second seizure, and some
children never require treatment. Most anticonvulsant drugs are effective as monotherapy.
Neurology
B&D Chapter 67

Question 35. D. The metabolism of tricyclic antidepressants by CYP 2D6 is an important topic that has led to an FDA-
recommended precaution. Tricyclics are metabolized by CYP 2D6, and as such, anything that decreases the
activity of CYP 2D6 will increase the plasma level of the tricyclic, even into the toxic range. Some people
are naturally “poor metabolizers” who have decreased CYP 2D6 activity and, as such, higher plasma levels.
Cimetidine inhibits CYP 2D6 and, as a result, will increase tricyclic levels. The same holds true for quinidine.
Several drugs are substrates for CYP 2D6 and will decrease its ability to clear tricyclics when present. These
include fluoxetine, sertraline, paroxetine, carbamazepine, phenothiazines, propafenone, and flecainide.
Psychopharmacology
K&S Chapter 29

Question 36. B. Irritable bowel syndrome presents with abdominal cramping, diarrhea, or constipation. Core symptoms
include changes in consistency or frequency of stool, feeling as if the bowel is not emptied completely, mucus
in stool but no blood, and abdominal distention. It is more common in women. The cause is unknown and
there tends to be comorbidity with psychiatric conditions. TCAs and SSRIs can be used to reduce symptoms.
For many patients, certain foods trigger symptoms, so diet changes can be beneficial. Remember that studies
show that the use of Nortriptyline in IBS leads to global improvement and reduced abdominal pain.
Somatic Symptom Disorders
K&S Ch. 13

362

Psychiatry Test Preparation and Review


Manual E-Book
Test Number Six

Question 37. A. Erikson developed the concept of the epigenetic principle. The epigenetic principle states that human
development occurs in sequential, clearly defined stages, and that each stage must be properly resolved
for development to proceed normally. Other famous therapists listed in this question are the subject of
their own questions elsewhere in this text.
Human Development
K&S Chapter 4

Question 38. C. An isolated, painful abducens palsy may represent microvascular ischemia, especially in older patients
with vascular risk factors. Spontaneous resolution over 8 to 12 weeks is typical. In the absence of com-
plete, spontaneous resolution, neuroimaging is essential. Head trauma, even if mild, is another relatively
common cause of abducens palsy. Impaired ability to abduct the eye past midline or a bilateral presenta-
tion predicts poor spontaneous recovery. The abducens nerve (sixth cranial nerve) is the cranial nerve
most commonly affected bilaterally in isolation. This occurs most often from trauma and increased
intracranial pressure.
A third-nerve palsy is the most commonly encountered diabetic cranial mononeuropathy. Pupillary spar-
ing is the hallmark of diabetic third-nerve palsy. With decreasing frequency, the fourth, sixth, and seventh
nerves are also affected by diabetes. Patients with Bell’s palsy have a significantly higher frequency of dia-
betes than an age-matched population. Most make a full recovery in 3 to 5 months.
Neurology
B&D Chapters 70 and 76

Question 39. D. Appropriate specifiers for substance-induced anxiety disorder in DSM 5 include “with onset during
intoxication,” “with onset during withdrawal,” and “with onset after medication use.” Though some
may think this question is picky or unfair, the details of the DSM are indeed fair game for a test of gen-
eral psychiatric knowledge. The small details of the DSM give the test writer ample opportunity to form
tricky questions. Know your DSM well!
Anxiety Disorders
K&S Chapter 9

Question 40. D. Herpes simplex virus-1 (HSV-1) encephalitis (HSE) is the most common cause of sporadic, fatal enceph-
alitis in the United States. Early recognition is important, because the antiviral drug acyclovir is effective
in reducing morbidity and mortality. HSV-1 strains are the causal agents in over 90% of cases. Fever
and headache are the most common features. Other common features include disorientation, personality
change, seizures, memory disturbance, motor deficit, and aphasia. The presence of olfactory hallucina-
tions should also suggest the possibility of HSE.
Examination of the CSF is the single most important diagnostic test in suspected cases of HSE. Other useful
tests in making the diagnosis include MRI, CT, and EEG. HSV DNA can be detected in the CSF with poly-
merase chain reaction (PCR) techniques. Amplification of HSV DNA from CSF by PCR testing has a sensitiv-
ity and specificity of greater than 95% for the diagnosis of HSE and is the diagnostic procedure of choice for
HSE. MRI is the neuroimaging procedure of choice in patients with suspected HSE. Approximately 90% of
patients with PCR-proven HSE will have MRI abnormalities involving the temporal lobes, as the virus tends
to attack the undersurface of both the frontal and temporal lobes. The EEG may be abnormal, demonstrating
diffuse slowing, focal abnormalities in the temporal regions, or periodic lateralizing epileptiform discharges.
EEG abnormalities involving the temporal lobes are seen in approximately 75% of patients. Therapy with
acyclovir should be started immediately in cases of suspected focal encephalitis. The mortality rate in un-
treated cases of HSE is about 70%, which is reduced to 19% to 28% in patients treated with acyclovir.
Neurology
B&D Chapter 53B

Question 41. E. This question is really asking why somatic symptom disorder with predominant pain is a psychiatric
condition rather than a medical one. And the answer is that in somatic symptom disorder with pre-
dominant pain, the pain is thought to be mediated by psychological factors. It also leads to maladaptive
thoughts, feelings, and behavior. In chronic pain syndromes, which are in the domain of medicine and
not psychiatry, there is thought to be a physical cause for the pain even if that cause cannot be identified.

363

Psychiatry Test Preparation and Review


Manual E-Book
Psychiatry Test Preparation & Review Manual

In somatic symptom disorder with predominant pain, psychological factors are felt to play a major role
in the precipitation, maintenance, or exacerbation of the pain. Any of the other choices could be true for
either condition and are not the key difference used to differentiate the two.
Somatic Symptom Disorders
K&S Chapter 13

Question 42. B. Inclusion body myositis (IBM) is the most common myopathy in patients over the age of 50 years. IBM is
much more common in men than in women. The disease weakens the distal muscles of the arms and legs. The
deep finger flexors, including the flexor pollicis longus and wrist flexors, are involved early. They are usually
more involved than the wrist and finger extensors. In the legs, early involvement of the quadriceps and ante-
rior tibial muscles occurs. Profound atrophy is appreciable in the flexor forearms and quadriceps. The disease
generally has a chronic progressive course and is considered unresponsive to prednisone and other immuno-
suppressive (e.g., methotrexate) and immunomodulating (e.g., IVIG) therapies. The clinical history and exami-
nation are the basis for suspecting the diagnosis of IBM. Muscle biopsy is confirmatory. The creatine kinase
(CK) concentration may be normal or only mildly elevated. The EMG demonstrates fibrillation potentials and
positive sharp waves. The muscle biopsy demonstrates endomysial inflammation and invasion of nonnecrotic
muscle fibers similar to polymyositis. In addition, characteristic “rimmed” vacuoles may be profuse.
Dermatomyositis is an illness in which weakness is associated with a characteristic skin rash. It is the com-
mon form of myositis occurring in childhood through middle adult life. The rash usually occurs with onset
of muscle weakness, although it may develop during the course of the disease. It is characteristically a pur-
plish discoloration of the skin over the cheeks and eyelids. It often has a butterfly distribution and blanches
on pressure. The weakness is symmetrical and affects the proximal more than distal muscles of the arms
and legs. Muscle pain is noted, but not in all patients. The serum CK concentrations are usually elevated in
dermatomyositis. The characteristic histological feature on muscle biopsy is perifascicular atrophy (a crust
of small fibers surrounding a core of more normal-sized fibers deeper in the fascicle).
Polymyositis is an acute or subacute illness that occurs in adults. It is more frequent in women than in men, as
are other autoimmune diseases. As in dermatomyositis, weakness is symmetrical and affects the proximal more
than distal muscles of the arms and legs. Systemic symptoms are common at onset, such as malaise, fever, and
anorexia. The diagnostic studies in polymyositis are similar to those in dermatomyositis and include serum CK
concentrations, serum autoantibodies, EMG, and muscle biopsy. Serum CK concentration should always be
elevated in polymyositis, unlike dermatomyositis or IBM, in which the CK concentration may be normal.
Neurology
B&D Chapter 79

Question 43. D. This question tests the four most commonly asked neural pathways on standardized psychiatry exam-
inations. The tuberoinfundibular pathway goes from the hypothalamus to the anterior pituitary. It is
important in the regulation of prolactin secretion. The nigrostriatal pathway goes from the substantia
nigra to the basal ganglia. It is important in the development of extrapyramidal symptoms. The meso-
cortical pathway goes from the ventral tegmental area to the frontal cortex and is involved in the nega-
tive symptoms of schizophrenia. The mesolimbic pathway goes from the ventral tegmental area to the
nucleus accumbens and is involved in positive psychotic symptoms. The ventral amygdalofugal path-
ways are a distractor. They are not commonly asked about on examinations and are part of the limbic
system, running from the amygdala to the thalamus and hypothalamus.
Basic Neuroscience
K&S Chapter 1

Question 44. C. Mitochondrial encephalopathy, lactic acidosis, and stroke-like episodes (MELAS) is a maternally inher-
ited encephalomyopathy clinically characterized by stroke-like episodes, vascular headaches, vomiting,
seizures, and lactic acidosis. The stroke deficits are sometimes transient but can be permanent and cause
progressive encephalopathy with dementia. The unique radiological feature is that the stroke involves
the cerebral cortex, sparing the white matter, mostly in the parietal and occipital regions. The onset is
generally in childhood or early adult life. Most patients have ragged-red fibers (RRF) on muscle biopsy,
but only rarely is there clinical weakness or exercise intolerance.
Myoclonic epilepsy with ragged red fibers (MERRF) myopathy is another maternally inherited encephalomy-
opathy characterized by myoclonus, epilepsy, cerebellar ataxia, and myopathy with RRF. Onset is usually in

364

Psychiatry Test Preparation and Review


Manual E-Book
Test Number Six

childhood or early adulthood. The syndrome begins generally with myoclonic epilepsy in childhood, and other
seizure patterns are added soon thereafter. Worsening ataxia and mental retardation are seen in later childhood.
The clinical deficit in Leber’s hereditary optic neuropathy (LHON) is usually an isolated bilateral optic
neuropathy. LHON is expressed predominantly in males of the maternal lineage. The age of onset is typi-
cally between 15 and 35 years, and the vision loss is painless and central. Funduscopic abnormalities may
be seen in patients with LHON. Especially during the acute phase of vision loss, there may be hyperemia
of the optic nerve head, dilatation and tortuosity of peripapillary vessels, circumpapillary telangiectasia,
nerve fiber edema, and focal hemorrhage. Vision loss in LHON affects central fields and is usually per-
manent. In most LHON patients, vision loss is the only manifestation of the disease. Some families have
additional members with associated cardiac conduction abnormalities, especially preexcitation syndromes.
There may also be a movement disorder or other minor neurologic or skeletal abnormalities.
Kearns-Sayre Syndrome (KSS) is defined by the triad of progressive external ophthalmoplegia, onset before
age 20, and at least one of the following: short stature, pigmentary retinopathy, cerebellar ataxia, heart
block, and elevated CSF protein (>100 mg/dL). Many patients with KSS are physically or mentally sub-
normal. Some clinical features of MELAS and MERRF may overlap with KSS. The clinical course in KSS
is progressive, and most patients with associated intellectual disability die in the third or fourth decade.
Neurology
B&D Chapter 63

Question 45. E. This question covers the paraphilias. Sexual masochism is a preoccupation with fantasies or urges of
being beaten, bound, or humiliated. Fetishistic disorder involves being sexually fixated on and aroused
by inanimate objects like shoes or pantyhose. Voyeuristic disorder involves preoccupation with fantasies
of observing an unsuspecting person who is naked or involved in sexual activity. Transvestic fetishism
involves fantasies and sexual urges to dress in clothing of the opposite gender to obtain sexual arousal.
The patient is aroused by the fabrics, materials, or garments. Frotteuristic disorder involves rubbing
one’s genitals against a nonconsenting person to obtain sexual arousal.
Paraphilic Disorders
K&S Chapter 17

Question 46. A. Botulinum toxin blocks acetylcholine release at peripheral synapses, leading to the paralytic and auto-
nomic clinical manifestations of botulism. Botulinum toxin causes irreversible blockade at peripheral
cholinergic synapses. Recovery requires sprouting of new nerve terminals, accounting for the protracted
clinical course of botulism.
Neurology
B&D Chapter 53C

Question 47. C. Signs of tobacco withdrawal include depressed mood, insomnia, irritability, anxiety, poor concentration,
restlessness, decreased heart rate, and increased appetite. To meet DSM 5 criteria for tobacco withdrawal,
four of these symptoms must be present within 24 hours of cessation of nicotine.
Substance Abuse and Addictive Disorders
K&S Chapter 20

Question 48. E. If stroke patients meet the appropriate criteria, thrombolytic therapy may be administered.
Thrombolytic therapy is able to recanalize acute intracranial occlusions. A strong correlation has been
shown between arterial recanalization and neurologic improvement in acute cerebral ischemia.
Intravenous t-PA administration requires close adherence to protocol guidelines. The management of pa-
tients after t-PA administration requires close neurologic and blood pressure monitoring and the capability
to handle potential hemorrhagic complications associated with thrombolytic therapy. Inclusion criteria for
administration of t-PA consist of acute ischemic stroke with a clearly defined time of onset (<3 hours) and
CT scan without evidence of intracranial hemorrhage. Exclusion criteria for administration of t-PA consist
of major surgery within the previous 14 days, blood pressure greater than 185/110, bleeding parameters
outside of a narrow range, or a CT showing signs of intracranial hemorrhage.
Neurology
B&D Chapter 51A

365

Psychiatry Test Preparation and Review


Manual E-Book
Psychiatry Test Preparation & Review Manual

Question 49. D. In illness anxiety disorder the patient is convinced that he or she has a serious disease based on mis-
interpretation of bodily symptoms and despite assurance that he or she is not ill. In somatic symptom
disorder the patient presents with somatic complaints and maladaptive thoughts, feelings, and behaviors
manifested by disproportionate thoughts about the seriousness of the symptoms, high anxiety about the
symptoms, or excessive time and energy devoted to the symptoms. In malingering the patient is inten-
tionally feigning symptoms for external incentives. In factitious disorder the patient is intentionally
feigning symptoms for the benefits of the sick role. This patient does not have Lou Gehrig’s disease, or
he would not have had several negative workups.
Somatic Symptom Disorders
K&S Chapter 13

Question 50. C. The cervical spinal column includes 37 joints that are continually in motion throughout life. Cervical
osteoarthritis and spondylosis are ubiquitous with increasing age. Myelopathy caused by compression of
the cervical spinal cord by the changes of spondylosis and osteoarthritis usually develops insidiously, but
it may be precipitated by trauma or progress in a stepwise fashion. Typical findings are a combination of
leg spasticity, upper extremity weakness or clumsiness, and sensory changes in the arms, legs, or trunk.
Either spinothalamic tract-mediated or posterior column-mediated sensory modalities may be impaired.
Sphincter dysfunction, if it occurs, usually is preceded by motor or sensory findings. Neck pain is often
not a prominent symptom, and neck range of motion may or may not be impaired. Some patients expe-
rience leg or trunk paresthesia induced by neck flexion (Lhermitte’s sign).

The natural history of cervical spondylotic myelopathy is variable. Some patients have stable neurologic
deficit for many years without specific therapy, whereas other patients have gradual or stepwise dete-
rioration. Some patients improve with treatments, such as bed rest, soft collars, or immobilizing collars.
Many patients are treated by surgical decompression with variable surgical results.
Neurology
B&D Chapter 73

Question 51. E. Oh joy! Mahler’s stages of separation–individuation are back again! This question focuses on Mahler’s
last stage of object constancy, which lasts from 2 to 5 years. In this stage the child understands the per-
manence of other people, even when they are not present.
Human Development
K&S Chapter 31

Question 52. E. Nonarteritic anterior ischemic optic neuropathy (AION) is the most common cause of unilateral optic
nerve swelling in adults older than 50 and commonly is associated with vascular risk factors, such as
diabetes or hypertension. Prognostically, many patients will have a stable deficit, although some may
experience progression over a month, and the expected rate of spontaneous improvement is high. In
30% to 40% of patients, subsequent involvement of the other eye occurs. Recurrence in an affected
eye, however, is very rare. Posterior (retrobulbar) ischemic optic neuropathy is rare but is often a sign of
giant cell arteritis. The workup should therefore include an evaluation for arteritis, as well as for inflam-
matory and infiltrative conditions. Sometimes ischemic optic neuropathy without significant disk edema
can occur after severe blood loss and shock. Arteritic AION usually is related to temporal arteritis and is
always associated with disk swelling. The prevalence of temporal arteritis increases with age, and most
patients are older than 70 years of age. Acute vision loss is often the presenting symptom in arteritic
AION.
Neurology
B&D Chapter 15

Question 53. E. Phencyclidine, inhalants, sedative hypnotics, and alcohol can all cause nystagmus during intoxication.
Substance Abuse and Addictive Disorders
K&S Chapter 20

366

Psychiatry Test Preparation and Review


Manual E-Book
Test Number Six

Question 54. B. A thorough knowledge of the criteria for brain death is essential for the physician whose responsibilities
include evaluation of comatose patients. The criteria for the establishment of brain death include the
following:
• C oma: The patient should exhibit an unarousable unresponsiveness. There should be no meaningful
response to noxious, externally applied stimuli. Spinal reflexes, however, may be retained.
• No spontaneous respirations: The patient should be removed from ventilatory assistance and carbon
dioxide should be allowed to build up because of the respiratory drive that hypercapnia produces.
The diagnosis of absolute apnea requires the absence of spontaneous respiration at a carbon diox-
ide tension of at least 60 mm Hg. A safe means of obtaining this degree of carbon dioxide reten-
tion involves the technique of apneic oxygenation, in which 100% oxygen is delivered endotracheally
through a thin sterile catheter for 10 minutes. Arterial blood gas levels should be obtained to confirm
the arterial carbon dioxide pressure.
• Absence of brain-stem reflexes: Pupillary, oculocephalic, corneal, and gag reflexes all must be absent,
and there should not be any vestibulo-ocular responses to cold calorics.
• Electrocerebral silence: An isoelectric EEG should denote the absence of cerebrocortical function.
• Absence of cerebral blood flow: Cerebral contrast angiography or radionuclide angiography can sub-
stantiate the absence of cerebral blood flow, which is expected in brain death. These tests are consid-
ered confirmatory rather than mandatory.
• Absence of any potentially reversible causes of marked CNS depression: Such causes include hypo-
thermia, drug intoxication, and severe metabolic disturbance.
The American Academy of Neurology has determined that a positive apnea test is an absolute necessity for
the declaration of brain death.
Neurology
B&D Chapter 5

Question 55. C. To get this question right, you must know the difference between an inhibitor and an inducer of the CYP
system. An inhibitor decreases the enzyme activity, leading to an increase in the plasma concentration of
the drug. An inducer increases the enzyme activity, leading to a decrease in the plasma concentration.
The question also touches on some other important concepts. For example, there are some patients or
ethnic groups who, because of a genetic polymorphism in the CYP genes, will have either increased or
decreased enzyme activity. This can change them into slow or rapid metabolizers.
Paroxetine and fluoxetine are potent CYP 2D6 inhibitors. Great caution must be shown mixing them with
TCAs, Mellaril, codeine, β-blockers, Risperdal, clozapine, and aripiprazole. In addition, Ritonavir is a 2D6
inhibitor that can affect levels of several psychiatric drugs in HIV patients. Thioridazine is contraindicated
with CYP 2D6 inhibitors.
The most important CYP enzymes for the psychiatrist to understand are CYP 2D6, CYP 1A2, and CYP
3A4. You will no doubt see more questions about them all throughout this book and on your examination.
Psychopharmacology
K&S Chapter 29

Question 56. D. Amyotrophic lateral sclerosis (ALS) is a neurodegenerative disorder of undetermined etiology that pri-
marily affects the motor neuron cell populations in the motor cortex, brain stem, and spinal cord. It is
progressive, and most patients eventually succumb to respiratory failure. Between 5% and 10% of ALS
is familial rather than sporadic, the most common inheritance pattern being autosomal dominant. The
incidence is estimated at ∼2 per 100,000 in the United States, and the prevalence is about 6 per 100,000.
In 1993 mutations in the gene encoding an enzyme called copper/zinc superoxide dismutase (SOD1)
were identified in patients with familial ALS. SOD1 mutations are identified in up to 20% of all patients
with familial ALS. A significant body of basic and clinical research lends strong support to a theory
of ALS pathogenesis, which proposes selective motor neuron damage from a complex chain of injuri-
ous events. Onset of muscle weakness is more common in the upper than the lower extremities (classic,
spinal ALS), but in approximately 25% of patients, weakness begins in bulbar-innervated muscles (bul-
bar-onset ALS). Pseudobulbar palsy may present with inappropriate or forced crying or laughter, which

367

Psychiatry Test Preparation and Review


Manual E-Book
Psychiatry Test Preparation & Review Manual

is often a source of great emotional distress for patients. The diagnosis of clinically definite ALS can
sometimes be reached based on the history and clinical examination alone, but owing to the seriousness
of the diagnosis, ancillary investigations are necessary to exclude other possibilities. The EMG exami-
nation characteristically reveals changes in a widespread distribution that is not in keeping with any
single root or peripheral nerve distribution. Fasciculation potentials are usually identified; their absence
should prompt an investigation for another disorder. Riluzole (Rilutek) was approved by the FDA as the
first specific drug for the treatment of ALS. It may inhibit presynaptic release of glutamate. Edaravone
(Radicava) is an antioxidant FDA approved for ALS treatment. Explanations of the other answer choices
to this question can be found in other question explanations in this volume.
Neurology
B&D Chapter 74

Question 57. E. Females develop tardive dyskinesia more commonly than males. Patients over the age of 50 are more
likely to develop tardive dyskinesia than those younger. Patients with brain damage and young children
are more likely to develop tardive dyskinesia than older healthier counterparts.
Psychopharmacology
K&S Chapter 29

Question 58. B. Brain tumors account for approximately 4% of epilepsy cases. Seizures are a common presenting symp-
tom of brain tumors. The incidence of seizures is greater in primary low-grade brain tumors than in
brain metastases or high-grade primary tumors. A study of 132 adult patients with newly diagnosed
glioblastoma reported that seizures were the presenting symptom in 31% of patients. Seizures are some-
times focal, reflecting tumor location, but many become secondarily generalized at onset. Patients with
temporal lobe tumors frequently describe auras; epigastric sensations and psychological phenomena
such as déjà vu and depersonalization are common. Complex partial seizures and secondarily general-
ized tonic–clonic seizures are common.
Neurology
B&D Chapter 52C

Question 59. B. Valproic acid will increase lamotrigine levels. Oral contraceptives will decrease lamotrigine levels.
Phenobarbital and phenytoin will decrease lamotrigine levels.
Psychopharmacology
K&S Chapter 29

Question 60. D. PET depends on detection of emitted radioactivity (derived from injection of a radiolabeled tracer). It
uses the selective binding or uptake and retention of a variety of radiopharmaceuticals. These tracers
may provide general information about physiological function, such as regional cerebral blood flow or
cerebral glucose metabolism. Brain glucose metabolism is correlated with synaptic activity, and PET
therefore provides a good measure of neural activity, but without specifying the particular neuronal sub-
type involved. The other major application of PET is in the labeling of pharmaceuticals that are sub-
strates for specific enzymes, such as those involved in neurotransmitter synthesis or breakdown, or those
that bind to specific neurotransmitter receptors or transporters. It is this capacity of PET to study spe-
cific neurochemical processes that is its unique strength. SPECT is the nuclear medicine equivalent of
CT, similarly combining a series of two-dimensional images obtained by moving the gamma camera on
a gantry in a circular or elliptical orbit around the patient. The information from this series of images is
then combined into a three-dimensional volume. SPECT is based on the detection of single gamma rays.
Brain SPECT can be used to study general physiologic functions, such as perfusion, and in labeling neu-
rotransmitter receptors and transporters. Brain perfusion often is measured using SPECT.
In addition to its superior spatial resolution, MRI takes advantage of changes in the paramagnetic proper-
ties of deoxyhemoglobin compared with oxygenated hemoglobin to deduce the degree of blood oxygen-
ation. This effect has been used to examine regional activation associated with a variety of cognitive pro-
cesses, as well as motor and sensory processing. Functional MRI can provide valuable insights into brain
plasticity and reorganization after injury such as trauma or stroke. Functional MRI has vastly superior
temporal resolution compared with PET or SPECT and is widely available. The spatial resolution also is

368

Psychiatry Test Preparation and Review


Manual E-Book
Test Number Six

favorable. In contrast to PET, which permits a quantifiable absolute measure (e.g., blood flow, metabolic
rate, ligand-binding potential), fMRI allows estimation of only the relative change in regional blood flow.
Neurology
B&D Chapter 33C

Question 61. A. The random movements of chorea are accentuated and often most noticeable during walking. The
superimposition of chorea on the trunk and leg movements of the walking cycle gives the gait a danc-
ing quality, and there is an exaggerated motion of the legs and arm swing. Chorea can also interrupt the
walking pattern, leading to a hesitant gait. Additional voluntary compensatory movements appear in
response to the chorea. Chorea in Sydenham’s chorea or chorea gravidarum may be sufficiently violent
to throw patients off their feet; severe chorea of the trunk may render walking impossible. The chorea
of Huntington’s disease usually causes a lurching or stumbling and stuttering gait with frequent steps
forward, backward, or to the side. Walking is slow, the stance is wide-based, the trunk sways excessively,
and steps are variable in length and timing. Spontaneous knee flexion and leg-raising movements are
common. Haloperidol reduces chorea but does not improve gait in Huntington’s disease. Balance and
equilibrium usually are maintained until the terminal stages of Huntington’s disease, when an akinetic-
rigid syndrome may supervene. Stereotactic thalamotomy is a neurosurgical technique designed to allevi-
ate symptoms of Parkinson’s disease. Pramipexole and benztropine are antiparkinsonian medications.
Lioresal (baclofen) is a potent muscle relaxant used in the treatment of muscle spasticity, for example, in
stroke and MS patients.
Neurology
B&D Chapter 22

Question 62. C. The most common akinetic-rigid gait disturbance is that encountered in Parkinson’s disease. The pos-
ture is stooped with flexion of the shoulders, neck, and trunk. During walking, arm swing is reduced or
absent, and the arms are held immobile at the sides or slightly forward of the trunk. The gait is slow and
shuffling, with small, shallow steps on a narrow base. The posture of generalized flexion of the patient
with Parkinson’s disease exaggerates the normal tendency to lean forward when walking. To maintain
balance when walking and avoid falling forward, the patient may advance with a series of rapid, small
steps (festination). Retropulsion and propulsion are similar manifestations of a flurry of small, shuffling
steps made in an effort to preserve equilibrium. Instead of a single large step, a series of small steps are
taken to maintain balance. Falls occur in Parkinson’s disease when festinating steps are too small to
restore balance.
Astasia–abasia refers to the inability to either stand or walk in a normal manner. Patients exhibit an un-
usual and dramatic gait disturbance, lurching wildly in various directions and falling only when a nearby
person or soft object will catch them. Astasia refers to the inability to maintain station (stand upright) un-
assisted. Abasia refers to lack of motor coordination in walking. When seen in conversion disorder, the gait
is bizarre and is not suggestive of a specific organic lesion; often the patient sways wildly and nearly falls,
recovering at the last moment. However, an acquired total inability to stand and walk can be seen in true
neurologic diseases, including stroke, Parkinson’s disease, damage to the cerebellum, Guillain–Barré syn-
drome, normal-pressure hydrocephalus, and others. In normal-pressure hydrocephalus, for example, the
patient’s gait becomes shortened, with frequent shuffling and falls; eventually standing, sitting, and even
rolling over in bed become impossible. This advanced state is referred to as “hydrocephalic astasia–abasia.”
Neurology
B&D Chapter 22

Question 63. E. Bacterial meningitis may be defined as an inflammatory response to bacterial infection of the subarach-
noid space. Because the subarachnoid space is continuous over the brain, spinal cord, nerves, and nerve
roots, infection in this space generally extends throughout the cerebrospinal axis. Worldwide, three main
pathogens, Haemophilus influenzae, Streptococcus pneumoniae, and Neisseria meningitidis, account for
up to 80% of cases after the neonatal period. Group B streptococci, Escherichia coli, and Listeria mono-
cytogenes are major pathogens in neonatal meningitis. The most common bacteria that cause meningitis,
S. pneumoniae and N. meningitidis, initially colonize the nasopharynx. At any time, S. pneumoniae can
be isolated from 5% to 10% of healthy adults and from 20% to 40% of healthy children. Bacterial

369

Psychiatry Test Preparation and Review


Manual E-Book
Psychiatry Test Preparation & Review Manual

meningitis occurs when pathogens colonizing the nasopharynx cause bacteremia and breach the blood–
brain barrier to infiltrate the brain.
The classical clinical presentation of adults with bacterial meningitis comprises headache, fever, and neck
stiffness (often with signs of cerebral dysfunction). Nausea, vomiting, myalgia, and photophobia are also
common. The neck stiffness may be subtle or marked, accompanied by Kernig’s and/or Brudzinski’s signs.
Cerebral dysfunction is manifested by confusion, delirium, and a declining level of consciousness that
ranges from lethargy to coma. Seizures occur in approximately 40% of cases. Cranial nerve palsies involv-
ing cranial nerves III, VI, and VII may also be seen in up to 20% of cases.
Neurology
B&D Chapter 53C

Question 64. A. The main clinical features of Guillain–Barré Syndrome (GBS) are motor weakness, areflexia, paresthesias
with minor sensory loss, and increased protein in the CSF without elevated WBCs. Nerve conduction
studies reveal widespread demyelination. Approximately two-thirds of patients report a preceding event,
most frequently an upper respiratory or gastrointestinal infection. The agent responsible for the prodro-
mal illness often remains unidentified. Specific infectious agents linked to GBS include Campylobacter
jejuni, HIV, Lyme, mononucleosis, hepatitis, CMV, and West Nile virus. The most common identifiable
bacterial organism linked to GBS is Campylobacter jejuni. Patients with classic GBS may initially pres-
ent with weakness with or without burning/prickling sensations. The fairly symmetrical weakness of the
lower limbs ascends proximally over hours to several days and may subsequently involve arm, facial,
and oropharyngeal muscles and, in severe cases, respiratory muscles. Autonomic instability can develop
and become severe. Hyporeflexia or areflexia are present. Cranial nerve involvement can occur. Facial
paresis, usually bilateral, is found in at least half of patients. In the first week of neurologic symptoms
the CSF protein may be normal but then becomes elevated on subsequent examinations. Abnormalities
of nerve conduction studies are found in approximately 90% of established cases and reflect an evolv-
ing picture of multifocal demyelination associated with secondary axonal degeneration. Among specific
therapeutic interventions aimed at mitigating the harmful effects of autoantibodies, plasma exchange
and high-dose IVIG infusions have been shown to be equally effective.
Neurology
B&D Chapter 76

Question 65. E. The clinical presentation of intracerebral hemorrhage (ICH) has two main elements: symptoms that
reflect the effects of intracranial hypertension and those that are specific for the location of the hema-
toma. The general clinical manifestations of ICH related to increased intracranial pressure (headache,
vomiting, and depressed level of consciousness) vary. CT is sensitive to the high-density fresh blood in
the brain parenchyma, along with associated features of local mass effect and ventricular extension. MRI
adds further precision, especially in determining the time elapsed between onset and time of MRI exami-
nation. Cerebellar hemorrhage represents approximately 5% to 10% of the cases. Its clinical presenta-
tion is characteristic, with abrupt onset of vertigo, headache, vomiting, and inability to stand and walk,
with absence of hemiparesis or hemiplegia. The physical findings that allow its clinical diagnosis are the
triad of appendicular ataxia (decreased coordination of extremities), horizontal gaze palsy, and periph-
eral facial palsy, all ipsilateral to the hemorrhage. There is a notorious tendency for abrupt deterioration
to coma and death after a period of clinical stability. The neurologist must be able to recognize when
neurosurgical intervention, which is frequently life-saving and function-restoring for these patients, is
indicated. To this effect, it is convenient to classify the clinical stages seen in this condition: (1) an initial
“cerebellar” stage, in which the only clinical deficits detected are referable to the cerebellum; (2) an inter-
mediate stage, in which, in addition to cerebellar signs, there are signs and symptoms referable to hydro-
cephalus; and (3) the final stage, brain-stem compression. Neurosurgical intervention is indicated when
the first signs and symptoms of hydrocephalus develop. The prognosis for good functional recovery is
excellent in patients with cerebellar hemorrhage who have had a timely suboccipital decompression
before they have lapsed into coma. Even comatose patients, however, can make an excellent functional
recovery, provided that the decompression is carried out expeditiously and soon after the onset of coma.
Neurology
B&D Chapters 46 and 51B

370

Psychiatry Test Preparation and Review


Manual E-Book
Test Number Six

Question 66. C. Stiff-person syndrome (SPS) is a rare syndrome of progressive rigidity of axial and proximal appendicu-
lar muscles with muscle hypertrophy and extreme lumbar lordosis. Intense spasms are superimposed
on a background of continuous muscle contraction. Gait is slow and stiff-legged. Spasms and stiff-
ness improve with sleep and are eliminated by general anesthesia and neuromuscular blocking agents.
Clinical criteria for diagnosis include insidious development of limb and axial stiffness, episodic spasms
superimposed on chronic stiffness, and no other underlying illness that would explain the symptoms.
EMG examination shows continuous firing of normal motor units. SPS is often a paraneoplastic syn-
drome and has been associated with anti-glutamic acid decarboxylase antibodies. It is thought that
SPS results from dysfunction of descending motor pathways, possibly secondary to immune-mediated
inhibition of GABA synthesis. Paraneoplastic SPS has been reported with breast and lung cancers and
Hodgkin’s disease. Untreated, SPS progresses to extreme disability. Diazepam is the most effective symp-
tomatic treatment. Clonazepam, baclofen, valproic acid, clonidine, vigabatrin, and tiagabine have also
been reported to be effective. Intrathecal baclofen and local intramuscular injections of botulinum toxin
have been helpful in some cases. Plasmapheresis, IVIG, and immunosuppression have been reported to
have variable effects on the condition.
Neurology
B&D Chapters 52G and 71

Question 67. D. Simple partial seizures of temporal lobe origin often are difficult to recognize as epileptic events.
Complex partial temporal lobe seizures may begin with impairment of consciousness, or by a simple
partial seizure, or with an aura. Auras vary in duration from seconds to several minutes before impair-
ment of consciousness. Most complex partial seizures last longer than 30 seconds, usually up to 1 to 2
minutes; few last less than 10 seconds, which is a distinguishing characteristic from absence seizures.
Postictal recovery usually is slow, with significant confusion that may last for several minutes or lon-
ger. Clinical symptomatology at the onset of the seizure may include auditory or olfactory hallucina-
tions, emotional or psychic symptoms, sensations of movement or rotation, or autonomic symptoms.
The cause of olfactory seizures (uncinate fits), usually an unpleasant odor, is discharges in the medial
temporal lobe. The cause of gustatory seizures is discharges deep in the sylvian fissure or the oper-
culum (insular cortex). Epigastric sensations of nausea, “butterflies,” emptiness, or tightness usually
are caused by temporal lobe activity. Emotional changes and psychic phenomena, often attributed to
simple partial seizures of temporal lobe origin, more commonly are associated with complex partial
seizures.
Neurology
B&D Chapter 67

Question 68. A. Carotid artery dissection commonly manifests with neck, face, and head pain ipsilateral to the dis-
section; frequently is associated with an ipsilateral Horner’s syndrome; and often follows head or
neck trauma. Most dissections involve the internal carotid artery or extracranial vertebral arteries.
Intracranial dissections may follow trivial trauma, closed head trauma, basilar skull fracture, or pen-
etrating injuries. Diagnosis is based on arteriographic findings, magnetic resonance angiography (MRA),
or CT angiography. Features on arteriography include the presence of a pearl and string sign; double-
lumen sign; short, smooth, tapered occlusion; or pseudoaneurysm formation. Therapeutic interventions
have included immediate anticoagulation with heparin followed by a 3- to 6-month course of warfarin,
as well as platelet antiaggregants instead of, or following, warfarin.
Neurology
B&D Chapters 18 and 51A

Question 69. B. Since the early 2000s, case reports have surfaced of patients developing pathologic gambling issues when
placed on dopamine agonists. Studies have demonstrated increased rates of pathologic gambling among
Parkinson’s patients treated with dopamine agonists, with rates as high as 8%. This pattern supports the
role of dopamine in addictive behaviors. As such, this is the most likely explanation for what is happen-
ing to Grandpa Joe. Though manic patients spend excessively, there is not enough information given in
the question to support a diagnosis of mania. Distortion is grossly reshaping the experience of external

371

Psychiatry Test Preparation and Review


Manual E-Book
Psychiatry Test Preparation & Review Manual

reality to suit internal needs and can include sustained feelings of delusional grandiosity, superiority, or
entitlement. This does not explain what is happening to Grandpa Joe. Kubler-Ross described the stages
of death and dying. Stage 1 is shock and denial. Stage 2 is anger. Stage 3 is bargaining. Stage 4 is depres-
sion. Stage 5 is acceptance. In the bargaining stage, the patient attempts to negotiate with physicians,
friends, or even God. This is not what Grandpa Joe is doing. Eric Erickson’s stages are covered through-
out this volume. In generativity vs stagnation, which tends to occur between age 40 and 60, generativity
refers to concerns for future generations and social institutions. The person invests the ego and libidinal
energy in groups, organizations, and society. This is not what Grandpa Joe is doing either.
Substance-Related and Addictive Disorders
K&S Ch. 20; Neurology Aug 2003

Question 70. A. Lorazepam (initially marketed under the brand names Ativan and Temesta) is a high-potency, short-
to intermediate-acting 3-hydroxy benzodiazepine drug that has all six intrinsic benzodiazepine effects:
anxiolytic, amnesic, sedative/hypnotic, anticonvulsant, antiemetic, and muscle relaxant. Lorazepam is
used for the short-term treatment of anxiety, insomnia, acute seizures, including status epilepticus, and
sedation of hospitalized patients, as well as sedation of aggressive patients. Lorazepam is considered to
be a short-acting drug that, similar to other benzodiazepines, exerts its therapeutic as well as adverse
effects via its interaction at benzodiazepine binding sites, which are located on GABA receptors in the
CNS. After its introduction in 1977, lorazepam’s principal use was in treating anxiety. Among benzo-
diazepines, lorazepam has a relatively high addictive potential. Lorazepam also has abuse potential; the
main types of misuse are for recreational purposes or continued use against medical advice. Lorazepam
may be safer than most benzodiazepines in patients with impaired liver function. Like oxazepam, it does
not require hepatic oxidation but only hepatic glucuronidation into lorazepam-glucuronide. Therefore,
impaired liver function is unlikely to result in lorazepam accumulation to an extent causing adverse
reactions. Similarly, renal disease has minimal effects on lorazepam levels. The three benzodiazepines
that are metabolized via glucuronidation are lorazepam, oxazepam, and temazepam, and these come up
often in this context on standardized examinations in psychiatry (mnemonic LOT).
Psychopharmacology
K&S Chapter 29

Question 71. C. Most of the body’s serotonin is in the gastrointestinal (GI) tract. This is why serotonergic drugs typically
cause varying degrees of stomach pain, nausea, vomiting, flatulence, and diarrhea. These side effects are
mediated mostly through effects on the serotonin 5-HT 3 receptor. Sertraline and fluvoxamine produce
the most intense GI symptoms. In most cases these side effects are transient, but in a small percentage of
individuals, these effects never abate, and a switch to another class of antidepressant must be made. The
most effective way to offset these side effects is to use initial low doses of these agents or to use delayed-
release preparations. Up to one-third of patients taking SSRIs will gain weight. This effect is mediated
through a metabolic mechanism, increase in appetite, or both. It happens slowly over time and is resis-
tant to diet and exercise regimens. Bonus fact: the SSRIs may rarely cause EPS symptoms, like akathisia,
dystonia, tremor, gait disorders, bradykinesia, or cogwheeling. There have been cases of patients with
well-controlled Parkinson’s who developed worsening of motor symptoms after being placed on an
SSRI. Additional bonus fact: the most common cardiac side effect of the SSRIs is bradycardia.
Psychopharmacology
K&S Chapter 29

Question 72. E. The standard and classic treatment for obsessive–compulsive disorder (OCD) is via pharmacotherapy,
behavior therapy, or a combination of the two. SSRIs and clomipramine (Anafranil) are the pharmaco-
logic agents of choice in OCD. Behavioral therapy techniques have been shown to be effective for OCD.
These include exposure and response prevention, desensitization, thought-stopping, flooding, implosion
therapy, and aversive conditioning. In behavior therapy, patients have to be truly committed to their
own improvement. Family and group therapy are often helpful in supporting the family and improv-
ing relationships in patients with OCD. For extreme cases that are unresponsive to all these therapies,
electroconvulsive shock therapy (ECT) and psychosurgery are considerations. ECT is not as effective as
psychosurgery, but it should still be tried before psychosurgery. Cingulotomy is a common psychosurgi-
cal procedure for extreme OCD and is effective in 25% to 30% of otherwise treatment-unresponsive
patients. Another surgical procedure that may also help intractable OCD is subcaudate tractotomy, also

372

Psychiatry Test Preparation and Review


Manual E-Book
Test Number Six

known as capsulotomy. The most common complication of psychosurgery is the development of sei-
zures. Some patients who do not respond to psychosurgery following failure of prior pharmacotherapy
and/or behavior therapy may respond to these interventions after the psychosurgery is done.
Obsessive Compulsive and Related Disorders
K&S Chapter 10

Question 73. B. Lithium has myriad side effects that crop up in multiple choice questions on psychiatry examinations.
Among its neurologic side effects, lithium can cause dysphoria, lack of spontaneity, slowed reaction
time, and memory difficulties. Patients often complain of “altered creativity,” and this is a well-described
adverse effect of lithium. These are usually benign side effects that can occur even when the serum lith-
ium level is below the toxic range. Lithium can also cause tremor, peripheral neuropathy, benign intra-
cranial hypertension, lowered seizure threshold, and a myasthenia gravis-like syndrome. Other common
nonneurologic side effects of lithium include weight gain, fluid retention, appetite loss, nausea, vomiting,
diarrhea, nephrogenic diabetes insipidus (polyuria/polydipsia), hypothyroidism, hyperparathyroidism,
benign T-wave changes, acne, rash, psoriasis, and hair loss.
Psychopharmacology
K&S Chapter 29

Question 74. D. Affective disorders (major depression disorder [MDD] and bipolar disorder) account for 50% of com-
pleted suicides. The other potentially tricky choice listed was personality disorders, which account for
only 5% of completed suicides, despite patients’ many attempts and gestures. Of the personality dis-
orders, 4% to 10% of borderline patients commit suicide, and 5% of antisocial personality disorder
patients commit suicide. Mixing the personality disorder with major depression or substance abuse
greatly increases the risk. The risk of completed suicide in affective disorders is 30 times greater than
that for the general population. Drug and alcohol abuse account for 25% of completed suicides, and 5%
to 10% of patients with schizophrenia complete suicide.
Management in Psychiatry
K&S Chapter 23

Question 75. E. The therapist’s tasks in a group therapy setting include determining the size and setting of the group,
choosing the frequency and length of sessions, deciding on an open vs a closed group, formulating
appropriate goals, selecting patients, and building a therapeutic environment for the group conducive to
identifying and dealing with problems within the group. We do not want to suppress catharsis; we want
patients to speak about their feelings and emotions and feel relief after doing so.
Psychotherapy
K&S Chapter 28

Question 76. C. Carbamazepine, in its extended-release form (Equetro), was finally FDA-approved in the United States
for treatment of bipolar disorder in 2004. It was first used to treat partial and generalized-onset epi-
lepsy and TN. Carbamazepine is structurally similar to the TCAs. Carbamazepine is believed to cause
blockade of type 2 sodium channels and have activity at A1 adenosine receptors. Carbamazepine
causes autoinduction of its own metabolism via hepatic CYP 3A4 enzymes. Carbamazepine has numer-
ous side effects. Those that are dose-dependent include blurred vision, vertigo, GI disturbances, and
task-performance impairment. Idiosyncratic adverse effects (which are not dose-dependent) include
agranulocytosis, Stevens–Johnson syndrome, aplastic anemia, hepatic failure, rash, and pancreatitis.
The drug’s hematologic effects are not dose-dependent. Severe blood dyscrasias (aplastic anemia and
agranulocytosis) occur in about 1 in 125,000 persons treated with carbamazepine. Patients should be
warned that symptoms such as fever, rash, petechiae, sore throat, bruising, and easy bleeding may her-
ald serious blood dyscrasia, and they should be urged to seek medical attention immediately. Routine
blood monitoring of the complete blood count is recommended at 3, 6, 9, and 12 months. If no evi-
dence of bone marrow suppression is noted at 12 months, most clinicians would reduce the interval of
monitoring.
Psychopharmacology
K&S Chapter 29

373

Psychiatry Test Preparation and Review


Manual E-Book
Psychiatry Test Preparation & Review Manual

Question 77. E. Privilege is the right to maintain confidentiality in the face of a subpoena. Conversations between doctor
and patient are considered privileged communications, which the law protects from forced disclosure on
the witness stand. The right of privilege belongs to the patient, not to the physician, so the patient has
the decision to waive that right. In 1996 the U.S. Supreme Court recognized the importance of thera-
pist–patient privilege, emphasizing that protecting privilege between a therapist and a patient serves the
public good.
Forensic Psychiatry
K&S Chapter 36

Question 78. A. Clozapine should not be taken in conjunction with any other drug that is associated with the devel-
opment of agranulocytosis or bone marrow suppression. Such agents include carbamazepine, phe-
nytoin, propylthiouracil, sulfonamides, and captopril. Lithium combined with clozapine can increase
risk of seizures, confusion, and movement disorders. Lithium should not be taken with clozap-
ine by patients who have previously experienced an episode of neuroleptic malignant syndrome.
Clomipramine can increase the seizure risk by lowering the seizure threshold and by increasing clo-
zapine plasma concentrations. Fluoxetine, paroxetine, risperidone, and fluvoxamine can all increase
the serum levels of clozapine. Paroxetine may trigger clozapine-associated neutropenia. Also keep in
mind that cigarette smoking decreases clozapine levels via induction of CYP 450 1A2. If a patient on
clozapine starts smoking (or begins smoking significantly more) then becomes psychotic, check a clo-
zapine level!
Psychopharmacology
K&S Chapter 29

Question 79. D. Memantine is a low- to moderate-affinity NMDA receptor antagonist. It is FDA-approved in the United
States for treatment of moderate to severe Alzheimer’s disease. It is purported that overexcitation of
NMDA receptors by the neurotransmitter glutamate may play a role in Alzheimer’s disease, because glu-
tamate plays an integral part in the neural pathways associated with learning and memory. Excess gluta-
mate overstimulates NMDA receptors to allow too much calcium into nerve cells, leading to cell death
that is observed in Alzheimer’s disease. Memantine is believed to have neuroprotective effects by par-
tially blocking NMDA receptors associated with abnormal transmission of glutamate, while allowing for
physiologic transmission associated with normal cell functioning. An often-tested fact is that the most
common side effect of donepezil is nausea and abdominal cramping. Galantamine, donepezil, tacrine,
and rivastigmine are all inhibitors of acetylcholinesterase.
Psychopharmacology
K&S Chapter 29

Question 80. E. A mnemonic for organophosphorus poisoning is DUMBELS:


D = Diarrhea
U = Urination
M = Miosis
B = Bradycardia
E = Emesis
L = Lacrimation
S = Salivation/sweating/secretion
Organophosphates are used mainly as pesticides and herbicides but are also used as petroleum additives,
lubricants, antioxidants, flame retardants, and plastic modifiers. Most cases of organophosphate toxic-
ity result from exposure in an agricultural setting, not only among those mixing or spraying the pesti-
cide or herbicide but also among workers returning prematurely to sprayed fields. Absorption may occur
through the skin, by inhalation, or through the GI tract. Organophosphates inhibit acetylcholinesterase by

374

Psychiatry Test Preparation and Review


Manual E-Book
Test Number Six

phosphorylation, with resultant acute cholinergic symptoms, with both central and neuromuscular mani-
festations. Symptoms include nausea, salivation, lacrimation, headache, weakness, and bronchospasm, in
mild instances, and bradycardia, tremor, chest pain, diarrhea, pulmonary edema, cyanosis, convulsions,
and even coma in more severe cases. Death may result from respiratory or heart failure. Treatment involves
intravenous administration of pralidoxime together with atropine given subcutaneously every 30 minutes
until sweating and salivation are controlled. Pralidoxime accelerates reactivation of the inhibited acetyl-
cholinesterase, and atropine is effective in counteracting muscarinic effects, although it has no effect on the
nicotinic effects, such as neuromuscular cholinergic blockade with weakness or respiratory depression. It
is important to ensure adequate ventilatory support before atropine is given.
Poisoning
B&D Chapter 86

Question 81. B. Clostridium tetani secretes tetanospasmin, also known as tetanus toxin. Tetanospasmin blocks release of
inhibitory neurotransmitters by spinal interneurons, causing the dramatic muscle contractions that char-
acterize tetanus. Tetanospasmin inhibits release of GABA and glycine, which are inhibitory neurotrans-
mitters in the brain stem and spinal cord. Cardinal features include muscle rigidity and spasms, which
may be accompanied by autonomic hyperactivity. Local tetanus, in which symptoms remain limited to a
limb, is a rare form. Far more common is generalized tetanus, also called lockjaw, as trismus heralds the
disorder in over 75% of cases. Sustained contraction of facial muscles causes a sneering grimace known
as risus sardonicus. Other early symptoms include dysphagia and axial muscle involvement, such as neck
stiffness, abdominal rigidity, and back pain. Laryngospasm compromises ventilation and makes intuba-
tion extremely difficult. Sustained contraction of back muscles causes an arching posture of the back. As
tetanus progresses, reflex muscle spasms develop, triggered by sensory stimuli, movement, or emotion.
Therapeutic goals include protecting the airway; neutralizing circulating tetanospasmin, and preventing
its further production; managing spasms and dysautonomia; and general supportive care. Initial treat-
ment of most patients with generalized tetanus includes endotracheal intubation, because laryngospasm
may appear abruptly even in mild cases. Human tetanus immune globulin neutralizes circulating toxin.
Infected wounds should be debrided after human tetanus immune globulin administration, because the
procedure may release further toxin. Metronidazole or penicillin should be given to eradicate C. tetani.
Poisoning
B&D Chapter 53C

Question 82. B. Sleep is composed of four distinct phases or stages: non-REM (NREM) sleep, which is divided into
stages N1 through N3, and REM sleep. Each phase has its own unique characteristics when measured
in three ways: EEG, electro-oculogram, and EMG. The table shown here breaks down these phases into
their distinct characteristics.
Stage EEG EOG EMG
N1 Slowing of alpha activity Slow, rolling Continual activity
N2 Sleep spindles and K complexes Slow, rolling Further reduction
N3 Slow wave delta Absent Further reduction
REM Low amplitude Rapid Silent
Sleep–Wake Disorders
K&S Chapter 16

Question 83. D. In the neonatal period, REM represents more than 50% of total sleep time, and the EEG goes directly
into REM phase without passing first through N1 through N3. Newborns sleep about 16 hours a day.
By about 4 months of age, the sleep pattern shifts. Infants now pass through NREM sleep before their
first REM episode and REM sleep drops to less than 40% of total sleep time. By young adulthood,
NREM makes up about 75% of total sleep time. Thus, in adulthood, REM makes up about 25% of
total sleep time. This distribution remains relatively constant into older age, although there is usually a
reduction in both slow-wave sleep and REM sleep in the elderly.
Sleep–Wake Disorders
K&S Chapter 16

375

Psychiatry Test Preparation and Review


Manual E-Book
Psychiatry Test Preparation & Review Manual

Question 84. C. Phentolamine is a medication usually reserved for hospital use in intensive care unit or cardiac care unit
settings. Its primary action is vasodilation due to α1 blockade. It also can lead to reflex tachycardia.
The primary application for phentolamine is the control of hypertensive emergencies, most notably
due to pheochromocytoma. It also has usefulness in the treatment of cocaine-induced hypertension, in
which one would generally avoid β-blockers and in which calcium channel blockers are not effective.
β-Blockers (e.g., metoprolol) or combined α- and β-adrenergic blocking agents (e.g., labetalol) should be
avoided in patients with a history of cocaine abuse. They can cause an unopposed α-adrenergic-mediated
coronary vasoconstriction, causing the worsening of myocardial ischemia and hypertension. It is also
used in the treatment of pheochromocytoma before the administration of β-blockers to avoid unopposed
α-stimulation. MAOI-induced hypertensive crisis should be treated with α-adrenergic antagonists, such
as phentolamine or chlorpromazine. These agents lower blood pressure within 5 minutes. Intravenous
furosemide (Lasix) can be used to reduce the fluid load, and β-adrenergic receptor antagonists can be
used for controlling tachycardia. A sublingual dose of nifedipine (Procardia) can be given and repeated
in 20 minutes. MAOIs should not be taken by patients with pheochromocytoma or thyrotoxicosis.
Bromocriptine and dantrolene are dopamine agonists and are used in the treatment of neuroleptic malig-
nant syndrome. They have no place in the treatment of MAOI-induced hypertensive crisis, and in fact
bromocriptine should be used with extreme caution in patients on MAOIs, as it can interact adversely
with MAOIs, worsening hypertensive crisis. Sedative–hypnotic agents, like diazepam, should also be
used with caution in patients taking MAOIs.
Diagnostic and Treatment Procedures in Psychiatry
K&S Chapter 29

Question 85. D. Concurrent administration of an SSRI with an MAOI antidepressant or lithium can raise plasma sero-
tonin to toxic levels, producing a serotonin syndrome. Serotonin syndrome is serious and can in some
cases be fatal. Symptoms tend to occur in the following order as the condition worsens: (1) diarrhea; (2)
restlessness; (3) extreme agitation, hyperreflexia, and autonomic instability, with possible rapid fluctua-
tions in vital signs; (4) myoclonus, seizures, hyperthermia, uncontrollable shivering, and rigidity; and (5)
delirium, coma, status epilepticus, cardiovascular collapse, and death. Treatment consists of promptly
removing the offending agents and immediately instituting comprehensive supportive care with nitro-
glycerine, cyproheptadine, methysergide, cooling blankets, chlorpromazine, dantrolene, benzodiazepines,
anticonvulsants, mechanical ventilation, and paralytic agents.
Management in Psychiatry
K&S Chapter 29

Question 86. A. The overwhelming factor that correlates with childhood physical abuse is poverty and psychosocial
stress, especially financial stress. Abuse and neglect of children also correlate with less parental edu-
cation, underemployment, poor housing, single parenting, and welfare reliance. Risk of child abuse
increases in families with many children and is also increased by other risk factors, such as physical
handicap, mental retardation, and prematurity. Parental mental illness, parental substance abuse (in
particular alcoholism), social isolation, and domestic violence also correlate with childhood abuse and
neglect. Families in which there are many of these problems coexisting tend to be more prone to child
abuse and neglect.
Public Policy
K&S Chapter 31

Question 87. B. IPT is a time-limited treatment (12 to 16 weeks) with three phases: a beginning (one to three sessions),
middle, and end (three sessions). The initial phase requires the therapist to identify the target diagnosis
(MDD) and the interpersonal context in which it presents. The therapist also elicits an “interpersonal
inventory,” a review of the patient’s patterns in relationships and capacity for intimacy, and particularly
an evaluation of current relationships from which a focus for treatment emerges. The therapist links the
target diagnosis to the interpersonal focus. This formulation defines the remainder of the therapy.
In the middle phase of treatment, the therapist uses specific strategies to deal with whichever of the poten-
tial problem areas is the focus. This might involve appropriate mourning in “complicated bereavement,”

376

Psychiatry Test Preparation and Review


Manual E-Book
Test Number Six

resolving an interpersonal struggle in a “role dispute,” helping a patient mourn the loss of an old role and
assume a new one in a “role transition,” or decreasing social isolation for “interpersonal deficits.” What-
ever the focus, the therapy is likely to address the patient’s ability to assert his or her needs and wishes in
interpersonal encounters, to validate the patient’s anger as a normal interpersonal signal and to encour-
age its efficient expression, and to encourage taking appropriate social risks. In the last few sessions, the
therapist reminds the patient that termination is nearing, helps the patient feel more capable and inde-
pendent by reviewing his or her often considerable accomplishments during the treatment, and notes that
ending therapy is itself a role transition, with inevitable good and painful aspects. Because IPT has also
demonstrated efficacy as a maintenance treatment for recurrent MDD, and because patients who have had
multiple episodes are very likely to have more, therapist and patient may decide to end acute treatment
as scheduled and then to recontract for ongoing treatment, perhaps of less intensive dosage, for example,
monthly rather than weekly sessions.
The IPT therapist’s stance is relaxed and supportive. The goal is to be the patient’s ally. The acute time limit
pressures the patient to take action. No formal homework is assigned, but the goal of solving the focal
interpersonal problem area provides an overall task. Treatment centers on the patient’s outside environ-
ment, not on the therapy itself. The scheduling of sessions once weekly accentuates that the emphasis is on
the patient’s real life, not the office. In sessions therapist and patient review the past week’s events. When
the patient succeeds in an interpersonal situation, the therapist acts as a cheerleader, reinforcing healthy
interpersonal skills. When the outcome is adverse, the therapist offers sympathy, helps the patient analyze
what went wrong in the situation, brainstorms new interpersonal options, and role-plays them with the
patient in rehearsal for real life. The patient then tests them out. Given this emphasis on interpersonal in-
teraction, it is not surprising that depressed patients learn new interpersonal skills from IPT that they have
not seen with pharmacotherapy. Note that psychotherapy questions come up frequently on psychiatry
examinations and candidates often find themselves underprepared for these important questions. Do make
it a point to understand IPT along with other psychotherapy modalities to maximize your examination
performance.
Psychotherapy
K&S Chapter 28

Question 88. E. The “dialectical” in dialectical behavioral therapy (DBT) refers to the way in which it emphasizes the
limitations of linear ideas about causation. It substitutes “both/and” for “either/or” and sees truth as
an evolving product of the opposition of different views. This arises from the mixed and shifting nature
of emotion in patients with borderline personality disorder (BPD). Interaction with such a patient is
unlikely to have the characteristics of a logical argument or even an orderly conversation. It is more
likely to be akin to a dance to rapidly changing music. The person with BPD is seen as having difficulties
in being detached from his or her experience and is often overwhelmed by it. Developing the capacity for
being mindful and living in the moment allows a greater potential for feeling in charge of the self. In this
way DBT combines behavioral therapy and mindfulness. A related concept is the balance between accep-
tance and change. The most difficult idea for some is that the world is as it is. But again, there is some
paradox in the notion that acceptance—for instance, of unchangeable traumatic events in the past—may
be necessary for change to be possible. DBT provides coping strategies to deal with an invalidating envi-
ronment. DBT therapists are often available 24 hours per day for crisis support.
Psychotherapy
K&S Chapter 28

Question 89. E. Most patients with OCD have both obsessions and compulsions—up to 75% according to some stud-
ies. Some psychiatrists feel that this statistic is closer to 100%, if the patient is properly assessed for
both mental and behavioral compulsions. Nevertheless, as per DSM-IV-TR, a patient need not have
both obsessions and compulsions to meet criteria for OCD; one or the other is sufficient. Obsessions
are intrusive thoughts, and compulsions are intrusive behaviors. In OCD, an idea or impulse intrudes
insistently and persistently into a person’s conscious awareness. Remember that by DSM diagnostic stan-
dards, compulsions are defined as repetitive behaviors or mental acts that the person feels driven to per-
form in response to an obsession or according to rules that must be applied rigidly. These behaviors (or
mental acts) are targeted at reducing distress or anxiety and are clearly excessive in their nature. In this

377

Psychiatry Test Preparation and Review


Manual E-Book
Psychiatry Test Preparation & Review Manual

particular question, the patient presents with compulsive orderliness, alphabetizing, and color-coding
behaviors. There is no mention of obsessions, though the criteria for OCD are met without that stipula-
tion in this case. While on the topic of OCD, keep an eye out for test questions involving the sudden
onset of OCD symptoms in a child. This could be PANDAS (Pediatric Autoimmune Neuropsychiatric
Disorder Associated with Streptococcal Infections)! In this syndrome the child develops OCD symptoms
and/or tics after a strep infection. If it looks like this may be the case, order a throat culture for strep!
Obsessive Compulsive and Related Disorders
K&S Chapter 10

Question 90. B. By DSM 5 criteria, pedophilia is defined as recurrent, intense sexually arousing fantasies, sexual urges,
or behaviors involving sexual activity with a prepubescent child or children, generally 13 years of age or
younger, persisting over at least 6 months. Either the perpetrator has acted on these sexual urges or the
urges or fantasies must cause significant distress or interpersonal difficulty. The perpetrator must be at
least 16 years of age and at least 5 years older than the victim. When a perpetrator is a late adolescent
involved with a 12- or 13-year-old, the pedophilia diagnosis does not apply. Ninety-five percent of pedo-
philes are heterosexual, and 50% are found to be using alcohol at the time of the incident. Keep in mind
that in terms of medication, pedophiles are most likely to respond to leuprolide, which has shown sup-
pression of pedophilic urges in studies.
Paraphilias
K&S Chapter 17

Question 91. D. The mainstay of pharmacologic treatment of childhood ADHD is the stimulant medication class of
agents. Methylphenidate and amphetamine preparations are dopamine agonists. Methylphenidate has
been shown to be effective in about 75% of all children with ADHD. Common side effects of the stimu-
lants are headaches, GI discomfort, nausea, and insomnia. Of course, stimulants also suppress appetite
and can induce weight loss. Some children experience a rebound effect after the wearing-off of the stim-
ulant, during which period they become irritable and hyperactive. Motor tics can also be exacerbated by
the use of stimulants, which warrants caution when the medications are given in this specific population
of children. Methylphenidate is also associated with growth stunting or suppression. This effect seems to
be offset when children are given drug holidays during the summer months when they are not in school.
About 75% of students on stimulant medications demonstrate improvement in attention as measured
by objective tests of their academic performance. Dextroamphetamine and dextroamphetamine/amphet-
amine salt combinations (Adderall) are generally the second choice when methylphenidate fails.
Disruptive, Impulse Control, Conduct Disorders, and ADHD
K&S Chapter 31

Question 92. B. The predominant disturbance in dissociative fugue is sudden, unexpected travel away from home or
one’s customary place of work, with inability to recall one’s past. There is also confusion about one’s
personal identity or the assumption of a new identity. The symptoms must cause significant distress or
impairment in social, occupational, or other important areas of functioning. The fugue state must not
occur during a period of substance abuse, or as part of dissociative identity disorder, or as a consequence
of a medical condition. These fugue episodes can last from minutes to months in duration. Traumatic
circumstances leading to an altered state of consciousness with a wish to flee are generally the underly-
ing cause of most fugue states. The disorder is seen more commonly during natural disasters, wartime, or
times of terrorism or social upheaval. Dissociative fugue is usually treated with psychodynamic psycho-
therapy, attempting to help the patient recover lost memory of his or her identity and recent experiences.
Hypnotherapy is at times also helpful in the process of recovery after fugue states.
Dissociative Disorders
K&S Chapter 12

Question 93. C. Eating disorders occur in about 4% of adolescents and young adults. Anorexia nervosa has its most
common age of onset in the middle teenage years, but up to 5% of cases begin in the 20s. The most
common age of onset is between 14 and 18 years of age. Anorexia nervosa is estimated to occur in
about 0.5% to 1% of adolescent girls. The disorder occurs about 10 to 20 times more frequently in
women than in men. It is most frequent in developed countries, and it is seen with highest frequency

378

Psychiatry Test Preparation and Review


Manual E-Book
Test Number Six

in women whose profession requires thinness, such as acting, modelling, and dance. Anorexia is asso-
ciated with depression in about 65% of cases.
Feeding and Eating Disorders
K&S Chapter 15

Question 94. A. Encopresis is defined by DSM 5 as the repeated passage of feces into inappropriate places, such as
clothes or the floor, whether intentionally or involuntarily. The child must be 4 years of age or older.
The episodes must occur at least monthly for 3 months or more. Encopresis is not due to a general
medical condition. The behavior must cause the child significant distress or social or academic impair-
ment. Boys are found to have encopresis six times more frequently than girls. Encopresis has been
shown to develop with much greater frequency in children with a known history of sexual abuse.
Some studies have associated encopresis with measures of maternal hostility and punitive and harsh
parenting.
Elimination Disorders
K&S Chapter 31

Question 95. E. Irritability, poor concentration, and poor sleep are common to generalized anxiety disorder, MDD, pre-
menstrual dysphoric disorder, and PTSD. Social anxiety disorder is characterized by a fear or anxiety
surrounding social situations in which individuals are exposed to possible scrutiny by others. The cri-
teria center on the patient’s anxiety or fear in specific social situations and do not include other specific
symptoms, such as irritability, decreased concentration, or sleep disturbance. Symptoms in generalized
anxiety disorder include restlessness, fatigue, difficulty concentrating, irritability, muscle tension, or
sleep disturbance. MDD and premenstrual dysphoric disorder contain depressed or irritable mood,
decreased interest in activities, poor concentration, sleep disturbance, and changes in appetite. In PTSD
the patient has experienced actual or threatened death or serious injury, which leads to symptoms of
intrusion, avoidance, negative alterations in cognition and mood, and alterations in arousal and reactiv-
ity. “Alterations in arousal and reactivity” include irritability, poor concentration, and sleep disturbance.
While we’re on the topic of PMDD, keep in mind that an important recommendation for making an
accurate diagnosis is to have the patient keep a daily journal of their symptoms for at least 2 months, so
that a clear correlation between mood symptoms and menses can be established.
Anxiety Disorders
K&S Chapters 9 and 11

Question 96. D. To be diagnosed with pathologic gambling, an individual must present with persistent and recurrent
maladaptive gambling that causes economic problems and significant disturbances in personal, social, or
occupational functioning with at least five of the following symptoms:

1. Preoccupation: The subject has frequent thoughts about gambling experiences, whether past, future,
or fantasy.
2. Tolerance: As with drug tolerance, the subject requires larger or more frequent wagers to experience
the same “rush.”
3. Withdrawal: Restlessness or irritability associated with attempts to cease or reduce gambling.
4. Escape: The subject gambles to improve mood or escape problems.
5. Chasing: The subject tries to win back gambling losses with more gambling.
6. Lying: The subject tries to hide the extent of his or her gambling by lying to family, friends, or
therapists.
7. Loss of control: The person has unsuccessfully attempted to reduce gambling.
8. Illegal acts: The person has broken the law to obtain gambling money or recover gambling losses.
This may include acts of theft, embezzlement, fraud, or forgery.
9. Risked significant relationship: The person gambles despite risking or losing a relationship, job, or
other significant opportunity.
10. Bailout: The person turns to family, friends, or another third party for financial assistance as a result
of gambling.

379

Psychiatry Test Preparation and Review


Manual E-Book
Psychiatry Test Preparation & Review Manual

Studies have pointed toward a neurobiological determinant in pathologic gamblers’ risk-taking behaviors.
Theories have focused on both serotonergic and noradrenergic receptor systems. Evidence supports the prob-
ability that male pathologic gamblers have low plasma MHPG concentrations, as well as increased CSF MHPG
concentrations and increased urinary output of norepinephrine. Chronic gamblers also have decreased platelet
MAO activity, which is a marker of serotonergic dysfunction, which is linked to difficulties with inhibition and
impulse control. Epidemiologic studies point to a prevalence rate of about 3% to 5% of problem gamblers in
the general population at large and about 1% who meet criteria for pathologic gambling.
Disruptive, Impulse Control, Conduct Disorders, and ADHD
K&S Chapter 19

Question 97. C. Conduct disorder is defined, as per the DSM 5, as a repetitive and persistent pattern of behavior in
which the basic rights of others or major age-appropriate societal norms or rules are violated, as mani-
fested by the presence of three (or more) of the following criteria in the past 12 months, with at least one
criterion present in the past 6 months:
Aggression to people and animals:

• O ften bullies, threatens, or intimidates others


• Often initiates physical fights
• Has used a weapon that can cause serious physical harm to others (e.g., a bat, brick, broken bottle,
knife, gun)
• Has been physically cruel to people
• Has been physically cruel to animals
• Has stolen while confronting a victim (e.g., mugging, purse snatching, extortion, armed robbery)
• Has forced someone into sexual activity
Destruction of property:

• H as deliberately engaged in fire setting with the intention of causing serious damage
• Has deliberately destroyed others’ property (other than by fire setting)
Deceitfulness or theft:

• H as broken into someone else’s house, building, or car


• Often lies to obtain goods or favors or to avoid obligations (i.e., “cons” others)
• Has stolen items of nontrivial value without confronting a victim (e.g., shoplifting, but without
breaking and entering; forgery)
Serious violations of rules:
• O ften stays out at night despite parental prohibitions, beginning before age 13 years
• Has run away from home overnight at least twice while living in a parental or parental surrogate
home (or once without returning for a lengthy period)
• Is often truant from school, beginning before age 13 years.
The disturbance of behavior causes clinically significant impairment in social, academic, or occupational
functioning. Many biopsychosocial factors contribute to the manifestation of childhood conduct disorder.
Some of these factors include harsh, punitive parenting; family discord; low socioeconomic status; lack of
proper parental supervision; and lack of social competence. The problems must begin to manifest before
13 years of age. In some children with conduct disorder, low plasma levels of dopamine-β-hydroxylase
have been found. This finding supports the notion of decreased noradrenergic functioning in conduct
disorder. Also, a Canadian study demonstrated greater right frontal EEG activity at rest correlated with
violent and aggressive behavior in children. There is also little doubt that children chronically exposed to
violence and abuse have a higher risk for being violent themselves.
Disruptive, Impulse Control, Conduct Disorders, and ADHD
K&S Chapter 31

Question 98. E. In binge eating disorder, there are recurrent episodes of binge eating consisting of eating rapidly, eat-
ing until uncomfortably full, eating large amounts of food when not hungry, eating alone because of
embarrassment about eating, or feeling disgusted with oneself, depressed, or guilty after eating. It is

380

Psychiatry Test Preparation and Review


Manual E-Book
Test Number Six

differentiated from bulimia nervosa by a lack of inappropriate compensatory behaviors, such as vomit-
ing, laxative use, or excessive exercise.
Feeding and Eating Disorders
K&S Chapter 15

Question 99. D. Carl Jung expanded on Freud’s work with concepts such as archetypes, the collective unconscious, com-
plexes, introverts, extroverts, anima and animus, the persona, and individuation. Erich Fromm defined
five personality types that he felt were common to and determined by Western society. Kurt Goldstein
gave us the term self-actualization, which refers to people using their creative powers to fulfill their
potential. Edith Jacobson proposed theories regarding an infant’s experience of pleasure or lack of plea-
sure and its effect on the mother–infant relationship. Otto Kernberg did a great deal of work concerning
object relations and borderline personality organization.
Psychological Theory and Psychometric Testing
K&S Chapter 4

Question 100. E. Of the options, only opium and organophosphorus insecticides cause miosis. Organophosphate poisons
irreversibly inhibit acetylcholinesterase and cause accumulation of acetylcholine at muscarinic and nico-
tinic receptors.
Muscarinic effects of organophosphorus poisons are:

•  iosis and blurred vision


M
• Increased sweating, salivation, lacrimation
• Increased bronchial secretions, bronchoconstriction
• Abdominal cramps, diarrhea, nausea, vomiting
• Urinary frequency and incontinence
Nicotinic effects of organophosphorus poisons are:

• O n striated muscles, twitching, fasciculations, cramps, and muscle paralysis


• On sympathetic ganglia, hypertension, and tachycardia
CNS effects of organophosphorus poisons are:

• A nxiety, restlessness, and confusion


• Seizures and coma
This child presents with tachycardia and hypertension, which are nicotinic manifestations. Sometimes the
presentation can vary owing to the muscarinic effect on the cardiovascular system, which causes bradycar-
dia and hypotension, but this is due to severe organophosphorus poisoning.
n-Hexane is used as a solvent in paints, lacquers, and printing inks and is used especially in the rubber in-
dustry and in certain glues. Workers involved in the manufacturing of footwear, laminating processes, and
cabinetry, especially in confined, unventilated spaces, may be exposed to excessive concentrations of this
substance. Exposure to this chemical by inhalation or skin contact leads to a progressive distal sensorimo-
tor axonal polyneuropathy; partial conduction block may also occur. Optic neuropathy or maculopathy
and facial numbness also have followed n-hexane exposure.
The major symptoms of botulism are blurred vision, dysphagia, and dysarthria. Pupillary responses to light
are impaired, and reduction of tendon reflex responses is variable. Weakness progresses for several days and
then reaches a plateau. Fatal respiratory paralysis may occur rapidly. Most patients have symptoms of auto-
nomic dysfunction, such as dry mouth, constipation, or urinary retention. The edrophonium test is positive in
approximately one-third of patients and does not distinguish botulism from other causes of neuromuscular
blockade. Wound cultures and serum assay for botulinum toxin confirm the diagnosis of wound botulism.
Treatment consists of bivalent (type A and B) or trivalent (A, B, and E) antitoxin. Antibiotic therapy is not
effective because the cause of symptoms (in all but infantile botulism) is the ingestion of toxin rather than
organisms. Otherwise, treatment is supportive; the need for respiratory assistance is unusual. Acetylcholines-
terase inhibitors are not beneficial. Recovery takes many months but usually is complete.
Poisoning
B&D Chapters 58 and 78

381

Psychiatry Test Preparation and Review


Manual E-Book
Psychiatry Test Preparation & Review Manual

Question 101. C. Oxazepam, lorazepam, and temazepam are only phase II metabolized (via glucuronidation) and as such have no
active metabolites before excretion. The other benzodiazepines first undergo phase I metabolism (oxidation via
the CYP system), which generates active metabolites that then go on to be phase II metabolized and excreted.
Psychopharmacology
K&S Chapter 29

Question 102. D. Benzodiazepines are in pregnancy category D. Category D indicates positive evidence of human fetal
risk. Weigh the risks and benefits of using medication in a pregnant patient.
Psychopharmacology
K&S Chapter 29

Question 103. C. Some of the most common reasons for psychiatric malpractice suits include suicide attempt or com-
pletion by a patient, incorrect or negligent treatment, medication error or drug reaction, and incorrect
diagnosis. Though other choices in this question are causes for malpractice suits, suicide is the most
common. An important fact to remember is that a “contract for safety” with the patient will not protect
the doctor from a malpractice suit if the patient commits suicide. The best defense for the doctor is a
thorough, well-documented suicide assessment.
Forensic Psychiatry
K&S Chapter 34

Question 104. A. SSRIs delay ejaculation. Yohimbine, alprostadil, and sildenafil can be used for impotence. Testosterone
can increase sexual desire but is masculinizing.
Sexual Dysfunction
K&S Chapter 17

Question 105. B. OCD occurs at equal rates in men and women. All other choices listed in this question occur more in
males. Don’t forget this important fact for your examination.
Statistics
K&S Chapter 10

Question 106. B. Here are commonly asked forensics terms. You must know them all.
A socially harmful act is not enough to have committed a crime. To be found guilty the accused must also
have mens rea (evil intent) and actus reus (voluntary conduct). In the case of many mentally ill patients
who commit socially harmful acts, they lack one of these elements.
Parens patriae is a doctrine that allows the state to intervene and act as a surrogate parent for those who
are unable to care for themselves.
Respondeat superior holds that a person occupying a high position in a hierarchy is responsible for those
in lower positions.
Substituted judgment is when a surrogate makes a decision on the basis of what the patient would have
wanted, taking into account his or her wishes and values.
Justice is the equitable distribution of social resources and benefits.
Forensic Psychiatry
K&S Chapter 34

Question 107. E. Studies support CBT as the best therapy for panic disorder. SSRIs are also considered first line because of
their effectiveness, safety, and low side effect profile. Studies also support the fact that CBT plus an SSRI
will deliver better results than either treatment given alone.
Anxiety Disorders
K&S Chapter 9

Question 108. B. Mahler’s stages of separation–individuation are back again! This question describes symbiosis, which
lasts from 2 to 5 months. In this stage the baby is developing the ability to distinguish the inner from the
outer world. The child perceives itself as being part of a single entity with its mother.

382

Psychiatry Test Preparation and Review


Manual E-Book
Test Number Six

Human Development
K&S Chapter 31

Question 109. E. The addition of a benzodiazepine to a patient on an SSRI for panic disorder will lead to a more rapid
resolution of the anxiety. The SSRI will take 2 to 4 weeks to work in most cases. The benzodiazepine
can effectively control the patient’s symptoms until the SSRI is fully working. The other choices will take
longer to titrate or become effective and are not good choices for rapid resolution of panic symptoms.
Propranolol is indicated only for performance anxiety and should not be used in other anxiety disorders.
Anxiety Disorders
K&S Chapter 9

Question 110. C. Carbamazepine acts like a vasopressin agonist and has antidiuretic actions. As such, it can cause hypo-
natremia, especially in the elderly. The other choices are not known for this particular side effect.
Psychopharmacology/Laboratory Tests in Psychiatry
K&S Chapter 29

Question 111. D. Risperidone does not affect thyroid function tests but may alter the other choices listed as part of a
metabolic syndrome. In addition the clinician may wish to consider an electrocardiogram if the patient
has significant cardiac risk factors, as well as hemoglobin A1c.
Psychopharmacology/Laboratory Tests in Psychiatry
K&S Chapter 29

Question 112. D. The psychotherapist should maintain neutrality with his or her patients. Personal information should
be limited and given only if it is to help the patient in some way. In this case, his telling the patients is
more about his grief than about the needs of the patients. It is therefore a boundary violation. Whenever
revealing personal information, therapists should ask themselves if they are doing it for the patient or
themselves. Choices A and B are too black and white to be true. There may be situations in which a
therapist can reveal personal information, but the therapist must examine why he or she is doing it first.
Choice E misses the central issue in the ethical dilemma.
Ethics
K&S Chapter 36

Question 113. D. Kappa is a number that is used for binary data and tells us whether a given procedure or test produces
reliable or reproducible results. It is often used to measure the degree of agreement between raters in a
study. Correlation coefficient indicates reliability for nonbinary data, such as continuous measurements.
Point prevalence is the number of cases at a specific point in time divided by the population at the same
time. Period prevalence looks at the number of cases both existing and new during a specific time period.
Lifetime prevalence reflects the proportion of people who have ever had a specific condition during their
lifetime.
Statistics
K&S Chapter 5

Question 114. C. This is a common case of transference that is expected to develop in the course of psychodynamic psy-
chotherapy. The patient is reenacting her relationship with her mother with the therapist. The therapist
is not rejecting the patient; her schedule does not allow her to meet three times per week. Nor is there
evidence to suggest that countertransference on the part of the therapist is the issue here. There is no
evidence of psychosis, or delusion, or of an erotomanic theme to the therapy. We have no evidence that
the patient is stalking the therapist, nor would it be reasonable to assume that this is the case given the
information provided. Displacement is taking the emotional energy from one object and placing it onto
another, unrelated object. The patient is not doing that in this question.
Psychotherapy
K&S Chapter 5

383

Psychiatry Test Preparation and Review


Manual E-Book
Psychiatry Test Preparation & Review Manual

Question 115. E. Neuroimaging is not considered standard of care for mania. It should be considered in all of the other
scenarios listed in this question, as well as before initial ECT treatments, with acute mental status
change after head trauma, and in dementia of unknown etiology.
Laboratory Tests in Psychiatry
K&S Chapter 5

Question 116. A. Exposure with response prevention is considered the “gold standard” for treatment of OCD. Make sure
you know this fact before you sit for your examination!
Obsessive Compulsive and Related Disorders
K&S Chapter 10

Question 117. B. The abnormal formation of the tricuspid valve you should be concerned with here is Ebstein’s anomaly.
It is the result of taking lithium during pregnancy, especially during the first trimester. Also worth noting
is that valproic acid taken during pregnancy causes neural tube defects in the fetus.
Psychopharmacology
K&S Chapter 29

Question 118. D. For many women, libido increases during pregnancy, and it is incorrect to say that most women expe-
rience absence of sexual desire during pregnancy. The other choices are part of normal adjustment to
pregnancy. Many women experience pregnancy as a means of self-realization, project hope onto the
child, are afraid of inadequate mothering, or have unconscious fears and ambivalence regarding the
effect of the mother-child relationship on the relationship between father and mother.
Human Development
K&S Chapter 27

Question 119. E. Competence is decided by the court. Capacity is decided by the psychiatrist. The four standards of
capacity are:

1. Ability to communicate a choice


2. Ability to understand the information presented
3. Ability to understand how information applies to one’s own circumstances
4. Ability to reason in a logical manner (even if most people wouldn’t agree with the final decision)
Forensic Psychiatry
K&S Chapter 34

Question 120. A. The Benton Visual Retention Test is one in which a patient is presented with geometric figures for 10
seconds and then asked to draw them from memory. It is a test of short-term memory. The Wechsler
Memory Scale is the most widely used memory test for adults. The Bender Visual-Motor Gestalt Test is
a test of visuomotor coordination and is useful in testing maturational levels in children. The Wisconsin
Card Sorting Test assesses abstract reasoning and difficulty in problem solving.
Psychological Theory and Psychometric Testing
K&S Chapter 5

Question 121. B. Schizoid fantasy is indulging in autistic-like retreat to resolve conflict. Interpersonal intimacy is avoided,
and others are driven away by the eccentricity. It is an immature defense. Anticipation is planning for
future inner discomfort and is mature. Altruism is using service to others to undergo an internally grati-
fying experience and is mature. Humor is using comedy to express feelings without becoming uncom-
fortable or causing discomfort in others and is mature. Suppression is consciously postponing attention
to an impulse or conflict and is mature.
Psychological Theory and Psychometric Testing
K&S Chapter 4

384

Psychiatry Test Preparation and Review


Manual E-Book
Test Number Six

Question 122. B. Lithium has been associated with hyperplasia and adenomas of the parathyroid glands leading to hyper-
parathyroidism and hypercalcemia. Neutropenia is associated with clozapine. Pancreatitis is a rare but
potentially deadly side effect of valproic acid. Hepatic failure is associated with valproic acid, hence the
black box warning in the Physician’s Desk Reference. Eosinophilic colitis is associated with clozapine.
Though these drugs are certainly not the only causes of these conditions, these associations would be
useful for the prudent student to know.
Psychopharmacology
K&S Chapter 29

Question 123. E. To be good candidates for brief psychodynamic therapy, patients must be able to identify and discuss
their feelings. The best candidates are highly motivated, have an area of focus for the therapy, and have
at least one solid relationship. You are looking for psychological mindedness and the ability to learn
through relationships. Patients who are too concrete, are regressed, and have no meaningful relation-
ships are usually not good candidates. The ability to take a daily medication is not particularly relevant.
Psychotherapy
K&S Chapter 28

Question 124. A. Object relations theory is best represented by the work of Melanie Klein. Object relations theory is
known for the schizoid, paranoid, and depressive positions, as well as tension between the true and the
false self. The other notable therapists listed in this question will undoubtedly be the subject of other
questions throughout this book.
Psychotherapy
K&S Chapter 4

Question 125. C. Motivational enhancement therapy is a treatment that has proven successful at treating drug addiction,
especially in cases in which there is denial and ambivalence on the part of the substance abuser. Also be
aware of Matrix therapy for substance abuse. It consists of relapse prevention groups, education groups,
social support groups, individual counseling, and urine and breath testing delivered in a structured man-
ner over a 16-week period. The treatment is a directive, non-confrontational approach that focuses on
current issues and behavior change. Matrix therapy has been known to show up on standardized exami-
nations. The patient does not suffer from depression or anxiety, so fluoxetine is not appropriate. There
is no evidence that Joe is a borderline, so dialectical behavior therapy is not warranted. ECT is used for
depression, psychosis, and mania, not crack addiction. We don’t have adequate information to make a
judgment call about involuntary hospitalization.
Substance Abuse and Addictive Disorders/Psychotherapy
K&S Chapters 20 and 28

Question 126. B. It is considered unethical to have any business involvement with patients or former patients. It is unethi-
cal to get patients to leave you anything in their will or to solicit them to leave anything to anyone in
their will. As such the answer here is that you will not ask any of your current or former patients to
make donations in any way.
Ethics
K&S Chapter 36

Question 127. C. In its most basic form, interpretation is making something that was unconscious conscious. It is an
explanatory statement that links a feeling, thought, behavior, or symptom to its unconscious meaning.
Confrontation is addressing something the patient does not want to accept. Clarification is reformu-
lating what the patient has said to create a coherent view of what has been communicated. Empathic
validation is a demonstration of the therapist’s empathic understanding of the patient’s internal state.
Affirmation is giving succinct comments in support of the patient’s statements or feelings.
Psychological Theory and Psychometric Testing
K&S Chapter 28

385

Psychiatry Test Preparation and Review


Manual E-Book
Psychiatry Test Preparation & Review Manual

Question 128. B. This is a common ethical situation encountered by physicians. Accepting gifts from a drug representa-
tive is a conflict of interest and is ethically questionable. There is evidence that gifts influence physician
behavior, which is why drug companies try to give you things! They’re not giving you things just to be
nice. They know it will influence prescribing habits. The monetary value of the gift is not the determin-
ing factor in whether it is unethical. Physicians have done very poorly at self-regulating with regard to
this issue. Doctors are just as influenced by gifts as anyone else in the office.
Ethics
K&S Chapter 36

Question 129. B. The proper way to work with an interpreter is to look at and direct questions to the patient. The inter-
preter should then translate the discussion for both sides. Don’t look at the interpreter. The interpreter is
not your patient. Having the family in the room is problematic because they may misinterpret or answer
the questions without asking the patient, and their involvement may pose a problem in terms of patient
confidentiality. Shouting at people who do not speak English is common but unsuccessful. They’re not
deaf; they can’t speak English, no matter how slowly or loudly you say it!
Psychotherapy
K&S Chapter 5

Question 130. B. Epigenetic is a term used to describe Erikson’s stages of development. Each stage contains a psycho-
logical crisis, which must be negotiated before moving on to the next phase. Levinson’s theories
divided adult development up into four stages, each lasting about 25 years. Vaillant found that a happy
childhood correlated with positive traits in middle life and that adaptive styles mature over time. He
postulated that this maturation was dependent on internal development rather than changes in the envi-
ronment. Neurodevelopmental theory concerns itself with physical brain development, formation and
connection of neurons, and brain plasticity. In normality-as-process, changes in personality over time are
an essential part of the developmental process. Emphasis is placed on the changes one undergoes over
time, rather than labeling where a person should be at a given age.
Human Development
K&S Chapter 2

Question 131. C. Routine monitoring for patients on lithium includes all of the choices except urine for creatinine and
protein. However, this test should be considered in patients who have renal disease and are on lithium.
Additional tests one might consider include pregnancy tests for women of childbearing age.
Psychopharmacology/Laboratory Tests in Psychiatry
K&S Chapter 29

Question 132. C. DSM 5 criteria for panic disorder state that the patient must have recurrent attacks of which at least one
is followed by 1 month or more of persistent concern about having more attacks, worry about the impli-
cations of the attack or its consequences, or a significant change in behavior related to the attacks.
Anxiety Disorders
K&S Chapter 9

Question 133. C. The DSM 5 states that to meet criteria for generalized anxiety disorder, excessive anxiety and worry
about a number of events or activities must occur more days than not for at least 6 months. The indi-
vidual finds it difficult to control the worry. The worry is associated with symptoms such as restlessness,
fatigue, difficulty concentrating, irritability, muscle tension, or sleep disturbance. It causes impairment in
social or occupational functioning. It is not attributable to substance abuse or another mental disorder.
Lifetime prevalence of generalized anxiety disorder is around 5%.
Anxiety Disorders
K&S Chapter 9

Question 134. D. Methylenedioxymethamphetamine (ecstasy) is known for a hyperthermic syndrome that can progress
to disseminated intravascular coagulation, rhabdomyolysis, liver and kidney failure, and death. Adverse
effects of ecstasy are not dose related, so this syndrome can occur at any dose. This is one of those facts

386

Psychiatry Test Preparation and Review


Manual E-Book
Test Number Six

that you must know for your examination and need to just memorize. Just in case you weren’t sure,
flunitrazepam is Rohypnol. The other drugs mentioned are found in questions throughout this book.
Substance Abuse and Addictive Disorders
K&S Chapter 20

Question 135. D. When treating social anxiety disorder, combining CBT and pharmacotherapy does not show a clear ben-
efit over using just one or the other for most initial treatments. There is evidence, however, that there are
some refractory cases that do respond better to a combination of both.
Anxiety Disorders
K&S Chapter 9

Question 136. D. Considering the choices given, it is hoped that lamotrigine jumped out at you. The other choices are all
very well known for causing significant weight gain. Clozapine and olanzapine cause the most weight
gain of all the antipsychotics and carry high rates of metabolic syndrome. Valproic acid and lithium are
both known to cause significant weight gain, among other troublesome side effects. Lamictal, however, is
relatively weight neutral, and significant weight gain is rare.
Psychopharmacology
K&S Chapter 29

Question 137. D. Skills training is part of a cognitive behavioral approach to substance abuse. Patients are helped to
change beliefs, improve interpersonal communication, resist social pressure, increase assertiveness, and
better manage stress. Unconscious symbolism is part of a psychodynamic approach to therapy, not part
of a cognitive behavioral approach.
Substance Abuse and Addictive Disorders
K&S Chapter 20

Question 138. B. Only physicians can order the start of a new seclusion once the previous seclusion order has ended. The
nurse cannot extend the time on his or her own. All other answer choices are correct.
Forensic Psychiatry
K&S Chapter 36

Question 139. D. The lowest effective methadone dose for most patients is 50 mg per day. The average methadone dose
for opiate addiction is 80 mg per day. Doses of 80 mg per day are twice as likely to lead to successful
outcomes as doses of 50 mg or less. Also never forget that carbamazepine lowers methadone levels. So if
you have a bipolar patient who is also on methadone, and they are placed on carbamazepine for mood,
you should expect their methadone levels to drop. So either the dose needs to be increased, or there is a
significant chance of opiate relapse. Another possible scenario is that a patient on methadone gets placed
on ketoconazole for a fungal infection and is suddenly over-sedated and appears intoxicated. This is
because ketoconazole will elevate methadone levels. Standardized test writers love to ask about these
scenarios, so they are valuable for the prudent student to know.
Substance Abuse and Addictive Disorders
K&S Chapter 20

Question 140. E. Patients who have a high likelihood of going into the delirium tremens based on history should be
detoxified on an inpatient basis. This is a major safety concern, because the delirium tremens can be life
threatening. As such, this takes precedence over other considerations.
Substance Abuse and Addictive Disorders
K&S Chapter 20

Question 141. E. The best medication for this patient would be an SSRI to control the OCD symptoms and a dopamine
blocker to control the tics. When we look at the choices, the one that fits is choice E. Fluvoxamine is an
SSRI with proven efficacy in treating OCD. Haloperidol is a strong dopamine blocker that can effec-
tively control tics. Benzodiazepines, buspirone, and naltrexone are not used for tics.
Psychopharmacology
K&S Chapter 29

387

Psychiatry Test Preparation and Review


Manual E-Book
Psychiatry Test Preparation & Review Manual

Question 142. B. The DSM 5 criteria for hypomania specifically state that there are no psychotic features. If psychosis
exists, then the mood episode would be considered mania. Major depression, mania, mixed episodes,
and delirium can all involve psychosis. In addition, it should be noted that dementia can also present
with psychosis.
Bipolar Disorders
K&S Chapter 8

Question 143. E. To meet criteria for bipolar I the patient must have at least one manic episode. So bipolar I is out in
this case. For bipolar II, the patient must have one hypomanic episode and one episode of MDD. So
bipolar II is out in this case because we have no depressive symptoms. To meet cyclothymic disorder,
one must have hypomanic symptoms as well as depressive symptoms that do not meet criteria for major
depression and that go on for at least 2 years. This patient has no depressive symptoms, so cyclothymic
disorder is out. There is no evidence of any substance abuse in the question stem, so substance-induced
mood disorder is out. The correct answer is other specified bipolar and related disorder, which includes
patients with clear bipolar symptoms who do not meet criteria for any specific bipolar disorder.
Bipolar Disorders
K&S Chapter 8

Question 144. E. Symptoms of pheochromocytoma can include anxiety, panic, tremulousness, flushing, feeling of impend-
ing doom, hypertension, and tachycardia. It can be mistaken for panic disorder. Hyperparathyroidism
presents as constipation, polydipsia, nausea, depression, paranoia, and confusion. Hypothyroidism pres-
ents with cold intolerance, weight gain, brittle hair, goiter, constipation, dry skin, lethargy, depressed
affect, hallucinations, and paranoia. Raynaud’s phenomenon presents with idiopathic paroxysmal bilat-
eral cyanosis of the digits and can be exacerbated or triggered by stress. Crohn’s disease does not have
psychiatric manifestations.
Somatic Symptom Disorders
K&S Chapter 13

Question 145. D. Compared with the general population, patients with social phobia tend to have fewer friendships,
lower levels of education, higher rates of suicide, and less success in career advancement. They also have
poorer marital function.
Anxiety Disorders
K&S Chapter 9

Question 146. C. Regularly scheduled primary care appointments are a crucial and standard part of treating illness anxi-
ety disorder. In illness anxiety disorder the patients are convinced that they have an illness that they
don’t have. If they are not followed closely by their primary care physician, they will go doctor shop-
ping. While regularly following up with the primary care doctor, they should also be engaged in psycho-
therapy to work on the underlying psychological causes of the condition. None of the other disorders
listed require regular follow-up by the primary care doctor. That is not to say these patients should not
regularly follow-up for routine medical care, but that it is not a crucial component of the treatment for
the psychiatric condition.
Somatic Symptom Disorders
K&S Chapter 13

Question 147. C. Alcohol dependence, being widowed, being unemployed, and being socially isolated are all factors that
increase chances of suicide in the elderly. Add to that list male gender, chronic illness, depression, irrita-
tion, rage, violence, and affective instability.
Management in Psychiatry
K&S Chapter 23

Question 148. B. For a patient on clozapine who develops an ANC between 500 and 999, the correct action is to stop the
clozapine. The clozapine may be tried again when the patient’s ANC improves. Monitor the ANC daily
until it is over 1000. For a patient whose ANC is between 1000 and 1499, you can switch to 3 times

388

Psychiatry Test Preparation and Review


Manual E-Book
Test Number Six

weekly monitoring until it is back above 1500. If the ANC is lower than 500, the patient can be rechal-
lenged after hematology consult and based on a risk-benefit analysis.
Psychopharmacology
K&S Chapter 29

Question 149. E. QTc interval is normally less than 450 milliseconds. Prolongation of the QTc interval longer than
500 milliseconds increases the risk for torsade de pointes. Under these circumstances, any medication
that may be increasing the QTc must be considered carefully and possibly discontinued, changed, or
decreased. A practical example of this would be if a patient you were working with became agitated.
Let’s say the ECG done earlier showed a QTc of 527. You might normally use Haldol to treat the agita-
tion, but it is now contraindicated because elevating the QTc any further could lead to an arrhythmia.
Under these circumstances, you would need to treat the agitation with a medication that does not pro-
long QTc and re-evaluate their baseline medication regimen to see if you could bring their QTc down
under 500 ms.
Psychopharmacology
K&S Chapter 29

Question 150. A. Interferon is well known to exacerbate depression. Side effects include depressed mood, suicidality, sui-
cidal ideation, insomnia, fatigue, and cognitive inefficiency. The other medications listed are not known
to cause the exacerbation of depressive symptoms listed in this question.
Psychopharmacology
K&S Chapter 29

389

Psychiatry Test Preparation and Review


Manual E-Book
This page intentionally left blank

     

Psychiatry Test Preparation and Review


Manual E-Book
Vignettes

Vignettes
Explanations

Vignette One
Gabriel Matthews is a 42-year-old construction worker who comes to you for help. Six months ago, he was working with a
chainsaw on a construction site and didn’t realize that the gas cap on the saw was loose. The cap came off, spilling gasoline
all over his clothes and the saw. The heat from the engine ignited the gas, setting his clothes on fire. He ran around the con-
struction site ablaze until three other workers came to his aid and extinguished the fire by smothering it with clothing and
dirt. He suffered severe burns and spent a significant amount of time in a burn unit.
Five weeks after the accident you are called to consult on him in the burn unit because he is having psychiatric symptoms
that started 2 days after the fire and are progressively getting worse. He is having distressing nightmares about being on fire
that wake him from sleep. His mood is low, and he feels unable to be happy about anything. He is hopeless about the future
and feels he has nothing to look forward to. He is getting more and more upset as his days in the unit go on, and he has a
short temper with the nurses. He keeps sending away visitors who come from his job and who were there the day that the
accident happened. He has had several incidents of yelling at various family members when they came to visit. You meet his
sister while on the unit, who cries as she tells you how hostile Gabriel has been toward the family lately.

1. Gabriel’s most accurate diagnosis is:


A. 
Major depressive disorder
B. 
Adjustment disorder with depressed mood
C. 
Posttraumatic stress disorder
D. 
Mood disorder secondary to a medical condition

2. 
Gabriel inquires about treatment available for his condition. Which treatments would you consider? (Choose three of five)
A. 
Family therapy
B. 
Selective serotonin reuptake inhibitors (SSRIs)
C. 
Benzodiazepines
D. 
Cognitive behavioral therapy
E. 
Dialectical behavior therapy

Full test - and additional VIDEO vignettes - available online - see inside front cover for details.

391

Psychiatry Test Preparation and Review


Manual E-Book
Psychiatry Test Preparation & Review Manual

3. 
Which of the following factors would be predictive of a poor prognosis for Gabriel? (Choose two of five)
A. 
Rapid onset of symptoms
B. 
Strong social supports
C. 
Absence of other Axis I disorders
D. 
Duration of symptoms greater than 6 months
E. 
Borderline personality disorder

4. 
Which one of the following symptoms is commonly found in patients with Gabriel’s disorder?
A. 
Tactile hallucinations
B. 
Thyroid abnormalities
C. 
Decreased norepinephrine turnover in the locus ceruleus
D. 
Alexithymia

5. 
In addition to medication, which of the following would be considered appropriate treatment approaches for this disor-
der? (Choose four of five)
A. 
Overcome the patient’s denial of the traumatic event
B. 
Use imaginal techniques or in vivo exposure
C. 
Encourage proper sleep, providing medication if necessary
D. 
Promote full discharge of aggression as a cathartic exercise to relieve irritability
E. 
Teach the patient cognitive approaches to dealing with stress

6. 
Which of the following symptoms can be found in both schizophrenia and posttraumatic stress disorder (PTSD)? (Choose
two of four)
A. 
Angry outbursts
B. 
Restricted affect
C. 
Decreased need for sleep
D. 
Exaggerated startle response

7. 
Which of the following illnesses can present with decreased sleep? (Choose three of four)
A. 
Bipolar I disorder
B. 
Generalized anxiety disorder
C. 
PTSD
D. 
Obsessive–compulsive disorder

8. 
In Gabriel’s case, he associated the trauma of the fire with chainsaws. For years afterward he would have severe anxiety
whenever he saw a chainsaw. He would avoid going near the outdoor power equipment whenever he was in a hardware
store. This is a good example of which one of the following?
A. 
Operant conditioning
B. 
Learned helplessness
C. 
Classical conditioning
D. 
Premack’s principle

9. 
Which one of the following is not considered a symptom of altered arousal and reactivity when diagnosing PTSD?
A. 
Poor concentration
B. 
Outbursts of anger
C. 
Feelings of detachment from others
D. 
Difficulty falling asleep

392

Psychiatry Test Preparation and Review


Manual E-Book
Vignettes

10. Which of the following should be considered in the differential for PTSD? (Choose four of five)
A. 
Panic disorder
B. 
Substance abuse
C. 
Major depressive disorder
D. 
Borderline personality disorder
E. 
Schizotypal personality disorder

Vignette Two
A 65-year-old woman presents to your office with a complaint of long-standing symptoms that have plagued her since her
adolescence. She reports chronic suicidal ideation, low mood, and inability to focus or concentrate. Her memory is fairly
good, but she doesn’t enjoy anything that she used to. She used to play cards with friends, drive herself to the mall to go
shopping, and take trips to visit her children and grandchildren in various cities. She denies hearing voices or having para-
noid or suspicious thoughts about people. Her sleep is very broken, and she gets only about 4 hours each night. She has no
motivation to shop, cook, or clean for herself, and she admits it to you. Her appetite is poor, and she has already lost 20 lb
over the past year from not eating properly.
Her eldest daughter, age 42, accompanies her to your office. Her daughter is quite concerned for her mother because
she has been on “every antidepressant you can imagine.” As a nurse, her daughter is able to rattle off a list of medications
that her mother has tried in the past: imipramine, doxepin, phenelzine, fluoxetine, paroxetine, venlafaxine, duloxetine. None
of these improved her back to her baseline. She has also had trials of several of these medications with other augmenting
agents, such as methylphenidate, lorazepam, aripiprazole, lithium, and buspirone.
On examination in your office, the patient is conversant and coherent but very slow to speak, and her affect is blunted
and speech is quiet and monotonous with marked alogia. She denies suicidal or homicidal thoughts or intentions at this time.

1. 
Your immediate clinical thoughts after interviewing this patient should be focused on:
A. 
Sending her home with a trial of bupropion and desvenlafaxine at high doses along with l-methylfolate for augmenta-
tion, because she has never been on these agents
B. 
Admitting her to the psychiatric hospital voluntarily for inpatient electroshock therapy
C. 
Getting her a bed in a local skilled nursing facility because she cannot manage her activities of daily living
appropriately
D. 
Considering reporting her daughter to the authorities for elder abuse
E. 
Enlisting a local assertive community treatment (ACT) team to pick up her care and service her needs in her home
instead of in a clinic setting

2. 
You decide to admit her to the hospital, and she agrees to go on a voluntary basis. Before considering electroshock ther-
apy, which of the following would be appropriate to do as a pretreatment evaluation? (Pick three of six)
A. 
Blood work for blood count and comprehensive chemistry (complete blood count [CBC] and chemistry screen
[chem-20])
B. 
Head computed tomography (CT) scan or magnetic resonance imaging (MRI)
C. 
Thyroid function tests
D. 
Electrocardiogram
E. 
Electroencephalogram
F. 
Neck and spine radiography

393

Psychiatry Test Preparation and Review


Manual E-Book
Psychiatry Test Preparation & Review Manual

3. 
Which of the following is a contraindication to electroshock therapy?
A. 
Pregnancy
B. 
Space-occupying brain lesion
C. 
Recent myocardial infarction within the past month
D. 
Hypertension
E. 
There are no absolute contraindications to electroshock therapy

4. 
For a seizure to be deemed effective in electroshock therapy sessions, its duration must be at least:
A. 
5 seconds
B. 
15 seconds
C. 
25 seconds
D. 
45 seconds
E. 
60 seconds

5. 
Which of the following are not generally considered to be adverse effects of electroshock therapy? (Pick two of six)
A. 
Death
B. 
Headache
C. 
Nausea and vomiting
D. 
Dizziness and lightheadedness
E. 
Hypertension
F. 
Delirium

6. 
Which of the following situations is an indication for maintenance electroshock therapy after an initial successful group of
treatments? (Pick three of six)
A. 
Severe medication side effects and intolerance
B. 
Profound memory loss after the initial treatment sessions
C. 
Psychotic or severe symptoms
D. 
Rapid relapse after successful initial treatment sessions
E. 
Delirium resulting from initial treatment sessions
F. 
Pregnancy

7. 
Which of the following medications should be discontinued before electroshock therapy administration? (Pick three of six)
A. 
Venlafaxine
B. 
Phenelzine
C. 
Clozapine
D. 
Fluoxetine
E. 
Bupropion
F. 
Alprazolam

8. 
Which of the following agents is not generally used as an anesthetic agent in electroshock therapy because of its strong
anticonvulsant properties?
A. 
Methohexital (Brevital)
B. 
Ketamine (Ketalar)
C. 
Etomidate (Amidate)
D. 
Propofol (Diprivan)
E. 
Alfentanil (Alfenta)

394

Psychiatry Test Preparation and Review


Manual E-Book
Vignettes

9. 
Which of the following is the typical course of electrode placement in electroshock therapy that is followed by most
practitioners?
A. 
Start with bilateral electrode placement always, because this is more effective
B. 
Start with bilateral electrode placement, but move to unilateral placement if persistent memory loss occurs after six
sessions
C. 
Start with unilateral electrode placement, but move to bilateral placement if no improvement is seen after four to six
unilateral treatments
D. 
Start with unilateral electrode placement always, because this is safer and causes fewer side effects
E. 
Start with unilateral electrode placement always, making sure placement is over the nondominant hemisphere to avoid
language and cognitive deficits

Vignette Three
Cathy Kelly is a 31-year-old computer programmer who works for a website design company. She comes to your office with
reports of decreased mood, poor appetite, poor concentration, and feelings of worthlessness. She states, “I haven’t gotten a
good night’s sleep in weeks, and I’ve lost about 10 pounds recently.” These symptoms have been present for the past 5 weeks.
On further questioning she describes a period 2 years ago when she “had some trouble” around her sister’s wedding. In the 4
days leading up to the wedding she was sleeping only 2 hours per night. She tells you, “I wasn’t tired and had enough energy
to make pastries and gifts for the wedding guests. I was up working almost all night.” She recalls that “brilliant ideas for new
projects” were running through her mind at that time. She continued going to work at her computer programming job dur-
ing those 4 days and felt that she was very productive. When the wedding came, she drank excessively and used her position
as a bridesmaid to meet single friends of the groom. She took several men into a secluded bathroom and had sex with them
during the wedding reception. When asked about substance abuse, she reports using both cocaine and alcohol in the past to
“make me feel better.” She denies any cocaine use around the time of her sister’s wedding, however.

1. 
Which one of the following would be the most appropriate diagnosis for Cathy?
A. 
Bipolar I disorder
B. 
Major depressive disorder
C. 
Bipolar II disorder
D. 
Cyclothymic disorder
E. 
Substance/medication-induced bipolar and related disorder

2. 
Which one of the following is a key differentiating factor between mania and hypomania?
A. 
Irritable mood
B. 
Decreased need for sleep
C. 
Marked impairment in social or occupational functioning
D. 
Flight of ideas

3. 
Which of the following factors should influence your choice of medications for Cathy? (Choose four of five)
A. 
The presence of psychosis
B. 
The presence of rapid cycling
C. 
The severity of symptoms
D. 
Pregnancy
E. 
Age

395

Psychiatry Test Preparation and Review


Manual E-Book
Psychiatry Test Preparation & Review Manual

4. 
In which of the following scenarios would you consider electroconvulsive therapy (ECT) for Cathy? (Choose two of four)
A. 
Cathy is pregnant and currently manic
B. 
Cathy has severe mania and psychosis that has responded poorly to medication
C. 
Cathy has mania secondary to a medical condition
D. 
Cathy has substance-induced mania

5. 
Which of the following medical conditions can be associated with mania? (Choose three of four)
A. 
Glioma
B. 
Cushing’s disease
C. 
Multiple sclerosis
D. 
Thiamine deficiency

6. 
Which of the following medications can cause a manic episode? (Choose four of five)
A. 
Isoniazid
B. 
Cimetidine
C. 
Metoclopramide
D. 
Steroids
E. 
Oxazepam

7. 
During the periods in which Cathy used cocaine, which of the following were true concerning her brain? (Choose three of
four)
A. 
Dopamine activity increased in the corpus striatum
B. 
Dopamine activity decreased in the mesocortical pathway
C. 
Dopamine activity increased in the mesolimbic pathway
D. 
There was both dopamine and norepinephrine reuptake inhibition

8. 
Which one of the following is not a potential sequelae of cocaine use?
A. 
Onset of hallucinations and paranoia
B. 
A significant appearance of lights in the central visual field
C. 
Hypersexuality
D. 
Itching and respiratory depression

9. 
Which of the following have a role in the treatment of cocaine overdose? (Choose three of four)
A. 
Intravenous diazepam
B. 
Haloperidol
C. 
Intravenous phentolamine
D. 
Clonidine

10. What percentage of patients with bipolar disorder have a co-occurring substance disorder like Cathy?
A. 
10%
B. 
30%
C. 
60%
D. 
80%

396

Psychiatry Test Preparation and Review


Manual E-Book
Vignettes

Vignette Four
Susan Walton is a 20-year-old college student. You interview her in the emergency room after an overdose of Tylenol. She
reports that she was happily shopping with her boyfriend when he spotted an attractive woman on the other side of the
street. “He’s such an asshole,” she tells you. She says he watched the woman closely as she walked away, and Susan was
certain that he was attracted to her. “I could tell by the way he was looking at her. She was such a whore. Is that what he
wants? A whore like that?” she screams at you. According to her boyfriend, Susan then reached into her purse, pulled out
a bottle of Tylenol, and swallowed as many pills as she could before her boyfriend wrestled the bottle from her hands. He
became panicked and brought her to the emergency room. On the drive in, she scratched up and down her arms using her
fingernails, breaking the skin. She then began biting her forearms until they bled. He tried to pull the car over and stop her,
but she scratched his face when he tried to intervene. When she arrived in the emergency room, she was crying hysterically
and cursing at her boyfriend. When he attempted to comfort her, she spat on him and smacked him in the face, scratching
him again with her nails. She called him a “piece of trash” and insisted that he wants to cheat on her “with that whore,”
which he denied.
When she is calmer, you take some further history from her. She tells you, “I was severely sexually abused as a small child.
But I didn’t tell anyone until I was a teenager. I started having sex at the age of 15. I used cocaine. I smoked. I really didn’t
care. I was extremely self-abusive and it got to the point where I wanted to kill myself to rid myself of the anger, the hurt, the
pain, the confusion.” She admits that she has made prior suicide attempts. She tells you, “When I was 17, I looked forward
to getting my driver’s license so I could run the car into a support column on the highway, or into a semi-truck. I drove very
recklessly; I didn’t want it to be an obvious suicide.” She informs you that when she was 18, she had a very severe car acci-
dent and ended up in the intensive care unit.

1. 
Which one of the following is Susan’s most likely diagnosis?
A. 
Major depressive disorder
B. 
Borderline personality disorder
C. 
Bipolar I disorder
D. 
Histrionic personality disorder
E. 
Social anxiety disorder

2. 
Which of the following are criteria for Susan’s diagnosis? (Choose four of five)
A. 
Chronic feelings of emptiness
B. 
Intense episodic dysphoria
C. 
Grandiosity
D. 
Severe dissociative symptoms
E. 
Transient stress-related paranoia

3. 
Susan’s suicide attempt could best be attributed to which one of the following?
A. 
Severe depressed mood
B. 
Overwhelming anxiety
C. 
Perceived rejection
D. 
Grandiose self-importance

4. 
What is the treatment of choice for Susan’s condition?
A. 
Psychoanalysis
B. 
Family therapy
C. 
Dialectical behavior therapy
D. 
Supportive psychotherapy

397

Psychiatry Test Preparation and Review


Manual E-Book
Psychiatry Test Preparation & Review Manual

5. 
Which of the following choices apply to patients with Susan’s condition? (Choose three of four)
A. 
They are in touch with reality only on a basic level
B. 
They have limited capacity for insight
C. 
They use many primitive defenses
D. 
They have an integrated sense of self

6. 
Which of the following defense mechanisms is Susan most likely to use based on her diagnosis? (Choose two of five)
A. 
Suppression
B. 
Sublimation
C. 
Humor
D. 
Acting out
E. 
Splitting

7. 
Which of the following are legitimate reasons borderline patients commit acts of self-mutilation? (Choose three of four)
A. 
To obtain social isolation
B. 
To express anger
C. 
To elicit help from others
D. 
To numb themselves to overwhelming affect

8. 
Susan takes intolerable aspects of herself and exports them onto her boyfriend, leading him over time to accept and play
that role. This phenomenon is known as which one of the following?
A. 
Displacement
B. 
Rationalization
C. 
Splitting
D. 
Projective identification

9. 
Which of the following are not considered part of Susan’s disorder? (Choose three of four)
A. 
Prolonged psychotic episodes
B. 
Marked peculiarity of thinking
C. 
Extreme suspiciousness
D. 
Impulsive behaviors

10. Which of the following medications may play a role in treating Susan’s condition? (Choose four of five)
A. 
Antipsychotics
B. 
SSRIs
C. 
Anticonvulsants
D. 
Monoamine oxidase inhibitors (MAOIs)
E. 
Stimulants

Vignette Five
Cathy Allen comes to Dr. Rupert Smith’s office for an initial appointment. Dr. Smith is a psychiatrist who comes highly rec-
ommended. With Cathy is her husband, Bob. Dr. Smith meets them in the waiting area. Cathy introduces herself and asks if
her husband Bob can come in to the appointment with her.

398

Psychiatry Test Preparation and Review


Manual E-Book
Vignettes

1. 
The most therapeutically appropriate response to Cathy’s request would be:
A. 
“No. I want to meet with only you, because you’re the patient.”
B. 
“Nice to meet you, Cathy. I’m Dr. Smith. Of course your husband can come in if you want him to.”
C. 
“I’m Dr. Smith. Cathy, you come in.”
D. 
“Sure, your husband can come in.”

2. 
After this exchange, Dr. Smith is now about to start the interview. Which of the following would be the most appropriate
way to begin? (Choose two of four)
A. 
“Tell me about the problems you’ve been having.”
B. 
“You look depressed to me. What’s going on?”
C. 
“Where would you like to begin?”
D. 
“You’re very thin. Is this your normal weight? How is your appetite?”

Cathy gives a short sentence or two in response to Dr. Smith’s initial question. She then sits silently and says no more. He
tries to get her to speak more but is unsuccessful. Her body language indicates that she is anxious and uncomfortable. She
gives very little information in response to follow-up questions.

3. 
How should Dr. Smith proceed?
A. 
“You obviously don’t want to be here. Maybe we should stop the interview.”
B. 
“Bob, if she doesn’t tell me what’s wrong, I can’t help her.”
C. 
“Do you have any pets? Tell me about them.”
D. 
“I can’t help but notice that you’re uncomfortable talking to me. Is there anything I could do to make you more
comfortable?”

4. 
After Dr. Smith’s intervention, Cathy opens up and tells him more about the problems she’s been having. Being an astute
psychiatrist, Dr. Smith pays attention to both the content and the process of the interview. Which of the following would
be considered process? (Choose three of four)
A. 
Cathy nervously tears a piece of paper into pieces
B. 
Cathy describes poor sleep for the past 2 weeks
C. 
Cathy changes the subject whenever the topic of her job comes up
D. 
Cathy’s body becomes tense and rigid while discussing her work

5. 
The interview moves forward, and Cathy describes some feelings of depression she’s been having. Dr. Smith says, “You say
you haven’t been sleeping. How many hours per night are you getting?” This question is an example of:
A. 
Confrontation
B. 
Facilitation
C. 
Clarification
D. 
Explanation

6. 
After discussing current symptoms of Cathy’s illness, Dr. Smith says, “I think I understand your current symptoms pretty
well. Now let’s talk about your medical history.” This comment is an example of:
A. 
Reassurance
B. 
Transition
C. 
Positive reinforcement
D. 
Advice

399

Psychiatry Test Preparation and Review


Manual E-Book
Psychiatry Test Preparation & Review Manual

7. 
After an hour-long interview, Dr. Smith seeks to wrap up. Which of the following are important steps he should keep in
mind while concluding the session? (Choose four of five)
A. 
Give Cathy a chance to ask questions
B. 
Thank the patient for sharing information
C. 
Review any prescriptions to make sure the patient understands why she is to take them and how to take them
D. 
Be clear about what the next step in the treatment will be
E. 
Discourage Cathy from calling with questions between sessions

8. 
Which of the following are essential elements for Dr. Smith to develop rapport with Cathy during the interview? (Choose
four of five)
A. 
Putting the patient at ease
B. 
Expressing compassion for pain
C. 
Showing expertise
D. 
Establishing authority as a physician
E. 
Know the answer to almost every question the patient asks

9. 
Which of the following variables are proven to be associated with decreased rates of patient compliance with treatment?
(Choose two of four)
A. 
Intelligence
B. 
Increased complexity of treatment regimen
C. 
Increased number of behavioral changes
D. 
Socioeconomic status

10. In which one of the following models of the doctor–patient relationship does the physician behave in a paternalistic
fashion?
A. 
Active–passive model
B. 
Teacher–student model
C. 
Mutual participation model
D. 
Friendship model

Vignette Six
You are asked to see a patient at your outpatient clinic. Pearl Probst comes to see you on a Monday morning. You quickly
realize that Pearl was not born a woman. She is a preoperative transgender woman of 24 years of age. She lives alone in an
apartment in the city where you work. Pearl tells you that she has felt like a woman trapped in a male body since her preteen
years. She began dressing as a woman in college and has begun the preoperative transition from male to female by taking
female sex hormones. She plans on following this up with gender reassignment surgery at some point in the future.
Pearl tells you that she and her girlfriend engage in sexual acts involving bondage, inflicting pain on each other, and step-
ping and spitting on each other. She asks you if you have any concerns about this behavior. She also reveals that she and her
partner enjoy taking showers together and urinating on each other. You ask her if it has been problematic for her, and she
states that she has had to end several relationships in the past because of it. You ask Pearl what her sexual orientation is, and
she states, “I am a gay woman of course!”

400

Psychiatry Test Preparation and Review


Manual E-Book
Vignettes

1. 
Pearl probably meets criteria for which of the following Diagnostic and Statistical Manual of Mental Disorders (DSM 5)
disorders? (Pick three of six)
A. 
Transvestic fetishism
B. 
Gender dysphoria
C. 
Urophilia (other specified paraphilic disorder)
D. 
Partialism
E. 
Sexual masochism disorder
F. 
Fetishistic disorder

2. 
Which of the following is not a poor prognostic factor in the paraphilias?
A. 
Onset of symptoms in middle age
B. 
Frequent recurrent acts
C. 
Concomitant substance abuse
D. 
Lack of guilt or shame about the acts
E. 
The act of intercourse does not occur with the paraphilia

3. 
Which of the following are good prognostic factors in the treatment dynamic of paraphilias? (Pick three of six)
A. 
Substance abuse
B. 
Successful relationships and adult attachments
C. 
Normal intelligence
D. 
The presence of multiple paraphilias
E. 
The presence of concomitant Axis I mental disorders
F. 
The absence of a personality disorder

4. 
Which of the following are not typically interventions that are used to treat paraphilias? (Pick two of six)
A. 
Prison
B. 
Insight-oriented psychotherapy
C. 
Cognitive behavioral therapy
D. 
Interpersonal psychotherapy
E. 
Twelve-step programs
F. 
Antiandrogen therapy

5. 
Those who identify with a gender different from their assigned gender are most accurately referred to as:
A. 
Genderqueer
B. 
Cross-dressers
C. 
Transsexuals
D. 
Transvestic disorder
E. 
Transgender

6. 
Which of the following is not true concerning hormone therapy for gender reassignment?
A. 
Gender reassignment surgery is more common than hormone therapy for transgender individuals
B. 
Hormone therapy will often leave the patient permanently sterile
C. 
Patients on testosterone need regular monitoring of liver function tests and CBC
D. 
Transgender women may take estrogen, progesterone, or testosterone blockers
E. 
Financial considerations are a significant impediment to many patients obtaining gender reassignment surgery

401

Psychiatry Test Preparation and Review


Manual E-Book
Psychiatry Test Preparation & Review Manual

Vignette Seven
You are a forensic psychiatrist working in private practice. You are faced with the evaluation of a fellow psychiatrist, Dr.
Dean Daniels, who is alleged to have had sexual relations with a former patient of his, Selena Victor. It is also alleged that Dr.
Daniels hired Selena to sell his house during their relationship, because she is a real estate agent.
Dr. Daniels has already been arrested and charged, and he is now out of jail on a $1 million bond posted by his high-pro-
file attorney, L. Lloyd Wolff, Esq. As per his lawyer, he is charged with one count of rape and two counts of sexual assault.
His lawyer informs you that Dr. Daniels has a history of depression and alcoholism and has been hospitalized psychiatrically
in the past. Dr. Daniels has never had a malpractice case brought against him, and his medical license has never been sanc-
tioned in any way. Dr. Daniels is now back in his office practicing as usual until his first court date comes up next month.

1. 
The American Psychiatric Association believes that sexual relations between a psychiatrist and their patient is: (Pick one of
five)
A. 
Permissible if the patient is no longer in active treatment with the psychiatrist
B. 
Permissible if a period of 10 years has passed since the patient has been in active treatment with the psychiatrist
C. 
Permissible when the patient signs a legal document stipulating that the relationship is consensual
D. 
Within the scope of the psychiatrist’s malpractice policy, should the patient sue the psychiatrist after the relationship
ends
E. 
Prohibited in all circumstances regardless of the details

2. 
The legal components that will dictate if Dr. Daniels can or should be declared not guilty by reason of insanity by a jury
are: (Pick three of six)
A. 
Duty to warn and protect
B. 
Mens rea
C. 
Competency to stand trial
D. 
Actus reus
E. 
The matter of Ford v Wainwright
F. 
M’Naghten rule

3. 
For you to declare him competent to stand trial, you must find Dr. Daniels able to: (Pick three of six)
A. 
Take the stand on trial in his own defense under the guidance of his attorney
B. 
Recognize and identify the persons involved in his case
C. 
Recall the various events surrounding the alleged crimes with accuracy
D. 
Collaborate with his attorney with a reasonable degree of rational understanding
E. 
Understand the charges that are being brought against him
F. 
State whether he would prefer a psychiatric plea or a regular plea of guilty versus not guilty

4. 
Which of the following is correct concerning Dr. Daniels hiring Selena to sell his house? (Pick two of five)
A. 
There is no ethical issue with his hiring her
B. 
He has violated American Psychiatric Association ethics guidelines
C. 
His hiring Selena is considered a form of exploitation
D. 
Because they are in a romantic relationship, standard ethics rules no longer apply
E. 
It would be permissible for Selena to barter professional real estate services for psychiatric services

402

Psychiatry Test Preparation and Review


Manual E-Book
Vignettes

5. 
In a case such as with Dr. Daniels, what would be the possible sanctions if he were to go to trial and be found “not guilty
by reason of insanity” by the jury on all three charges? (Pick three of six)
A. 
He could continue to practice psychiatry as before
B. 
He could lose his medical license and be remanded to outpatient treatment by the court
C. 
He could be allowed to retain his medical license and be remanded by the court to an intensive outpatient psychiatric
day program for treatment
D. 
He could eventually practice psychiatry again after completion of appropriate treatment of his disorder, based on man-
dated future psychiatric evaluation
E. 
His name would be inscribed on a computer-based list of sex offenders if his state maintains such a list
F. 
He could continue to practice psychiatry as before, but not with female patients

Vignette Eight
Steven Geller is a 30-year-old male with paranoid schizophrenia. He stopped his medications 3 weeks ago. In response to the
voices that followed, he then stopped eating and drinking. The voices have been telling him that his food is poisoned. “They
put rat poison in your food,” the voices told him. His mother became concerned when he wouldn’t eat for 2 days. She made
him several of his favorite foods and tried to convince him to eat, but he barricaded himself in his room and would not come
out for 24 hours. His mother then called emergency medical services. You interview him in the emergency room after he is
brought into the hospital.

1. 
Which of the following statements would you include in Steven’s mental status examination, given what you know at this
point?
A. 
Thought content includes paranoid delusions
B. 
Thought content includes auditory hallucinations
C. 
Thought process includes flight of ideas
D. 
Perceptions include auditory hallucinations

On further interview, Steven screams at you that he is “God’s chosen one” and further states that people are trying to
poison him to prevent him from revealing his identity to the world. He states that he will be “raised into the heavens on
clouds.” You question the veracity of this statement, and he insists that it is true. He does not believe that it is a creation of
his mind and is certain that those who doubt him are wrong. “I will burn the disbelievers,” he tells you.

2. 
Where should this new information be included in the mental status examination?
A. 
Thought content
B. 
Perceptions
C. 
Attitude
D. 
Thought process

3. 
During your examination, you notice that Steven is malodorous and is wearing dirty clothes. You note a rancid odor in the
room and observe brown streaks on his pant legs. On closer inspection it appears to be feces. His family verifies that he
has been failing to maintain hygiene since stopping his medication. When asked about this, he tells you, “I’m gonna drop
to the kimnet and fly with the glaspin.” His response is best characterized as:
A. 
Logorrhea
B. 
Glossolalia
C. 
Laconic speech
D. 
Verbigeration

You begin to perform a mini mental status examination on Steven. You ask him to count backward from 100 by 7’s. He
replies “93, 89, 81, 74.” You ask him if he can go further and he replies, “Go to hell. I am the chosen one. I won’t do any-
thing I don’t want to.”

403

Psychiatry Test Preparation and Review


Manual E-Book
Psychiatry Test Preparation & Review Manual

4. 
How would you document this exchange? (Choose two of four)
A. 
Memory is intact
B. 
Concentration is impaired
C. 
Abstract thinking is intact
D. 
Attitude is hostile

5. 
Steven’s belief about being the chosen one would best be described as:
A. 
Pseudologia phantastica
B. 
Delusion of grandeur
C. 
Algophobia
D. 
Nihilistic delusion

6. 
The mesocortical pathway, which is responsible for the __________ symptoms of schizophrenia, begins at the
_____________.
A. 
Positive; ventral tegmental area
B. 
Negative; nucleus accumbens
C. 
Positive; nucleus accumbens
D. 
Negative; ventral tegmental area

7. 
Which of the following are correct concerning schizophrenia? (Choose three of four)
A. 
Positive symptoms are associated with frequency of hospitalization
B. 
Cognitive symptoms are directly related to long-term functional outcome
C. 
Positive symptoms are directly related to long-term functional outcome
D. 
Schizophrenia is associated with a 10% suicide rate.

8. 
Which of the following would not be consistent with a diagnosis of schizophrenia? (Choose three of four)
A. 
Increased prevalence of diabetes
B. 
Symptoms present for 2 months
C. 
Pressured speech, racing thoughts, excessive energy
D. 
Phencyclidine use during symptom onset

9. 
Which of the following are common aspects of appearance for schizophrenic patients? (Choose four of five)
A. 
Lack of spontaneous movement
B. 
Echopraxia
C. 
Agitation
D. 
Bizarre posture
E. 
Bright clothing

10. Which of the following are often found as part of thought form for schizophrenic patients? (Choose three of four)
A. 
Verbigeration
B. 
Ideas of reference
C. 
Word salad
D. 
Mutism

404

Psychiatry Test Preparation and Review


Manual E-Book
Vignettes

Vignette Nine
Judy Albanese, a local college student, is brought into the emergency room when her roommate called EMS after she col-
lapsed at the gym. She appears malnourished and emaciated. Her roommate told EMS that she hadn’t been eating recently.
She had cut down to one meal per day to lose weight. Yesterday the only thing she ate all day was a cereal bar. She has been
spending 3 hours each day at the gym after classes in an effort to lose weight. Despite being emaciated, she believes that she
is overweight. She recently told her roommate, “I’m so gross! I don’t know how anyone stands to look at me. All the skinny
girls get the boyfriends, the attention, and what do I get?” When you ask her more questions, she admits to you, “I feel cold
all the time. I have terrible headaches, and when I shower, big clumps of hair fall out of my head.” She goes on to tell you,
“During class, instead of listening to lectures or taking notes, I think about what I have eaten that day, when I will eat again,
what I will eat. I like to bake and bring the treats to school the next day, to give to my friends. I watch them eat. I’m really
jealous of them when they eat. I read cookbooks for fun and have collected hundreds of recipes. I never look in the mirror
without thinking ‘Fat.’”

1. 
Which of the following factors would you consider essential to make a diagnosis of anorexia nervosa? (Choose three of
four)
A. 
Body weight less than minimally normal for height and age
B. 
A disturbance in how body weight is experienced
C. 
Amenorrhea
D. 
Binge eating and purging behavior

2. 
Based on diagnostic criteria, you determine that Judy has anorexia. Which of the following medical complications are
likely to be associated with the diagnosis? (Choose five of six)
A. 
Bradycardia
B. 
Pancytopenia
C. 
Lanugo
D. 
Osteopenia
E. 
Metabolic encephalopathy
F. 
Ulcerative colitis

3. 
Which of the following would be considered indications that Judy should be admitted to a hospital? (Choose three of
four)
A. 
Significant hypokalemia
B. 
Body mass index (BMI) less than 15 kg/m2
C. 
Growth arrest
D. 
Osteopenia

4. 
As part of your evaluation of Judy you wish to calculate her BMI. How do you do that?
A. 
100 lb for the first 5 feet in height + 5 lb/inch over 5 feet ± 10%
B. 
Height (m2)/weight (kg)
C. 
(Age [years] × 0.375 + height [m]) × 0.093/0.09 (daily caloric intake [kcal])
D. 
Weight (kg)/height (m)

405

Psychiatry Test Preparation and Review


Manual E-Book
Psychiatry Test Preparation & Review Manual

5. 
You consider treatment options for Judy. Which of the following have proven efficacy in patients with anorexia? (Choose
four of five)
A. 
Cognitive behavioral therapy
B. 
Family therapy
C. 
Fluoxetine
D. 
Olanzapine
E. 
Bupropion

6. 
Which of the following are possible complications of self-induced vomiting? (Choose three of four)
A. 
Russell’s sign
B. 
Mallory–Weiss syndrome
C. 
Spontaneous abortion
D. 
Atonic colon

7. 
Which of the following are possible complications of ipecac abuse? (Choose three of four)
A. 
Skeletal muscle atrophy
B. 
Rectal prolapse
C. 
Cardiomyopathy
D. 
Prolonged QTc interval

8. 
Which of the following statements are correct concerning anorexia? (Choose two of four)
A. 
Risk of anorexia increases when family members have anorexia
B. 
Patients with anorexia often demonstrate traits of paranoid personality disorder
C. 
Patients with anorexia are characterized by emotional flexibility
D. 
Adolescence is a time of increased risk for anorexia

9. 
Which of the following should be included in the differential diagnosis for anorexia? (Choose four of five)
A. 
Major depressive disorder
B. 
Anxiety disorders
C. 
Bulimia nervosa
D. 
Substance abuse
E. 
Brief psychotic disorder

10. Medical treatment for anorexia should include which of the following? (Choose three of four)
A. 
Combination estrogen and progesterone
B. 
Dental follow-up
C. 
Electrocardiogram
D. 
Correction of hypokalemia

406

Psychiatry Test Preparation and Review


Manual E-Book
Vignettes

Vignette Ten
Lisa is a 22-year-old barista at a local coffee shop who comes to your office seeking help after feeling that she did not get any
better with her primary care physician. She gives a long history of anxiety around other people dating back to childhood.
At one point, while in high school, her mother pressured her to become a camp counselor to “overcome shyness.” Lisa was
able to force herself to do it for a few weeks but then became overwhelmed by the anxiety and quit. She also went through
a period of time during her school years when she wouldn’t use public restrooms or would use them only if they were com-
pletely empty. She got into trouble for leaving class to go to the restroom all of the time. When the restroom was empty dur-
ing classes, she felt the most comfortable using it.
Now she reports being very anxious at work and at parties. She snuck out of the holiday party for her job because she
was so uncomfortable. She worries that other people are judging her and won’t like her. She says that she feels stupid inter-
acting with others, especially at work. She had quit a previous job because there were weekly meetings, which she had to
attend and during which she had to speak in front of 30 people. Her anxiety about these meetings led her to quit the job.
When you ask about her personal life, she tells you, “I’ve gone on dates once or twice but have never had any long-term rela-
tionships. Dates are excruciating for me. Making conversation with new people makes me so uncomfortable and anxious.”
Lisa’s primary care physician had tried her on sertraline in the past. She comes to you to see if there is anything else you
can offer her.

1. 
Which of the following should be included in Lisa’s differential diagnosis? (Choose three of four)
A. 
Panic disorder
B. 
Schizoaffective disorder
C. 
Social phobia
D. 
Generalized anxiety disorder

2. 
Given Lisa’s medication history, which other medications may be worth trying? (Choose three of four)
A. 
Paroxetine
B. 
Clonazepam
C. 
Citalopram
D. 
Bupropion

3. 
Which of the following has the best evidence to support its use for Lisa’s condition?
A. 
Cognitive behavioral therapy
B. 
Supportive psychotherapy
C. 
Motivational interviewing
D. 
Psychodynamic psychotherapy

4. 
Lisa is most likely to be misdiagnosed with which of the following? (Choose two of four)
A. 
Schizoid personality disorder
B. 
Avoidant personality disorder
C. 
Schizotypal personality disorder
D. 
Dependent personality disorder

5. 
Which diagnosis best explains Lisa’s avoidance of public restrooms during her school years?
A. 
Specific phobia
B. 
Panic disorder
C. 
Social phobia
D. 
Agoraphobia

407

Psychiatry Test Preparation and Review


Manual E-Book
Psychiatry Test Preparation & Review Manual

6. 
The performance-only subtype of social phobia is most successfully treated by which one of the following?
A. 
Benztropine
B. 
Olanzapine
C. 
Propranolol
D. 
Lorazepam

7. 
The major concern of patients with social phobia is which one of the following?
A. 
Avoidance of relationships
B. 
The need for someone to be with them in stressful situations
C. 
Fear of rejection
D. 
Fear of embarrassment

8. 
How long does Lisa need to have symptoms to meet DSM 5 criteria for social phobia?
A. 
2 weeks
B. 
2 months
C. 
6 weeks
D. 
6 months

9. 
Which of the following are common side effects of Lisa’s condition? (Choose three of four)
A. 
Blushing
B. 
Dry mouth
C. 
Sweating
D. 
Fear of dying

10. As many as one-third of patients with Lisa’s condition also meet criteria for which one of the following disorders?
A. 
Major depressive disorder
B. 
Agoraphobia
C. 
Cocaine abuse
D. 
Body dysmorphic disorder

Vignette Eleven
Carl Freeman is an obese 59-year-old male who is referred to you by his primary care physician for complaints of depres-
sion. Carl lives with his girlfriend Heidi Schmitz and her three children, Blair, Denny, and Rao. He works as a customer ser-
vice representative at a health insurance company. He tells you that, “My coworkers resent me because I keep falling asleep
at my desk during the day. I’ve even fallen asleep in the middle of phone calls with customers.” Because he has had difficulty
at work, he was referred for a medical evaluation. He reports decreased energy, fatigue, and poor sleep. He states that he
had difficulty concentrating at work. He tells you, “I’ve been irritable and fatigued. I’m having terrible headaches. I’ve been
gaining weight recently and I can’t concentrate. Basically everything is going wrong right now.” He tried to sleep more at
night, but this did not make him feel any better. He tried taking naps in his car during his lunch hour, but this didn’t help.
His primary care physician felt that he was depressed and referred him to you. When you interview him on his sleep habits,
he reports that his wife stopped sleeping in the same room as him because of his snoring. He tells you that she calls him a
“water buffalo” because of the noises he makes while he sleeps.

408

Psychiatry Test Preparation and Review


Manual E-Book
Vignettes

1. 
Which of the following should be included in the differential diagnosis for Carl? (Choose two of four)
A. 
Major depressive disorder
B. 
Sleep apnea
C. 
Klein–Levin syndrome
D. 
Narcolepsy

2. 
Which of the following would you include in a workup for this patient? (Choose two of four)
A. 
Thyroid-stimulating hormone
B. 
Periodic limb movements of sleep test
C. 
Continuous positive airway pressure (CPAP)
D. 
Nocturnal polysomnography

3. 
Which of the following are possible complications of Carl’s condition? (Choose three of four)
A. 
Increased risk of cardiovascular complications
B. 
Decreased mood
C. 
Increased neck girth
D. 
Decreased cognition

4. 
Which one of the following statements is correct concerning Carl’s condition?
A. 
Carl has a parasomnia
B. 
Carl has a dyssomnia
C. 
Modafinil would be considered first-line treatment for Carl’s condition
D. 
Carl’s condition places him at increased risk for Parkinson’s disease

5. 
Which one of the following is Carl most likely to be misdiagnosed with?
A. 
Pavor nocturnus
B. 
Somnambulism
C. 
Jactatio capitis nocturna
D. 
Gastroesophageal reflux

6. 
Which of the following choices are true concerning obstructive sleep apnea? (Choose two of four)
A. 
Airflow ceases during apneic episodes
B. 
Respiratory effort decreases during apneic episodes
C. 
Patients need at least three apneic episodes per hour to meet criteria
D. 
Respiratory effort increases during apneic episodes

7. 
Which of the following complications are common with obstructive sleep apnea? (Choose three of four)
A. 
Arrhythmias
B. 
Changes in blood pressure during apneic episodes
C. 
Pulmonary hypotension
D. 
Chronic increase in systemic blood pressure

8. 
Which of the following are true concerning rapid eye movement (REM) sleep behavior disorder? (Choose two of four)
A. 
It occurs primarily in females
B. 
Loss of atonia during REM is a major component
C. 
Violent behavior can be a complication
D. 
Symptoms improve after treatment with stimulants or fluoxetine

409

Psychiatry Test Preparation and Review


Manual E-Book
Psychiatry Test Preparation & Review Manual

9. 
Which of the following are symptoms of sleep-related gastroesophageal reflux? (Choose three of four)
A. 
Awakening from sleep
B. 
Cough
C. 
Chest tightness
D. 
Cessation of airflow

10. Which of the following would be considered sleep hygiene measures? (Choose three of four)
A. 
Avoid daytime naps
B. 
Exercise during the day
C. 
Use of zolpidem
D. 
Arise at the same time each morning

Vignette Twelve
Ryan Huang is a 35-year-old male who is unemployed and lives with his mother. He comes to your clinic with complaint of
voices telling him to kill his mother because “She is the fiend. She is the devil.” He reports sleeping in his car for the past few
days because he is trying to stay away from his mother so that he doesn’t hurt her. “I don’t want to go to jail,” he tells you.
Ryan has a history of violence toward his mother. When he was in his 20s, she took out an order of protection against
him after an incident in which he choked her in response to command auditory hallucinations. When the order of protection
expired, she did not renew it. In subsequent years he began to do better. Eventually she allowed him to move back into the
home.
Ryan reported that the voices began when he was 16 years of age. He doesn’t like them because they tell him to harm
others as well as himself. During his first psychiatric hospitalization, he developed oculogyric crisis from multiple PRN (as
needed) Haldol injections. Since this experience, he has demonstrated an unwillingness to maintain compliance with medica-
tions. He is currently prescribed ziprasidone but has only intermittent compliance.
Ryan’s mother also tells you that he has a significant alcohol problem. She says he drinks daily. Ryan himself admits to
periods of “the shakes” and previous blackouts. His drinking tends to get worse at certain points. He will go on binges and
drink excessively for up to a week at a time.
Ryan’s trauma history includes being raped repeatedly by an older male cousin between the ages of 13 and 16. He admits
that this has had an effect on him but has difficulty explaining how. He denies current flashbacks.
Medical history is significant for smoking one pack per day of cigarettes, hypertension, high cholesterol, and poorly con-
trolled diabetes. He endorses a head injury, which happened when he was 13. When he tried to resist a rape attempt by his
cousin, his cousin beat him until he was unconscious. On examination he recalls one of three objects after 5 minutes. He is
oriented to person, place, and time.

1. 
Which of the following should be considered in Ryan’s differential diagnosis? (Choose four of five)
A. 
Schizophrenia
B. 
Substance-induced psychotic disorder
C. 
PTSD
D. 
Generalized anxiety disorder
E. 
Dementia not otherwise specified (NOS)

2. 
Which of the following would you include in a medical workup for Ryan? (Choose five of six)
A. 
Thyroid function tests
B. 
Thiamine level
C. 
Head CT
D. 
Electrocardiogram (ECG)
E. 
Urine toxicology
F. 
Prolactin level

410

Psychiatry Test Preparation and Review


Manual E-Book
Vignettes

3. 
Which of the following are the most likely side effects Ryan will experience from treatment with ziprasidone? (Choose
two of four)
A. 
Weight gain
B. 
Extrapyramidal symptoms
C. 
Sedation
D. 
Cardiac effects

4. 
Ryan’s history includes significant substance abuse. Heavy use of which one of the following drugs before the age of 16
has been correlated with an increased relative risk of schizophrenia?
A. 
Phencyclidine
B. 
Alcohol
C. 
Lysergic acid diethylamide
D. 
Cannabis

5. 
If a patient presents with psychosis for more than 1 month but does not meet criterion A for schizophrenia, what diagno-
ses are possible? (Choose two of four)
A. 
Schizoaffective disorder
B. 
Delusional disorder
C. 
Schizotypal disorder
D. 
Unspecified schizophrenia spectrum and other psychotic disorders

6. 
Which one of the following medications is not an antagonist at the 5-HT 2A receptor?
A. 
Haloperidol
B. 
Aripiprazole
C. 
Olanzapine
D. 
Ziprasidone

7. 
Which of the following choices are true concerning delusional disorder? (Choose two of four)
A. 
Auditory hallucinations may be present
B. 
Memory impairment may be seen
C. 
Tactile hallucinations may be present
D. 
Unnecessary medical interventions may be part of the picture

8. 
Which one of the following is correct concerning brief psychotic disorder?
A. 
The patient will not return to normal functioning
B. 
Primary preventative measures can involve treatment with low-dose risperidone
C. 
Hallucinations may be present, but delusions are not
D. 
Symptoms last between 1 day and 1 month

9. 
Which of the following are true concerning psychotic symptoms? (Choose four of five)
A. 
Tactile hallucinations are more common in medical and neurologic conditions than in schizophrenia
B. 
Illusions are sensory misperceptions of actual stimuli
C. 
Delusions are fixed false beliefs that are not supported by cultural norms
D. 
Word salad is the violation of basic rules of grammar seen in severe thought disorder
E. 
Cataplexy is synonymous with waxy flexibility

411

Psychiatry Test Preparation and Review


Manual E-Book
Psychiatry Test Preparation & Review Manual

10. A differential diagnosis for new-onset psychosis may include which of the following medical conditions? (Choose four of
five)
A. 
Systemic lupus erythematosus
B. 
Temporal lobe epilepsy
C. 
Neurosyphilis
D. 
Wilson’s disease
E. 
Phymatous rosacea

Vignette Thirteen
John Jameson, a 42-year-old man, is brought by ambulance, accompanied by police, to your emergency room at 2:00 am.
Upon arrival, he is agitated, and the police and emergency medical technicians cannot manage his aggression. He refuses to
answer your questions and gives you a verbal tongue-lashing when you try to approach him. Nobody else accompanies him,
and there are no collaterals present from whom you can obtain information. You discover he has never been to your hospital
before, because there is no medical record at your facility in his name. Mental status examination is impossible at this time,
as he is completely uncooperative with you. He shouts obscenities at you and yells out that his mother should be shot for
what she has done to him.

1. 
What are your very next steps in management with respect to this patient? (Pick two of six)
A. 
Draw blood for basic labs and obtain a CT scan of the head to rule out organic causes for his agitation
B. 
Ask the police and emergency technicians why they brought him
C. 
Admit him to the psychiatry unit on an involuntary basis
D. 
Do your best to obtain his mother’s contact information and call her as soon as possible to find out what “she has
done to him”
E. 
Have him restrained by police and/or hospital security so you can administer him an intramuscular injection of halo-
peridol and lorazepam to help calm him
F. 
Obtain medical consultation and clearance from the emergency room physician

2. 
Which of the following is not a predictor of dangerousness to others in violent patients, such as in the patient scenario
depicted in this vignette? (Pick two of six)
A. 
Prior violent acts
B. 
Chronic anger, hostility, or resentment
C. 
Female gender
D. 
Numerous medical problems
E. 
Childhood brutality or deprivation
F. 
Access to weapons or instruments of violence

3. 
When attempting to interview this violent and agitated patient, the psychiatrist should do which of the following? (Pick
three of six)
A. 
Conduct the interview in a quiet, nonstimulating area of the emergency room
B. 
Avoid asking the patient if he has weapons on him to avoid further anger and agitation
C. 
Request that security personnel give their assistance during the interview, if needed
D. 
Avoid any behavior that could be misconstrued by the patient as menacing, such as standing over the patient
E. 
Explain to the patient that any refusal to answer questions will result in him being medicated and admitted to the hos-
pital over his objections
F. 
Interview the patient in an enclosed locked room to prevent the patient from fleeing

412

Psychiatry Test Preparation and Review


Manual E-Book
Vignettes

4. 
Once the patient is sedated, the emergency room medical physician completes a workup on the patient. The workup,
including drug screen and head CT, is negative, and the patient is cleared from a medical and surgical perspective. If you
are unable to obtain any further history on this patient, your disposition for him should be to: (Pick three of six)
A. 
Discharge him home with outpatient psychiatric follow-up
B. 
Admit him to the psychiatric inpatient unit on an involuntary basis
C. 
Start him on aripiprazole and divalproex sodium in the emergency room
D. 
Contact police to return to the emergency room to arrest the patient
E. 
Obtain a social work consultation after holding him overnight in the emergency room pending further information
F. 
Attempt to call his mother and tell her about his feelings toward her

5. 
Once admitted to the psychiatric unit, it is discovered that the patient has a lengthy history of schizophrenia, paranoid
type, since 17 years of age and has been hospitalized in this fashion no less than 25 times since the onset of his illness
began. These recurrent hospitalizations have mostly been due to his refusal to take medications upon discharge. Currently,
he is refusing to take medication on the unit when it is offered to him by nursing staff. Which of the following would cur-
rently be good choices of medication for this patient, on the inpatient unit? (Pick two of six)
A. 
Risperidone
B. 
Olanzapine
C. 
Asenapine
D. 
Ziprasidone
E. 
Quetiapine
F. 
Lurasidone

6. 
If the patient is given risperidone (Risperdal Consta) biweekly intramuscular injections and is well stabilized in the hospi-
tal on this agent, which of the following would be the best discharge disposition for him for ongoing treatment and care
once he is ready to leave the acute inpatient psychiatry unit? (Choose three of seven)
A. 
State psychiatric inpatient facility (long-term admission)
B. 
ACT team home visits
C. 
Partial hospital program
D. 
Continuing day treatment program
E. 
Outpatient mental health department of a university/teaching hospital
F. 
Outpatient freestanding mental health clinic
G. 
Mental health practitioner in the patient’s primary care physician’s group practice

Vignette Fourteen
Robert Bradbury is a 30-year-old male with a history of chronic paranoid schizophrenia who is being treated with clozapine.
He goes to an outpatient psychiatry appointment and has the following discussion with his psychiatrist.
Doctor: How are things going, Robert?
Robert: Fine. I’ve been working in the afternoons after my program, and it’s going very well. I’m continuing to drool a lot,
like I told you last time, but its manageable.
Doctor: How are your symptoms? Are you hearing any voices?
Robert: No. I haven’t heard voices in about a year now. I’m really glad about that. (smiles)
Doctor: Good. Good. Tell me about this job you’ve been doing.
Robert: Well, I’m doing a patient work program through the hospital. We move furniture, run errands, deliver mail within the
hospital. Stuff like that.
Doctor: Do you like it?

413

Psychiatry Test Preparation and Review


Manual E-Book
Psychiatry Test Preparation & Review Manual

Robert: I do, but there is this one woman that I work with who is so nasty. (frowns) She talks down to the patient workers
like she’s better than us or as if we’re not as good as other people. It gets me upset sometimes.
Doctor: How do you handle it?
Robert: My boss tells me just to ignore her, that it’s her problem, not mine, and that she’s not worth getting upset over.
Doctor: Are you able to do that?
Robert: Yeah. If she says something nasty, I just walk away. I try not to let it bother me as much as it used to. There are plenty
of people at work who are friendly, so it doesn’t matter.
Doctor: Good. I like your attitude about this. Sounds like you’re handling it well.
Robert: Thanks. Oh, before we finish, I need a refill on my clozapine. I went for blood work 2 days ago.

The next seven questions are regarding Robert’s mental status examination:

1. 
Robert’s attitude is best described as: (Choose two of six)
A. 
Guarded
B. 
Hostile
C. 
Apathetic
D. 
Cooperative
E. 
Friendly
F. 
Ingratiating

2. 
Robert’s affect is best described as:
A. 
Incongruent
B. 
Within normal range
C. 
Constricted
D. 
Blunted
E. 
Flat

3. 
Robert’s thought process is best described as: (Choose two of six)
A. 
Flight of ideas
B. 
Tangentiality
C. 
Circumstantiality
D. 
Linear
E. 
Goal directed
F. 
Thought blocking

4. 
Robert’s impulse control is best described as:
A. 
Good
B. 
Fair
C. 
Poor

5. 
Robert’s insight is best described as:
A. 
Good
B. 
Fair
C. 
Poor

414

Psychiatry Test Preparation and Review


Manual E-Book
Vignettes

6. 
Robert’s judgment is best described as:
A. 
Good
B. 
Fair
C. 
Poor

7. 
Robert’s perceptions are best described as:
A. 
No auditory hallucinations
B. 
No visual hallucinations
C. 
No olfactory hallucinations
D. 
No tactile hallucinations
E. 
No gustatory hallucinations

8. 
What is the best feedback the doctor can give Robert about his excessive drooling?
A. 
It is expected; we should continue to follow it
B. 
It will go away once your dose of clozapine is increased
C. 
It is a clear indication to stop the medication
D. 
It is most likely unrelated to your medications

9. 
If Robert is on clozapine for 8 months, how often should he have his absolute neutrophil count (ANC) done?
A. 
Every month
B. 
Every week
C. 
Every 2 weeks
D. 
Every 2 months

10. In addition to monitoring Robert’s WBC/ANC, which other tests would be appropriate to monitor Robert over time?
(Choose seven of eight)
A. 
ECG
B. 
Liver function tests
C. 
Clozapine level
D. 
Fasting glucose
E. 
Weight
F. 
Waist circumference
G. 
Triglycerides and cholesterol
H. 
Echocardiogram

Vignette Fifteen
A woman 35 years of age presents to the emergency room in a state of acute anxiety and agitation. After administration
of an intramuscular injection of 2 mg of lorazepam, she calms down a bit and is able to give you more of her history. For
the past 10 years, she has been functioning as a bank teller. She lives alone in a studio apartment and is self-sufficient. She
reports that for the past decade she has felt that she is not one person but three different persons. She feels that her self-states
take over her being whenever she is in an extremely stressful situation. When asked about her parents and youth, she closes
her eyes and begins to talk in a more youthful voice stating, “I have to run away. I can’t be home when Papa gets here.” She
seems distant as if in a trance. When she finally comes to her senses, she admits that she was repeatedly beaten and raped
from ages 5 to 11 years by her stepfather. She admits to flashbacks and excessive easiness to startle, and these symptoms per-
sist even now in her. She states that this child-like voice is that of “Melanie,” one of her self-states, who comes out when she
is under stress at work or in her relationships with men. She denies suicidal or homicidal ideation. She denies experiencing
auditory or visual hallucinations, past or present. No delusions or ideas of reference are noted. She is much calmer now in
the emergency room after your intervention with her.

415

Psychiatry Test Preparation and Review


Manual E-Book
Psychiatry Test Preparation & Review Manual

1. 
Which of the following clinical features of this patient’s disorder are correct? (Pick two of six)
A. 
Clinical studies report female-to-male ratios of up to 10 to 1 in diagnosed cases
B. 
Fifteen percent of cases are associated with childhood trauma and maltreatment
C. 
Psychotherapies of choice include dynamic, cognitive, and hypnotherapy
D. 
Studies have shown a strong genetic component to the disorder
E. 
Inability to recall important personal information is not part of this disorder
F. 
About 10% of patients also meet criteria for somatization disorder

2. 
Which of the following would be expected to worsen the prognosis of this patient’s disorder?
A. 
Concomitant diabetes and hypertension
B. 
Concomitant eating disorder
C. 
Recommending clonazepam for anxiety symptoms
D. 
The patient forcing herself to maintain a high level of daily functioning despite having serious symptoms
E. 
Group therapy for patients with the same disorder only
F. 
Past traumatic brain injury from a motor vehicle accident

3. 
Which one of the following is not a recommended pharmacologic choice for this patient’s disorder?
A. 
Quetiapine
B. 
Fluoxetine
C. 
Divalproex sodium
D. 
Lithium carbonate
E. 
Zolpidem

4. 
Which of the following symptoms is not typically seen in this patient’s disorder?
A. 
Seizure-like episodes
B. 
Survivor guilt
C. 
Suicidal thoughts
D. 
Asthma and breathing problems
E. 
Manic episodes

5. 
Which of the following would help you rule out a factitious or malingered disorder in this patient’s case? (Pick two of six)
A. 
Marked inconsistencies in her story and symptom presentation
B. 
The patient prevents you from speaking to collaterals
C. 
Marked dysphoria about her symptoms
D. 
A significant history of legal problems
E. 
Feeling confused and ashamed about her symptoms
F. 
A history of poor work performance by the patient

Vignette Sixteen
Kevin Moran is a 75-year-old man who is brought to your office by his 38-year-old daughter Susan for a consultation. Mr.
Moran has not been himself for at least a year, his daughter states. He lost his wife to cancer 18 months ago, and they were
married for 50 years. Susan tells you that her father cannot live on his own anymore, and she had to take him into her home
where she has a spare bedroom for him. The reason for his inability to live independently is because he gets easily confused,
forgetful, loses his sense of direction, and starts to wander alone in the street with no purpose. The police brought him
home once after they found him wandering in his neighborhood late at night and the poor man couldn’t find his way home.

416

Psychiatry Test Preparation and Review


Manual E-Book
Vignettes

Luckily he was able to remember his own name and his daughter’s name, which helped police trace him back to her home.
Susan says her father cannot really cook or clean for himself because he forgets that he leaves the stove on and burns pots
and pans, which could result in a severe fire hazard. He can eat, but he forgets the names of common household items like
forks and cups, and sometimes even forgets what they are used for.
His medical history is significant for coronary artery disease since age 68, hypertension controlled on medication, type 2
diabetes for which he takes oral medications only, and a small stroke a few years ago for which he has been given aspirin. He
also has high serum cholesterol and elevated serum triglycerides.

1. 
Given his history, the most likely diagnosis is: (Pick two of six)
A. 
Major depressive disorder
B. 
Generalized anxiety disorder
C. 
Vascular dementia
D. 
Diffuse Lewy-body disease
E. 
Alzheimer’s dementia
F. 
Pick’s disease

2. 
What would be your next maneuver with respect to this patient in the outpatient setting? (Pick three of seven)
A. 
Start sertraline 25 mg daily
B. 
Start trazodone 50 mg at bedtime
C. 
Start donepezil 5 mg daily
D. 
Obtain an electroencephalogram
E. 
Start risperidone 0.25 mg at bedtime
F. 
Obtain an outpatient brain MRI
G. 
Obtain neuropsychological consultation

3. 
His daughter is concerned that she cannot manage her father properly in the home. What suggestions can you make to
help her with this situation? (Pick three of six)
A. 
Refer him to an ACT team for ongoing management
B. 
Obtain a visiting-nurse consultation
C. 
Refer her to caregiver support programming and groups
D. 
Convince her to get family members to provide coverage in the home to monitor the patient more closely
E. 
Consult a physiatrist to have the patient placed in a subacute rehabilitation facility
F. 
Seek skilled nursing facility or assisted-living facility placement for the patient

4. 
If the patient has a dementia of the Alzheimer type, what would be his expected prognosis if he were to remain untreated?
A. 
1 to 3 years
B. 
4 to 6 years
C. 
7 to 10 years
D. 
11 to 15 years
E. 
15 to 20 years

417

Psychiatry Test Preparation and Review


Manual E-Book
Psychiatry Test Preparation & Review Manual

5. 
The treatment of choice for a case of dementia believed to have features of both Alzheimer and vascular type would be:
(Pick three of seven)
A. 
An antiplatelet aggregant agent
B. 
An atypical antipsychotic agent
C. 
A sedative–hypnotic anxiolytic agent
D. 
A cholinesterase-inhibiting agent
E. 
Vitamin B complex supplementation
F. 
An antidepressant agent
G. 
An antihypertensive agent

6. 
Which of the following is not typically a complication of this man’s illness?
A. 
Agitation and sundowning
B. 
Personality changes
C. 
Aggression
D. 
Hallucinations and delusions
E. 
Depression
F. 
Mania

7. 
Which of the following drugs should be avoided in this patient? (Pick two of seven)
A. 
Rivastigmine
B. 
Aspirin
C. 
Memantine
D. 
Benztropine
E. 
Diphenhydramine
F. 
Fluoxetine
G. 
Galantamine

Vignette Seventeen
Wanda Reardon is a 55-year-old woman who is hospitalized for acute relapse of a multiple sclerosis (MS) flare-up. She has
had the disease for 25 years, and it has been classified as relapsing and remitting in variety. You are called to her bedside
because she is in an acute confusional state. Upon admission to the hospital yesterday, her neurologist started her on intrave-
nous methylprednisolone and omeprazole for the MS exacerbation. Her history reveals that she is a former cigarette smoker
who quit 10 years ago. She also has a history of hypertension and gastritis. Her medications at home include enalapril, lanso-
prazole, and interferon β-1a (Avonex) weekly injections for her MS.
When you arrive at the bedside to examine Wanda after reviewing her chart, you find that she is unable to attend to you
or your questions. She is talkative, but what she says makes no grammatical or logical sense. Her eyes are rolling back in
her head, and her eyelids are drooping frequently during your interaction with her. She is rocking side to side in her bed
in a hyperactive manner. She is unable to engage you or answer any of your questions appropriately. She is disoriented to
time, place, and person. When you call her name, she is able to look at you briefly, but her attention wanes, and in a brief
moment she looks away and is unable to respond further to you. Her rocking behavior is quite severe, and you fear that she
may fall out of her bed, despite the fact that her bed rails are up.

1. 
Which of the following symptoms are not generally characteristic of Wanda’s present syndrome? (Pick two of six)
A. 
Mood stability
B. 
Irritability
C. 
Sleep–wake cycle disturbance
D. 
Language disturbance
E. 
Gradual onset over weeks to months
F. 
Memory impairment
418

Psychiatry Test Preparation and Review


Manual E-Book
Vignettes

2. 
Which of the following risk factors predispose this patient to the current condition you now find her in? (Pick three of six)
A. 
Smoking history
B. 
Female gender
C. 
Her age
D. 
Her current medications
E. 
Her MS
F. 
Hypertension

3. 
The central nervous system (CNS) area(s) believed to be most closely implicated in this patient’s present condition is (are)
the:
A. 
Cerebellum
B. 
Frontal and parietal lobes
C. 
Midbrain and nigrostriatal pathway
D. 
Reticular formation and dorsal tegmental pathway
E. 
Hippocampus and amygdala

4. 
Which of the following neurotransmitters is probably the least likely to be implicated in the pathophysiology of delirium?
A. 
Norepinephrine
B. 
Dopamine
C. 
Serotonin
D. 
Acetylcholine
E. 
Glutamate

5. 
Which of the following electroencephalography findings would you expect to find in this patient?
A. 
Temporal lobe spikes
B. 
Hypsarrhythmia
C. 
Generalized background slowing
D. 
Triphasic waves
E. 
Periodic lateralizing epileptiform discharges (PLEDs)

6. 
Which of the following agents would not be appropriate treatment for Wanda’s current condition? (Pick two of six)
A. 
Haloperidol
B. 
Risperidone
C. 
Diphenhydramine
D. 
Quetiapine
E. 
Benztropine
F. 
Olanzapine

7. 
Which of the following are true about the course and prognosis of delirium? (Pick three of six)
A. 
Prodromal symptoms can occur months before onset of florid symptoms
B. 
Symptoms usually persist as long as causally relevant factors are present
C. 
Delirium usually progresses to dementia, according to longitudinal studies
D. 
Delirium does not adversely affect mortality in patients who develop it
E. 
Prognosis of delirium worsens with increased patient age and longer duration of the episode
F. 
Periods of delirium are sometimes followed by depression or PTSD

419

Psychiatry Test Preparation and Review


Manual E-Book
Psychiatry Test Preparation & Review Manual

Vignette Eighteen
Allan Newbold is a 30-year-old man who consults you at your private office for anxiety. He describes his anxiety as an
excessive preoccupation with his appearance. He is always worried that he isn’t attractive enough to the opposite sex. He
has no medical or surgical history at all. He has never seen a psychiatrist before. He denies depression, but he is very upset
because he always feels his body could be better. He exercises twice a day and has a physique that is similar to most fit mod-
els or competitive bodybuilders. You ask him to identify his specific shortcomings, and he tells you that his skin doesn’t tan
evenly so he has to resort to artificial spray-on tans, which “look fake,” he says. He also feels that his body has hair in the
“wrong places,” and “I always have to go to the laser hair removal salon or get it waxed off. Even then, there’s always some
left over.” He also feels that because he is a natural bodybuilder who doesn’t use artificial means of building muscle, like
steroids, his muscles are unable to develop evenly and symmetrically. He opens his shirt and shows you his bare chest, point-
ing out to you how his abdominal muscles are uneven and lumpy and asymmetrical. To your eye and superficial glance, they
look perfectly normal and you tell him so. He replies, “Of course you think they’re normal! Everyone I ask tells me they’re
normal, but I know they’re just lying to me.”
Allan tells you he is actually a physical therapist by training and says, “I love my job because I work with kids mostly and
help them in ways that no one else really can.” He also reveals that he does TV and magazine modeling on the side and has
even posed nude in Playgirl magazine. He was named “man of the year” for that publication a few years back. He tells you
he is heterosexual and has a girlfriend, Eve Chandler, who is a 24-year-old fitness model and hedge fund associate. He tells
you that his sex life and sexual functioning “are fantastic! No problems there!” When you ask if Eve thinks his body has
imperfections, he says, “She tells me it’s all in my head and that my body rocks, but I know she’s only saying it to be kind
to me!” He spends a huge amount of time and effort at esthetic salons, at tanning salons, and with the dermatologist look-
ing for creams, lotions, injectables, and any other procedures that he feels might enhance his appearance. He spends at least
$2000 a month on such products and services.

1. 
The basic pathophysiology of the disorder that Allan is suffering from is believed to be related to:
A. 
Dopamine
B. 
Norepinephrine
C. 
Epinephrine
D. 
Serotonin
E. 
Glutamate

2. 
The psychodynamic explanation of Allan’s disorder and behavior is best described as:
A. 
Early parental losses that lead to a self-focused neurosis
B. 
Acting out behavior due to poor impulse control and poor frustration tolerance
C. 
The displacement of a sexual or emotional conflict onto nonrelated body parts
D. 
Arrested development in the anal phase of psychosexual development
E. 
An unresolved Oedipal complex

3. 
Which of the following factors are atypical of Allan’s most likely diagnosis? (Pick two of six)
A. 
The fact that he is a man
B. 
The fact that the onset of his disorder presented before 30 years of age
C. 
The fact that Allan has never suffered a major depressive episode in the past
D. 
The fact that Allan is unmarried
E. 
The fact that Allan is a professional and is high-functioning
F. 
The fact that Allan spends an extraordinary amount of money on himself

420

Psychiatry Test Preparation and Review


Manual E-Book
Vignettes

4. 
Which of the following are considered appropriate treatments for Allan’s primary disorder? (Pick three of six)
A. 
Clomipramine
B. 
Bupropion
C. 
Fluoxetine
D. 
Modafinil
E. 
Phenelzine
F. 
Carbamazepine

5. 
Which of the following is typically true about the course and prognosis of Allan’s primary disorder?
A. 
It is usually gradual and insidious in onset
B. 
It is usually of short duration and self-limited
C. 
It has an undulating course with few symptom-free intervals
D. 
The part of the body on which concern is focused typically remains the same over time
E. 
The preoccupation with imagined defects is not usually associated with significant distress or impairment

Vignette Nineteen
Grace Hanover is a 50-year-old woman who is referred to you by her primary care physician. He has no clue what’s going
on with Grace medically, because she comes for follow-up every single month with a new physical complaint despite the fact
that he has told her so many times that there is nothing that he can find that’s wrong with her. Her physician tells you that
Grace indeed has suffered from hypertension and hyperlipidemia for the past 3 years, but that she has been taking enalapril
and simvastatin daily since then, which have normalized her blood pressure and serum lipid levels quite nicely. He tells you
that her many complaints have been going on since he has known her, which is 20 years now, but he knows that these com-
plaints predated her being his patient.
You ask her physician what symptoms she presents with, and he runs off a ridiculously long shopping list that is over-
whelming and implausible. Her biggest (and longest complaint) is sexual dysfunction on the order of low libido, poor sexual
arousal, and inability to orgasm through either masturbation or intercourse. This has been going on for 20 years, or even
more. Gynecologic consultations have been multiple over the years, and testing has never revealed any organic cause to these
problems.
Over the years she has complained of gastrointestinal discomfort after eating, though not all the time; periodic dizziness
and feeling weak in her legs for no particular reason; generalized body aches and pains, particularly in her neck, lower back,
arms, and legs; and chronic constipation, despite drinking plenty of water every day and eating a very well-balanced diet.
Her physician says, “She’s just weird, and I never know what she’s going to come up with next when she comes to the
office. I think it’s all in her head, but I’m not sure. I think her insurance company must hate her, because with all the tests she’s
undergone over the past 20 years (and all of them were negative!), it must have cost them hundreds of thousands of dollars!”
Grace works in public relations and has a six-figure-salary job. She is rarely absent from work, despite her many physical
problems. She has never been married and has no children. She was an only child and was doted on by her father, whereas
her mother was actually the breadwinner in the household and was more distant with Grace. Grace has no psychiatric his-
tory to speak of. She denies depression, mania, and psychosis. She does have some anxiety but denies panic attacks or social
phobia.

1. 
Which of the following symptoms would push your differential diagnosis away from a conversion disorder (functional
neurological symptom disorder) in Grace’s case? (Pick three of seven)
A. 
Backaches
B. 
Headaches
C. 
Anorgasmia
D. 
Dizziness
E. 
Constipation
F. 
Gastrointestinal discomfort
G. 
Arm and leg pain

421

Psychiatry Test Preparation and Review


Manual E-Book
Psychiatry Test Preparation & Review Manual

2. 
Which of the following etiological theories are believed to be possible contributors to Grace’s primary problem? (Pick
three of six)
A. 
Abnormal regulation of the cytokine system
B. 
Only personality disordered patients have Grace’s disorder
C. 
Decreased metabolism in the frontal lobes and nondominant hemisphere
D. 
Apoptosis and gliosis of brain-stem neurons
E. 
Catecholaminergic deficits or imbalance in the CNS
F. 
Genetic predisposition of the disorder in first-degree female relatives of probands of patients with Grace’s disorder

3. 
Which of the following statements about the epidemiology of Grace’s primary disorder are not true? (Pick three of six)
A. 
Men outnumber women with the disorder by about 5 to 20 times
B. 
The lifetime prevalence of somatic symptom disorder among women is about 1% to 2%
C. 
The disorder occurs more frequently in patients of upper class and higher socioeconomic status
D. 
The disorder usually begins in adulthood after the age of 30
E. 
Concomitant personality traits associated with somatic symptom disorder include obsessive–compulsive, paranoid, and
avoidant features
F. 
Bipolar I disorder and substance abuse occur no more frequently in somatic symptom disorder patients than in the
general population at large

4. 
Which of the following facts are true about the course and prognosis of Grace’s primary disorder? (Pick three of six)
A. 
The course of the disorder is typically acute and static in its presentation
B. 
Patients with the disorder have a 20% chance of being diagnosed with this disorder 5 years later
C. 
Patients with the disorder are no more likely to develop another medical illness in the next 20 years than people with-
out the disorder
D. 
The disease rarely remits completely
E. 
It is unusual for a patient with the disorder to be free of symptoms for longer than 1 year
F. 
The overall prognosis of the disorder is good to excellent in most cases

5. 
The only treatment maneuver for Grace’s primary disorder that seems to be able to decrease personal health care expendi-
ture by about 50% is:
A. 
Atypical antipsychotics
B. 
Antidepressant medications
C. 
Electroshock therapy
D. 
Group and individual psychotherapy
E. 
Mood stabilizers like lithium and divalproex sodium
F. 
Opioid antagonists like naltrexone

Vignette Twenty
Kerry Fields is a 26-year-old man who comes to you because he is an extreme athlete and is addicted to opioid painkillers.
He skis, snowboards, drives motocross motorcycles and all-terrain vehicles, and knows only too well how to abuse his body
from all this physical activity. He has had a shoulder surgery for severe rotator cuff tear and at least three knee surgeries on
each knee for meniscal tears and repairs. He also has spinal scoliosis and has herniated two cervical and three lumbar inter-
vertebral disks in the past. He has never undergone back surgery, though he has been used to living with chronic pain.
He tells you his problem is balancing pain with narcotic overuse. He is currently taking Roxicodone 30 mg, two tablets four
times a day, and they are barely keeping him stable. He confesses to you that he also drinks every night, at least two or three
vodkas with soda and sometimes more. He also smokes cannabis, about an ounce a week on average. He tells you his back pain,
neck pain, and knee pain are typically a 7 out of 10 most days, unless he is doing some extreme sporting activity or other, when
his pain can climb to 9 out of 10 after the activity is over. He wants help from you and your honest recommendations.

422

Psychiatry Test Preparation and Review


Manual E-Book
Vignettes

1. 
Which of the following would be good recommendations for treating Kerry’s problems? (Pick three of six)
A. 
Go to an inpatient facility for alcohol, narcotic, and cannabis detoxification
B. 
Do an outpatient narcotic taper and switch Kerry to a Vivitrol (naltrexone) monthly injection
C. 
Send Kerry to a specialized pain clinic for appropriate recommendations and management
D. 
Continue the Roxicodone as prescribed for pain and give Kerry disulfiram (Antabuse) for alcohol relapse prevention
E. 
Send Kerry for orthopedic and neurologic consultation to determine the etiology of his pain
F. 
Consider a switch from Roxicodone to oral methadone to address both pain and narcotic dependence, along with
acamprosate calcium to address alcohol relapse prevention

2. 
What is the correct dosing strategy if Kerry is going to begin taking disulfiram?
A. 
500 mg once daily
B. 
250 mg once daily
C. 
Start 500 mg once daily for 1 to 2 weeks and then reduce to 250 mg daily for maintenance
D. 
Start 250 mg once daily for 1 to 2 weeks and then increase to 500 mg daily for maintenance
E. 
500 mg twice a day

3. 
What are the disadvantages of buprenorphine/naloxone tablets for opioid relapse prevention in Kerry’s case? (Pick three of
six)
A. 
The naloxone content of the tablet may trigger a precipitated withdrawal on its own
B. 
Buprenorphine/naloxone tablets cannot be taken with full opioid agonist painkillers
C. 
Buprenorphine/naloxone tablets are generally not as effective as full opioid agonist painkillers for the management of
moderate to severe chronic pain
D. 
Buprenorphine/naloxone tablets can easily be diverted and injected for recreational purposes by intravenous drug users
E. 
Buprenorphine/naloxone tablets must be taken under the tongue, and patients frequently complain that they have a
bad taste
F. 
Buprenorphine/naloxone tablets can be abused and overused for their euphoric effects by recreational narcotic users
who take excessive quantities of this medication

4. 
Which of the following agents decreases serum methadone levels?
A. 
Phenytoin
B. 
St. John’s wort (Hypericum)
C. 
Dextromethorphan
D. 
Erythromycin
E. 
Verapamil
F. 
Disulfiram

5. 
Which of the following substances of abuse causes a withdrawal syndrome when stopped abruptly that manifests with
insomnia, irritability, drug craving, restlessness, nervousness, depressed mood, tremor, malaise, myalgia, and increased
sweating?
A. 
Cocaine
B. 
Opiates
C. 
Phencyclidine
D. 
Cannabis
E. 
Alcohol

423

Psychiatry Test Preparation and Review


Manual E-Book
Psychiatry Test Preparation & Review Manual

Vignettes
Answer Key

Vignette One 3. ABCD 5. C


1. C 4. AB 6. B
2. ABD 5. ABC 7. ABCD
3. DE 6. ABCD 8. ABCD
4. D 7. ACD 9. BC
5. ABCE 8. D 10. B
6. AB 9. ABC
7. ABC 10. C Vignette Six
8. C 1. BCE

9. C Vignette Four 2. A

10. ABCD 1. B 3. BCF


2. ABDE 4. AD
Vignette Two 3. C 5. E
1. B 4. C 6. B
2. ABD 5. ABC
3. E 6. DE Vignette Seven
4. C 1. E
7. BCD
5. DE 2. BDF
8. D
6. ACD 3. BDE
9. ABC
7. CEF 4. BC
10. ABCD
8. D 5. BCD

9. C Vignette Five
1. B Vignette Eight
1. D
Vignette Three 2. AC
1. C 2. A
3. D
2. C 3. B
4. ACD

Full test - and additional VIDEO vignettes - available online - see inside front cover for details.

424

Psychiatry Test Preparation and Review


Manual E-Book
Vignettes

4. BD 8. BC 4. E
5. B 9. ABC 5. CE
6. D 10. ABD
7. ABD Vignette Sixteen
Vignette Twelve 1. CE
8. BCD
1. ABCE 2. CFG
9. ABCD
2. ABCDE 3. BCF
10. ACD
3. CD 4. C
Vignette Nine 4. D 5. ADG
1. ABC 5. BD 6. F
2. ABCDE 6. A 7. DE
3. ABC 7. CD
4. B 8. D Vignette Seventeen
1. AE
5. ABCD 9. ABCD
2. ADE
6. ABC 10. ABCD
3. D
7. ACD
Vignette Thirteen 4. B
8. AD
1. BE 5. C
9. ABCD
2. CD 6. CE
10. BCD
3. ACD 7. BEF
Vignette Ten 4. BEF
1. ACD 5. AB Vignette Eighteen
1. D
2. ABC 6. BCD
2. C
3. A
Vignette Fourteen 3. AC
4. AB
1. DE 4. ACE
5. C
2. B 5. C
6. C
3. DE
7. D Vignette Nineteen
4. A
8. D 1. CEF
5. A
9. ABC 2. ACF
6. A
10. A 3. ACD
7. A
4. CDE
Vignette Eleven 8. A
5. D
1. AB 9. C
2. AD 10. ABCDEFG Vignette Twenty
3. ABD 1. ACF
4. B Vignette Fifteen
2. C
1. AC
5. D 3. BCE
2. BF
6. AD 4. ABC
3. D
7. ABD 5. D

425

Psychiatry Test Preparation and Review


Manual E-Book
Psychiatry Test Preparation & Review Manual

Vignettes
Explanations
Vignette One
1. C. PTSD is the best explanation for the scenario in this vignette. In PTSD the person experienced or wit-
nessed an event that involved actual or threatened death or serious injury, witnessed a traumatic event
occur to others, learned that a traumatic event occurred to close family or friends, or experienced repeated
or extreme exposure to aversive details of a traumatic event. The patient then needs at least one intrusion
symptom, one symptom of avoidance, two symptoms of increased arousal, and two symptoms of nega-
tive alterations in cognition and mood to meet DSM criteria. In this case the nightmares are a symptom
of intrusion. The restricted range of affect (low mood, unable to be happy) is evidence of negative altera-
tions in cognition or mood. Avoiding people associated with the trauma counts as an avoidance symptom.
Irritability and outbursts of anger with his family and the nurses and difficulty staying asleep are consid-
ered symptoms of increased arousal.
Although he may have some symptoms of depression, he does not meet major depressive disorder (MDD)
criteria, and the overall picture is better explained by PTSD. There is more going on symptom-wise than
would be explained by an adjustment disorder with depressed mood. Although he no doubt has several
medical issues at this point, mood disorder secondary to a medical condition does not fully account for the
full picture we are seeing. The most comprehensive explanation is found in PTSD.
K&S Chapter 11

2. ABC. Most patients with PTSD receive both medication and therapy. SSRIs and serotonin–norepinephrine reup-
take inhibitors (SNRIs) are first-line treatment for PTSD. Cognitive behavioral therapy (CBT) is the type
of therapy with the most evidence of effectiveness for PTSD. Psychodynamic psychotherapy does not have
significant evidence of effectiveness in PTSD. Dialectical behavior therapy is first-line for borderline per-
sonality disorder, not PTSD. Benzodiazepines should be avoided in PTSD, both because of the high risk
for potential addiction and because some studies have shown that benzodiazepines slow recovery rates for
PTSD. Patients take longer to recover when they are on benzodiazepines, so they are not a good choice. In
this vignette, family therapy is certainly a good idea, given the stress the family is under and the patient’s
reaction to them.
K&S Chapter 11

3. DE. Rapid onset of symptoms, strong social supports, absence of other psychopathology or substance abuse,
short duration of symptoms (less than 6 months), and good premorbid functioning would all be considered
good prognostic factors for PTSD. The opposite of any of these would be considered poor prognostic fac-
tors. As a general rule, the very old and very young have the highest likelihood of developing PTSD, with
those in the middle of life faring best.
K&S Chapter 11

Full test - and additional VIDEO vignettes - available online - see inside front cover for details.

426

Psychiatry Test Preparation and Review


Manual E-Book
Vignettes

4. D. Alexithymia is an inability to describe feeling states and can be a part of the PTSD picture. Tactile halluci-
nations and thyroid abnormalities are not usually associated with PTSD. We see increased norepinephrine
turnover in the locus ceruleus in PTSD, not decreased.
K&S Chapter 11

5. ABCE. Overcoming denial, use of imaginal techniques or in  vivo exposure, encouragement and help for proper
sleep, and learning cognitive approaches to stress can all be helpful to patients with PTSD as part of a suc-
cessful therapeutic approach. Cathartic expression of aggression is not a common component of PTSD
treatment, and patients are encouraged to verbalize feelings rather that act them out aggressively.
K&S Chapter 11

6. AB. Angry outbursts and restriction of affect can occur in both schizophrenia and PTSD. Restricted affect
would be considered a symptom of avoidance in PTSD. Decreased need for sleep is characteristic of mania.
Exaggerated startle response is a symptom of arousal/reactivity seen in PTSD.
K&S Chapter 11

7. ABC. Decreased sleep, and most importantly decreased need for sleep, can be seen in bipolar I disorder.
Generalized anxiety disorder may present with sleep disturbance as one of the physical symptoms (the oth-
ers are muscle tension and fatigue). PTSD can present with difficulty falling or staying asleep as part of the
increased arousal symptoms. Obsessive–compulsive disorder does not have sleep disturbance as part of its
DSM criteria.
K&S Chapter 11

8. C. This is an example of classical conditioning. Classical conditioning is when a neutral (conditioned) stim-
ulus is paired with a stimulus that evokes a response (unconditioned stimulus) such that over time the
neutral stimulus eventually elicits the same response as the unconditioned stimulus. In this case the uncon-
ditioned stimulus was the fire, and the reaction was fear and horror. In time the neutral stimulus of the
chainsaw takes on the power to generate the fear originally caused by the fire. Classical conditioning is
part of the behavioral model for PTSD.

In operant conditioning, voluntary behavior is modified as the patient actively tries different behaviors to
see which will deliver a desired reward. Learned helplessness is a model of depression in which a patient
repetitively fails at a task and eventually stops trying, adopting a hopeless, apathetic position. Premack’s
principle states that behavior engaged in with high frequency can be used to reinforce behavior that occurs
with low frequency.
K&S Chapters 2 and 11

9. C. Feelings of detachment from others are considered symptomatic of negative alterations in cognition and
mood when diagnosing PTSD. All other choices in the question are symptoms of increased arousal.
K&S Chapter 11

10. ABCD. The differential diagnosis for PTSD should include all of the choices mentioned in this question, except
schizotypal personality disorder. Panic disorder can present with symptoms that could be considered in line
with avoidance and hyperarousal. Substance abuse can mimic anxiety symptoms, and there is a high over-
lap between PTSD and substance abuse, especially alcohol. MDD presents with symptoms similar to the
avoidance symptoms of PTSD (restricted affect, diminished interest in activities, feelings of detachment).
Borderline personality disorder may present with mood lability, irritability, and angry outbursts, which
could be mistaken for hyperarousal symptoms in PTSD.
K&S Chapter 11

Vignette Two
1. B. This question asks you to focus on the most clinically appropriate maneuver in this case. You could cer-
tainly send the patient home with yet another medication trial, but the fact that she has already failed
multiple medication trials makes the prognosis for success very poor. Clearly, her daughter has done the
patient no harm, so there is no question here of reporting her to state authorities as a case of possible

427

Psychiatry Test Preparation and Review


Manual E-Book
Psychiatry Test Preparation & Review Manual

elder abuse. ACT teams are an excellent treatment modality, principally for patients who have poor insight
into their mental disorder and, as a result, are poorly adherent to medication regimens that would prevent
decompensation. ACT teams, because of their mobility and their high staff-to-patient ratio, are able to
service such patients in their home environment and at the same time ensure that the medications are filled
and taken appropriately. ACT teams are not a substitute for skilled nursing care or skilled nursing facil-
ity placement. Such teams are unable to handle and manage patients at home who have multiple medical
problems and need structure and help with basic activities of daily living, such as shopping, cooking, and
cleaning. Such patients are much better served by skilled nursing facility placement or, if less severe, by
visiting nurse services complemented by home health care assistance. In this particular case, though, plac-
ing the patient in a skilled nursing facility is not the best immediate maneuver. The patient is profoundly
depressed and in need of intensive psychiatric care before nursing home placement. If placed in such a
facility in her present mental state, the patient will surely not be able to be managed by a nursing home
consulting psychiatrist, given the profound severity of her depression and disintegration of her activities
of daily living. The best maneuver in this case would be to convince the patient and her daughter that
a voluntary psychiatry inpatient admission would be best for the patient. Consideration of electroshock
therapy, once admitted, is quite appropriate, given the patient’s failure to improve with numerous trials of
antidepressant medications. The most common indication for ECT is MDD, for which ECT is the fastest
and most effective available therapy. ECT should be given consideration when patients have failed mul-
tiple medication trials; are acutely suicidal, homicidal, or psychotic; or have severe symptoms of stupor or
agitation.
K&S Chapter 29

2. ABD. The pretreatment evaluation for ECT includes physical examination, neurologic examination, anesthesia
consultation, and complete medical and surgical history. Laboratory testing should include blood and urine
chemistries, chest radiography, and an ECG. A dental examination is advisable, particularly in the elderly,
who may have poor dentition or poor dental hygiene. Spine radiography should be done only if there is
history or suspicion of preexisting spinal disorder. Brain CT or MRI scan should be done if there is history
or suspicion of a seizure disorder or a space-occupying lesion. In this particular vignette, the patient may
well have a brain tumor of some sort that is causing her refractory depression. Even though a brain tumor
is not an absolute contraindication to ECT, a brain scan should be performed in this case to rule out that
possibility. Thyroid function tests and routine electroencephalogram are not needed for pretreatment ECT
evaluation.
K&S Chapter 29

3. E. Electroshock therapy has no absolute contraindications. Patients with situations that put them at increased
risk merely need closer and more careful monitoring before, during, and after the procedure. Pregnancy
is not a contraindication for ECT. Patients with space-occupying brain lesions are at risk for brain edema
and herniation after the procedure. Those patients with smaller mass lesions can be premedicated with
dexamethasone (Decadron), which reduces the risks after the procedure. Patients with recent myocardial
infarction are a high-risk group, but the risk is diminished 2 weeks after the myocardial infarction, and
even further diminished 3 months after the infarction. Patients with hypertension should be well stabilized
on their antihypertensive medications before ECT is administered.
K&S Chapter 29

4. C. For a seizure to be effective in the course of ECT, it should last at least 25 seconds. Proper objective sei-
zure monitoring must be undertaken by the physician conducting the ECT. There must be evidence that a
bilateral generalized seizure has taken place after the electrical stimulation has been applied. The electroen-
cephalogram (EEG) and electromyogram enable this to be monitored objectively.
K&S Chapter 29

5. DE. The most worrisome side effect of ECT is memory loss. About 75% of patients given ECT complain that
memory impairment is the worst adverse effect. Most patients with memory impairment report a return
to baseline within 6 months after treatment. Fractures and muscle or back soreness are possible with ECT,
but with routine use of muscle relaxants, fractures of long bones do not generally occur. A minority of

428

Psychiatry Test Preparation and Review


Manual E-Book
Vignettes

patients experience nausea, vomiting, and headache after ECT. Mortality rate with ECT is about 0.002%
per treatment and 0.01% for each patient. Hypertension can occur during the seizure but can be controlled
by antihypertensive agents administered at that time. Hypertension is not typically a long-term adverse
effect of ECT.
K&S Chapter 29

6. ACD. The indications for maintenance ECT treatments are severe medication side effects and intolerance, psy-
chotic or severe symptoms, and rapid relapse after a successful initial round of treatments. Maintenance
ECT should always be considered after a remission of symptoms from a first round of treatment, because
initial positive response is rarely maintained and relapses often occur despite an initial response to a first
round of treatments. While on the topic of ECT, don’t forget that evidence supports the use of ECT in
treatment-resistant Parkinson’s disease.
K&S Chapter 29

7. CEF. Benzodiazepines should be tapered and withdrawn before ECT because of their anticonvulsant properties.
Lithium must be withdrawn before ECT because it can cause a postictal delirium and can prolong seizure
activity. Clozapine and bupropion should also be withdrawn before ECT, because they are known to be
associated with late-appearing seizures. Antidepressants in the class of the SSRIs, SNRIs, tricyclics, and
MAOIs are not contraindicated with ECT.
K&S Chapter 29

8. D. Methohexital (Brevital) is the most commonly used anesthetic agent for ECT because of its shorter dura-
tion of action and lower association with postictal arrhythmias. Etomidate (Amidate) is sometimes used
in elderly patients, because it does not increase the seizure threshold, and it is well understood that sei-
zure threshold increases as patients age. Ketamine (Ketalar) is sometimes used because it doesn’t raise the
seizure threshold. It is, however, associated with the emergence of psychotic symptoms after anesthesia.
Alfentanil (Alfenta) is used concomitant with barbiturates in some cases, because it allows for lower dosing
of the barbiturates, which lowers the seizure threshold further. It is, however, associated with an increased
incidence of nausea. Propofol (Diprivan) is less useful as an anesthetic agent in ECT, because it raises the
seizure threshold.
K&S Chapter 29

9. C. Electrode placement for ECT can be either unilateral or bilateral. Bilateral placement leads to a more rapid
therapeutic response in most cases, but it also results in a higher frequency of memory impairment. Most
practitioners will begin treatment with unilateral ECT because of its more favorable adverse effect profile.
If the patient does not improve after four to six unilateral treatments, most clinicians will strongly con-
sider moving to bilateral electrode placement thereafter. Initial bilateral electrode placement is considered
in cases of severe depressive symptoms, catatonic stupor, acute suicide risk, manic symptoms, treatment-
resistant schizophrenia, and marked agitation.
K&S Chapter 29

Vignette Three
1. C. Cathy would best be described as having bipolar II disorder. To meet criteria for bipolar II requires at
least one MDD episode and one hypomanic episode. Cathy clearly meets MDD criteria. Hypomania is
defined by a clear period of irritable, expansive, or elevated mood lasting for at least 4 days but that does
not cause marked impairment in functioning. In addition, there must be at least three of the same symp-
toms that define manic episodes (pressured speech, decreased need for sleep, grandiosity, flight of ideas,
distractibility, increased goal-directed activity, excess involvement in behaviors with high potential for
harmful consequences). Cathy fits this profile. If she were to have mania, her symptoms would continue
for 1 week or more and/or she would have marked impairment in functioning, neither of which are true
in this case. MDD is incorrect, because it does not explain the entire picture we are seeing. Cyclothymic
disorder is defined as hypomania plus subthreshold depressive symptoms for a period of 2 years or longer.
This doesn’t fit Cathy’s case. Substance/medication-induced bipolar and related disorder would certainly

429

Psychiatry Test Preparation and Review


Manual E-Book
Psychiatry Test Preparation & Review Manual

be included in a differential for Cathy but would not be the best choice because no data on the connection
between substance abuse and mood changes have been given. She also denies the use of substances that
would probably precipitate a manic episode around the time of her sister’s wedding (cocaine).
K&S Chapter 8

2. C. Key differentiating factors between hypomania and mania would include the time period (4 to 6 days for
hypomania vs 7+ days for mania) and the presence or absence of marked impairment in social or occupa-
tional functioning. Irritable mood, decreased need for sleep, and flight of ideas could be present in either.
K&S Chapter 8

3. ABCD. Psychosis, rapid cycling, severity of symptoms, and pregnancy are all valid concerns that would have an
effect on the choice of medications in the bipolar patient. For patients in their 30s, age wouldn’t be a major
factor.
K&S Chapter 8 and 29

4. AB. ECT for bipolar has been proven very effective in severe mania with psychosis and in pregnancy. Mania
secondary to a medical condition or substance abuse would not necessarily lead to ECT and would most
likely be treated with medications.
K&S Chapter 8 and 29

5. ABC. Glioma, Cushing’s disease, and MS have all been associated with mania. Thiamine deficiency is not associ-
ated with mania but is a crucial component of Wernicke–Korsakoff syndrome, seen in alcoholics.
K&S Chapter 8

6. ABCD. Isoniazid, cimetidine, metoclopramide, and steroids can all cause a manic episode. Of course there are oth-
ers, such as bronchodilators, antidepressants, anticonvulsants, stimulants, barbiturates, and several drugs
of abuse. Benzodiazepines would tend to lessen mania, not cause it.
K&S Chapter 8

7. ACD. Cocaine use causes rapid dopamine and norepinephrine reuptake inhibition. It increases dopamine in the
mesolimbic and mesocortical pathways, as well as the corpus striatum.
K&S Chapter 20

8. D. Sequelae of cocaine use include hallucinations, paranoia, euphoria, increased energy, hypersexuality, and
irritability. With heavy cocaine use, patients can experience a shower of lights in their central vision, as
well as visual hallucination of black dots on their skin and in the environment (coke bugs). Itching and
respiratory depression come from opiate abuse, not cocaine.
K&S Chapter 20

9. ABC. Treatment for cocaine overdose can include cold blankets and ice packs for hyperthermia, intravenous
diazepam for seizures, intravenous phentolamine for malignant hypertension, and both haloperidol and
lorazepam for agitation. Clonidine is useful in treating the autonomic effects of opiate withdrawal but is
not used in cocaine overdose.
K&S Chapter 20

10. C. Sixty percent of patients with bipolar disorder have a co-occurring substance abuse disorder. As such,
Cathy is not unusual in this regard. It means that it is very important to screen for substance abuse in any
bipolar patients.
K&S Chapter 8

Vignette Four
1. B. Susan has borderline personality disorder. Characteristics of the disorder include frantic efforts to avoid
abandonment, unstable and intense interpersonal relationships, idealization and devaluation, unstable
self-image, impulsivity in self-damaging ways, affective instability, chronic feelings of emptiness, intense

430

Psychiatry Test Preparation and Review


Manual E-Book
Vignettes

inappropriate anger, and transient stress-related paranoia or dissociation. They can have short-lived psy-
chotic episodes. In Susan’s case, we see impulsivity in self-damaging ways, unstable interpersonal rela-
tionships, intense inappropriate anger, efforts to avoid abandonment, affective instability, and unstable
self-image.
K&S Chapter 22

2. ABDE. Characteristics of Susan’s diagnosis are frantic efforts to avoid abandonment, unstable and intense inter-
personal relationships, idealization and devaluation, unstable self-image, impulsivity in self-damaging
ways, affective instability, chronic feelings of emptiness, intense inappropriate anger, and transient stress-
related paranoia or dissociation. Grandiosity is not a characteristic of borderline personality disorder but
can be seen in bipolar disorder and psychotic disorders.
K&S Chapter 22

3. C. Perceived rejection is the centerpiece of many borderline suicide attempts and is certainly the precipitating
factor in Susan’s case.
K&S Chapter 22

4. C. The treatment of choice for borderline personality disorder is dialectical behavior therapy.
K&S Chapter 22

5. ABC. Borderline patients may be in touch with reality only on a basic level, have limited capacity for insight, and
use primitive defenses, such as splitting. They do not have an integrated sense of self. Their unstable sense
of self is an important part of the disorder.
K&S Chapter 22

6. DE. Acting out and splitting are two of the defenses most commonly associated with borderline personality dis-
order. The others have no particular association with borderline personality disorder.
K&S Chapter 22

7. BCD. Patients with borderline personality self-harm to express anger, elicit help from others, and numb them-
selves to overwhelming affect. Their goal is not to socially isolate themselves, and they may feel quite
dependent on others, despite the fact that they eventually do drive others away by their extreme behavior.
K&S Chapter 22

8. D. Projective identification is a defense mechanism used by borderline patients that was first described by
Otto Kernberg. Intolerable aspects of the self are projected onto another person. The other person is then
induced to play the projected role. This is of particular concern for psychiatrists working with border-
line patients, as they can be pulled into this dynamic if they lose their neutral stance. Displacement is a
defense mechanism whereby emotional energy is taken from one object and placed onto another unrelated
object. Rationalization is using rational explanations to justify beliefs and attitudes that would otherwise
be socially unacceptable. Splitting is seeing other people or situations as either all good or all bad. It is a
commonly used defense of the borderline patient.
K&S Chapters 8 and 22

9. ABC. Impulsive behaviors are certainly part of the borderline picture. Prolonged psychotic episodes are not.
Borderline patients may have brief episodes of psychosis when under significant stress, but prolonged
psychosis is something seen on Axis I. Marked peculiarity of thinking is more a descriptor of schizotypal
personality disorder than of borderline. Extreme suspiciousness is more a descriptor of paranoid personal-
ity disorder than of borderline. That is not to say that a borderline patient may never be suspicious, but
extreme suspiciousness is not one of the defining DSM criteria.
K&S Chapter 22

10. ABCD. All of the choices except stimulants may have a role in treating Susan’s condition. Antipsychotics have
been used for anger, hostility, and brief psychotic episodes. SSRIs have been used for depressive features
and aggression. Anticonvulsants are helpful for mood lability and aggression. MAOIs have been helpful in

431

Psychiatry Test Preparation and Review


Manual E-Book
Psychiatry Test Preparation & Review Manual

modulating impulsive behavior in some patients. Stimulants are not considered effective treatment for bor-
derline personality disorder. But if asked on an examination which drug would be the first-line choice for
treating anger and impulsivity in a borderline patient, the answer is an SSRI.
K&S Chapter 22

Vignette Five
1. B. During the initial contact, the doctor wishes to establish rapport quickly, put the patient at ease, and show
respect. If the patient wishes to have someone else in the initial session with him or her, then that should be
respected. Patients have a right to know the position and professional status of the people involved in their
care. As such, introducing yourself is important. The answer choice that best addresses all of these issues is
choice B.
K&S Chapter 5

2. AC. Starting the interview with an open-ended question is best. It allows patients to describe what has brought
them in and signals to them that you are interested in hearing what they have to say. Choices A and C are
open ended and allow the patient to tell Dr. Smith what is wrong in her own words. Choices B and D are
too closed ended and specific for an opening question. They are both phrased so directly the patient may
interpret them as rude.
K&S Chapter 5

3. D. A patient may be frightened or anxious at the beginning of an interview. The first step in addressing this
is by acknowledging the patient’s anxiety and offering reassurance. Of the choices given, the one that does
this best is choice D. Choices A and B have an accusatory tone, which may make the patient shut down
more. Choice C is ignoring the dynamic in the room and talking about a completely unrelated subject that
does not address the patient’s underlying discomfort.
K&S Chapter 5

4. ACD. The content of an interview is what gets said between doctor and patient, such as subjects discussed and
topics mentioned. The process of the interview is what happens nonverbally, such as body language, behav-
iors, or avoidance of difficult topics. Choices A, C, and D are all behaviors, body language, or avoidance of
difficult topics. Choice B is a topic that was spoken about in the session and, as such, is content.
K&S Chapter 5

5. C. In clarification the therapist tries to get more details about what the patient has already said. That is what
Dr. Smith is doing in this question. Confrontation is pointing out something that the patient is missing
or denying. Facilitation is using both verbal and nonverbal cues to encourage a patient to keep talking.
Explanation is when the doctor describes the treatment to the patient in clear understandable language and
gives the patient the opportunity to ask questions.
K&S Chapter 5

6. B. In transition the doctor lets the patient know that he or she has gathered enough information on one subject
and encourages the patient to move on to another subject. In reassurance the doctor informs the patient of his
or her condition in a way that leads to increased trust and compliance; reassurance is experienced by the patient
as empathic and caring. In positive reinforcement the doctor makes the patient feel as if the doctor is not upset
no matter what the patient says, so as to facilitate an open exchange of information. In advice the doctor rec-
ommends a course of action to the patient. This should always be done after the patient has had time to speak
freely about his or her problems. If done before this, it can be received as inappropriate or intrusive.
K&S Chapter 5

7. ABCD. In ending the session, it is important to give patients a chance to ask questions and explain future plans
and next steps. You should thank the patient for sharing information. Any prescriptions should be
reviewed, and the patient should understand why a medication is being given and how to take it. Patients
should be encouraged to call with questions if they need clarification of their treatment, if anything emer-
gent arises, or if they have side effects to medications and need guidance.
K&S Chapter 5

432

Psychiatry Test Preparation and Review


Manual E-Book
Vignettes

8. ABCD. Putting the patient at ease, expressing compassion for pain, showing expertise, and establishing authority
as a physician are all necessary parts of developing rapport with a patient. To do this well the doctor must
balance the roles of empathic listener, expert, and authority. Doctors are not expected to be all-knowing
and should be honest with the patient when they do not know the answer to a patient’s question.
K&S Chapter 5

9. BC. Decreased rates of patient compliance have been proven to be associated with increased complexity of the
treatment regimen and an increased number of behavioral changes required for the treatment to succeed.
Intelligence, gender, marital status, race, religion, socioeconomic status, and education level have not been
proven to correlate with compliance rates.
K&S Chapter 5

10. B. All of these answer choices represent different models of the doctor–patient relationship. In the teacher–
student model the physician’s dominance is emphasized. The physician is paternalistic and controlling. The
patient’s role is one of dependence and acceptance. In the active–passive model, patients assume no respon-
sibility for their care and take no part in their treatment. This is appropriate for patients who are uncon-
scious or delirious. In the mutual participation model, there is equality between both parties, and each
depends on the other for cooperation and input. Long-term management of chronic diseases often leads to
this model. In the friendship model, patient and doctor become friends. This is an arrangement that is con-
sidered unethical and dysfunctional. It often reflects psychological problems on the part of the physician.
K&S Chapter 5

Vignette Six
1. BCE. Gender dysphoria is defined as a marked incongruence between one’s experienced/expressed gender and
one’s assigned gender. In adolescents and adults, the disturbance is manifested by symptoms such as a
stated desire to be the other gender, frequent passing as the other gender, desire to live or be treated
as the other gender, or the conviction that the individual has the typical feelings and reactions of the
other gender. There is a persistent discomfort with his or her gender or sense of inappropriateness in the
birth-assigned gender role. In adolescents and adults, the disturbance is manifested by symptoms such as
preoccupation with getting rid of primary sex characteristics or belief that he or she was born the wrong
gender.
Transvestic fetishism is a disorder in which the patient has recurrent, intense sexually arousing fan-
tasies, sexual urges, or behaviors involving cross-dressing over at least a 6-month period. Pearl is not
cross-dressing for sexual arousal. She feels that she is a gender other than her birth-assigned gender.
Partialism, also called oralism, is categorized under other specified paraphilic disorder. People with this
disorder concentrate their sexual activity on one part of the body to the exclusion of all others. The
typical presentation involves preference for oral sex without intercourse. Urophilia is also a paraphilia,
which would fall under other specified paraphilia disorder in DSM 5. Pearl does meet criteria for uro-
philia, because it involves the intense desire to urinate on a partner or be urinated on. In both men
and women, this may also be associated with sexual arousal via the insertion of foreign objects into
the urethra for the purpose of sexual stimulation. To meet DSM 5 criteria, it needs to cause distress or
impairment.
Sexual sadism disorder and sexual masochism disorder are two different paraphilias, but they are related.
The former involves at least 6 months of recurrent, intense sexually arousing fantasies, sexual urges, or
behaviors involving acts in which psychological or physical suffering of the victim is sexually exciting to
the person. The latter involves at least 6 months of similar fantasies, urges, or behaviors involving acts of
being humiliated, beaten, bound, or otherwise made to suffer. To meet DSM 5 criteria, it needs to cause dis-
tress or impairment. It would seem that Pearl and her partner meet criteria for these paraphilias. Fetishistic
disorder is again a paraphilia of at least 6 months’ duration, during which time the person has recurrent,
intense sexually arousing fantasies, urges, or behaviors involving the use of nonliving objects (for example,
female undergarments or fake rubber penises). Pearl does not seem to have a fetishistic disorder.
K&S Chapter 17

433

Psychiatry Test Preparation and Review


Manual E-Book
Psychiatry Test Preparation & Review Manual

2. A. Poor prognosis for paraphilias is associated with early age of onset, a lack of guilt or shame about the act, a
high frequency of acts, and concomitant substance abuse. The prognosis is also better when there is a history of
the act of coitus with the paraphilia and when the patient is self-referred rather than referred by a legal agency.
K&S Chapter 17

3. BCF. Good prognostic indicators for paraphilias include normal intelligence, the absence of substance abuse, the
absence of personality disorders, the presence of normal adult attachments and relationships, the absence
of concomitant Axis I mental disorders, and the presence of only one paraphilia.
K&S Chapter 17

4. AD. There are essentially five types of psychiatric intervention used to treat paraphilias. These include reduction
of sexual drives, external control, treatment of comorbid conditions, dynamic therapy, and CBT. Prison is
an external control mechanism to prevent sexual crimes, but it does not usually involve a treatment ele-
ment. Interpersonal psychotherapy is not usually useful in helping prevent paraphiliac behaviors, urges,
and fantasies. Sex therapy can be effective and is considered an adjunctive modality to dynamic therapy.
Twelve-step programs based on the Alcoholics Anonymous model are of course useful in supporting indi-
viduals with sexual addiction and troublesome behaviors. Antiandrogen therapy may reduce sex drive and
aberrant sexual behavior by decreasing serum testosterone levels to subnormal concentrations.
K&S Chapter 17

5. E. The term transgender is used to refer to those who identify with a gender different from the one they were
born with (assigned gender). Within transgender individuals there are different groups. Those who wish to
have the body of another sex are referred to as transsexuals. Those who feel they are between genders, of
both genders, or of neither gender are referred to as genderqueer. Those who dress as the opposite gender
but continue to identify with their birth-assigned gender are cross-dressers. Transgender people may be of
any sexual orientation. They can be attracted to the same sex, opposite sex, or both sexes. Transvestic dis-
order is defined as a period of at least 6 months of intense sexual arousal from cross-dressing that causes
significant distress or impairment.
K&S Chapter 18

6. B. Contrary to what many people think, most transgender people do not end up having gender reassignment
surgery. One of the major barriers is financial. The most common type of surgery is chest surgery, with
genital surgery being less common because it is expensive, often involves multiple procedures, and is lim-
ited in terms of the results it can provide. More frequently, transgender people obtain hormone therapy
rather than surgery. Hormone treatment of transgender men is accomplished with testosterone. It requires
monitoring of the CBC, because increased hemoglobin and hematocrit levels can lead to stroke from an
increase in red blood cell count. Testosterone is processed in the liver, so routine liver function tests should
also be obtained. Testosterone treatment may increase the likelihood of lipid abnormalities and diabetes
as well. Transgender women may take estrogen, progesterone, or testosterone blockers. These can also be
given in combination. Women on these hormone treatments should routinely have their prolactin tested, as
patients on estrogen therapy can develop prolactinomas. Reproductive counseling is very important before
beginning hormone treatment, because permanent sterility is frequently the outcome. Therapy for gender
dysphoria is aimed at helping patients become comfortable with their gender identity and supporting adap-
tive behaviors.
K&S Chapter 18

Vignette Seven
1. E. The American Psychiatric Association forbids sexual relations between psychiatrists and their patients in
all circumstances. It doesn’t matter whether they are currently in treatment, how long ago they were in
treatment, or whether the patient considers it consensual. It is absolutely forbidden. Ethically it will always
be considered an exploitative relationship because of the power and authority difference between doc-
tor and patient. Psychiatrists will not be covered by malpractice in a situation where they had sex with a
patient and can face loss of a malpractice suit, criminal charges, loss of their medical license, and expulsion
from the American Psychiatric Association.
K&S Ch. 36

434

Psychiatry Test Preparation and Review


Manual E-Book
Vignettes

2. BDF. The commission of a criminal act has two components: voluntary conduct (actus reus) and evil intent
(mens rea). Evil intent cannot be present if the offender’s mental status is so impaired or diseased as to
have deprived the offender of the rational intent to commit the act. The law can be invoked only in the
presence of evil or malicious intent. Intent to do harm is not sufficient on its own as grounds for criminal
action. The M’Naghten rule is the statute derived from the 1843 case of Daniel M’Naghten in the British
courts. The question brought to bear in this rule is “Did the defendant understand the nature and qual-
ity of the act and the difference between right and wrong with respect to the act?” at the time the act was
committed. If a mental illness causes the defendant to be unaware of the consequences of his or her acts, or
if he or she was incapable of understanding that these acts were wrong, the person could then be absolved
of criminal responsibility for the acts. Ford v Wainwright is a landmark case in the matter of competence
to be executed and has nothing to do with the case in this vignette. The duty to warn and protect derives
from the case of Tarasoff v Regents of the University of California, which was the landmark case set-
ting the precedent for clinicians to be responsible for warning and protecting intended victims of patients
expressing intent to harm others.
K&S Chapter 36

3. BDE. The standard for competence to stand trial is set quite low to enable as many defendants as possible to
have their day in court. This standard was set by the U.S. Supreme Court in the landmark case of Dusky v
United States. The defendant must be able to demonstrate knowledge of the charges brought against him
or her. The defendant must demonstrate knowledge of the penalties associated with being found guilty of
each of these charges. He or she must demonstrate a knowledge and recognition of the various persons
involved in the case—the defense attorneys, the prosecuting attorneys, the judge, the witnesses who will be
called, and the jury. The defendant must be able to collaborate with his or her attorney with a reasonable
degree of rational understanding of the proceedings against him or her. Note that the defendant’s current
mental status (and not his or her mental status at the time the alleged acts were committed) is what is
brought to bear in determining competency to stand trial. The defendant does not have to recall every min-
ute detail of the acts that he or she is alleged to have committed. The defendant does not have to be able to
take the stand as a witness in his or her own defense. The defendant does not have to have an opinion on
what kind of plea the defense team should enter on his or her behalf. Defense and prosecuting attorneys
usually have the discretion to hire and appoint their own psychiatric expert witness to attest to the defen-
dant’s competency to stand trial. The judge has the final word on which, if any, of these opinions he or she
is willing to hear and entertain in the decision-making process on competency to stand trial. The judge also
has the discretion to appoint his or her own psychiatric expert witness(es) to lend further weight to the
decision in cases in which the defendant’s competency to stand trial may be difficult to determine.
K&S Chapter 36

4. BC. Exploitation involves using the therapeutic relationship for personal gain, such as hiring a patient or going
into business with a patient. Psychiatrists cannot have any business interactions with their patients other
than the patient paying them for psychiatric treatment. For Dr. Daniels to hire Selena to sell his home is
a violation of ethics guidelines and would be considered exploitation. The situation is made even worse
by the fact that he is in a romantic relationship with her, which is absolutely forbidden. The fact that he
is behaving unethically does not mean that other ethics rules don’t apply. Bartering for business services is
still entering into a business interaction with her outside of treatment, so would also be forbidden.
K&S. Ch. 36

5. BCD. If Dr. Daniels’ defense team has put in a psychiatric plea on all three charges and the prosecution has
allowed this to go to trial before a jury, then there is almost certainly a psychiatric impediment in the good
doctor’s case. If the jury finds the good doctor not guilty by reason of insanity on all three counts, then
there was certainly some compelling reason for the doctor’s mental status to have been clouded during
these events, given that the jury felt he did not know right from wrong at the time these acts were commit-
ted. With this knowledge alone, we know that Dr. Daniels needs psychiatric treatment of some kind. We
don’t know why his mental status was clouded, but a psychiatric disorder was probably the root cause at
that time.
Based on these facts, there is little doubt that the judge will mandate Dr. Daniels to some sort of psychiat-
ric treatment. Dr. Daniels is highly unlikely to be permitted to return to psychiatric practice immediately
without some sort of treatment mandate. Dr. Daniels will not be classified as a sex offender because he was

435

Psychiatry Test Preparation and Review


Manual E-Book
Psychiatry Test Preparation & Review Manual

found not guilty of these charges, albeit by reason of psychiatric incapacitation. The decision to remove or
maintain the doctor’s license comes not from the court but from the state licensing department, who upon
reviewing the facts of the case will determine the destiny of the doctor’s practice privileges. It is absolutely
possible (though it may come as a shock) that the state licensing department could allow the doctor to
return to practice after treatment is successfully completed, despite the nature of these charges.
K&S Chapter 36

Vignette Eight
1. D. Thought process refers to the form of the patient’s thoughts. Descriptions of thought process would
include the terms goal-directed, linear, circumstantial, tangential, loosening of associations, flight of ideas,
thought blocking, neologisms, racing thoughts, or word salad. Thought content includes delusions, preoc-
cupations, obsessions, compulsions, phobias, hypochondriacal symptoms, or antisocial urges. Perceptions
primarily refers to hallucinations, be they auditory, visual, tactile, olfactory, or gustatory. In this case the
only statement that is both correct and accurate is that the perceptions section of the mental status exami-
nation should include auditory hallucinations.
K&S Chapter 7

2. A. The beliefs that Steven is having qualify as delusions. A delusion is a fixed false belief that is not supported
by social norms. They are listed under the thought content section of the mental status examination.
K&S Chapter 7

3. B. The patient statement contains the phrases “drop to the kimnet” and “fly with the glaspin.” As such the
correct answer is glossolalia, which is unintelligible jargon that has meaning to the speaker but not the
listener. Glossolalia is common in schizophrenia. Logorrhea is copious, pressured, coherent speech and
uncontrollable excessive talking as is often seen in mania. Laconic speech is characterized by a reduction in
the quality of spontaneous speech. Replies to questions are brief, and no unprompted additional informa-
tion is provided. Verbigeration is the meaningless and stereotyped repetition of words or phrases.
K&S Chapter 7

4. BD. Concentration may be impaired in several psychiatric disorders. It is tested by asking the patient to start at
100 and count backward by 7’s or by asking the patient to spell the word “world” backward and forward.
Attitude in the mental status examination describes the patient’s attitude toward the examiner. Words com-
monly used include cooperative, friendly, interested, seductive, defensive, hostile, evasive, apathetic, ingra-
tiating, and guarded. Memory would be tested by giving the patient three words to remember and then
coming back to them in 5 minutes to see if he or she remembered them. Abstract thinking is tested through
the use of proverbs. It is a reflection of the patient’s ability to handle concepts.
K&S Chapter 5

5. B. A delusion of grandeur is a person’s exaggerated conception of his or her importance, identity, or power.
Pseudologia phantastica is a type of lying in which a person appears to believe in the reality of his or her
fantasies and acts on them. Algophobia is the dread of pain. A nihilistic delusion is a false feeling that the
self or others are nonexistent or that the world is coming to an end.
K&S Chapter 7

6. D. The mesocortical pathway, which is responsible for the negative symptoms of schizophrenia, begins at the
ventral tegmental area and extends to the frontal lobes.
K&S Chapter 7

7. ABD. Positive symptoms in schizophrenia are most associated with frequency of hospitalization but are not good
predictors of long-term functional outcome. Cognitive symptoms have the strongest correlation with long-
term functional outcome. Schizophrenia is associated with a 10% suicide rate. Most schizophrenics who
commit suicide do so in the first few years of their illness and are therefore young.
K&S Chapter 7

436

Psychiatry Test Preparation and Review


Manual E-Book
Vignettes

8. BCD. The presence of schizophrenia-like symptoms for only a 2-month period would rightly be diagnosed as
schizophreniform disorder. Schizophreniform disorder is characterized by the presence of delusions, hal-
lucinations, disorganized speech, disorganized or catatonic behavior, or negative symptoms, which last
between 1 and 6 months. Once the symptoms last for more than 6 months, the diagnosis changes to
schizophrenia. In a case in which the symptoms were present for less than 1 month the correct diagnosis
would be brief psychotic disorder. Pressured speech, racing thoughts, and excessive energy would be more
consistent with mania found in bipolar disorder than with schizophrenia. Phencyclidine use during symp-
tom onset would present a very clear cause of the symptoms and would be appropriately diagnosed as
substance-induced psychotic disorder, with onset during intoxication. The prevalence of diabetes in people
with schizophrenia is twice as high as in the general population.
Psychotic Disorders
K&S Chapter 7

9. ABCD. All of the choices given except bright clothing are commonly seen in schizophrenics. Lack of spontaneous
movement, odd stiffness or clumsiness, echopraxia (imitation of posture or behavior of the examiner), agi-
tation, and bizarre posture are all possible in addition to tics and stereotypies.
K&S Chapter 7

10. ACD. Thought form (aka thought process) of schizophrenic patients can include verbigeration (meaningless rep-
etition of specific words or phrases), word salad (incomprehensible connection of thoughts with loss of
normal grammatical structure), and mutism (voicelessness without any physical impediments of speech).
Ideas of reference are part of thought content, not thought form.
K&S Chapter 7

Vignette Nine
1. ABC. Anorexia nervosa can be divided into restricting type and binge-eating/purging type. Refusal to maintain
body weight above minimally normal for height and age is an essential criterion. If BMI is ≥17 kg/m2, it
is considered mild. If BMI is 16 to 16.99 kg/m2, it is considered moderate. If BMI is 15 to 15.99 kg/m2, it
is considered severe. If BMI is <15 kg/m2, it is considered extreme. In addition the anorexic patient has an
intense fear of becoming fat, has a disturbance in the way his or her body is experienced, and presents with
amenorrhea (if female). Binge eating and purging can occur in either anorexia or bulimia.
K&S Chapter 15

2. ABCDE. Medical complications associated with anorexia include but are not limited to bradycardia, pancytopenia,
lanugo, osteopenia, metabolic encephalopathy, arrhythmias, elevated liver function test liver function tests
(LFT), elevated blood urea nitrogen (BUN), decreased T3 and T4, parotid gland enlargement, seizures, and
peripheral neuropathy.
K&S Chapter 15

3. ABC. Indications that anorexia should be managed on an inpatient basis include significant hypokalemia, weight
loss to under 75% of expected weight for height and age (a BMI of 15 kg/m2 is considered extreme),
growth arrest, risk of self-harm or development of psychosis, rapid weight loss, or the failure of outpatient
management.
K&S Chapter 15

4. B. BMI is calculated as weight (kg)/height (m)2. Other choices are just distractors and are nothing you should
memorize.
K&S Chapter 15

5. ABCD. Treatment for anorexia should address psychiatric, medical, and nutritional issues. Weight restoration
is a major goal. Psychopharmacology can include the use of both antipsychotics and antidepressants.
Psychotherapy and family therapy are important, and there is evidence for the use of CBT in all eating
disorders, especially bulimia. The most effective psychotherapy focuses on helping the patient develop

437

Psychiatry Test Preparation and Review


Manual E-Book
Psychiatry Test Preparation & Review Manual

alternative coping strategies and defenses, as well as changing problematic eating behaviors. Bupropion
should be avoided in patients with eating disorders because it lowers the seizure threshold, which can
increase risk of seizures during periods of electrolyte disturbances, which eating disorder patients are
prone to.
K&S Chapter 15

6. ABC. Complications of self-induced vomiting found in anorexia or bulimia can include esophagitis, scars and
abrasions on the back of the hand (Russell’s sign), Mallory–Weiss syndrome (bleeding from tears in the
esophageal mucosa caused by repetitive retching), Barrett’s esophagus, erosion of tooth enamel, parotid
gland swelling, increased serum amylase, hypokalemia, and an increased rate of spontaneous abortion
and low birth weight during pregnancy. Atonic colon can be found in anorexics but is a result of laxative
abuse, not self-induced vomiting.
K&S Chapter 15

7. ACD. Abuse of ipecac syrup can lead to skeletal muscle atrophy, prolonged QTc interval, cardiomyopathy, and
tachycardia. Rectal prolapse can be seen in eating disorders after severe laxative abuse.
K&S Chapter 15

8. AD. The risk of anorexia is higher in families that contain anorexics. Anorexia is unrelated to paranoid per-
sonality disorder. Eating disorder patients tend toward personality traits that are rigid and perfectionistic.
They tend to be emotionally inflexible. Adolescence is a time of heightened risk. Patients may control eat-
ing as a reaction to other changes in their lives that are outside of their control.
K&S Chapter 15

9. ABCD. Disorders that may be misdiagnosed as an eating disorder, or vice versa, include all listed in this question,
except brief psychotic disorder. In MDD, weight loss often accompanies loss of appetite, and sometimes
severe weight loss. A thorough evaluation for symptoms of MDD should be done, including an under-
standing of how the patient feels about the weight loss. In MDD the patient is not afraid of gaining weight.
In anxiety disorders, patients may lose weight owing to changes in appetite due to anxiety or issues sur-
rounding obsessions. These patients should be screened carefully for areas in which anxiety may impede
normal eating. Bulimia may be misdiagnosed as anorexia, especially in cases of binge-eating/purging
anorexia. Keep the patient’s body weight in mind as an important marker. Below minimally expected body
weight (<17 kg/m2) is considered anorexia, whether the patient is purging or not. Substance abuse can
often come along with severe weight loss, as patients do not eat properly and become malnourished. These
patients do not fear gaining weight as an anorexic does.
K&S Chapter 15

10. BCD. There are a whole range of medical considerations that come along with anorexia. Combinations of estro-
gen and progesterone have not been shown to be successful at reversing osteopenia in anorexia. Dental
follow-up is essential, particularly for those who purge, because the stomach acid eats away at tooth
enamel over time and greatly increases risk of caries and tooth decay. ECG is important, as many anorexics
develop hypokalemia leading to arrhythmias as well as changes in QTc interval. Correcting hypokalemia is
necessary to prevent significant cardiac issues. Checking all electrolytes, administering vitamin supplemen-
tation, checking a CBC, and evaluation for severity of bone loss due to osteopenia are some of the other
measures that should be considered.
K&S Chapter 15

Vignette Ten
1. ACD. Lisa clearly has some form of anxiety disorder based on the symptoms given in the vignette. As such, panic
disorder, social phobia, and generalized anxiety disorder (GAD) should be included in a differential diag-
nosis, and more questions should be asked to better determine the correct diagnosis. There is no mention
of psychosis in the vignette, so schizoaffective disorder should not be included.
K&S Chapter 9

438

Psychiatry Test Preparation and Review


Manual E-Book
Vignettes

2. ABC. Based on Lisa’s history the most likely diagnosis is social anxiety disorder. First-line pharmacotherapy con-
sists of SSRIs and SNRIs. Benzodiazepines can also be very effective at decreasing anxiety when the patient
has to function in a specific social situation. Lisa has already been tried on sertraline with poor results.
However, we don’t know what dosage was tried and for how long she was on the medication. She has
also tried bupropion, which tends to be very activating and can make anxiety worse. Our best bet would
be to return to first-line treatments and make sure they are given adequate therapeutic trials. Paroxetine
and citalopram would both be considered first line. Clonazepam is a very reasonable add-on to one of
these medications to control acute anxiety in specific social situations. Bupropion should be avoided for its
potential to make anxiety worse.
K&S Chapter 9

3. A. CBT has solid evidence behind its use in social anxiety disorder as well as other anxiety disorders. The
other answer choices do not. CBT should be considered the first-line psychotherapy for social anxiety
disorder.
K&S Chapter 9

4. AB. Social anxiety disorder can overlap with or easily be misdiagnosed as schizoid personality disorder or
avoidant personality disorder. Important to keep in mind is that in social phobia the patient fears embar-
rassment in social situations. In avoidant personality disorder the person fears rejection in relationships.
In schizoid personality disorder the patient does not desire close relationships and is very happy without
them.
K&S Chapter 9

5. C. Patients with social phobia can demonstrate avoidance of public restrooms. Their fear is not a specific pho-
bia of the sink, toilet, or room. Their fear is of being embarrassed if someone hears, sees, or smells them
using the bathroom. As such it is a form of social phobia.
K&S Chapter 9

6. C. The performance-only subtype of social phobia is performance anxiety. Successful treatment consists of
β-adrenergic antagonists such as propranolol. These will decrease the physical manifestations of the anxi-
ety. Always keep in mind that β-blockers are contraindicated in asthma because of their ability to cause
bronchoconstriction. As such, if given an asthmatic with performance anxiety one could choose a low-dose
benzodiazepine or SSRI (which would be considered second-line agents for social phobia), but don’t give
them a β-blocker. Also be careful with benzodiazepine doses if the patient has to speak publicly because of
cognitive impairment.
K&S Chapter 9

7. D. Fear of embarrassment or scrutiny by others is the major fear of those with social phobia. It is
present in all situations that they fear, whether it is using a public restroom or talking at a party.
Fear of rejection is found most prominently in those with avoidant personality disorder. They
often don’t form relationships for fear of rejection. The need for someone to be with the patient in
stressful or anxiety-producing situations is a part of agoraphobia. The example is the person who
won’t leave his or her front gate without a friend or family member with him or her. Having some-
one with them doesn’t help avoidant or social phobia patients. Avoidance of relationships is most
characteristic of the schizoid personality disorder patient, who neither has nor seeks close rela-
tionships. He or she is a loner and is happy that way. One could argue that avoiding relationships
could also describe the avoidant patient, but the underlying motivation is different. The avoid-
ant patient wants relationships but is afraid of rejection. The schizoid patient doesn’t want them
at all.
K&S Chapter 9

8. D. According to DSM 5 criteria the symptoms of social anxiety disorder must last for 6 months before the
diagnosis is made.
K&S Chapter 9

439

Psychiatry Test Preparation and Review


Manual E-Book
Psychiatry Test Preparation & Review Manual

9. ABC. Blushing, dry mouth, and sweating are all commonly seen in social phobia, as are muscle twitching and
anxiety over scrutiny and embarrassment. Fear of dying is a more severe symptom, which is seen in panic
attacks, as would be dizziness and a sense of suffocation. Panic attacks can co-occur with social phobia but
should be diagnosed as such if present. They are not a necessary part of the social phobia picture.
K&S Chapter 9

10. A. As many as one-third of patients with social anxiety disorder also meet criteria for MDD. Many social phobia
patients also have alcohol problems. This makes sense when you think about the availability of alcohol in social
situations and its ability to take the edge off their anxiety and allow them to better tolerate social interaction.
K&S Chapter 9

Vignette Eleven
1. AB. Of the choices given, the most likely to be in Carl’s differential are sleep apnea and MDD. Carl is experienc-
ing decreased energy, fatigue, decreased concentration, and poor sleep. Sleeping longer hours is not making
him feel rested or better. Both of these diagnoses can present this way. The warning signs suggesting sleep
apnea are Carl’s obesity and the snoring, which drove his wife out of their bedroom. Sleep apnea can include
morning headaches, long pauses without breathing during sleep, and waking up gasping for air. Because of
the effect on mood and irritability, it can masquerade as a psychiatric disorder and should be considered in
overweight people who present looking depressed. Treatment for the disorder is a CPAP machine.
Klein–Levin syndrome is a rare condition characterized by hypersomnia and hyperphagia. These patients
can have mood changes but also show confusion, lack of sexual inhibitions, hallucinations, disorientation,
memory impairment, and incoherent speech. Periods of excessive sleeping can extend from days to months.
This is not the picture we are seeing with Carl.
Narcolepsy is a pattern of excessive daytime sleepiness, sleep attacks, cataplexy, hypnagogic/hypnopompic
hallucinations, and direct descent into REM sleep during sleep attacks. Sleep attacks are most common
during heightened emotional states.
K&S Chapter 16

2. AD. A workup for Carl should include thyroid function tests, as thyroid disorders can mimic mood disorders
and must be ruled out. Nocturnal polysomnography (sleep study) is the test of choice to diagnose sleep
apnea. A periodic limb movements of sleep test is not a real test. There is a periodic limb movements of
sleep disorder (restless legs syndrome). It is diagnosed with nocturnal polysomnography, but there is not
a special test with that name. CPAP is the machine used to treat sleep apnea. It does not diagnose sleep
apnea. Therefore it would not be used as part of a workup.
K&S Chapter 16

3. ABD. Possible complications of sleep apnea include decreased mood, increased risk of stroke, and cardiovascular
complications. Patients with obstructive sleep apnea may have large necks, but the sleep apnea doesn’t cause
their necks to get bigger. The big neck contributes to the collapse of airway when they’re asleep.
K&S Chapter 16

4. B. Sleep apnea is considered a dyssomnia. Dyssomnias are disorders relating to duration, quality, or timing of
sleep. They can lead to too much sleep or too little sleep. Examples would be narcolepsy, sleep apnea, and
circadian rhythm disorder. Parasomnias are disorders in which undesired behaviors occur during sleep or
sleep transitions. Examples would be nightmare disorder, sleep terrors, or sleepwalking (non-REM sleep
arousal disorder). First-line treatment for obstructive sleep apnea is CPAP machine. Modafinil has FDA
approval for narcolepsy, obstructive sleep apnea, and shift work disorder. Modafinil may be used in cases
where residual sleepiness exists despite treatment with CPAP machine. An increased risk for Parkinson’s
disease is seen in REM behavior disorder, not in sleep apnea.
K&S Chapter 16

5. D. The differential diagnosis for sleep apnea should include insufficient sleep, gastroesophageal reflux, and
nighttime panic attacks. Pavor nocturnus is another name for sleep terror disorder. It presents as sudden
arousal from sleep with behavior indicative of extreme fear. The patient may be awake but disoriented. He

440

Psychiatry Test Preparation and Review


Manual E-Book
Vignettes

or she won’t remember the event the next day. Somnambulism is another name for sleepwalking disorder.
It is more common in males. It can include retrograde amnesia as well as confusion. Treatment primarily
involves maintaining safety for the patient. Jactatio capitis nocturna is a rhythmic movement disorder that
includes head banging during sleep. Treatment involves preventing injury.
K&S Chapter 16

6. AD. In obstructive sleep apnea, airflow ceases during episodes, and respiratory effort increases. This is in con-
trast to central sleep apnea, in which airflow stops, and respiratory effort decreases. To be diagnosed with
sleep apnea, the apneic episodes must last for 10 seconds or longer and occur a minimum of 30 times per
night. In severe cases the patient may have as many as 300 episodes per night.
K&S Chapter 16

7. ABD. Complications of obstructive sleep apnea include pulmonary and cardiovascular death, arrhythmias, tran-
sient alterations in blood pressure during each episode, pulmonary hypertension, and over time, an increase
in systemic blood pressure, which may be mistaken for essential hypertension.
K&S Chapter 16

8. BC. REM sleep behavior disorder is a chronic progressive condition that is most often seen in men. There is a
loss of atonia during REM sleep, which leads to sometimes violent behaviors in which the patients act out
their dreams. Serious injury to the patients and those sleeping next to them is a risk. It tends to get worse
with the use of stimulants, tricyclics, and fluoxetine. Clonazepam and carbamazepine have proven effective
in decreasing the symptoms.
K&S Chapter 16

9. ABC. Sleep-related gastroesophageal reflux can cause awakening from sleep due to burning substernal pain or a
sense of tightness in the chest. Coughing and choking sensations can occur repeatedly. Despite all of these
symptoms, airflow is not impeded as it is in sleep apnea. However, given the symptoms, it is clear how this
could be mistaken for sleep apnea.
K&S Chapter 16

10. ABD. Sleep hygiene measures should be instituted in all patients with primary insomnia. They consist of aris-
ing at the same time each day, limiting daily time in bed, discontinuing any CNS-activating drugs such
as caffeine, avoiding daytime naps, exercising, avoiding evening stimulation, taking hot baths before bed,
avoiding regular meals near bedtime, practicing evening relaxation routines, and maintaining comfortable
sleeping conditions. The use of medication for sleep is considered treatment for insomnia but is not consid-
ered part of sleep hygiene.
K&S Chapter 16

Vignette Twelve
1. ABCE. Ryan is a complex case with a great deal of missing information. As such, there are several diagnoses to be
included on the differential. Schizophrenia should definitely be considered as a result of his command audi-
tory hallucinations and aggression history. Substance-induced psychotic disorder should be included, as he
has a significant alcohol history and we don’t have a urine toxicology on him, so we can’t say for sure that
he hasn’t been using other drugs. PTSD needs to be explored because he has a very clear trauma history.
He denies current flashbacks, but that is not enough data to rule out the diagnosis. Dementia NOS should
be included because he remembers only one of three objects on memory testing. This is clearly a poor
result, but we have no idea what is causing it. He clearly had a concussion/head trauma in the past, he
could have memory impairment due to chronic alcohol use, or his poor performance on the examination
could be a result of thought disorganization due to psychosis. His medical and cardiovascular risk factors
place him at higher risk for vascular dementia. We don’t have a clear understanding of his executive func-
tioning. We just don’t have enough information to know. As such, dementia NOS should be included until
more information can be collected and we can clearly understand the pattern of symptoms we are seeing.
The only disorder in the question that should not be included is GAD. The vignette does not mention a
single symptom of GAD.
K&S Chapters 21, 7, and 9

441

Psychiatry Test Preparation and Review


Manual E-Book
Psychiatry Test Preparation & Review Manual

2. ABCDE. A workup for Ryan should include, but is not limited to, the following items. Thyroid function tests to
rule out a biological cause for a mood disorder that could be presenting with psychotic symptoms. (We
don’t have enough information in the vignette to rule this out as a possibility at this point.) Thiamine
level should be included owing to the possibility of Wernicke–Korsakoff, given the picture of heavy alcohol
use and memory impairment. Head CT should be included to rule out a possible organic cause of hallu-
cinations and memory impairment, particularly with a history of head trauma. ECG should be included
because of the need to treat command auditory hallucinations (CAH) with antipsychotic medication and
the need to monitor cardiac status during this process. In addition, his hypertension, hypercholesterolemia,
and diabetes put him at higher risk for heart disease. Urine toxicology should be included to rule out the
possibility that other substances have been involved in this complex picture. The only one that should not
be considered is prolactin level. The patient is not on any medication that would elevate prolactin at this
point, and as such, there is no current need to monitor it.
K&S Chapters 21, 7, and 9

3. CD. If we were to treat Ryan with ziprasidone the most likely side effects would include sedation and cardiac effects.
Ziprasidone does not cause significant weight gain or extrapyramidal symptoms like some of the other antipsy-
chotics. The patient should be instructed to take ziprasidone with food for adequate absorption.
K&S Chapter 29

4. D. There is evidence that heavy cannabis use during early adolescence increases the relative risk of developing
schizophrenia to as much as six times that of the general population.
K&S Chapter 7

5. BD. Criteria A for schizophrenia are defined as two or more of the following for 1 month: delusions, hallucina-
tions, disorganized speech, disorganized behavior, and negative symptoms. Only one is needed if the initial
psychotic symptom is a bizarre delusion, voices commenting directly on behavior, or two voices convers-
ing. Psychosis lasting longer than a month that does not meet criteria A for schizophrenia can be either a
delusional disorder or an unspecified schizophrenia spectrum and other psychotic disorder. All of the other
diagnostic options would meet criteria A. Schizotypal personality disorder is characterized by bizarre, odd,
and magical thinking, but florid psychosis is not part of the picture. As such, it is not the right choice.
K&S Chapter 7

6. A. This question is important because it is antagonism at the 5-HT 2A receptor that makes the atypical anti-
psychotics atypical. It is through this mechanism that they help with negative symptoms rather than wors-
ening them. Haloperidol is a typical neuroleptic and has no action on the 5-HT 2A receptor. Aripiprazole
can be confusing because it is a partial agonist at D2 and 5-HT 1A but antagonizes 5-HT 2A.
K&S Chapter 29

7. CD. In delusional disorder, auditory or visual hallucinations are not present but tactile, olfactory, or gustatory
hallucinations may be if related to the patient’s delusions. There is no memory impairment as in dementia
(which may also present with psychosis). If the patient has delusional disorder somatic type, then unneces-
sary medical interventions are quite likely.
K&S Chapter 7

8. D. In brief psychotic disorder, symptoms last between 1 day and 1 month, after which the patient returns to
normal functioning. It may be characterized by hallucinations, delusions, paranoia, disorganized speech
and behavior, and even catatonia. Antipsychotics are used to treat the episode while it is ongoing. There are
no known primary preventative measures.
K&S Chapter 7

9. ABCD. All of the following statements are true. Tactile hallucinations are more common in medical and neurologic
conditions than in schizophrenia. Illusions are sensory misperceptions of actual stimuli. Delusions are fixed
false beliefs that are not supported by cultural norms. Word salad is the violation of basic rules of gram-
mar seen in severe thought disorder. Cataplexy is the sudden loss of muscle tone seen in narcolepsy attacks.
Catalepsy is synonymous with waxy flexibility.
K&S Chapter 7

442

Psychiatry Test Preparation and Review


Manual E-Book
Vignettes

10. ABCD. The differential diagnosis for new-onset psychosis should include many medical conditions. Among them
are systemic lupus erythematosus, temporal lobe epilepsy, neurosyphilis, Wilson’s disease, AIDS, B12 defi-
ciency, heavy metal poisoning, delirium, dementia, Huntington’s disease, pellagra, tumor, stroke or bleed,
herpes encephalitis, or autism. Phymatous rosacea is a skin disorder that has nothing to do with psychosis.
K&S Chapter 7

Vignette Thirteen
1. BE. This is a straightforward case with respect to initial early management. The predominant determinant of
your actions as an emergency room psychiatrist is based on the principles of maintaining staff and patient
safety and collecting more data before approaching more complex workup and disposition decisions. As an
emergency room psychiatrist, it is imperative to collect as much history and collateral information as possi-
ble. The primary source of that information in this case is from police and emergency medical technicians,
because there are no identified family members or collaterals from whom one might obtain information.
Once a proper history of events is obtained, further workup, management, and disposition planning can be
made.
Even before obtaining a history, though, the psychiatrist must ensure that the patient and staff in the emer-
gency room are safe. Safety of the patient and others comes before all in an acute or emergency room
setting. To this end, the psychiatrist’s first maneuver should be to have the agitated or aggressive patient
restrained and medicated so that no acute harm comes to the patient or the staff that are attending to him.
Haloperidol, with or without lorazepam, when given intramuscularly, is an excellent choice of manage-
ment. The initial dosage depends upon the age and body size of the individual, as well as the severity of the
agitation. An initial dose of haloperidol 5 mg intramuscularly can be repeated every 20 to 30 minutes (with
or without addition of lorazepam) until sedation ensues. With the advent of short-acting, atypical antipsy-
chotic agents, such as aripiprazole, ziprasidone, and olanzapine, strong consideration should be given to
these agents, given their superior safety and tolerability profile.
K&S Chapter 23

2. CD. There are many well-described risk factors for aggressive behavior in patients. Many major mental illnesses
predispose the patient to potential acting out and acts of violence directed against others. These conditions
include mental retardation, attention deficit hyperactivity disorder, conduct disorder, delirium, dementia,
psychotic disorders, mood disorders, intermittent explosive disorder, adjustment disorder with disturbance
of conduct, and cluster B personality disorders. The likelihood of violence against others increases during
periods of decompensation in major mental disorders. Acute use of alcohol or other substances of abuse
can also trigger acute dangerousness to others. The frequency of violence among males outweighs that in
females when it comes to homicide, assault and battery, or rape. With respect to domestic violence epi-
sodes, the rates of outward violence are about equal in both sexes. The most telltale predictor of violence
is, of course, a history of prior violent behavior. Being a battered, underprivileged child predisposes the
individual to a future likelihood of violent behavior. Low educational level, having poor family supports,
unstable housing, unemployment, and poor coping skills or a lack of resources for help can all be contribu-
tors to homicidality and violence. Medical problems, unless they involve acute or chronic physical pain, do
not typically predict violent acts of aggression in individuals.
K&S Chapter 23

3. ACD. The psychiatrist should attempt to conduct an assessment with a potentially violent patient in a way that pro-
motes containment of the behavior and limits the potential for harm. There are several steps that a psychia-
trist can take to try to minimize the patient’s agitation and potential risk. The interview should be conducted
in a calm, quiet, and nonstimulating area. Sufficient physical space should be available for both patient and
psychiatrist, with no physical barrier to leaving the room for either one. The psychiatrist should avoid any
kind of behavior that might be misconstrued by the patient as threatening, such as standing over the patient,
touching the patient, or staring at the patient. The psychiatrist should ask whether the patient is carrying
weapons but should not ask the patient to hand over any weapons. Assistance from security personnel, as
well as physical or chemical restraints, may be helpful if the psychiatrist deems these to be appropriate.
K&S Chapter 23

443

Psychiatry Test Preparation and Review


Manual E-Book
Psychiatry Test Preparation & Review Manual

4. BEF. The patient is voicing homicidal ideation and perhaps even intention directed at his mother. Psychiatrists
can be sued for failing to protect society from the violent acts of their patients if it was reasonable for
the psychiatrist to have known about the patient’s violent tendencies and if the psychiatrist could have
done something to safeguard the public. The landmark case of Tarasoff v Regents of the University of
California resulted in the California Supreme Court ruling that mental health professionals have a duty
to protect identifiable, endangered third parties from threats of imminent and serious harm made by their
outpatients. In this case, the psychiatrist is certainly acting quite responsibly in wanting to contact the
patient’s mother to inform her of the patient’s feelings and possible intentions of harm directed against her.
Of course, the psychiatrist in this case should not discharge the patient, particularly if the patient continues
to voice these feelings and intentions.
In this case, the most conservative course of action would be to admit the patient to the psychiatry unit,
most likely on an involuntary basis, given his lack of cooperation with you in the emergency room. This
would enable safety, stabilization, and further history-taking to take place simultaneously in a confined
clinical setting, with the highest level of care and clinical attention to the patient. It would not be wrong
to detain the patient overnight in the emergency room so that a social work consultation that is focused
on obtaining more detailed history and collateral information about the patient and also adding more
weight to the disposition plan for the patient could be obtained in the morning. There is no evidence in
this vignette that the police have intentions to arrest and charge the patient with a crime once he is psychi-
atrically cleared in the emergency room. Typically, police officers will inform the psychiatrist or the triage
nurse of this so that they can be contacted to pick the patient up upon discharge to bring the patient to the
police station for booking. Starting aripiprazole and divalproex sodium is preemptive and presumptive of a
psychotic or bipolar disorder that warrants ongoing medication management of this kind. There is no indi-
cation of any such diagnosis in this vignette, given that too little symptomatic information is presented.
K&S Chapter 36

5. AB. Given the patient’s refusal of oral medication, the psychiatrist should begin thinking about the need for
a long-acting, injectable antipsychotic agent to ensure the patient’s compliance and stability. Of the six
agents listed in answers A through F, only olanzapine and risperidone have long-acting injectable formula-
tions (Zyprexa Relprevv and Risperdal Consta). These would be the two agents of choice in this case, given
the current presentation of patient’s refusal to take oral medications on a voluntary basis. Olanzapine and
risperidone also come in oral disintegrating tablet formulations (Zydis and M-Tabs, respectively). This may
be an advantage to the psychiatrist, if the patient eventually agrees to take oral medication. The melting
tablets can be given by nursing staff while the patient is directly observed. This formulation prevents the
cheeking or spitting out of medication once it has been placed in the oral cavity. If the patient is not acutely
agitated, but refuses oral medication, a court petition for medication-over-objection may need to be pre-
sented to a judge, depending on the state.
K&S Chapters 36 and 29

6. BCD. In this case, the patient needs a higher level of outpatient mental health care than a simple outpatient clinic
setting. This stems from the fact that he has failed this level of care previously and has relapsed numerous
times in the past. He also has a history of noncompliance with medication and very poor insight and judg-
ment with respect to his mental illness and how to cope with it. The ACT team model exists in most states
in the United States. It is truly the highest level of outpatient mental health care because the treatment team
visits the patient at the patient’s residence multiple times every month. The team helps the patient with
medication monitoring and management and in many cases can prevent patient noncompliance that may
result in an acute decompensation. The program was developed in the 1970s by researchers in Madison,
Wisconsin. The patient is assigned to a treatment team that is composed of a psychiatrist, nurse, social
workers, case managers, and other possible interventionists. The team can provide care and clinical cover-
age virtually 24 hours a day, 7 days a week. The high staff-to-patient ratio (anywhere from 6 to 12 staff
members per patient) enables the ACT team to help the patient with a rich variety of case management
modalities and options.
Partial hospital programs are useful for refractory patients who are discharged from the hospital. These
programs are designed to be time-limited and are a step down from the acute inpatient setting for the
patient. These partial hospitals offer a less restrictive level of care for the chronic psychiatric patient, while
maintaining a well-structured framework that is close to that offered on an inpatient hospital unit. The

444

Psychiatry Test Preparation and Review


Manual E-Book
Vignettes

patient sleeps at home and comes to the partial hospital each day to attend programming and get physi-
cian and nursing care during the day, as well as case management and psychotherapy. Classically, patients
can spend up to 12 to 16 weeks in these programs after an acute hospitalization. They should then be dis-
charged to a less-restrictive outpatient level of care for ongoing maintenance care.
The continuing day treatment program for the chronically mentally ill is also an excellent choice for the
patient presented in this vignette. These programs run 5 to 7 days a week and provide a less structured,
though comprehensive, treatment modality for these patients. Patients sleep at home and attend the pro-
gram during the day, where they receive an array of therapies similar to that which is available in the par-
tial hospital setting. Key to this modality (and that of the partial hospital as well) is the fact that nursing
staff can administer oral and injectable medication on-site and while directly observing the patient. The
census of a day program is usually greater than that of a partial hospital program, because the psychiatrist
will usually see and evaluate the patient less often in the day treatment program setting.
K&S Chapter 17

Vignette Fourteen
1. DE. Attitude in the mental status examination describes the patient’s attitude toward the examiner. Words com-
monly used include cooperative, friendly, interested, seductive, defensive, hostile, evasive, apathetic, ingra-
tiating, and guarded. Based on the information in this vignette the patient is best described as cooperative
and friendly.
K&S Chapter 5

2. B. Affect is the patient’s present emotional responsiveness based on facial expressions and expressive behav-
ior. It may or may not be congruent with mood. Words used to describe affect include full range (within
normal range), constricted, blunted, and flat. Full range would be considered normal. Flat would be used to
describe a patient who is showing no signs of affective expression. A patient with constricted affect shows
less emotion than someone who is full range but more than someone who is blunted. Someone who is
blunted shows less emotion than someone who is constricted but more than someone who is flat.
K&S Chapter 5

3. DE. Thought process refers to the form of the patient’s thoughts. Examples of this would be goal directed,
linear, circumstantial, tangential, loosening of associations, flight of ideas, thought blocking, neologisms,
racing thoughts, or word salad. In this case, Robert’s thoughts are completely normal and as such, the best
description would be linear or goal directed.
K&S Chapter 5

4. A. Impulse control refers to the patient’s ability to control sexual, aggressive, and other impulses. It may be
determined based on recent history and the patient’s behavior during the interview. It is often rated as
good, fair, or poor.
K&S Chapter 5

5. A. Insight is the patient’s understanding and awareness of his or her own illness. It can be rated as good, fair,
or poor.
K&S Chapter 5

6. A. Judgment is reflection of patients’ capacity for good social judgment. What kinds of decisions are they
making? Do they understand the outcome of their behavior? Can they predict what they would do in
imaginary situations? It can be rated as good, fair, or poor.
K&S Chapter 5

7. A. Perceptions primarily refers to hallucinations, be they auditory, visual, tactile, olfactory, or gustatory. In
Robert’s case, there are none present. Choices B, C, D, and E may be true for Robert, but they were never
asked about in the vignette, and Robert never mentioned any of them. Therefore, we have no evidence of
whether he is experiencing them or not.
K&S Chapter 5

445

Psychiatry Test Preparation and Review


Manual E-Book
Psychiatry Test Preparation & Review Manual

8. A. Sialorrhea (excessive drooling) is a common and predictable side effect of clozapine. Robert and his doctor
should keep an eye on it, but it is not a reason to stop the medication if the patient can tolerate it. It may
get worse with a dosage increase.
K&S Chapter 5

9. C. The appropriate schedule for measuring ANC in a patient on clozapine would include a baseline ANC,
weekly ANC for the first 6 months, biweekly ANC for the next 6 months, and monthly ANC thereafter.
K&S Chapter 29

10. ABCDEFG. All of the tests listed except an echocardiogram would be appropriate to monitor in Robert over time.
ECG is important for anyone on antipsychotics, paying special attention to the QTc interval. Clozapine
is metabolized through the liver so LFTs are important. A clozapine level to ensure a therapeutic dose
is important. Fasting glucose, weight, waist circumference, triglycerides, and cholesterol are all important
parts of monitoring for metabolic syndrome, which we know clozapine has a high likelihood of causing.
K&S Chapter 29

Vignette Fifteen
1. AC. There are very few studies on dissociative identity disorder (DID), so very few epidemiological data exist
on the subject. Studies have pointed toward a female-to-male ratio of diagnosed cases as somewhere
between 5 and 10 to 1. The disorder is principally linked to severe early childhood trauma experiences,
usually abuse, neglect, or maltreatment. The rates of reported trauma in adult sufferers of DID range from
85% to 97%. Physical and sexual abuse are most frequently reported as the type of childhood trauma
sustained by sufferers of DID. Studies have not, as of this writing at any rate, demonstrated much in the
way of a genetic component to the disorder. About 70% of DID patients also meet criteria for PTSD. A
key DSM 5 criterion for DID is the inability of the patient to recall important personal information that
is too extensive to be explained by ordinary forgetfulness. About 40% to 60% of DID patients also meet
criteria for somatization disorder. Psychotherapy is a key component in the progress and recovery of the
DID patient. Those modalities that have been seen to be useful to DID patients include supportive psycho-
therapy, dynamic psychotherapy, cognitive therapy, and hypnotherapy.
K&S Chapter 12

2. BF. Group psychotherapy can be very useful in DID, but it is better conducted only with patients with this
disorder because of the tendency of other patients to be overly fascinated or frightened by these patients.
Prognosis of DID is poorer in patients with comorbid organic mental disorders, psychotic disorders,
and severe medical illnesses. Refractory substance abuse disorders and eating disorders are also thought
to worsen the prognosis of patients with DID. As far as appropriate medication choices are concerned,
patients with DID may benefit from a low-dose benzodiazepine, which may help diminish anxiety, hyper-
arousal, panic, and intrusive symptoms suffered by these patients. A proper substance abuse history should
be taken before starting any benzodiazepine, of course. Patients who are able to maintain a high level of
daily functioning tend to do better with this condition than those that are lower functioning.
K&S Chapter 12

3. D. There are many valid choices of medication to address the myriad of symptoms that may accompany DID.
Lithium is not one of them. It has no place as a mood stabilizer in DID, unless there is a concomitant
diagnosis of bipolar disorder. On the other hand the anticonvulsant and antipsychotic mood-stabilizing
agents are excellent choices for DID patients. These include divalproex sodium, lamotrigine, gabapentin,
topiramate, carbamazepine, risperidone, quetiapine, olanzapine, and ziprasidone. The SSRI class of antide-
pressants also has been shown to be efficacious in reducing the symptoms associated with DID. These of
course include fluoxetine, paroxetine, sertraline, citalopram, and escitalopram. These are useful for depres-
sive symptoms and mood lability, but may be less effective for the intrusive hyperarousal symptoms of
PTSD often associated with DID.
K&S Chapter 12

446

Psychiatry Test Preparation and Review


Manual E-Book
Vignettes

4. E. DID is conceptualized as a trauma spectrum disorder. Many patients with DID, up to 70%, meet criteria
for PTSD. About 40% to 60% of patients with DID also meet criteria for somatization disorder. Many
patients with DID also meet criteria for a mood disorder, in particular the depressive disorders. Frequent
rapid mood swings are also seen commonly in patients with DID. These mood swings are not believed to
be bipolar in nature but are believed to be posttraumatic and dissociative in nature and not true cyclic
mood disorder.
K&S Chapter 12

5. CE. Patients with DID must be ruled out for malingering, factitious disorder, conversion disorder, and somato-
form disorders. The following indicators may point a clinician away from DID as a diagnosis and toward
one of these psychosomatic disorders: increased symptoms when under observation, refusal to allow collat-
eral contacts, symptom exaggeration, outright lies, using fabricated symptoms to excuse antisocial behav-
ior, or a history of legal issues. Patients with veritable DID are generally conflicted, confused, ashamed,
and distressed by their symptoms. Patients with nongenuine DID often show little dysphoria about their
disorder.
K&S Chapter 12

Vignette Sixteen
1. CE. Mr. Moran is showing the signs and symptoms of progressive memory and executive function decline that
are compatible with a dementia. Dementia is defined as a progressive loss of cognitive functioning in the
presence of a clear sensorium. The basic faculties that are affected include memory, thinking, attention, and
comprehension. Dementia can be caused by various etiological factors and can be multifactorial. About
15% of demented patients have reversible illness if treatment is rendered before the development of irre-
versible damage. Mr. Moran’s presentation could well be that of a dementia of the Alzheimer type. He
has been confused and forgetful, which are cardinal features of Alzheimer’s dementia. He has also been
wandering aimlessly and cannot remember his way around familiar territory. This is essentially a visual
agnosia; he is presented with places that are familiar to him, but he does not remember them or recognize
them and he cannot negotiate his way around them. Part of the essential criteria for Alzheimer’s dementia
is the classic cognitive disturbances of aphasia, apraxia, agnosia, and disturbance in executive functioning.
A patient must have at least one of these four deficits, with marked memory impairment, to meet criteria
for dementia of the Alzheimer type. Mr. Moran also forgets the names of household items. This is both an
agnosia (he cannot name/recognize objects) and an apraxia (he has forgotten how to use certain common
items).
Now, Mr. Moran could certainly be suffering from a vascular dementia or a combination of a vascular
dementia with Alzheimer’s dementia. The differential diagnosis of vascular dementia presents itself in
this case by the patient’s medical history. The patient, we are told, has a history of hypertension, diabe-
tes, hyperlipidemia, and cardiovascular disease. He is a vasculopath and certainly these cardiovascular
risk factors are also stroke risk factors. He could be having repeated silent strokes that eventually begin
to compromise his cognitive functioning. The medical and neurologic examination and workup will help
you clarify which type of dementia is the more likely to be the culprit of his decline in functioning in this
case.
Note that there is no mention in this vignette of any manifestation of depression or anxiety, which make
these two answers incorrect in this case. As for the possibility of a dementia due to diffuse Lewy body dis-
ease or a frontotemporal dementia, these are very specific dementing entities that present with their own
specific set of symptoms and signs, none of which are really mentioned in this vignette. We urge you to
refer to other question explanations in this volume that elaborate on these two special types of dementias.
They do appear frequently on standardized examinations, and it is essential to know their features.
K&S Chapter 21

2. CFG. A medical and a neurologic workup are essential for this patient to help you clarify your differential diag-
nosis. You should be thinking about a dementia of the Alzheimer type, a vascular dementia, or a dementia
with features of both Alzheimer and vascular types. An outpatient brain imaging study is essential. The
MRI is best, because it affords the most anatomic detail, and its sensitivity to ischemic lesions is much

447

Psychiatry Test Preparation and Review


Manual E-Book
Psychiatry Test Preparation & Review Manual

greater than CT scan. A brain tumor is not completely out of the realm of possibility in this case, and the
MRI can be performed with gadolinium contrast administration to see if there is a presence of any enhanc-
ing space-occupying lesion that could be responsible for this patient’s cognitive impairment.
Neuropsychological consultation for a testing battery can be extremely valuable in a case such as this. A
thorough battery includes testing for cognitive and executive functioning and is highly sensitive to subtle
deficits in cognition, visuospatial abilities, and multitasking and reasoning skills. In the face of a nega-
tive or nonspecific medical workup, such a battery may lend weight to a subtle diagnosis of Alzheimer’s
dementia.
An EEG is not a high priority in this case. There is no evidence of seizure activity, or anything remotely
resembling ictal events, from the history given in this vignette. For such a test to be useful, a picture of
repeated discrete events that involve waxing and waning of attention and awareness should be a part of
the presenting history, which it is not in this case.
Starting medication can be useful for this patient, but which one(s)? There is no evidence of depressive
symptoms or sleep disturbance presented in this case history. Therefore sertraline and trazodone would
not be very helpful at this juncture. On the other hand, the case history points strongly in the direction of
a dementia, and donepezil would be an excellent choice to start, even before a workup is conducted and a
diagnosis solidified. Donepezil (Aricept) is FDA-approved for Alzheimer’s dementia, and it may have effi-
cacy in vascular dementia as well, though the FDA did not approve the agent for this indication. The dos-
ing is simple: 5 mg daily for the first month, increasing to 10 mg daily thereafter. Some evidence suggests
further titration to the 23-mg daily dose may be more beneficial in maintaining cognition than the former
10-mg dosing. Donepezil can always be stopped if the diagnosis of Alzheimer’s dementia is brought into
question once a workup has been completed on this particular patient.
K&S Chapter 21

3. BCF. This patient is getting to be too much to handle at home, without outside support, assistance, and guid-
ance. Soon, his daughter will become a patient herself, because her emotional and physical burden of car-
ing for the patient will cause caregiver burnout. Thus referring the patient’s daughter to caregiver support
programming and groups is a wise thing to do. Obtaining a visiting nurse consultation in the patient’s
home is also an excellent maneuver in this case. A home care nurse can assess the patient’s basic home care
needs and make appropriate recommendations to his physicians for these services. Ultimately, if around-
the-clock home care is not feasible for the patient and his family members, consideration will have to be
given to skilled nursing facility placement, if his condition continues to deteriorate, which we might expect
to happen in this particular case.
An ACT team referral is quite inappropriate in this case. Such teams are unable to provide the broad-based
medical, nursing, and home care that this patient will eventually need. ACT teams are based on a psychi-
atric model and are charged with the responsibility of helping mental health patients to maintain their
stability in the outpatient setting, despite a lack of insight and ability to do so themselves. Medical issues
that are deeply complex, such as in this case, are far too burdensome for ACT teams, which have only a
psychiatric nurse and a psychiatrist among their team members. Family members can certainly provide
coverage at home for this patient, if he begins to become home-bound. This, however, is not ideal, as fam-
ily members are not usually trained to this intense medical task, and they generally have to sacrifice their
own lives and well-being to tend to their relative. A subacute rehabilitation facility is hardly an appropriate
placement for this patient. Such facilities are geared toward patients who need intensive physical therapy
to restore their ambulatory functioning. These placements are deemed to be temporary, and the expecta-
tion is that these patients will recover functional ability and eventually make their way home, whenever
possible.
K&S Chapter 21

4. C. Alzheimer-type dementia has an average survival expectancy of about 8 years from the time of initial onset
of the disease. Studies typically point to a range from about 5 to 10 years of survival for these patients.
Note that more than 50% of nursing home beds in the United States are occupied by patients with
Alzheimer’s dementia.
K&S Chapter 21

448

Psychiatry Test Preparation and Review


Manual E-Book
Vignettes

5. ADG. Alzheimer’s dementia is treated with either an acetylcholinesterase inhibitor or an N-methyl-d-aspartate


(NMDA) antagonist or both together. The three acetylcholinesterase inhibitors currently in use in the
United States include donepezil, rivastigmine, and galantamine. The NMDA antagonist that is FDA-
approved for Alzheimer’s dementia is memantine.
With respect to the treatment and prevention of vascular dementia, the approach is to treat modifi-
able underlying risk factors for cerebrovascular disease. These measures include prescribing antiplate-
let aggregants such as aspirin, clopidogrel, or aspirin/dipyridamole combination therapy (Aggrenox).
Antihypertensive medication, antidiabetic medications, and insulin, as well as lipid-lowering medications,
all have a place in the treatment and prevention of further worsening of cerebrovascular disease. Smoking
cessation is ultimately a very important recommendation to patients in this regard as well.
The other agents mentioned in answer choices B, C, E, and F have no real place in the treatment of demen-
tia itself, though they may be useful in treatment-associated comorbid symptoms and problems.
K&S Chapter 21

6. F. Dementia is often accompanied by other devastating symptoms that are associated with cognitive decline.
Patients can often be seen to exhibit subtle or gross personality changes that are uncharacteristic for them
when at baseline. They may become depressed and unmotivated for self-care, even if they still have the
cognitive capacity to engage in this behavior. Aggression and violent verbal and physical outbursts can be
seen in moderate- to late-stage Alzheimer’s dementia. Sundowner syndrome can also be seen, which occurs
when the day–night cycle is disrupted by a lack of external light or lack of cues as to the date, season, and
time of day. Patients in this instance can become delirious, aggressive, agitated, and acutely more confused
as the day proceeds and may need sedation to abort such episodes. Such episodes may also be accompa-
nied by frank delusions and hallucinations. Mania, however, is not typically seen in dementing patients,
unless they have a history of bipolar or schizoaffective disorder with such episodes in the past.
K&S Chapter 21

7. DE. Benztropine (Cogentin) and diphenhydramine (Benadryl) are dangerous in the demented patient because
their anticholinergic properties can worsen cognitive impairment and cause greater confusion in these
patients. Patients exposed to these agents may also develop an anticholinergic delirium and may require
sedation due to agitation and disinhibition of their behavior. The rest of the medication choices listed are
all viable therapies for the patient presented in this vignette.
K&S Chapter 21

Vignette Seventeen
1. AE. Wanda, of course, is suffering from a delirium. Delirium is characterized by a waxing and waning of level
of consciousness and a cognitive impairment that evolves over a short period of time. Common associ-
ated psychiatric symptoms include mood disturbance, perceptual disturbance, and behavioral disturbance.
Delirium usually evolves rapidly over a period of hours or days. Irritability, impulsivity, anger, and rage
can all be seen at times within the constellation of symptoms that make up a delirium. Cognitive deficits
can manifest as memory and language impairment. There may be noted incoordination (apraxia) in certain
cases of delirium as well. Patients with delirium often have disturbance of sleep and/or of the sleep–wake
circadian cycle. Identification and reversal of underlying causes of delirium are the cornerstones of treat-
ment of the disorder.
K&S Chapter 21

2. ADE. Wanda is suffering from an acute delirium. Delirium is defined as an acute onset of waxing and waning
level of consciousness, with impairment in cognitive functioning. Recognizing a delirium is important so
that identifying the underlying causes can be undertaken and appropriate measures taken to treat and pre-
vent these causes. Delirium prevalence is highest in postcardiotomy patients. Some studies point to rates as
high as 90%. Advanced age is a risk factor for delirium, but generally in those over 70 years of age. About
30% to 40% of hospitalized patients over age 65 have an episode of delirium. Preexisting brain damage
or disease is a serious risk factor for delirium. This patient has MS and is in the midst of an acute exac-
erbation of that disease. This certainly predisposes her to an acute delirium. A history of alcohol abuse or

449

Psychiatry Test Preparation and Review


Manual E-Book
Psychiatry Test Preparation & Review Manual

tobacco smoking is also a risk factor for an acute delirium. Diabetes, cancer, blindness, and malnutrition
are also delirium risk factors. Hypertension, in and of itself, is not a risk factor for delirium. Male gender
is an independent risk factor for delirium. Intoxication or withdrawal from pharmacologic or toxic agents
is certainly a strong risk factor for delirium. The fact that Wanda is on interferon at home and methylpred-
nisolone in the hospital can predispose her to a delirium. Classic medications that can predispose patients
to delirium include narcotic painkillers, steroids, anesthetic agents, antineoplastic agents, anticholinergic
agents, antibiotics, antifungals, and antiviral agents.
K&S Chapter 21

3. D. The exact pathophysiology of delirium is not well understood. The major neuroanatomical area implicated in
delirium is the reticular formation. The reticular formation is the area of the brain stem responsible for regulation
of attention and arousal. The major pathway implicated in the etiology of delirium is the dorsal tegmental path-
way. This pathway projects from the mesencephalic reticular formation to the tectum and thalamus.
K&S Chapter 21

4. B. The complex pathophysiology of delirium is not well understood. Studies tend to point toward decreased
acetylcholine activity in the brain as a causative factor in delirium. The delirium associated with alco-
hol withdrawal has been associated with hyperactivity of noradrenergic neurons in the locus ceruleus.
Serotonin and glutamate have also been implicated in the pathophysiology of delirium.
K&S Chapter 21

5. C. The EEG characteristically shows generalized background slowing in delirium. Triphasic waves are mostly
specific for a hepatic encephalopathy and not for delirium of other etiologies. PLEDs is the characteristic
EEG finding in herpes simplex virus encephalitis. Temporal lobe spikes would be indicative of a seizure
focus in the temporal lobe. Hypsarrhythmia is an EEG pattern of high-amplitude waves and spikes on a
chaotic disorganized background that is characteristic of infantile spasms.
K&S Chapter 21

6. CE. Psychosis and insomnia are the two major symptoms of delirium that are likely to warrant medication
management. Haloperidol, a butyrophenone antipsychotic agent, is the most commonly used sedative
agent used in cases of hospital-based delirium. It can be given orally, intramuscularly, or intravenously,
though the intravenous route of administration is not FDA-approved. Despite that fact, intravenous halo-
peridol is a standard of care for this type of patient in an intensive care unit setting. Note that a moni-
tored bed is essential when giving intravenous haloperidol because of the risk of torsades de pointes, a
potentially fatal cardiac arrhythmia. Use of atypical antipsychotics such as risperidone, olanzapine, que-
tiapine, and aripiprazole is felt to be effective in delirium management, though there is a dearth of studies
to lend evidence to this management. Anticholinergic drugs like diphenhydramine and benztropine, along
with phenothiazine antipsychotics such as chlorpromazine, are very poor choices in delirium management.
Use of such drugs can prolong or even worsen the delirium, rather than improve it, and so they are to be
avoided in delirium management. Insomnia can be treated with short- or intermediate-half-life benzodi-
azepines such as lorazepam. However, any benzodiazepine can have a paradoxically agitating effect on
patients with delirium. Thus the use of these agents is generally reserved for patients with known alcohol-
withdrawal delirium (delirium tremens).
K&S Chapter 21

7. BEF. Even though the onset of delirium is usually acute and sudden in nature, premonitory or prodromal symp-
toms can be seen in the days (not months!) preceding the onset of florid symptoms. The symptoms of
delirium usually persist while causally relevant factors are present, though typical duration of episodes is
usually about 1 week. The more advanced the age of the patient and the longer the delirium episode lasts,
the longer the delirium usually takes to resolve. Delirium is known to increase patient mortality within the
first year after an episode. This is due mostly to the serious nature of the concomitant medical problems
that lead to the delirium. Controlled studies have not been conducted to demonstrate that delirium typi-
cally progresses to a dementia. Nevertheless, many clinicians believe this to be the case. It is, however, well
understood that periods of delirium may lead to symptoms of depression or PTSD in the aftermath of the
episode.
K&S Chapter 21

450

Psychiatry Test Preparation and Review


Manual E-Book
Vignettes

Vignette Eighteen
1. D. Allan is of course suffering from body dysmorphic disorder (BDD). The cause of this disorder is unknown.
There is a high comorbidity with depressive disorders. Patients with BDD often have family histories of
mood disorders and obsessive–compulsive disorder. In many patients with BDD, the symptoms are respon-
sive to serotonin-specific medications, which leads researchers to feel that the disorder itself is related to
imbalances in serotonin.
K&S Chapter 10.

2. C. All of the answers are valid psychodynamic explanations for adulthood neurosis. However, the most accu-
rate explanation of BDD symptoms falls under the defense mechanism of a displacement of sexual or emo-
tional conflict onto body parts unrelated to the issue. The defense mechanisms of repression, projection,
distortion, dissociation, and symbolization also are in play in this dynamic.
K&S Chapter 10.

3. AC. Data indicate that BDD typically has its initial onset between the ages of 15 and 30 years. In this regard,
Allan is quite typical. Women are more affected than men, though slightly. So in this regard, Allan would
be considered atypical. Most sufferers of BDD are unmarried, and in this regard Allan is among the major-
ity. The fact that Allan has never suffered a major depressive episode in the past is quite atypical of BDD.
There is a high comorbidity of BDD with mood and depressive disorders. One study found that 90% of
BDD sufferers had experienced a major depressive episode in their lifetimes. The fact that Allan is high-
functioning and spends a lot of money on himself and his appearance has no bearing on his BDD at all.
K&S Chapter 10.

4. ACE. Pharmacotherapy for BDD is usually approached first line with serotonergic agents such as SSRIs, MAOIs,
or tricyclic antidepressants. These serotonergic agents are typically effective at reducing symptoms in about
50% of patients. Augmentation with buspirone, lithium, methylphenidate, or an atypical antipsychotic is
appropriate if first-line therapy is ineffective.
K&S Chapter 10.

5. C. BDD usually begins during the adolescent years. The onset can be gradual or abrupt. The disorder usu-
ally has an undulating course with few symptom-free intervals. The part of the body on which concern
is focused may remain the same, or it may change over time. The preoccupation with imagined defects is
almost always associated with significant distress and impairment.
K&S Chapter 10

Vignette Nineteen
1. CEF. Of course, Grace meets DSM 5 criteria for a somatic symptom disorder and not for a conversion disorder
(functional neurological symptom disorder). Remember that a somatic symptom disorder begins before age
30, and symptoms persist over a period of years. The somatic symptom disorder diagnosis requires the pres-
ence of somatic symptoms and maladaptive thoughts, feelings, and behaviors related to those symptoms.
Conversion disorder (functional neurological symptom disorder) differentiates from somatic symptom dis-
order by manifesting as only a pseudoneurologic symptom or symptoms, which typically involve motor or
sensory functioning. Note that pain symptoms fall in a nebulous category that encompasses both the physical
and the neurologic and so could be a part of either a somatic symptom or a conversion disorder. Recall that
conversion and somatic symptom disorders are both disorders in which the patient’s symptoms are not being
voluntarily produced. They are manifested involuntarily as a product of psychological distress and interper-
sonal problems. Most often, these triggering conflicts are subconscious, though patients may at times be able
to identify stressors and triggers when asked about them. Another classic difference between these two dis-
orders is the way in which patients react emotionally to their symptoms. Somatic symptom disorder patients
often describe their symptoms in a more histrionic, emotional, and exaggerated fashion. Conversion disorder
patients typically display la belle indifference, which is simply an inappropriately cavalier attitude toward
serious symptoms and a marked unconcern regarding what may appear to be serious impairment.
K&S Chapter 13

451

Psychiatry Test Preparation and Review


Manual E-Book
Psychiatry Test Preparation & Review Manual

2. ACF. There are multiple etiological theories that attempt to explain the complexity of somatic symptom disor-
der. Certainly, psychosocial factors can play a great role in the presentation of this disorder. Many such
patients are found to originate from unstable homes with histories of physical abuse. Psychodynamically,
the symptom presentation may be conceptualized as a manifestation of repressed instinctual impulses.
Biological factors seem to point to a neuropsychological relation of somatization disorder to attention
deficit symptoms or disorders. Studies using evoked potentials have proposed a link between the somatic
symptoms and those of excessive distractibility, inability to habituate to repetitive stimuli, and lack of stim-
ulus selectivity. A small number of neuroimaging studies have demonstrated decreased frontal lobe and
nondominant hemisphere metabolism.
Genetic factors may also play a role in the etiology of somatic symptom disorder. The disorder tends to
run in families and occurs in approximately 10% to 20% of the first-degree female relatives of probands
of patients with somatic symptom disorder. One study demonstrated a 29% concordance rate of somatic
symptom disorder in monozygotic twins and 10% in dizygotic twins. Male relatives of women with
somatic symptom disorder show an increased risk of antisocial personality disorder and substance-related
disorders. The other answer choices have no proven relationship to the etiology of somatic symptom disor-
der and are merely nonsense distractors.
K&S Chapter 13

3. ACD. Women with somatic symptom disorder tend to outnumber men with the disorder by about 5 to 20 times.
The lifetime prevalence of the disorder in the general population is about 1% or 2%. Among general pri-
mary care patients, patients with somatic symptom disorder may number about 5% to 10%. The disorder
is inversely related to social position and occurs most often in patients with low income and little educa-
tion. Somatic symptom disorder must begin before age 30 and typically begins in the teenage years. Two
disorders that are not more commonly seen in patients with somatic symptom disorder include bipolar I
disorder and substance abuse.
K&S Chapter 13

4. CDE. Somatic symptom disorder is a chronic, undulating, and relapsing disorder that rarely remits completely.
Patients with the disorder are no more likely to develop another medical illness in the next 20 years than
patients who do not have somatic symptom disorder. A patient with somatic symptom disorder has about
an 80% chance of being diagnosed with the disorder 5 years later. It is unusual for patients with somatic
symptom disorder to be symptom-free for more than a year. The prognosis of the disorder overall is poor
to fair at best.
K&S Chapter 13

5. D. Both individual and group psychotherapy seem to be the most useful strategy for somatic symptom disor-
der, as these have been shown to reduce patients’ personal health expenditure by 50%. This is so because
psychotherapy helps decrease rates of hospitalization in these patients. Giving psychotropic medications to
these patients, even when somatic symptom disorder is accompanied by a mood or anxiety disorder, is a
risk. This is because these patients are notorious for marginal or poor compliance with such medications.
There are very few data to support the use of psychotropic medications in somatic symptom disorder when
it is not comorbid with other mental disorders.
K&S Chapter 13

Vignette Twenty
1. ACF. Pain and opioid use, when they coexist, make up one of the most delicate and difficult disease combina-
tions to treat effectively. This is even more complex when alcohol and cannabis use cloud and complicate
the clinical scenario. Detoxification of the patient off the painkillers, as well as alcohol and cannabis, on
an inpatient basis is certainly an excellent maneuver. Once the patient comes out of the inpatient facility,
a subsequent referral to pain management and perhaps physiatry would be ideal, because the patient will
certainly need to have his pain addressed at that point. Outpatient narcotic tapers typically don’t work
well for a number of reasons. The patient is not being monitored and so is being asked to use his judg-
ment and willpower to adjust the medication on his own. We already know that Kerry has a problem

452

Psychiatry Test Preparation and Review


Manual E-Book
Vignettes

with use of a variety of substances. Asking him to monitor his own home-based taper off narcotics is a bit
naïve on the part of any good clinician. There is no doubt that he will fail to manage such a taper appro-
priately, because of chronic pain and a lack of willpower and a predisposition to addictive behavior. Also,
giving Kerry naltrexone long-acting monthly injections (Vivitrol) is a pipe dream. This medication is FDA-
approved for both alcohol and opioid relapse prevention, but it has no analgesic properties whatsoever.
Naltrexone will certainly reduce the cravings for both narcotics and alcohol; however, one cannot take
narcotics with Vivitrol because, at best, they won’t work because they will be blocked by the antagonistic
effect of the Vivitrol and, at worst, the combination of narcotics with Vivitrol could induce a precipitated
withdrawal state. Guidelines dictate that at least 5 days must elapse between the last dose of a short-acting
narcotic (or 10 days if a long-acting narcotic is being taken) and the first dose of naltrexone.
Sending Kerry to a specialized pain clinic is an outstanding option. Pain specialists, who are neurologists,
psychiatrists, internists, anesthesiologists, and physiatrists, are well poised to put Kerry on a strict pain-
killer regimen that will be tailored to his abusive tendencies. Options for this include fentanyl transdermal
patch therapy or methadone for chronic pain. They will also recommend ancillary services like physical
therapy to help Kerry improve his condition and his pain.
Continuing Roxicodone is absolutely not an option in this case. Kerry will merely continue to overuse
and abuse this medication, which is a short-acting opioid narcotic useful for only short-term, acute treat-
ment of intense pain, such as postoperative pain. Disulfiram (Antabuse) cannot be given to Kerry until he
abstains from alcohol for several days to at least a week. Recall that Antabuse blocks aldehyde dehydro-
genase in the liver, which is the hepatic enzyme responsible for alcohol metabolism. Thus levels of acetal-
dehyde build up in the bloodstream. As such, if alcohol is consumed while the patient is on Antabuse, an
aversive reaction of nausea, vomiting, hypertension, headache, flushing, thirst, sweating, and dyspnea is the
result. For Antabuse to be started, Kerry must either be detoxed off the alcohol first or be abstinent of his
own accord to avoid this adverse reaction. The minimum safe duration off alcohol before starting to take
disulfiram is 12 hours.
Orthopedic and neurologic consultations are useful, but they won’t help Kerry in the present with his sub-
stance use issues or with his pain management. These are more pressing issues at the moment, and these
consultations can take place after both pain and addiction are directly addressed.
Methadone and acamprosate calcium (Campral) are excellent choices for outpatient management of
Kerry’s problems with narcotic painkillers and alcohol. Methadone is a long-acting synthetic narcotic that
can be prescribed lawfully in an outpatient office when it is given for chronic pain. When methadone is
prescribed only as opioid replacement therapy for narcotic addiction, it can be given only in a federally
licensed and regulated methadone clinic. Methadone is thought to be most effective for opioid relapse pre-
vention at or above doses of 60 mg daily. Campral is a wonderful medication for alcohol relapse pre-
vention. Campral effectively reduces cravings for alcohol consumption. Campral’s mechanism of action is
believed to involve the antagonism of glutamate and the NMDA receptor. For Campral to be effective, the
patient must have already stopped drinking alcohol for a short period, perhaps about 1 week.
K&S Chapter 29

2. C. The correct dosing strategy for disulfiram (Antabuse) is to start the patient on 500 mg daily for the first 1
to 2 weeks, and then lower the maintenance dose to 250 mg daily. Disulfiram should not be administered
until the person has abstained from alcohol for at least 12 hours.
K&S Chapter 29

3. BCE. Suboxone (buprenorphine/naloxone) is a sublingual tablet or disintegrating film strip that is rapidly
absorbed under the tongue. The unique property of buprenorphine is that it is both an agonist and an
antagonist at the μ opiate receptor site. This property makes buprenorphine a very modest painkiller,
and it also renders the medication very difficult to abuse. At doses above 40 mg daily, buprenorphine
gates the μ opiate receptor, and no further agonistic effects occur. This essentially breaks up most of the
medication’s potential for building tolerance and inducing euphoria. Also, if full opioid agonists are con-
sumed in conjunction with Suboxone, the user may well experience a disturbing precipitated withdrawal
syndrome.

453

Psychiatry Test Preparation and Review


Manual E-Book
Psychiatry Test Preparation & Review Manual

The naloxone content of Suboxone is a fail-safe mechanism to prevent the diversion of the product by
intravenous opiate users for the purposes of self-injection for recreational purposes. When absorbed by the
two large veins under the tongue, the naloxone has no real clinical effect; however, if the product is emulsi-
fied and injected intravenously, the naloxone component of the medication acts as a potent and rapid opi-
oid antagonist and abusers can experience a significant precipitated withdrawal if any attempt is made to
inject it.
Suboxone tablets and film strips must be taken under the tongue to be fully effective. Typical maintenance
doses for opioid relapse prevention range from 4 to 16 mg daily.
K&S Chapter 29

4. ABC. Among the agents that can decrease methadone blood levels are phenytoin, Hypericum, dextrometho-
rphan, abacavir, carbamazepine, cocaine, dexamethasone, nevirapine, rifampin, spironolactone, and
tobacco products. Among the agents that can increase methadone blood levels are ciprofloxacin, erythro-
mycin, disulfiram, verapamil, dihydroergotamine, grapefruit, moclobemide, and Echinacea.
K&S Chapter 29

5. D. Cannabis intoxication produces memory impairment, perceptual distortions, decreased problem-solving


ability, loss of coordination, increased heart rate, anxiety, and panic attacks. Abrupt cessation of canna-
bis after prolonged heavy use may cause a characteristic withdrawal syndrome that encompasses insom-
nia, irritability, drug craving, restlessness, depressed mood, nervousness, and anxiety. This can be followed
by anxiety, nausea, tremors, muscle twitches, sweating, myalgia, and general malaise. Typically, the with-
drawal syndrome begins about 24 hours after the last use, peaks at about 2 to 4 days, and diminishes after
about 2 weeks.
K&S Chapters 29 and 31.

454

Psychiatry Test Preparation and Review


Manual E-Book
Topic Index

A Diagnostic and Treatment Procedures in


Anxiety Disorders Psychiatry
Test I: 77, 119 Test I: 38, 68
Test II: 57, 69 Test II: 13, 23, 75, 83, 136
Test III: 15, 31, 37 Test III: 107, 117, 122, 133, 145
Test IV: 15, 31, 111 Test IV: 115
Test V: 95, 110, 137 Test V: 31, 33, 93, 94, 117, 118, 129
Test VI: 17, 39, 95, 107, 109, 132, 133, 135, 145 Test VI: 84
Vignette 10 Disruptive, Impulse Control disorders, Conduct Disorders,
Video Vignette 6 and ADHD
Test I: 120, 140
B Test II: 47, 114
Basic Neuroscience Test III: 6, 39, 43, 45, 47, 110
Test I: 24, 41, 46, 93, 96, 97, 99 Test IV: 49, 57, 148
Test II: 15, 33, 35, 37, 44, 91, 95, 97, 117, Test V: none
125, 127 Test VI: 91, 96, 97
Test III: 13, 29, 44, 50, 69, 78, 80, 81, 93 Video Vignette 8, 12
Test IV: 3, 25, 41, 71, 76, 102, 108, 134, 144 Dissociative Disorders
Test V: 1, 106 Test I: 128, 130
Test VI: 43 Test II: none
Bipolar Disorders Test III: 33, 118
Test I: none Test IV: none
Test II: 71, 73 Test V: none
Test III: 134 Test VI: 92
Test IV: 35, 114 Vignette 15
Test V: 13
Test VI: 142, 143 E
Vignette 3 Elimination Disorders
Video Vignette 5 Test I: none
Test II: 124, 150
C Test III: none
Cultural Issues in Psychiatry Test IV: none
Test I: 39 Test V: none
Test II: none Test VI: 94
Test III: 41, 150 Ethics
Test IV: 139 Test I: 19, 139
Test V: none Test II: 109, 111
Test VI: 19 Test III: 87, 103
Test IV: 75
D Test V: 21, 92, 103, 116, 146
Depressive Disorders Test VI: 7, 112, 126, 128
Test I: 34, 137 Vignette 7
Test II: 17, 53, 54, 55, 77, 81, 104,
105, 140 F
Test III: 35, 42, 146, 147 Feeding and Eating Disorders
Test IV: 47, 69 Test I: 74, 110, 134
Test V: 63 Test II: none
Test VI: none Test III: none
Video Vignette 7, 9 Test IV: 27, 29

455

Psychiatry Test Preparation and Review


Manual E-Book
Topic Index

Feeding and Eating Disorders (Continued) Vignette 16


Test V: none Vignette 17
Test VI: 23, 93, 98 Neurodevelopmental and Pervasive Developmental
Vignette 9 Disorders
Video Vignette 13 Test I: 42, 85
Forensic Psychiatry Test II: none
Test I: 30 Test III: 32, 54, 108
Test II: 65, 102, 106, 146 Test IV: 39, 55
Test III: 71, 101, 126 Test V: 29, 30, 78, 134
Test IV: 53, 67, 83 Test VI: none
Test V: 37, 41, 143, 149 Neurology
Test VI: 77, 103, 106, 119, 138 Test I: 3, 8, 13, 16, 18, 22, 27, 32, 35, 40, 43, 47, 48, 49,
Vignette 7 53, 54, 57, 60, 64, 78, 79, 92, 98, 103, 107, 115, 124,
125, 133, 143, 146
H Test II: 2, 4, 6, 8, 10, 12, 14, 16, 18, 20, 24, 26, 28, 30, 32,
History of Psychiatry 34, 36, 38, 40, 42, 46, 48, 50, 52, 56, 58, 60, 62, 64,
Test I: 55 68, 70, 72, 74, 76, 78, 80, 82, 84, 86, 88, 90, 94, 110,
Test II: none 112, 116, 118, 120
Test III: 25 Test III: 2, 4, 12, 14, 16, 18, 20, 28, 30, 34, 36, 38, 40, 52,
Test IV: 141 56, 58, 60, 62, 64, 66, 70, 72, 74, 76, 80, 84, 86, 88,
Test V: none 90, 92, 94, 98, 100
Test VI: none Test IV: 2, 4, 6, 8, 10, 12, 14, 18, 20, 22, 24, 26, 28, 30, 32,
Human Development 34, 36, 38, 40, 42, 44, 48, 50, 52, 54, 56, 58, 60, 66,
Test I: 2, 7, 14, 31, 83, 84, 86, 88 68, 70, 72, 74, 80, 82, 88, 90, 92, 94, 96, 98, 100, 106,
Test II: 1, 31, 89, 93, 119 118, 142
Test III: 1, 21, 67, 138 Test V: 2, 4, 8, 10, 12, 14, 16, 18, 19, 20, 22, 24, 26, 28,
Test IV: 1, 11, 78, 127 32, 34, 36, 38, 39, 40, 42, 44, 46, 48, 50, 51, 54, 56,
Test V: 15, 82, 84, 85, 86, 88, 107 57, 58, 59, 60, 62, 64, 66, 68, 69, 70, 72, 74, 76, 77,
Test VI: 27, 37, 51, 108, 118, 130 79, 80, 81
Test VI: 2, 6, 8, 10, 12, 14, 15, 16, 18, 20, 22, 24, 26, 28,
L 30, 32, 34, 38, 40, 42, 44, 46, 48, 50, 52, 54, 56, 58,
Laboratory Tests in Psychiatry 60, 61, 62, 63, 64, 65, 66, 67, 68, 69
Test I: 111, 113, 114, 117
Test II: 85, 135 O
Test III: 65, 79, 85, 123, 127, 149 Obsessive-Compulsive and Related Disorders
Test IV: 19, 124 Test I: 66
Test V: 27, 47, 65, 102, 111 Test II: 115, 142
Test VI: 23, 110, 111, 115, 131 Test III: 5, 112
Test IV: 45
M Test V: none
Management in Psychiatry Test VI: 72, 89, 116
Test I: 20, 144, 149 Vignette 18
Test II: 3, 63, 148
Test III: 8, 48, 123 P
Test IV: 16, 21, 65, 99 Paraphilic Disorders
Test V: 71, 108 Test I: 52
Test VI: 74, 85, 147 Test II: none
Vignette 13 Test III: none
Vignette 14 Test IV: none
Test V: none
N Test VI: 45
Neurocognitive Disorders Vignette 6
Test I: 4, 69, 112, 147 Video Vignette 3
Test II: 92, 130, 144 Personality Disorders
Test III: 9, 49, 55, 68, 106 Test I: 63, 70, 118, 138
Test IV: 42, 126, 147 Test II: 59, 107, 143, 149
Test V: 6, 87, 142 Test III: 17, 53, 63, 73
Test VI: none Test IV: 33, 61, 149

456

Psychiatry Test Preparation and Review


Manual E-Book
Topic Index

Personality Disorders (Continued) Public Policy


Test V: 17, 140 Test I: none
Test VI: none Test II: 7, 11, 61, 123
Vignette 4 Test III: 124
Video Vignette 1 Test IV: none
Video Vignette 2 Test V: 138, 142, 144, 145
Video Vignette 4 Test VI: 5, 86
Poisoning
Test I: none S
Test II: none Sexual Disorders
Test III: 22, 24, 26 Test I: 76
Test IV: none Test II: none
Test V: 148 Test III: 51
Test VI: 80, 81, 100 Test IV: 79
Psychological Theory and Psychometric Testing Test V: 83
Test I: 17, 21, 26, 29, 101, 102 Test VI: 33, 104
Test II: 5, 39, 45, 67, 99, 101, 138 Sleep Wake Disorders
Test III: 83, 113, 135, 144 Test I: 23, 80, 148
Test IV: 43, 85, 91, 110, 116, 133, 135 Test II: 22, 145
Test V: 35, 89, 99, 127 Test III: 11, 77, 128, 140
Test VI: 29, 31, 99, 120, 121, 127 Test IV: 136, 146
Psychopharmacology Test V: 67, 125, 133
Test I: 1, 9, 10, 25, 28, 33, 37, 44, 50, 58, 59, 61, 62, 87, Test VI: 4, 82, 83
89, 91, 94, 95 Vignette 17
Test II: 25, 27, 29, 64, 66, 96, 100, 121, 122, 126, 133, Somatic Symptom Disorders/Psychosomatics
137, 141, 147 Test I: 11, 56, 81, 82, 122, 145
Test III: 3, 10, 46, 57, 59, 89, 91, 95, 96, 102, 104, 114, Test II: 19, 108, 134
115, 125, 129, 132, 139, 142, 143, 148 Test III: 82, 109, 120
Test IV: 77, 81, 86, 87, 89, 93, 97, 101, 103, 105, 107, Test IV: 17, 23, 46, 73, 84
112, 113, 121, 122, 123, 125, 128, 129, 130, 131, Test V: 5, 49, 52, 75, 97, 104, 139, 141
137, 138, 143, 145 Test VI: 36, 41, 49, 144, 146
Test V: 3, 7, 9, 27, 43, 55, 61, 73, 91, 96, 100, 101, 105, Vignette 19
109, 112, 114, 115, 120, 121, 123, 124, 126, 128, Video Vignette 10
130, 131, 135, 147, 150 Statistics
Test VI: 1, 3, 9, 11, 13, 21, 35, 55, 57, 59, 70, 71, 73, 76, Test I: 6, 15, 105, 106, 108, 109
78, 79, 101, 102, 110, 111, 117, 122, 131, 136, 141, Test II: 41, 87
148, 149, 150 Test III: 23, 61, 97, 99, 116, 137
Psychosocial Interventions Test IV: 5, 37
Test I: 132 Test V: 23, 90, 122
Test II: none Test VI: 105, 113
Test III: none Substance Related and Addictive Disorders
Test IV: none Test I: 90, 123, 127, 129, 131, 135, 141, 150
Test V: none Test II: 98, 132
Test VI: none Test III: 75, 105, 130
Psychotherapy Test IV: 7, 9, 13, 62, 64, 95, 104, 117, 120, 140
Test I: 12, 36, 45, 51, 100, 104, 142 Test V: 45, 53, 98, 113, 132
Test II: 43, 113, 128, 129, 131, 139 Test VI: 47, 53, 69, 125, 134, 137, 139, 140
Test III: 131, 141 Vignette 20
Test IV: 109, 119, 132, 150 Video Vignette 11
Test V: 119
Test VI: 75, 87, 88, 114, 123, 124, 125, 129 T
Psychotic Disorders Trauma and Stress Related Disorders
Test I: 5, 65, 67, 73, 75, 116, 121 Test I: 71, 72, 126, 136
Test II: 9, 21, 49, 51, 79, 103 Test II: none
Test III: 7, 19, 27, 111, 119, 121, 136 Test III: none
Test IV: 63 Test IV: 51, 59
Test V: 11, 136, 138 Test V: 25
Test VI: none Test VI: 25
Vignette 12 Vignette 1, 4

457

Psychiatry Test Preparation and Review


Manual E-Book
Index

A Acute intermittent porphyria, 140, 173,


α2 adrenergic agonism, opioid withdrawal mediation 325, 357
of, 147, 180 manifestation of, 280, 318
α1 Adrenergic receptors, blocking of, 284 symptoms, 173
α-Aminobutyric acid (GABA) Acute ischemic stroke
anxiety associated with, 9 aspirin for, 324, 354–355
α-Aminobutyric acid (GABA)-A receptor recombinant tissue plasminogen activator (r-TPA)
benzodiazepines in, 207 for, 64, 95
α-Amino-3-hydroxy-5-methyl-4-isoxazolepropionic Acute stress disorder, 25, 58, 131, 161
acid (AMPA), glutamate and, 80, 116, diagnosis of, 272, 310
198, 229 posttraumatic stress disorder vs., 133, 165
Abasia, 369 Addison’s disease, 264, 297
Abducens palsy, diabetes and, 328, 363 characteristic symptoms of, 264
Abilify. See Aripiprazole ADEM. See Acute disseminated encephalomyelitis
Abreaction, in cognitive therapy, 86, 124 (ADEM)
Absolute neutrophil count (ANC), clozapine and, 349, Adenylosuccinase deficiency, 121
388–389 Adenylyl cyclase, second messenger and, 79, 115
Abuse, child, 337, 376 ADHD. See Attention deficit hyperactivity disorder
ACA. See Anterior cerebral artery (ACA) (ADHD)
Acceptance, Bowlby’s stages of childhood attachment Adolescence
disorder and, 152 greatest psychological impact from deformity during,
Acetylcholine, 209, 245 17, 50
catabolism of, blocking, 6, 36 myoclonic epilepsy of, 362
receptor(s) suicidal rate, 154, 189
antibodies (AChR-ab), in myasthenia gravis, 284 Adopted siblings, alcoholism and, 23, 56
blocking of, 284 Adrenal insufficiency, symptoms of, 274, 310–311
Acetylcholinesterase, organophosphates inhibit, Adrenocorticotropic hormone (ACTH) stimulation
162–163 test, 297
Acquired immunodeficiency syndrome (AIDS) Advanced directives, 140–141, 173
CNS lymphoma in, 195, 224 Affect, 413, 444
dementia complex, 195, 225 Affective disorders, suicides and, 373
distal sensory polyneuropathy in, 225 Affirmation, 385
vacuolar myelopathy in, 196, 225 Aggression
Actus reus, 240, 382, 401, 434 cerebrospinal fluid serotonin level associated with,
Acute abortive therapy, of migraine headache, agents 216, 252
for, 263, 296 patient with, neurochemical changes
Acute benzodiazepine overdose, 168 in, 274, 312
Acute disseminated encephalomyelitis (ADEM), Agnosia, 236–237
220–221 Agoraphobia, 75, 110
Acute dystonia, definition of, 276, 313 Agraphia, alexia without
Acute inflammatory demyelinating polyneuropathy in arterial territories, 258, 289
(AIDP), 33, 115, 171. See also Guillain-Barré Gerstmann’s syndrome and, 7, 38
syndrome (GBS) Agyria. See Lissencephaly

Note: Page numbers followed by “f ” indicate figures, “t” indicate tables, and “b” indicate boxes.

458

Psychiatry Test Preparation and Review


Manual E-Book
Index

AIDP. See Acute inflammatory demyelinating Amnesia, 57


polyneuropathy (AIDP) anterograde, 208, 242–243
AIDS. See Acquired immunodeficiency syndrome dissociative, 57, 161
(AIDS) transient global, 128, 157, 323, 353
AIDS-related retinopathy, cytomegalovirus in, 66, 97 Amnestic syndrome, transient amnesia and, 353
Ailurophobia, 31 Amok, 8, 38
Akathisia, 160 Amoxapine, 138, 171, 256, 286
definition of, 313 causing parkinsonian symptoms, 149, 182
propranolol for, 212, 248 AMPA. See α-Amino-3-hydroxy-5-methyl-4-
Alcohol abuse isoxazolepropionic acid (AMPA)
lab tests for, 213, 249 Amphetamine
naltrexone for, 211, 247 intoxication, 58
Alcohol, for essential tremor, 359 side effects of, 192, 221
Alcohol intoxication, 235 Amygdala
Alcohol level, in blood, 262, 295 emotional memory and, 205, 238
Alcohol withdrawal Papez’ circuit and, 137, 169
carbamazepine for, 207, 241 Amygdaloid bodies, stroke to, 147, 180
symptoms, management of, 275, 313 Amylase test, 259, 291
Alcoholic dementia, 114 Amyloid plaques, 230
Alcoholic neuropathy, as complication of chronic Amyloid precursor protein (APP)
alcohol abuse, 78, 114 Alzheimer’s disease associated with, 21, 54
Alcoholic-nutritional cerebellar degeneration, 114 gene of, in chromosome, 21, 213, 249
Alcoholism Amyotrophic lateral sclerosis (ALS), 33–34, 166, 332,
adopted siblings and, 23, 56 367–368
Wernicke’s encephalopathy and, Analgesia, patient-controlled, 148, 181
326, 359 Analysis of variance (ANOVA), 163, 197, 227
Alexia, without agraphia usage of, 289
in arterial territories, 258, 289 ANC. See Absolute neutrophil count (ANC)
Gerstmann’s syndrome and, 7, 38 Anencephaly, 269, 303
Alexithymia, in posttraumatic stress Angelman’s syndrome, 303–304
disorder, 391, 426 Anger, Kübler-Ross stages of reaction and, 154, 189
Alogia, 152, 186 Anomic aphasia, 352
Alopecia, 164 Anorexia nervosa, 53
Alprazolam, 313 diagnosis of, 404, 436
ALS. See Amyotrophic lateral sclerosis (ALS) differential diagnosis for, 405, 437
Altruism, 82, 119 head CT scan and, 21, 54
Alzheimer’s disease hospital admission for, indications for, 404, 436
amyloid precursor protein and, 21, 54 medical complications associated
chromosome, 21 with, 404, 436
defects and, 9, 39 treatment for, 405, 436–437
dementia, 301 Anorgasmia, 327, 362, 420, 450
Parkinson’s disease and, 286–287 Anosognosia, 57, 221
prevalence of, 281, 319 ANOVA. See Analysis of variance (ANOVA)
Amaurosis fugax, 191 Anterior cerebral artery (ACA), 322, 353
Ambien. See Zolpidem ischemia, 219–220
Ambivalence, patient’s, as therapeutic focus of right, stroke in, 68, 68f, 100, 100f
motivational enhancement therapy, 19, 51 Anterior choroidal artery syndrome, 353
American Psychiatric Association, 401 Anterior horn cell degeneration, of central nervous
ethics guidelines of, 4 system, 4, 33–34
psychiatrist’s unethical behavior and, 204, 238, Anterior spinal artery syndrome, 297–298
322, 352 Anterograde amnesia, 208, 242–243
Amitriptyline, for gastric ulcer, 67, 99 Antiplatelet aggregant agent, 417, 448

459

Psychiatry Test Preparation and Review


Manual E-Book
Index

Antipsychotics Assertive community treatment, 64, 95


atypical, 67, 98–99 Astasia, 369
properties of, 272, 310 Astasia-abasia, 369
dopaminergic pathways associated with, 145, 178 Astereognosis, 221
in ECT, 292 Asterixis, clinical bedside and, 146, 179
for psychotropic medications, 271, 308–309 α-Synuclein, in Parkinson’s disease, 286–287
Anton’s syndrome, 194, 223 AT. See Ataxia-telangiectasia (AT)
symptoms of, 202, 234 Ataque de nervios, 135, 167–168
Anxiety disorder(s) Atarax. See Hydroxyzine
central serous chorioretinopathy in, 196, 226 Ataxia
generalized, 391, 426 as cerebra lesion, 164
DSM criteria for, 346, 386 in Miller-Fisher syndrome, 78, 114
in males and females, 275, 312 truncal, 133, 164
neurotransmitters associated with, 9, 40 with vertigo, carotid territory transient ischemic
prevalence of, 22, 55 attack or stroke and, 76, 111
specific phobia and, 324, 356 Wernicke’s encephalopathy and, 359
Apathy, cortical dementia and, 79, 115 Ataxia-telangiectasia (AT), 106, 292–293
Aphasia, 234, 236–237 Ataxic dysarthria. See Scanning speech
Aphemia, 204, 236–237 α-Tocopherol. See Vitamin E (α-tocopherol)
Apnea test, positive, brain death and, 332, 367 Atomoxetine, 158–159, 180, 278, 315
Apolipoprotein E4, in Alzheimer-type dementia, 13, 45 for attention deficit hyperactivity disorder with
Apoptosis, 106 Tourette’s syndrome, 145, 179
mediated by NMDA receptor complex, 136, 168 Atonia, 89
APP. See Amyloid precursor protein (APP) Atropine, 163
Apraxia, 234, 236–237 Attachment, 1, 30
Aprosodia, 221 between parents and children, 17, 49
Arachnoid (pacchionian) granulations, function of, 70, Attachment theory, 306
102 Attention deficit hyperactivity disorder (ADHD)
Archetypes, 340, 381 diagnosis of, 231
Arenaviruses, 244 symptoms of, 71, 103
Arginase deficiency, 121 Attitude, in mental status exam, 413, 444
Argyll Robertson pupils, neurological examination of, Attribution theory, 34
143, 177 Audiometry, for phonological disorders, 211, 247
Aripiprazole, 18, 32, 51, 159 Auscultation, in arteriovenous malformation, 207, 242
Armodafinil, 98 Autism, normal, Mahler’s stage of separation-
Arnold-Chiari type I malformation, 138, 138f, 170, individuation, 140, 173, 307
170f Autism spectrum disorder, diagnosis of, 198, 228
Arsenic poisoning, 132, 132f, 162 Autistic symptoms, neurologic cause of, 260, 292
Arterial territories, stroke in, 258, 289 Autoantibodies, against glutamic acid decarboxylase,
Arteriovenous malformation (AVM)auscultation in, stiffperson syndrome and, 334, 370
207, 242 Autonomy, 59, 276, 313
Arteritis, temporal, 263, 295–296 vs. shame and doubt, 92
serum sedimentation rate for, 11, 42 AVM. See Arteriovenous malformation (AVM)
Arylsulfatase A (ASA), metachromatic leukodystrophy Avoidant personality disorder, 15, 47, 139, 172,
and, 65, 96 280, 318, 406, 438
ASA. See Arylsulfatase A (ASA) in social phobia, 196–197, 226
Asenapine, 136, 168 Avoidant/restrictive food intake disorder, 21, 53
Aseptic meningitis, cerebrospinal fluid findings for,
131, 162 B
Aspergillus, in hypersensitivity pneumonitis, 210 Babinski’s sign, 38–39, 134, 166
Aspergillus, in hypersensitivity pneumonitis, 245–246 Baclofen, 145, 179
Aspirin, for acute ischemic stroke, 324, 354–355 for trigeminal neuralgia, 354

460

Psychiatry Test Preparation and Review


Manual E-Book
Index

Bacterial meningitis, 109, 162 Bipolar disorder (Continued)


Bálint’s syndrome, 201, 233, 352 prevalence of, 211, 247
Bannwarth’s syndrome, 227–228 suicides and, 335, 373
Basic metabolic panel, in carbamazepine, Bipolar I disorder, 111, 227, 391, 426
277, 314 rapid-cycling specifier in, 152, 187
Basilar migraine, 105 Bipolar II disorder, 197, 227, 394, 428–429
Basilar skull fracture, 145, 178 choice of medications for, factors influencing, 394,
“Battle sign,” 145, 178 429
β-Blockers, for akathisia, 354 diagnosis of, 257, 287
BDD. See Body dysmorphic disorder (BDD) lifetime prevalence of, 227
Beck, Aaron, cognitive therapy and, 52, 186 Blepharospasm, definition of, 313
Becker’s muscular dystrophy, 141, 174–175 Blood alcohol level, 262, 295
BECTS. See Benign childhood epilepsy with BMI. See Body mass index (BMI)
centrotemporal spikes (BECTS) Body dysmorphic disorder (BDD), 129, 158, 194, 223,
Bell’s palsy, 227–228, 237–238 419, 450
diabetes and, 363 course and prognosis of, 420, 450
diagnosis of, 265, 298 psychodynamic explanation of, 419, 450
stroke-related facial paresis vs., 192, 222 Body mass index (BMI), 404, 436
Beneficence, 26, 59, 82, 119 Bonding, 1, 30
Benign childhood epilepsy with centrotemporal spikes Borderline personality disorder, 22, 55, 131, 161,
(BECTS), 362 396b, 429–431
Benign familial neonatal convulsions, calcium channel dialectical behavioral therapy for, 7, 38, 338, 377
system defects and, 137, 169 inpatient hospitalization for, 27, 60
Benign paroxysmal positional vertigo, 135, 167 self-mutilation in, 12, 44
Benign prostatic hypertrophy, as side effect of suicides and, 209, 244
tricyclic Borrelia burgdorferi, 128, 157–158
antidepressant drugs, 86, 123 Bell’s palsy and, 227–228
Benton visual retention test, 344, 384 Boston diagnostic aphasia examination, 360
Benzodiazepine(s), 181–182, 275, 308–309, 312 Botulinum toxin
for delirious patient, 32 for focal and segmental dystonia, 327, 361
for impaired liver function, 210, 245 mechanism of action of, 330, 365
overdose of, acute, 168 poisoning, 201, 228–229, 233
in pregnancy category, 340, 382 type A, 71, 104
Benztropine, 188, 354, 417, 448 Botulism, symptoms of, 381
Bereavement, 134, 166 BPRS. See Brief Psychiatric Rating Scale (BPRS)
Bethanechol, 211, 247 Brachial plexopathy, 356
Bilateral acoustic schwannomas, neurofibromatosis Brachial plexus, 297–298
type 1 (von Recklinghausen’s disease) and, 27, 60 Bradycardia, 174
Bilateral leg weakness, in transverse myelitis, 191, Bradykinesia, 232
220–221 Brain death, criteria for, 332, 367
Bilateral VIIIth nerve tumor, 357t Brain-stem reflexes, absence of, brain death and, 367
Binet, Alfred, 125 Brain tumors, 368
Binge eating disorder, 340, 380–381 Brandt-Daroff exercises, 135, 167
Binswanger’s disease, clinical features of, 326, 358 Brexpiprazole, 146, 179
Biofeedback therapy, 150, 184 Brief Psychiatric Rating Scale (BPRS), 251, 315
Biological factors, in child’s temperament, 198, 230 Bright artificial light exposure, for seasonal affective
Bipolar and related disorder, other specified, 348, 388 disorder, 72, 105–106
Bipolar disorder, 76, 267, 394b, 430. See also Bipolar I British anti-Lewisite. See Dimercaprol
disorder; Bipolar II disorder Broca’s aphasia, 93, 289
in children, diagnosis of, 231 Bromocriptine, 354
with co-occurring substance disorders, 395, 429 Brown-Séquard syndrome, due to hemisection of
group therapy and CBT, 301 spinal cord, 68, 100

461

Psychiatry Test Preparation and Review


Manual E-Book
Index

Bulimia, 25, 58 Carotid artery


Buprenorphine, 188, 208, 242 dissection, 334, 371
Buprenorphine/naloxone tablets, 422, 452–453 occlusion, 197, 227
Bupropion, 179 territory, ischemia of, 191, 219–220
for depression and attention deficit hyperactivity Carpal tunnel syndrome, 355, 355f
disorder, 153, 187 Case-control study, 31
for obsessive-compulsive disorder, 85, 122 Catalepsy, as immobile position, 148, 182
Buspirone, serotonin, 1A and, 80, 116 Cataplexy, 98
Butyrophenone, 159 Catastrophizing, 307–308
Catatonia, 161–162, 182
C Catechol-O-methyltransferase, antiparkinsonian
Calcium channel system defects, neurologic disorders effects and, 143, 176
in, 137, 169 Catharsis, in group therapy setting, 335, 373
Calcium (Ca2+) ion, 85, 122–123 Caudate nucleus
elevated intracellular levels of, 136, 168 anomalies in, 194, 223
Campylobacter jejuni, 139, 171, 370 atrophy of, 230
Candida albicans, 245–246 neurons, Tourette’s syndrome and development of
Cannabis, 422, 451–452 tics and, 140, 173
intoxication, 212, 248 CDT. See Carbohydrate-deficient transferrin (CDT)
signs of, 264, 297 Central nervous system (CNS)
use of, 410, 441 cysticercosis, 78, 113
as widely abused recreational substance among U.S. lymphoma in, AIDS and, 195, 224
high school students, 151, 186 toxoplasmosis, AIDS and, 195, 225
Capacity, standards of, 384 tuberculosis of, 257, 288
Capgras’ syndrome, 176 Central serous chorioretinopathy, in anxiety disorder,
CAPS. See Clinician-Administered PTSD 196, 226
Scale (CAPS) Cerebellar hemispheric lesions, 164, 237–238
Capsulotomy, for obsessive-compulsive disorder, 335, Cerebellar hemorrhage, 334, 370
372–373 Cerebellar vermis lesion, 133, 164
Carbamazepine, 87, 125, 160, 243, 311–312, Cerebral aneurysmal rupture, 227
342, 383, 387 Cerebral aneurysm, risk in electroconvulsive shock
for alcohol withdrawal, 207, 241 therapy, 87, 124–125
basic metabolic panel and, 277, 314 Cerebral venous thrombosis (CVT), diagnosis of, 140,
clozapine efficacy and, 336, 374 173
complete blood count and, 21, 54, 336, 373 Cerebrospinal fluid (CSF)
erythromycin and, 214, 250, 264, 297 decreased, 5-hydroxyindoleacetic acid in, associated
for facial pain, 324, 354 with violence and aggression, 65, 88, 96,
metabolism of, 6, 35 125–126
serum level of, 208, 243 herpes simplex encephalitis and, 363
sodium imbalance and, 342, 383 high cell count with absent protein in, in acute
for trigeminal neuralgia, 11, 41 inflammatory demyelinating polyneuropathy, 79,
Carbamoyl phosphate synthase 115
deficiency, 121 opening pressure, lumbar puncture with, for
Carbidopa, mechanism of action of, in combination pseudotumor cerebri, 63, 94
agent carbidopa-levodopa, 73, 106 pleocytosis, Guillain-Barré syndrome and, 333, 370
Carbohydrate-deficient transferrin (CDT), in Ceruloplasmin, serum level of, in Wilson’s disease, 19,
Parkinson’s disease, 256, 286–287 51
Carbon monoxide, exposure to, 299 Cervical dystonia, 327f, 361, 361f
Carcinoid syndrome, 226 Cervical spondylosis, 331, 366
Carcinoma, dermatomyositis and, 13, 44 Chelation therapy, 320
Cardiac disease, in men Chiari malformations, 303
risk of, 268 Child guidance clinics, in United States, 216, 251

462

Psychiatry Test Preparation and Review


Manual E-Book
Index

Childhood incest, major depression and, 200, 231–232 Clostridium tetani, 228–229, 375
Children Clozapine, 279, 317
language and, 326, 359 agents contraindicated with, 336, 374
normal development of, 16, 49 carbamazepine and, 74, 108
sexual abuse in, 202, 234 white blood cell (WBC) count and, 151, 185, 275,
Cholinergic therapies, vascular dementia and, 89, 126 312
Cholinergic toxicity, 132, 162–163 Cluster headache, 42
Cholinesterase-inhibiting agent, 417, 448 treatment for, 268, 302
Cholinesterase inhibitors, 36 CMV. See Cytomegalovirus (CMV)
side effects, 36 CNS. See Central nervous system (CNS)
Chorea, 297, 369 Cobalamin. See Vitamin B12 (cobalamin)
Choreoathetosis, in Parkinson’s disease, 201, 232 Cocaine
Choroidal artery syndrome, anterior, 353 hypodopaminergic state of, 17, 50
Chromosome lowering seizure threshold with, 203, 235
dementia pugilistica and, 85, 122 overdose with, treatment of, 395, 429
migraine and, 77, 113 use of
Chromosome 22 brain and, 395, 429
NF2, 36, 60 potential sequelae of, 395, 429
Chromosome, 21, amyloid precursor protein gene in, Cogentin. See Benztropine
213, 249 Cognitive behavioral therapy, 217, 253, 406, 438
Chronic fatigue syndrome for anorexia nervosa, 405, 436–437
diagnosis of, 16, 49 for panic disorder, 341, 382
symptoms, 49 for posttraumatic stress disorder, 390, 425
treatment, 49 for social anxiety disorder, with SSRI, 346, 387
Chronic pain syndromes, 363–364 Cognitive distortions, 307–308
Cigarette smoking, in decreasing tricyclic depression and, 20, 52
antidepressant concentrations, 7, 37 interpersonal psychotherapy and, 83, 120
Cimetidine Cognitive dysfunction, 159
cytochrome P450 2D6, 328 Cohort study, 2, 31, 78, 114
increasing clozapine levels, 151, 186 Collaborative care
Cingulate gyrus, Papez’ circuit and, 137 mental disorders, treatment for, 85
Cingulotomy, for obsessive-compulsive disorder, 149, primary care setting, treatment for, 85
183, 335, 372–373 Collaborative care model, 136, 169
Circadian rhythm sleep-wake disorder, shift-work psychiatrist’s role in, 193, 223
type, 35 Collaborative care models, 93
Citalopram, 406, 438 Collective unconscious, 340, 381
CJD. See Creutzfeldt-Jakob disease (CJD) Colloid cyst of the third ventricle, 242, 242f
CK. See Creatine kinase (CK) Coma, brain death and, 367
Clarification in interview, 385, 398, 431 Comatose patients, head CT scan for, 204, 237–238
Classical conditioning, 35, 391, 426 Communication stage, for children, 359
Clinician-Administered PTSD Scale (CAPS), 315 Competence to stand trial, 81, 117–118, 401
Clites vs State case, 118 Complete blood count (CBC), carbamazepine and, 21,
Clomipramine, 125–126, 199, 206, 230, 240, 420, 450 54, 336, 373
Clonazepam, 406, 438 Complex partial seizures, 371
for panic disorder, with SSRI, 342, 383 Compliance, patient, decreased rates of, 399, 432
Clonidine, 354 Complicated bereavement, 376
for reducing symptoms of opioid withdrawal, Computed tomography (CT), head, 409, 441
213, 249 anorexia nervosa and, 21, 54
Clonidine (Catapres), 158–159, 180 noncontrast, for lobar hemorrhage, 70, 102
Clopidogrel, for stroke, 355 Comtan. See Entacapone
Clostridium botulinum, 228–229 Concentration, impaired, 403, 435
Clostridium tetani, 336 Concrete operations, 128, 157, 270, 305

463

Psychiatry Test Preparation and Review


Manual E-Book
Index

Conduct disorder Cri-du-chat syndrome, 165–166


diagnosis of, 200, 231 Crossover studies, 31
juvenile, 340, 380 Cross-sectional surveys, 31
Conduction aphasia, 62, 93, 352 CSF. See Cerebrospinal fluid (CSF)
Confabulation, in Anton’s syndrome, 202, 234 χ2 test, usage of, 289
Confidentiality, limits of, 143–144, 177 Culture
Conflict of interest, for psychiatrist, 345, 386 depression and, 216, 251
Confrontation, 32, 385 ethnicity and, 325, 357
Confusion, Wernicke’s encephalopathy and, 359 importance of, understanding, 155, 190
Consciousness, alteration of, in differentiating delirium Cushing’s disease, 109, 206, 240
and dementia, 27, 60–61 mania and, 395, 429
Conservation, 16, 49 symptoms of, 206
Constipation, 420, 450 CVT. See Cerebral venous thrombosis (CVT)
as side effect of tricyclic antidepressants, 123 Cyclothymic disorder, 76, 110, 287
Construct validity, of diagnosis, 221, 314 CYP. See Cytochrome P450 (CYP)
Consultation-liaison psychiatrist, obligation of, 75, Cysticercosis, CNS, 78, 113
108 Cytochrome P450 (CYP), 332, 367
Contralateral hemisensory loss, 232 3A4, 321, 351
Control group inducer of, 276, 313
does not receive treatment, 152, 187 1A2, inhibitor of, 255, 285
statistics and, 163 2D6, 328, 362
Conversion disorder, 176 inhibitors, 367
diagnosis of, 195, 224 substrate for, 262, 295
neurologic symptoms of, 16, 48 inhibitor, 332, 367
prognosis of, 204, 237 Cytomegalovirus (CMV)
risk factor for, 273, 310 in AIDS-related retinopathy, 66, 97
social phobia, 134 congenital viral infection caused by, 141, 174
Cooperative attitude, 413, 444 Cytomel, 310–311
Coprolalia, 31
Correlation coefficient, 383 D
direction and strength measurement and, 146, 179 Daily functioning, impairment of, in delusional
Correlation, usage of, 289 disorder, 15, 46
Cortical atrophy, 288 “Dancing eyes-dancing feet” syndrome, 193, 222
Corticosteroids(s), 82 Dandy-Walker malformation, 269, 269f, 303
intravenous, for multiple sclerosis, 27, 27f, 60, 60f diagnosis of, 269, 269f, 303, 303f
Countertransference, 3, 32 Dangerousness, in violent patients, 411
examination of, in response to patient’s behavior, Dawson’s fingers, 31
10, 41 Daytime attacks, cluster headaches and, 11, 42
Court, competence and, 344, 384 DBT. See Dialectical behavioral therapy (DBT)
Court procedure, understanding of, in making Death, in families, 266, 300
of will, 7 Decision-making, in right to die and surrogacy,
CPK. See Creatine phosphokinase (CPK) 74, 108
C-6 radiculopathy, diagnosis of, 266, 300 Declarative memory, 270, 307
Cranial neuropathies, in sarcoidosis, 216, 252 Decrementing response, to repetitive stimulation,
Creatine kinase (CK), in inclusion body myositis, 364 myasthenia gravis and, 254, 284
Creatine phosphokinase (CPK), level of, in neuroleptic Deep brain stimulation, for essential tremor, 359
malignant syndrome (NMS), 284 Deep finger flexors, inclusion body myositis and, 364
Creatinine, urine test for, 346, 386 Defendant
Creutzfeldt-Jakob disease (CJD), sanctions for, 402, 434–435
54, 226–227, 304 Defense mechanism
characteristic of, 261, 293 immature, 81, 117, 344, 361, 384
electroencephalograms of, 131, 161 mature, 260, 293, 327, 361

464

Psychiatry Test Preparation and Review


Manual E-Book
Index

Deformity, psychological impact of, during Descriptive validity, of diagnosis, 221, 314
adolescence, 17, 50 Desipramine, 87, 124, 205, 238
Dejerine-Roussy syndrome, 164 essential tremor and, 76, 111
Delirium, 129, 159, 217, 252, 418, 448 side effects, 238
characteristic of, 293 Despair, ego identity vs., 92
course and prognosis of, 418, 448 Desvenlafaxine, extended-release tablets, 277, 314
dementia vs., 27, 60–61 Developmental milestones, of children, 205, 238–239
EEG pattern of, 10, 40 Devic’s disease, 226–227
reticular formation and dorsal tegmental pathway Dextroamphetamine, 158–159
in, 418, 449 for attention deficit hyperactivity disorder, 338, 378
risk factors predisposing to, 418, 448–449 Dextromethorphan, 422, 453
Delirium tremens, 347, 387 Dextrose, intravenous, 313
Delusional disorder, 15, 46, 55, 72, 104, 133, 164, Diabetes insipidus, 83, 244
279, 317, 410, 441 Diabetes, type 2, 111
diagnosis of, 202, 234–235 Diabetic third-nerve palsy, 328, 363
Delusion, example of, 2, 31 Diagnostic and Statistical Manual of Mental
Dementia, 297 Disorders, fifth edition (DSM, 5)
Alzheimer type, 416, 447 criteria for somatic symptom disorder, 255, 285
neurologic signs of, 267, 301 substance-induced anxiety disorder specifier and,
in Binswanger’s disease, 326, 358 329, 363
delirium vs., 27, 60–61 Diagnostic and Statistical Manual of Mental
diagnosis of, 416, 446 Disorders, fifth edition (DSM, 5)
HIV-related, 262, 294 catatonia and, 131, 161–162
with Lewy bodies, 255, 285 Dialectical behavioral therapy (DBT), 338, 377
with Lewy bodies, 110 for borderline personality disorder, 7, 38
maneuver for, in outpatient setting, 416, 447 Diazepam, compromised liver function and, 9, 39–40
normal-pressure hydrocephalus and, 75, 110 Differentiation, Mahler’s stage of separation-
NOS (not otherwise specified), 409, 440 individuation, 307
vascular, neurologic signs of, 267, 301 Diffuse Lewy-body disease, 110
Dementia praecox, 42, 132, 163 Dilantin. See Phenytoin sodium
Demerol. See Meperidine Dimercaprol, 168
Depakote, in pancreatitis, 291 Diphenhydramine, 417–418, 448–449
Dependent personality disorder, 141, 174 Disability, 281, 319
violent behavior and, 202, 234 disability, criterion for, 281, 319
Depersonalization, 24, 57 Discrimination, 35
Depression, 12–13, 18, 45, 51, 72, 105 Disinhibited behavior, 77, 113
absence of causes of, brain death and, 367 Disinhibited social engagement disorder, 23, 56
in children, diagnosis of, 231–232 Dissection, carotid artery, 334, 371
cortical dementia and, 79, 115 Dissociative amnesia, 57, 161
in different cultures, 216, 251 Dissociative disorders, 134, 166
“distorted negative thoughts” and, 151, 186 Dissociative fugue, 57, 339, 378
ECT for, 260, 276, 313 Dissociative identity disorder (DID), 24, 57, 149, 183,
major, in childhood incest, 200, 231–232 445–446
management of, 263, 280, 296 factitious or malingered disorder in, ruling out, 415,
pancreatic cancer in, 199, 230 446
postpartum, 236 prognosis of, 415, 445
severity of, evaluating, 278, 315 Distal sensory polyneuropathy, in AIDS, 225
stroke risk factor and, 76, 110–111 Distortion, 371–372
Depressive position, 249 Disulfiram, 313, 451–452
Derealization, 57 Divalproex sodium, 159
Dermatomes, sensory, of nipples and umbilicus, 80, 116 for primary generalized seizures, 67, 99
Dermatomyositis, 44, 364 for uncomplicated absence seizures, 5, 34

465

Psychiatry Test Preparation and Review


Manual E-Book
Index

Dix-Hallpike maneuver, 167 ECG. See Electrocardiogram (ECG)


D2-like group, of dopamine receptors, 310 ECT. See Electroconvulsive therapy (ECT)
D1-like receptors, 310 Edrophonium chloride, myasthenia gravis
Doctor-patient relationship, models of, 399, 432 and, 9, 40, 284
Donepezil, 416, 446–447 EEG. See Electroencephalogram (EEG)
mechanism of action of, 215, 251 Ego identity
DOPA decarboxylase, inhibition of, 73, 106 vs. despair, 92
Dopamine, 418, 449 vs. role confusion, 92
in association with induction of Egocentric, 157
aggression, 5, 35 Egomania, 31
receptors, blocking of, 284 Eldepryl. See Selegiline
synthesis, 175 Electrocardiogram (ECG), 409, 441
type, 2 antagonism, 180 Electrocerebral silence, brain death and, 367
Dopamine agonist therapy, 334 Electroconvulsive therapy (ECT), 129, 147, 159,
Doubt, autonomy vs., 92 180–181
Down’s syndrome, 165–166, 291–292 for bipolar II disorder, 395, 429
features of, 258, 291–292 for depression, 260, 276, 313
Doxepin, antihistaminic activity of, 85, 122 electrodes in, placement of, 260, 292
Draw a person test, 207, 242 lithium in, 260, 292
Drive theory, 19, 52 Electrodes, placement of, in electroconvulsive therapy
DSM. See Diagnostic and Statistical Manual of Mental (ECT), 260, 292
Disorders, fifth edition (DSM, 5) Electroencephalogram (EEG)
Duchenne’s muscular dystrophy, 141, 174–175 epileptic seizure
Gowers’ maneuver or sign in, 10, 40–41 vs. psychogenic seizure, 56
Duloxetine, description of, 278, 316 generalized background slowing in, in delirium, 418,
Dupuytren’s contractures, 179 449
Durable power, 240 in juvenile myoclonic epilepsy, 362
Durham Rule, 118 triphasic waves on, 179
Dysdiadochokinesis, 164 video monitoring, for nonepileptic seizures, 353–354
Dysfunctional member, family therapy and, 9, 40 Electromagnetic radiation, 162–163
Dyskinesia, tardive, 130, 160 Electromyography (EMG), needle, repetitive nerve
age and, 332, 368 stimulation by, 284
definition of, 313 Electroshock therapy, 392, 426–427
neuroleptic-induced, risk factor for, 275, 312 adverse effects of, 393, 427–428
Dyspareunia, 169 contraindications to, 393, 427
Dyssomnia, 408, 439 duration of, 393, 428
Dysthymic disorder, 126 electrode placement, typical course of, 394, 428
lifetime prevalence of, 118 maintenance of, 393, 428
Dysthymic disorder, hallucinations and, 82, 118 medications discontinued prior to, 393, 428
Dystonia pretreatment evaluation for, 392, 427
acute, definition of, 276, 313 Embarrassment, fear of, 407, 438
musculorum deformans, 288 Emergency room (ER) psychiatrist, evaluation by, vs.
prevalence of, 361 forensic psychiatrist and, 261
Dystonic tremor, cervical dystonia and, 361 Emergency room (ER) psychiatrist, evaluation by, vs.
Dystrophin, as muscle membrane protein, 141, 174–175 forensic psychiatrist and, 293
EMG. See Electromyography (EMG)
E Emil Kraepelin, 132, 163
Early morning awakening, of depression, 135, 168 Emotional incontinence, 361
Eating disorders, 339, 378–379 Emotional lability, 361
purging in, 196, 225 Emotion, exaggerated expression of, narcissistic
weight loss in, 195, 225 personality disorder and, 14, 45
Ebstein’s anomaly, 384 Empathic failures, in mother, 278, 315

466

Psychiatry Test Preparation and Review


Manual E-Book
Index

Empathic validation, 385 F


Empathy and warmth, outcome of psychodynamic Fabry’s disease, 106
therapy, 70, 102–103 Face-to-face interview, for depression, 64, 95
Encephalitis, from herpes simplex virus-1, 329, 363 Face validity, definition of, 277, 314
Encephalomyopathy, maternally inherited, 364 Facial diplegia, 128
Encopresis, 339, 379 Facial dysmorphism, phenytoin sodium causing, 15, 47
Endarterectomy, for internal carotid artery stenosis, Facial nerve, palsy of. See Bell’s palsy
138, 171 Facial pain, 324, 354
Endurance tasks, timed, in myasthenia gravis, 284 Facial paresis, stroke-related, Bell’s palsy vs., 192, 222
Entacapone, 176 Facioscapulohumeral dystrophy (FSHD), 174
Epidural hematoma(s), 131, 161, 294 Factitious disorder, 366
Epidural spinal cord compression (ESCC), acute, MRI Familial hemiplegic migraine (FHM), 109
in, 199, 206, 231, 240 calcium channel system defects and, 169
Epigenetic principle, 328, 363 Family therapy, 9, 40
Epigenetic view, of development, 345, 386 for anorexia nervosa, 405, 436–437
Epileptic seizure for posttraumatic stress disorder, 390, 425
vs. psychogenic seizure, 56 Fatal familial insomnia, 226–227
Episodic memory, 307 Feelings of detachment, 391, 426
EPS. See Extrapyramidal side effects (EPS) “Fee splitting,” 83, 119
Epstein-Barr virus (EBV), 304 Female orgasmic disorder, 137, 169, 362
infection, 227–228 vaginismus and, 15, 47
Erikson, Erik, 92, 328, 363 Female sexual interest/arousal disorder, 137, 169
Erythromycin, carbamazepine and, 214, 250, 264, 297 Femoral cutaneous nerve, lateral, meralgia paresthetica
Esketamine, 32 and, 140, 173
Essential tremor (ET), 111, 326, 359 Fetishistic disorder, 10, 41, 365, 400, 432
Eszopiclone Fetzima
half-life of, 154, 189 Levomilnacipran, 34
sleep walking and, 273, 311 Fibromyalgia, 199, 230
ET. See Essential tremor (ET) Fiduciary duty, 82, 119
Ethics Filoviruses, 244
of psychiatrist’s on-air affirmation, 271, 309 Fish-scale disease, 44
sexual relationship between psychiatrist and patient, Flooding, 86, 123
258, 289 Flumazenil, 136, 168
Ethnicity, 325, 357 Fluoxetine, 367, 420, 450
Ethosuximide for anorexia nervosa, 405, 436
for juvenile myoclonic epilepsy, 362 side effects of, 210, 246
for uncomplicated absence seizures, 34 Fluoxetine-phenelzine, 145, 178
Euphoria, as behavioral effect of opioids, 141, 174 Flurazepam, for acute delirium, 86, 124
Excitatory neurotransmitters, 80, 116 Fluvoxamine, 347, 387
Excoriation (skin-picking) disorder, 13, 44 Focal dystonia, 361
Executions, psychiatrist participating in, 4, 34 Folate deficiency, 248
Exhibitionistic disorder, 41, 239 Folate levels, low, effect of, 263, 296
Exocytosis, 121 Folic acid supplementation, 159
Exploitation, of patient, 352 Folie-à-deux disorder, 164, 176
Exposure with response prevention, for OCD, 343, 384 Foot flexion, L5 motor nerve root in, 67, 99
Extinction, prevention of, 6, 35 Ford vs. Wainwright case, 236
Extracranial vertebral artery dissection, 297–298 Forensic psychiatrist consultant, 401b, 433–435
Extrapyramidal side effects (EPS), 160 Forensic psychiatrist, evaluation by,
Eye movement desensitization and reprocessing emergency room (ER)
(EMDR), 358–359 psychiatrist vs., 261, 293
Eye movement desensitization, and reprocessing, for Formal operations, Piaget’s stages of,
PTSD, 358–359 84, 121

467

Psychiatry Test Preparation and Review


Manual E-Book
Index

Fornix, Papez’ circuit and, 137 Gerstmann-Sträussler-Scheinker syndrome,


Fragile X syndrome, 134, 165–166, 291 226–227
Fresh-frozen plasma, in lumbar puncture, 165 γ-Glutamyl transpeptidase (GGT)
Freud, Anna, 81, 117 for alcoholic patient, 274, 311
Freud’s theories of human development, 92, 191, 219 in detection of chronic alcohol
Friedrich’s ataxia, 77, 112 abuse, 78, 114
Frontal lobe(s) GGT. See γ-Glutamyl transpeptidase
damage of, 18, 50 (GGT)
dysfunction of, loss of executive functioning Gifts, ethics and, 386
and, 142, 176 Gingival hypertrophy, phenytoin sodium causing,
“Frontal lobe syndrome,” 1 15, 47
Frontal release signs, 38–39 Gingival margin, blue line at, 132, 164
Frotteuristic disorder, 41, 365 Glatiramer acetate, 358
FSHD. See Facioscapulohumeral dystrophy Glioblastoma multiforme, 332, 368
(FSHD) in elderly, 194, 223
Functional neurological symptom disorder. See Glioma, mania and, 395, 429
Conversion disorder Global aphasia, 93
Fungal meningitis, 162 Globus pallidus, lesions in, 265, 299
Glossolalia, 402, 435
G Glucuronidation, lorazepam metabolism
GABA. See γ-Aminobutyric acid (GABA) by, 335, 372
Gabapentin, 157, 160, 242, 323, 353 Glue, sniffing
for migraine headache, 296 neuropathy and, 202, 234
Gag reflex, complete loss of, left-hemispheric stroke side effects of, 193, 222
and, 8, 38–39 Glutamate, inhibitory neurotransmitter and, 8, 39
Gait disturbance, normal-pressure hydrocephalus and, Glycine, 130, 160
75, 110 for aggression, 35
γ-Aminobutyric acid (GABA), 128, 157 Clostridium tetani and, 336, 375
for aggression, 35 as neurotransmitter, 133, 165
anxiety associated with, 40 “Good enough mothering” theory, 2, 31
biogenic amine neurotransmitter and, 69, 101 Gowers’ maneuver or sign, in Duchenne’s muscular
Clostridium tetani and, 336, 375 dystrophy, 10, 40–41
γ-Aminobutyric acid (GABA)-A receptor, 142, 175 Grandeur, delusion of, 403, 435
benzodiazepines in, 241 Grasp reflex, positive, frontal lobe and, 237–238
as chloride channel, 210, 246 Grief, period of, 154, 189
Ganser’s syndrome, 143, 176 Guanfacine, 158–159
Gastric ulcer, amitriptyline for, 67, 99 Guillain-Barré syndrome (GBS), 33, 139, 171
Gastroesophageal reflux, 408, 439–440 clinical features of, 333, 370
sleep-related, 409, 440 intensive care unit for, 237–238
Gastrointestinal discomfort, 420, 450–451 Guilt, initiative vs., 92
Gastrointestinal distress, from SSRIs, 335, 372 Gustatory seizures, 371
Gaucher’s disease, 104 Gustatory special sensory auras, 334, 371
GBS. See Guillain-Barré syndrome (GBS)
Gegenhalten, 239–240 H
Gender dysphoria, 400, 432 Habituation theory, 34
Gender reassignment, hormone therapy, 400, 433 Hair-pulling disorder. See Trichotillomania
Generalization, 35 Hallervorden-Spatz syndrome, 208, 242
Generalized anxiety disorder, 391, 426 Hallucinations
DSM criteria for, 346, 386 dysthymic disorder and, 82, 118
Generativity, vs. stagnation, 92 hypnagogic, 98
Geodon. See Ziprasidone mental status examination and, 8, 38
Gerstmann’s syndrome, 38, 352 olfactory, schizophrenia and, 66, 97

468

Psychiatry Test Preparation and Review


Manual E-Book
Index

Haloperidol, 153, 159, 188, 308–309, 347, 387, 410, Hirsutism, phenytoin sodium causing, 15, 47
441, 449 Histamine H1 receptor, associated with weight gain
for Huntington’s disease, 333, 369 and sedation, 69, 101
for tranquilizing agitated patients, 3, 32 Histoplasma, 245–246
Halstead-Reitan battery, 36 Histrionic personality disorder, 45
Hamilton depression rating scale (HAM-D), 315 HIV disease, complications of, 269, 304
Hand paresthesias, nocturnal, 356 HIV-related dementia, diagnosis of,
Handwriting, in essential tremor, 359 262, 294
Headache(s) Hoarding disorder, 84, 120
intractable post-lumbar puncture, 139, 172 Hoffmann’s sign, 239–240
meningitis and, 333, 370 Hoffman’s sign, 38–39
tension-type, 72, 105 Homeless mentally ill, 281, 319
Head atrophy, caudate, Huntington’s disease and, 255, Homocystinuria, 290
285 Homonymous hemianopsia, left, 84, 120
Head CT scan. See Computed tomography (CT), head Hoover’s sign, 205, 239–240
Head trauma Hormone therapy, for gender reassignment,
abducens palsy from, 363 400, 433
subdural hematoma from, 134, 166 Horner’s syndrome, 192, 221
Head tremor, in essential tremor, 359 in carotid artery occlusion, 227
Health Insurance Portability and Accountability Act classic triad of, 298
(HIPAA), 282, 33202 Hostile attitude, 403, 435
Heat intolerance, 137, 169 Hostility, cardiac disease, in men, 268, 302
Hematoma H1 receptor, blocking of, weight gain and, 254, 284
epidural, 131, 161, 294 HTLV-1. See Human T-lymphotropic virus type-1
subdural, 161, 256, 256f, 286, 286f (HTLV-1)
Heme synthesis Human T-lymphotropic virus type-1 (HTLV-1),
acute intermittent porphyria, 140, 173 tropical spastic paraparesis and, 144, 178
Hemianesthesia, following stroke, 293 Huntington’s disease (HD),
Hemiballismus, left-arm, 69, 101 297, 333, 369
Hemicranial headache, of acute onset, with left eye caudate head atrophy and, 255, 285
visual loss, management of, 263, 295–296 diagnosis of, 264, 297
Hemodialysis, lithium toxicity and, 150, 184 likelihood of transmission of, 3, 32
Hepatic function, impaired, psychosis and, medications Hydrocephalic astasia-abasia, 369
for, 277, 314 Hydrocephalus
Hepatolenticular degeneration, 287 noncommunicating, 170–171
Herniation, transtentorial, 165 normal-pressure, 75, 110
Heroin, in urine toxicology screen, 147, 180 symptoms of, 261, 294
Herpes simplex encephalitis, 329, 363 Hydrochlorothiazide, lithium and, 325
Herpes simplex virus, Bell’s palsy and, 197, 227–228 3-Hydroxy benzodiazepines, 177
Herpes simplex virus-1, encephalitis from, 363 5-Hydroxyindoleacetic acid (5-HIAA)
Herpetic meningitis, 162 for aggression, 35
Heschl’s gyrus. See Superior temporal gyrus (Heschl’s decreased, in CSF, associated with violence and
gyrus) aggression, 65, 88, 96, 125–126
Hiccups, 234 Hydroxyzine, 160
HIPAA. See Health Insurance Portability and Hyperekplexia, 160, 165
Accountability Act (HIPAA) Hyperemotionality, 361
Hippocampal volume, decreased, in posttraumatic Hyperkalemic periodic paralysis, deficits with, 301
stress disorder, 259, 290 Hypersensitivity pneumonitis, 210, 245–246
Hippocampus Hypertension
memory formation in, 215, 250 bupropion and, 12, 43
Papez’ circuit and, 137 as cause of intracerebral hemorrhage, 19, 52
Hirano bodies, 230 idiopathic intracranial, complication of, 269, 305

469

Psychiatry Test Preparation and Review


Manual E-Book
Index

Hypertensive crisis, monoamine oxidase inhibitor- Industry, vs. inferiority, 92


related, 337, 376 Infancy, motor development in, 274, 312
Hyperthermia, malignant, calcium channel system Infantile mental retardation, inherited cause of, 259,
defects and, 169 291
Hyperthyroidism, 226 Infantile secure attachment, 270, 306
Hyperventilation syndrome, 226 Infantile spasms, 137
Hypervitaminosis A, 243 Infarcts
Hypnagogic hallucinations, 98 lacunar, 303
Hypnotherapy, 183 of subcortical white matter, 258, 288
Hypochloremic alkalosis, 225 in middle cerebral artery, 289
Hypoglycemia, 226 Inferiority, industry vs., 92
Hypokalemic periodic paralysis, calcium channel Information, releasing of, to insurance company, 352
system defects and, 169 Inhalant(s)
Hypomania, 287, 348, 388 causing dementia, 24, 57
mania and, 394, 428–429 intoxication, diagnosis of, 279, 316
Hypomania/mania, 119 Inhibitory neurotransmitters, 86, 123
Hyponatremia, rapid sodium replacement in, Initiative, vs. guilt, 92
63, 93–94 Inpatient hospitalization, for borderline personality
Hypothyroidism disorder, 27, 60
serum thyroid-stimulating hormone increasing in, Insecticide, organophosphate, treatment
87, 124 of, 282, 320
symptoms of, 296 Insight, 413, 444
Hypoxanthine-guanine phosphoribosyltransferase, Insomnia, 35, 449
deficiency of, 84, 121 treatment of, 28, 61
Hypsarrhythmia, infantile spasm and, 137 Insular cortex, gustatory special sensory auras in, 334,
371
I Insurance company, patient’s request, 274, 311
Ibuprofen, lithium levels and, 143, 177 Intellectual disability, 165–166, 170, 181
Identity diffusion, 211, 246 diagnosis of, 279, 317
Idiopathic intracranial hypertension, complication of, Intelligence quotient (IQ), mental age and, 88, 125
269, 305 Intention tremor, in Wilson’s disease, 257, 287
Illness anxiety disorder, 11, 42, 331, 366 Interferon β-1b, 325, 358
misinterpretation of bodily sensations and, 23, Interferon, depression and, 349, 389
55–56 Intermittent explosive disorder, 136, 148, 168,
regularly scheduled primary care appointments for, 181–182
348, 388 differential diagnosis for, 136, 169
Illness management, 122 Internal capsule
Imipramine, 211, 247 in pure motor hemiparesis, 200, 232
for childhood enuresis, 90, 127 stroke localizing to, 69, 101
Imitrex. See Sumatriptan Internal carotid artery stenosis, endarterectomy for,
Immature defense mechanism(s), 81–82, 117, 344, 138, 171
361, 384 Internuclear ophthalmoplegia, 264, 296–297
Impaired hepatic function, psychosis and, medications in multiple sclerosis, 23, 56
for, 277, 314 Interpersonal deficits, 376
Imprinting, 79, 115, 250 Interpersonal inventory, 376
Impulse control, 413, 444 Interpersonal psychotherapy therapy (IPT), 120, 214,
Impulse control disorder, of chronic pathological 250, 338, 376
gambling, 339, 379 Interpretation, definition of, 345, 385
Incidence, 102 Interpreter, 345, 386
Inclusion body myositis (IBM), 141, 174, 364 Intimacy, vs. isolation, 92
Incontinence, normal-pressure hydrocephalus and, 75, Intracerebral hemorrhage (ICH), 370
110 hypertension causing, 19, 52

470

Psychiatry Test Preparation and Review


Manual E-Book
Index

Intracranial hemorrhage Korsakoff’s syndrome, 360


signs of, 64, 95 Krabbe’s disease, 96, 104
Intracranial metastatic carcinoma, in brain tumors, Kraepelin, Emil, in concept of mental disorders, 11, 42
285–286 Kuru, 226–227
Introjection, 327, 361
Ionizing radiation, 162–163 L
Ipecac, abuse of, 405, 437 Labile affect, 361
cardiomyopathy from, 325, 358 Lacunar infarcts, 303
Ipsilateral peripheral facial palsy, 164 of subcortical white matter, 258, 288
IQ. See Intelligence quotient (IQ) Lacunar strokes, prevention of, 268, 303
Irritability, in depressed children, 25, 59 Lacunar thalamic infarcts, 164
Irritable bowel syndrome Lambert-Eaton myasthenic syndrome, 4, 33
diagnosis of, 328, 362 calcium channel system defects and, 169
symptoms, 362 Lamotrigine, 30, 267, 311–312, 321, 351
Ischemia side effects of, 30
amaurosis fugax and, 191, 219–220 for trigeminal neuralgia, 354
anterior cerebral artery, 219–220 valproate and, 17, 49
carotid artery territory, 191, 219–220 valproic acid and, 333, 368
lenticulostriate territory, 219–220 weight gain and, 346, 387
middle cerebral artery territory, 219–220 Language, children and, 326, 359
vertebrobasilar territory, 219–220 Lasègue’s sign, 239–240
Ischemic heart disease, sumatriptan (Imitrex) and, 7, Latency, in Freud’s theories of human development,
37 191, 219
Ischemic optic neuropathy, 331, 366 Lateral medullary syndrome. See also Wallenberg’s
Isolation, intimacy vs., 92 syndrome
Isoniazid (INH) exposure, vitamin B6 (pyridoxine) left, 65, 96
deficiency and, 209, 243 L-DOPA therapy, in dementia with Lewy bodies, 285
Lead encephalopathy, 162–163
J Lead poisoning, 132, 164
Jaundice, lithium intoxication and, 88, 125 Learned helplessness, 34, 143, 176, 307–308
JC virus, 304 Leber’s hereditary optic neuropathy (LHON), 365
Jerk epilepsy, 362 Lenticulostriate territory ischemia, 219
Judgment of line orientation test, 326, 360 Leptospirosis, 157–158
Judgment, patient’s, 414, 444 Lesch-Nyhan syndrome, 71, 84, 104, 121
Jung, Carl, 92, 340, 381 Levinson’s developmental theory, 386
Justice, 59, 382 Levodopa, for progressive supranuclear palsy, 361
Juvenile myoclonic epilepsy (JME), 328, 362 Levomilnacipran, 34, 395
Lewy bodies, 230
K dementia with, diagnosis of, 255, 285
Kappa, 102, 342, 383 in Parkinson’s disease, 286–287
Kearns-Sayre syndrome (KSS), 365 Lewy body disease, 89, 126
Ketoconazole, 243, 387 Lhermitte’s sign, 366
Kinetic tremor, in essential tremor, 359 LHON. See Leber’s hereditary optic neuropathy
Klein, Melanie, 328, 385 (LHON)
Kleine-Levin syndrome, 217, 252 Libido, increased, major depressive episode and, 72,
Kleinfelter syndrome chromosome (XXY), presence of, 105
147, 181 Lidocaine transdermal patch (Lidoderm), 160
Kleptomania, diagnostic criterion for, 135, 167 Lifetime prevalence, 383
Klinefelter’s syndrome, 181 Limb dystonia, 361
Klüver-Bucy syndrome, 180 Lioresal, 145, 179. See also Baclofen
Kohut’s theories of self-psychology, 213, 249, 306 Liothyronine, 310–311
Koro, 38 Lisch nodules, 356

471

Psychiatry Test Preparation and Review


Manual E-Book
Index

Lissencephaly, 170–171 Major depressive disorder (MDD) (Continued)


Lithium, 159 social phobia, 167
for aggression, 144, 178 suicides and, 203, 235, 335, 373
ECT and, 260, 292 symptoms of, 77, 112
hydrochlorothiazide, 325 vortioxetine, 187
hyperparathyroidism associated with, 344, 385 Major depressive episode, 77, 113
intoxication, 209, 244, 277, 314–315 Mal de ojo, 38
MAOIs combination with, 12, 43 Malingering, 33, 366
during pregnancy, 343, 384 diagnosis of, 273, 310
routine monitoring for, 346, 386 feature of, 149, 183
side effects of, 1, 30, 275, 313, 335, 373 Mallory-Weiss syndrome, 405, 437
therapeutic range for, 142, 175 Malpractice, causes of, 341, 382
thiazide diuretic and, 358 Mammillary body, Papez’ circuit and, 137
Lithium carbonate, 415, 445 Manganese miners, 164
Locked-in syndrome, acute, 63, 93–94 Mania, 417, 448
Locus ceruleus, norepinephrine synthesis in, 68, 100 hypomania and, 394, 428–429
Logorrhea, 66, 98 medical conditions associated with, 395, 429
Long-acting injection Manic bipolar patient, thought process of, 76, 111
medications, 153 Manic episode, 415, 446
Lorazepam, 335, 340, 372, 382 medications causing, 395, 429
for a mute catatonic patient, 149, 183 MAOIs. See Monoamine oxidase inhibitors
Lorenz, Konrad, 115 (MAOIs)
Lou Gehrig’s disease. See Amyotrophic lateral sclerosis Marchiafava-Bignami disease, 114, 360
Lower motor neuron signs, 134, 166 Marcus Gunn pupil, 177
Lumbar puncture Marfan’s syndrome, 292
with cerebrospinal fluid opening pressure, for Mature defense mechanism(s),
pseudotumor cerebri, 63, 94 119, 260, 293, 327, 361
contraindications to, 133, 165 McGarry instrument, 117–118
Lung carcinoma, brain metastases and, 139, 172, 255, MDD. See Major depressive disorder (MDD)
285–286 Mean, 102
Lyme disease, 157–158 Measles virus, in subacute sclerosing panencephalitis,
Bell’s palsy and, 227–228 71, 103
Lymphoma, in acquired immunodeficiency syndrome Medial longitudinal fasciculus (MLF), lesion to, as sign
(AIDS), 195, 224 of internuclear ophthalmoplegia, 23, 56
Lyrica. See Pregabalin Median, 102
Lysergic acid (LSD), 57 Median neuropathies, at the wrist, 324, 324f, 355
Medical chart, subpoenaed by court, 200, 232
M Medicare, maximum fee for, 74, 107
Magnetic resonance imaging (MRI) Medroxyprogesterone acetate, for exhibitionistic
in acute epidural spinal cord compression, 199, 206, disorder, 205, 239
231, 240 Medulloblastoma, diagnosis of,
functional, 368–369 266, 266f, 299, 299f
Magnetic resonance imaging (MRI), neonate brain, Mees’ lines, of fingernails, 132, 132f
269, 269f Meige’s syndrome, 203, 236
Mahler’s stages, of separation-individuation, 132, 140, MELAS. See Mitochondrial encephalopathy,
162, 331, 341, 366, 382 myopathy, lactic acidosis, and stroke-like episodes
object constancy, 331, 366 (MELAS)
Major depressive disorder (MDD), 407, 439–440 Melatonin, 151, 189
childhood incest and, 200, 231–232 production of, in pineal gland, 70, 103
lifetime prevalence of, 112, 118 Memantine, 6, 36, 206, 241, 336, 374
pathological gambling and, 22, 55 Memory, 307
prevalence of, 247 deficits, in Binswanger’s disease, 358

472

Psychiatry Test Preparation and Review


Manual E-Book
Index

Meningitis, 333, 369–370 Middle cerebral artery infarction, 289


cerebral venous thrombosis and, 173 Middle cerebral artery stroke, left-sided,
primary or secondary, 157–158 258, 289
suspected, with obtundation, 133, 165 Middle cerebral artery territory ischemia, 219–220
Meningoencephalitis, cerebral venous thrombosis Middle cerebral artery territory stroke in impaired
and, 173 naming, repetition, 258, 289
Mens rea, 240, 341, 382, 434 Middle meningeal artery
Mens rea, 401 damage to, 261, 294
Mental disorders trauma to, 131, 161
treatment, 85 Midline cerebellar vermian lesions, 164
Mental disorders, concept of, 11, 42 Migraine
Mental illness, symptoms of, 146, 179–180 acute abortive therapy of, agents for, 263, 296
Mental retardation, infantile, inherited cause of, 259, generator of, 72, 105
291 without aura, diagnostic criterion of, 63, 94
Mental status exam Miller-Fisher syndrome, 114
hallucinations and, 8, 38 Minnesota Multiphasic Personality Inventory (MMPI),
thought content in, 78, 113, 402, 435 4, 33, 71, 103, 273
Meperidine Miosis, 174
analgesic potency of, intramuscular vs. oral, 3, 32 Mirroring, in Kohut’s theory of self-psychology, 270,
combine with MAOI, 150, 185 306
Meralgia paresthetica, 140, 173 Mirtazapine, 217, 252, 282, 320
Mercury poisoning, 162–163 in nausea, 265, 299
MERRF myopathy. See Myoclonus epilepsy with receptor for, 268, 302
ragged-red fibers (MERRF) myopathy Mistrust, trust vs., 92
Mesial temporal sclerosis, 230 Mitochondrial encephalopathy, myopathy, lactic
Mesocortical pathway, 329, 364, 403, 435 acidosis, and stroke-like episodes
Mesolimbic pathway, 364 (MELAS), 330, 364
Mesolimbic-mesocortical pathway, associated with MLF. See Medial longitudinal fasciculus (MLF)
antipsychotic medications, 145, 178 MMPI. See Minnesota Multiphasic Personality
Metabolic encephalopathy, 363 Inventory (MMPI)
Metachromatic leukodystrophy (MLD), 104, 290 M’Naghten rule, 236, 401, 434
arylsulfatase A and, 65, 96 Modafinil, 98
Methadone, 347, 387 for narcolepsy, 35
3-Methoxy-4-hydroxyphenylglycol (MHPG), plasma, Mode, 102
pathological gambling and, 339, 379 Monoamine oxidase (MAO) activity, platelet,
Methylenedioxyamphetamine, as common drug of pathological gambling and, 339, 380
abuse at clubs and raves, 28, 61 Monoamine oxidase inhibitors (MAOIs), 2, 32, 279,
Methylenedioxymethamphetamine (ecstasy), 216, 251, 316
287, 346, 386–387 drug interactions of, 150, 185
Methylenedioxypyrovalerone, 354 hypertensive crisis induced by, 337, 376
Methylphenidate, 158–159 lithium combination with, 12, 43
for attention deficit hyperactivity Mood disorders, rating scale for, 215, 251
disorder, 338, 378 Mood stabilizers, administering, 274, 311–312
causing anorgasmia, 327, 362 Motivational enhancement therapy, 345, 385
decreasing TCA levels, 12, 43 therapeutic focus of, 19, 51
Methysergide, for cluster headaches, 302 Motor development, in infancy, 274, 312
Metoclopramide, 130, 160, 167 MRI. See Magnetic resonance imaging (MRI)
Microdeletion, autosomal, in chromosome Multiple sclerosis
characteristics of, 260, 292 mania and, 395, 429
Midbrain, lesion of, 164 progressive phase of, CSF finding in, 268, 302
Midbrain stroke syndrome. See Weber’s syndrome relapsing remitting, 325, 358
Middle adulthood, developmental task of, 3, 33 steady progression from initial onset and, 25, 58

473

Psychiatry Test Preparation and Review


Manual E-Book
Index

Multiple system atrophy, trinucleotide repeat Neurofibromatosis, 36


sequences and, 144, 178 type 1, 357t
Muscarinic cholinergic receptors, blockade type 1 (NF1), 324, 356, 357t
of, 213, 249 type 2, 357t
Muscarinic effects, of organophosphorus poisons, 381 type 2 (NF2), 356, 357t
Muscle fasciculations, 277, 314–315 Neurofibromin, 356
Muscle inflammation, 230 Neurogenic thoracic outlet syndrome, 356
Mutations, in Parkinson’s disease, 286–287 Neuroimaging, mania and, 343, 384
Mutism, 150, 183, 403, 436 Neuroleptic-induced tardive dyskinesia, risk factor for,
Myasthenia gravis (MG), 40 275, 312
electromyographic studies for, 254, 284 Neuroleptic malignant syndrome (NMS), 138, 171
Myelopathy serotonin syndrome vs., 254, 284
cervical spondylosis and, 366 Neuronal glucose metabolism, positron emission
tropical spastic paraparesis and, 144, 178 tomography for, 333, 368
Myerson’s sign, 38–39 Neuronal programmed cell death, 73, 106
Myocardial infarction, electroconvulsive therapy for, Neurons
147, 180–181 chemical signaling between, 84, 121
Myoclonic epilepsy, juvenile, 328, 362 deficits in, neurotoxicity and, 273
Myoclonic seizures, 362 Neurontin. See Gabapentin
Myoclonic sleep disorder, 167–168 Neuropathy, inhalant toxication and,
Myoclonus epilepsy with ragged-red fibers (MERRF) 202, 234
myopathy, 364–365 Neuropsychological consultation, 416, 446–447
Myotonic dystrophy (MD) type I, 112 Neurosarcoidosis, 216, 252
NeuroStar TMS Therapy. See Transcranial magnetic
N stimulation (NeuroStar TMS Therapy)
Naloxone, 168 Neurotransmitters, anxiety associated with, 9, 40
as opioid antagonist, 153, 188 Niacin deficiency (pellagra), 212, 248
Naltrexone Nicotine withdrawal, symptoms of, 330, 365
for alcohol abuse, 211, 247 Nicotinic effects, of organophosphorus poisons, 381
extended-release injectable suspension, 278, 315. See Niemann-Pick disease, 259, 290–291
also Vivitrol Nighttime awakening, 66, 98
Namenda. See Memantine Nigrostriatal pathway, 178, 364
NAMI. See National Alliance for the Mentally Ill Nigrostriatal tract, 138, 170
(NAMI) NMDA receptor. See N-methyl-D-aspartate (NMDA)
Naratriptan, for migraine headache, 263, 296 receptor
Narcissistic disorder personality patients, 14, 45 N-methyl-D-aspartate (NMDA) receptor, 204, 237
Narcissistic personality disorder, 89, 126 memantine and, 206, 241
Narcolepsy, 5, 16, 35, 48, 66, 98, 130, 160 phencyclidine and, 80, 116
clinical features of, 279, 316 N-methyl-D-aspartate (NMDA) receptor, 32
symptom of, 267, 300 complex, 136, 168
National Alliance for the Mentally Ill (NAMI), NMS. See Neuroleptic malignant syndrome (NMS)
63, 94 Nocturnal polysomnography, 408, 439
Nausea, mirtazapine in, 265, 299 Noncommunicating hydrocephalus, 170–171
Nefazodone, trazodone and, 323, 353 Noncompliance, 33
Neglect, child, 337, 376 Nonepileptic seizures, videotelemetry and, 323,
Neuralgia, postherpetic, 130 353–354
treatment of, 265, 298–299 Nonmalfeasance, 82, 119
Neurocognitive disorder, 89 Noradrenergic neurons, 40
Neurodegenerative disorders, associated with Norepinephrine
expansion of genetic trinucleotide, 178 for aggression, 35
Neurodevelopmental theory, 386 anxiety and, 9, 40
Neurofibrillary plaques and tangles, 229–230 locus ceruleus and, 191, 220

474

Psychiatry Test Preparation and Review


Manual E-Book
Index

Norepinephrine (Continued) Oedipal conflict, 249


reuptake of, 209, 245 Olanzapine, 153, 188, 412, 418, 445, 449
synthesis of, in locus ceruleus, 68, 100 for anorexia nervosa, 405, 436
Normal autism, Mahler’s stage of separation- in treating enuresis in childhood, 85, 122
individuation, 307 Olfactory hallucinations, schizophrenia and, 66, 97
Normality-as-process, 386 Olfactory seizures, 371
Normal-pressure hydrocephalus, 75, 110 Ondansetron (Zofran), 160
symptoms of, 261, 294 Open-ended questions, 88, 125
Nortriptyline, 362 Operant conditioning, 5, 34, 391, 426
Noxious stimuli, preserved response to, 135, 167 theories of, 271, 307
Nucleus accumbens, cocaine and, 19, 51 Ophthalmoplegia
Null hypothesis, 132, 163 internuclear, 264, 296–297
rejection of, in type I error, 20, 52 in multiple sclerosis, 23, 56
Null point, 361 in Miller-Fisher syndrome, 78, 114
Nuplazid Wernicke’s encephalopathy and, 359
Pimavanserin, 35 Opioid(s)
Nursing facility, 416, 447 behavioral effect of, 141, 174
Nystagmus, 331, 366 intoxication, 235
vertical, PCP intoxication associated with, 24, 57 Oppenheim’s dystonia, 288
Oppositional defiant disorder (ODD)
O conduct disorder and, 231
Obesity rates diagnosis of, 201, 233
the U.S., 352 Opsoclonus-myoclonus, 193, 222
Obesity-related illnesses, 322, 352 Optic ataxia, 352
Object constancy, in Mahler’s stages of separation Oral phase, in Freud’s theories of human
individuation, 307, 331, 366 development, 219
Object permanence, in Piaget’s developmental stages, Orap. See Pimozide
193, 222 Orbitofrontal lobe lesions, presentation of, 1, 31
Object relations theory, 344, 385 Organophosphate poisoning, 162–163, 340, 381
Obsessions, as part of thought content in mental status exposure and, 336, 374–375
exam, 78, 113 treatment for, 282, 320
Obsessive-compulsive disorder (OCD), 44, 129, 131, Orgasmic disorder, 362
136, 158–159, 161–162, 169, 199, 224, 338, 341, Ornithine transcarbamylase deficiency, 121
377–378, 382 Orthostatic hypotension, side effect of
cingulotomy and, 149, 183 MAOIs, 32
Clomipramine, 199, 230 Other Specified Somatic Symptom and Related
exposure with response prevention for, 343, 384 Disorders, pseudocyesis and, 66, 97
lifetime prevalence of, 120 Outpatient commitment program, for noncompliant
neuroimaging of, 194, 223 patient with schizophrenia and history of violence,
serotonin in, 88, 125–126 150, 184–185
treatment for, 335, 372–373 Oxazepam, active metabolites and, 144, 177
Obsessive-compulsive personality disorder (OCPD), Oxcarbazepine, 160
diagnosis of, 217, 253 side effects of, 30
Obstructive sleep apnea, 408, 439
complications of, 408, 439 P
OCD. See Obsessive-compulsive disorder (OCD) Pacchionian granulations. See Arachnoid
O’Connor vs Donaldson, 118 (pacchionian) granulations
Ocular motor apraxia, 352 Pain, somatic symptom disorder and, 329, 363–364
Oculopharyngeal muscular dystrophy, 174 Paliperidone, 153, 188, 314
Oculovestibular reflexes, in diagnosis of brain death, Pancreatic cancer, in depression, 199, 230
9, 39 Pancreatitis, Depakote in, 291
ODD. See Oppositional defiant disorder (ODD) Panic attack, 73, 106

475

Psychiatry Test Preparation and Review


Manual E-Book
Index

Panic disorder, 224 Pellagra. See Niacin deficiency (pellagra)


DSM criteria for, 346, 386 Perceptions, 414, 444
lifetime prevalence of, 106 Pergolide, 18, 50
organic differential for, 106 Period prevalence, 383
treatment for, 341, 383 Periodic high-amplitude sharp wave complexes, 131,
PANSS. See Positive and Negative Symptom Scale 161
(PANSS) Persecution, feelings of, in psychotic patient, 69, 101
Parachute response, in 6 months of age, 65, 97 Persistent vegetative state, 135, 167
Paralysis Personal information, of therapist, 342, 383
hypokalemic periodic, calcium channel system Personality disorder
defects and, 169 defense mechanisms, 73, 106
Paranoia, amphetamine intoxication and, 25, 59 Personality disorders, 181–182
Paranoid personality disorder, 73, 106, 137, 170 characteristics of, 396, 429–430
Paranoid-schizoid position, 249 defense mechanisms in, 397, 430
Paraoxonase status, measurement of, 320 diagnosable, percentage of, 257, 288
Paraphilias, 365 medications for, 397, 430–431
interventions for, 400, 433 self-mutilation in, 397, 430
Paraphilic disorders, 399b, 432 treatment for, 396, 430
prognostic factor Personal needs, of therapist, 342, 383
good, 400, 433 PET. See Positron emission tomography (PET)
poor, 400, 433 Phalen’s maneuver, 355
Parens patriae, 82, 119, 206, 240, 382 Phallic stage, in Freud’s theories of human
Parental empathy, 249 development, 219
Paresthesias, nocturnal hand, 356 Phencyclidine (PCP), 331, 366
Parinaud’s syndrome, 164 effect of, 192, 221
Parkinsonian disease, 171 hallucinogenic effects of, 210, 245
Parkinson’s disease, 286–287, 293–294, 317, 334, intoxication, 235
371–372 vertical nystagmus associated with, 24, 57
Parkinson’s disease (PD), 333, 369 NMDA receptor and, 80, 116
carbohydrate-deficient transferrin (CDT) in, in urine, 21, 54
256, 286–287 Phenelzine, 176, 420, 450
characteristics of, 262, 295 for atypical depression, 214, 250
choreoathetosis in, 201, 232 combine with fluoxetine, 145, 178
core features of, 286–287 Parkinson’s disease and, 81, 117
idiopathic, 199, 230, 286–287 Phentolamine, 337, 376
Paroxetine, 12, 43, 367, 406, 438 Phenylketonuria (PKU), 290
sedative effects of, 10, 41 clinical feature of, 137–138, 170
Paroxysmal hemicranias, 105 Phenytoin, 130, 160, 422, 453
Partial isolation monkeys, 148, 182 intramuscular, for status epilepticus, 67, 98
Pathologic gambling issues Phenytoin sodium, 15, 47
Parkinson’s disease patients, 371–372 Pheochromocytoma, 348, 388
Pathological gambling, 152, 187, 339, 379 Phobia, specific, 324, 356
major depressive disorder and, 22, 55 Phonological disorders, diagnosis of, 211, 247
Pathological grief, 14, 46 Phosphodiesterase, 5 inhibition, mechanisms of, 148,
Patient 182
psychiatrist vs. Piaget’s developmental stages, 193, 222
sexual relations, 401, 433 cognitive development, piaget’s stages of, 68, 92,
Patient compliance, decreased rates of, 399, 432 100, 128, 157
PCP. See Phencyclidine (PCP) concrete operations, 128, 157, 270, 305
PCR. See Polymerase chain reaction (PCR) Piblokto, 38
Pearson correlation coefficient, 139, 171–172 Pick’s disease, 198, 229–230
Pedophilia, 338, 378 diagnosis of, 267, 267f, 301, 301f

476

Psychiatry Test Preparation and Review


Manual E-Book
Index

Pimavanserin, 6, 35 Practicing, Mahler’s stage of separation-individuation,


Pimozide, for Tourette’s disorder, 213, 249 270, 307
Pineal gland, melatonin in, production of, 70, 103 Prader-Willi, 303–304
Pinpoint pupils, heroin withdrawal and, 87, 124 Prader-Willi syndrome, 165–166
Pituitary macroadenoma, 8, 38 diagnosis of, 269, 303–304
PKU. See Phenylketonuria (PKU) Pralidoxime, 162–163, 320
Platelet antiaggregants, for stroke, 355 for organophosphorus poisoning, 374–375
Platelet transfusion, in lumbar puncture, 165 Predictive validity, of diagnosis, 221, 314
Pleocytosis, cerebrospinal fluid, Guillain-Barré Prednisone, 358
syndrome and, 333, 370 Prefrontal cortex, left, damage of, depression and, 18,
Plexopathy, brachial, 356 51
Point prevalence, 20, 53, 102, 383 Pregabalin, 160, 230
Poliomyelitis, 166 Pregnancy
Polycystic ovarian syndrome, 72, 105 categories of
cause of, 105 benzodiazepines in, 340, 382
depression, 72, 105 manic episode during, therapies for, 129, 159
Polymerase chain reaction (PCR) Premature ejaculation, treatment for, 341, 382
for herpes simplex encephalitis, 363 Preoperational thought, 68, 100
serum, for Huntington’s disease, 1, 30 Cognitive development, Piaget’s stages of, 92, 100,
Polymyositis, 33, 329, 364 128, 157
Pontine hemorrhage, 227 cognitive development, Piaget’s stages of, 68
Porphobilinogen and aminolevulinic Piaget’s stages of, 79, 115
in urine, blood, and stool, 140 Presynaptic dopaminergic depletion, 69, 101
Porphyria, acute intermittent, 325, 357 Primary auditory cortex, 130, 160
manifestation of, 280, 318 Primary care physicians
Positive and Negative Symptom Scale (PANSS), 315 vs. psychiatry physicians, 28, 61–62, 93
Positron emission tomography (PET), neuronal glucose collaborative care, goal of, 62
metabolism and, 333, 368 Primary care setting, 169
Posterior cerebral artery territory, stroke in, 258, 289 treatment, 85
Postexertive malaise, 16, 49 Primary prevention, 33
Postherpetic neuralgia, 130 Primary torsion dystonia, dementia and, 257, 288
treatment of, 265, 298–299 Prion disease(s), 197, 226–227
Post-lumbar puncture headache, treatment of central nervous system, diagnosis of, test for, 21, 54
intractable, 139, 172 Privilege, 336, 374
Postpartum blues, diagnosis of, 203, 236 as psychiatrist’s right, 150, 185
Postpartum depression, 236 Probability, 102
Postpartum psychosis, 236 Procedural memory, 307
Posttraumatic dementia, 122 Progressive multifocal leukoencephalopathy, diagnosis
Posttraumatic stress disorder (PTSD), 161, 390, 409, of, 196, 226, 226f
425, 440 Progressive supranuclear palsy, 327, 327f, 360–361
acute stress disorder vs., 133, 165 Projection, 32, 73, 106
differential diagnosis for, 392, 426 Projective identification, 397, 430
duration of disturbance of, 14, 45–46 Prolactin level, 409, 441
hippocampal volume and, 259, 290 risperidone and, 22, 55
poor prognosis for, 391, 425 Propofol, 393, 428
psychotherapeutic treatment for, 358–359 Propranolol, 248
risk factor for, 280, 317 for akathisia, 212, 248, 323, 354
symptoms of, 391, 426 for performance anxiety, 407, 438
treatment approaches for, 391, 426 Prosopagnosia, 192, 221, 234
Postural tremor, in essential tremor, 359 Protein, urine test for, 346, 386
Potassium, serum level of, in anorexia nervosa, 14, 46 Protriptyline, in treatment of tics in Tourette’s
Power, 20, 53 syndrome, 83, 120

477

Psychiatry Test Preparation and Review


Manual E-Book
Index

Pseudallescheria boydii, 245–246 Pure motor hemiparesis, 69, 101, 200, 232
Pseudobulbar palsy, 361 Purging, in eating disorders, 196, 225
Pseudocyesis, 66, 97 Pyridoxine. See Vitamin B6 (pyridoxine)
Pseudoseizures, in conversion disorder, 195, 224 Pyromania, symptoms of, 135, 168
Pseudotumor cerebri, 63, 94, 173
complication of, 269, 305 Q
Psychiatric rehabilitation-social skills training, 25, 58 QTc interval, prolongation of, 349, 389
Psychiatrist, 309 Quality of relationships, patient, in psychodynamic
ethical behavior for, 281, 319 psychotherapy, 26, 59
legal team’s actions and, 89, 126
on-air affirmation of, ethics of, 271, 309 R
responsibility of, 262, 294 Rabies virus infection, diagnosis of, 198, 228–229
suicide and, 281, 319 Radial nerve entrapment, 15, 47–48
vs. patient Radiculopathy
sexual relations, 401, 433 C-6, diagnosis of, 266, 300
Psychiatrist’s unethical behavior Ramelteon
with patient, 434 half-life of, 189
Psychiatry physicians mechanism of action of, 151, 186
vs. primary care physicians, 28, 61–62, 93 Random reinforcement, pathological gambling and,
collaborative care, benefits of, 28 152, 187
Psychoanalytic psychotherapy, 86, 123 Randomization, 20, 53
Psychodynamic psychotherapy Raphe nuclei, as sites of serotonergic cell bodies, 18,
error in, 210, 246 51
missed session in, 212, 248 Rapid eye movement (REM) sleep behavior disorder,
quality of relationships in, 26, 59 89, 126, 408, 440
Psychodynamic therapy Rapport, development of, 399, 432
candidates for, 344, 385 Rapprochement, Mahler’s stage of separation-
prognostic outcome in, 70, 102–103 individuation, 132, 162, 307
Psychogenic polydipsia, 43 RAS. See Reticular activating system (RAS)
Psychogenic seizure, 23, 56 Rash, in dermatomyositis, 364
epileptic seizure vs., 56 Rationalization, 63, 94
Psychological debriefing, for traumatic event, Reading, impairment in, specific learning disorder
326, 358–359 with, 201, 232–233
Psychosis, 449 Recombinant tissue plasminogen activator (r-TPA),
differential diagnosis for, 411, 442 contraindication to, in acute ischemic stroke, 64,
impaired hepatic function and, medications for, 277, 95
314 Reflex, primitive, in 6 months of age, 65, 97
postpartum, 236 Reflex sympathetic dystrophy, characteristics of, 74,
Psychotherapy 107
group and individual, 421, 451 Refractory epilepsy, vagal nerve stimulation for, 207,
notes, separate set of, 352 241
self-disclosure in, 276, 314 Reglan. See Metoclopramide
Psychotic disorder(s), 159, 402b, 409b, 436, 441 Regression analysis, 70, 102, 145, 163, 179, 289
brief, 410, 441 Rejection, perceived, and suicide, 396, 430
consideration in, 256, 286 Relative risk, 149, 182
substance-induced, 409, 440 Reliable diagnosis, 192, 221
symptoms of, 410, 441 REM sleep, 337, 375
Psychotic or paranoid patient, approach to, 74, 108 Repetitive stimulation, decrementing response to,
PTSD. See Posttraumatic stress disorder (PTSD) myasthenia gravis and, 254, 284
Puberty Reprocessing, eye movement, for PTSD, 358–359
age of onset of, 7, 37 Requip. See Ropinirole
in Freud’s theories of human development, 219 Reserpine, mechanism of action of, 69, 101

478

Psychiatry Test Preparation and Review


Manual E-Book
Index

Resistance, 32 Schizophrenia, 163, 187, 403, 409, 435, 440


Respondeat superior, 236, 240, 382 appearance of patients with, 403, 435
Respondent conditioning, 35 chronic paranoid, mental status exam for,
Restless legs syndrome (RLS), 153, 188 412b–413b
treatment of, 64, 95 committing homicide, description of, 280, 318
Restricted affect, 391, 426 DSM criterion for, 71, 104
Reticular activating system (RAS), 105 early age at onset of, 13, 44–45
Reticular formation, in brain, unconsciousness and, late-onset, 152, 187
266, 300 lifetime prevalence of, 64, 95
Retinal hemorrhages, 166 neuroimaging of, 266, 300
Retrograde ejaculation, in thioridazine use, 215, 251 prevalence of, 77, 112
Retroviruses, 244 symptoms of, 391, 426
Rett’s disorder, 36 thought form for, 403, 436
Rickettsia rickettsii, 157–158 ventricles enlargement in, 14, 46
Right hemisphere disease, 326, 360 Schizophrenia spectrum, unspecified, 410, 441
Right-left disorientation, 83, 119 Schizophreniform disorder, 436
Rigidity consideration in, 256, 286
“clasp-knife” type, 239–240 Schizotypal personality disorder, 90, 127
Riluzole, 194, 224 School-age years, development during, 257, 288
for ALS, 367–368 SCID. See Structured Clinical Interview for DSM
Risk-taking behavior, 3, 33 (SCID)
Risperidone, 153, 188, 412, 445 Seasonal affective disorder (SAD), 65, 96–97
recommended monitoring for, 342, 383 bright artificial light for, 72, 105–106
side effects of, 30 Seclusion and restraint, 347, 387
tobacco smoking and, 75, 108 Secondary gain, medical help in, 87, 124
RLS. See Restless legs syndrome (RLS) Secondary prevention, 33
Rocky Mountain spotted fever, 157–158 Second messengers, 115
Rolandic epilepsy, benign, 362 Secure attachment, infantile, 270, 306
Role confusion, ego identity vs., 92 Sedation, histamine H1 receptor and, 69, 101, 284
Role dispute, 376–377 Segmental dystonia, 361
Role transition, 376–377 Seizures, brain tumors and, 368
Romazicon. See Flumazenil Selective attention bias, in cognitive psychology, 271,
Romberg’s sign, 177, 239–240 307–308
Ropinirole, for “crawling” sensation, 64, 95 Selective perception, 307–308
Rozerem. See Ramelteon Selective serotonin reuptake inhibitor (SSRI), 43
r-TPA. See Recombinant tissue plasminogen activator Selective serotonin reuptake inhibitors (SSRIs), 199
(r-TPA) for panic disorder
Russell’s sign, 405, 437 with CBT, 341, 382
with clonazepam, 342, 383
S for posttraumatic stress disorder, 390, 425
Saccades, abnormal smooth pursuit, for premature ejaculation, 341, 382
schizophrenia and, 343 side effect of, 335, 372
Sacroiliac joint syndrome, 297–298 for social anxiety disorder, with CBT, 346, 387
SAD. See Seasonal affective disorder (SAD) Selegiline, 178
“Saturday night palsy,” 47–48 as inhibitor of monoamine oxidase type B, 17, 50
Scanning speech, 203, 236 Self-disclosure, in psychotherapy, 276, 314
Schizencephaly, 170–171 Self-induced vomiting, complications of, 405, 437
Schizoaffective disorder, 129, 159 Self-mutilation, in borderline personality disorder, 12,
major mood episode in, 81, 118 44
Schizoid fantasy, 344, 384 Self-psychology, Kohut’s theories of, 213, 249, 306
Schizoid personality disorder, 59, 89, 126, 172, 406, Semantic memory, 307
438 Sensitivity, 102, 271, 308

479

Psychiatry Test Preparation and Review


Manual E-Book
Index

Sensitization theory, 34 Sleep wake disorders, in patient over, 65 years of age,


Sentence completion test, 360 151, 185
Separation-anxiety disorder, in children, 211, 246–247 Sleepwalking, 351
Serotonergic neurons, 40 Slow-wave sleep, 351
Serotonin, 273, 310 Smoking
aggression and, 35 effect on psychotropic medications, 75, 108
anxiety and, 9, 40 partial nicotine agonist for, 282, 320
obsessive-compulsive disorder and, 88, 125–126 Smooth pursuit saccades, abnormal, schizophrenia
Serotonin, 1A, buspirone and, 80, 116 and, 343, 367
Serotonin syndrome, 337, 376 Social anxiety disorder, 339, 379
neuroleptic malignant syndrome (NMS) vs., 254, CBT plus SSRI for, 346, 387
284 prevalence of, 47
supportive care, 376 Social learning theory, 34
supportive care, for serotonin syndrome, 337 Social phobia, 134, 167, 193, 222–224, 406, 438
Sertraline, 27, 60 avoidant personality disorder in, 197, 226
for heart attack, 27 poorer marital function and, 348, 388
Serum sedimentation rate, for temporal arteritis, 11, side effects of, 407, 439
42 Socioeconomic status, child abuse and, 337, 376
Severe white matter demyelination, 229–230 SOD1 gene, 367–368
Sexual abuse Sodium, 12, 43
in children, 202, 234 Sodium oxybate, 175
encopresis and, 339, 379 Solicitation, 345, 385
Sexual behavior, male, 270, 305–306 Somatic symptom disorder, 42, 224, 366, 420, 420b,
Sexual desire, during pregnancy, 343, 384 450–451
Sexual masochism, 330, 365 course and prognosis of, 421, 451
Shaken-baby syndrome, 166 DSM 5, criteria for, 255, 285
Shame, autonomy vs., 92 epidemiology of, 421, 451
Sheehan’s syndrome, 75, 109, 318 pain and, 329, 363–364
Short-acting injection Somnambulism, 322, 351
Ziprasidone, 188 Sonata. See Zaleplon
Short palpebral fissures, fragile X syndrome and, 16, Specific learning disorder, with impairment in reading,
49 201, 232–233
Sialorrhea, 317 Specificity, 271, 308
Simple partial seizures, of temporal lobe origin, 334, SPECT. See Single-photon emission computerized
371 tomography (SPECT)
Simultanagnosia, 221, 352 Speech therapy, 247
Single-photon emission computerized tomography Sphincter dysfunction, cervical spondylosis and, 366
(SPECT), 368 Sphingomyelinase, deficiency in, 259, 290–291
Skills training, for substance abuse, 347, 387 Spina bifida, valproic acid causing, 17, 50
Skinner, B. F., operant conditioning developed by, 5, 34 Spinal artery infarction, anterior, 351
SLE. See Systemic lupus erythematosus (SLE) Spinal cord
Sleep cervical, 366
decreased, 391, 426 metastases in, 220–221, 351
in elderly, 278, 315 posterior, in vitamin B12 deficiency, 208, 243
changes in, 217, 252 Spirochete, 157–158
stages of, 337, 375, 375t Spondylosis, cervical, 331, 366
Sleep apnea, 142, 175, 296, 408, 439 Spongiform encephalopathy, characteristics of, 269,
complications of, 408, 439 304
Sleep attacks, 130, 160 Spontaneous abortion, 405, 437
Sleep hygiene measures, 409, 440 SSPE. See Subacute sclerosing panencephalitis (SSPE)
Sleep paralysis, 98 SSRIs. See Selective serotonin reuptake inhibitors
Sleep terrors, 215, 251 (SSRIs)

480

Psychiatry Test Preparation and Review


Manual E-Book
Index

St. John’s Wort, 422, 453 Sydenham’s chorea, 110


Stagnation, generativity vs., 92 Symbiosis, Mahler’s stage(s) of separation-
State-dependent learning, 212, 247 individuation, 162, 307, 341, 382
Status epilepticus, intramuscular phenytoin for, 67, 98 Synapse formation, time period of, 195, 224
Stem occlusion, 289 Syringomyelia, 193, 222
Stereotactic thalamotomy, for essential tremor, 359 Systemic lupus erythematosus (SLE), 82, 119, 159
Steroids, 119 symptoms, 119
Stiff-person syndrome, 265, 299, 334, 371
Stimulus-response therapy, 184 T
Strattera. See Atomoxetine T4 dermatome, 80, 116
Stroke, hemianesthesia following, 261, 293 T10 dermatome, 80, 116
Structured Clinical Interview for DSM (SCID), 276, Tabes dorsalis, Argyll Robertson pupils and, 143, 177
314 Taenia solium, in CNS cysticercosis, 78, 113
Stupor, 204, 237 Tarasoff rule, 203, 236
Sturge-Weber syndrome, 73, 106 Tarasoff v Regents of the University of California,
Subacute combined degeneration, of spinal cord, 200, 82, 118
208, 231 Tardive dyskinesia, 130, 160
Subacute sclerosing panencephalitis (SSPE), 103 age and, 332, 368
Subarachnoid hemorrhage, 109 definition of, 313
characteristic of, 263, 296 neuroleptic-induced, risk factor for, 275, 312
Subcortical arteriosclerotic encephalopathy (SAE), 358 Tardive dystonia, definition of, 313
Subcortical ischemic vascular dementia, 358 Tasmar. See Tolcapone
Subcortical white matter, lacunar infarct of, 258, 288 Tay-Sachs disease, 96, 104, 290
Subdural hematoma, 161, 256, 256f, 286, 286f Teacher-student model, 399, 432
from head trauma, 134, 166 Telangiectasias, 292–293
Sublimation, 81–82, 117, 119 Temperament, child’s, biological factors in,
Substance abusers, 26, 59 198, 230
Substance-induced anxiety disorder, specifiers for, in Temporal arteritis (TA), 263, 295–296
DSM 5, 329, 363 serum sedimentation rate for, 11, 42
Substance-related and addictive disorders, 24, 56–57 Temporal lobe, 130, 160
Substituted judgment, 382 Tensilon injection, in myasthenia gravis, 284
Subthalamic nucleus, right, hemiballismus localizing Tertiary prevention, 4, 33, 122
to, 69, 101 Tetanospasmin, 375
Suicidal behavior, 154, 189 Tetanus infection, 228–229
insurance and, 146, 180 Tetanus toxin, 375
past, as indicator for future suicidal behavior, Tetrahydrocannabinol (THC), intoxication, 235
90, 127 Thalamic deep brain stimulation, for essential tremor,
previous, as best predictor for future suicide, 7, 37 359
Suicidal ideation, 76, 111 Thalamic lesions, 164, 293
Suicides, 335, 373 Thalamus, 160
borderline personality disorder and, 209, 244 Thallium poisoning, 164
in elderly, factors that increase chances of, THC. See Tetrahydrocannabinol (THC)
348, 388 Thematic apperception test, 360
major depressive disorder and, 203, 235 Thenar atrophy, in carpal tunnel syndrome, 355f
psychiatric malpractice and, 341, 382 Thiamine
psychiatrist and, 281, 319 intravenous, 313
Sullivan, Harry Stack, social interaction theory of, 92 level of, 409, 441
Sumatriptan, contraindication to, 7, 37 for Wernicke’s encephalopathy, 212, 247, 359
Superior temporal gyrus (Heschl’s gyrus), 160 Thiazide diuretic, lithium and, 358
Supportive psychotherapy, 153, 188 Thioridazine, side effects of, 215, 251
Supranuclear vertical gaze palsy, 360–361 Third-nerve palsy, diabetic, 328, 363
Suvorexant, 64, 95 Thoracic outlet syndrome, neurogenic, 356

481

Psychiatry Test Preparation and Review


Manual E-Book
Index

Thought content, in mental status exam, 78, 113, Transferrin, carbohydrate-deficient, in Parkinson’s
402, 435 disease, 256, 286–287
Thought process, 402, 413, 436, 444 Transgender, 400, 433
Thrombocytopenia, 165 Transient global amnesia (TGA),
Thrombocytosis, as potential side effect of valproic 128, 157, 323, 353
acid, 146, 180 Transition, in interview, 398, 431
Thrombolytic therapy, for stroke, 365 Transmissible spongiform encephalopathies. See Prion
Thymoma, associated with myasthenia gravis, 77, 112 disease(s)
Thymus gland, myasthenia gravis and, 112 Transtentorial herniation, 165
Thyroid function tests, 409, 441 Transverse myelitis
risperidone and, 342, 383 acute, 321, 351
Thyroid-stimulating hormone (TSH), 408, 439 diagnosis of, 191, 220–221
serum, increasing of, in hypothyroidism, 87, 124 Transvestic fetishism, 365
Tiagabine, 157 Transvestism, 41
Tic douloureux. See Trigeminal neuralgia (tic Trauma, psychological debriefing for,
douloureux) 326, 358–359
Ticlopidine, for stroke prevention, 355 Trazodone
Timed endurance tasks, in myasthenia gravis, 284 nefazodone and, 323, 353
Tinbergen, Nikolaas, experiments by, 215, 250 side effects of, 216, 252
Tinel’s sign, 355 Tremor, 334, 371–372
Tissue plasminogen activator (t-PA) essential, 111, 326, 359
intravenous, for ischemic stroke, 330, 365 intention, in Wilson’s disease, 257, 287
recombinant, contraindication to, in acute ischemic in Parkinson’s disease, 232
stroke, 64, 95 Treponema pallidum, 157–158
Titubation, 164 Trichotillomania, 44
Tizanidine, 160, 358 habit reversal therapy for, 148, 182
Tobacco, cytochrome P450 (CYP) 1A2 Tricyclic antidepressants (TCAs),
and, 255, 285 238, 256
Tobacco smoking. See Smoking concentration of, decreasing, 7, 37
Toddler, synapse formation in, 195, 224 side effects of, 67, 90, 99, 123, 126
Todd’s paralysis, 144, 177 use of, 138
Tolcapone, 176 Trigeminal neuralgia (tic douloureux), 354
Tonic-clonic seizures, generalized, 362 carbamazepine for, 11, 41
Topiramate, 157 Triiodothyronine, tricyclic antidepressant
Torsion dystonia, primary, dementia and, 257, 288 and, for
Torticollis, 361f augmentation, 273, 310–311
Total sleep time, REM sleep in, 337, 375 Triptans, in migraine headache, 296
Tourette’s syndrome (TS), Trisomy, 165–166. See Down’s syndrome
158–159, 182 Tropical spastic paraparesis, 144, 178
pimozide for, 213, 249 Truncal ataxia, 133
vocal tics and, 26, 59 Trust, vs. mistrust, 92
Toxoplasmosis, 288 TSH. See Thyroid-stimulating
Toxoplasmosis, in AIDS, 195, 225 hormone (TSH)
t-PA. See Tissue plasminogen activator (t-PA) T test, usage of, 258, 289
Trail-making test, 206, 241 Tuberculomas, 288
for testing executive function, 6, 36 Tuberculosis, of central nervous
Transcortical aphasias, 93 system, 257, 288
Transcortical mixed aphasia, 93 Tuberculous meningitis, 162
Transcortical sensory aphasia, 93 Tuberoinfundibular pathway, 178, 364
Transcranial magnetic stimulation (NeuroStar TMS Tuberous sclerosis, 6, 36
Therapy), 272, 309 feature of, 260, 292–293
Transference, 32, 343, 383 Tyrosine hydroxylase, 142, 175

482

Psychiatry Test Preparation and Review


Manual E-Book
Index

U Vincristine therapy, acquired peripheral neuropathy and,


Unconscious symbolism, skills training and, for 12, 43
substance abuse, 347, 387 Violence
Unethical behavior, 83 dependent personality disorder and, 202, 234
Urinary retention, sniffing glue and, 193, 222 predictive factor for, 194, 224
Urine test, for creatinine and protein, 346, 386 Violent patients
Urine test, paroxetine, 43 admission of, 412, 443
Urine toxicology, 409, 441 dangerousness of, 442
Urophilia, 400, 432 discharge disposition for, 412, 443–444
interviewing, 411, 442
V social work consultation for, 412, 443
Vacuolar myelopathy, 220–221 Visiting-nurse consultation, 416, 447
HIV-associated, 196, 225 Vistaril. See Hydroxyzine
Vagal nerve stimulation, for refractory Visual field defect, with anterior choroidal artery
epilepsy, 207, 241 syndrome, 353
Vaginismus, 169 Vitamin A deficiency, 243
female orgasmic disorder and, 15, 47 Vitamin B12 (cobalamin)
Vaillant theory, of happy childhood, 386 deficiency in, 243, 267, 300–301
Valbenazine, 67, 99 triad of, 248
Validity, 59 in subacute combined degeneration of spinal cord,
diagnosis, 221, 314 200, 231
Valproic acid, 17, 49, 311–312 Vitamin B6 (pyridoxine) deficiency, isoniazid (INH)
for juvenile myoclonic epilepsy, 362 exposure and, 209, 243
lamotrigine levels and, 333, 368 Vitamin E (α-tocopherol) deficiency, triad of, 248
during pregnancy, 214, 250 Vivitrol, 278, 315
for primary generalized seizures, 67, 99 Vocal tics, Tourette’s syndrome and, 26, 59
side effects of, 180 von Hippel-Lindau syndrome (VHL), 106
therapeutic range for, 154, 189 Vocal tics, Tourette’s syndrome and von
Varenicline, 282, 320 Recklinghausen’s disease. See Neurofibromatosis,
Vascular dementia, 126, 301 type, 1 (NF1)
neurologic signs of, 267, 301 Vortioxetine, 153, 187
VDRL test. See Venereal Disease Research Laboratory Voyeuristic disorder, 41, 365
(VDRL) test
Venereal Disease Research Laboratory (VDRL) test, W
139, 172 Wallenberg’s syndrome, 65, 96. See also Lateral
Venlafaxine, increased blood pressure medullary syndrome
and, 214, 250 Warfarin, for stroke prevention, 355
Ventral amygdalofugal pathways, 364 Water intoxication, 13, 44
Ventricles, enlargement of, in schizophrenia, 14, 46 WBC count. See White blood cell (WBC)
Verbigeration, 403, 436 Weber’s syndrome, 164
Vertebral artery dissection, chiropractic adjustments Wechsler Adult Intelligence Scale, 19, 52
in, 264, 297–298 Weight gain, 322
Vertebrobasilar territory ischemia, 219–220 Weight gain, histamine H1 receptor and, 69, 101, 254,
Vertical nystagmus, PCP intoxication associated 284
with, 24, 57 Weight loss, eating disorders and, 195, 225
Vertigo Wellbutrin. See Bupropion
ataxia with, carotid territory transient ischemic Wernicke-Korsakoff syndrome, 242–243
attack or stroke and, 76, 111 Wernicke’s aphasia, 93, 322, 352
benign paroxysmal positional, 135, 167 Wernicke’s encephalopathy, 110, 242–243, 247, 326,
Videotelemetry, nonepileptic seizures and, 323, 359–360
353–354 brain autopsy of, 2, 31
Vilazodone, 67, 99 West Nile virus infection, 198, 228–229, 244

483

Psychiatry Test Preparation and Review


Manual E-Book
Index

White blood cell (WBC) count, clozapine and, 151, X


185, 275, 312 Xyrem, 98
White matter, subcortical, lacunar infarct of, 258, 288
Wihtigo, 38 Y
Williams’ syndrome, 36, 292 Yersinia pestis, 157–158
Wilson’s disease, 51, 257, 287
Winnicott, Donald, in development theory of “good Z
enough mothering,” 2, 31 Zaleplon, 189
Wisconsin card-sorting test, 36 Zanaflex. See Tizanidine
With onset during withdrawal, as specifier for Ziprasidone, 18, 50, 67, 98–99, 153, 188, 410, 441
substance induced anxiety disorder, 329, 363 Zolpidem, 189
Word salad, 352, 403, 436
Wrist flexors, inclusion body myositis and, 364

484

You might also like

pFad - Phonifier reborn

Pfad - The Proxy pFad of © 2024 Garber Painting. All rights reserved.

Note: This service is not intended for secure transactions such as banking, social media, email, or purchasing. Use at your own risk. We assume no liability whatsoever for broken pages.


Alternative Proxies:

Alternative Proxy

pFad Proxy

pFad v3 Proxy

pFad v4 Proxy